You are on page 1of 407

EBD_7587

• Corporate Office : 45, 2nd Floor, Maharishi Dayanand Marg, Corner Market,
Malviya Nagar, New Delhi-110017
Tel. : 011-49842349 / 49842350

By
Dheeraj Pandey

Typeset by Disha DTP Team

DISHA PUBLICATION
ALL RIGHTS RESERVED

© Publisher
No part of this publication may be reproduced in any form without prior permission of the publisher.
The author and the publisher do not take any legal responsibility for any errors or misrepresentations
that might have crept in. We have tried and made our best efforts to provide accurate up-to-date
information in this book.

For further information about the books from DISHA,


Log on to www.dishapublication.com or email to info@dishapublication.com
CONTENTS
Sl. No. Chapter Name Page No.

1. Some Basic Concepts of Chemistry ..................................................... 1-12

2. Structure of Atom ................................................................................... 13-24

3. Classification of Elements and Periodicity in Properties.................... 25-32

4. Chemical Bonding and Molecular Structure........................................ 33-42

5. States of Matter ...................................................................................... 43-54

6. Thermodynamics .................................................................................... 55-72

7. Equilibrium ............................................................................................. 73-92

8. Redox Reactions .................................................................................... 93-100

9. Hydrogen ................................................................................................ 101-108

10. The s-Block Elements ........................................................................... 109-116

11. The p-Block Elements (Group-13 and Group14) ................................ 117-124

12. Organic Chemistry — Some Basic Principles and Techniques ......... 125-140

13. Hydrocarbons ......................................................................................... 141-158

14. Environmental Chemistry ..................................................................... 159-166

15. The Solid State ....................................................................................... 167-176


EBD_7587
16. Solutions ................................................................................................. 177-190

17. Electrochemistry .................................................................................... 191-206

18. Chemical Kinetics .................................................................................. 207-222

19. Surface Chemistry .................................................................................. 223-232

20. General Principles and Processes of Isolation of Elements .............. 233-242

21. The p-Block Elements (Group 15, 16, 17 & 18) ................................. 243-254

22. The d-and f-Block Elements ................................................................. 255-262

23. Co-ordination Compounds .................................................................... 263-282

24. Haloalkanes and Haloarenes ................................................................ 283-300

25. Alcohols, Phenols and Ethers ............................................................... 301-324

26. Aldehydes, Ketones and Carboxylic Acids .......................................... 325-348

27. Amines .................................................................................................... 349-370

28. Biomolecules .......................................................................................... 371-378

29. Polymers ................................................................................................. 379-386

30. Chemistry in Everyday Life .................................................................. 387-392

31. Analytical Chemistry .............................................................................. 393-400


Some Basic Concepts
1
of Chemistry
1. If the true value for an experimental result is 6.23 (a) 1.084 × 1018 (b) 6.023 × 1019
and the results reported by three students X, Y (c) 4.84 × 1017 (d) 6.023 × 1023
and Z are : 7. 1.44 g of titanium (At. mass = 48) reacted with
X : 6.18 and 6.28 excess of O2 and produce x g of non stoichiometric
Y : 6.20 and 6.023 compound Ti1.44O. The value of x is:
Z : 6.22 and 6.24
Which of the following option is correct?
(a) X precise, Y accurate, Z precise and (a) 2 (b) 1.77
accurate. (c) 1.44 (d) none of these
(b) X precise and accurate, Y not precise, Z 8. Silver oxide (Ag2O) decomposes at temperature
precise 300 °C yielding metallic silver and oxygen gas. A
(c) Both X & Z precise & accurate, Y not 1.60 g sample of impure silver oxide yields
precise. 0.104 g of oxygen gas. What is the per cent by
(d) Both X & Y neither precise nor accurate, Z mass of the silver oxide in the sample?
both precise and accurate. (a) 5.9 (b) 47.125 (c) 94.25 (d) 88.2
2. Irrespective of the source, pure sample, of water 9. A sample of AlF3 contains 3.0 × 1024 F– ions. The
always yields 88.89% mass of oxygen and 11.11% number of formula unit of this sample are
mass of hydrogen. This is explained by the law of (a) 9 × 1024 (b) 3 × 1024
(c) 0.75 × 1024 (d) 1.0 × 1024
10. Dissolving 120 g of a compound of mol. wt. 60 in
(a) conservation of mass 1000 g of water gave a solution of density 1.12 g/ mL.
(b) multiple proportions The molarity of the solution is:
(c) constant composition (a) 1.00 M (b) 2.00 M
(d) constant volume (c) 2.50 M (d) 4.00 M
3. The percentage of Se in peroxidase anhydrous 11. A mixture of O2 and gas "Y" mol. mass 80 in the
enzyme is 0.5% by weight (atomic weight = 78.4). mole ratio a : b has a mean molecular mass 40.
Then minimum molecular weight of peroxidase What would be mean molecular mass, if the gases
anhydrous enzyme is are mixed in the ratio b : a under, identical
(a) 1.568 × 103 (b) 1.568 × 104 conditions? (Assume that gases are
(c) 15.68 (d) 3.136 × 104 non-reacting):
4. The amount of zinc required to produce 224 mL of
H2 at STP on treatment with dil. H2SO4 will be (a) 40 (b) 48 (c) 62 (d) 72
(a) 6.5 g (b) 0.65 g 12. If 3.01 × 1020 molecules are removed from 98 mg
(c) 65 g (d) 0.065 g of H2SO4, then the number of moles of H2SO4
5. Assuming fully decomposed, the volume of CO2 left are
released at STP on heating 9.85 g of BaCO3 (a) 0.1 × 10–3 (b) 0.5 × 10–3
(Atomic mass, Ba = 137) will be (c) 1.66 × 10–3 (d) 9.95 × 10–2
(a) 1.12 L (b) 2.24 L (c) 4.06 L (d) 0.84 L 13. The number of molecules in 8.96 litre of a gas at
6. The number of water molecules present in a drop 0 °C and 1 atm. pressure is approximately
of water (volume 0.0018 mL) density = 18 mL–1 at (a) 6.023 × 1023 (b) 12.04 × 1023
room temperature is (c) 18.06 × 10 23 (d) 24.08 × 1022
EBD_7587
2 Chemistry Objective MCQs
14. The density of 3M solution of sodium chloride is 21. The concentrated sulphuric acid that is peddled
1.252 g mL–1. The molality of the solution will be: commercial is 95% H2SO4 by weight. If the density
(molar mass, NaCl = 585 g mol–1) of this commercial acid is 1.834 g cm–3, the
molarity of this solution is
(a) 2.60 m (b) 2.18 m (c) 2.79 m (d) 3.00 m (a) 17.8 M (b) 12.0 M
15. Indium (atomic mass = 114.82) has two naturally (c) 10.5 M (d) 15.7 M
occurring isotopes, the predominant one form 22. 9 moles of "D" and 14 moles of E are allowed to
has isotopic mass 114.9041 and abundance of react in a closed vessel according to given
95.72%. Which of the following isotopic mass is reactions. Calculate number of moles of B formed
the most likely for the other isotope? in the end of reaction, if 4 moles of G are present
(a) 112.94 (b) 115.90 in reaction vessel. (Percentage yield of reaction
(c) 113.90 (d) 114.90 is mentioned in the reaction)
16. A compound contains 54.55 % carbon, 9.09%
hydrogen , 36.36% oxygen. The empirical formula
of this compound is: 80%
Step-1 3D + 4E ¾¾¾ ® 5C + A
(a) C3H 5O (b) C 4 H 8O 2 50%
Step-2 3C + 5G ¾¾¾ ® 6B + F
(c) C 2 H 4O 2 (d) C 2 H 4O (a) 2.4 (b) 30 (c) 4.8 (d) 1
17. An organic compound whose empirical and 23. A mixture of NH4NO3 and (NH4)2HPO4 contain
molecular formula are same, contains 20% 30.40% mass per cent of nitrogen. What is the
carbon, 6.7% hydrogen, 46.7% nitrogen and the mass ratio of the two components in the mixture?
rest oxygen. On heating it yields ammonia, (a) 2 : 1 (b) 1 : 2 (c) 3 : 4 (d) 4 : 1
leaving a solid residue. The solid residue gives 24. The mass of N2F4 produced by the reaction of
a violet colour with dilute solution of alkaline 2.0 g of NH3 and 8.0 g of F2 is 3.56 g. What is the
copper sulphate. The organic compound is per cent yield?
2NH 3 + 5F2 ¾¾
® N 2 F4 + 6HF
(a) 79.0 (b) 71.2
(a) NH2COONH4 (b) HCOONH4
(c) 84.6 (d) None of these
(c) NH2NHCHO (d) NH2CONH2
25. The ppm level of F– in a 500 g sample of a tooth
18. 10 mL of 2 M NaOH solution is added to 200 mL
paste containing 0.2 g F– is
of 0.5 M of NaOH solution. What is the final
concentration ? (a) 400 (b) 1000 (c) 250 (d) 200
(a) 0.57 M (b) 5.7 M 26. Two glucose solutions are mixed. One has a
(c) 11.4 M (d) 1.14 M volume of 480 mL and a concentration of 1.50 M
and the second has a volume of 520 mL and
19. Arrange the following in the order of increasing
concentration 1.20 M. The molarity of final
mass (atomic mass: O = 16, Cu = 63, N = 14)
solution is
(a) 1.20 M (b) 1.50 M
(c) 1.34 M (d) 2.70 M
I. one atom of oxygen
27. What is the volume of CO2 liberated (in litres) at
II. one atom of nitrogen
1 atmosphere and 0 °C when 10 g of 100% pure
III. 1 × 10–10 mole of oxygen
calcium carbonate is treated with excess dilute
IV. 1 × 10–10 mole of copper
sulphuric acid?
(a) II < I < III < IV (b) I < II < III < IV
(Atomic mass Ca : 40, C : 12, O : 16)
(c) III < II < IV < I (d) IV < II < III < I
(a) 0.224 (b) 2.24 (c) 22.4 (d) 224
20. The number of moles of oxygen in one litre of air
28. Complete combustion of 0.858 g of compound X
containing 21% oxygen by volume, under
standard conditions are gives 2.64 g of CO2 and 1.26 g of H2O. The lowest
(a) 0.0093 mole (b) 0.21 mole molecular mass X can have:
(c) 2.10 mole (d) 0.186 mole (a) 43 g (b) 86 g (c) 129 g (d) 172 g
Some Basic Concepts of Chemistry 3
29. The impure 6 g of NaCl is dissolved in water and CO in the mixture? (All volumes were measured
then treated with excess of silver nitrate solution. under identical conditions)
The mass of precipitate of silver chloride is found (a) 60 (b) 40 (c) 6 (d) 4
to be 14 g. The % purity of NaCl solution would 39. When burnt in air, 14.0 g mixture of carbon and
be: sulphur gives a mixture of CO2 and SO2 in the
volume ratio of 2:1, volume being measured at
(a) 95% (b) 85% (c) 75% (d) 65% the same conditions of temperature and pressure
30. How many of 0.1N HCl are required to react moles of carbon in the mixture is
completely with 1 g mixture of Na2CO3 and
(a) 0.75 (b) 0.5 (c) 0.40 (d) 0.25
NaHCO3 containing equimolar amounts of two ?
(a) 157.7 mL (b) 15.77 mL 40. What is the molarity of SO24- ion in aqueous
(c) 147.7 mL (d) 14.77 mL solution that contain 34.2 ppm of Al2(SO4)3?
31. 5 g of benzene on nitration gave 6.6 g of (Assume complete dissociation and density of
nitrobenzene. The theoretical yield of the
solution 1g/mL)
nitrobenzene will be
(a) 4.5 g (b) 5.6 g (c) 8.09 g (d) 6.6 g
32. An aqueous solution of oxalic acid dihydrate (a) 3 × 10–4 M (b) 2 × 10–4 M
(c) 10 M –4 (d) None of these
contains its 6.3 g in 250 mL. The volume of 0.1 N
NaOH required to completely neutralize 10 mL of 41. A transition metal M forms a volatile chloride
this solution which has a vapour density of 94.8. If it contains
(a) 4 mL (b) 20 mL (c) 2 mL (d) 40 mL 74.75% of chlorine the formula of the metal
33. Choose the incorrect formula out of the four chloride will be
compounds for an element X below : (a) MCl3 (b) MCl2 (c) MCl4 (d) MCl5
(a) X2O3 (b) X2Cl3 42. 50 mL 10 N H2SO4, 25 mL 12 N HCl and 40 mL 5 N
(c) X2(SO4)3 (d) XPO4 HNO3 were mixed together and the volume of the
34. A gaseous compound of nitrogen and hydrogen mixture was made 1000 mL by adding water. The
contains 12.5% (by mass) of hydrogen. The normality of the resultant solution will be
density of the compound relative to hydrogen is (a) 2 N (b) 1 N (c) 3 N (d) 4 N
16. The molecular formula of the compound is:
43. 250 mL of a sodium carbonate solution contains
(a) NH2 (b) N3H (c) NH3 (d) N2H4 2.65 g of Na2CO3. If 10 mL of this solution is
35. In an organic compound of molar mass 108 g mol–1 diluted to one litre, what is the concentration of
C, H and N atoms are present in 9 : 1 : 3.5 by mass. the resultant solution?
Molecular formula can be:
(mol. wt. of Na2CO3 = 106)
(a) C6H8N2 (b) C7H10N
(c) C5H6N3 (d) C4H18N3 (a) 0.1 M (b) 0.01 M
36. What is the empirical formula of vanadium oxide, (c) 0.001 M (d) 10–4 M
if 2.74 g of the metal oxide contains 1.53 g of 44. 3.92 g of ferrous ammonium sulphate crystals are
metal? dissolved in 100 mL of water. 20 mL of this
(a) V2O3 (b) VO (c) V2O5 (d) V2O7 solution requir es 18 mL of potassium
37. 100 cm3 of 0.1 N HCl is mixed with 100 cm3 of 0.2 permaganate during titration for complete
N NaOH solution. The resulting solution is oxidation. The weight of KMnO4 present in one
litre of the solution of
(a) 3.47 g (b) 12.38 g
(a) 0.1 N and the solution is basic
(c) 1.23 g (d) 34.76 g
(b) 0.1 N and the solution is acidic
(c) 0.05 N and the solution is basic 45. 1.12 mL of a gas is produced at S.T.P. by the
action of 4.12 mg of alcohol ROH with methyl
(d) 0.05 N and the solution is acidic
magnesium Iodide. The molecular mass of alcohol
38. On subjecting 10 mL mixture of N2 and CO to
is
repeated electro spark, 7 mL of O2 was required
for combustion. What was the mole percent of (a) 16.0 (b) 41.2 (c) 82.4 (d) 156.0
EBD_7587
4 Chemistry Objective MCQs
46. How many moles of P4 can be produced by 250 mL. On titration 16.68 mL of this solution
reaction of 0.10 moles Ca5(PO4)3F, 0.36 moles requires 25 mL of N/15 NaOH solution for
SiO2 and 0.90 moles C according to the following complete neutralization, calculate x.
reaction? (a) 3 (b) 2 (c) 4 (d) 5
4 Ca 5 (PO 4 )3 F + 18 SiO 2 + 30 C ¾¾
® 53. 2 g of a mixture of CO and CO2 on reaction with
3P4 + 2CaF2 + 18CaSiO3 + 30 CO excess I2O5 produced 2.54 g of I 2 . What will be
the mass % of CO2 in the original mixture ?
(a) 0.060 (b) 0.030 (c) 0.045 (d) 0.075
(a) 35 (b) 70 (c) 30 (d) 60
47. 1 mole of mixture of CO and CO 2 requires exactly 54. Which statement is false for the balanced
28 g KOH in solution for complete conversion of equation given below?
all the CO 2 into K 2 CO 3 . How much amount CS2 + 3O2 —® 2SO2 + CO2
more of KOH will be required for conversion into (a) One mole of CS2 will produce one mole of
CO2
K 2 CO 3 . If one mole of mixture is completely
(b) The reaction of 16 g of oxygen produces
oxidized to CO 2 7.33 g of CO2
(c) The reaction of one mole of O2 will produce
(a) 112 g (b) 84 g (c) 56 g (d) 28 g 2/3 mole of SO2
48. 2.0 g of a sample contains mixture of SiO2 and (d) Six molecules of oxygen requires three
Fe2O3. On very strong heating, it leaves a residue molecules of CS2
weighing 1.96 g. The reaction responsible for 55. Number of moles of KMnO4 required to oxidize
loss of mass is given below. one mole of Fe(C2O4) in acidic medium is
Fe2O3 (s) ® Fe3O4 (s) + O2 (g), (unbalance
equation)
What is the percentage by mass of SiO2 in (a) 0.167 (b) 0.6 (c) 0.2 (d) 0.4
original sample? 56. Haemoglobin contains 0.33% of iron by weight.
(a) 100 % (b) 20 % (c) 40 % (d) 60 % The molecular weight of haemoglobin is
49. 20 g of CaCO3 on heating gave 8.8 g of CO2 and approximately 67200. The number of iron atoms
(at. wt. of Fe = 56) present in one molecule of
11.2 g of CaO. This is in accordance with
haemoglobin is
(a) The law of conservation of mass.
(b) The law of constant composition. (a) 6 (b) 1 (c) 2 (d) 4
(c) The law of reciprocal proportion. 57. A 25.0 mm × 40.0 mm piece of gold foil is 0.25 mm
(d) None of these thick. The density of gold is 19.32 g/cm3. How
50. If 224 mL of a triatomic gas has a mass of 1 g at many gold atoms are in the sheet? (Atomic
273K and 1 atmospheric pressure then the mass weight : Au = 197.0)
of one atom is
(a) 7.7 × 1023 (b) 1.5 × 1023
(a) 8.30 × 10–23 g (b) 2.08 × 10–23 g 21
(c) 4.3 × 10 (d) 1.47 × 1022
(c) 5.53 × 10–23 g (d) 6.24 × 10–23 g 58. What volume of oxygen gas (O2) measured at
51. Specific volume of cylindrical virus particle is 0°C and 1 atm, is needed to burn completely 1L
6.02 × 10 –2 cc/g. whose radius and length are of propane gas (C3H8) measured under the same
7 Å & 10 Å respectively. conditions ?
If NA = 6.02 × 1023mol–1, find molecular weight of (a) 7 L (b) 6 L (c) 5 L (d) 10 L
virus 59. 25.4 g of I2 and 14.2 g of Cl 2 are made to react
completely to yield a mixture of ICl and ICl3.
(a) 3.08 × 103 kg/mol (b) 3.08 × 104 kg/mol Calculate moles of ICl and ICl3 formed
(c) 1.54 × 104 kg/mol (d) 15.4 kg/mol (a) 0.1, 0.1 (b) 0.2, 0.2
52. 1.575 g of oxalic acid (COOH) 2.xH2O are (c) 0.1, 0.2 (d) 0.2, 0.1
dissolved in water and the volume made upto
Some Basic Concepts of Chemistry 5
60. 10 moles SO2 and 15 moles O2 were allowed to solution formed by adding 4 mole of SO2Cl2 to
react over a suitable catalyst. 8 moles of SO3 excess of water ?
were formed. The remaining moles of SO2 and (a) 1 (b) 2 (c) 3 (d) 4
O2 respectively are - 66. 1 g mixture of equal number of mole of Li 2CO3
(a) 2 moles, 11 moles (b) 2 moles, 8 moles and other metal carbonate (M2CO3) required 21.6
(c) 4 moles, 5 moles (d) 8 moles, 2 moles mL of 0.5 N HCl for complete neutralisation
61. In the reaction reaction. What is the approximate atomic mass
® 4NO(g) + 6H 2 O(l ) ,
4NH 3 (g) + 5O 2 (g) ¾¾ of the other metal?
when 1 mole of ammonia and 1 mole of O2 are
made to react to completion (a) 25 (b) 23 (c) 51 (d) 118
(a) 1.0 mole of H2O is produced 67. When 2.5 g of a sample of Mohr's salt reacts
(b) 1.0 mole of NO will be produced N
(c) all the ammonia will be consumed completely with 50 mL of KMnO4 solution.
10
(d) all the oxygen will be consumed
The % purity of the sample of Mohr's salt is:
62. An ideal gaseous mixture of ethane (C2H6) and (a) 78.4 (b) 70 (c) 37 (d) 40
ethene (C2H4) occupies 28 litre at 1 atm and
273 K. The mixture reacts completely with 128 g 68. If potassium chlorate is 80% pure, then 48 g of
O2 to produce CO2 and H2O. Mole fraction at oxygen would be produced from (atomic mass of
K =39)
C2H6 in the mixture is:
(a) 153.12 g of KClO3 (b) 122.5 g of KClO3
(a) 0.6 (b) 0.4 (c) 0.5 (d) 0.8 (c) 245 g of KClO3 (d) 98 g of KClO3
63. Wood's metal contains 50.0% bismuth, 25.0% 69. 12 g of Mg (atomic mass 24) will react completely
lead, 12.5% tin and 12.5% cadmium by mass. with hydrochloric acid to give
What is the mole fraction of tin? (a) One mol of H2
(Atomic mass : Bi = 209, Pb = 207, Sn = 199, Cd = 112)
(b) 1/2 mol of H2
(a) 0.202 (b) 0.158 (c) 0.176 (d) 0.221
64. The mass of BaCO3 produced when excess CO2 (c) 2/3 mol of O2
is bubbled through a solution of 0.205 mol (d) both 1/2 mol of H2 and 1/2 mol of O2
Ba(OH)2 is : 70. 2.76 g of silver carbonate (at mass of Ag 108) on
(a) 81 g (b) 40.5 g being heated strongly yield a residue weighing
(c) 20.25 g (d) 162 g
65. Sulfuryl chloride (SO2Cl2) reacts with water to
give a mixture of H2SO4 and HCl. How many moles (a) 2.16 g (b) 2.48 g
of baryta would be required to neutralize the (c) 2.32 g (d) 2.64 g

Answer KEY
1 (d) 8 (c) 15 (a) 22 (a) 29 (a) 36 (c) 43 (c) 50 (c) 57 (d) 64 (b)
2 (c) 9 (d) 16 (d) 23 (a) 30 (a) 37 (c) 44 (a) 51 (d) 58 (c) 65 (b)
3 (b) 10 (b) 17 (d) 24 (d) 31 (c) 38 (a) 45 (c) 52 (b) 59 (a) 66 (d)
4 (b) 11 (d) 18 (a) 25 (a) 32 (d) 39 (b) 46 (a) 53 (c) 60 (a) 67 (a)
5 (a) 12 (b) 19 (a) 26 (c) 33 (b) 40 (a) 47 (b) 54 (d) 61 (d) 68 (a)
6 (b) 13 (d) 20 (a) 27 (b) 34 (d) 41 (c) 48 (c) 55 (b) 62 (b) 69 (b)
7 (b) 14 (c) 21 (a) 28 (a) 35 (a) 42 (b) 49 (a) 56 (d) 63 (c) 70 (a)
EBD_7587
6 Chemistry Objective MCQs

1. (d) Both Y and X are neither precise nor


16
accurate as the two values in each of them ´ x = 0.104 Þ x = 1.508 g
are not close. With respect to X & Y, the 232
values of Z are close & agree with the true 1.508
value. Hence, both precise & accurate. % of Ag2O = ´ 100 = 94.25%
1.6
2. (c) The H : O ratio in water is fixed, irrespective
of its source. Hence it is law of constant 9. (d) An, AlF3 the number of F is 3 for one AlF3
composition. molecule 3F– º 1 formula unit of AlF3
3. (b) 0.5% by weight means if Mol. wt. is 100 then 1
mass of Se is 0.5. If at least one atom of Se
3.0 × 1024 F– º ´ 3.0 ´ 10 24 AlF3 units
3
is present in the molecule then 10. (b) Given
100 mass of solute (w) = 120 g
M. Wt = ´ 78.4 = 1.568 ´ 10 4
0.5 mass of solvent (w) = 1000 g
Mol. mass of solute = 60 g
4. (b) Zn + H 2SO 4 ¾
¾® ZnSO 4 + H 2 density of solution = 1.12 g/ mL
65 g Zn gives 1 mole of H2 = 22400 mL of H2 From the given data,
224 mL of H2 will be obtained from 0.65 g Mass of solution = 1000 + 120 = 1120 g
Zn. Mol. mass Mol. mass
Q d= or V =
5. (a)
V d
BaCO 3 ¾
¾® BaO + CO 2
1120
192 g of BaCO3 gives 1 mol of CO2 = 22.4 L Volume of solution V = = 1000 mL
9.85 g of BaCO3 will give 0.05 mol of CO2 1.12
which is equal to 1.12 litre. or = 1 litre
6. (b) 0.0018 mL = 0.0018 g = 0.0001 mole of water W
Now molarity (M) =
-4
= 10 mole Mol. mass ´ V ( lit )
\ Number of water molecules 120
= = 2M
= 6.023 ´ 10 23 ´ 10 -4 = 6.023 × 1019 60 ´ 1
7. (b) Ti + O2 —® Ti1.44O 11. (d) Let mole fraction of O2 is x
40 = 32 × x + 80 (1 – x)
1.44 x
mole mole or x = 5 / 6
48 48 (1.44 ) + 16
5 1
:
a : b = x : (1 – x) =
1.44 1.44x 6 6
\ 48 = 48 1.44 + 16
( ) When ratio is changed
x = 1.77 g 1 5
8. (c) Let mass of Ag2O = x g M mixture = 32 ´ + 80 ´ = 72
60 6
1 12. (b) Moles of H2SO4 in 98 mg of H2SO4
Ag2O ¾® 2Ag + O2
2 1
xg = ´ 0.098 = 0.001
1 98
232 g ´ 32 = 16 g O2
2 Moles of H2SO4 removes
Some Basic Concepts of Chemistry 7
When an aqueous solution of biuret is
3.01 ´ 10 20 -3 treated with dilute sodium hydroxide and a
= = 0.5 ´ 10 = 0.0005
6.02 ´ 10 23 drop of copper sulphate, a violet colour is
Moles of H2SO4 left produced. This test is known as biuret test,
= 0.001 – 0.0005 = 0.5 × 10–3 and is characteristic of compounds having
13. (d) At S.T.P. 22.4 litre of gas contains 6.023 × the group –CONH–
1023 molecules 18. (a) From molarity equation
\ molecules in 8.96 litre of gas M1V1 + M2V2 = MV(total)
10 200 210
6.023 ´10 23 ´ 8.96 2´ + 0.5 ´ = M´
= = 24.08 ´10 22 1000 1000 1000
22.4
120 = M × 210
14. (c) The relation between molarity (M) and
molality (m) is 120
M= = 0.57 M
210
æ1 M ö
d = M ç + 2 ÷ , M2 = Mol. mass of 19. (a) Mass of 6.023 × 1023 atoms of oxygen = 16 g
è m 1000 ø
Mass of one atom of oxygen
solute
On putting value 16
= = 2.66 ´ 10 -23 g
6.023 ´ 10 23
æ 1 58.5 ö
1.252 = 3 ç + ÷ Mass of 6.023 × 1023 atoms of nitrogen
è m 1000 ø
= 14 g
on solving m = 2.79 Mass of one atom of nitrogen
15. (a) Let atomic weight of other isotope is M
14
114.9041´ 95.72 + M ´ 4.28 = 23
= 2.32 ´ 10 -23 g
114.82 = 6.023 ´ 10
100
Mass of 1 mole of oxygen = 16 g
M = 112.94
Mass of 1 × 10–10 mole of oxygen = 16 × 10–10
16. (d) C 54.55 54.55/12 = 4.5 4.5/2.27 = 2
Mass of 1 mole of copper = 63 g
H 9.09 9.09/1= 9.09 9.09/2.27 = 4 Mass of 1 × 10–10 mole of copper = 63 × 1 × 10–10
O 36.36 36.36/16 = 2.27 2.27/2.27= 1 = 63 × 10–10
Hence empirical formula of the So, the order of increasing mass is
compound = C2H4O II < I < III < IV.
20. (a) 21% of 1 litre is 0.21 litre.
17. (d) 22.4 litres = 1 mole at STP
Atomic Relative no. Simplest 0.21
Element Percentage \ 0.21 litre = = 0.0093 mol
mass of atoms ratio 22.4
C 20% 12 1.66 1
21. (a) 95% H2SO4 by weight means 100g H2SO4
H 6.7% 1 6.7 4
solution contains 95g H2SO4 by mass.
N 46.7% 14 3.33 2
O 26.6% 16 1.66 1 Molar mass of H2SO4 = 98g mol–1
95
Empirical formula = Molecular formula Moles in 95g = = 0.969 mole
98
= CH4N2O or NH2CONH2
Volume of 100g H 2SO 4
D
H 2 NCONH 2 + H 2 NCONH 2 ¾¾®
mass 100g
= =
H2 NCONHCONH 2 + NH3 density 1.834g cm –3
biuret,
= 54.52 cm3 = 54.52 × 10–3 L
EBD_7587
8 Chemistry Objective MCQs

Moles of solute 1.26


Molarity = 2× = 0.14 Þ mass of H = 0.14
Volme of solute in L 18
Compound does not contains oxygen.
0.969
= = 17.8 M So EF ® C0.06 H0.14 Þ C3H17
54.52 ´ 10 –3
Þ Lowest M.M. = 43
80% 29. (a) The reaction that takes place is
22. (a) 3D + 4E ¾¾¾ ® 5C + A
5 NaCl + AgNO3 ¾¾
® AgCl ¯ + NaNO 3
9 mole 14 mole ´ 9 ´ 0.8 = 12 mole \ 143.5 g of AgCl is produced from 58.5
3
50% g NaCl
3C + 5G ¾¾¾
® 6B + F
\ 14 g of AgCl will be produced from
12 mole 4 mole
58.5 ´ 14
Limiting Reagent is G = 5.70 g NaCl
143.5
6
\ Moles of B formed = ´ 4 ´ 0.5 = 2.4 This is the amount of NaCl in common salt;
5
23. (a) Let wt. of NH4NO3 and (NH4)2HPO4 are x 5.70
% purity = ´ 100 = 95%
and y gram respectively 6
x y 30. (a) Na 2CO 3 NaHCO3
´ 2 ´ 14 + ´ 2 ´ 14
80 132 ´100 = 30.4 x (1 – x)
x+y
x 1- x
Þ x : y= 2 : 1 = given (moles are equal )
106 84
24. (d) 2NH3 + 5F2 ¾¾ ® N2F4 + 6HF
2 × 17 g NH3 gives 66 g N2F4 x = 0.557

66 0.557 0.443 V ´ 0.1


2 g will give – ´ 2 = 3.88 g N 2 F4 + =
34 53 84 1000

3.56 V = 157.7 mL
% yield = ´ 100 = 91.75% 31. (c) C6H6 + HNO3 ® C6H5NO2 + H2O
3.88
78g 123g
wt. of solute Now since 78g of benzene on nitration
25. (a) ppm = ´ 106
wt. of solvent give = 123g nitrobenzene
hence 5g of benzene on nitration give
0.2
= ´ 106 = 400 123
500 = ´ 5 = 7.88g
26. (c) M1 V1 + M2V2 = M3V3 (V3 = V1 + V2) 78
1.5 × 480 + 1.2 × 520 = M3 × 1000 nearest answer is (c) i.e. theoritical yield
= 7.88 g
M3 =1.34 M
27. (b) CaCO3 + H2SO4 ® CaSO4 + H2O + CO2 32. (d) Normality of oxalic acid solution
1 mol 100 g 22.4 litre 6.3 ´ 1000
1 mol 10 g 2.24 litre = = 0.4 N
63 ´ 250
2.64 Now from
28. (a) Moles of C = nCO2 = = 0.06
44 N1V1 = N2V2
Þ mass of C = 0.72 0.4 × 10 = 0.1 × V2
Mole of H = 2 × Moles of H2O = V2 = 40 mL
Some Basic Concepts of Chemistry 9
37. (c) Normality
33. (b) X 2 O3 Þ X3∗O2,
= N1V1 - N 2 V2 = 0.2 ´ 100 - 0.1´100
X 2 Cl3 Þ X3+ Cl2 - V1 + V2 100 + 100
X 2 (SO 4 )3 Þ X3+ SO 4 2- 10
= = 0.05 N NaOH
3+ 3- 200
XPO 4 Þ X PO4
Because Cl2– does not exist. So, X2Cl3 is 38. (a) N 2 + O 2 ® 2 NO
incorrect. The correct formula should be a a 2a
XCl3.
34. (d) In an unknown compounds containing N 1
CO + O2 ® CO 2
and H 2
given % of H = 12.5%
(10 - a )
\ % of N = 100 – 12.5 = 87.5% (10 – a) (10 – a)
2
Element Percentage Atomic ratio Simple ratio
12.5 12.5
(10 - a )
H 12.5% = 12.5 =2 a+ =7
1 6.25 2
N 87.5
87.5
= 6.25
6.25
=1 \ a= 4
14 6.25 volume of CO = 6 mL
2 × vapour density = Mol. wt = mol wt. 6 ´ 100
= 16 × 2 = 32. Mole % of CO = = 60
10
Molecular formula = n × empirical formula
39. (b) Let weight of C be x g, then S will be (14 – x)g
32
mass n = =2 x / 12 2
16 =
\ Molecular formula of the compound will be (14 - x) / 32 1
= (NH2)2 = N2H4
6
\ x = 6 g ; Moles of C = = 0.5
C H N 12

35. (a) Moles


9 1 3.5
n=
108
=2 40. (a) mass of Al2 (SO 4 )3
12 1 14 54 ´106 = 34.2
mass of water
3 1
: 1 : 1 litre solution contains 1000 g of water
4 4
Þ In 1 litre solution, mass of
3 : 4 : 1
34.2 ´1000
Þ C3H4N Þ M.F. = (C3H4N)n Al 2 (SO4 )3 = = 34.2 mg
106
MF = (C3H4N)2 = C6H8N2
molarity of
36. (c) Mass of oxide = Mass of metal + Mass of
oxygen 34.2 ´ 10-3
Al 2 (SO4 )3 = M = 10-4 M
2.74 = 1.53 + WOxygen Þ WOxygen = 1.21 g 342
1.53
= 0.03 Al 2 (SO 4 )3 ( aq ) ¾¾
® 2Al3+ ( aq ) + 3SO 24 - ( aq )
Moles of V =
51 10–4 M 2 × 10–4 M 3 × 10–4 M
1.21 éSO24- ù = 3 ´10-4 M
Moles of O = = 0.075
16 ë û
V0.03 O0.075 41. (c) 74.75% of chlorine means 74.75g chlorine
V O 2.5 Þ V2O5 is present in 100g of metal chloride.
EBD_7587
10 Chemistry Objective MCQs
Weight of metal = 100g – 74.75g
16 ´ 4.12 16 ´ 1.12
= 25.25g \ = = x = 82.4
Equivalent weight x ´ 1000 22400
L.R.
weight of metal 46. (a) 4Ca 5 ( PO4 )3 F+ 18SiO 2 + 30C ¾¾
® 3P4 + 2CaF2
= ´ 35.5 0.1 0.36 0.9
weight of chlorine
+ 18CaSiO3 + 30CO
25.25
= ´ 35.5 = 12 18 moles of SiO2 gives 3 moles of P4
74.75
0.36 moles of SiO2 will give
Valency of metal
3
2 ´ V.D. = ´ 0.36 = 0.06 mole
= 18
Equivalent wt. of metal + 35.5
1
2 ´ 94.8 47. (b) CO + O 2 ® CO 2 ;
= =4 2
12 + 35.5
CO 2 + 2KOH ® K 2CO 3 + H 2O
\ Formula of compound = MCl4
28
N V + N 2 V2 + N 3 V3 Moles of KOH = = 0.50
42. (b) N= 1 1 56
Total volume
It corresponds to 0.25 mol of CO 2
50 ´10 + 25 ´ 12 + 40 ´ 5 Hence mol of CO = 1 – 0.25 = 0.75 º mole of
= = 1N
1000 CO 2 formed
43. (c) Molarity Mol of KOH requred = 2 × 0.75 = 1.5
Wt ´ 1000 2.65 ´ 1000 = 1.5 × 56 = 84 g
= = = 0.1 M
MW ´ V 106 ´ 250 1
48. (c) 3Fe 2O3 (s ) ® 2Fe3O 4 + O2
M1V1 = M2V2 2
\ 10 × 0.1 = 1000 × M2 = 0.001 M 480 g Fe2O3 provide 16 g O2. For loss of
44. (a) Normality of ferrous amm. sulphate 480
0.04 g O 2 ® 0.04 ´ = 1.2 g Fe 2 O3
= 3.92 ´1000 = 0.1 (Eq. wt of FAS is 392)
16
392 ´ 100
0.8
N1V1 = N2V2 % by mass of SiO2 = ´ 100 = 40%
2.0
20 × 0.1 = 18 × N2 N2 = 0.111 49. (a) CaCO3 ® CaO + CO2
1 g ev. of KMnO4 = 31.6 g
20 g 8.8 g 11.2 g
0.111 g ev. of KMnO4 = 31.6 × 0.111 = 3.5 g. mass of reactant = mass of products = 20g.
45. (c) Let the alcohol be ROH and x its molecular Hence the law of conservation of mass is
weight obeyed.
ROH + CH 3 MgI ® CH 4 + ROMgI 50. (c) The conditions given are standard
xg 16 g conditions
4.12 224 mL has mass = 1g ;
g of alcohol will produce
1000 22400 mL will have mass = 100g. This is mol.
16 4.12 wt of gas
´ g of methane
x 1000 6.023 × 1023 molecules have 3 × 6.023 × 1023
atoms since gas is triatomic
16 ´ 1.12
Methane actually obtained is = g \ weight of one atom
22400
Some Basic Concepts of Chemistry 11

100 2KMnO 4 + 3H 2SO 4 ¾


¾®
= = 5.5 ´ 10 - 23 g
23
3 ´ 6.023 ´ 10
K 2SO 4 + 2MnSO 4 + 3H 2O + 5 [O ]
51. (d) Specific volume (volume of 1 g) of nascent oxygen
cylindrical virus particle = 6.02 × 10–2 cc/g
Radius of virus (r) = 7 Å = 7 × 10–8 cm 2Fe(C2 O 4 ) + 3H 2SO 4 + 3 [O ] ¾¾
®
Length of virus = 10 × 10–8 cm Fe 2 (SO 4 ) 3 + 2CO 2 + 3H 2 O
Volume of virus = O required for 1 mol. of Fe(C2O4) is 1.5, 5 O
are obtained from 2 moles of KMnO4
22
pr 2 l = ´ (7 ´ 10 -8 ) 2 ´ 10 ´ 10 -8 \ 1.5 [O] will be obtained from
7
2
= 154 × 10–23 cc = ´ 1.5 = 0.6 moles of KMnO .
5 4
volume 56. (d) Weight of Iron in 67200
Wt. of one virus particle =
specific volume
0.33
\Mol. wt. of virus = Wt. of NA particle = ´ 67200 = 221.76
100
Number of atoms of Iron
154 ´ 10 -23
= ´ 6.02 ´ 10 23 = 15400 g/mol
6.02 ´ 10 -2 221.76
= = 3.96 º 4
56
= 15.4 kg/mol
57. (d) Volume of gold foil = 25 × 40 × 0.25 mm3
52. (b) Meq of oxalic acid in 16.68 ml = Meq of = 250 × 10–3 cm3
NaOH Mass of gold foil = 19.32 × 250 × 10–3 g
1 = 4.83 g
= 25 ×
15 4.83
No. of gold atoms = ´ NA
Meq of oxalic acid in 250 ml 197
1 250 = 1.47 × 1022
= 25 × × = 24.98 58. (c) Writing the equation of combustion of
15 16.68
propane (C3H8), we get
1.575
× 1000 = 24.98 C3H8 + 5O 2 ® 3CO2 + 4H 2O
(90 + 18x ) / 2 1vol 5vol
1L 5L
\ x= 2
From the above equation we find that we
53. (c) 5CO + I 2 O 5 ® 5CO 2 + I 2 need 5 L of oxygen at NTP to completely
25.4 burn 1 L of propane at N.T.P.
Moles of I 2 O 5 = If we change the conditions for both the
254
gases from N.T.P. to same conditions of
= 0.01 º 0.05 moles of CO temperature and pressure. The same results
Weight of CO = 0.05 × 28 = 1.4 g; are obtained. i.e. 5 L is the correct answer.
Weight of CO 2 = 2 – 1.4 = 0.6 g
59. (a) I 2 + 2Cl 2 ¾
¾® ICl + ICl3
0.6
Hence % of CO 2 = × 100 = 30% 25.4 14.2
2 No. of moles 0 0
54. (d) 3 molecules of O2 = 1 molecules of CS2 254 71
6 molecules of O2 = 2 molecules of CS2 initially 0.1 0.2 0 0
55. (b) The required equation is No. of moles 0 0 0.1 0.1
after reaction
EBD_7587
12 Chemistry Objective MCQs
60. (a) 2SO2 + O2 ¾® 2SO3 2HCl + Ba (OH) 2 ® BaCl 2 + 2H 2O
10 15 0
Total moles of Ba (OH) 2 required = 2
10 – 2x 15 – x 2x
66. (d) Let x g of Li2CO3 and (1 – x) g of M2CO3
Q 2x = 8 x = 4 present in given mixture
Hence, remaining, SO2 = 10 – 8 = 2 moles, 2 × Total moles of carbonates = moles of
HCl and
O2 = 15 – 4 = 11 moles
æ x 1- x ö -3
61. (d) 2´ç + ÷ = 21.6 ´ 0.5 ´10
è 74 2M + 60 ø
4 NH 3 (g ) + 5O 2 (g ) ¾
¾® 4 NO(g ) + 6H 2 O (l) x = 0.20
4 moles 5 moles 4 moles 6 moles x 1- x
Given 1 Mole 1 Mole Q 74 = 2M + 60
Reacting 0.8 1 0.8 1.2
M = 118
All O2 consumed being limiting reagent. 67. (a) m-eq.FeSO4 (NH4)2SO4.6H2O
62. (b) C2H6 + 3.5O2 —® 2CO2 + 3H2O; = m-eq. of KMnO4
C2H4 + 3O2 —® 2CO2 + 2H2O (n = 1)
Let volume of ethane is x litre, W
´ 1 ´ 1000 = 0.1 ´ 50
22.4 × 4 = 3.5x + 3(28 – x) 392
Þ x = 11.2 litre Hence, % purity of Mohr's salt
at constant T and P, V µ n; 1.96 W = 1.96 g
= ´ 100 = 78.4%
Þ Mole fraction of C2H6 in mixture 2.5
11.2 68. (a) heat
= = 0.4 2KClO3 ¾¾¾
® 2KCl + 3O 2
28 2´122.5g 3´32g
245 96
12.5 / 119
63. (c) XSn = 50 = 0.176 48 g of oxygen will be produced from
25 12.5 12.5
+ + + 122.5 g of KClO3
209 207 119 112
\ Amount of 80% KClO3 needed
64. (b) Ba(OH)2 + CO2 ¾¾
® BaCO3 + H2 O
100
1 mol 1 mol = ´ 122.5 = 153.12g
80
1 mol Ba(OH)2 = 1 mol BaCO3
69. (b) Mg + 2HCl ® MgCl2 + H 2 ­
\ 0.205 mol Ba (OH) 2 º 0.205 mol BaCO3 1 mole 1 mole
1 1 1
Wt. of substance = No. of moles × Molecular mole mole (12g of Mg = mol)
2 2 2
mass= 0.205 × 197.3 = 40.5 g 70. (a) Decomposition of Ag2CO3 gives silver
65. (b) SO 2Cl 2 + 2H 2O ® H 2SO 4 + 2HCl
2Ag 2CO3 ® 4Ag + 2CO2 + O 2
H 2SO 4 + Ba (OH) 2 ® BaSO 4 + 2H 2 O 2´276g 4´108g
2.76g 2.16g
2 Structure of Atom
1. What is the angular velocity (w) of an electron
occupying second orbit of Li2+ ion? 1.956 ´1016 1.19 ´ 108
(a) (b)
8p3 me4 8p3 me4 l l
(a) K2 (b) K2
h3 9h 3
2.859 ´ 105 2.859 ´ 1016
64 p me 3 4 3
9p me 4 (c) (d)
(c) ´ K 2 (d) K2 l l
9 3 3
h h
7. The energy of an electron in first Bohr orbit of H-
2. In a collection of H-atoms, all the electrons jump
from n = 5 to ground level finally (directly or atom is – 13.6 eV. The energy value of electron in
indirectly), without emitting any line in Balmer the excited state of Li2+ is:
series. Th e number of possible different (a) – 27.2 eV (b) 30.6 eV
radiations is : (c) – 30.6 eV (d) 27.2 eV
(a) 10 (b) 8 (c) 7 (d) 6 8. Based on the equation:
3. The wavelength of Ha line of Balmer series is X
Å. What is the X of Hb line of Balmer series. æ 1 1 ö
DE = -2.0 ´10 -18 J ç - ÷
çn 2 2÷
è 2 n1 ø

108 80 the wavelength of the light that must be absorbed


(a) X Å (b) X Å to excite hydrogen electron from level n = 1 to
80 108
level n = 2 will be:
1 80 1 108
(c) Å (d) Å (h = 6.625 × 10–34 Js, C = 3 × 108 ms–1)
X 108 X 80
(a) 1.325 × 10–7 m (b) 1.325 × 10–10 m
4. The number of photons of light having wave –7
number 'x' in 10 J of energy source is : (c) 2.650 × 10 m (d) 5.300 × 10–10 m
9. In an atom, an electron is moving with a speed of
hc
(a) 10hcx (b) 600 m/s with an accuracy of 0.005%. Certainity
10x
with which the position of the electron can be
10 located is ( h = 6.6 × 10–34 kg m2s–1, mass of
(c) (d) None of these
hcx
5. The values of Planck's constant is 6.63 × 10–34 Js. electron, em = 9.1 × 10–31 kg)
The velocity of light is 3.0 × 108 m s–1. Which
value is closest to the wavelength in nanometres of (a) 5.10 × 10 –3 m (b) 1.92 × 10 –3 m
a quantum of light with frequency of (c) 3.84 × 10 –3 m (d) 1.52 × 10 –4 m
8 × 1015 s–1? 10. Which of the following is not permissible
(a) 5 × 10–18 (b) 4 × 101 arrangement of electrons in an atom?
(c) 3 × 10 7 (d) 2 × 10–25
(a) n = 5, l = 3, m = 0, s = + 1/2
6. According to law of photochemical equivalence
the energy absorbed (in ergs/mole) is given as (b) n = 3, l = 2, m = – 3, s = – 1/2
(h = 6.62 × 10–27 ergs, c = 3 × 1010 cm s–1, (c) n = 3, l = 2, m = – 2, s = – 1/2
NA = 6.02 × 1023 mol–1) (d) n = 4, l = 0, m = 0, s = – 1/2
EBD_7587
14 Chemistry Objective MCQs
11. A dye absorbs a photon of wavelength l and re- 17. An electron, e1 is moving in the fifth stationary
emits the same energy into two photons of state, and another electron e2 is moving in the
wavelength l 1 an d l 2 r espectively. The fourth stationary state. The radius of orbit of
wavelength l is related with l1 and l2 as: electron, e1 is five times the radius of orbit of
electron, e2 calculate the ratio of velocity of
electron e1 (v1) to the velocity of electron e2 (v2).
l1 + l 2 l1l 2
(a) l= (b) l=
l1l 2 l1 + l 2 (a) 5 : 1 (b) 4 : 1 (c) 1 : 5 (d) 1 : 4
18. Among the following groupings which
l12 l22 l1l 2
(c) l= (d) l= represents the collection of isoelectronic species?
l1 + l 2 ( l1 + l 2 )2 (a) NO + , C 22 - , O -2 , CO
12. The Bohr's energy equation for H atom reveals (b) N 2 , C 22 - , CO, NO
that the energy level of a shell is given by
E = –13.58/n2eV. The smallest amount that an H (c) CO, NO + , CN - , C 22 -
atom will absorb if in ground state is (d) NO, CN - , N 2 , O -2
(a) 1.0 eV (b) 3.39 eV 19. The wave number of the first emission line in the
(c) 6.79 eV (d) 10.19 eV Balmer series of H-Spectrum is :
13. If m and e are the mass and charge of the revolving (R = Rydberg constant) :
electron in the orbit of radius r for hydrogen atom, 5 9 7 3
(a) R (b) R (c) R (d) R
the total energy of the revolving electron will be: 36 400 6 4
20. What is the maximum wavelength line in the
1 e2 e2
(a) (b) - Lyman series of He+ ion?
2 r r 1
(a) 3R (b)
me2 1 e2 4 3R
(c) (d) - (c) (d) None of these
r 2 r 4R
14. Excited hydrogen atom emits light in the 21. The position of both, an electron and a helium
ultraviolet region at 2.47 × 1015 Hz. With this atom is known within 1.0 nm. Further the
frequency, the energy of a single photon is: momentum of the electron is known within
(h = 6.63 × 10–34 Js) 5.0 × 10–26 kg ms–1. The minimum uncertainty
(a) 8.041 × 10–40 J (b) 2.680 × 10–19 J in the measurement of the momentum of the
(c) 1.640 × 10 J –18 (d) 6.111 × 10–17 J helium atom is
15. The orbital diagram in which the Aufbau principle is (a) 50 kg ms–1 (b) 80 kg ms–1
violated is : (c) 8.0 × 10 kg ms (d) 5.0 × 10–26 kg ms–1
–26 –1
2s 2p 22. What will be the difference between
electromagnetic radiation shown in A and B
(a) ­¯ ­¯ ­
respectively ?
(b) ­ ­¯ ­ ­

(c) ­¯ ­ ­ ­
(A)
(d) ­¯ ­¯ ­ ­
16. Calculate the velocity of ejected electron from
the metal surface when light of frequency
2 × 1015 Hz fall on the metal surface and the (B)
threshold frequency is 7 × 1014 Hz for metal ?

(i) Velocity (ii) Wavelength


(iii) Frequency (iv) Energy
(a) 1.37 × 106 (b) 1.26 × 106 (a) (ii) only (b) (ii) and (iv)
(c) 1.45 × 107 (d) 1.37 × 107 (c) (ii), (iii) and (iv) (d) (iv) only
Structure of Atom 15
23. Arrange the electromagnetic radiations a, b, c, 29. What is the maximum number of electrons that
d and e in increasing order of energy. can be accomodated in an atom in which the
Frequencies of a, b and c are 1015, 1014 and highest principal quantum number value is 4?
1017 respectively whereas wavelength of (d) (a) 10 (b) 18 (c) 36 (d) 54
and (e) are 350 nm and 100 nm respectively ? 30. The de Broglie wavelength of a car of mass
(a) a, b, c, d, e (b) a, b, d, e, c 1000 kg and velocity 36 km/hr is :
(c) a, d, b, e, c (d) b, a, d, e, c (a) 6.626 × l0–34 m (b) 6.626 × 10–38 m
(c) 6.626 × 10 m –31 (d) 6.626 × 10– 30 m
24. When electronic transition occurs from higher
energy state to lower energy state with energy 31. If the radius of first orbit of H atom is a0, the de-
difference equal to DE electron volts, the Broglie wavelength of an electron in the third
wavelength of the line emitted is approximately orbit is
equal to
12395 12395 (a) 4pa0 (b) 8pa0 (c) 6pa0 (d) 2pa0
(a) ´ 10 -10 m (b) ´ 1010 m 32. Which of the following radial distribution graphs
DE DE correspond to l = 2 for the H atom ?
12395 12395
(c) ´ 10-10 cm (d) ´ 1010 cm
DE DE
25. Which of the following should be the number of (a)
electrons present in X2+ on the basis of electronic
configuration, if the ion X3– has 14 protons ?
(a) 12 (b) 14 (c) 16 (d) 18
26. In a hydrogen atom, if energy of an electron in
ground state is 13.6. ev, then that in the 2nd excited (b)
state is
(a) 1.51 eV (b) 3.4 eV
(c) 6.04 eV (d) 13.6 eV.
27. At temperature T, the average kinetic energy of
3
any particle is KT . The de Broglie
2 (c)
wavelength follows the order :
(a) Visible photon > Thermal neutron >
Thermal electron
(b) Thermal proton > Thermal electron >
Visible photon
(c) Thermal proton > Visible photon > Thermal (d)
electron
(d) Visible photon > Thermal electron >
Thermal neutron
28. Which one of the following set of quantum
numbers is not possible for 4p electron? 33. In ground state of Cu + . The no. of shells
occupied, subshells, filled orbitals and unpaired
1
(a) n = 4, l = 1, m = –1, mS = + electrons respectively are :
2 (a) 4, 8, 15, 0 (b) 3, 6, 15, 1
1 (c) 3, 6, 14, 0 (d) 4, 7, 14, 2
(b) n = 4, l = 1, m = 0, mS = + 34. The Li2+ ion is moving in the third stationary
2 state, and its linear momentum is 7.3 × 10–34 kgms–1.
1 Angular momentum is.
(c) n = 4, l = 1, m = 2, mS = +
2
(a) 1.158 × 10–45 kg m2s–1
1 (b) 11.58 × 10–48 kg m2s–1
(d) n = 4, l = 1, m = –1, mS = -
2 (c) 11.58 × 10–47 kg m2s–1
(d) 12 × 10–45 kg m2s–1
EBD_7587
16 Chemistry Objective MCQs
35. The de Broglie wavelength of a tennis ball of
3 3
mass 60 g moving with a velocity of 10 metres (a) + x (b) - x
per second is approximately 2 8
(a) 10–31 (b) 10–16 (c) 10–25 (d) 10–33 3 3
36. In an atom how many orbital(s) will have the (c) + x (d) - x
4 4
quantum numbers; n = 3, l = 2 and ml = + 2 ?
42. The kinetic and potential energy (in eV) of an
(a) 5 (b) 3 (c) 1 (d) 7
electron present in third Bohr's orbit of hydrogen
37. According to Bohr's theory, the angular atom are respectively:
momentum of an electron in 5th orbit is (a) –1.51, –3.02 (b) 1.51, –3.02
(c) –3.02, 1.51 (d) 1.51, –1.51
43. What atomic number of an element "X" would
(a) 10 h/p (b) 2.5 h/p have to become so that the 4th orbit around X
(c) 25 h/p (d) 1.0 h/p would fit inside the I Bohr orbit of H atom?
(a) 3 (b) 4 (c) 16 (d) 25
38. The energy of electron in first energy level is
44. Which combinations of quantum numbers, n, l,
- 21.79 ´ 10 -12 erg per atom. The energy of m and s for the electron in an atom does not
electron in second energy level is : provide a permissible solution of the wave
(a) - 54.47 ´10-12 erg atom-1 equation ?
1 1
(b) - 5.447 ´ 10-12 erg atom-1 (a) 3,2,1, + (b) 3,1,1, -
2 2
(c) - 0.5447´10-12 erg atom-1
1 1
-12 -1 (c) 3, 3,1, - (d) 3,2, - 2, +
(d) - 0.05447´10 erg atom 2 2
39. Which of the following electronic configuration 45. If the nitrogen atom had electronic configuration
of d-orbital will have highest affinity for gaining 1s7 it would have energy lower than that of the
an electron? normal ground state configuration 1s2 2s2 2p3
because the electrons would be closer to the
(a) nucleus. Yet 1s7 is not observed. It violates
(a) Heisenberg’s uncertainty principle
(b) (b) Hund’s rule
(c) Pauli exclusion principle
(c) (d) Bohr postulate of stationary orbits
46. The position of both, an electron and a helium
atom is known within 1.0 nm. Further the
(d)
momentum of the electron is known within
40. If the de-Broglie wavelength of a particle of mass 5.0 × 10–26 kg ms–1. The minimum uncertainty
m is 100 times its velocity, then its value in terms in the measurement of the momentum of the
of its mass (m) and Planck’s constant (h) is helium atom is
(a) 50 kg ms–1
(b) 80 kg ms–1
(c) 8.0 × 10–26 kg ms–1
1 m h (d) 5.0 × 10–26 kg ms–1
(a) (b) 10 47. Two fast moving particles X and Y are associated
10 h m
with de Broglie wavelengths 1 nm and 4 nm
1 h m respectively. If mass of X is nine times the mass
(c) (d) 10 of Y, the ratio of kinetic energies of X and Y would
10 m h
41. An electron in an atom jumps in such a way that be
x
its kinetic energy changes from x to . The (a) 3 : 1 (b) 9 : 1 (c) 5 : 12 (d) 16 : 9
4
change in potential energy will be:
Structure of Atom 17
48. In a multi-electron atom, which of the following
orbitals described by the three quantum members 56. The velocity of particle A is 0.1 ms–1 and that of
will have the same energy in the absence of particle B is 0.05 ms–1. If the mass of particle B is
magnetic and electric fields? five times that of particle A, then the ratio of de-
(A) n = 1, l = 0, m = 0 (B) n = 2, l = 0, m = 0 Broglie wavelengths associated with the particles
(C) n = 2, l = 1, m = 1 (D) n = 3, l = 2, m = 1
(E) n = 3, l = 2, m = 0 A and B is
(a) (D) and (E) (b) (C) and (D)
(c) (B) and (C) (d) (A) and (B) (a) 2 : 5 (b) 3 : 4 (c) 6 : 4 (d) 5 : 2
49. A system irradiated for 10 min. is found to absorb 57. Let mp be the mass of a proton, mn that of a
3 × 10 18 quanta per sec. If the amount of
20
neutron, M1 that of a 10
substance decomposed is 3 × 10–3 mol Ne nucleus and M2 that
( NA = 6 × 1023). The quantum efficiency of the 40
of a nucleus. Then
20 Ca
reaction is
(a) 2.5 (b) 2 (c) 1.5 (d) 1 (a) M2 = 2M1 (b) M1<10(mp + mn)
50. The radii of maximum probability for 3s, 3p and (c) M2 > 2M1 (d) M1 = M2
3d electrons are in the order 58. If radius of second stationary orbit (in Bohr's
(a) (rmax )3d > (rmax )3 p > (rmax )3s atom) is R then radius of third orbit will be :
(b) (rmax )3d > (rmax )3s > (rmax )3 p (a) R/3 (b) 9 R (c) R/9 (d) 2.25 R
(c) ( rmax ) 3 s > ( rmax )3 p > ( rmax )3 d 59. The dissociation energy of H2 is 430.53 kJmol–1.
(d) none of these If hydrogen is dissociated by illumination with
51. If in Bohr's model, for unielectronic atom, time radiation of wavelength 253.7 nm the fraction of
period of revolution is represented as Tn, z where
n represents shell no. and Z represents atomic the radiant energy which will be converted into
number then the value of T 1, 2 : T2, 1 will be : kinetic energy is given by
(a) 8 : 1 (b) 1 : 8 (c) 1 : 1 (d) 1 : 32 (a) 100% (b) 8.82% (c) 2.22% (d) 1.22%
52. Find the value of wave number ( v ) in terms of 60. The threshold frequency of a metal is
Rydberg's constant, when transition of electron
takes place between two levels of He+ ion whose 1 × 1015 s -1 . The ratio of the maximum kinetic
sum is 4 and difference is 2. energies of the photoelectrons when the metal is
irradiated with radiations of frequencies
8R 32R
(a) (b) 1.5 × 1015s–1 and 2.0 × 1015s–1 respectively would
9 9
be
3R
(c) (d) None of these (a) 4 : 3 (b) 1 : 2 (c) 2 : 1 (d) 3 : 4
9
61. The wave number of electromagnetic radiation
53. The uncertainty in the position of an electron
(mass = 9.1 × 10–28 g) moving with a velocity of emitted during the transition in between two
3.0 × 104 cm s–1 accurate upto 0.011% will be energy levels of Li2+ ion whose principal quantum
(a) 1.92 cm (b) 7.68 cm number sum is 4 and difference is 2, is
(c) 0.175 cm (d) 3.84 cm
54. The energy of e – in first orbit of He + is
–871.6 × 10–20 J. The energy of e– in first orbit of
H is: 8
(a) 3.5 RH (b) 4 RH (c) 8 RH (d) RH
(a) – 871.6 × 10–20 (b) –435.8 × 10–20 J 9
(c) – 217.9 × 10 J (d) –108.9 × 10–20
–20

55. If a proton and a-particle are accelerated through 62. An electron travels with a velocity of x ms–1. For
the same potential difference, the ratio of de- a proton to have the same de-Broglie
Broglie wavelengths lp and la is wavelength, the velocity will be approximately :
(a) 3 (b) 1840 x
2 2 (c) 1 (d) 2 (a) (b) (c) 1840 x (d) x
x 1840
EBD_7587
18 Chemistry Objective MCQs
63. The total spin and magnetic moment for the atom electron comes back to the ground state ?
with atomic number 24 are: (a) 4 ® 1
± 3, 35 BM (b) 4 ® 2, 2 ® 1
(a) ± 3, 48 BM (b)
(c) 4 ® 3, 3 ® 2, 2 ® 1
3 3 (d) All of the above
(c) ± , 48 BM (d) ± , 35 BM
2 2 66. What should be the kinetic energy and total
64. Select the incorrect graph for velocity of e– in an energy of the electron present in hydrogen atom,
1 if its potential energy is –5.02 eV?
orbit vs. Z, and n :
n (a) –2.51 eV, +2.51eV
(b) + 0.251 eV, –0.251eV
(c) –2.51 eV, –2.51 eV
(d) +2.51 eV, +2.51eV
(a) v 67. Ionization potential of hydrogen atom is 13.6eV.
Hydrogen atom in ground state are excited by
n monochromatic light of energy 12.1 eV. The
spectral lines emitted by hydrogen according to
Bohr's theory will be

(a) one (b) two (c) three (d) four


68. If the subsidiary quantum number of a subenergy
v
(b) level is 4, the maximum and minimum values of
the spin multiplicities are :
1/n (a) 9, 1 (b) 10, 1 (c) 10, 2 (d) 4, –4
69. If the wavelength of the electromagnetic radiation
is increased to thrice the digital value, then what
will be the percent change in the value of
frequency of the electromagnetic radiation.
v
(c)

Z (a) Increases by 33% (b) Decreases by 33%


(c) Increases by 66% (d) Decreases by 66%
70. According to Bohr’s theory the energy required
for the transition of H-atom from n = 6 to n = 8
state is
(d) v (a) equal to the energy required for the
transition from n = 5 to n = 6 state
n (b) larger than in (a)
(c) less than in (a)
65. An electron has been excited from the first to the
(d) equal to the energy required for the
fourth energy state in an atom. Which of the
transition from n = 7 to n = 9 state.
following transitions are possible when the
Structure of Atom 19

Answer KEY
1 (d) 8 (a) 15 (b) 22 (c) 29 (c) 36 (c) 43 (c) 50 (c) 57 (a) 64 (d)
2 (d) 9 (b) 16 (a) 23 (d) 30 (b) 37 (b) 44 (c) 51 (d) 58 (d) 65 (d)
3 (b) 10 (b) 17 (d) 24 (a) 31 (c) 38 (b) 45 (c) 52 (b) 59 (b) 66 (a)
4 (c) 11 (b) 18 (c) 25 (a) 32 (c) 39 (d) 46 (d) 53 (c) 60 (b) 67 (c)
5 (b) 12 (d) 19 (a) 26 (a) 33 (c) 40 (b) 47 (d) 54 (c) 61 (c) 68 (c)
6 (b) 13 (d) 20 (b) 27 (d) 34 (b) 41 (a) 48 (a) 55 (b) 62 (b) 69 (d)
7 (c) 14 (c) 21 (d) 28 (c) 35 (d) 42 (b) 49 (d) 56 (d) 63 (a) 70 (c)

n 2h2 - 1 æ 1 1 ö 3R
1. (d) v n = rn w where rn = v= = Rç 2 - 2÷ =
2 2
4 p me Z × K λβ è2 4 ø 16
2
2p× Z × e × K 16
and v n = \ λβ = =X
n×h 3R
2pZe 2 × K n 2h 2 36
\ = ´ w; when =X
n ×h 4p2 me2 Z × K 3R
8p3 me 4 × Z2 × K 2 16 X ´ 5R ´16 80X
w= Then = = Å
3R 36 ´ 3R 108
n 3 × h3
9 p3 me 4 × K 2
nhc æ 1ö
E= = nhcv çQ v= ÷
= (Q n = 2 and Z = 3) 4. (c)
l è lø
h3
5 10 10
2. (d) (2) (4)
\ 10 = nhcv or n = =
hcv hcx
4 ch c
(1) (3) (6) 5. (b) E = hn = ;and n =
3 l l
8
2 3.0 ´ 10
(5) 8 ´ 1015 =
l
1
Total radiations are = 6 3.0 ´ 108
3. (b) Ha line of Balmer series means first line of \ l= = 0.37 ´10 -7
8 ´1015
Balmer series.
n1 = 2, n2 = 3 = 37.5 ´ 10-9 m = 4 ´ 101 nm

- 1 æ 1 1 ö 5R hc
v= = Rç - ÷= 6. (b) E = ´ NA
la è2 2
32 ø 36 l

36 6.62 ´ 10-27 ´ 3 ´ 1010 ´ 6.02 ´ 10 23


\l a = =X =
5R l
1.19 ´ 108
Hb line of Balmer series means, second line =
l
of Balmer series, n1 = 2, n2 = 4.
EBD_7587
20 Chemistry Objective MCQs
7. (c) For Li2+ ion 14. (c) E = hn
Z 2
( 3)2 = 6.63 × 10–34 × 2.47 × 1015
E = – 13.6 × 2
= -13.6 ´ = 1.640 × 10–18 J
n ( 2 )2 15. (b) According to Aufbau principle, the orbital
-13.6 ´ 9 of lower energy (2s) should be fully filled
= = -30.6 eV
4 before the filling of orbital of higher energy
æ 1 1ö starts.
8. (a) DE = -2.0 ´ 10 -18 ´ ç 2 - 2 ÷
è2 1 ø 1 2
-18 -3
16. (a) mn = hn - hn0
= -2.0 ´ 10 ´ 2
4
= 1.5 × 10–18 1
Þ mn 2 = h ( n - n 0 )
hc 2
DE =
l
2h
hc 6.6 ´ 10 -34 ´ 3 ´ 108 Þ n= (n - n 0 )
l= = m
DE 1.5 ´ 10 -18
= 1.325 × 10 m–7
2´ 6.6 ´10-34
9. (b) According to Heisenberg uncertainty = (
2 ´1015 - 7 ´1014 )
principle. 9.1´10-31
h = 1.37 × 106 ms–1
Dx.mDv = 17. (d) From the expression of Bohr ’s theory, we
4p
know that
h
Dx = h
4 pm Dv me v1r1 = n1
600 ´ 0.005 2p
Here Dv = = 0.03
100 h
6.6 ´ 10 -34 & me v 2 r2 = n2
So, Dx = 2p
4 ´ 3.14 ´ 9.1´ 10-31 ´ 0.03 me v1r1 n1 h 2p
= 1.92 × 10–3 = ´
10. (b) m = – l to +l, through zero thus for l = 2, me v2 r2 n2 2p h
values of m will be – 2, –1, 0, + 1, + 2. Given, r1 = 5 r2, n1 = 5, n2 = 4
Therefore for l = 2, m cannot have the
value –3. me ´ v1 ´ 5r2 5
=
hc hc hc me ´ v2 ´ r2 4
11. (b) E = E1 + E 2 ; = +
l l1 l2 v1 5 1
Þ = = = 1: 4
hc æ l + l1 ö l1l 2 v2 4 ´ 5 4
Þ = hc ç 2 ÷; l =
l l l
è 1 2 ø l1 + l2 18. (c) The species CO, NO+, CN– and C22– contain
12. (d) The smallest value of energy of an electron 14 electrons each.
in H atom in ground state can absorb is æ 1 1 ö
= E2 – E1. 19. (a) v = RZ2 ç - ÷
2
è2 32 ø
- 13.58 æ - 13.58 ö
= -ç ÷ = 10.19 5R
= R æç - ö÷ =
4 1 1
è 1 ø
13. (d) Total energy of a revolving electron is the è4 9ø 36
sum of its kinetic and potential energy.
1 é 1 1 ù é1 1 ù
Total energy = K.E. + P. E. 20. (b) = RZ2 ê - ú = R ´ 2 2 ê - ú
l ëê 1
n 2
n 2
2 úû ë 12
2 2
û
e 2 æ e2 ö 2
= +ç- ÷ = - e
2r è r ø 2r 1
Þ 3R; l =
3R
Structure of Atom 21
21. (d) According to Heisenberg uncertainty X = 14 = 1s2, 2s2 2p6, 3s2 3p2
principle, X2+ = 1s2, 2s2 2p6, 3s2 = 12 electrons
h 26. (a) 2nd excited state will be the 3rd energy level.
Dx ´ Dp = (which is constant).
4p 13.6 13.6
As Dx for electron and helium atom is same, En = 2
eV or E = eV = 1.51 eV.
n 9
thus momentum of electron and helium will
also be same, therefore the momentum of 3
27. (d) Kinetic energy of any particle = KT
helium atom is equal to 5 × 10–26 kg ms–1. 2
22. (c) e/m waves shown in figure A has higher 1
wavelength in comparison to e/m waves Also K.E. = mv 2
2
shown in figure B.
Thus these waves also differ in frequency 1 2 3 3KT
mv = KT Þ v2 =
c 2 2 m
and energy. n =
l 3KT
v=
l1 m
De-broglie wavelength
(A) h h
= l= =
mv 3KT
m
m
hc h 1
Þ E1 = l= lµ
l1 3KTm m
Mass of electron < mass of neutron
l (electron) > l (neutron)
l2 28. (c) For 4p electron n = 4, l = 1, m = –1, 0 + 1 and
(B) s = +½ or –½
29. (c) Energy of subshells follow the order 1s2,
2s2, 2p6, 3s2, 3p6, 4s2, 3d10, 4p6, 5s, 4d, 5p,
6s..... we can fill them by electrons upto 4p
hc
Þ E2 = l1 > l2 Þ E1 < E2 hence 36 electrons.
l2 h
30. (b) l =
23. (d) E = hn mv
h = 6.6 × 10–34 J-s
æcö
and n=ç ÷ m = 1000 kg
èlø
36 ´ 103
na = nb =
1015, 1014, v = 36 km/hr = m/sec = 10 m/sec
60 ´ 60
nc = 1017, nd = 0.85 × 1015 6.6 ´ 10-34
and ne = 3 × 1015, \ l= = 6.6 ´ 10-38 m
103 ´ 10
hc 31. (c) rn = a0n2
24. (a) l= r = a0 × (3)2 = 9a0
DE
nh
6.62 ´ 10-34 J s ´ 3 ´ 108 ms -1 mvr = ;
= 2p
E ´ 1.602 ´ 10 -19 J 3h h
nh = =
12395 mv =
2 pr 2 p ´ 9 a 6 p a0
= ´10-10 m 0
E
h h
25. (a) X3– has 14 protons, i.e., X also has 14 l= = ´ 6 pa0 = 6 pa0
mv h
protons and therefore 14 electrons.
EBD_7587
22 Chemistry Objective MCQs
32. (c) l = 2 represent d orbital for which E1
En = –
n2
r 2 y2 where E1 is energy of first energy level

E2 = -
E1
=-
E1 -21.79 ´ 10 -12
a0 =
22 4 4
33. (c) The ground state electronic configuration,
= – 5.447 × 10–12 erg per atom.
of Cu+ ion is
39. (d) The d-orbital represented by option (d) will
1s2, 2s2, 2p6, 3s2, 3p6, 3d10
become completely filled after gaining an
n = 3 (number of shells) ; Number of
electron. Therefore option (d) is correct.
subshells occupied = 6, Number of filled
40. (b) Let wavelength of particle be x
orbitals = 14 ; There is no unpaired electron.
34. (b) Z = 3 for Li2+ ions x
So, velocity (v) =
100
52.9 ´ n2
So rn = pm
Z h h ´ 100
l= ; x=
n = 3, Z = 3 mv m´x
2
52.9 ´ (3) x 2 = 100
h
or x = 10
h
rn = pm m m
3
= 158.7 pm 2x 3
Also, linear momentum (mv) = 7.3 × 10–34 41. (a) Change in P.E. = - + (2x) = x
4 2
kg ms–1
Then angular momentum will be -13.6
42. (b) Total energy of third shell =
w = (mv) × r 32
= (7.3 × 10–34 kg ms–1) (158.7 pm) = –1.51 eV
= 7.3 × 10–34 kg ms–1 × (158.7 × 10–12 m) K.E. = – Total energy Þ 1.51 eV
= 11.58 × 10–48 kg m2 s–1 P.E. = 2 × T.E. = –3.02 eV

h 6.6 ´ 10 -34 n2
35. (d) l= = = 10 -33 m 43. (c) r1 = 0.529 Å; R4(x) = R1 × ;
mv 60 ´ 10 -3 ´ 10 Z
0.529 ´ ( 4)
36. (c) n = 3, l = 2 means 3d orbital 2
R 4( x ) Þ ; Z = 16
Z
44. (c) Possible values of l and m depend upon the
+2 +1 0 –1 –2 value of n
i.e. in an atom only one orbital can have the l = 0 to (n – 1)
value ml = + 2 m = –l to + l through zero
37. (b) Angular momentum of an electron in nth 1 1
orbit is given by s = + and -
2 2
nh Thus for n = 3,
mvr =
2π l may be 0, 1 or 2; but not 3
For n = 5, we have m may be –2, –1, 0, +1 or +2
Angular momentum of electron 1 1
s may be + or -
5h 2.5h 2 2
= =
2p p 45. (c) Not more than two electrons can be present
38. (b) If we assume the atom to be hydrogen like, in same atomic orbital. This is Pauli's
energy of nth energy level exclusion principle.
Structure of Atom 23
46. (d) By Heisenberg uncertainty Principle
h 54. (c) E1 + = E1 ´ Z2
Dx ´ Dp = (which is constant) He H
4p
As Dx for electron and helium atom is same \ -871.6 ´ 10
-20 = E ´4
1 H
thus momentum of electron and helium will
also be same therefore the momentum of -20 J
\ E1 = -217.9 ´10
helium atom is equal to 5 × 10–26 kg. m.s–1. H
h h
47. (d) de Broglie wavelength l = lp =
mv 55. (b) ;
2eVmp
l1 m2 v2 1 1 v2
= ; = ´
l 2 m1v1 4 9 v1 h h
l
v2 9 v1 4 He 2 + = 2 ´ 2eVm
=
2 ´ 2eV ´ 4m p
= ; = He2 +
v1 4 v2 9
1 2 lp
KE = mv \ = 2 2
2 l
He 2 +
2
KE1 m1 v12 9 æ 4 ö 16 56. (d) Given, vA = 0.1 ms–1 and vB = 0.05 ms–1 also,
= ´ = ´ç ÷ =
2
KE2 m2 v2 1 è 9 ø 9 mB = 5mA
48. (a) The energy of an orbital is given by (n + l). h
de-Broglie wavelength, l =
In (D) and (E), (n + l) value is (3 + 2) = 5, mv
hence they will have same energy, since here
n values are also same. l A h / m A v A m B vB
\ = =
49. (d) Quantum efficiency l B h / m B v B m A vA
Number of moles of substance reacted 5m A ´ 0.05
= = = 5 ´ 0.5 = 2.5 = 5 / 2
Number of Quanta absorbed m A ´ 0.1

3 ´ 10-3 ´ 6 ´ 1023 \ lA : lB = 5 : 2
= 18 =1
3 ´ 10 ´ 60 ´ 10 57. (a) 20
10 Ne contains 10 protons and 10 neutrons
50. (c) Radius of maximum probability decrease \ M1 = 10 mp + 10mn
with increase in azimuthal quantum no. 40
20 Ca contains 20 protons and 20 neutrons
3 T1,2
n 1 1 1 \ M2 = 20 mp + 20 mn
51. (d) Tµ ; = ´ =
Z2 T2,1 4 8 32 \ M2 = 2M1
52. (b) n1 + n2 = 4; n2 – n1= 2; \ n1 = 1, n2 = 3 2
r1 æ n1 ö 4
2é1 1 ù 32R 58. (d) =ç ÷ =
n = R ( 2) ê - ú = r2 è n 2 ø 9
2
ë1 32 û 9
h h 59. (b) Energy of 1 mole of photons,
53. (c) Dx . Dp = or Dx . mDv = ;
4p 4p E = N0 × h u
0.011 N0 ´ h ´ c
Dv = ´ 3 ´ 10 4 = 3.3 cms -1 =
100 l
6.6 ´10-27 6.023 ´ 10 23 ´ 6.63 ´ 10 -34 ´ 3 ´ 10 8
Dx = = 0.175 cm
4 ´ 3.14 ´ 9.1´ 10-28 ´ 3.3
=
253.7 ´ 10 -9
EBD_7587
24 Chemistry Objective MCQs
Energy converted into KE = (472.2 – 430.53) kJ 67. (c) After absorbing 12.1 eV the electron in H
% of energy converted into atom is excited to 3 shell.
(472.2 - 430.53) -13.6
KE =
472.2
× 100 = 8.82% E n - E1 = - (- 13.6) = 12.1
n2
(KE )1 ν1 - ν 0
60. (b) =
ν 2 - ν0
-13.6
(KE ) 2 + 13.6 = 12.1
n2
(1.5 ´1015 - 1 ´1015 ) 1 \n=3
= =
(2.0 ´ 1015 - 1 ´1015 ) 2 The possible transitions are = 3
61. (c) n1 + n2 = 4 ...... (i) 68. (c) l = 4;
number of degenerate orbitals = 2l + 1 = 9;
n2 - n1 = 2 ..... (ii)
n1 = 1, n2 = 3 1
maximum total spins = 9 ´
2
2
æ 1 1ö
u = RH Z ç 2 - 2 ÷ maximum multiplicity = 2s + 1
è n1 n2 ø
9
= 2 ´ + 1 = 10
é1 1 ù 2
= RH ´ 32 ´ ê - ú = 8 RH
ë11 32 û
1
minimum total spins =
h h h 2
62. (b) l= = =
me x m p V 1840 me V
1
é\ m p = 1840 me ù minimum multiplicity = 2 ´ + 1 = 2
ë û 2
x c
Hence, V = 69. (d) n1 =
1840 ll
63. (a) Atomic number = 24
Electronic configuration is 3d 5, 4s1
c c
No. of valence e–1(s) = 6 n2 = =
l 2 3l1
1
Total spin = ± ´ 6 = ± 3 n 2 - n1
2 % change in frequency = ´ 100
Magnetic moment n1

= n ( n + 2 ) = 6 (6 + 2 )
c c 2c
- - ´ 100
=48 BM 3l1 l1 3l1
= ´ 100 =
64. (d) v = 2.188 × 106 Z/n c c
65. (d) Electron can undergo transition from higher l1 l1
state to all lower states by loss of energy.
Potential energy -5.02 = – 66%
66. (a) Total energy = =
2 2 70. (c) The difference between the energy of
Total energy = – 2.51 eV adjacent energy levels decreases on moving
Kinetic energy = – Total energy away from nucleus.
= – (–2.51 eV) = + (2.51 eV)
Classification of Elements
3 and Periodicity in Properties
1. Na+, Mg2+, Al3+ and Si4+ are isoelectronic. The 7. In the long form of the periodic table, the valence
order of their ionic size is shell electronic configuration of 5s 2 5p 4
(a) Na+ > Mg2+ < Al3+ < Si4+ corresponds to the element present in :
(b) Na+ < Mg2+ > Al3+ > Si4+ (a) Group 16 and period 6
(c) Na+ > Mg2+ > Al3+ > Si4+ (b) Group 17 and period 6
(d) Na+ < Mg2+ > Al3+ < Si4+ (c) Group 16 and period 5
2. Consider the following changes (d) Group 17 and period 5
8. Consider the following ionization enthalpies of
A ® A + + e - : E1 and A + ® A 2 + + e - : E 2 two elements ‘A’ and ‘B’.
The energy required to pull out the two electrons Element Ionization enthalpy (kJ/mol)
are E 1 and E 2 r espectively. The correct
1st 2nd 3rd
relationship between two energies would be
A 899 1757 14847
(a) E1 < E2 (b) E1 = E2
B 737 1450 7731
(c) E1 > E2 (d) E1 ³ E2
3. The elements with zero electron affinity are Which of the following statements is correct ?
(a) Boron and Carbon
(b) Beryllium and Helium
(c) Lithium and Sodium (a) Both ‘A’ and ‘B’ belong to group–1 where
(d) Fluorine and Chlorine ‘B’ comes below ‘A’.
4. Ionization energies of atoms A and B are 400 and (b) Both ‘A’ and ‘B’ belong to group–1 where
300 k cal mol –1 respectively. The electron ‘A’ comes below ‘B’.
affinities of these atoms are 80.0 and 85.0 k cal (c) Both ‘A’ and ‘B’ belong to group–2 where
mol–1 respectively. Then which is the correct ‘B’ comes below ‘A’.
statement regarding electronegativity c ? (d) Both ‘A’ and ‘B’ belong to group–2 where
‘A’ comes below ‘B’.
9. Sum of first three ionization energies of Al is
(a) cA < cB (b) c A > cB 53.0 eV atom–1 and the sum of first two ionization
energies of Na is 52.2 eV atom–1. Out of Al(III)
(c) c A = cB (d) none of these
and Na(II)
5. The set of three elements having successive
atomic numbers and having the ionization
(a) Na (II) is more stable than Al (III)
energies of 2372, 520 and 890 kJ per mol is
(b) Al (III) is more stable than Na (II)
(a) H, He, Li (b) He, Li, Be
(c) Both are equally stable
(c) Li, Be, B (d) B, C, N
(d) Both are equally unstable
6. Similarity in chemical properties of the atoms of
elements in a group of the Periodic table is most 10. Which pair of elements belongs to same group ?
closely related to: (a) Elements with atomic no. 17 and 38
(a) atomic numbers (b) Elements with atomic no. 20 and 40
(b) atomic masses (c) Elements with atomic no. 17 and 53
(c) number of principal energy levels (d) Elements with atomic no. 11 and 33
(d) number of valence electrons
EBD_7587
26 Chemistry Objective MCQs

11. In a given energy level, the order of penetration 18. Which of the following sequence correctly
effect of different orbitals is represents the decreasing acidic nature of
(a) ¦ < d < p < s (b) s = p = d = ¦ oxides ?
(c) s < p < d < ¦ (d) p > s > d > ¦ (a) Li2O > BeO > B2O3 > CO2 > N2O3
12. Consider the following changes : (b) N2O3 > CO2 > B2O3 > BeO > Li2O
(1) M(s) ¾® M(g) (c) CO2 > N2O3 > B2O3 > BeO > Li2O
(2) M(s) ¾® M2+(g) + 2e– (d) B2O3 > CO2 > N2O3 > Li2O > BeO
(3) M(g) ¾® M+(g) + e– 19. Correct order of first IP among following elements
Be, B, C, N, O is
(4) M+(g) ¾® M2+(g) + e–
(a) B < Be < C < O < N
(5) M(g) ¾® M2+(g) + 2e– (b) B < Be < C < N < O
The second ionization energy of M could be (c) Be < B < C < N < O
calculated from the energy values associated with: (d) Be < B < C < O < N
20. In which of the following process highest
(a) 1 + 3 + 4 (b) 2 – 1 + 3 energy is absorbed?
(c) 1 + 5 (d) 5 – 3
(a) Cu ® Cu + (b) Br ® Br -
13. The correct order of decreasing electronegativity +
values among the elements I-beryllium, (c) I ® I- (d) Li ® Li
II-oxygen, III-nitrogen and IV-magnesium is 21. The correct order of acidic strength :
(a) II > III > I > IV (b) III > IV > II > I
(c) I > II > III > IV (d) I > II > IV > III
(a) Cl2O7 > SO2 > P4O10
14. What is the order of ionisation energies of the
(b) K2O > CaO > MgO
coinage metal
(c) CO2 > N2O5 > SO3
(a) Cu > Ag < Au (b) Cu > Ag > Au
(d) Na2O > MgO > Al2O3
(c) Cu < Ag < Au (d) Au > Ag < Cu
22. An element of atomic weight 40 has 2, 8, 8, 2 as
15. First three ionisation energies (in kJ/mol) of three
the electronic configuration. Which one of the
representative elements are given below :
following statements regarding this element is
Element IE1 IE2 IE3 not correct?
P 495.8 4562 6910 (a) it belongs to II group of the periodic table
Q 737.7 1451 7733 (b) it has 20 neutrons
(c) the formula of its oxide is MO2
R 577.5 1817 2745
(d) it belongs to 4th period of the periodic table
Then incorrect option is : 23. Which of the following series corr ectly
(a) Q : Alkaline earth metal represents relations between the elements from
(b) P : Alkali metals X to Y?
(c) R : s-block element X®Y
(d) They belong to same period (a) 3 Li ® 19 K Ionization enthalpy increases
16. What is the correct order of electronegativity?
(a) M– < M2– < M3– < M4– (b) 9 F ® 35 BrElectron gain enthalpy (negative
(b) M– > M2– > M3– > M4– sign) increases
(c) M– < M2– > M3– < M4– (c) 6 C ® 32Ge Atomic radii increases
(d) M4– < M2– < M3– < M– (d) 18 Ar ® 54 Xe Noble character increases
17. The chemistry of lithium is very similar to that
24. Which of the following arrangements represents
of magnesium even though they are placed in
the increasing order (smallest to largest) of ionic
different groups. Its reason is: radii of the given species O2–, S2–, N3–, P3–?
(a) Both are found together in nature
(a) O2– < N3– < S2– < P3–
(b) Both have nearly the same size
(b) O2– < P3– < N3– < S2–
(c) Both have similar electronic configuration
(c) N3 < O2– < P3– < S2–
(d) The ratio of their charge and size (i.e. charge
density) is nearly the same (d) N3– < S2– < O2– < P3–
Classification of Elements and Periodicity in Properties 27

25. Which one of the following has largest ionic 32. Which is the correct order of second ionization
radius? potential of C, N, O and F in the following ?
(a) O > N > F > C (b) O > F > N > C
(a) Li+ (b) O 22 - B3+ (d) F
(c) –
(c) F > O > N > C (d) C > N > O > F
26. Elements X, Y and Z have atomic numbers 19, 37 33. The first (DiH1) and second (DiH2) ionization
and 55 respectively. Which of the following enthalpies (in kJ mol–1) and the electron gain
statements is true about them ? enthalpy (DegH) (in kJ mol–1) of the elements I, II,
III, IV and V are given below.
(a) Their ionization potential would increase
with increasing atomic number
(b) ‘Y’ would have an ionisation potential Element DiH1 DiH2 DegH
between those of ‘X’ and ‘Z’ I 520 7300 – 60
(c) ‘Z’ would have the highest ionization II 419 3051 – 48
potential III 1681 3374 – 328
(d) ‘Y’ would have the highest ionization IV 1008 1846 – 295
potential. V 2372 5251 + 48
27. An element X occurs in short period having The most reactive metal and the least reactive
configuration ns2 np1. The formula and nature of
non-metal of these are respectively
its oxide is
(a) XO3, basic (b) XO3 acidic (a) I and V (b) V and II
(c) X2O3, amphoteric (d) X2O3 basic (c) II and V (d) IV and V
28. The decreasing order of the ionization potential 34. Aqueous solutions of two compounds M1 – O – H
of the following elements is and M2 – O – H are prepared in two different
(a) Ne > Cl > P > S > Al > Mg beakers. If, the electronegativity of M1 = 3.4,
(b) Ne > Cl > P > S > Mg > Al M2 = 1.2, O = 3.5 and H= 2.1 then the nature of
(c) Ne > Cl > S > P > Mg > Al two solutions will be respectively :
(d) Ne > Cl > S > P > Al > Mg (a) acidic, basic (b) acidic, acidic
29. The incorrect statement among the following is (c) basic, acidic (d) basic, basic
(a) the first ionization potential of Al is less than
35. An element having electronic configuration
the first ionization potential of Mg
1s22s22p63s2 3p64s1 forms
(b) the second ionization potential of Mg is
greater than the second ionization potential (a) Acidic oxide (b) Basic oxide
of Na (c) Amphoteric oxide (d) Neutral oxide
(c) the first ionization potential of Na is less 36. The van der Waal and covalent radii of fluorine
than the first ionization potential of Mg atom respectively from the following figure are.
(d) the third ionization potential of Mg is greater
than the third ionization potential of Al.
30. The screening effect of inner electrons of the 294 pm
nucleus causes
(a) decrease in the ionization energy
(b) increase in the ionization energy
(c) no effect on the ionization energy F F F F
(d) increases the attraction of the nucleus for
the electrons
31. The order of increasing sizes of atomic radii 144 pm 144 pm
among the elements O, S, Se and As is :
(a) 219 pm, 72 pm (b) 75 pm, 72 pm
(a) As < S < O < Se (b) Se < S < As < O
(c) 147 pm, 72 pm (d) 147 pm, 144 pm
(c) O < S < As < Se (d) O < S < Se < As
EBD_7587
28 Chemistry Objective MCQs

37. Consider the following four elements, which are 44. The electronegativity follows the order
represented according to long form of periodic (a) F > O > Cl > Br (b) F > Cl > Br > O
table. (c) O > F > Cl > Br (d) Cl > F > O > Br
Here W, Y and Z are left, up and right elements 45. The first ionisation potential of aluminium is
with respect to the element ‘X’ and ‘X’ belongs smaller than that of magnesium because
to 16th group and 3rd period. Then according to (a) Atomic size of Al > Atomic size of Mg.
given information the incorrect statement (b) Atomic size of Al < Atomic size of Mg.
regarding given elements is: (c) Al has one electron in p - orbital
(d) None of these
(a) Maximum electronegativity : Y
(b) Maximum catenation property : X 46. Which one of the following statements is
(c) Maximum electron affinity : Z incorrect in relation to ionisation enthalpy?
(d) Y exhibits variable covalency (a) Ionisation enthalpy increases for each
38. The electron affinity of chlorine is 3.7 eV. 1 g of successive electron.
chlorine is completely converted to Cl– ion in a (b) The greatest increase in ionisation enthalpy
gaseous state. (1 eV = 23.06 kcal mol–1). is experienced on removal of electron from
Energy released in the process is core noble gas configuration.
(c) End of valence electrons is marked by a big
jump in ionisation enthalpy.
(d) Removal of electron from orbitals bearing
(a) 4.8 kcal (b) 7.2 kcal lower n value is easier than from orbital
(c) 8.2 kcal (d) 2.4 kcal having higher n value.
39. In which of the following arrangements, the 47. Highest electron affinity is shown by
sequence is not strictly according to the property
(a) O– (b) F– (c) Cl2 (d) F2
written against it?
(a) CO2 < SiO2 < SnO2 < PbO2: increasing 48. Which of the following order is wrong?
oxidising power (a) NH 3 < PH 3 < AsH 3 — Acidic
(b) NH3 < PH3 < AsH3 < SbH3: increasing basic (b) Li < Be < B < C — IE1
strength (c) Al 2 O3 < MgO < Na 2 O < K 2 O — Basic
(c) HF < HCl < HBr < HI: increasing acid (d) Li + < Na + < K + < Cs + — Ionic radius
strength 49. The electronic configuration of an element is .
(d) B < C < O < N: increasing first ionisation The atomic number and the group number of the
enthalpy. element ‘X’ which is just below the above element
40. Which among the following elements has the in the periodic table are respectively
highest first ionization enthalpy? (a) 23 and 5 (b) 23 and 15
(a) Nitrogen (b) Boron (c) 33 and 15 (d) 33 and 5
(c) Carbon (d) Oxygen 50. Consider the following information about element
41. Which of the following is not the correct order
P and Q:
for the stated property ?
(a) Ba > Sr > Mg; atomic radius Period number Group number
(b) F > O > N : first ionization enthalpy P 2 15
(c) Cl > F > I; electron affinity
Q 3 2
(d) O > Se > Te; electronegativity
(a) QP (b) Q2P3 (c) Q3P2 (d) Q2P
42. The increasing order of electron affinity of N, P
and As is 51. Which one of the following statements is
incorrect ?
(a) N < P < As (b) As < P < N (a) Greater the nuclear charge, greater is the
(c) As < N < P (d) As < N > P electron affinity
43. Which of the following ionisation energy values (b) Nitrogen has zero electron affinity
for calcium show a sudden increase?
(c) Electron affinity decreases from fluorine to
(a) Third (b) Second
(c) First (d) Fourth iodine in 17th group
(d) Chlorine has highest electron affinity
Classification of Elements and Periodicity in Properties 29

52. Which of the following statements is wrong ? 57. If the ionization enthalpy and electron gain enthalpy
(a) van der Waal’s radius of iodine is more than of an element are 275 and 86 kcal mol –1
its covalent radius respectively, then the electronegativity of the
(b) All isoelectronic ions belong to same period element on the Pauling scale is:
of the periodic table
(c) I.E.1 of N is higher than that of O while I.E.2 (a) 2.8 (b) 0.0 (c) 4.0 (d) 2.6
of O is higher than that of N
58. Which transition involves maximum amount of
(d) The electron gain enthalpy of N is almost
energy ?
zero while that of P is 74.3 kJ mol–1
53. Electron affinity is positive for (a) M - (g) ¾¾
® M(g) + e-
(a) O(g) + e - ® O- (g)
(b) M- (g) ¾¾
®M+ (g) + 2e-
(b) S(g) + e- ® S- (g)
(c) M+ (g) ¾¾
®M2+ (g) + e-
(c) O+ (g) + e - ® O(g)
(d) O - (g) + e - ® O 2 - (g)
(d) M2+ (g) ¾¾
® M3+ (g) + e-
59. Which ionisation potential (IP) in the following
54. Which one of the following ionic species has the
equations involves the greatest amount of
greatest proton affinity to form stable compound?
energy ?
(a) Cl – (b) F– (c) I– (d) Br –
55. Which property decreases from left to right (a) Na ® Na + + e - (b) K + ® K 2 + + e-
across the periodic table and increases from top
(c) C 2 + ® C3+ + e- (d) Ca + ® Ca 2 + + e -
to bottom?
(i) Atomic radius (ii) Electronegativity 60. Consider the following ionisation reactions :
(iii) Ionisation energy (iv) Metallic character I.E. (kJ mol–1) I.E. (kJ mol–1)
(a) (i) only (b) (i), (ii) and (iii) A (g) ® A (g) + e , A1 A(g) ® A+(g) + e–, A1
+ –

(c) (i), (iii) and (iv) (d) (i) and (iv) B+ (g) ® B2+(g) + e–, B2 C(g) ® C+(g) + e–, C1
56. The basic character of MgO, BaO, Na2O and FeO C+ (g) ® C2+(g) + e–, C1 C2+(g) ® C3+(g) + e–, C3
follow the order If monovalent positive ion of A, divalent positive
ion of B and trivalent positive ion of C have zero
(a) MgO < FeO < BaO < Na2O
electron. Then incorrect order of corresponding
(b) FeO < MgO < Na2O < BaO I.E. is :
(c) FeO < MgO < BaO < Na2O (a) C3 > B2 > A1 (b) B1 > A1 > C1
(d) Na2O < MgO < FeO < BaO (c) C3 > C2 > B2 (d) B2 > C3 > A1

Answer KEY
1 (c) 7 (c) 13 (a) 19 (a) 25 (a) 31 (d) 37 (d) 43 (a) 49 (c) 55 (d)
2 (a) 8 (c) 14 (a) 20 (a) 26 (b) 32 (b) 38 (d) 44 (a) 50 (c) 56 (c)
3 (b) 9 (b) 15 (c) 21 (a) 27 (c) 33 (c) 39 (b) 45 (c) 51 (c) 57 (a)
4 (b) 10 (c) 16 (b) 22 (c) 28 (b) 34 (a) 40 (a) 46 (d) 52 (b) 58 (d)
5 (b) 11 (a) 17 (d) 23 (c) 29 (b) 35 (b) 41 (b) 47 (a) 53 (d) 59 (b)
6 (a) 12 (d) 18 (b) 24 (a) 30 (a) 36 (c) 42 (c) 48 (b) 54 (b) 60 (d)
EBD_7587
30 Chemistry Objective MCQs

1. (c) Among the isoelectronic species, the cation 13. (a) Electronegativity values of given elements
having more positive charge would be the are as follows:
smaller in size. Be – 1.5 (I) Mg – 1.2 (IV)
2. (a) IE1 is always less than IE2. O – 3.5 (II) N – 3.0 (III)
3. (b) Fully filled electronic configuration. i.e. II > III > I > IV
14. (a) Cu = 3d10 4s1, Ag = 4d10 5s1, Au = 4¦14 5d10 6s1
4. (b) Electronegativity of an atom = IE + EA In all the above given cases, unpaired
2 ´ 62.5 s-electron has to be removed. In the case of
where IE and EA are in k cal mol . –1
Cu, a 4s electron is to be removed which is
5. (b) He (Z = 2) is a noble gas and has highest closer to the nucleus than the 5s electron of
I.E. The I.E. of Be (Z = 4) is more than that of Ag.
Li (Z = 3) So, IE1 of Cu > IE1 of Ag.
6. (a) If elements are arranged in order of their However, in case of Au, due to imperfect
increasing atomic numbers, element coming screening effect of 14 e– s of 4¦ orbitals, the
at intervals of 2, 8, 8, 18, 18, 32 and 32 will nuclear charge increases and therefore
have similar physical and chemical 5s e– of Au is more tightly held.
properties and thus grouped in one Thus, the order of IE1 is Cu > Ag < Au.
particular group. 15. (c) R is p-block element, because difference
7. (c) Tellurium (Te) has 5s25p4 valence shell between IE2 and IE3 is not very high as
configuration. It belongs to group 16 and compared to between IE1 and IE2; hence stable
present in period 5 of the periodic table. oxidation state of R will be higher than +2.
8. (c) Generally, the ionization enthalpies or 16. (b) Higher the positive charge the greater is the
energy increases from left to right in a period EN and higher the negative charge the lesser
and decreases from top to bottom in a group. is the EN. So, EN order is
Several factor such as atomic radius, nuclear M– > M2– > M3– > M4–
charge, shielding effect are responsible for 17. (d) Due to diagonal relationship.
change of ionization enthalpies. 18. (b) On passing from left to right in a period,
Here, Ist ionization enthalpy of A and B is acidic character of the normal oxides of the
greater than group I (Li = 520 kJmol–1 to Cs elements increases with in crease in
= 374 kJmol–1), which means element A and electronegativity.
B belong to group –2 and all three given 19. (a) Be – 1s22s2; B – 1s22s22p1; C – 1s22s22p2;
ionization enthalpy values are less for N – 1s22s22p3; O – 1s22s22p4. IP increases
element B means B will come below A. along the period. But IP of Be > B. Further
9. (b) Ionization energy is not the only the criteria IP of O < N because atoms with fully or
for the stability of an oxidation state. partly filled orbitals are most stable and
10. (c) Atomic no. 17 (Cl) and 53 (I) are present in hence have high ionisation energy.
the same group 20. (a) Cu has completely filled d-orbital, so highest
11. (a) Penetration effect order is s > p > d > f. energy is absorbed when it converts to Cu+
12. (d) Second ionization energy is amount of ion.
energy required to take out an electron from 21. (a) Acidic character of oxide Non-metallic
the monopositive cation. nature of element.
Classification of Elements and Periodicity in Properties 31

Non-metallic character increases along the 32. (b) The second ionization potential means
period. Hence order of acidic character is removal of electron from cation
Cl2O7 > SO2 > P4O10. C+ = 1s2 2s2 2p1, N+ = 1s2 2s2 2p2
22. (c) Its valency is 2. So it will form MO type
O+ = 1s2 2s2 2p3, F+ = 1s2 2s2 2p4
compound.
Therefore O > F > N > C
23. (c) On moving down a group atomic radii
increases. 33. (c) I represents Li, II represents K
III represents Br, IV represents I
24. (a) For isoelectronic species ionic radii
decreases as the charge on ion decreases. V represents He
So, amongst these, II represents most
Further on moving down in a group ionic
reactive metal and V represents least reactive
radii increases. Hence the correct order is
non-metal.
O2– < N3– < S2– < P3–
34. (a) The electronegativity difference between
25. (a) On moving along a period ionic radii
M1 and O is 0.1, which indicates M1– O
decreases due to increase in effective
bond will be covalent, since O–H bond
nuclear charge.
having more ionic character thus bond will
26. (b) Elements X, Y, Z with atomic numbers 19,
break and H+ ions gets release and acidic
37, 55 lie in group 1 (alkali metals). On
solution is formed and whereas difference
moving down a group from the size of atoms between electronegativity of M2O is 2.3,
increases, the outermost electrons become thus, M2–OH bond will break. Hence,
less strongly held. So the ionization energy
solution will be basic in nature.
decreases. Therefore, IE of Y could be
between those of X and Z. 35. (b) It is electronic configuration of alkali metal.
X Y Z Hence it will form basic oxide.
K (19) Rb (37) Cs (55) 36. (c) Covalent radius is radius of an atom in its
I.E. 4.3 4.2 3.9 bound state i.e., in fluorine it is half of
27. (c) ns2 np1 is the electronic configuration of III distance between two covalently bonded
A period. Al2O3 is amphoteric oxide fluorine atoms; van der Waal radii is one-
28. (b) Closed shell (Ne), half filled (P) and half of the distance between the nuclei of
completely filled configuration (Mg) are the two identical non-bonded isolated atoms.
cause of higher value of I.E. These atoms are attracted toward each
other through weak van der Waal’s force
29. (b) IE2 of Mg is lower than that of Na because
hence van der Waal radii are very large.
in case of Mg +, 3s-electron has to be
37. (d)
removed where as in case of Na+, an electron
W : Phosphorus Y : Oxygen X : Sulphur Z : Chlorine
is removed from the stable inert gas Electronegativity O > Cl > S > P Catenation : S > P > O > Cl
configuration which is difficult. Electron Affinity : Cl > O > S > P Oxygen exhibits covalency of two only

30. (a) Higher the screening effect, lower is the I.E. 1


38. (d) Number of moles =
31. (d) On moving down in a group atomic radii 35.5
increases due to successive addition of Given, 1 eV = 23.06 kcal mol–1
extra shell hence 3.7 eV = 3.7 × 23.06 kcal mol–1
O < S < Se i.e. 1 mole realease energy
Further As is in group 15 having one less = 3.7 × 23.06 kcal
electron in its p orbital hence have higher \ Energy released
atomic radii than group 16 elements. 1
= ´ 3.7 ´ 23.06 kcal = 2.4 kcal
i.e., O < S < Se < As 35.5
EBD_7587
32 Chemistry Objective MCQs

39. (b) Correct order of increasing basic strength 50. (c) P is a trivalent non-metal and Q is a divalent
is NH3 > PH3 > AsH3 > SbH3 > BiH3 metal.
40. (a) Due to stable 2s2 2p3 configuration (half Hence formula of compound is Q3P2.
filled p-orbital). Nitrogen atom has highest 51. (c) Electron affinity of 9F is less than that of
17Cl
energy.
41. (b) On moving along the period, ionization 52. (b) In the isoelectronic species, all isoelectronic
enthalpy increases. anions belong to the same period and
In second period, the order of ionization cations to the next period.
enthalpy should be as follows : F > O > N 53. (d) O– ion repel the incoming electron thus
But N has half-filled structure, therefore, it energy is required to add incoming electron.
is more stable than O. That’s why its 54. (b) Proton affinity decreases in moving across
ionization enthalpy is higher than O. Thus, the period from left to right due to increase
the correct order of IE is in charge, within a group the proton
F > N > O. affinities decreases from top to bottom.
42. (c) Phosphorus has vacant ‘d’-orbitals due to 55. (d) Atomic radius and metallic character
which it has higher electron affinity than decreases from left to right across the period
nitrogen. As also has vacant d-orbitals but and increases from top to bottom down the
its atomic size is very big thus its electron group.
affinity will be lesser than N. 56. (c) The basic character of metal oxides
43. (a) Ca = 3s2 3p6 4s2 decreases from left to right in a period and
Ca2+ = 3s2 3p6 4s0 increases down the group.
(Stable noble gas configuration) 57. (a) I.E. + E.A. = 275 + 86 = 361 kcal mol–1
Hence, there would be sudden increase in = 361 × 4.184 = 1510.42 kJ mol–1
IE3 value.
44. (a) F and O belong to 2nd period whereas Cl 1510.42
\ Electronegativity =
and Br belong to 3rd and 4th periods 540
respectively. Hence the sequence of the E.N. = 2.797 = 2.8
is F > O > Cl > Br 58. (d) The energy involved is ionisation energy
45. (c) Al (3s2 p1) and Mg (3s2). Lower energy is (I.E.). Further the 3rd ionisation energy will
required to remove 3p1 electron than 3s1 be greater than the 2nd and 1st.
electron (penetrating effect is s > p > d > f ).
59. (b) K + ® K 2 + e - . Since e– is to be removed
Secondly Mg has stable electronic
from stable configuration.
configuration than Al
46. (d) As the value of n increases, the nuclear 60. (d) A Þ H(1s1)
attraction over the outermost shell B Þ He(1s2)
decreases, therefore removal of an electron C Þ Li(1s2 2s1)
will be easier. A1 = IE1(A) B2 = IE2(B)
47. (a) Due to inert gas configuration molecules
B1 = IE1(B) C2 = IE2(C)
F2, Cl2 and anion F– have almost zero electron
affinity. C1 = IE1(C) C3 = IE3(C)
48. (b) The right sequence of I.E1 of Li < B < Be < C. B1 > A1 > C1 C3 > B2 > A1
49. (c) Atomic number of the given element = 15 ; He > H > Li Li2+ He+ H
atomic number of element ‘X’ will be 1s2 1s1 2s1 1s1 1s1 1s1
18 + 15 = 33; C3 > C2 > B2 Li2+ Li+ He+
group no. of ‘X’ = 10 + 5 (valence electrons)
1s2 1s2 1s1
= 15 , period = 4th
Chemical Bonding and
4 Molecular Structure
1. Which of the following contains both covalent 6. The compound MX4 is tetrahedral. The number
and ionic bond? of ÐXMX formed in the compound are
(a) NH4Cl (b) H2O
(c) CCl4 (d) CaCl2
2. Which of the following is not a correct
statement? (a) three (b) four (c) five (d) six
(a) Ionic compounds are electrically neutral. 7. The correct order of hybridization of the central
(b) Boiling point of an ionic compound is atom in the following species NH3, [PtCl4]2–,
more than a covalent compound. PCl5 and BCl3 is
(c) Melting point of a covalent compound is (a) dsp2, dsp3, sp2 and sp3
more than an ionic compound. (b) sp3, dsp2, sp3d, sp2
(d) Ionic compounds are soluble in polar (c) dsp2, sp2, sp3, dsp3
solvent. (d) dsp2, sp3, sp2, dsp3
3. Main axis of a diatomic molecule is Z. AO's px 8. The bond dissociation energy of B – F in BF3 is
and py overlap to form which of the following 646 kJ mol–1 whereas that of C – F in CF4 is 515
orbitals? kJ mol–1. The correct reason for higher B – F
(a) p-MO (b) s-MO bond dissociation energy as compared to that of
(c) d-MO (d) No bond will form C – F is
(a) stronger s bond between B and F in BF3 as
+
+ compared to that between C and F in CF4.
(b) significant pp – pp interaction between B
and F in BF3 whereas there is no possibility
of such interaction between C and F in CF4.
py (c) lower degree of pp – pp interaction between
px B and F in BF3 than that between C and F
in CF4.
4. The group having triangular planar structures (d) Smaller size of B-atom as compared to that
is : of C-atom.
9. Amongst LiCl, RbCl, BeCl 2 and MgCl 2 the
(a) BF3, NF3, CO32 - compounds with the greatest and the least ionic
-
(b) CO32 - , NO3 , SO3 character, respectively are:
(c) NH3, SO3, CO32 -
(a) LiCl and RbCl (b) RbCl and BeCl2
(d) NCl3, BCl3, SO3 (c) MgCl2 and BeCl2 (d) RbCl and MgCl2
10. Which of these statements is not true?
5. In piperidine , the hybrid state assumed (a) NO+ is not isoelectronic with O2
N (b) B is always covalent in its compounds
(c) In aqueous solution, the Tl+ ion is much
H more stable than Tl (III)
by N is (d) LiAlH4 is a versatile reducing agent in
(a) sp (b) sp2 (c) sp3 (d) dsp2 organic synthesis.
EBD_7587
34 Chemistry Objective MCQs

11. Which of the following does not apply to 16. Which compound exhibits maximum dipole
metallic bond ? moment among the following ?
(a) Overlapping valence orbitals
(b) Mobile valency electrons
(c) Delocalized electrons
(d) Highly directed bonds. NO2 NO2
12. In PO 34– , the formal charge on each oxygen NH 2
atom and the P - O bond order respectively are (a) (b)

(a) –0.75, 0.6 (b) –0.75, 1.0


(c) –0.75, 1.25 (d) –3, 1.25 NO2 NO2
13. Which one of the following formulae does not
correctly represent the bonding capacities of the
two atoms involved? (c) (d)
+
é H ù NH 2
ê | ú
êH — P — H ú NH 2
(a) |
ê H ú
ë û 17. Molecular AB has a bond length of 1.61Å and
a dipole moment of 0.38 D. The fractional
(b) F F charge on each atom (absolute magnitude) is:
O (e0 = 4.802 × 10–10 esu)
O
(c) O ¬N (a) 0.5 (b) 0.05 (c) 0 (d) 1.0
O–H
18. In a regular octahedral molecule, MX 6 the
H number of X - M - X bonds at 180° is
| O (a) three (b) two (c) six (d) four
(d) H – C = C 19. Arrange the following ions in the order of
O–H decreasing X – O bond length, where X is the
14. The dipole moments of diatomic molecules AB central atom
and CD are 10.41D and 10.27 D, respectively
while their bond distances are 2.82 and 2.67 Å, (a) ClO -4 ,SO 24 - , PO34- ,SiO -4
respectively. This indicates that (b) SiO44 - , PO34- ,SO24 - , ClO-4

(c) SiO 44 - , PO34- ,ClO4- ,SO42 -


(a) bonding is 100% ionic in both the
molecules (d) SiO44 - ,SO42- , PO34- , ClO-4
(b) AB has more ionic bond character than CD 20. Which of the following statements is correct in
(c) AB has lesser ionic bond character than the context of the allene molecule, C3H4?
CD (a) The central carbon is sp hybridized
(d) bonding is nearly covalent in both the (b) The terminal carbon atoms are sp 2
molecules hybridized
15. Among the following, the species having the (c) The planes containing the CH2 groups are
smallest bond is mutually perpendicular to permit the
(a) NO– (b) NO+ (c) O2 (d) NO formations two separate p-bonds
(d) All are correct
Chemical Bonding and Molecular Structure 35

21. Which of the following pairs have identical bond 29. Although CN – ion and N 2 molecule are
order ? isoelectronic, yet N2 molecule is chemically inert
(a) N 2 , O 22+ (b) N 2 , O2– because of
(a) presence of more number of electrons in
(c) N 2– , O 2 (d) O 2+ , N 2 bonding orbitals
22. Which of the following molecular orbitals has (b) lone bond energy
two nodal planes ? (c) absence of bond polarity
(a) s2s (b) p2py (c) p*2py (d) s*2px (d) uneven electron distribution.
23. Which one of the following molecules is polar ? 30. The number and type of bonds in ion in CaC2
(a) XeF4 (b) IF5 (c) SbF5 (d) CF4
are:
24. Bond distance in HF is 9.17 × 10–11 m. Dipole
moment of HF is 6.104 × 10 –30 Cm. The (a) One s bond and one p-bond
percentage ionic character in HF will be : (b) One s bond and two p-bond
(electron charge = 1.60 × 10–19 C) (c) Two s bond and two p-bond
(a) 61.0% (b) 38.0% (d) Two s bond and one p-bond
(c) 35.5% (d) 41.5% 31. Among the following compounds the one that
25. Dipole moment of H2O is 1.85 D. If the bond is polar and has the central atom with sp 2
angle is 105° and O — H bond length is hybridisation is
0.94 Å, what is the magnitude of charge on the
(a) H2CO3 (b) SiF4
oxygen atom in water molecule?
(c) BF3 (d) HClO2
32. Among the following ions the pp–dp overlap
(a) 2 × 10–10 esu (b) 4.28 × 10–10 esu could be present in
(c) 3.22 × 10 esu (d) 1.602 × 10–19 C
–10
(a) NO -2 (b) NO 3-
26. Which of the correct increasing order of lone
pair of electrons on the central atom? (c) PO34- (d) CO 23-
(a) IF7 < IF5 < ClF3 < XeF2
(b) IF7 < XeF2 < ClF2 < IF5 33. In the reaction 2PCl5 ƒ PCl4+ + PCl 6- , the
(c) IF7 < ClF3 < XeF2 < IF5 change in hybridisation is from
(d) IF7 < XeF2 < IF5 < ClF3 (a) sp3d to sp3 and sp3d2
27. The molecules BF3 and NF3 are both covalent
(b) sp3d to sp2 and sp3
compounds, but BF3 is non polar whereas NF3
is polar. The reason for this is (c) sp3d to sp3d2 and sp3d3
(a) atomic size of boron is larger than nitrogen (d) sp3d2 to sp3 and sp3d
(b) Boron is metal while nitrogen is gas 34. In an octahedral structure, the pair of d orbitals
(c) B – F bonds are non-polar while N – F involved in d 2sp3 hybridization is
bonds are polar d d
(a) x 2 - y2 , z2
(b) d xz, d
(d) BF3 is planar but NF3 is pyramidal x2 -y2
28. Among the following chloro-compound having (c) d d (d) d xy, d yz
z 2 , xz
the lowest dipole moment is
(a) CH3Cl 35. N2 and O2 are converted into monocations, N +2
and O +2 respectively. Which of the following
Cl H
(b) C=C statements is wrong?
H3C Cl (a) In N +2 , N - N bond weakens
(c) CH2Cl2 (b) In O +2 , the O - O bond order increases
Cl (c) In O +2 , paramagnetism decreases
Cl
(d) C=C (d) N +2 becomes diamagnetic
H3C H
EBD_7587
36 Chemistry Objective MCQs

36. The correct order of bond dissociation energy 44. Which of the following statement is correct?
among N2, O2, O-2 is shown in which of the (a) FeCl2 is more covalent than FeCl3.
following arrangements? (b) FeCl3 is more covalent than FeCl2.
(c) Both FeCl2 and FeCl3 are equally covalent.
(a) N 2 > O-2 > O2 (b) O -2 > O 2 > N 2 (d) FeCl2 and FeCl3 do not have any covalent
character.
(c) N 2 > O2 > O2- (d) O 2 > O-2 > N 2
45. The maximum number of 90º angles between
37. In XeF2, XeF4 and XeF6, the number of lone bond pair-bond pair of electrons is observed in
pairs on Xe are respectively
(a) 2, 3, 1 (b) 1, 2, 3 (c) 4, 1, 2 (d) 3, 2, 1
38. Which of the following represents the given
mode of hybridisation sp2 –sp2 – sp - sp from (a) dsp2 hybridization
left to right ? (b) sp3d hybridization
(a) H 2 C = CH - C º N (c) dsp3 hybridization
(d) sp3d2 hybridization
(b) HC º C – C º CH
46. The true statements from the following are
(c) H 2 C = C = C = CH 2 1. PH5 and BiCl5 do not exist
CH2 2. pp - dp bond is present in SO2
(d) H2C 3. Electrons travel with the speed of light
4. SeF4 and CH4 have same shape
39. In BrF 3 molecule, the lone pairs occupy
equatorial positions to minimize 5. I3+ has bent geometry
(a) lone pair - bond pair repulsion only
(b) bond pair - bond pair repulsion only (a) 1, 3 (b) 1, 2, 5
(c) 1, 3, 5 (d) 1, 2, 4
(c) lone pair - lone pair repulsion and lone pair
- bond pair repulsion 47. KF combines with HF to form KHF2. The
compound contains the species
(d) lone pair - lone pair repulsion only
40. The group of molecules having identical shape (a) K+, F– and H+ (b) K+, F– and HF
is : (c) K+ and [HF2]– (b) [KHF]+and F2
(a) PCl5, IF5, XeO2F2 48. Which of the following statements is incorrect?
(b) BF3, PCl3, XeO3 (a) NH3 is more basic than PH3.
(c) SF4, XeF4, CCl4 (b) NH3 has a higher boiling point than that
(d) ClF3, XeOF2, XeF3+ of HF.
41. SF2, SF4 and SF6 have the hybridisation at (c) N2 is less reactive than P4.
sulphur atom respectively as : (d) The dipole moment of NH3 is less than that
(a) sp2, sp3, sp2d 2 (b) sp3, sp3, sp3d 2 of SO2.
3 3 3
(c) sp , sp d, sp d 2 (d) sp3, spd 2, d 2sp3 49. In which of the following sets, all the given
42. Which of the following molecules has two sigma species are isostructural ?
(s) and two pi (p) bonds? (a) CO2, NO2, ClO2, SiO2
(a) C2H4 (b) N2F2 (b) PCl3, Al Cl3, BCl3, SbCl3
(c) C2H2Cl2 (d) HCN (c) BF3, NF3, PF3, Al F3
_
43. The shape of IF6– is : (d) BF4 , CCl 4 , NH 4+ , PCl 4+
(a) Trigonally distorted octahedron
50. The formation of molecular complex BF3 – NH3
(b) Pyramidal
results in a change in hybridization of boron
(c) Octahedral
(a) from sp2 to dsp2 (b) from sp2 to sp3
(d) Square antiprism
(c) from sp3 to sp2 (d) from sp3 to sp3d
Chemical Bonding and Molecular Structure 37

51. Which of the following substances has the least 57. Which one of the following pairs of molecules
ionic character ? will have permanent dipole moments for both
(a) FeCl2 (b) ZnCl2 members ?
(c) CdCl2 (d) MgCl2
(a) NO2 and CO2 (b) NO2 and O3
52. Sulphur reacts with chlorine in 1 : 2 ratio and
forms X. Hydrolysis of X gives a sulphur (c) SiF4 and CO2 (d) SiF4 and NO2
compound Y. What is the hybridisation state of 58. In which of the following ionization processes
central atom in the compound. the bond energy has increased and also the
magnetic behaviour has changed from
(a) sp3 (b) sp2 (c) sp (d) dsp2 paramagnetic to diamagnetic ?
53. CaO and NaCl have the same crystal structure
and approximately the same ionic radii. If U is N2 ® N2 +
(a) NO ® NO + (b)
the lattice energy of NaCl, the approximate
lattice energy of CaO is (c) C2 ® C2 + (d) O2 ® O2 +
59. Bond order normally gives idea of stability of a
(a) U/2 (b) U (c) 4 C (d) 2 U molecular species. All the molecules viz. H2, Li2
54. Dipole moment is shown by and B2 have the same bond order yet they are
(a) 1, 4-dichlorobenzene not equally stable. Their stability order is
(b) cis -1, 2-dichlorobenzene
(a) H2 > Li2 > B2 (b) Li2 > H2 > B2
(c) trans -1, 3-dichlorobenzene
(c) Li2 > B2 > H2 (d) B2 > H2 > Li2
(d) trans -2, 3-dichloro -2- butene
60. The internuclear distances in O – O bonds for
55. Which of the following has the square planar
structure? O+2 , O2 , O2- and O 22 - respectively are :
(a) 1.30 Å, 1.49 Å, 1.12 Å, 1.21 Å
(a) XeF4 (b) NH +4 (c) BF4- (d) CCl4
(b) 1.49 Å, 1.21 Å, 1.12 Å, 1.30 Å
56. Which one of the following molecules is
paramagnetic? (c) 1.21 Å, 1.12 Å, 1.49 Å, 1.30 Å
(a) N2 (b) NO (c) CO (d) O3 (d) 1.12 Å, 1.21 Å, 1.30 Å, 1.49 Å

Answer KEY
1 (a) 7 (b) 13 (d) 19 (b) 25 (c) 31 (a) 37 (d) 43 (a) 49 (d) 55 (a)
2 (c) 8 (b) 14 (c) 20 (d) 26 (a) 32 (c) 38 (a) 44 (b) 50 (b) 56 (b)
3 (d) 9 (b) 15 (b) 21 (a) 27 (d) 33 (a) 39 (c) 45 (d) 51 (b) 57 (b)
4 (b) 10 (a) 16 (c) 22 (c) 28 (c) 34 (a) 40 (d) 46 (b) 52 (a) 58 (a)
5 (c) 11 (d) 17 (b) 23 (b) 29 (c) 35 (d) 41 (c) 47 (c) 53 (c) 59 (a)
6 (d) 12 (c) 18 (a) 24 (d) 30 (b) 36 (c) 42 (d) 48 (b) 54 (b) 60 (d)
EBD_7587
38 Chemistry Objective MCQs

1. (a) NH4Cl contains both covalent and ionic 7. (b) Hybridisation in NH3 = sp3,
[PtCl4]2– = dsp2 (inner complex);
+
é H ù PCl5 = sp3d and BCl3 is sp2.
ê | ú
– Hú Cl – 8. (b) The delocalised pp - pp bonding between
bonds êH – N | filled p-orbital of F and vacant p-orbital of
ê H ú
ë û B leads to shortening of B–F bond length
2. (c) Due to electrostatic force of attraction, the which results in higher bond dissociation
melting points and boiling points of ionic energy of the B–F bond.
compounds are high.
3. (d) px and py orbitals do not have proper
orientation to overlap and hence no bond F
is formed. B F
F
+
+
Vacant Filled
2p-orbital 2p-orbital

py F F
+
px B=F B–F
+
4. (b) GroupHybridization Shape F F
(1) BF3 sp2 Triangular Planar (TP) + +1/3
NF3 sp3 Tetrahedral (T) F F
+1/3
B–F B F
CO32- sp2 T.P. +1/3
F F
(2) CO32- sp2 T.P.
ü
9. (b) According to Fajan’s rule smaller, highly
ïï æ All have same ö charged cation has greatest covalent
NO3- sp2 T.P. ý ç hybridization ÷ character while large cation with smaller
ïè ø
ïþ charge has greatest ionic character.
SO3 sp2 T.P. 10. (a) NO+ = 7 + 8 – 1 = 14 e–.
(3) NH3 sp3 T O2 = 16 e–
SO3 sp2 T.P. i.e not iso-electronic
CO32- sp2 T.P. (b) Boron forms only covalent compounds.
(4) NCl3 sp3 T This is due to its extremely high ionisation
BCl3 sp2 T.P. energy.
SO3 sp2 T.P. (c) Compounds of Tl+ are much more stable
5. (c) Hybridisation of N = ½ [5 + 3 + 0 – 0] = 4 than those of Tl3+.
hence sp3 (d) LiAlH4 is a versatile reducing agent in
organic synthesis
X 11. (d) In metallic bonds, each ion is surrounded
6. (d) M three angle below M and three by equal number of oppositely charged
X electrons, hence have equal electrostatic
X X
attraction from all sides and hence do not
above M hence = 6 have directional characteristics.
Chemical Bonding and Molecular Structure 39

Dipole moment = (Distance between


3- 3-
é O ù é O ù opposite charges) × (charge, q)
ê || ú ê | ú µ=q×d
12. (c) êO - P - Oú « êO = P - O ú
So, greater the distance between the
| |
ê O ú ê O ú
ë û ë û opposite charges higher the dipole. Due to
Bond order the resonance the greater charge separation
occurs between charges due to linearity.
Number of bonds
= – – –
Number of Resonating structures O O O O
N N
5 Å Å
= = 1.25
4
Three unit negative charge is being shared
by four O atoms. Formal charge = –3/4
= – 0.75 NH
.. 2 NH2
Å
H O 17. (b) 1 e = 1.602 × 10–19C
| P
13. (d) H , C < C*, O , H 1esu = 3.33 × 10–10C

The star marked carbon has a valency of 5 1e 1.602 ´ 10 -19 C


=
and hence this formula is not correct. 1esu 3.33 ´ 10-10 C
14. (c) As dipole moment = electric charge × bond 1 e = 4.802 × 10–10 esu
length Dipole moment = q × distance
D. M. of AB molecule Þ 1 D » 10–18 esu cm
= 4.8 ´ 10 -10 ´ 2.82 ´ 10 -8 = 13.53D 0.38 × 10–18 esu cm = q × (1.61 × 10–8cm)
D.M. of CD molecule q = 2.36 × 10–11esu

< 4.8´10,10 ´ 2.67 ´10,8 < 12.81D 2.36 ´ 10 -11 esu


q=
Now % ionic character 4.802 ´ 10 -10 esu
Actual dipole moment of the bond q =0.049
= ´100
Dipole moment of pure ionic compound q » 0.05 fractional charge
then % ionic character in AB X5
18. (a)
10.41
= ´ 100 = 76 .94 %
13.53 X4 X1
% ionic character in CD
10.27 M
= ´ 100 = 80.17%
12.81
15. (b) NO–(16) – B.O. – 2
O2(16) – B.O. – 2 X3 X2
NO+(14) – B.O. – 3
NO(15) – B.O. – 2.5 X6
Higher the bond order lower is the bond
length. Hence NO+ will have smallest Thus here bond angles between
bond. X 4 - M - X 2 = 180°
16. (c) NO2 X1 - M - X 3 = 180°
X 5 - M - X 6 = 180°
19. (b) More will be the electronegativity of X,
lesser will be the bond length of X-O bond.
NH2
EBD_7587
40 Chemistry Objective MCQs

20. (d) H
p 0.94Å
p
52.5°
H H O
d
C C C
H H d
cos 52.5° = Þ d = 0.609 ´ 0.94Å
0.94Å
= 0.572Å
21. (a) Bond order in N2 and O 22+ is 3 (calculate
by energy level diagram) \ μ = q´d
μ 1.85D
q1 = =
d 0.572Å
22. (c) It has two nodal
1.85 ´ 10 -18 esu cm
=
0.572 ´ 10 -8 cm
planes. It is p*2py
= 3.2 × 10–10 esu
23. (b) The geometry of IF5 is square Pyramidal 26. (a) The number of lone pairs of electrons on
with an unsymmetric charge distribution central atom in various given species are
therefore this molecule is polar. Species Number of lone
pairs on central
F atom
F F
IF7 nil
I
IF5 1
F F ClF3 2
XeF2 3
24. (d) Given e = 1.60 × 10–19 C
Thus the correct increasing order is
d = 9.17 × 10–11 m
IF7 < IF5 < ClF3 < XeF2
From m = e × d 0 1 2 3
m = 1.60 × 10–19 × 9.17 × 10–11
27. (d) The shape of BF3 is trigonal planar
= 14.672 × 10–30
d-
% ionic character F d + d-
B – F and m = 0 hence it is non polar..
Observed dipole moment d- F
= ×100
Calculated Dipole moment The shape of NF 3 is pyramidal
+
:

6.104 ´ 10-30 Nd
= ´ 100 d- F F d- and m ¹ 0 hence it is polar..
14.672 ´ 10 -30 F d-
= 41.5% Cl H
25. (c) m = 1.85 D = 1.85 × 10–18 esu cm = q × d 28. (c) C=C
H Cl
+q
H Dipole moment (m) = 0
2 29. (c) In nitrogen molecule, both the nitrogen
105° – atoms have same electronegativity. So it
O has zero polarity and hence less tendency
q1
to break away and form ions.
H
+q 30. (b) The structure of CaC2 is Ca2+ [: C º C :]2 –
2
i.e, one s and two p bonds
Chemical Bonding and Molecular Structure 41

31. (a) H2CO3 and BF3 both have triangular planar 39. (c) F
F H–O :
structure B – F C=O
Br F
F H–O
BF3 is symmetrical
:
\ m = 0 hence non polar, H2CO3 is not F
symmetrical hence polar in nature. In BrF3, both bond pairs as well as lone
32. (c) In P–O bond, p bond is formed by the pairs of electrons are present. Due to the
sidewise overlapping of d-orbital of P and presence of lone pairs of electrons (lp) in
p-orbital of oxygen. Hence it is formed by the valence shell, the bond angle is
pp and dp overlapping. contracted and the molecule takes the T-
O shape. This is due to greater repulsion
s p between two lone pairs or between a lone
P pair and a bond pair than between the two
bond pairs.
O – O–
40. (d) ClF3 ¾¾ ® Hybridisation
O–
1
33. (a) 2PCl5 ƒ PCl4+ + PCl6- = 3 + [7 - 3] = 5(sp3 d )
2
­ sp 3 d ­ sp 3 ­ sp 3 d 2 XeOF2 ¾¾ ® Hybridisation
34. (a) Only those d orbitals whose lobes are 1
= 3 + [8 - 4] = 5(sp3 d )
directed along X, Y and Z directions 2
hybridise with s and p orbitals. In other XeF3+ ¾¾
® Hybridisation
three, d orbitals namely dxy, dyz and dxz,
1
the lobes are at an angle of 45° from both = 3 + [8 - 3 - 1] = 5( sp 3 d )
axis, hence the extent of their overlap with 2
All molecules have sp3d hybridization and
s and p orbitals is much lesser than d 2 2 2 lone pairs. Hence all have identical
x -y
and d 2 orbitals. (T-shape).
z
35. (d) s2b s *a2 s 2b s*2 2 2 1 + 41. (c) Hybridisation :
a ( p b = p b ) s b (N 2 = 13electrons)
gg
it contains one unpaired electron hence
paramagnetic
1. : SF2 Þ 1 (6 + 2 ) = 4 = sp3
2
36. (c) The bond order of N2, O2, and O 2- are 1
respectively 3, 2 and 1.5 2. : SF4 Þ (6 + 4 ) = 5 = sp3d
2
Since higher bond order implies higher 1
bond dissociation energy hence the correct 3. SF6 Þ ( 6 + 6 ) = 6 = sp 3 d 2
order will be N 2 > O 2 > O 2- 2
p, s
37. (d) In XeF2 Total number of valence electrons 42. (d) H s C p N
of Xe = 8, two electrons shared with 2F i.e 2 p and 2 s bonds.
atoms, 6 electrons left hence 3 lone pairs, 43. (a) The structure of IF6– is distorted octahedral
in XeF4 4 shared with 4 F atoms 4 left hence This is due to presence of a lone pair.
2 lone pairs; in XeF6 6 shared with 6 F
atoms 2 left hence 1 lone pair. F
F F
38. (a) CH 2 = C H - C º N
1 2 3 4
3 s bonds (sp2
hybridisation); I
2 s bonds (sp-hybridisation)
C1 = 3 s bonds, C2 = 3 s bonds, F F
C3 = 2 s bonds
F
EBD_7587
42 Chemistry Objective MCQs

44. (b) According to Fajan's rule, higher charge 55. (a) XeF4 has square planar structure, while
on the ions, more covalent is the compound. NH +4 , BF4– and CCl4 have tetrahedral
structure.
1 2 56. (b) The molecular orbital configuration of the
5 10 6
molecules given is
9 M 11
Total no. of electrons in NO = 7(N) + 8(O)
45. (d) 7 12 8
3 4 = 15
Hence E.C. of NO
sp3d2 hybridisation = KK s(2s)2 s *(2s)2 s(2 pz )2
Number of 90° angle between bonds = 12 p(2 p x )2 = p(2 p y )2 p *(2 p x )1 = p *(2 p y )o
46. (b) SeF4 has distorted tetrahedral geometry
while CH4 has tetrahedral geometry. Due to presence of one unpaired electron
47. (c) Since F form H-bond [HF 2 ] – exists. NO is paramagnetic.
Therefore KHF2 gives K+ + HF2– Except NO all are diamagnetic due to
48. (b) Due to high EN of F than N, H-bonding is absence of unpaired electrons.
stronger in HF than NH3 thus b.pt. of HF > NH3. 57. (b) Both NO2 and O3 have angular shape and
49. (d) All have tetrahedral structure. hence will have net dipole moment.
50. (b) In BF3, B is sp2 hybridized with one 58. (a) For NO
empty pz orbital. The empty pz orbital of Total no. of electrons = 15
BF3 can be filled by lone pair of molecules B.O = 2.5
such as NH 3 . When this occurs a Mag. Behaviour = Paramagnetic
tetrahedral molecule or ion is formed For NO+
which is sp3 hybridized. Total no. of electrons = 14
51. (b) Polarizing power of Zn 2+ is more than B.O = 3
others, hence ZnCl2 is least ionic in nature.
Mag. Behaviour = Diamagnetic
52. (a) S + 2Cl2 ® SCl4 H O 59. (a) The molecular orbital configuration of the
2

OH given molecules is
HO OH H2 = s(1s)2 (no electron anti-bonding)
HO — S = O S Li2 = s(1s)2 s*(1s)2 s(2s)2
(H2SO3) HO OH (two anti-bonding electrons)
Hybridisation of B2 = s(1s)2 s*(1s)2 s(2s)2 s*(2s)2
1 p(2p x )1 = p(2p y )1
H 2SO3 = (6 + 2 + 0 ) = 4 = sp3
2 (4 anti-bonding electrons)
Product of charges Though the bond order of all the species
53. (c) Lattice energy = are same (B.O = 1) but stability is different.
interionic distance
This is due to difference in the presence of
In NaCl the product of charges = 1 × 1; In
no. of anti-bonding electron.
CaO product of charges = 2 × 2 = 4 while
Higher the no. of anti-bonding electron
the inter ionic distance is almost same in
both. Thus lattice energy of CaO is almost lower is the stability hence the correct order
four times the lattice energy of NaCl. is H2 > Li2 > B2
54. (b) Cis1, 2 - dichloro benzene will have some 60. (d) The bond length follows the order
Cl O+2 < O2 < O2– < O 2–
2
Cl
D.M. According to this the possible values are
1.12Å, 1.21Å, 1.30Å, 1.49Å
5 States of Matter
1. Densities of two gases are in the ratio 1:2 and 3. Which gas shows real behaviour?
their temperatures are in the ratio 2:1 then the (a) 16 g O2 at 1 atm and 273 K occupies
ratio of their respective pressures is 11.2 L
(a) 1:1 (b) 1:2 (c) 2:1 (d) 4:1 (b) 1 g H2 in 0.5 L flask exerts pressure of 24.63
atm at 300 K
2. Which of the following volume (V) - temperature
(c) 1 mole NH3 at 300 K and 1 atm occupies
(T) plots represents the behaviour of one mole
volume 22.4 L
of an ideal gas at one atmospheric pressure ?
(d) 5.6 L of CO2 at 1 atm and 273 K is equal to
11 g
V(L) 4. For a real gas (mol. mass = 60) if density at critical
(36.8 L 4 ´105
373 K) point is 0.80 g/cm3 and its Tc = K, then
(22.4 L 821
van der waals’ constant a (in atm L2 mol–2) is
(a) 273K)
(a) 0.3375 (b) 3.375 (c) 1.68 (d) 0.025
5. 600 c.c. of a gas at a pressure of 750 mm of Hg is
compressed to 500 c.c. Taking the temperature to
remain constant, the increase in pressure, is
V(L) T(K) (a) 150 mm of Hg (b) 250 mm of Hg
(26.8 L (c) 350 mm of Hg (d) 450 mm of Hg
373 K) 6. The volume of 0.0168 mol of O2 obtained by
(22.4 L decomposition of KClO 3 and collected by
273K)
(b) displacement of water is 428 mL at a pressure of
754 mm Hg at 25 °C. The pressure of water vapour
at 25 °C is
(a) 18 mm Hg (b) 20 mm Hg
V(L) T(K) (c) 22 mm Hg (d) 24 mm Hg.
7. Three different gases X, Y and Z of molecular
(30.6 L masses 2, 16 and 64 were enclosed in a vessel at
(22.4 L 373 K) constant temperature till equilibrium is reached.
(c) 273K) Which of the following statement is correct?
(a) Gas Z will be at the top of the vessel
(b) Gas Y will be at the top of the vessel
(c) Gas Z will be at the bottom and X will be at
the top
V(L) T(K) (d) Gases will form homogenous mixture
8. When 2 g of a gas A is introduced into an
evaluated flask kept at 25 ºC, the pressure is
(22.4 L found to be one atmosphere. If 3 gm of another
(d) 273K) gas B is then added to the same flask, the total
(14.2 L pressure becomes 1.5 atm. Assuming ideal gas
373 K) behaviour, calculate the ratio of the molecular
weights MA : MB.
T(K)
(a) 1 : 3 (b) 1 : 1 (c) 2 : 1 (d) 3 : 1
EBD_7587
44 Chemistry Objective MCQs
9. When does a gas deviate the most from its seconds. The volume of O2 in dm3 which diffuses
ideal behaviour? under the similar condition in 30 seconds will be
(a) At low pressure and low temperature (atomic mass of sulphur = 32 u):
(b) At low pressure and high temperature (a) 7.09 (b) 14.1 (c) 10.0 (d) 28.2
(c) At high pressure and low temperature 16. For 1 mol of an ideal gas at a constant temperature
(d) At high pressure and high temperature T, the plot of log P against log V is a (P : Pressure,
10. The initial volume of a gas cylinder is 750.0 mL. If V : Volume)
the pressure of gas inside the cylinder changes
(a) Straight line parallel to x-axis.
from 840.0 mm Hg to 360.0 mm Hg, the final volume
the gas will be: (b) Straight line with a negative slope.
(c) Curve starting at origin.
(a) 1.750 L (b) 3.60 L
(d) Straight line passing through origin.
(c) 4.032 L (d) 7.50 L
17. The value of van der waals constant ‘a’ for gases
11. Which of the following assumption of kinetic
O2, N2 , NH3 and CH4 are 1.360, 1.390, 4.170 and
molecular theory states that gases do not have
2.253 litre 2 atm mol–2 respectively. The gas which
fixed shape ?
can most easily be liquefied is :
(a) Particles of a gas move in all possible
(a) O2 (b) N2 (c) NH3 (d) CH4
directions in straight line.
18. A manifestation of surface tension is :
(b) Particles of a gas are always in constant and
(a) rise of liquid in a capillary tube
random motion.
(b) spherical shape of liquid drops
(c) Total energy of molecules before and after
(c) upward movement of water in soils
the collision remains same.
(d) All the above
(d) None of these
19. A bubble of the gas released at the bottom of a
12. Which one of the following is the wrong
lake increases to eight times the original volume
assumption of kinetic theory of gases ?
when it reaches at the surface. Assuming that
(a) Momentum and energy always remain
the atmospheric pressure is equivalent to
conserved.
pressure exerted by a column of water 10 m high,
(b) Pressure is the result of elastic collision of
molecules with the container’s wall. what is the depth of the lake
(c) Molecules are separated by great distances
compared to their sizes. (a) 80 m (b) 90 m (c) 10 m (d) 70 m
(d) All the molecules move in straight line 20. Two vessels containing gases A and B are
between collision and with same velocity. interconected as shown in the figure. The stopper
13. Equal masses of methane and oxygen are mixed is opened, the gases are allowed to mix
in an empty container at 25°C. The fraction of the homogeneously. The partial pressures of A and
total pressure exerted by oxygen is B in the mixture will be, respectively

(a) 1/2 (b) 2/3 Gas A Gas B


1 273
(c) ´ (d) 1/3 12 L 8L
3 298 8 atm 5 atm
14. A He atom at 300 K is released from the surface
of the earth to travel upwards. Assuming that it
(a) 8 and 5 atm (b) 9.6 and 4 atm
undergoes no collision with other molecules,
(c) 4.8 and 2 atm (d) 6.4 and 4 atm
how high will it be before coming to the rest?
21. A neon-dioxygen mixture contains 70.6 g O2 and
(a) 9.53 m (b) 95.3 m
167.5 g neon. If pressure of the mixture of gases
(c) 953 m (d) 9.53 × 104 m
in the cylinder is 25 bar. What is the partial
15. Sulphur dioxide and oxygen were allowed to
pressure of O2 and Ne in the mixture respectively?
diffuse through a porous partition. 20 dm3 of SO2
(a) 5.25 bar, 10 bar (b) 19.75 bar, 5.25 bar
diffuses through the porous partition in 60
(c) 19.75 bar, 10 bar (d) 5.25 bar, 19.75 bar
States of Matter 45
22. When CO2(g) is passed over red hot coke it 28. The temperature at which oxygen molecules have
partially gets reduced to CO(g). Upon passing the same root mean square speed as helium atoms
0.5 L of CO2(g) over red hot coke, the total volume have at 300 K is:
of the gases increased to 700 mL. Atomic masses: He = 4 u, O = 16 u)
The composition of the gaseous mixture at STP (a) 300 K (b) 600 K
is (c) 1200 K (d) 2400 K
29. By how many folds the temperature of a gas
(a) CO2 = 300 mL; CO = 400 mL would increase when the root mean square
velocity of the gas molecules in a container of
(b) CO2 = 0.0 mL; CO = 700 mL
fixed volume is increased from 5 × 104 cm/s to
(c) CO2 = 200 mL; CO = 500 mL 10 × 104 cm/s ?
(d) CO2 = 350 mL; CO = 350 mL (a) Two (b) Three (c) Six (d) Four
23. Which of the following is not an assumption of 30. A high- altitude balloon is filled with 1.41 × 104 L
the kinetic theory of gases? of hydrogen at a temperature of 21°C and a
(a) Gas particles have negligible volume. pressure of 745 torr. What is the volume of the
(b) A gas consists of many identical particles balloon at a height of 20 km, where the
which are in continual motion. temperature is – 48 °C and the pressure is 63.1
torr?
(c) At high pressure, gas particles are difficult
(a) 1.247 × 105 L (b) 1.66 × 105 L
to compress. (c) 1.66 × 10 L4 (d) None of these
(d) Collisions of gas particles are perfectly 31. A spherical balloon of 21 cm diameter is to be
elastic. filled with H2 at NTP from a cylinder containing
24. Two vessels of volumes 16.4 L and 5 L contain the gas at 20 atm at 27°C. The cylinder can hold
two ideal gases of molecular existence at the 2.82 L of water at NTP. The number of balloons
respective temperature of 27 °C and 227 °C and that can be filled up is
exert 1.5 and 4.1 atmospheres respectively. The (a) 15 (b) 10 (c) 20 (d) 25
ratio of the number of molecules of the former to 32. Equal volumes of the gases which do not react
that of the later is together are confined in separate vessels. The
pressure is 200 mm and 400 mm of Hg
1 1 respectively. If the two gases are mixed together
(a) 2 (b) 1 (c) (d)
2 3 what will be the pressure of the resulting mixture
25. If 10–4 dm3 of water is introduced into a 1.0 dm3 (temperature remaining constant)
flask at 300 K, how many moles of water are in the
vapour phase when equilibrium is established ? (a) 400 mm (b) 400 mm
(Given : Vapour pressure of H2O at 300 K is (c) 300 mm (d) 200 mm
3170 Pa; R = 8.314 J K–1 mol–1) 33. In case of CO and CH4 curve goes to minima
(a) 5.56× 10–3 mol (b) 1.53 × 10–2 mol then increases with increase in pressure but in
–2
(c) 4.46 × 10 mol (d) 1.27 × 10–3 mol case of H2 and He the curve is linear because:
26. At STP, a container has 1 mole of He, 2 mole Ne, CO CH4
3 mole O2 and 4 mole N2. without changing total
H2
pressure if 2 mole of O2 is removed. The partial
pressure of O2 will be decreased by: He
(a) 26% (b) 40%
(c) 58.33% (d) 66.66% PV Ideal gas
27. A mixture of Ne and Ar kept in a closed vessel at
250 K has a total K.E. = 3kJ. The total mass of Ne
and Ar is 30 g. Find mass % of Ne in gaseous
mixture at 250 K.
(a) 61.63 (b) 38.37 (c) 50 (d) 28.3 P
EBD_7587
46 Chemistry Objective MCQs
(a) Intermolecular interactions for H2 and He (1) The gas behaves non-ideally
are very low. (2) The gas dimerises
(b) Molecular size or atomic size for H2 and (3) The gas is adsorbed into the vessel walls
He is small.
(a) 1, 2 and 3 (b) 1 and 2 only
(c) Both (a) and (b)
(d) Neither (a) nor (b) (c) 2 and 3 only (d) 1 only
34. Following table represents critical temperature 39. What is the ratio of pressure of the 2 g of
of some gases. Arrange these gases in their hydrogen to that of 4 g of helium at temperature
increasing order of liquifaction. of 298 K, 20 mL volume? (consider the ideal
behaviour)
Gas H2 He N2 O2
(a) 1 : 2 (b) 2 : 1 (c) 1 : 1 (d) 2 : 2
Tc / K 33.2 5.3 126 154.3
40. A drop of oil is placed on the surface of water.
(a) He < N2 < H2 < O2 (b) H2 < He < N2 < O2 Which of the following statement is correct ?
(c) He < H2 < N2 < O2 (d) O2 < N2 < H2 < He (a) It will remain on it as a sphere
35. a, v and u represent most probable velocity, (b) It will spread as a thin layer
average velocity and root mean square velocity (c) It will be partly as spherical droplets and
respectively of a gas at a particular temperature. partly as thin film
The correct order among the following is
(d) It will float as a distorted drop on the water
(a) u > v > a (b) v > u > a
surface
(a) a > u > v (d) u > a > v 41. The correct order of viscosity of the following
36. van der Waal’s equation for a gas is stated as, liquids will be
2 (a) Water < methyl alcohol < dimethyl ether <
nRT ænö
P= -aç ÷ . glycerol
V - nb èVø
(b) methyl alcohol < glycerol < water < dimethyl
This equation reduces to the perfect gas equation,
nRT ether
P= when, (c) dimethyl ether < methyl alcohol < water <
V
glycerol
(a) temperature is sufficient high and pressure (d) glycerol < dimethyl ether < water < methyl
is low. alcohol
(b) temperature is sufficient low and pressure 42. At 0°C and one atm pressure, a gas occupies
is high. 100 cc. If the pressure is increased to one and a
(c) both temperature and pressure are very half-time and temperature is increased by one-
high. third of absolute temperature, then final volume
(d) both temperature and pressure are very low. of the gas will be:
37. What is the value of X in °C for given volume vs (a) 80 cc (b) 88.9 cc
temperature curve ? (c) 66.7 cc (d) 100 cc
P1 P < P < P < P
P2 1 2 3 4 43. At 27 °C a sample of ammonia gas exerts a
P3 pressure of 5.3 atm. What is the pressure when
Volume P4 the volume of the gas is reduced to one-tenth of
the original value at the same temperature?
X Temperature (°C) (a) 0.53 atm (b) 5.3 atm
(a) 0° C (b) 273.15° C (c) 53 atm (d) None of these
(c) – 273.15° C (d) 300° C
44. The most probable speed of 8 g of H2 is
38. When a sample of gas is compressed at constant
temperature from 15 atm to 60 atm, its volume 200 ms–1. Average kinetic energy (neglect
changes from 76 cm3 to 20.5 cm3. Which of the rotational and vibrational energy) of H2 gas is :
following statements are possible explanations (a) 480 J (b) 240 J
of this behaviour? (c) 120 J (d) none of these
States of Matter 47
45. Oxidation of succinate ion produces ethylene and 51. Reducing the pressure from 1.0 atm to 0.5 atm
carbon dioxide gases. On passing 0.2 Faraday would change the number of molecules in one
electricity through an aqueous solution of mole of ammonia to
potassium succinate, the total volume of gases (a) 25% of its initial value
(at both cathode and anode) at STP (1 atm and (b) 50% of its initial value
273 K) is : (c) 75% of its initial value
(a) 8.96 L (b) 4.48 L (d) None of the above
(c) 6.72 L (d) 2.24 L 52. For real gases van der Waals equation is written
46. A compressed cylinder of gas contains as
1.50 × 103 g of N2 gas at a pressure of 2.0 ×107 Pa
and a temperature of 17.1 °C. What volume of gas
has been released into the atmosphere if the final æ an 2 ö
pressure in the cylinder is 1.80 ×105 Pa? Assume ç p + 2 ÷ (V– nb) = nRT
ideal behaviour and that the gas temperature is è V ø
unchanged. where ‘a’ and ‘b’ are van der Waals constants.
(a) 1264 L (b) 126 L Two sets of gases are :
(c) 12600 L (d) 45 L (I) O2, CO2, H2 and He
47. Dimethyl ether decomposes as (II) CH4, O2 and H2
The gases given in set-I in increasing order of ‘b’
CH 3OCH 3 (g) ¾¾
® CH 4 (g) + CO(g) + H 2(g) and gases given in set-II in decreasing order of
when CH3OCH3 decomposes to 20% extent ‘a’, are arranged below. Select the correct order
under certain conditions, what is the ratio of from the following :
diffusion of pure CH3OCH3 with methane? (a) (I) He < H2 < CO2 < O2 (II) CH4 > H2 > O2
(a) 0.59 : 1 (b) 1.18 : 1 (b) (I) O2 < He < H2 < CO2 (II) H2 > O2 > CH4
(c) 2.36 : 1 (d) 1.77 : 1 (c) (I) He < H2 < O2 < CO2 (II) CH4 > O2 > H2
48. Air entering the lungs ends up in tiny sacs called (d) (I) H2 < O2 < He < CO2 (II) O2 > CH4 > H2
alveoli. From the alveoli, the oxygen diffuses into 53. Which of the following statements are correct?
the blood. The average radius of the alveoli is (i) Hydrogen bonding is a special case of
0.0050 cm and the air inside contains 14 per cent dipole - dipole interaction.
oxygen. Assuming that the pressure in the alveoli (ii) Energy of hydrogen bond varies between
is 1.0 atm and the temperature is 37°C, calculate 10 to 100 kJ mol–1.
the number of oxygen molecules in one of the (iii) Hydrogen bonds are powerful force in
alveoli. determining the structure and properties of
compounds like proteins, nucleic acids etc.
(a) 6 × 1013 (b) 1024 (iv) Strength of the hydrogen bond is
(c) 1.7 × 10 22 (d) 1.7 × 1012 determined by the coulombic interaction
49. A graph is plotted between PV m (along Y-axis) between the lone-pair electrons of the
and P (along X–axis) at 27° ( Vm being the molar electronegative atom of one molecule and
the hydrogen atom of other molecule.
volume of an ideal gas). The intercept of the graph
(a) (i) and (ii) (b) (i), (ii) and (iii)
at Y–axis is (R = 0.082 atm LK–1 mol–1)
(c) (ii), (iii) and (iv) (d) All of these
(a) 22.4 atm L mol–1
54. On a ship sailing in pacific ocean where
(b) 24.6 atm L mol–1
temperature is 23.4°C, a balloon is filled with 2 L
(c) 24.6 atm LK–1 mol–1
air. What will be the volume of the balloon when
(d) None of these
the ship reaches Indian ocean, where temperature
50. A flask containing air (open to the atmosphere)
is heated from 300 K to 500 K. The percentage of is 26.1°C ?
the air escaped into the atmosphere is (a) 2.018 L (b) 2.8 L
(a) 16.6 (b) 40 (c) 60 (d) 20 (c) 3.5 L (d) 1.5 L
EBD_7587
48 Chemistry Objective MCQs
55. A bubble of air is underwater at temperature 15°C
and the pressure 1.5 bar. If the bubble rises to the
surface where the temperature is 25°C and the R=10cm
pressure is 1.0 bar, what will happen to the volume
of the bubble ? Sphere
(a) Volume will become greater by a factor of (hollow)
1.6. (III)
(b) Volume will become greater by a factor of If all the containers are placed at the same
1.1. temperatures, then find the incorrect options –
(c) Volume will become smaller by a factor of (a) Pressure of the gas is minimum in (III)
0.70. container
(d) Volume will become greater by a factor of (b) Pressure of the gas is equal in I and II
2.5. container
56. Cyclopropane and oxygen at partial pressures
(c) Pressure of the gas is maximum in (I)
170 torr and 570 torr respectively are mixed in a
(d) The ratio of pressure in II and III container
gas cylinder. What is the ratio of the number of
is 4 : 3
moles of cyclopropane to the number of moles of
58. Helium atom is two times heavier than a hydrogen
oxygen (nC3H6/nO2)? molecule at. 298K. The average KE of helium is
(a) 2 times of H2 molecule
170 ´ 42 (b) same as that of H2 molecule
(a) = 0.39
570 ´ 32 (c) 4 times that of hydrogen molecule
1
170 æ 170 570 ö (d) that of H2 molecule
(b) ç + ÷ » 0.19 2
42 è 42 32 ø 59. A 3 : 2 molar mixture of N2 and CO is present in a
vessel at 500 bar pressure. Due to hole in the
170 vessel, the gas mixture leaks out. Th e
(c) = 0.23
740 composition of mixture effusing out initially is
170
(d) = 0.30
570 n N : n CO ::1: 2 (b) n N : n CO :: 6 :1
(a) 2 2
57. There are three closed containers in which equal
amount of the gas are filled. (c) n CO : n N ::1: 2 (d) n CO : n N2 :: 2 : 3
2
60. The molecular velocities of two gases at the same
temperature are u1 and u2 and their masses are
m1 and m2 respectively. Which of the following
expressions are correct?
10cm
m1 m2
(a) = (b) m1u1 = m 2 u 2
u12 u 22
10cm
(cube) 10 cm m1 m 2
(I) (II) (c) = (d) m1u12 = m 2 u 22
u1 u2
States of Matter 49

Answer KEY
1 (a) 7 (d) 13 (d) 19 (d) 25 (d) 31 (b) 37 (c) 43 (c) 49 (b) 55 (a)
2 (c) 8 (a) 14 (d) 20 (c) 26 (c) 32 (c) 38 (d) 44 (b) 50 (b) 56 (d)
3 (c) 9 (c) 15 (b) 21 (d) 27 (d) 33 (c) 39 (c) 45 (a) 51 (d) 57 (b)
4 (b) 10 (a) 16 (b) 22 (a) 28 (d) 34 (c) 40 (b) 46 (a) 52 (c) 58 (b)
5 (a) 11 (b) 17 (c) 23 (c) 29 (d) 35 (a) 41 (c) 47 (c) 53 (d) 59 (d)
6 (d) 12 (d) 18 (d) 24 (a) 30 (a) 36 (a) 42 (b) 48 (d) 54 (a) 60 (d)

1. (a) P µ d and T,, 750 ´ 600


or P2 = = 900 mm of Hg
500
P1 d T 1 2 Therefore increase of pressure
= 1 1 = ´ Þ P1 : P2 = 1 : 1
P2 d 2 T2 2 1 = (900 – 750) = 150 mm of Hg
6. (d) Volume of 0.0168 moles at STP
V1 V2
2. (c) = at const. pressure = 0.0168 × 22400 = 376.3mL.
T1 T2
P1V1 P2 V2
22.4 V2 <
Þ = , V2 = 30.6 litre T1 T2
273 373
3. (c) For ideal gas PV = nRT 760 ´376.3 P2 ´428
or <
Volume of 1 mole gas at 1 atm pressure and 273 298
273 K is 22.4 L.
or P2 < 730mm
60
4. (b) Vc = = 75cm3 mol–1; Pressure of water = 754 – 730 = 24 mm Hg
0.80
7. (d) All the gases occupy the available volume
V and will form homogeneous mixture.
b = c = 25cm3 mol–1; = 0.25 L mol–1 8. (a) From ideal gas equation, PV = nRT
3

8a æmö RT
\ Tc = PV = ç ÷ RT or M = m
27Rb èMø PV
Let the molecular wt. of A and B be MA and
4 ´ 105 2.45
= = 5.76 gL-1 MB respectively.
821 0.425
RT 3 ´ RT
Þ a = 3.375 Then MA = 2 ´ 1 ´ V ; M B = 0.5 ´ V
5. (a) Given initial volume (V1) = 600 c.c.; Initial
M 2RT 0.5V 2 ´ 0.5 1
pressure (P1) = 750 mm of Hg and final A
\ M = V ´ 3RT = =
3 3
volume (V2) = 500 c.c. according to Boyle’s B

law, Therefore, the ratio MA : MB = 1 : 3


P1V1 = P2V2 9. (c) At high pressure and low temperature,
gaseous atoms or molecules get closer to
or 750 × 600 = P2 × 500 each other and van der Waal forces operates.
EBD_7587
50 Chemistry Objective MCQs
So molecules or atoms start attracting each
other. Hence a gas deviate the most from its r1 V1 / t1 d2
or = =
ideal behaviour. While in ideal behaviour r2 V2 / t2 d1
we consider that gaseous molecules do not
attract each other, i.e., there is no 20 / 60 16 / 2 1
intermolecular forces of attraction. = = =
V2 / 30 32 / 2 2
10. (a) According to Boyle’s law
Q Mol. wt = 2 × V.D
V1 P2 750 360
= ; = Mol.wt
V2 P1 V2 840 \ V.D =
2
V2 = 1750 mL = 1.750 L
11. (b) Particles of a gas are always in constant On calculating,
and random motion. If the particles were at V2 = 14.1
rest and occupy fixed positions, then a gas 16. (b) According to Boyle's law, PV = constant
would have a fixed shape which is not \ log P + log V = constant
observed. log P = – log V + constant
12. (d) Molecules move very fast in all directions Hence, the plot of log P vs log V is
in a straight line by colliding with each other straight line with negative slope.
but with different velocity.
13. (d) Let the mass of methane and oxygen = m
gm.
Mole fraction of O2 log P
Moles of O2
= log V
Moles of O 2 + Moles of CH 4
17. (c) The higher the value of ‘a’, more the value
m / 32 m / 32 1 of Tc , easy is the liquefaction
= = = 18. (d) All the given phenomenon occurs due to
m / 32 + m /16 3m / 32 3
Partial pressure of O2 = surface tension
Total pressure × mole fraction of O2, 19. (d) Let V be the original volume of bubble. The
final volume will be 8 V. Let p be the
1 1
PO2 = P × = P atmospheric pressure and p1 the pressure
3 3
at the bottom
3
14. (d) k BT = mgh \ p × 8 V = p1 × V;;
2
8p = p1
3 1.38 ´ 10-23 ´ 300
h= ´ = 9.53 ´ 10 4 m p1 = atmospheric pressure + pressure due
2 9.81´ 4 ´ 1.66 ´ 10-27
to water lake
15. (b) According to Graham’s Law Diffusion:
= p + 7p
r1 d2 The p = 10 m high, the 7p will be = 70 m high
= So the depth = 70 m
r2 d1
Since rate of diffusion PA VA 8 ´12 96
20. (c) Moles of A, (n A) = = =
RT RT RT
Vol. of gas diffused (V )
= PB VB 8 ´ 5 40
Time taken for diffusion(t ) Moles of B, (n B) = = =
RT RT RT
r1 V1 / t1 Total pressure × total volume = (nA + nB) ×
\ =
r2 V2 / t2 RT
States of Matter 51

1 \ Now, n(CO final) + n(CO final) = 0.031


p ´ (12 + 8) = (96 + 40)RT
RT n(CO final) = 0.031 – n(CO final) ...(iii)
2
P = 6.8 Substituting (ii) in eq. (i)
Partial pressure ofA = P × mole fraction of A n(CO final) = 0.044 – 2[0.031 – n(CO final)]
æ 96 96 + 40 ö n(CO final) = 0.044 – 0.062 + 2n(CO final)
= 6.8 ç ÷
è RT RT ø n(CO final) = 0.018 mol.
= 4.8 atm Volume of
Partial pressure of B = 6.8 – 4.8 = 2 atm.
nRT 0.018 ´ 0.0821 ´ 273
CO = V = =
P 1
21. (d) Number of moles of O2
= 0.40 Litre
70.6 g and volume of CO2 = 0.7 litre – 0.4 litre
= = 2.21 mole
32 g mol -1 = 0.3 litre
Number of moles of Ne \ CO2 = 300 mL, CO = 400 mL
23. (c) At high pressure real gas particles are easily
167.5 g compressed.
= = 8.375 mole
20g mol -1 24. (a) Given conditions
V1 = 16.4 L, V2 = 5 L
Mole fraction of O2
P1 = 1.5 atm, P2 = 4.1 atm
2.21 T1 = 273 + 27 = 300 K,
= = 0.21 mole
2.21 + 8.375 T2 = 273 + 227 = 500 K
Mole fraction of Ne = 1 – 0.21 = 0.79
P1V1 nT
Partial pressure of a gas Applying gas equation, = 1 1
P2 V2 n 2 T2
= Mole fraction × total pressure
Partial pressure of O2 = 0.21 × 25 = 5.25 bar
n1 PVT
Partial pressure of Ne = 0.79 × 25 = 1 1 1
n 2 P2 V2 T2
= 19.75 bar
22. (a) CO 2 + C ¾¾ ® 2CO 1.5 ´16.4 ´ 500 2
\ =
Stoichoimetry ratio is 1 : 2 4.1 ´ 5 ´ 300 1
AT STP, P = 1 atm, T = 273 K, R = 0.0821 25. (d) From the ideal gas equation :
PV PV = nRT
Initial moles of CO2; n(CO2initial) =
RT
PV 3170×10-3
1 ´ 0.5 or n = = = 1.27 × 10–3
= = 0.022 mole RT 8.314×300
0.0821 ´ 273
In final mixture no. of moles; n(CO2/CO 3
mixture) 26. (c) PO = ´ PT ;
2 10
1 ´ 0.7 After removing 2 mole of O2,
= = 0.031
0.0821 ´ 273
1
Increase in volume is by = 0.031 – 0.022 P 'O = ´ PT
10
2
= 0.009 mole of gas
Final no. of moles of CO i.e. n(CO final) Decrease in partial pressure of O2
n(CO final) = 2n(CO initial) – n(CO final)
2 2 3PT PT
= 2(0.022 – n(CO ...(i) -
2 final) = 10 8 ´ 100 = 58.33 %
n(CO final) = 0.044 – 2n(CO ...(ii) 3PT
2 final)
10
EBD_7587
52 Chemistry Objective MCQs

3 3000 ´ 2 After filling the cylinder will have H 2 equal


27. (d) K= n T RT Þ n T =
2 3 ´ 250 ´ 8.314 to its volume = 2820 mL
3 \ Volume of H 2 for filling balloons
3 ´ 103 = n T ´ 8.314 ´ 250
2 = 51324 – 2820 = 48504 mL
Hence no. of balloon to be filled
x 30 - x 2
+ =
20 40 8.314 48504
= » 10
% Ne = 28.3 4851
3RT 32. (c) When vessels are joined the volume is
28. (d) Vrms = doubled and pressure is reduced to half \
M
Pressure of mixture
Vrms(O ) = Vrms (He)
2
200 400
3RTO2 3RTHe = + = 300 mm
= 2 2
M O2 M He
33. (c) Due to small size of these species (H2 and
TO 2 THe He) intermolecular interactions (van der
or = Waal forces) are very low, therefore it is
M O2 M He
difficult to compress these.
300 ´ 32 34. (c) More will be critical temperature easier is
\ TO 2 = = 2400K the liquifaction of the gas. Hence correct
4
order will be
3RT He < H2 < N2 < O2
29. (d) r.m.s. velocity Vrms =
M
3RT 8RT 2 RT
35. (a) u :v :a = : :
V1 T1 M pM M
i.e., V = T2
2
8
= 3: : 2
5 ´ 104 1 T1 p
= =
4 2 T2
10 ´ 10
2
\ T2 = 4T1 36. (a) Given P = nRT - a æ n ö
ç ÷
v - nb èvø
P1V1 P2 V2
30. (a) = Which can also be written as
T1 T2
é n2 a ù
745 ´ 1.41 ´ 10 4 ´ 225 ê P + 2 ú (V - nb ) = nRT
V2 = = 1.274 ´ 105 L
294 ´ 63.1 êë V úû
4 3 At low pressure and high temperature the
31. (b) The volume of the balloon = pr
3 a
3 effect of 2 and b is negligible hence
4 22 æ 21 ö V
= ´ ´ ç ÷ = 4851 mL PV = nRT.
3 7 è2ø
37. (c) At any given pressure, graph of volume vs
Volume of the cylinder = 2820 mL temperature (in °C) is a straight line and on
Volume of H 2 at NTP extending to zero volume each line
intercepts the temperature axis at
20 ´ 2820 ´ 273
= mL = 51324 mL – 273.15° C.
300 ´ 1
States of Matter 53
38. (d) Given, P1 = 15 atm, P2 = 60 atm
0.2 0.2 0.2
V1 = 76 cm3, V2 = 20.5 cm3. = ∗ ∗ < 0.4
2 1 2
If the gas is an ideal gas, then according to
Boyle's law, it must follow the equation, nRT 0.4´0.0821´273
V< < < 8.96 L.
P1V1 = P2V2 P 1
P1 × V1 = 15 × 76 = 1140 46. (a) n N 2 = 53.57 mole
P2 × V2 = 60 × 20.5 = 1230
P1 n1
\P1V1 ¹ P2 V2 =
P2 n 2
\ The gas behaves non-ideally.
The given information is not sufficient to 1.8
n2 = = 0.009 ´ 53.57 = 0.48213
comment on other statements. 200
39. (c) Number of moles, temperature and volume
n N 2 escape = 53.087
are same.
40. (b) nRT
41. (c) The correct order of viscosity of the given Volume = = 1264 L
P
liquids is dimethyl ether < methyl alcohol <
water < glycerol. rCH 3 OCH 3 MCH 4 p CH 3OCH3
47. (c) = ´
P1V1 P2 V2 rCH 4 MCH 3 OCH 3 pCH 4
42. (b) =
T1 T2 16 0.8
= ´ = 2.36 :1
P ´ 100 1.5P ´ V2 46 0.2
=
T 4T 4 3
48. (d) Volume of alveoli = pR
3 3
V2 = 88.9 mL = 88.9 cc
4
43. (c) P1V1 = P2V2 = ´ 3.14 ´ (5 ´10-3 )3 mL
3
V1 = V, V2 = V/10
V = 5.23 ´ 10-7 mL = 5.23 ´ 10-7 L
5.3 atm 100 14 PV
= ; P2 = 5.3 ´ 100 = 53 atm n2 = ´ n air = 0.14 ´
P2 V 100 RT

5.23 ´10-10
2RT = 0.14 ´1´
44. (b) 200 = ; RT = 40 0.0821´ 310
2 ´ 10-3
= 2.87 ´ 10-12
3 3 8 Molecules = 2.87 × 6.02 ×1023 × 10–12
Average K.E. = nRT = ´ ´ 40 = 240 J
2 2 2 = 1.7 × 1012
CH 2COO, 49. (b) The graph is a straight line parallel to the
45. (a) | + 2H2O ¾® axis of pressure with intercept
CH 2COO,
CH 2 –
= PVm = RT = 0.082
|| + 2CO2 + H2 + 2OH atm L K -1mol-1 ´ 300K
CH2
anode cathode = 24.6 atm L mol -1 .

Total equivalent of 50. (b) P,V and R are constant


C2H4 + CO2 + H2 = 0.2 + 0.2 + 0.2 = 0.6 \ n l T1 = n 2T2 ,100 ´ 300 = n 2 ´ 500, : n 2 = 60
Total moles of gases Air escaped is 40%.
EBD_7587
54 Chemistry Objective MCQs
51. (d) One mole of a substance contains the 56. (d) By ideal gas equation,
number of molecules which is independent
1 = n1RT
PV
of pressure.
n1 µ P1 and n2 µ P2
52. (c) Molar mass ­ , ‘a’ increases
n1 P1 n 170
size of molecule ­ ,‘b’ increase = Þ 1 = = 0.30
n2 P2 n2 570
b(L/mol) a (bar. L2/mol2)
1
H2 ¾® 0.02661 CH4 ¾® 2.25 57. (b) n, T same hence P µ ,
V
He ¾® 0.0237 O2 ¾® 1.36
V1 = 1000 cm3
O2 ¾® 0.03183 H2 ¾® 0.244 V2 = p (10)2 × 10 = 1000 p cm3
CO2 ¾® 0.04267 4 4
V3 = p (10)3 = p 1000 cm3
53. (d) All of the given statements are correct for 3 3
hydrogen bond. \ Pressure of the gas is minimum in (III)
54. (a) V1 = 2 L, T2 = (26.1 + 273) K = 299.1 K, container, Pressure of the gas is maximum
V2 = ? in (I),
T1 = (23.4 + 273) K = 296.4 K
3 n
From Charle's law, 58. (b) KE = RT . Average KE is the same
2 No
V1 V2 VT
= Þ V2 = 1 2 for all gaseous molecules
T1 T2 T1
59. (d) Molar ratio of N2 and CO is 3 : 2, i.e., 300 bar
2L ´ 299.1K and 200 bar, respectively.
Þ V2 = = 2L ´1.009
296.4K n N2 m CO PN 2
= 2.018 L = ´
n CO mN 2 PCO
55. (a) Given
P1 = 1.5 bar, T1 = 273 + 15 = 288 K, V1= V 300 3
P2 = 1.0 bar ,T1 = 273 + 25 = 298K, V2 = ? = =
200 2
P1V1 P2 V2
= 1 1
T1 T2 60. (d) U1 µ and U 2 µ
m1 m2
1.5 ´ V 1 ´ V2
=
288 298
V2 = 1.55 V i.e., volume of bubble will be U12 m2
\ = \ m1 U12 = m 2 U 22
almost 1.6 time to initial volume of bubble. U 22 m1
6 Thermodynamics
1. Among the following the state function(s) is 7. One mole of an ideal gas at 300 K is expanded
(are) isothermally from an initial volume of 1 litre to
(i) Internal energy 10 litres. The DE for this process is (R = 2 cal
(ii) Irreversible expansion work mol–1 K–1)
(iii) Reversible expansion work (a) 163.7 cal (b) zero
(iv) Molar enthalpy (c) 1381.1 cal (d) 9 lit atm
(a) (ii) and (iii) (b) (i), (ii) and (iii) 8. The latent heat of vapourization of a liquid at
(c) (i) and (iv) (d) (i) only 500 K and 1 atm pressure is 10.0 kcal/mol. What
2. If a reaction involves only solids and liquids will be the change in internal energy (DU) of 3
which of the following is true ? moles of liquid at the same temperature
(a) DH < DE (b) DH = DE (a) 13.0 kcal/mol (b) –13.0 kcal/mol
(c) DH > DE (d) DH = DE + DnRT (c) 27.0 kcal (d) –7.0 kcal/mol
3. One mole of a non-ideal gas undergoes a change 9. Equal volumes of methanoic acid and sodium
of state (2.0 atm, 3.0 L, 95 K) ® (4.0 atm, hydroxide are mixed. If x is the heat of formation
5.0 L, 245 K) with a change in internal energy, of water, then heat evolved on neutralization is
DU = 30.0 L atm. The change in enthalpy DH (a) more than x (b) equal to x
of the process in L atm is. (c) less than x (d) twice x.
10. A cyclic process ABCD is shown in P–V
(a) 40.0 diagram for an ideal gas. Which of the following
(b) 42.3 diagram represents the same process?
(c) 44.0
(d) Not defined because pressure is not
constant
4. If bond enthalpies of N º N, H –H and N –H B
A
bonds are x1, x2 and x3 respectively, DH of for
NH3 will be P
(a) x1 + 3x2 – 6x3 (b) 1/2x1 + 3/2x2 – 3x3 C
(c) 3x3 – 1/2x1 – 3/2x2(d) 6x3 – x1 – 3x2 D
5. Heat of neutralization of a strong acid HA
V
and a weaker acid HB with KOH are – 13.7
A D
and – 12.7 k cal mol –1. When 1 mole of KOH B C

was added to a mixture containing 1 mole each C A


V V
of HA and HB, the heat change was – 13.5 kcal.
In what ratio is the base distributed between HA (a) (b)
D B
and HB. T T
D C A B
(a) 3 : 1 (b) 1 : 3 (c) 4 : 1 (d) 1 : 4
6. A gas expands adiabatically at constant pressure V
B
V
such that T µ V-1/2. The value of g (Cp,m/Cv,m) D
(c) (d)
of the gas will be : A
C
(a) 1.30 (b) 1.50 (c) 1.70 (d) 2 T T
EBD_7587
56 Chemistry Objective MCQs
11. What is the value of change in internal energy (a) qAB = 450 R and qCA = – 450 R
at 1 atm in the process? (b) qAB = 450 R and qCA = – 225 R
(c) qAB = 450 R and qCA = – 375 R
H2O (l,323 K ) ¾¾
® H2O ( g, 423 K ) (d) qAB = 375 R and qCA = – 450 R
Given : Cv,m (H2O, l) = 75.0 JK–1 mol–1; 16. The standard enthalpy of formation of NH3 is
– 46.0 kJ/mol. If the enthalpy of formation of
Cp,m (H2O, g) = 33.314 JK–1 mol–1
H2 from its atoms is – 436 kJ/mol and that of N2
DHvap at 373 K = 40.7 kJ/mol is – 712 kJ/mol, the average bond enthalpy of N
(a) 42.91 kJ/mol (b) 43086 kJ/mol - H bond in NH3 is:
(c) 42.6 kJ/mol (d) 49.6 kJ/mol (a) – 1102 kJ/mol (b) – 964 kJ/mol
12. Which of the following statements/relationships (c) + 352 kJ/mol (d) + 1056 kJ/mol
is not correct in thermodynamic changes? 17. The standard enthalpy of formation (Df H° 298)
(a) DU = 0 (isothermal reversible expansion for methane, CH4 is – 74.9 kJ mol–1. In order to
of a gas) calculate the average energy given out in the
V2 formation of a C – H bond from this it is
(b) w = – nRT ln (isothermal reversible necessary to know which one of the following?
V1
(a) The dissociation energy of the hydrogen
expansion of an ideal gas)
molecule, H2.
V2 (b) The first four ionisation energies of carbon.
(c) w = nRT ln (isothermal reversible
V1 (c) The dissociation energy of H2 and enthalpy
of sublimation of carbon (graphite).
expansion of an ideal gas)
(d) The first four ionisation energies of carbon
(d) For a system of constant volume heat
involved directly changes to internal energy. and electron affinity of hydrogen.
13. The heat of atomization of methane and ethane 18. Assuming that water vapour is an ideal gas, the
are 360 kJ/mol and 620 kJ/mol, respectively. internal energy change (DU) when 1 mol of
The longest wavelength of light capable of water is vapourised at 1 bar pressure and 100°C,
breaking the C – C bond is (Avogadro number (given : molar enthalpy of vapourisation of wa-
= 6.02 × 1023 mol–1, h = 6.62 × 10–34 J s) : ter at 1 bar and 373 K = 41 kJ mol –1 and
R = 8.3 J mol–1 K–1) will be
(a) 41.00 kJ mol–1 (b) 4.100 kJ mol–1
(a) 2.48 × 104 nm (b) 1.49 × 103 nm
(c) 2.48 × 10 nm 3 (d) 1.49 × 104 nm (c) 3.7904 kJ mol–1 (d) 37.904 kJ mol–1
14. Following reaction occurrs in an automobile 19. An ideal gas expands against a constant
2C8 H18 ( g ) + 25O 2 (g ) ® 16CO 2 (g ) + 18H 2O (g ) .
external pressure of 2.0 atmosphere from 20
litre to 40 litre and absorbs 10 kJ of heat from
The sign of DH, DS and DG would be surrounding. What is the change in internal
energy of the system?
(given : 1atm-litre = 101.3 J)
(a) +,–,+ (b) –,+,– (c) –,+,+ (d) +,+,– (a) 4052 J (b) 5948 J
15. A heat engine carries one mole of an ideal (c) 14052 J (d) 9940 J
mono-atomic gas around the cycle as shown in 20. For which one of the processes represented by
the figure. Select the correctoption:
the following equations the enthalpy (heat)
T = 600 K change is likely to be negative
B 2
(a) Cl - (g) + aq ® Cl - (aq)
(b) Cl(g) ® Cl + (g) + e -
Pressure

A C (c) 1 / 2Cl 2 (g ) ® Cl (g )
T1 = 300 K T3 = 450 K
(d) Cl 2 (l) ® Cl 2 (g)
Volume
Thermodynamics 57
21. The ‘thermite reaction’ involving the reaction if equilibrium pressure is 3X bar then DrG°
between ferric oxide and metallic aluminium would be
produces molten iron. Given that
2Al + 3 / 2 O 2 ® Al 2 O 3 ; DH1 = -400 kcal/ mol (a) – RT ln 9 – 3RT ln X
(b) RT ln 4 – 3RT ln X
2Fe + 3 / 2 O 2 ® Fe 2O 3 ; DH 2 = -200 kcal/mol.
(c) – 3RT ln X
What is DH for the formation of 1 mole of iron? (d) None of these
(a) –100 kcal (b) –200 kcal 26. Stan dard Gibb’s free energy change for
(c) +100 kcal (d) +200 kcal isomerization reaction
22. Given : cis-2-pentene trans-2-pentene
1 is – 3.67 kJ/mol at 400 K. If more trans-2-
(I) H 2 (g) + O2 (g) ® H 2 O(l );
2 pentene is added to the reaction vessel, then
(a) more cis-2-pentene is formed
DH°298K = -285.9kJ mol-1
(b) equilibrium remains unaffected
1 (c) additional trans-2-pentene is formed
(II) H 2 (g) + O2 (g) ® H 2 O(g);
2 (d) equilibrium is shifted in forward direction
DH°298K = -241.8kJ mol-1 27. The dissolution of KCl in water is endothermic
yet it dissolves in water spontaneously. Which
The molar enthalpy of vapourisation of water
one of the following best explain s th is
will be :
behaviour?
(a) 241.8 kJ mol–1 (b) 22.0 kJ mol–1
(c) 44.1 kJ mol –1 (d) 527.7 kJ mol–1 (a) Endothermic processes are energetically
23. Given favoured.
Reaction Energy Change (b) The electrostatic bonds between the ions
(in kJ) are not too strong.
Li(s) ® Li(g) 161 (c) Energy changes have nothing to do with
Li(g) ® Li+(g) 520 the dissolution processes.
1 (d) The entropy driving force causes the
F (g) ® F(g) 77
2 2 dissolution.
F(g) + e– ® F–(g) (Electron gain 28. Hydrogen has an ionisation energy of 1311 kJ
enthalpy) mol –1 and for chlorine it is 1256 kJ mol –1.
Li+ (g) + F–(g) ® Li F(s) – 1047 Hydrogen forms H+ (aq) ions but chlorine does
1 not form Cl+ (aq) ions because
Li (s) + F2(g) ® Li F(s) – 617
2 (a) H+ has lower hydration enthalpy
Based on data provided, the value of electron (b) Cl+ has lower hydration enthalpy
gain enthalpy of fluorine would be : (c) Cl has high electron affinity
(d) Cl has high electronegativity
29. At 25°C, when 1 mole of MgSO4 was dissolved
(a) – 300 kJ mol–1 (b) – 350 kJ mol–1 in water, the heat evolved was found to be
(c) – 328 kJ mol–1 (d) – 228 kJ mol–1 91.2 kJ. One mole of MgSO 4 . 7H 2O on
24. Fixed mass of an ideal gas contained in a 24.63 dissolution gives a solution of the same
L sealed rigid vessel at 1 atm is heated from – composition accompanied by an absorption of
73°C to 27°C. Calculate change in gibb's energy 13.8 kJ. The enthalpy of hydration, i.e., DHh
if entropy of gas is a function of temperature as for the reaction
S = 2 + 10–2 T (J/K): (Use 1 atm L = 0.1 kJ) MgSO 4 (s) + 7H 2 O (l) ¾¾
® MgSO 4 .7H 2 O(s) is :
(a) 1231.5 J (b) 1281.5 J
(c) 781.5 J (d) 0 is:
25. For the reaction taking place at certain temperature
(a) – 105 kJ/mol (b) – 77.4 kJ/mol
NH 2 COONH 4 (s) ‡ˆˆˆˆ† 2NH 3 (g) + CO 2 (g),
(c) 105 kJ/mol (d) None of these
EBD_7587
58 Chemistry Objective MCQs
30. Calculate the heat produced (in kJ) when 224 g 35. 36 mL of pure water takes 100 sec to evaporate
of CaO is completely converted to CaCO3 by from a vessel and heater connected to an electric
reaction with CO2 at 27°C in a container of source which delivers 806 watt. The DH vap of
fixed volume. H2O is :
(a) 40.3 kJ/mol (b) 43.2 kJ/mol
(c) 4.03 kJ/mol (d) None of these
Given : DH°f (CaCO3,s) = – 1207 kJ/mol; 36. For the combustion reaction at 298 K
DH°f (CaO,s) = – 635 kJ/mol, DH°f (CO2,g) = 2Ag(s) + 1 / 2O 2 (g) ¾¾
® 2Ag 2O(s)
– 394 kJ/mol; [Use R = 8.3 JK–1 mol–1] Which of the following alternatives is correct?
(a) 702.04 kJ (b) 721.96 kJ
(c) 712 kJ (d) 721 kJ
(a) DH = DU
31. Given that: (b) DH > DU
(i) D f H° of N 2O is 82 kJ mol–1 (c) DH < DU
(d) DH and DU has no relation with each other
(ii) Bond energies of N º N, N = N, O = O
37. A gas undergoes change from state A to state B.
and N = O are 946, 418, 498 and 607 kJ
In this process, the heat absorbed and work done
mol–1 respectively,
by the gas is 5 J and 8 J, respectively. Now gas
The resonance energy of N2O is :
is brought back to A by another process during
(a) –88kJ (b) –66kJ (c) –62kJ (d) –44kJ
which 3 J of heat is evolved. In this reverse
32. The difference between the reaction enthalpy
process of B to A :
change (D rH) and reaction internal energy
(a) 10 J of the work will be done by the gas.
change (DrU) for the reaction:
(b) 6 J of the work will be done by the gas.
2C6 H 6 (l) + 15O 2 (g) ¾¾
®12CO 2 (g) + 6H 2 O(l) (c) 10 J of the work will be done by the
at 300 K is (R = 8.314 J mol–1K–1) surrounding on gas.
(a) 0 J mol–1 (b) 2490 J mol–1 (d) 6 J of the work will be done by the
(c) –2490 J mol–1 (d) –7482 J mol–1 surrounding on gas.
33. From given following equations and DH° 38. The amount of energy released when 20 mL of
values, determine the enthalpy of reaction at 0.5 M NaOH are mixed with 100 mL of 0.1 M
298 K for the reaction : HCl is x kJ. The heat of neutralisation (in kJ
mol-1) is
C 2 H 4 (g) + 6F2 (g) ¾¾
® 2 CF4 (g) + 4HF(g)
(a) –100x (b) –50x
® 2HF(g); DH1° = -537 kJ
H 2 (g) + F2 (g) ¾¾ (c) + 100 x (d) +50x
39. Which of the following value of D f H° represent
® CF4 (g); DH °2 = -680 kJ
C(s) + 2F2 (g) ¾¾
that the product is least stable ?
2C(s) + 2H 2 (g) ¾¾® C 2 H 4 (g); DH 3° = 52 kJ (a) –94.0 kcal mol–1 (b) –231.6 kcal mol–1
(a) – 1165 (b) – 2486 (c) +21.4 kcal mol–1 (d) +64.8 kcal mol–1
(c) + 1165 (d) + 2486 40. Enthalpy of neutralisation of HCl with NaOH
34. Substance A2B(g) can undergoes decomposition is x. The heat evolved when 500 mL of 2 N HCl
to form two set of products : are mixed with 250 mL of 4N NaOH will be.
(a) 500 x (b) 100x (c) x (d) 10 x
A2 (g) + B(g) ; DH° = 40 kJ / mol 41. The enthalpy of neutralisation of NH4OH
A2 B(g) with HCl is –51.46 kJ mol–1 and the enthalpy
A (g) + AB(g) ; DH° = 50 kJ / mol of neutralisation of NaOH with HCl is –55.90
kJ mol–1. The enthalpy of ionisation of NH4OH
If the molar ratio of A2 (g) to A (g) is 5 : 3 in a
set of product gases, then the energy involved is
in the decomposition of 1 mole of A2B (g) is :
(a) – 107.36 kJ mol–1 (b) – 4.44 kJ mol–1
(a) 48.75 kJ/mol (b) 43.73 kJ/mol
(c) + 107.36 kJ mol–1 (d) + 4.44 kJ mol–1
(c) 46.25 kJ/mol (d) None of these
Thermodynamics 59
42. The molar heat capacity (Cp) of CD2O is 10 cals TB
at 1000 K. The change in entropy associated TB
DS DS
with coolin g of 32 g of CD 2 O vapour
(a) (b) TA
from 1000 K to 100 K at constant pressure will TA
be: (D = deuterium, atomic mass = 2 u) T T
(a) 23.03 cal deg–1 (b) – 23.03 cal deg–1
(c) 2.303 cal deg –1 (d) – 2.303 cal deg–1 TB
TA
43. The (S°) of the following substances are: DS TA
TB
DS
CH4 (g) 186.2 JK–1 mol–1
O2 (g) 205.2 JK–1 mol–1 (c) (d)
CO2 (g) 213.6 JK–1 mol–1 T T
H2O (g) 69.9.JK–1 mol–1
49. What is the free energy change for the
The entropy change (DS°) for the reaction
CH4(g) + 2O2(g) ® CO2(g) + 2H2O(l) is: conversion of 1 mole of water into steam at
373.2 K. The heat of vaporization (DHv) of water
of 373.2 K is 9.1 kcal mol–1. The entropy change
is 25.5 cal/mol deg.
(a) – 312.5 J K–1 mol–1
(b) – 242.8 J K–1 mol–1 (a) – 401.6 cal/mol (b) – 416.6 cal/mol
(c) – 108.1 J K–1 mol–1 (c) 516.5 cal/mol (d) – 516.5 cal/mol
(d) – 37.6 J K–1 mol–1
44. The enthalpy of the reaction forming PbO 50. A certain reaction is non spontaneous at 298K.
according to the following equation is 438 kJ. The entropy change during the reaction is 121
What heat energy (kJ) is released in formation JK –1 . Is the reaction is endothermic or
of 22.3 g PbO(s)? exothermic ? The minimum value of DH for
(Atomic masses : Pb = 207, O = 16.0) the reaction is
2Pb(s) + O 2 (g) ¾¾ ® 2PbO(s) (a) endothermic, DH = 36.06 kJ
(a) 21.9 (b) 28.7 (c) 14.6 (d) 34.2 (b) exothermic, DH = – 36.06 kJ
45. The species which by definition has zero
(c) endothermic, DH = 60.12 kJ
standard molar enthalpy of formation at 298 K
is (d) exothermic, DH = – 60.12 kJ
(a) Br2 (g) (b) Cl2 (g) 51. The entropy of a sample of a certain substance
(c) H2O (g) (d) CH4 (g) increases by 0.836 J K–1 on adding reversibly
46. What is Dngas for the combustion of 1 mole of
0.3344 J of heat at constant temperature. The
benzene, when both the reactants and the
temperature of the sample is:
products are at 298 K ?
(a) 0 (b) 1/2 (c) 3/2 (d) –3/2 (a) 2.5 K (b) 0.3 K
47. The enthalpy of neutralisation of NH4OH and (c) 0.016 K (d) 0.4 K
52. One mole of an ideal gas is expanded
CH 3COOH is – 10.5 kcal mol–1 and enthalpy
isothermally and reversibly to half of its initial
of neutralisation of CH3COOH with strong base pressure. DS for the process in J K–1 mol–1 is
is – 12.5 kcal mol–1. The enthalpy of ionisation [ln 2 = 0.693 and R = 8.314, J/(mol K)]
of NH 4OH will be (a) 6.76 (b) 5.76 (c) 10.76 (d) 8.03
(a) 3.2 kcal mol–1 (b) 2.0 kcal mol–1 53. The enthalpy of hydrogenation of cyclohexene
(c) 3.0 kcal mol –1 (d) 4.0 kcal mol–1 is – 119.5 kJ mol –1. If resonance energy of
48. If for a given substance melting point is TB and benzene is –150.4 kJ mol –1, its enthalpy of
freezing point is T A, then correct variation
hydrogenation would be
shown by, graph between entropy change and
temperature is (a) – 208.1 kg mol–1 (b) – 269.9 kg mol–1
(c) – 358.5 kg mol–1 (d) – 508.9 kg mol–1
EBD_7587
60 Chemistry Objective MCQs
54. The enthalpies of formation of Al2O3 and Cr2O3 If in a container (operating at constant pressure)
are –1596 kJ and –1134 kJ respectively. DH for which is isolated from the surrounding, mixture
the reaction 2Al + Cr2 O3 ® 2Cr + Al2 O3 is of H2 (gas) and O2 (gas) are passed over excess
of B(s), then calculate the molar ratio (O2 : H2)
so that temperature of the container do n o t
change :
(a) – 2730 kJ (b) – 462 kJ (a) 15 : 3 (b) 42 : 1 (c) 1 : 42 (d) 1 : 84
(c) – 1365 kJ (d) + 2730 kJ 61. Combustion of sucrose is used by aerobic
55. The favourable conditions for a spontaneous organisms for providing energy for the life
reaction are sustaining processes. If all the capturing of
(a ) TDS > DH, DH = + ve, DS = + ve energy from the reaction is done through
electrical process (non P–V work) then calculate
(b) TDS > DH, DH = + ve, DS = - ve maximum available energy which can be
(c) TDS = DH, DH = -ve, DS = - ve captured by combustion of 34.2 g of sucrose
(d) TDS = DH, DH = + ve, DS = + ve Given : DH combustion (sucrose) = – 6000 kJ mol–1
56. The standard Gibb’s free energy change, DG° is DS combustion = 180 J/Kmol and body temperature
related to equilibrium constant, K P as is 300 K
DG o
é e ù
(a) K P = - RT ln DG o (b) KP = ê ú (a) 600 kJ (b) 594.6 kJ
ë RT û
(c) 5.4 kJ (d) 605.4 kJ
DG 62. The factor of DG values is important in
KP = -
o
(c) (d) K P = e -DG / RT
RT metallurgy. The DG values for the following
57. Which of the following salts should cause reactions at 800ºC are given as :
maximum cooling when 1 mole of it is dissolved
S2 (s) + 2O 2 (g) ¾¾
® 2SO 2 (g) ; DG = – 544 kJ
in the same volume of water ?
(a) NaCl; DHº = 5.35 kJ mol–1 ® 2ZnS(s) ; DG = – 293 kJ
2Zn(s) + S2 (s) ¾¾
(b) KNO3; DHº = 53.5 kJ mol–1
(c) KOH; DHº = –56.0 kJ mol–1 ® 2ZnO(s) ; DG = – 480 kJ
2Zn (s) + O 2 (g) ¾¾
(d) HBr; DHº = –83.3 kJ mol–1 Then DG for the reaction :
58. The molar enthalpies of combustion of isobutane
2ZnS(s) + 3O 2 (g) ¾ ¾
® 2ZnO(s) + 2SO 2 (g)
and n-butane are –2870 kJ mol–1 and –2875 kJ
mol–1 respectively at 298 K and 1 atm. Calculate will be :
DHº for the conversion of 1 mole of
n-butane to 1 mole of isobutane (a) –357 kJ (b) –731 kJ
(a) –8 kJ mol–1 (b) +8 kJ mol–1 (c) –773 kJ (d) –229 kJ
(c) –5748 kJ mol–1 (d) +5748 kJ mol–1 63. 1 gram equivalent of H2SO4 is treated with
59. The enthalpy of atomisation of CH4 and C 2H6 112 g of KOH for complete neutralization.
are 360 and 620 kcal mol–1 respectively. The Which of the following statementsis correct?
C – C bond energy is expected to be (a) 13.7 kcal of heat is evolved with the
formation of 87 g of K2SO4, leaving no
KOH.
(b) 27.4 kcal of heat is evolved with the
(a) 210 kcal mol–1 (b) 80 kcal mol–1 formation of 87 g of K2SO4, leaving 4 gram
(c) 130 kcal mol –1 (d) 180 kcal mol–1 equivalent of KOH.
60. Boron can undergo the following reactions with (c) 15.7 kcal of heat is evolved with the
the given enthalpy changes : formation of 1 gram equivalent of K2SO4,
3 leaving 56 g of KOH.
2B (s)
s + O O (g)g ¾¾ ®B s ;DH
B22O 3 (s); DH= = -1260
–1260 kJkJ
2 22 (d) 13.7 kcal of heat is evolved with the
s ++ 3H
2B (s) g ¾¾
O22 (g) ® BB22O s ; DH
H36 (g); DH == 30
-1260
kJ kJ formation of 87g of K2SO4, leaving 1 gram
Assume no other reactions are occurring. equivalent of KOH.
Thermodynamics 61
64. From the following data DH of the following 70. For the reaction
reactions 2C 6 H 5 CO 2 H(s) + 15O 2 (g) ¾¾® 14CO 2 (g) + 6H 2 O(g)
1
C(s) + O 2 (g ) ¾¾® CO(g); 2C 6 H 5 CO 2 H(s) + 15O 2 (g) ¾¾ ® 14CO 2 (g) + 6H 2 O(g)
2
DH = – 110 kJ and DUº = –772.7 kJ mol at 298 K. Calculate DHº
–1

C(s) + H 2 O(g) ¾¾® CO(g) + H2(g); (a) +760.3 kJ mol–1 (b) –760.3 kJ mol–1
DH = 132 kJ (c) +670.3 kJ mol–1 (d) –790.3 kJ mol–1
Calculate the mole composition of the mixture 71. DHof of water is – 285.8 kJ mol–1. If enthalpy
of steam and oxygen on being passes over coke
of neutralisation of monoacidic strong base is
at 1273 K, keeping the temperature constant.
–57.3 kJmol - 1 . DHf of OH - ion will be
o
(a) 1 : 0.6 (b) 0.6 : 1 (c) 2 : 3 (d) 3 : 2
65. Gasoline has an enthalpy of combustion 24000
kJ/ gallon. When gasoline burn s in an
automobile engine, approximately 30% of the
energy released is used to produce mechanical (a) – 114.25 kJ mol–1 (b) 114.25 kJ mol–1
work. The remainder is lost as heat transfer to (c) 228.5 kJ mol–1 (d) – 228.5 kJ mol–1
the engine's cooling system. As a start on 72. The heat of atomization of PH3(g) is 228 kcal
estimating how much heat transfer is required, mol–1 and that of P2H4(g) is 335 kcal mol–1. The
calculate what mass of water could be heated energy of the P–P bond is
from 25°C to 75°C by the combustion of 1.0 (a) 102 kcal mol–1 (b) 51 kcal mol–1
gallon of gasoline in an automobile? (c) 26 kcal mol–1 (d) 204 kcal mol–1
73. For the reaction
(Given : C (H2O) = 4.18 J/g°C) 1
CO(g) + O 2 (g) ¾¾ ® CO 2 (g), DH, and DS
(a) 34.45 kg (b) 80.383 kg 2
(c) 22 kg (d) 224 kg are –283 kJ and –87 JK–1, respectively. It was
66. 2 moles of an ideal gas at 27 ºC temperature is intended to carry out this reaction at 1000, 1500,
expanded reversibly from 2 L to 20 L. Find 3000 and 3500 K. At which of these temperatures
entropy change (R = 2 cal/mol K) would this reaction be thermodynamically
(a) 92.1 (b) 0 (c) 4 (d) 9.2 spontaneous?
67. One mole of solid iron was vaporized in an oven (a) 1500 and 3500 K
at its boiling point of 3433 K and enthalpy of (b) 3000 and 3500 K
vaporization of iron is 344.3 kJ mol–1. The value (c) 1000, 1500 and 3000 K
of entropy vaporization (in J mol–1) of iron is (d) 1500, 3000 and 3500 K
(a) 100 (b) 10 (c) – 100 (d) 110 74. The molar entropies of HI (g) and I (g) at
68. A reaction is spontaneous at low temperature 298 K are 206.5, 114.6, and 180.7 J mol–1 K–1
but non-spontaneous at high temperature. respectively. Using the DG° given Below,
Which of the following is true for the reaction? calculate the bond energy of HI.
HI (g ) ¾¾® H ( g ) + I (g ) ; DG° = 271.8 kJ
(a) DH > 0, DS > 0 (b) DH < 0, DS > 0 (a) 282.4 kJ mol–1 (b) 298.3 kJ mol–1
(c) 290.1 kJ mol –1 (d) 315.4 kJ mol–1
(c) DH > 0, DS = 0 (d) DH < 0, DS < 0
69. Which of the following pairs of processes is 75. What is the equilibrium constant if ATP
certain to occur in a spontaneous chemical hydrolysis by water produce standard free
reaction? energy of –50 kJ/mol under normal body
(a) Exothermic and increasing disorder conditions ?
(b) Exothermic and decreasing disorder
(c) Endothermic and increasing disorder (a) 2.66 × 108 (b) 5.81 × 108
(d) Endothermic and decreasing disorder (c) 1.18 × 107 (d) 1.98 × 108
EBD_7587
62 Chemistry Objective MCQs
76. 0.5 mole each of two ideal gases 82. Temperature of 5 moles of a gas is decreased by
æ 5 ö 2K at constant pressure. Indicate the correct
A ç Cv,m = R ÷ and B (Cv,m = 3R) are taken statement
è 2 ø
in a container and expanded reversibly and
(a) Work done by gas is = 5 R
adiabatically, during this process temperature
(b) Work done over the gas is = 10 R
of gaseous mixture decreased from 350 K to
250 K. Find DH (in cal/mol) for the process: (c) Work done by the gas = 10 R
(d) Work done = 0
83. An ideal gas occuping a volume of 2dm3 and a
(a) – 100 R (b) – 137.5 R pressure of 5 bar undergoes isothermal and
(c) – 375 R (d) None of these irreversible expansion against external pressure
77. What is the free energy change ' DG' when 1.0 of 1 bar. The final volume of the system and the
mole of water at 100 ºC and 1 atm pressure is work involved in the process is
converted into steam at 100 °C and 1 atm. (a) 10 dm3, 1000 J (b) 8 dm3, – 800 J
pressure (c) 10 dm3, – 800 J (d) 10 dm3, – 1000 J
(a) 540 cal (b) – 9800 cal 84. A piston filled with 0.04 mol of an ideal gas
(c) 9800 cal (d) 0 cal expands reversibly from 50.0 mL to 375 mL at
a constant temperature of 37.0ºC. As it does so,
78. In an irreversible process taking place at constant
it absorbs 208 J of heat. The values of q and w
T and P and in which only pressure-volume work
for the process will be:
is being done, the change in Gibbs free energy
(dG) and change in entropy (dS), satisfy the criteria (R = 8.314 J/mol K) (ln 7.5 = 2.01)
(a) q = + 208 J, w = – 208 J
(a) ( D S)V, E > 0, ( D G)T, P < 0 (b) q = – 208 J, w = – 208 J
(b) ( D S)V, E = 0, ( D G)T, P = 0 (c) q = – 208 J, w = + 208 J
(d) q = + 208 J, w = + 208 J
(c) ( D S)V, E = 0, ( D G)T, P > 0
85. The heats of neutralisation of CH 3COOH ,
(d) ( D S)V, E < 0, ( D G)T, P < 0
HCOOH, HCN and H2S are – 13.2, – 13.4, –
79. For the auto-ionization of water at 25°C,
2.9 and – 3.8 kcal per equivalent respectively.
ˆˆ† H + ( aq ) + OH - ( aq )
H 2 O (l ) ‡ˆˆ Arrange the acids in increasing order of strength
equilibrium constant is 10–14. (a) HCOOH > CH 3COOH > H 2S > HCN
What is DG° for the process? (b) CH 3COOH > HCOOH > H 2 S > HCN
(a) ; 8 × 104 J mol–1 (b) ; 3.5 × 104 J mol–1
(c) ; 2 × 104 J mol–1 (d) None of these (c) H 2S > HCOOH > CH 3COOH > HCN
80. For which of the following process, DS is (d) HCOOH > H 2S > CH 3COOH > HCN
negative? 86. The enthalpy of neutralisation of a weak acid in
(a) H2(g) ¾¾ ® 2H(g) 1 M solution with a strong base is – 56.1 kcal
(b) 2SO3(g) ¾¾ ® 2SO2(g) + O2(g) mol–1. If the enthalpy of ionisation of acid is
compressed 1.5 kcal mol–1 and enthalpy of neutralisation
(c) N 2 (4l ) ¾¾¾¾¾® N 2 (2l ) of the strong acid with a strong base is –
(d) C(diamond) ¾¾ ® C(graphite) 57.3 kJ eq–1 . What is the % ionisation of the
81. What is the entropy change (in JK–1 mol –1) weak acid in molar solution (assume the acid is
when one mole of ice is converted into water at monobasic)
0º C? (The enthalpy change for the conversion
of ice to liquid water is 6.0 kJ mol–1 at 0 ºC)
(a) 21.98 (b) 20.13 (c) 2.013 (d) 2.198 (a) 25 (b) 20 (c) 15 (d) 10
Thermodynamics 63
87. The free energy change for the following 93. Standard enthalpy of combustion of CH4 is
reactions are given below, – 890 kJ mol –1 and standard enthalpy of
5 vaporisation of water is 40.5 kJ mol –1. The
C2 H 2 (g) + O 2 (g) ® 2CO2 (g) + H 2 O(l); DG ° = -1234kJ
2 enthalpy change of the reaction
C2 H 2 (g) + O2 (g) ® 2CO 2 (g) + H 2O(l); DG ° = -1234kJ CH 4 (g) + 2 O 2 (g) ¾
¾® CO 2 (g) + H 2 O (g)

C(s) + O 2 (g) ® CO 2 (g); DG° = -394kJ (a) – 809.5 kJ mol–1 (b) – 890 kJ mol–1
(c) 809 kJ mol–1 (d) – 971 kJ mol–1
1
H 2 (g) + O2 (g) ® H 2O(l); DG ° = -237kJ 94. Standard enthalpy and standard entropy
2 changes for the oxidation of ammonia at 298 K
What is the standard free energy change for the
are – 382.64 kJ mol–1 and – 45.6 JK–1 mol–1,
reaction H2(g) + 2C (s) ¾¾ ® C2H2(g)
respectively. Standard Gibb's energy change for
(a) – 209 kJ (b) –2259 kJ
the same reaction at 298 K is
(c) + 2259 kJ (d) 209 kJ
88. For the reaction (a) - 22.1 kJ mol -1 (b) - 339.3 kJ mol -1
C3 H8 (g) + 5O2 (g) ® 3CO2 (g) + 4H 2 O(l) (c) - 439.3 kJ mol-1 (d) - 523.2 kJ mol-1
at constant temperature, DH – DE is 95. DG in Ag2O ® 2 Ag + 1/2O2 at a certain
(a) –RT (b) +RT (c) –3 RT (d) +3 RT temperature is –10 kJ mol–1. Pick the correct
89. Among the following, the intensive properties statement
are
(a) Ag2O decomposes to Ag and O2
(i) molar conductivity
(ii) electromotive force (b) Ag and O2 combines to form Ag2O
(iii) resistance (c) Reaction is in equilibrium
(iv) heat capacity (d) Reaction does not take place
(a) (i) and (ii) (b) (i), (ii) and (iii) 96. For complete combustion of ethanol,
(c) (i) and (iv) (d) (i) only C2 H5OH ( l ) + 3O 2 ( g ) ¾¾
® 2CO2 ( g ) + 3H 2 O ( l ) ,
90. What is the normal boiling point of mercury? the amount of heat produced as measured in
Given : DH°f (Hg, l) = 0; S°(Hg, l) = 77.4 J/K- bomb calorimeter, is 1364.47 kJ mol–1 at 25ºC.
mol DH°f (Hg, g) = 60.8 kJ/mol; S°(Hg, g) = Assuming ideality the enthalpy of combustion,
174.4 J/K–mol
DcH, for the reaction will be:
(a) 624.8 K (b) 626.8 K
(c) 636.8 K (d) None of these
(R = 8.314 kJ mol–1)
91. When 1 mole of oxalic acid is treated with excess
of NaOH in dilute aqueous solution, 106 kJ of (a) -1366.95 kJ mol-1
heat is liberated. Predict the enthalpy of (b) -1361.95 kJ mol-1
ionisation of the acid (c) -1460.95 kJ mol-1
(d) -1350.50 kJ mol-1
(a) 4.3 kJ mol–1 (b) – 4.3 kJ mol–1 97. Diborane is a potential rocket fuel which
(c) 8.6 kJ mol–1 (d) – 8.6 kJ mol–1 undergoes combustion according to the equation
B2 H 6 (g) + 3O 2 (s) ¾¾
® B2 O3 (s) + 3H 2O(g)
92. The lattice energy of solid NaCl is 180 kcal mol–1
Calculate the enthalpy change for the
and enthalpy of solution is 1 kcal mol–1. If the
hydration energies of Na + and Cl - ions are in combustion of diborane. Given
the ratio 3 : 2, what is the enthalpy of hydration
of sodium ion ? 3
(a) – 107.4 kcal mol–1 (b) 107.4 kcal mol–1 (i) 2B(s) + O 2 (g) ¾¾
® B2 O3 (s);
2
(c) 71.6 kcal mol–1 (d) – 71.6 kcal mol–1 DH = –1273 kJ per mol
EBD_7587
64 Chemistry Objective MCQs

1 99. How many molecules of ATP, undergo


(ii) H 2 (g) + O 2 (g) ¾¾® H 2O(l); hydrolysis to raise the temperature of 180 kg of
2
water which was originally at room temperature
DH = –286 kJ per mol
by 1°C ? C{P,m} water = 75.32 J/mol/K, DH{P}
(iii) H 2 O(l) ¾¾® H 2 O(g); for ATP hydrolysis = 7 kcal/mol
DH = 44 kJ per mol
(iv) 2B (s) + 3H2 (g) ¾¾ ® B2H6 (g);
DH = 36 kJ per mol (a) 1.5 × 1025 (b) 2.00 × 1023
(a) + 2035 kJ per mol (b) – 2035 kJ per mol (c) 3.4 × 1025 (d) 4.0 × 1024
(c) + 2167 kJ per mol (d) – 2167 kJ per mol
100. Consider the DG °f and DH°f (kJ/mol) for the
98. What is the amount of heat (in Joules) absorbed
following oxides. Which oxide can be most
by 18 g of water initially at room temperature
heated to 100°C ? If 10 g of Cu is added to this easily decomposed to form the metal and oxygen
water , than decrease in temperature (in Kelvin) gas?
of water was found to be? C (p,m) for water (a) ZnO (DG° = – 318.4, DH° = – 348.3)
75.32 J/mol K ; C (p,m) for Cu = 24.47 J/mol K. (b) Cu2O (DG° = – 146.0, DH° = – 168.8)
(c) HgO (DG° = – 58.5, DH° = – 90.8)
(a) 5649, 369 (b) 5544, 324 (d) PbO (DG° = – 187.9, DH° = – 217.3)
(c) 5278, 342 (d) 3425, 425

Answer KEY
1 (c) 11 (c) 21 (a) 31 (a) 41 (d) 51 (d) 61 (d) 71 (d) 81 (a) 91 (c)
2 (b) 12 (c) 22 (c) 32 (d) 42 (b) 52 (b) 62 (b) 72 (b) 82 (b) 92 (a)
3 (c) 13 (b) 23 (c) 33 (b) 43 (b) 53 (a) 63 (d) 73 (c) 83 (c) 93 (a)
4 (b) 14 (b) 24 (c) 34 (b) 44 (a) 54 (b) 64 (a) 74 (b) 84 (a) 94 (b)
5 (c) 15 (c) 25 (d) 35 (a) 45 (b) 55 (a) 65 (b) 75 (a) 85 (a) 95 (a)
6 (b) 16 (b) 26 (a) 36 (c) 46 (d) 56 (d) 66 (d) 76 (c) 86 (b) 96 (a)
7 (b) 17 (c) 27 (d) 37 (d) 47 (b) 57 (b) 67 (a) 77 (d) 87 (d) 97 (b)
8 (c) 18 (d) 28 (b) 38 (a) 48 (a) 58 (a) 68 (d) 78 (a) 88 (c) 98 (a)
9 (c) 19 (b) 29 (a) 39 (d) 49 (b) 59 (b) 69 (a) 79 (a) 89 (a) 99 (a)
10 (c) 20 (a) 30 (a) 40 (c) 50 (a) 60 (d) 70 (b) 80 (c) 90 (b) 100 (c)
Thermodynamics 65

1. (c) Internal energy and molar enthalpy are state BC : Isothermal expansion
functions. Work (reversible or irreversible) CD : Isochoric
is a path function. DA : Isothermal compression
2. (b) DH = DE + PDV, for solid and liquid, DV = 0
DU
or DH = DE + DnRT, for solids and liquids 11. (c) H2O(1, 323 K) ¾¾®
1
H2O(1, 373 K)
Dn = 0

¾®
DU2
3. (c) DH = H 2 - H1 = (E 2 + P2 V2 ) - (E1 + P1V1 ) DU3
H2O(g, 323 K) ¬¾¾ H2O(1, 373 K)
= (E 2 - E1 ) + (P2 V2 - P1V1 ) Cv,m(H2O, g) = 33.314 – 8.314
= 30 + (4 ´ 5 - 2 ´ 3) = 44 L atm = 25 J/K mol
DU2 = DH2 – DngRT = 37.6
1 3
DUtotal = Du1 + Du2 + Du3
4. (b) 2 N2 +
2 H2 ¾® NH3
= Cv, m (l) DT + DVvap + Cv,m (g) DT
1 3
2 x1 + 2 x2 – 3x3 75 ´ 50
= + 37.6 +
25 ´ 50
5. (c) Let x mole of KOH be neutralized by the 1000 1000
strong acid HA. Then, moles neutralized = 42.6 kJ/mol
12. (c) For isothermal reversible expansion.
by HB = 1 – x
V2
Hence, – 13.7 × x + (– 12.7) × (1 – x) = – 13.5 w = –nRT ln
V1
x 0.8 4 13. (b) In CH4, 4 × BE(C – H) = 360 kJ/mol
Þ x = 0.8; = = 4:1
1 - x 0.2 1
1 \ BE(C – H) = 90 kJ/mol
6. (b) T µ ; TV1/2 = constant In C2H6, BE(C – C) + 6 × BE(C – H) = 620 kJ/mol
V
For adiabatic process, \ BE(C – C) = 80 kJ/mol
80 ´ 103
TV g-1 = constant \ BE(C – C) = J/molecule
6.023 ´ 10 23
1 3 hc
\ g -1 = , g= Now, E =
2 2 l
7. (b) For a finite change DE = C v DT . For an 6.626 ´ 10 -34 ´ 3 ´108 ´ 6.023 ´10 23
\ l=
isothermal process DT = 0. Hence DE = 0 80 ´ 103
8. (c) 3H 2O (l) ® 3H 2O (g) ; \ l = 1.49 × 103 nm
14. (b) For combustion reaction, DH is negative,
Dn = 3, DE = DH - DnRT Dn = (16 + 18 ) – ( 25 + 2 ) = +7, so DS is + ve,
2 reaction is spontaneous, hence DG is –ve.
= 30 - 3 ´ ´ 500 = 27 kcal 3
1000 15. (c) For monoatomic gas Cv = 2 R Cp = 5 R
2
9. (c) Neutralisation of weak acid with strong base
At constant volume, DU = qv = nCv,m DT
hence < x. Extra heat is utilised to effect
the ionisation of weak acid. At constant pressure, DH = qp = nCp,m DT
10. (c) AB : Isobaric expansion 1 3 ˆˆ† NH 3 ;
16. (b) Given N + H ‡ˆˆ
2 2 2 2
EBD_7587
66 Chemistry Objective MCQs

1 3 DH° = – 285.9 kJ mol–1 … (1)


16. (b) Given ˆˆ† NH 3 ;
N 2 + H 2 ‡ˆˆ
2 2 1
H2(g) + O2 ( g ) ¾® H2O(g) ;
DHf = - 46.0 kJ / mol 2
ˆˆ† H 2 ; DH f = - 436 kJ / mol
H + H ‡ˆˆ DH° = – 241.8 kJ mol–1 … (2)
We have to calculate
ˆˆ† N 2 ; DH f = - 712 kJ / mol
N + N ‡ˆˆ H2O(l) ¾® H2O (g) ; DH° = ?
1 3
DH f ( NH3 ) = DH N - N + DH H - H - DH N - F On substracting eqn. (2) from eqn. (1) we
2 2 get
1 3 H2O(l) ¾® H2O(g) ;
-46 = ( -712 ) + ( -436 ) - DH N - F DH° = – 241.8 – (– 285.9)
2 2
On calculation = 44.1 kJ mol–1
DH N - F = - 964 kJ / mol 23. (c) Applying Hess’s Law
17. (c) To calculate average enthalpy of C – H 1
D f H ° = D sub H + D diss H + I.E. + E.A + D lattice H
bond in methane following informations 2
are needed -617 = 161 + 520 + 77 + E.A. + (–1047)
(i) dissociation energy of H2 i.e. E.A. = –617 + 289 = –328 kJ mol–1
1 \ electron affinity of fluorine
H ( g ) ¾¾ ® H( g ); DH = x (suppose )
2 2 = –328 kJ mol–1
(ii) Sublimation energy of C(graphite) to C(g)
P1 P2
® C( g ); DH = y (Suppose)
C( graphite) ¾¾ 24. (c) At constant volume, =
T1 T2
Given
300 3
Þ P2 = 1´ =
C( graphite) + 2H 2 ( g ) ¾¾
® CH 4 ( g ); 200 2
and V1 = 24.63 L
DH = 75 kJ mol-1
for single phase
18. (d) Given DH = 41 kJ mol–1 = 41000 J mol–1
Q dG = Vdp - SdT
T = 100°C = 273 + 100 = 373 K
Dn = 1 (
DG = VDP - ò 2 + 10-2 T dT )
DU = DH – DnRT = 41000 – (1 × 8.314 × æ 10-2 ´ 50, 000 ö
373) = 1231.5 - 200 - ç ÷
ç 2 ÷
= 37898.88 J mol–1 ; 37.9 kJmol–1 è ø
= 781.5 J
19. (b) DU = q + w
=10 × 1000 – 2 × (20) × 101.3 = 5948 J 25. (d) DG° = – RT ln KP; KP = (2x)2 X = 4x3
20. (a) Gaseous ions, when dissolved in water, get DG° = – RT ln (4X3)
hydrated and heat is evolved (heat of DG° = – RT ln 4 – 3RT ln X
hydration).
26. (a) If more trans-2-pentene is added, then its
Cl - (g) + aq ® Cl- (aq) is such reaction . concentration in right hand side will
21. (a) I – II, gives, 2Al + Fe2O3 ® 2Fe + Al2O3 increase. But in order to maintain the K
constant, concentration of cis-2-pentene
we have -400 - (-200) = -200; For one will also increase. Therefore more cis-2-
mole of iron the value is –100 cal pentene will be formed.
22. (c) Given 27 (d)
1 28. (b) Hydration energy of Cl+ is very less than
H2(g) + O2 ( g ) ¾® H2O(l) ; H+, hence it doesn’t form Cl+ (aq) ion.
2
Thermodynamics 67
29. (a) Given that 35. (a) 1 watt = 1 J/sec
MgSO4 (s) + nH2O ® MgSO4 nH2O; Total heat supplied for 36 mL H2O
= 806 × 100
Dr H1 = – 91.2 kJ/mol ...(i) = 80600 J
MgSO4 .7H2O (s) + (n – 7)H2O
80600
® MgSO4 (nH2O) DH vap = ´ 18
36
DrH2 = 13.8 kJ/mol ...(ii) = 40300 J or 40.3 kJ/mol
or DHhyd = DrH1 – DrH2 equation (i) – (ii) 36. (c) DH = DU + DnRT
= – 91.2 kJ/mol – 13.8 kJ/mol Dn = nP – nR
= – 105 kJ/mol 5 1
Now, Dn = 2 - =-
30. (a) CaO (s) + CO2 (g) – CaCO3 (s) 2 2
D f H ° = DH f° (CaCO 3 ) - DH °f (CaO ) - DH °f (CaCO 2 ) 1
\ DH = DU - RT
= – 1207 – (– 635) – (– 394) 2
= – 178 kJ/mol
\ DU = DH – Dng RT 1
Thus, DU = DH + RT
2
æ (-1) ´ 8.3 ´ 300 ö
DU = -178 - ç ÷ \ DU > DH
è 1000 ø q =+5, w = –8J
37. (d) A ¾¾¾¾¾¾ ®B
= – 175.51 kJ
224 DU AB = q + w = +5 + ( -8) = -3
n CaO = =4
56
q = – 3, DU BA = +3
\ qv = n DrU = 4 × (– 175.51)
= – 702.04 kJ DUBA = q + w
1 Þ 3 = -3 + w Þ w = + 6 J (work done on
31. (a) N 2 ( g ) + O 2 ® N 2 O (g )
2 + .. the system).
1 ××
N º N(g) + (O = O ) ® N ..
= N = O
..
(g) 38. (a) In question 0.01 gev of NaOH is being
2
o = [Energy required for breaking of neutralised by 0.01 gev of HCl and value
DH f
bonds]–[Energy released for forming of is x kJ, for 1 gev the value is –100x
bonds] (exothermic process).
39. (d) As Df H increases stability decreases.
1
= ( DH N º N + D H O = O - ( D H N = N + DH N = O ) 40. (c) Enthalpy of neutralisation of HCl with
2
NaOH is x. In question gev of HCl
1
= (946 + ´ 498) - (418 + 607) æ 500 ´ 2 ö æ 250 ´ 4 ö
2 ç = 1gev ÷ and NaOH ç = 1gev ÷
=170 kJ mol -1 è 1000 ø è 1000 ø
hence the value x.
Resonance energy = 82 - 170 = - 88 kJ mol
Resonance energy = 82 - 170 = - 88 kJ mol -1 41. (d) HCl ¾¾
® H + + Cl – ...(i)
Strong acid (Complete ionisation)
32. (d) DH = DU + DngRT
For the reaction Dng = 12 – 15 = – 3 ˆˆ† NH 4+
NH 4 OH ‡ˆˆ + OH –
Weak base
DH – DU = – 3 × 8.314 × 300
DH = x kJ mol–1 ...(ii)
= – 7482 J mol–1
+ –
33. (b) DH° = 2 × DH1° + 2 × DH°2 – DH°3 = –2486 kJ H + OH ¾¾
® H 2O
DH = –55.90 kJ mol–1 ...(iii)
5 3
34. (b) D r H = ´ 40 + ´ 50 = 43.75 kJ / mol (from neutralisation of strong acid and strong base)
8 8
EBD_7587
68 Chemistry Objective MCQs
From equation (i), (ii) and (iii) = – 10.5 – (– 13.7) – D H ion (CH 3COOH )
NH 4 OH + HCl ¾¾® NH 4+ –
+ Cl + H 2O = 13.7 – 10.5 – 1.2
DH = –51.46 kJ mol–1 = 2 kcalmol -1
\ x + (– 55.90) = – 51.46 48. (a) For a pure substance TA and TB represent
x = – 51.46 + 55.90 the same temperature. Hence A is a correct
= 4.44 kJ mol–1
choice.
\ Enthalpy of ionisation of NH4OH is
49. (b) DG =DH – TDS
Þ 4.44 kJ mol–1
42. (b) Given, Cp = 10 cals at 1000 K \ DG = 9100 - 373.2 ´ 25.5 = - 416.6 cal
T1 = 1000 K. T2 = 100 K mol–1
m = 32 g 50. (a) For non spontaneous reaction
DS = ? DG = + ve
at constant pressure DG = DH – T DS and
T
DS = C p ln 2 DS = 121 J K -1
T1
For DG = + ve
T2
= 2.303 × Cp log DH has to be positive. Hence the reaction is
T1
endothermic.
100
= 2.303 × 10 log The minimum value of DH can be obtained
1000
by putting DG = 0
= –23.03 cal deg–1
DH = TDS = 298 × 121 J
(
43. (b) DS° = S° CO 2 + 2 ´ S° H 2 O - S°CH 4 + 2 ´ S°O 2 ) = 36.06 kJ
= (213.6 + 2 × 69.9) – (186.2 + 2 × 205.2)
DH DH 0.3344
= – 242. 8 J K–1 mol–1. 51. (d) DS = ; T= = = 0.4K
T DS 0.836
1 22.3
44. (a) Q = 438 ´ = 21.9 kJ 52. (b) For isothermal process (DT = 0)
2 223
45. (b) The species in its elemental form has zero P1
DS = R ln = 8.314 ln 2
standard molar enthalpy of formation at P2
298 K. At 298K, Cl2 is gas while Br2 is
liquid. = 8.314 × 0.693 = 5.76

15
46. (d) C6 H 6 (l) + O2 (g) ® 6CO 2 (g) + 3H 2O (l)
53. (a) + H2 ¾
¾® ; DH= – 119.5 kJ
2

C6 H 6 (l) + O2 (g) ® 6CO 2 (g) + 3H 2O (l)


+ 3H2 ¾
¾® ; DH= 3(–119.5)
Dn = 6 – 15/2 = –3/2
47. (b) DneuH for strong base and strong acid = – 358.5 kJ
-1 The resonance energy provides extra
= – 13.7 kcal eq
stability to the benzene molecule so it has
DH ion (CH 3COOH ) to be overcome for hydrogenation to take
-1 place. So
= – 12.5 – (– 13.7) = 1.2 kcalmol
DH = – 358.5 – (–150.4) = –208.1 kJ
DH ion ( NH 4 OH)
Thermodynamics 69

3 34.2
54. (b) 2Al + O2 ® Al2 O3 , DH = -1596 kJ 61. (d) No. of moles of sucrose = = 0.1
2 342
.....(i) – (DG)T,P = useful work done by the system
3 – DG = – DH + TDS
2Cr + O 2 ® Cr2 O 3 , DH = -1134 kJ
2 180 ´ 0.1 ´ 300
....(ii) = + (6000 × 0.1) +
1000
By (i) – (ii) = 605.4 kJ
2Al + Cr2O3 ® 2Cr + Al 2O3 , DH = -462 kJ. 62. (b) For the reaction
2 ZnS ® 2 Zn + S2 ; DG1º = 293 kJ
55. (a) ΔG = ΔH-TΔS, ΔH is +ve,
..........(i)
ΔS is + ve; TΔS > ΔH
2 Zn + O2 ®2 ZnO ; DG2º = – 480 kJ
for spontaneous process.
..........(ii)
It will make DG, –ve
S2 + 2 O2 ®2 SO2 ; DG3º = – 544 kJ
56. (d) DG = -RT ln Kp or Kp = e–DG/RTT
.........(iii)
57. (b) Dissolution of KNO3 is endothermic, hence
heat is absorbed and cooling is observed. DGº for the reaction
58. (a) Isobutane + oxygen ® CO 2 + H 2 O 2 ZnS + 3 O2 ®2 ZnO + 2 SO2
can be obtained by adding eqn. (i), (ii) and
DH = -2870 kJ mol -1 ......(i) (iii)
n-butane + oxygen ® CO2 + H 2O Þ DGº = 293 – 480 – 544 = – 731 kJ

DH = -2878 kJ mol -1 ....(ii) 63. (d) H 2SO 4 + 2KOH ® K 2SO 4 + 2H 2 O


98 112 174 2 mole
(ii) – (i); n-butane – Isobutane, 49 56 87 1 mole
DH = (–2878 + 2870) 13.7 kcal is the heat evolved when 1 gev of
= – 8 kJ mol–1. strong acid is neutralised by 1 gev of strong
59. (b) Atomisation of methane base.
CH 4 (g) ¾
¾® C(g) + 4H(g); 64. (a) The first reaction is exothermic and the
DH = 360 kcal second reaction is endothermic. If on
360 passing the mixture of O2 and H2O (steam)
\ C – H bond energy =
4 over coke while keeping the temperature
-1 constant DH of both the reactions must be
= 90 kcal mol
same.
C 2 H 6 (g) ¾
¾® 2C(g) + 6H(g);
Moles of O2 needed to evolve 132 kJ
DH = 620 kcal
0.5 ´ 132
or H C -C + 6 H C -H = 620 = = 0.6
110
\ H C- C = 620 – 6 H C -H Hence steam : O2 ratio must be 1 : 0.6
= 620 – 6 × 90 = 80 kcal mol–1 65. (b) q = m × c × DT, m = q/(c × DT)
60. (d) No. of moles of O2 required to supplied 30 = (24 × 106 × 0.7)/(4.18 × 50)
kJ heat to second reaction = 80383 g or 80.383 kg
30 3 1 66. (d) Entropy change at constant temperature
= ´ =
1260 2 28 V1 20
1 = 2.303nR log V = 2.303 × 2 × 2 log
n O : nH = : 3 or 1 : 84 2 2
2 2 28 = 9.2 cal K–1 mol–1
EBD_7587
70 Chemistry Objective MCQs
= 355 kcal mol–1
D H 344.3 ´ 103
67. (a) DS = = = 100 J mol- 1 or 4 × 76 + (P – P) = 355 kcal mol–1
T 3443
P–P bond energy = 51 kcal mol–1
68. (d) We know that DG = DH – TDS
73. (c) Q Δ G =Δ H –T ×ΔS
When DH < 0 and DS < 0 then DG will be
For a spontaneous reaction, DG should be
negative at low temperatures (positive at
negative
high temperature) and the reaction will be
DH = – 238 kJ, DS = – 87 JK –1
spontaneous.
Hence, reaction will be spontaneous when
69. (a) Measure of disorder of a system is nothing
ΔH>T×ΔS. Therefore, at 1000, 1500 and
but entropy. For a spontaneous reaction,
3000 K the reaction would be spontaneous.
DG < 0. As per Gibbs Helmholtz equation,
74. (b) DS° = – 206.5 + 114.6 + 180.7 = 88.8
DG = DH – TDS
DG° = DH° – TDS°
Thus DG is –ve only DH° = 271.8 + 298 × 88.8 ´ 10–3
when DH = –ve (exothermic) DH° = 298.3 kJ mol–1
and DS = +ve (increasing disorder)
75. (a) DG = -RT ln K eq :
70. (b) DH º = DU º + DnRT
Normal body temperature = 37oC
5 ´ 8.314 ´ 298
DHº = -772.7 + kJ J
1000 Þ - 50 = 8.314 ´ 310 lnK eq
mol K mol
= -760.3 kJ mol -1 Þ 19.39 = lnKeq
1
71. (d) H 2 (g) + O 2 (g ) ¾
¾® H 2 O (l); Þ K eq = 2.6 ´ 108
2
DH = – 285.8 kJ .... (i) 76. (c) DH = (n1Cp,m + n2Cp,m ) DT
1 2
+
H (aq) + OH (aq) ¾
– ¾® H 2 O (l); æ 7 ö
= ç 0.5 ´ R + 0.5 ´ 4 R ÷ ( -100 )
DH = – 57.3 kJ .... (ii) è 2 ø
= – 375 R
1
H 2 (g) + aq ¾ ¾® H + (aq) + e - ; DH = 0 77. (d) Condition of equilibrium, hence DG = 0
2
78. (a) For spontaneous reaction, dS > 0 and DG
(by convention) .... (iii) and DG should be negative i.e. < 0.
(i) – (ii) – (iii) gives, 79. (a) DG° = –RT lnK
1 1 = –8.314JK–1mol–1 × 298 K × 2.303log 10–14
H 2 (g) + O 2 (g) + e - + aq ¾
¾® = 7.98 × 104 ; 8 × 104 J mol–1
2 2
80. (c) When gas is compressed, its entropy
OH - (aq) decreases so, DS is negative.
DH = – 285.8 + 57.3 = – 228.5 kJ DH
72. (b) Bond dissociation energy of 81. (a) DS =
T
PH3(g) = 228 kcal mol–1 DH per mole 6000
DS(per mole ) = =
T 273
228
P – H bond energy = = 76 kcal mol-1 = 21.98 JK mol-1
-1
3
82. (b) For 5 moles of gas at temperature T,
H H
PV1 = 5RTT
P P
For 5 moles of gas at temperature T – 2,
H H
Bond energy of 4 (P – H) + (P –P) PV2 = 5R(T – 2)
Thermodynamics 71

\ P (V2 - V1 ) = 5R(T – 2 – T); capacity are mass dependent, hence


extensive properties.
PDV = – 10R,
– PDV = 10R ˆˆ† Hg (g ),
90. (b) Hg (l ) ‡ˆˆ
When DV is negative, W is + ve. D RS° = 174.4 – 77.4 = 97 J/K-mol
83. (c) P1V1 = P2 V2 Q DG° = DH° – TDS° = 0
DH °
5bar ´ 2dm 3 T=
\ V= = 10 dm 3 DS°
1bar
60.8 ´1000
Work, W = – Pext ( (Vfinal - Vinitial ) = = 626.8 K
97
= – 1 bar ( 10 – 2)
91. (c) H + (aq) + OH - (aq) ‡ˆˆ
ˆˆ† H 2 O (l);
= – 1 × 10 5 Pa × 8 × 10 -3 m 3
= – 800 J DH = – 57.3 kJ .... (i)
84. (a) Process is isothermal reversible expansion, ˆˆ†
H2C2O4 + 2OH– ‡ˆˆ
hence
DU = 0, therefore q = – w. C 2 O -4 - + 2H2O; DH = –106 kJ
Since q = + 208 J, W = – 208 J .... (ii)
85. (a) The greater the (negative value) of heat of For the second reaction the value should
neutralisation, the more is the strength of have been
the acid. Hence, 2 × (– 57.3) = – 114.6 kJ
HCOOH > CH 3COOH > H 2S > HCN The difference
86. (b) The enthalpy of ionisation of weak acid is (114.6 – 106) = 8.6 kJ mol-1 is used to
given by effect of the ionisation of oxalic acid.
DHion (HA) DH hyd. = DH sol . – DH lattice
92. (a)
= DH N (weak acid / strong base) –
= 1 – 180 = – 179 kcalmol-1
DH N (strong acid / strong base)
Then DH hyd. ( Na + ) + DH hyd. (Cl - ) = – 179
= – 56.1 – (– 57.3) = 1.2 kJ mol-1

DH (ionisation ) = 1.5 kJ mol-1 2


or DH hyd. ( Na + ) + DHhyd = – 179
3
Hence % ionisation in 1 M solution
(1.5 - 1.2) or DH hyd. ( Na + ) = – 107.4 kcal mol-1
= × 100 = 20%
1.5 93. (a) CH 4 (g) + 2 O 2 (g) ¾
¾®
87. (d) By 2× (ii) – (i) + (iii) CO 2 (g) + 2 H 2 O (l)
H2(g) + 2C(s) ® C2H2(g), DG° = 209 kJ
DH = – 890 kJ .... (i)
88. (c) DH = DE + DnRT
2 H 2 O (l) ¾
¾® 2 H 2 O (g);
Dn = 3 – (1 + 5)
DH = 2 × 40.5 = 81 kJ .... (ii)
= 3 – 6 = –3
From (i) + (ii), we get
DH - DE = (-3RT )
CH 4 (g) + 2 O 2 (g) ¾
¾®
89. (a) Mass independent properties (molar
CO 2 (g) + 2 H 2 O (g)
conductivity and electromotive force) are
intensive properties. Resistance and heat DH = – 890 + 81 = – 809 kJ
EBD_7587
72 Chemistry Objective MCQs

94. (b) DG = DH - TDS Amount of heat gained by Cu

= -382.64 - (298 ´ 145.6 ´10 -3 ) = 24.47


J
´
10g
(373 – 298) K
K mol 63g / mol
= -339.3 kJ mol -1
= 291.3 J
95. (a) DG = –ve means the process is spontaneous. Heat lost by water = 291.30 J
96. (a) C 2 H 5 OH(l) + 3O 2 (g) ¾¾
® J
– 291.30 J = 75.32 ´ (T2 - 373K )
2CO2 (g) + 3H2O(l) K
Bomb calorimeter gives DU of the reaction
Þ –3.947 K = T2 –373 K
Given, DU = –1364.47 kJ mol–1
Þ T2 = 369.05 K
Dng = – 1
DH = DU + DngRT 99. (a) q p = DH = Cp dT
1 ´ 8.314 ´ 298
= -1364.47 - J
1000 Þ q p = 75.32 ´ (299 - 298) K
K mol
= – 1366.93 kJ mol–1
97. (b) For the equation J
Þ q p = 75.32
B 2 H 6 (g) + 3O 2 (g) ¾¾
® B 2 O 3 (g) + 3H 2 O(g) K mol
Eqs. (i) + 3 (ii) + 3 (iii) – (iv) For 180 kg of water, no. of moles of water
DH = – 1273 + 3(–286) + 3(44) – 36 180 ´ 103 g
= – 1273 – 858 + 132 – 36 = = 104 g moles
18g / mol
= – 2035 kJ/mol
J
98. (a) 18gm of water at 100oC q p = 75.32 ´ 10 4 moles
10gm of Cu at 25oC is added. mol
= 753.2 × 103 J = 753.2 kJ
DH for ATP = 7 kcal / mol
= 7 × 4.184 kJ/mol
o
100 C = 29.2 kJ/mol
6.022 × 1023 molecules of ATP produce
= 29.2 kJ
qp = Cp, m dT
29.2 kJ produced from
J 18g
= 75.32 ´ ´ (373 - 298) K 6.022 × 1023 molecules
K mol 18g / mol
753.2 kJ produced from
J 75.8
= 75.32 × 75 K 6.022 × 1023 ×
K 29.2
= 5.649 × 103 J 25
= 1.5 × 10 molecules
If now 10g of copper is added 100. (c) DG° = – RT ln K ;
Cp, m = 24.47 J/ mol K
K ­ stability of reactant ¯
7 Equilibrium
1. The equilibrium constant for the reaction 6. If 1.0 mole of I2 is introduced into 1.0 litre flask at
W + X ƒ Y + Z is 9. If one mole of each of w 1000 K, at quilibrium (Kc = 10–6), which one is
and x are mixed and there is no change in volume, correct
the number of moles of y formed is (a) [I2 (g)] > [I– (g)] (b) [I2 (g)] < [I– (g)]
(a) 0.10 (b) 0.50 (c) 0.75 (d) 0.54
1 -
2. When heated, ammonium carbamate decomposes (c) [I2 (g)] = [I– (g)] (d) [I2 (g)] = [I (g )]
as follows: 2
NH 4COONH2 (s) ‡ˆˆ ˆˆ† 2NH3 (g) + CO 2 (g) . At 7. In a reaction A + B C + D, the initial
a certain temperature, the equilibrium pressure concentrations, of A and B were 0.9 mol. dm–3
of the system is 0.318 atm. KP for the reaction is: each. At equilibrium the concentration of D was
(a) 0.128 (b) 0.426 found to be 0.6 mol dm–3. What is the value of
(c) 4.76 × 10–3 (d) None of these equilibrium constant for the reaction
3. One mole of SO3 was placed in a one litre reaction (a) 8 (b) 4 (c) 9 (d) 3
flask at a given temperature when the reaction 8. The partial pressure of CH3OH (g), CO (g) and
equilibrium was established in the reaction. H2 (g) in equilibrium mixture for the reaction,
2SO3 ƒ 2SO2 + O2 . The vessel was found CO (g) + 2H2 (g) CH3OH (g) are 2.0, 1.0
to contain 0.6 mole of SO2. The value of the and 0.1 atm respectively at 427°C. The value of
equilibrium constant is Kp for the decomposition of CH3OH to CO and
H2 is
(a) 0.36 (b) 0.675 (c) 0.45 (d) 0.54
(a) 102 atm (b) 2 × 102 atm–1
4. For the equilibrium system
(c) 50 atm2 (d) 5 × 10–3 atm2
ˆˆ†
2HX(g) ‡ˆˆ ˆ
ˆ H 2 (g) + X 2 (g)
9. 3.2 moles of hydrogen iodide were heated in a
the equilibrium constant is 1.0×10–5. What is the sealed bulb at 444°C till the equilibrium state
concentration of HX if the equilibrium was reached. Its degree of dissociation at this
concentration of H2 and X2 are 1.2 × 10–3 M, and
temperature was found to be 22% The number of
1.2 × 10–4 M respectively.
moles of hydrogen iodide present at equilibrium
(a) 12 × 10–4 M (b) 12 × 10–3 M
–2 are
(c) 12 × 10 M (d) 12 × 10–1 M
5. If K1 and K2 are respective equilibrium constants
for the two reactions (a) 2.496 (b) 1.87 (c) 2 (d) 4
XeF6 (g) + H2O (g) XeOF4 (g) + 2HF (g)
10. K c for PCl5 (g) PCl3 (g) + Cl 2 (g) is
XeO4 (g) + XeF6 (g) ƒ XeOF4 (g) + XeO3F2 (g)
0.04 at 250°C. How many moles of PCl5 must be
the equilibrium constant for the reaction
added to a 3 L flask to obtain a Cl2 concentration
XeO4 (g) + 2HF (g) XeO3F2 (g) + H2O (g) will
of 0.15 M
be
K1 (a) 4.2 moles (b) 2.1 moles
K1 K2
(a) (b) K1.K 2 (c) (d) (c) 5.5 moles (d) 6.3 moles
K 22 K2 K1
EBD_7587
74 Chemistry Objective MCQs
11. Gaseous N2O4 dissociates into gaseous NO2
ˆˆ† 2NH3 (g )
N 2 (g ) + 3H 2 (g ) ‡ˆˆ
according to the reaction

éë N 2 O 4 ( g ) ƒ 2NO 2 ( g )ùû
-2
At 300 K and 1 atm pressure, the degree of 2 æ 800R ö
dissociation of N2O4 is 0.2. If one mole of N2O4 (a) 16 ´ (800R ) (b) ç ÷
è 4 ø
gas is contained in a vessel, then the density of 2
æ 1 ö
the equilibrium mixture is : (c) ç ÷ (d) None of these
(a) 1.56 g/L (b) 6.22 g/L è 4 ´ 800R ø
(c) 3.11 g/L (d) 4.56 g/L 17. At a certain temperature, only 50% HI is
12. The value of Kp for the equilibrium reaction dissociated into H2 and I2 at equilibrium. The
equilibrium constant is:
N 2O 4 ( g ) ƒ 2NO2 ( g ) is 2. (a) 1.0 (b) 3.0 (c) 0.5 (d) 0.25
The percentage dissociation of N2O4(g) at a 18. Some inert gas is added at constant volume to
pressure of 0.5 atm is the following reaction at equilibrium
(a) 25 (b) 88 (c) 50 (d) 71 ˆˆ† NH3 (g) + H 2S(g)
NH 4HS(s) ‡ˆˆ
13. If CuSO 4 .5H 2O (s) Predict the effect of adding the inert gas :
(a) The equilibrium shifts in the forward
CuSO 4 .3H2O(s) + 2H 2O(g) direction
Kp= 1.086 × 10–4 atm 2 at 25° C. The efforescent (b) The equilibrium shifts in the backward
nature of CuSO4.5H2O can be noticed when the direction
vapour pressure of H2O in atmosphere is (c) The equilibrium remains unaffected
(d) The value of KP is increased
(a) > 9.72 mm (b) < 7.92 mm
19. The standard state Gibb's free energy change for
(c) > 7.92 mm (d) < 11.92 mm the isomerisation reaction
14. 28g N2 and 6.0 g of H2 are heated over catalyst in cis-2-pentene trans-2-pentene
a closed one litre flask of 450°C. The entire –1
is –3.67 kJ mol at 400 K. If more trans-2-pentene
equilibrium mixture required 500 mL of 1.0 M
is added to the reaction vessel
H2SO4 for neutralisation. The value of Kc for the
reaction (a) more cis-2-pentene is formed
(b) equilibrium shifts in the forward direction
N 2 (g) + 3H 2 (g ) 2 NH 3 (g) is (c) equilibrium remains unaltered
(a) 0.06 mol -2 L2 (b) 0.59 mol -2 L2 (d) more trans-2-pentene is produced
(c) 1.69 mol L2 -2
(d) 0.03 mol 2 L-2 20. (1) N2(g) + 3H2(g) ‡ˆˆˆˆ† 2NH3(g), K1
15. A gaseous compound of molecular mass 82.1 ˆˆ† 2NO(g), K2
(2) N2(g) + O2(g) ‡ˆˆ
dissociates on heating to 400 K as 1
(3) H2(g) + O2(g) ‡ˆˆ ˆˆ† H2O(g), K3
X 2 Y4 (g) X 2 (g) + 2 Y2 (g) 2
The equation for the equilibrium constant of the
The density of the equilibrium mixture at a reaction
pressure of 1 atm and temperature of 400K is
2.0gL-1 . The percentage dissociation of the
compound is 5
2NH3(g) + ˆˆ† 2NO(g) + 3H O(g),
O2 (g) ‡ˆˆ 2
2
(a) 12.5% (b) 48.5% (c) 90.1% (d) 25.0% (K4) in terms of K1, K2 and K3 is :
16. At 527 °C, the reaction given below has Kc = 4 K1.K 2 K1.K 32
1 3 (a) K3 (b)
ˆˆ† N 2 ( g ) + H 2 ( g )
NH3 (g ) ‡ˆˆ K2
2 2 K 2 .K 33
What is the KP for the reaction? (c) K1 K2 K3 (d)
K1
Equilibrium 75
21. On the basis of Le-Chatelier's principle, predict 27. For a reversible gaseous reaction
which of the following conditions would be ˆˆ† 2NH3 at equilibrium, if some
N2 + 3H 2 ‡ˆˆ
unfavourable for the formation of SO3? Given that
moles of H2 are replaced by same number of
2SO2 + O2 2SO3; DH = –42 kcal
moles of T2 (T is tritium, isotope of H and assume
isotopes do not have different chemical
properties) without affecting other parameter,
(a) Low pressure and low temperature then :
(b) High pressure and low temperature
(a) the sample of ammonia obtained after
(c) High temperature and low pressure
sometime will be radioactive.
(d) High concentration of SO2
(b) moles of N2 after the change will be different
22. The reaction, SO 2 + Cl 2 ¾ ¾® SO 2 Cl 2 is
as compared to moles of N2 present before
exothermic and reversible. A mixture of SO2 (g), the change
Cl2 (g) and SO2Cl2 (l) is at equilibrium in a closed (c) the value of KP or Kc will change
container. Now a certain quantity of extra SO2 is (d) the average molecular mass of new
introduced into the container, the volume equilibrium will be same as that of old
remaining the same. Which of the following is/ equilibrium
are true? Kp
(a) The pressure inside the container will not 28. The ratio for the reaction
change. Kc
1
(b) The temperature will not change. ˆˆ† CO 2 ( g ) is:
CO( g ) + O2 ( g ) ‡ˆˆ
(c) The temperature will increase. 2
(d) The temperature will decrease. 1
ˆˆ† 2C + D, initial (a) (b) ( RT )1/ 2
23. In reaction A + 2B ‡ˆˆ RT
concentration of B was 1.5 times of [A], but at (c) RT (d) 1
equilibrium the concentrations of A and B became 29. Which of the following is true at chemical
equal. The equilibrium constant for the reaction equilibrium?
is: (a) (DG)T, P is minimum and (DS)U,V is also
minimum
(a) 8 (b) 4 (c) 12 (d) 6 (b) (DG)T,V is minimum and (DS)U,V is maximum
24. For the reaction : (c) (DG)T,V is maximum and (DS)U,V is zero
ˆˆ† 2BaO(s) + O 2 (g);
2BaO 2 (s) ‡ˆˆ (d) (DG)T,P is zero and (DS)U,V is also zero
DH = +ve. In equilibrium condition, pressure of 30. K1, K2 and K3 are the equilibrium constants of
O2 is dependent on the following reactions (I), (II) and (III)
(a) mass of BaO2 respectively:
(b) mass of BaO (I) N2 + 2O2 2NO2
(c) temperature of equilibrium
(d) mass of BaO2 and BaO both (II) 2NO2 N2 + 2O2
25. For the reaction 1
(III) NO2 N 2 + O2
NH4HS(g) NH3(g) + H2S(g) in a closed 2
flask, the equilibrium pressure is P atm. The The correct relation from the following is
standard free energy of the reaction would be: 1 1
(a) K1 = =
K 2 K3
(a) – RT ln p (b) – RT (ln p – ln 2) 1 1
(b) K1 = =
(c) – 2 RT ln p (d) – 2 RT (ln p – ln 2) K 2 ( K3 ) 2
26. ˆˆ† 2HI(g) at
For the reaction H 2 (g) + I 2 (g) ‡ˆˆ
721 K, the value of equilibrium constant is 50, (c) K1 = K 2 = K3
when equilibrium concentration of both is 5M. 1
(d) K1 = = K3
Value of Kp under the same conditions will be K2
(a) 0.02 (b) 0.2 (c) 50 (d) 50 RT
EBD_7587
76 Chemistry Objective MCQs
31. If Ksp of CaF2 at 25°C is 1.7 × 10–10, the 35. A solution of 0.1 M NaZ has pH = 8.90. The Ka of
combination amongst the following which gives HZ is
a precipitate of CaF2 is

(a) 1 × 10–2 M Ca2+ and 1 × 10–3 M F– (a) 6.3 × 10–11 (b) 6.3 × 10–10
(c) 1.6 × 10–5 (d) 1.6 × 10–6
(b) 1 × 10–4 M Ca2+ and 1 × 10–4 M F–
(c) 1 × 10–2 M Ca2+ and 1 × 10–5 M F– 36. The following equilibrium is established when
hydrogen chloride is dissolved in acetic acid.
(d) 1 × 10–3 M Ca2+ and 1 × 10–5 M F–
+
32. The solubility of PbI2 at 25°C is 0.7 g L–1. The HCl + CH3COOH ƒCl– + CH 3COOH 2
solubility product of PbI2 at this temperature is The set that characterises the conjugate acid-
(molar mass of PbI2 = 461.2 g mol–1) base pairs is
(a) 1.40 × 10–9 (b) 0.14 × 10–9 +
(c) 140 × 10 –9 (d) 14.0 × 10–9 (a) (HCl, CH3COOH) and ( CH 3 COOH 2 , Cl–)
32. I– ions react with iodine in aqueous solution to +
form I3- ion as (b) (HCl, CH 3COOH 2 ) and (CH3COOH, Cl–)
I–(aq) + I 2 (aq) I3- (aq). +
(c) ( CH 3COOH 2 , HCl) and (Cl–, CH3COOH)
When L of solution containing 1 mol of KI and
0.25 mol of I 2 was reacted with excess of +
(d) (HCl, Cl–) and ( CH 3COOH 2 , CH3COOH)
AgNO 3 , 0.80 mol of yellow ppt. was obtained.
The stability constant of I3- ion is 37. A solution contains 10 mL 0.1 N NaOH and 10 mL
(a) 0.20 (b) 5.0 (c) 20.0 (d) 0.05 0.05 N H 2SO 4 , pH of this solution is :
33. Consider the expression DG = –RTlnKp+RTlnQp (a) less than 7 (b) 7
and indicate the correct statement at equilibrium (c) zero (d) greater than 7
(a) DG = 0, Qp> Kp the equilibrium reaction will 38. 8 mol of AB3(g) are introduced into a 1.0 dm3
shift from left to right vessel. If it dissociates as
(b) DG = 0, Qp=Kp the equilibrium reaction will 2AB 3(g) A2(g) + 3B 2(g) . At equilibrium, 2
shift from left to right mol of A2 are found to be present. The equilibrium
(c) DG = ¥,Qp<Kp the equilibrium reaction will constant of this reaction is
shift from right to left
(d) DG<0, Qp>Kp the equilibrium reaction will
shift from right to left where Qp and Kp term (a) 2 (b) 3 (c) 27 (d) 36
39. Assuming that the degree of hydrolysis is small,
refer to reaction quotient and equilibrium
the pH of 0.1 M solution of sodium acetate
constant at constant pressure respectively.
(Ka = 1.0 × 10–5) will be:
34. If the synthesis of ammonia from Haber's process
(a) 5.0 (b) 6.0 (c) 8.0 (d) 9.0
is carried out with exactly the same starting
conditions (of partial pressure and temperature) 40. The hydrogen ion concentration of 0.2 N
CH3COOH which is 40% dissociated is
but using D2 (deuterium) in place of H2. Then
(a) the equilibrium will be disturbed (a) 0.08 N (b) 0.12 N (c) 0.80 N (d) 1.2 N
(b) the composition of reaction mixture will 41. Which one of the following is the correct
remain same at equilibrium. statement ?
(c) Use of isotope in reaction will not produce (a) HCO3– is the conjugate base of CO32–.
ammonia. (b) NH2– is the conjugate acid of NH3.
(d) At equilibrium rate of forward reaction will (c) H2SO4 is the conjugate acid of HSO4–.
be greater than the rate of reverse reaction (d) NH3 is the conjugate base of NH2–.
Equilibrium 77
42. Which equilibrium can be described as an acid- 48. The first and second dissociation constants of
base reaction using th e Lewis acid-base an acid H2A are 1.0 × 10–5 and 5.0 × 10–10
definition but not using the Bronsted-Lowry respectively. The overall dissociation constant
definition? of the acid will be
(a) 2NH3 + H2SO4 2NH4+ + SO42– (a) 0.2 × 105 (b) 5.0 × 10–5
(c) 5.0 × 10 15 (d) 5.0 × 10–15
(b) NH3 + CH3COOH NH4+ + CH3COO–
(c) H2O + CH3COOH H3O+ + CH3COO– 49. Values of dissociation constant, Ka are given as
follows :
(d) [Cu(H2O)4]2+ + 4NH3 ƒ [Cu(NH3)4]2+ + 4H2O Acid Ka
43. At a certain temperature the dissociation constants HCN 6.2 × 10–10
of formic acid and acetic acid are 1.8 × 10–4 and HF 7.2 × 10–4
1.8 × 10–6 respectively. The concentration of acetic HNO2 4.0 × 10–4
acid solution in which the hydrogen ion has the Correct order of increasing base strength of the
same concentration as in 0.001 M formic acid _
base CN–, F– and NO2 will be :
solution is equal to _
(a) F- < CN - < NO2
(a) 0.001 M (b) 0.01 M _
(b) NO2 < CN - < F-
(c) 0.1 M (d) 0.0001 M _
44. One mole of O2(g) and two moles of SO2(g) were (c) F- < NO2 < CN -
_
heated in a closed vessel of one-litre capacity at (d) NO2 < F- < CN -
1098 K. At equilibrium 1.6 moles of SO3(g) were 50. Given
found. The equilibrium constant Kc of the
reaction would be (i) HCN(aq) + H 2 O(l) ƒ H3O + (aq) + CN - (aq)
Ka = 6.2 × 10–10
(a) 30 (b) 40 (c) 80 (d) 60
45. The increase of pressure on ice ƒ water system (ii) CN - (aq) + H 2 O(l) ƒ HCN(aq) + OH - (aq)
at constant temperature will lead to Kb = 1.6 × 10–5.
(a) a decrease in the entropy of the system These equilibria show the following order of the
(b) an increase in the Gibb’s energy of the relative base strength,
system (a) OH– > H2O > CN– (b) OH– > CN– > H2O
(c) no effect on the equilibrium (c) H2O > CN– > OH– (d) CN– > H2O > OH–
(d) a shift of the equilibrium in the forward 51. Which of the following statements about pH and
direction H+ ion concentration is incorrect?
46. What is the decreasing order of basic strengths (a) Addition of one drop of concentrated HCl
of in NH4OH solution decreases pH of the
OH–, NH2–, H–C º C– and CH3 – CH2– solution.
(a) CH 3 - CH -2 > NH -2 > H - C º C - > OH - (b) A solution of the mixture of one equivalent of
each of CH3COOH and NaOH has a pH of 7
(b) H - C º C - > CH 3 CH -2 > NH -2 > OH - (c) pH of pure neutral water is not zero
(c) OH - > NH -2 > H - C º C - > CH 3 - CH -2 (d) A cold and concentrated H2SO4 has lower
H+ ion concentration than a dilute solution
(d) NH -2 > H - C º C - > OH - > CH 3 - CH -2
of H2SO4
47. When CO2 dissolves in water, the following 52. Assuming that the buffer in the blood is
equilibrium is established
CO2 - HCO3– . Calculate the ratio of conjugate
CO2 + 2H2O H3O+ + HCO3–
base to acid necessary to maintain blood at its
for which the equilibrium constant is 3.8 × 10–7
proper pH of 7.4.
and pH = 6.0. The ratio of [HCO3–] to [CO2] would
be K1 (H 2 CO 3 ) = 4.5 × 10–7
(a) 3.8 × 10–13 (b) 3.8 × 10–1
(c) 6.0 (d) 13.4
(a) 11 (b) 8 (c) 6 (d) 14
EBD_7587
78 Chemistry Objective MCQs
53. If the reaction between CO2 and H2O is 60. The conjugate base of hydrazoic acid is:
ˆˆ† H 2 CO3 ‡ˆˆ
CO 2 + H 2O ‡ˆˆ ˆˆ† H + + HCO3– (a) N–3 (b) N 3- (c) N -2 (d) HN 3-
If CO2 escapes from the system 61. NaOH is a strong base. What will be pH of
(a) pH will decrease 5.0 × 10–2 M NaOH solution ? (log 2 = 0.3)
(b) H+ concentration will decrease (a) 14.00 (b) 13.70 (c) 13.00 (d) 12.70
(c) H2CO3 concentration will be altered
62. The solubility product of Ag2CrO4 is 32 ´ 10 -12 .
(d) The forward reaction is promoted
54. Calculate the pH of a solution containing 0.1 M What is the concentration of CrO24 - ions in that
HCO 3- and 0.2 M CO 32- solution (in g L–1)
[K1(H2CO3) = 4.2×10–7×10 and (a) 2 × 10–4 (b) 8 × 10–4
K2( HCO 3- ) = 4.8×10–11]. (c) 8 × 10 –8 (d) 16 × 10–4
(a) 3.18 (b) 10.62 (c) 6.62 (d) 9.31 63. What is the minimum concentration of SO42–
55. What is the pH of a 10–4 M OH– solution at 330K, required to precipitate BaSO4 in a solution
if Kw at 330K is 10–13.6 ? containing 1.0 × 10–4 mole of Ba2+ ?
Ksp for BaSO4 = 4 × 10–10
(a) 4 (b) 9.0 (c) 10 (d) 9.6 (a) 4 × 10–10 M (b) 2 × 10–7 M
56. What happens when an inert gas is added to an (c) 4 × 10 M–6 (d) 2 × 10–3 M
equilibrium keeping volume unchanged?
64. Equimolar solutions of the following compounds
(a) More product will form
are prepared separately in water. Which will have
(b) Less product will form
the lowest pH value ?
(c) More reactant will form
(a) BeCl2 (b) SrCl2
(d) Equilibrium will remain unchanged
(c) CaCl2 (d) MgCl2
57. The degree of dissociation of 0.1M weak acid
HA is 0.5%. If 2 mL of 1.0 M HA solution is diluted 65. Addition of which chemical will decrease the
to 32 mL the degree of dissociation of acid and hydrogen ion concentration of an acetic acid
H3O+ ion concentration in the resulting solution solution
will be respectively (a) NH4Cl (b) Al2(SO4)3
(a) 0.02 and 3.125 × 10–4 (c) AgNO3 (d) NaCN
(b) 1.25 × 10–3 and 0.02 66. What is the percentage hydrolysis of NaCN in
(c) 0.02 and 1.25 × 10–3 N/80 solution when the dissociation constant for
(d) 0.02 and 8.0 × 10–12 HCN is 1.3 × 10–9 and Kw = 1.0 × 10–14
58. 20 mL of 0.2 M NaOH are added to 50 mLof (a) 2.48 (b) 5.26 (c) 8.2 (d) 9.6
0.2 M CH3COOH (K a = 1.8 ´10 -5 ) the pH of 67. The dissociation constant of 0.1 M acetic acid
the solution is solution is 1.8 × 10–5. If 1 L of this solution is
(a) 4.56 (b) 4.73 (c) 9.45 (d) 6.78 mixed with 0.05 mole of HCl, what will be pH of
59. Why only As3+ gets precipitated as As2S3 and mixture ? [log 5 = 0.7]
not Zn 2+ as ZnS when H2S is passed through an (a) 1.3 (b) 2.6 (c) 1.9 (d) 3.4
acidic solution containing As3+ and Zn 2+? 68. For preparing a buffer solution of pH 6 by mixing
(a) Solubility product of As2S3 is less than that sodium acetate and acetic acid, the ratio of the
of ZnS concentration of salt and acid should be
(b) Enough As3+ are present in acidic medium (Ka = 10–5)
(c) Zinc salt does not ionise in acidic medium
(d) Solubility product changes in presence of (a) 1 : 10 (b) 10 : 1 (c) 100 : 1 (d) 1 : 100
an acid
Equilibrium 79
69. In a saturated solution of the sparingly soluble 76. A buffer solution is prepared by mixing 10 mL of
strong electrolyte AgIO3 (molecular mass = 283) 1.0 M CH3COOH and 20 mL of 0.5 M CH3COONa
the equilibrium which sets is and then diluted to 100 mL with distilled water. If
AgIO 3(s) Ag + (aq) + IO 3- (aq) . If the pKa of CH3COOH is 4.76, what is the pH of the
solubility product constant Ksp of AgIO3 at a buffer solution ?
given temperature is 1.0 × 10–8, what is the mass
of AgIO3 contained in 100 mL of its saturated
solution? (a) 5.8 (b) 4.34 (c) 5.21 (d) 4.76
77. Which one of the following arrangements
(a) 1.0 × 10– 4 g (b) 28.3 × 10–2 g represents the correct order of the proton affinity
(c) 2.83 × 10–3 g (d) 1.0 × 10–7 g. of the given species :
70. At 25°C, the dissociation constant of a base, (a) I– < F– < HS– < NH -2
BOH, is 1.0 ´ 10-12. The concentration of
hydroxyl ions in 0.01 M aqueous solution of (b) HS– < NH -2 < F– < I–
the base would be
(a) 1.0 ´ 10- 5 mol L-1 (b) 1.0 ´ 10-6 mol L-1 (c) F– < I– < NH -2 < HS–
(c) 2.0 ´ 10-6 mol L-1 (d) 1.0 ´ 10-7 mol L-1
71. What would be the pH of a solution obtained by (d) NH 2- < HS- < I - < F -
mixing 5g of acetic acid and 7.5g of sodium 78. Consider the following equilibrium
acetate and making the volume equal to 500 mL? +
(Ka = 1.75 × 10–5, pKa = 4.76) AgCl ¯ +2NH3 ƒ éëAg ( NH 3 )2 ùû + Cl -
(a) pH =4.70 White precipitate of AgCl appears on adding
(b) pH < 4.70 which of the following?
(c) pH of solution will be equal to pH of acetic (a) NH3 (b) aqueous NaCl
acid (c) aqueous HNO3 (d) aqueous NH4Cl
(d) 4.76 < pH < 5.0 79. The degree of hydrolysis in hydrolytic equilibrium
72. In some solutions, the concentration of H3O+ A - + H 2O HA + OH -
remains constant even when small amounts of at salt concentration of 0.001 M is :
strong acid or strong base are added to them.
( K a = 1´10 -5 )
These solutions are known as:
(a) 1 ´ 10 -3 (b) 1 ´ 10 - 4
(a) Ideal solutions (b) Colloidal solutions
(c) 5 ´ 10 - 4 (d) 1 ´ 10 - 6
(c) True solutions (d) Buffer solutions
73. If degree of dissociation of pure water at 100 °C 80. A litre of solution is saturated with AgCl. To this
is 1.8 × 10–8, then the dissociation constant of solution if 1.0 ´ 10 -4 mole of solid NaCl is added,
water will be (density of H2O = 1 g/cc) what will be the [Ag + ] , assuming no volume
change?
(a) More (b) Less (c) Equal (d) Zero
(a) 1 × 10–12 (b) 1 × 10–14 81. If the equilibrium constant of the reaction of weak
(c) 1.8 × 10 –12 (d) 1.8 × 10–14 acid HA with strong base is 10–7, then pOH of
74. The dissociation constant of two acids HA1 and the aqueous solution of 0.1M NaA is
HA2 are 3.14 × 10– 4 and 1.96 × 10– 5 respectively.
The relative strength of the acids will be
approximately (a) 8 (b) 10 (c) 4 (d) 5
(a) 1 : 4 (b) 4 : 1 (c) 1 : 16 (d) 16 : 1
82. How many gms of CaC2O4will dissolve in one
75. At 298K a 0.1 M CH3COOH solution is 1.34 % litre of saturated solution. Ksp of CaC2O4 is
ionized. The ionization constant Ka for acetic acid 2.5 × 10–9 mol2lit–2
will be
(a) 0.0064 g (b) 0.0128 g
(a) 1.82 × 10– 5 (b) 18.2 × 10 –5
–5 (c) 0.0032 g (d) None of these
(c) 0.182 × 10 (d) None of these
EBD_7587
80 Chemistry Objective MCQs
83. Zirconium phosphate [Zr3(PO4)4] dissociates 89. What will be the H+ ion concentration in a
into three zirconium cations of charge + 4 and solution prepared by mixing 50 mL of 0.20 M NaCl,
four phosphate anions of charge – 3. If molar 25 mL of 0.10 M NaOH and 25 mL of 0.30 N HCl?
solubility of zirconium phosphate is denoted by (a) 0.5 M (b) 0.05 M
S and its solubility product by Ksp then which of
(c) 0.02 M (d) 0.10 M
the following relationship between S and Ksp is
correct? 90. The concentration of hydroxyl ion in a solution
left after mixing 100 mL of 0.1 M MgCl 2 and
100 mL of 0.2 M NaOH
-11
(a) S = {Ksp/ (6912)1/7}(b) S = {Ksp/ 144}1/7 (Ksp of Mg (OH ) 2 = 1.2 ´10 ) is
(c) S = {Ksp/ 6912}1/7 (d) S = {Ksp/ 6912}7 (a) 2.8 ´ 10 -4 (b) 2.8 × 10 - 3
84. Solid Ba(NO3)2 is gradually dissolved in a (c) 2.8 ´ 10 -2 (d) 2.8 ´ 10 -5
1.0 × 10 –4 M Na 2CO 3 solution. At which
91. The 0.001M solution of Mg(NO3)2 is adjusted to
concentration of Ba 2+, precipitate of BaCO3
pH 9, Ksp of Mg(OH)2 is 8.9 × 10–12. At this pH
begins to form ? (Ksp for BaCO3 = 5.1 × 10–9)
(a) 5.1 × 10–5 M (b) 7.1 × 10–8 M (a) Mg(OH)2 will be precipitated
(c) 4.1 × 10–5 M (d) 8.1 × 10–7 M (b) Mg(OH)2 is not precipitated
85. A solution of NH 4Cl and NH 3 has pH = 8.0. (c) Mg(OH)3 will be precipitated
Which of the following hydroxides may be (d) Mg(OH)3 is not precipitated
precipitated when this solution is mixed with equal 92. The solubility (in mol L–1) of AgCl
volume of 0.2 M of metal ion. (Ksp = 1.0 × 10–10) in a 0.1 M KCl solution will be
(a) Ba(OH)2 (K sp = 1.1 ´ 10-4 ) (a) 1.0 × 10–9 (b) 1.0 × 10–10
(b) Mg(OH)2 (Ksp = 3.5 ´ 10-4 ) (c) 1.0 × 10 –5 (b) 1.0 × 10–11
(c) Fe(OH) 2 (K sp = 8.1´ 10-16 ) 93. A solution is saturated with respect to
-5
(d) Ca (OH ) 2 (K sp = 2.1 ´ 10 ). SrCO3 and SrF2. The [CO32–] was found to be
1.2 × 10–3 M. The concentration of F– in the
86. On addition of increasing amount of AgNO3 to
0.1 M each of NaCl and NaBr in a solution, what solution would be
% of Br - ion get precipitated when Cl - ion Given Ksp of SrCO3 = 7.0 ´ 10–10 M2,
starts precipitating. Ksp (AgCl) = 1 .0 ´ 10 - 10 , Ksp of SrF2 = 7.9 ´ 10–10 M3,
Ksp (AgBr) = 1 ´ 10 -13 (a) 1.3 × 10–3 M (b) 2.6 × 10–2 M
(a) 0.11 (b) 99.9 (c) 0.01 (d) 9.99
(c) 3.7 × 10–2 M (d) 5.8 × 10–7 M
87. The percentage hydrolysis of 0.15 M solution
94. If pKb for fluoride ion at 25°C is 10.83, the
of ammonium acetate, Ka for CH3COOH is
ionisation constant of hydrofluoric acid in water
1.8 × 10–5 and Kb for NH3 is 1.8 × 10–5
at this temperature is
(a) 0.55 (b) 4.72 (c) 9.38 (d) 5.56
(a) 3.52 × 10 -3 (b) 6.75 × 10 -4
88. Which one of the following arrangements -2 -5
(c) 5.38 × 10 (d) 1.74 × 10
represents the correct order of solubilities of
sparingly soluble salts Hg2Cl2, Cr 2(SO4)3, BaSO4 95. If Ksp (PbSO4) = 1.8 × 10–8 and Ka (HSO -4 )
and CrCl3 respectively ? = 1.0 × 10–2 the equilibrium constant for the
reaction.
(a) BaSO4 > Hg 2 Cl2 > Cr2 (SO 4 )3 > CrCl3
(b) BaSO4 > Hg 2 Cl2 > CrCl3 > Cr2 (SO4 )3 PbSO 4 (s) + H + (aq) ƒ HSO -4 (aq) + Pb 2+ (aq) is
(c) BaSO4 > CrCl3 > Hg 2Cl 2 > Cr2 (SO4 )3 (a) 1.8 × 10–6 (b) 1.8 × 10–10
(d) Hg 2Cl 2 > BaSO4 > CrCl3 > Cr2 (SO4 )3 (c) 2.8 × 10–10 (d) 1.0 × 10–2
Equilibrium 81
96. The dissociation constants of a weak acid HA if the degree of dissociation is a at equilibrium
and weak base BOH are 2 × 10–5 and 5 × 10–6 pressure P, then the equilibrium constant for the
respectively. The equilibrium constant for the reaction is
neutralisation reaction of the two is
a2 a 2P 2
(a) Kp = (b) Kp =
1 + a 2P 1 - a2
(a) 1.0 × 104 (b) 1.0 × 10–4
(c) 1.0 × 10–10 (d) 2.5 × 10–1 P2 a 2P
(c) Kp = (d) Kp =
97. Solid AgNO3 is slowly added to a solution 1 - a2 1 - a2
containing each of 0.01 M NaCl and 0.001 M NaBr.
What will be the concentration of Cl– ions in 100. The equilibrium constant for a reaction is K, and
solution when AgBr will just start to precipitate ? the reaction quotient is Q. For a particular
Ksp (AgBr) = 3.6 × 10–13, Ksp (AgCl) = 1.8 × 10–10. K
reaction mixture, the ratio is 0.33. This means
Q
that :
(a) 1.8 × 10–7 (b) 3.6 × 10–10 (a) The reaction mixture will equilibrate to form
more reactant species
(c) 0.01 (d) 2 × 10–4
(b) the reaction mixture will equilibrate to form
98. What is the unit of KP for the reaction?
more product species
ˆˆ† CH 4 (g) + 2H 2S(g)
CS2 (g) + 4H 2 (g) ‡ˆˆ
(c) the equilibrium ratio of reactant to product
(a) atm (b) atm–2 (c) atm2 (d) atm–1 concentrations will be 3
99. In the dissociation of PCl5 as (d) the equilibrium ratio of reactant to product
PCl5 (g) ƒ PCl3 (g) + Cl2 (g) concentrations will be 0.33

Answer KEY
1 (c) 11 (c) 21 (c) 31 (a) 41 (c) 51 (b) 61 (d) 71 (d) 81 (c) 91 (b)
2 (c) 12 (d) 22 (c) 32 (d) 42 (d) 52 (a) 62 (a) 72 (d) 82 (a) 92 (a)
3 (b) 13 (b) 23 (b) 33 (b) 43 (b) 53 (b) 63 (c) 73 (d) 83 (c) 93 (c)
4 (c) 14 (b) 24 (c) 34 (b) 44 (c) 54 (b) 64 (a) 74 (b) 84 (a) 94 (b)
5 (d) 15 (a) 25 (d) 35 (c) 45 (d) 55 (d) 65 (d) 75 (a) 85 (c) 95 (a)
6 (a) 16 (c) 26 (c) 36 (d) 46 (a) 56 (d) 66 (a) 76 (d) 86 (b) 96 (a)
7 (b) 17 (a) 27 (a) 37 (b) 47 (b) 57 (c) 67 (a) 77 (a) 87 (a) 97 (c)
8 (d) 18 (c) 28 (a) 38 (c) 48 (d) 58 (a) 68 (b) 78 (c) 88 (b) 98 (b)
9 (a) 19 (a) 29 (d) 39 (d) 49 (c) 59 (a) 69 (c) 79 (a) 89 (b) 99 (d)
10 (b) 20 (d) 30 (b) 40 (a) 50 (b) 60 (b) 70 (d) 80 (b) 90 (a) 100 (a)
EBD_7587
82 Chemistry Objective MCQs

1. (c) -7
[ HX ] = 1.2 ´ 1.2 ´ 10
10-5
W + X ˆˆ†
ˆ
‡ˆˆ
ˆ Y + Z
= 1.2 × 10–1
Initial 1 1 0 0
= 12 × 10–2 M
At eqn. 1- a 1- a a a
1 K
5. (d) K = K 2 ´ = 2 .
a2 K1 K1
Keq = 9 =
(1 - a)2 6. ˆˆ† 2I–
(a) I2 ‡ˆˆ
\ a = 0.75 1–x 2x
Moles of Y = 0.75
(2x ) 2
2. (c) Ptotal = 3P Kc = = 10 -6
(1 - x)
0.318
Þ P= = 0.106
3 Soln. shows that (1 – x) > 2x
\ Kp = 4P3 = 4.76 × 10–3
\[I2 (g)] > [I - (g)]
3. (b)
7. (b) A+ B C + D
ˆˆ†
ˆ
‡ˆˆ
ˆ 2SO2
2SO3 + O2 Initial 0.9 0.9 0 0
Initial 1 0 0 At eqm. 0.3 0.3 0.6 0.6
At equilibrium 1 - 2a 2a a
0.6 ´ 0.6
Kc = =4
\ 2a = 0.6, \ a = 0.3 0.3 ´ 0.3
[SO3] = 1 – 2a = 1 – 0.6 = 0.4
p CH 3OH 2
[SO2] = 2a = 0.6 8. (d) Kp = = = 200 ;
p CO ´ p H 2 1 ´ (0.1) 2
[O2] = a = 0.3
2 For reverse reaction
a ´ ( 2a ) 0.3 ´ 0.6 ´ 0.6
K eq = 2
= = 0.675
(1 - 2a ) 0.4 ´ 0.4 1 1
= = 5 ´10 -3 atm 2
K p 200
4. (c)
2HX(g) ƒ H 2 (g) + X 2 (g) 9. ˆˆ† H + I .It is 22% decomposed ,
(a) 2HI ‡ˆˆ 2 2
At eqm. 1.2 ´ 10-3 M 1.2 ´ 10 -4 M
3.20 ´ 22
[H2 ][X2 ] \ = 0.704
K eq = 100
[HX ]2
(3.2–0.704) is equal to HI present at
-5 1.2 ´ 10-3 ´ 1.2 ´ 10-4 equilibrium which is = 2.496
10 =
[ HX ]2
Equilibrium 83

10. (b) At equilibrium the moles of Cl 2 must be 2a


p NO2 = ´P
= 0.15 × 3 = 0.45 (1 + a)

ˆˆ† PCl + Cl
PCl 5 ‡ˆˆ 2
3 2 æ 2a ö
´ P÷
( p NO2 )2 çè (1 + a ) ø = 4a P
2

x - 0.45 0.45 0.45 KP = =


p N 2O4 æ1– a ö 1 – a2
Eqm. Conc.
3 3 3 ç ÷´ P
è1+ a ø

éPCl3 ùû [Cl 2 ] Given, KP = 2, P = 0.5 atm


Kc = ë
ëéPCl5 ûù 4a 2 P
\ KP =
1 – a2
0.15´ 0.15
\ 0.04 = 4a 2 ´ 0.5
(x - 0.45) / 3 =
1 – a2
\ x = 2.1 moles
a = 0.707 » 0.71
11. (c) N 2 O4 (g) ƒ 2NO2 (g)
\ Percentage dissociation
t=0 1 0
= 0.71 × 100 = 71
t = eq. 1–a 2a
13. (b) The efflorescent salts loss water to
Where a = Degree of dissociation.
atmosphere
Mol. wt. of mixture
K p = p2 = 1.086 ´ 10-4 ;
(1 - a ) ´ M N2O4 + 2a ´ M NO2 H 2O
=
(1 + a )
p H 2O = 1.042 ´ 10 -2 atm = 7.92 mm

(1 - 0.2 ) 92 + 2 ´ 0.2 ´ 46 If H2O pressure at 25°C is less than 7.92 mm.


= = 76.66
(1 + 0.2 ) The reaction
CuSO 4 .5H 2O (s) ¾
¾®
Now, As per ideal gas equation
PV = nRT CuSO 4 .3H 2 O (s) + H2O(g)
PMmixture = dRT will not proceed in RHS.
PM mix 1 ´ 76.66 28
\d = = = 3.11g / L
14. (b) Moles of N 2 = = 1,
RT 0.0821 ´ 300 28
12. (d) ˆˆ† 2NO 2 (g)
N 2 O 4 (g) ‡ˆˆ
6
Initial moles 1 0 Moles of H 2 = =3
2
Moles at equil. (1 – a) 2a
(a = degree of dissociation) 500 ´ 1
Moles of H 2SO 4 required = = 0.5
Total number of moles at equil. 1000
= (1 – a) + 2a Moles of NH 3 neutralised by H2SO4 = 1.0
= (1 + a)

p N 2O 4 =
(1 – a )
´P
(2NH 3 + H 2SO 4 ¾¾
® (NH 4 ) 2 SO 4 )
(1 + a )
EBD_7587
84 Chemistry Objective MCQs

Hence 1 mol of NH 3 by the reaction 19. (a) Equilibrium shifted in the backward
direction. (Le Chatelier's principle)
between N2 and H2.
20. (d) To calculate the value of K4 in the given
ˆˆ† 2 NH
N 2 + 3H 2 ‡ˆˆ equation we should apply :
3
eqn. (2) + eqn.(3) × 3 – eqn. (1)
initial 1 3 0
at eqm 1 – 0.5 3 – 0.5 × 3 1 K 2 K33
hence K4 =
K1
1´ 1
Kc = = 0.592
0.5 ´ (1.5) 3 21. (c) Since reaction is exothermic hence low
temperature will favour forward reaction
PM 1 ´ 82.1 also volume is decreased by applying high
15. (a) D = = = 2.5 g L-1
RT 0.0821´ 400 pressure.
22. (c) By addition of SO2 equilibrium will shift to
d = 2.0 g L-1 (given) RHS which is exothermic. Hence temperature
D-d 2.5 - 2 will increase.
a= = = 0.125 = 12.5%
d(n - 1) 2(3 –1) 23. (b) A + 2B 2C + D
a 1.5a 0 0
(a – x) (1.5a – 2x) 2x x
[N 2 ]1/2 [H 2 ]3/2
16. (c) If, K c = =4
[NH3 ]1 (2 x) 2 ´ x
Hence KC =
(a - x ) (1.5a - 2 x) 2
Given, at equilibrium
[NH3 ]2 = 4 -2 = 1 \ (a – x) = (1.5a – 2x)
then, ( )
[N 2 ][H 2 ]3 16 \ a = 2x
On solving KC = 4
Dn g
also K P = K c . ( RT ) 24. (c) For the reaction
ˆˆ† 2BaO( s ) + O 2 (g);
2BaO 2 ( s ) ‡ˆˆ
2
1 -2 æ 1 ö DH = + ve.
= ´ (800R ) = ç ÷
16 è 4 ´ 800R ø At equilibrium K p = PO 2
Hence, the value of equilibrium constant
17. (a) ˆˆ† 1 H 2 + 1 I 2
HI ‡ˆˆ depends only upon partial pressure of O2.
2 2 Further on increasing temperature formation
at t = 0 0 0 of O2 increases as this is an endothermic
at eq. (1 – 0.5) (0.5) (0.5) reaction. Hence, pressure of O2 is dependent
on temperature.
æ 50 ö
çèQ a = = 0.5÷
ø P
100 25. (d) PNH 3 = PH 2S = atm
2
(0.5)1/2 (0.5)1/2
Now KC = =1 2
0.5 æ Pö P2
K p = PNH3 PH 2S = ç ÷ =
18. (c) Addition of inert gas at constant volume è 2ø 4
has no effect on equilibrium.
Equilibrium 85

2
\ from equation (i), (ii) and (iii)
æpö 1 1
ΔG = -RT ln K p = -RT ln ç ÷ K1 = =
è2ø K 2 ( K3 )2
31. (a) When ionic product i.e. the product of
= -2RT [ln p - ln 2]
the concentration of ions in the solution
26. (c) ˆˆ† 2HI(g) exceeds the value of solubility product,
H 2 (g) + I2 (g) ‡ˆˆ
formation of precitpiate occurs.
Kp = Kc (RT) Dn ;
ˆˆ† Ca 2+ + 2F–
CaF2 ‡ˆˆ
Dn = 2 - 2 = 0; \ K p = K c
Ionic product = [Ca2+] [F–]2
27. (a) Since T2 has similar chemical properties as when, [Ca2+] = 1× 10–2 M
H2 so upon mixing, NH2T and NHT2 are [F–]2 = (1× 10–3)2 M
formed. = 1× 10–6 M
Dn g \ [Ca2+] [F–]2 = (1×10–2) (1×10–6) = 1×10–8
28. (a) K P = K C ( RT )
In this case,
For the reaction
Ionic product (1× 10–8) >
1 solubility product (1.7× 10–10)
CO(g) + O2 ( g ) CO2(g)
2 32. (d) PbI2 Pb++ + 2I–
s 2s
æ 1ö 1
Dng = 1 - ç1 + ÷ = - Ksp = s × (2s)2 = 4s3
è 2ø 2
3
æ 0.7 ö
= 4´ç = 14.0 ´ 10 –9
\ KP =
KC
;
KP
=
1 è 461.2 ø÷
RT KC RT
33. (b) Van’t Hoff reaction isotherm is DG = DGº+RT
29. (d) The condition for the equilibrium is
ln Qp
( DG) T,P = 0 and ( DS) U,V = 0 When the reaction is in a state of equilibrium
DG = 0
30. (b) (I) ˆˆˆ
K1
N 2 + 2O 2 ‡ˆˆ †
ˆ 2NO 2 Then DGº = – RT ln Qp = – RT ln Kp
[NO 2 ]2 34. (b) The reaction mixtures starting either with
K1 = ...(i)
[N 2 ][O 2 ]2 H2 or D2 reach equilibrium with the same
composition, except that D2 and ND3 are
(II) ˆˆˆ†
K2
2NO 2 ‡ˆˆˆ N 2 + 2O 2
present instead of H2 and NH3.
[N 2 ] [O2 ]2 35. (c) NaZ is salt of WA/SB
K2 = ...(ii)
[NO 2 ]2
1
1
\ pH = ( pK w + pK a + log C )
ˆˆˆ
K3
† N2 + O2 2
(III) NO2 ‡ˆˆˆ
2 8.9 × 2 = 14 + pKa + log 0.1
[N ]1/2 [O2 ] 17.8 = 14 + pKa – 1
K3 = 2
[NO 2 ] pKa = 4.8,
[N 2 ][O 2 ]2 Ka = Antilog (– 4.8)
\ ( K3 )2 = ...(ii) = 1.585 × 10–5 » 1.6 × 10–5.
[NO2 ]2
EBD_7587
86 Chemistry Objective MCQs

36. (d) HCl is stronger acid than CH3COOH and 42. (d) [Cu(H2O)4]2+ + 4NH3 [Cu(NH3)4]2+
+
Cl– is a stronger base than CH 3COOH 2 + 4H2O involves lose and gain of electrons.
and is the conjugate base of HCl. H2O is coordinated to Cu by donating
HCl + CH COOH ‡ˆˆ ˆˆ† electrons (LHS). It is then removed by
3
acid1 base2 withdrawing electrons.
+
Cl– + CH 3COOH 2 43. (b) [H+] = C ´ K a = 0.001 ´ 1.8 ´ 10 -4 for
base1 acid2 formic acid
37. (b) Milliequivalents of NaOH = 10 × 0.1 = 1
Milliequivalents of H2SO4 = 10 × 0.05 = 0.5 [H+] = C2 ´ 1.8 ´10-5 for acetic acid
2NaOH + H 2SO4 ¾¾
® Na 2SO4 + H 2 O Equating and solving, C2 = 0.01 M.
2 equ. 1 equ.
44. (c) ˆˆ† 2SO3
O 2 + 2SO 2 ‡ˆˆ
\ 1 equivalent of NaOH reacts with 0.5 eq.
of H2SO4 to give neutral (pH = 7) solution. at t = 0 1 2 0
38. (c) 2 AB3 ( g ) A2 ( g ) + 3B2 ( g ) at equil. (1 – a) 2(1 – a) 2a
Given at equilibrium
at t = 0 8 0 0
2a = 1.6
at eq. (8 – 2 × 2) 2 3×2
a = 0.8
= 4 2 6 (2a )2 (0.8)2
now KC = =
(1 – a )(2 – 2a )2 (1 – 0.8) (1 – 0.8)2

[ A2 ][ B2 ]3 2 / 1 ´ [6 / 1]3 0.64
KC = = = 27 =
0.008
[ AB3 ]2 [4 / 1]2
KC = 80
39. (d) Sodium acetate is a salt of strong base and 45. (d) Volume of ice is greater than that of water.
weak acid. The direction in which the reaction will
1 1 proceed can be predicted by applying
\ pH = 7 + pK a + log c
2 2 Le-Chatelier's principle
where pka = - log K a = – log 10–5 = 5 1
Pressure µ
Volume
log c = log10–1 = – 1 So equilibrium, will shift forward.
5 1 46. (a) Follow Lowry Bronsted concept for
pH = 7 + - = 9.0
2 2 conjugate pair. Acid strength follows the
40. (a) [H+] = Ca = 0.2 × 0.40 = 0.08 N. order H2O > C2H2> NH3> C2H6 conjugate
base will follow the order
40
(a = = 0.40 ) C 2 H 5- > NH 2- > C 2 H- > OH-
100
41. (c) HSO4– accepts a proton to form H2SO4. 47. (b) CO2 + 2H2O H3O+ + HCO3– ;
Thus H2SO4 is the conjugate acid of HSO4–.
[H3 O + ][HCO3- ]
Kc = ;
+ H+
HSO 4- ¾¾¾® H 2SO4 [CO 2 ]
base conjugate acid [HCO 3- ] 3.8 ´ 10 -7
of HSO-4 = = 3.8 ´10 -1
[CO 2 ] -6
10
Equilibrium 87

48. (d) ˆˆ† H + + HA -


H 2 A ‡ˆˆ \ [H + ] = 4.0 ´10 -8
+ -
\ K = 1.0 × 10–5 = [H ][HA ] (Given) [HCO3- ] 4.5 ´10-7
1
[H 2 A] \ = = 11
[CO 2 ] 4 ´ 10-8
HA - ¾¾
® H + + A2- 53. (b) ˆˆ† H 2 CO3
CO 2 + H 2 O ‡ˆˆ
ˆˆ† H + + HCO3-
‡ˆˆ
-10 [H + ][A2 - ]
\ K 2 = 5.0 ´10 = (Given) If CO2 escapes, the equilibrium will shift to
[HA - ] LHS and [H+] concentration will decrease

[H + ]2 [A 2 - ]
54. (b) HCO 3- ® H + + CO 32-
K= = K1 ´ K2
[H 2 A]
[H + ][CO32- ]
K2 = = 4.8 ´10-11
= (1.0 × 10–5) × (5 × 10–10) = 5 × 10–15 -
[HCO3 ]
49. (c) Higher the value of Ka lower will be the value
4.8 ´ 10-11[HCO3- ]
of pKa i.e. higher will be the acidic nature. [H + ] =
Further CN–, F– and NO2– are conjugate base [CO32- ]
of the acids HCN, HF and HNO2 respectively = 4.8 ´10-11 (0.1 / 0.2)
hence the correct order of base strength will
be pH = – log [H+]
= – log (4.8 × 10–11 × 0.5) =10.62
F– < NO2– < CN–
55. (d) Given at 330 K
(Q stronger the acid weaker will be its
conjugate base) Kw = 10–13.6
50. (b) The more is the value of equilibrium i.e. pKw = pH + pOH
constant, the more is the completion of Q pOH = – log [OH–]
reaction or more is the concentration of 13.6 = pH + pOH
products i.e. the order of relative strength pOH= – log 10–4
would be pOH= 4
OH– > CN– > H2O \ pH = 13.6 – 4 = 9.6
51. (b) CH3COOH is weak acid while NaOH is 56. (d) On adding inert gas at constant volume the
strong base, so one equivalent of NaOH total pressure of the system is increased,
can not be neutralized with one equivalent but the partial pressure of each reactant
of CH3COOH. Hence the solution of one and product remains the same. Hence no
equivalent of each does not have pH value effect on the state of equilibrium.
as 7. Its pH will be towards basic side as
NaOH is a strong base hence conc. of OH– 57. (c) a1 = 0.005 = K a ´ C1
will be more than the conc. of H+. Molarity of diluted solution; 2 × 1 = 32 × M,
52. (a) CO 2 with H 2 O forms H 2 CO 3
1
M= (C 2 )
ˆˆ† H + + HCO 3- 16
CO 2 + H 2 O ‡ˆˆ
[H + ][HCO3- ] Ka
K1 = = 4.5 ´10-7 a2 = = 0.005 16 = 0.02
[CO 2 ] C2
Again pH = - log[H + ] = 7.4 + 0.02
H3 O = C 2 a 2 = = 1.25 ´ 10 -3 M
16
EBD_7587
88 Chemistry Objective MCQs

58. (a)
Be(OH)2 Mg(OH) 2
NaOH ˆˆ† CH 3COONa
+ CH 3COOH ‡ˆˆ + H 2O Amphoteric Weak base

20 × 0.2 50 × 0.2 0 0
Ca(OH)2 Sr(OH)2 Ba(OH)2
4 10 0 0 Strong base
0 6 4 4
Millimoles Hence, Be(OH)2 will have lowest pH.
Conc. =
Total Volume 65. (d) NaCN + H2O NaOH + HCN
6 4 (hydrolysis)
[CH 3 COOH] = ;[CH 3COONa] =
70 70 NaOH will give OH– ions and react with H+ to
4 / 70 form H2O hence decrease the concentration
pH = – log 1.8 ´ 10 -5 + log , pH = 4.56
6 / 70 of H+ ions.
59. (a) Ksp of As2S3 is less than ZnS. In acidic
medium ionisation of H2S is suppresed Kw 10-14
66. (a) a= = = 2.48%.
(common ion effect) and Ksp of ZnS does Ka ´ c 1
1.3 ´10 -9 ´
not exceed. 80

60. (b) N 3H ˆˆ† N3- + H +


‡ˆˆ 67. (a) The pH of mixture is due to the HCl only,
Hydrazoic acid
because CH3COOH is negligibly ionised due

i.e, conjugate base of hydrazoic acid is N3 . to common ion effect. Thus,
61. (d) Given [OH– ] = 5 × 10–2 [H+] = 0.05 M = 5 × 10–2 M
\ pOH = – log 5 × 10–2 pH = – 1og (5 × 10–2) = – [log 5 + log 10–2]
= – log 5 + 2 log 10 = 1.30 pH = 1.3
Q pH + pOH = 14 -5 salt salt
68. (b) 6 = - log10 + log = 5 + log
Q pH = 14 – pOH acid acid
= 14 – 1.30 = 12.70
62. (a) For ternary electrolyte; solubility salt salt 10
log must be 1. \ = or 10 :1 .
acid acid 1
1 1
æ K sp ö 3 æ 32 ´10 -12 ö3 69. (c) Let s = solubility
-4
= çç ÷÷ = çç ÷ = 2 ´10
è 4 ø è 4 ÷
ø ˆˆ† Ag + + IO-
AgIO3 ‡ˆˆ 3
s s
63. (c) [Ba2+] [SO42 –] Ksp = [Ag+] [IO3–] = s × s = s2
Ksp = (1.0 × 10–4) [SO42 –] = 4 × 10–10 Given Ksp = 1 × 10–8
\[SO42- ] = 4 ´10 -6 M \ s = K sp = 1´10-8
64. (a) Metal halide on hydrolysis with water form
= 1.0 × 10–4 mol/lit = 1.0 × 10–4 × 283 g/lit
corresponding hydroxides.
The basic strength of hydroxide increases (Q Molecular mass of AgIO3 = 283)
as we move down in a group. This is because
1.0 ´10-4 ´ 283 ´100
of the increase in size which results in = g/100mL
decrease of ionization energy which 1000
weakens the strength of M – O bonds in = 2.83 × 10–3 g/100 mL
MOH and thus increases the basic strength.
Equilibrium 89
70. (d) Given Kb = 1.0 × 10–12 2
æ 1.34 ö
[BOH] = 0.01 M [OH]– = ? 75. (a) K a = ca 2 = 0.1 ´ ç = 1.8 ´ 10 -5
è 100 ø÷
ˆˆ† B+ + OH -
BOH ‡ˆˆ 76. (d) Total volume = 100 mL
t=0 c 0 0
t = t eq c (1–α) ca ca 1.0
[acid] = 10 mL ´ = 0.1
100
c2a 2 ca 2 0.5
Kb = = [salt] = 20 mL ´ = 0.1
c(1 - a ) (1 - a ) 100

0.01a 2
[salt ]
pH of acidic buffer = pK a + log acid
Þ 1.0 × 10–12 =
(1 - a )
[ ]
On calculation, we get, a = 1.0 × 10–5 0.1
= 4.76 + log = 4.76
Now, [OH–] = ca = 0.01 × 10–5 0.1
= 1 × 10–7mol L–1 77. (a) The species with the greatest proton
[salt] affinity will be the strongest base, and its
71. (d) pH = pKa + log
[acid] conjugate acid will be the weakest acid. The
weakest acid will have the smallest value of
7.5 Ka. Since HI is a stronger acid than HF which
= 4.76 + log 500 = 4.7 + log 1.5 = 4.87 is a stronger acid than H2S, a partial order
5 of proton affinity is
500
I– < F– < HS–
Hence correct answer is 4.76 < pH < 5.0 Since NH3 is a very weak acid, NH2– must
72. (d) Solutions which resist the change in the be a very strong base. Therefore the correct
value of pH when small amount of acid or order of proton affinity is
base is added to them are known as buffers. I– < F– < HS– < NH2–
73. (d) As, molarity, 78. (c) 2HNO3(aq) + [Ag(NH3)2]+ + Cl–
Wt. of solute per litre of solution
= ® AgCl (s ) ¯ +2NH 4 + 2NO3-
¾¾
Mol. wt. of solute
When nitric acid is added to amine solution,
1000
Molarity of H2O = mol/L solution is made acidic and the complex ion
18
dissociates and liberate silver ion to
H2O H+ + OH– recombine with chloride ion. This is the
c (1 – a) ca ca conformatory test for silver in group 1.
ca 2 79. (a)
Thus, K a = = ca 2 A- + H 2 O ƒ HA + HA
1- a
(Initial) 0.001M 0 0
(At eq.) (0.001- R) (0.001 ´ h) (0.001 ´ h)
1000
= ´ (1.8 ´ 10 -8 ) 2 (0.001 ´ h)(0.001 ´ h)
18 Kh =
= 1.8 × 10–14 (0.001 - h)

or, K h = 0.001 ´ h 2 (as, 0.001 – h » 0.001)


a1 K a1 3.14 ´ 10 -4
74. (b) = = = 4 :1
a2 K a2 1.96 ´ 10-5 10 -9 = 0.001 ´ h 2
EBD_7587
90 Chemistry Objective MCQs

æ K 10-14 -
ö 85. (c) pH = 8, pOH = 6; [OH - ] = 10 -6 M;
ç K h = w = -5 = 10 9 ÷
ç Ka 10 ÷
è ø -6 2
Ionic product of Fe(OH)2 = 0.2 × (1´ 10 )
h2 = 10–6 ; \ h = 10 -3
= 2 ´ 10 -13 > K sp ( = 8.1 ´ 10 -16 )
80. (b) The dissociation of a weak electrolyte
(AgCl) is suppressed on adding the strong 86. (b) To precipitate the AgCl
electrolyte having a common ion (Cl –),
hence concentration of Ag+ ion will be less. [Ag + ] required
81. (c) Hydrolysis of a salt is reverse reaction of
acid-base neutralization reaction. K sp (AgCl) 1.0 ´10-10
= = = 1.0 ´10-9 M
[Cl - ] 0.1
K w 10-14
\ Kh = = -7 = 10–7
Ka 10 K sp (AgBr)
[Br - ] left at this stage =
Kh [Ag + ]
[OH - ] = ch = c ´ = c ´ Kh
c
1.0 ´ 10 –13
= 10-8 = 10-4 = = 1.0 × 10–4 M
1.0 ´ 10-9
Þ pOH– = –log [OH–]
= – log [10–4] = 4 % of remaining
2+
ˆˆ† Ca + C2O 42– ; Ksp = S2 1.0 ´ 10 -4
82. (a) CaC2 O4 ‡ˆˆ [ Br - ] = ´100 = 0.1
0.1
\ S = K sp = 2.5 ´ 10-9 = 5 ´ 10-5 mol/litre
% of Br - to be precipitated
= 100 – 0.1 = 99.9
solubility = 5 ´10 -5 ´128 = 0.0064 g
Kw
83. (c) é Zr3 ( PO 4 ) ù ‡ˆˆ
ˆˆ† 3Zr 4+ + 4PO 43- 87. (a) a=
ë 4û 3S Ka ´ Kb
4S
Ksp = (3S)3 (4S)4
= 27S 3 × 256 S 4 1´ 10-14
= 6912 S 7. = = 0.55%
1.8 ´10-5 ´ 1.8 ´ 10-5
1/7
æ Ksp ö
\ S = çç ÷÷ 88. (b) Cr2(SO4)3 2Cr 3+ + 3SO2–
è 6912 ø 2s
4
3s
84. (a) Given Na2CO3 = 1.0 × 10–4 M
\ [CO32– ] = 1.0 × 10–4 M Ksp = (2s)2 (3s)3 = 4s2 × 27s3 = 108s5
i.e. s = 1.0 × 10–4 M 1/ 5
æ K sp ö
At equilibrium s= ç ÷
[Ba2+] [CO32–] = Ksp of BaCO3 è 108 ø

K sp 5.1 ´ 10 -9 Hg2Cl2 2Hg 2+ + 2Cl-


[Ba2+] = = 2s 2s
[CO23 - ] 1.0 ´ 10 -4
Ksp = (2s)2 × (2s)2 = 16s4
= 5.1 × 10–5 M
Equilibrium 91

1/ 4
92. (a) Let solubility of AgCl = x mole/L
æK ö
s = ç sp ÷ ˆˆ† Ag + + Cl –
AgCl ‡ˆˆ
è 16 ø
i.e., Ksp(AgCl) = x× x
BaSO4 Ba 2+ + SO 2–
4
s s ® K + + Cl –
KCl ¾¾
Ksp = s2
0.1
s= K sp [Cl–] from KCl = 0.1 m
Total [Cl–] in solution = x + 0.1
CrCl3 Cr3+ + 3Cl -
s 3s Ksp(AgCl) = [Ag+] [Cl–] = x (x + 0.1)
Ksp = s × (3s)3 = 27 s4 1.0 × 10–10 = x(x + 0.1)
1.0 × 10–10 = x2 + 0.1x
1/ 4
s = æç sp ö÷
K 1.0 × 10–10 = 0.1x (as x2 << 1)
è 27 ø x = 1.0 × 10–9 mol/L
Hence the correct order of solubilities of 93. (c) The two Ksp values do not differ very much.
salts is So it is a case of simultaneous equilibria,
where the concentration of any species can
1/ 4 1/ 4 1/ 5
æ K sp ö æ Ksp ö æ K sp ö not be neglected.
K sp >ç ÷ >ç ÷ >ç ÷
è 16 ø è 27 ø è 108 ø
2
éSr 2+ ù éF- ù Ksp
(25 ´ 0.3 - 25 ´ 0.1) ë ûë û = SrF
[H + ] =
2
89. (b) meq = 0.05M + -
100 éSr ù é CO3 ù K sp
2 2
ë ûë û SrCO
2
90. (a) MgCl 2 + 2NaOH ® Mg(OH) 2 + 2NaCl
100 × 0.1 100 × 0.2 0 0 Initial moles 7.9 ´ 10-10
= = 1.128
0 0 10 2 0 After mixing 7.0 ´ 10-10

é 10 ù é 20 ù -4
2 \ [F- ]2 = 1.128 × 1.2 × 10–3 = 13.5 × 10–4
\ Ksp = ê ú ê 200 ú = 5 ´10 .
ë 200 ûë û
\ [F - ] = (13.5 × 10–4)1/2
But actually Ksp of = 3.674 × 10–2 » 3.7 × 10–2 M.
Mg (OH ) 2 = 1.2 ´ 10 - 11 94. (b) K w = Ka ´ K b
\ 4S3 = 1.2 ´ 10 -11 Kb = 10–10.83 = 1.48 × 10–11
Find S then [OH–] = 2S which is 2.8 ´ 10–4. Kw 10-14 -4
\ Ka = = = 6.75 ´ 10
91. (b) pH = 9 ; [H+ ] = 10-9 ;[OH- ] = 10-5 ; K b 1.48 ´ 10-11

[Mg 2 + ] = 1´10-3 ; 95. (a) PbSO 4 (s ) ƒ


2+ - 2 -13
[Mg ][OH ] = 1 ´10
Pb2+ (aq) + SO42- (aq) (Ksp ) ...(i)
-12
given Ksp of Mg (OH ) 2 = 8.9 ´ 10
HSO 4– (aq) ƒ
which is more than 1 ´ 10 - 13 .
Hence Mg(OH)2 will not precipitate H + (aq) + SO 42 - (aq) (K a ) ...(ii)
EBD_7587
92 Chemistry Objective MCQs
Dn g 3- 5
Subtracting equation (ii) from (i), then 98. (b) K P ¾¾ ® (atm ) = (atm )
+ -2
PbSO4 (s) + H (aq) ƒ = ( atm )
Pb2+ (aq) + HSO-4 (aq) (K eq)
Ksp 99. (d)
\ K eq = PCl5 ƒ PCl3 + Cl2
Ka
Initial 1 0 0
1.8 ´ 10 -8 At eqn. 1- a a a
= = 1.8 ´ 10-6
1.0 ´ 10 -2 1- a a a
Partial = .P .P .P
96. ˆˆ† BA + H 2 O
(a) HA + BOH ‡ˆˆ pressure 1 + a 1+ a 1+ a

Kw Total mole = 1 – a + a + a = 1 + a
\ KH = .
K a ´ Kb
a a
Ka ´ Kb P. .P 2
KP = 1 + a 1 + a = a P
The inverse of K H is Kc = K
w 1- a 1 - a2
P
1+ a
2 ´ 10 -5 ´ 5 ´ 10 -6
= = 1.0 × 10 4
1 ´ 10 -14 K
100. (a) Q = 0.33
97. (c) Ksp (AgBr) < Ksp (AgCl) Q

Therefore, AgBr will precipitate first and at ÞK<Q


that time all Cl– will be present. i.e., reaction moves in backward direction
to achieve equilibrium.
8 Redox Reactions
1. Which of the following represents a redox (a) I2 will be reduced to I–
reaction? (b) There will be no redox reaction
(a) NaOH
NaOH + HClHCl ¾¾ ® NaCl + H2OO (c) I– will be oxidised to I2
NaOH
(b) BaCl 2 + +
H2
HCl
SO4
¾¾ ® NaCl
BaSO+ H O
4 + 2HCl (d) Fe2+ will be oxidised to Fe3+
NaOH
(c) CuSO + HCl ¾¾
4 + 2H2O
® NaCl +H
Cu(OH) O
2 + H2SO4 7. Which of the following is correct code for x
Zn ++2HCl
(d)NaOH HCl ¾¾ ® NaCl + H
ZnCl2 + H2 O and y in the following reaction.
2. Which reaction involves neither oxidation nor x
reduction?
(a) CrO24, ¾¾ 2Na(s) + S(s) ¾¾
® Na2S(s)
↑ Cr2 O 72,
(b) Cr ¾¾ ↑ CrCl3 y
(c) Na ¾¾ ↑ Na∗ (i) x = oxidation reaction, y = reduction
reaction
(d) 2S2 O 32, ¾¾↑ S4 O 62,
(ii) x = gain of two electrons, y = loss of two
3. Zn gives H2 gas with H2SO4 and HCl but not electrons,
with HNO3 because (iii) x = reduction reaction, y = oxidation
reaction
(a) Zn acts as an oxidising agent when it reacts (iv) x = loss of two electrons, y = gain of two
with HNO3 electrons
(b) HNO3 is weaker acid than H2SO4 and HCl (a) (i) and (ii) (b) (i) and (iv)
(c) In electrochemical series, Zn is above
(c) (ii) and (iii) (d) (iii) and (iv)
hydrogen
8. One mole of N2H4 loses 10 moles of electrons to
(d) NO3- is reduced in preference to hydronium
form a new compound, y. Assuming that all
ion nitrogen appear in the new compound, what is
4. A compound of Xe and F is found to have 53.5% the oxidation state of nitrogen in y (There is no
of Xe. What is oxidation number of Xe in this change in the oxidation state of hydrogen )
compound?
(a) –4 (b) 0 (c) +4 (d) +6
5. Which of the following is not a (a) –1 (b) –3 (c) +3 (d) +5
disproportionation reaction?
9. The oxidation state of nitrogen is correctly given
Al(OEt )
3 ® PhCOOCH Ph for
(a) 2PhCHO ¾¾¾¾¾ 2
– Compound Oxidation state
(b) CHO CH2OH COO
+ OH –¾¾
® + (a) éë Co ( NH3 )5 Cl ùû Cl 2 0
COOH COO– COO –
(b) NH2OH –2
(c) NaH + H 2 O ¾¾
® NaOH + H 2 (c) (N2H5)2SO4 +2
(d) Mg3N2 –3
(d) All of the above. 10. The equivalent mass of oxidising agent in the
6. A solution contains Fe2+, Fe3+ and I– ions. This following reaction is
solution was treated with iodine at 35°C. E° for
Fe3+ / Fe2+ is + 0.77 V and E° for I2/2I– = 0.536 V. SO2 + 2H2S ¾¾® 3S + 2H 2O
The favourable redox reaction is : (a) 32 (b) 64 (c) 16 (d) 8
EBD_7587
94 Chemistry Objective MCQs
11. Amongst the following, identify the species with Which is the strongest reducing agent?
an atom in + 6 oxidation state: (a) Zn (s) (b) Cr (s)
(a) [MnO4]– (b) [Cr(CN)6]3– (c) H2(g) (d) Fe3+ (aq)
(c) Cr2O3 (d) CrO2Cl2 18. The correct decreasing order of oxidation number
12. Oxidation state of sulphur in anions of oxygen in compounds BaO2, O3, KO2 and OF2
SO32 - , S2 O42 - and S2 O62- increases in the is :
orders: (a) BaO2 > KO2 > O3 > OF2
(a) S2 O26 - < S2 O24 - < SO32 - (b) OF2 > O3 > KO2 > BaO2
(c) KO2 > OF2 > O3 > BaO2
(b) SO26 - < S2 O42 - < S2 O62 -
(d) BaO2 > O3 > OF2 > KO2
(c) S2 O42 - < SO32 - < S2 O62 - 19. In the balanced chemical reaction
(d) S2 O24 - < S2 O26 - < SO32 - IO3- + aI - + bH - ¾¾ ® cH 2 O + dI 2
a, b, c and d, respectively, correspond to
13. In the reaction 3Mg + N2 ® Mg3N2
(a) magnesium is reduced (a) 5, 6, 3, 3 (b) 5, 3, 6, 3
(b) magnesium is oxidized (c) 3, 5, 3, 6 (d) 5, 6, 5, 5
(c) nitrogen is oxidized 20. In the reaction shown below, oxidation state of
(d) None of these the carbon in reactant and product are (i) and
14. One gas bleaches the colour of flowers by (ii) respectively? Is the given reaction a redox
reduction, while the other by oxidation reaction?
(a) CO and Cl2 (b) SO2 and Cl2 Na2CO3(aq) + HCl (aq) ¾¾ ®
(c) H2S and Br2 (d) NH3 and SO2
15. In the reaction : Na + (aq ) + Cl - (aq ) + H 2 O (l ) + CO 2 (g )
C + 4HNO3 ® CO2 + 2H 2O + 4NO2 (a) (i) 6, (ii) 4, yes (b) (i) 6, (ii) 6, No
HNO3 act as (c) (i) 4, (ii) 4, No (d) (i) 4, (ii) 4, yes
(a) an oxidizing agent 21. In the disproportionation reaction
(b) an acid 3 HClO3 ¾® HClO4 + Cl2 + 2O2 + H2O, the
equivalent mass of the oxidizing agent is (molar
(c) an acid as well as oxidizing agent
(d) a reducing agent. mass of HClO3 = 84.45)
16. Which of the following represents redox
reactions? (a) 16.89 (b) 32.22 (c) 84.45 (d) 28.15
I. Cr2 O 72 - ® 2Cr 42 -
+ 2OH ¾¾ + H 2O 22. Th e oxidation number of phosphorus in
Ba(H2PO2)2 is
II. Zn + CuSO 4 ¾¾
® ZnSO 4 + Cu (a) +3 (b) +2 (c) +1 (d) –1
- 23. How many electrons are involved in the following
III. 2MnO 4- + 3Mn 2 + + 4 OH ¾¾®
redox reaction?
5MnO 2 + 2H 2 O
IV. 2Cu + ¾¾ Cr2 O27 - + Fe2 + + C2 O42 -
® Cu + Cu 2 +
(a) I, II (b) I, III
® Cr3+ + Fe3+ + CO 2 (Unbalanced)
(c) III, IV (d) II, III, IV
17. The standard reduction potentials at 298K for the (a) 3 (b) 4 (c) 6 (d) 5
following half reactions are given against each 24. Given :
XNa2HAsO3 + YNaBrO3 + ZHCl ®
Zn2+ (aq) + 2e ƒ Zn(s) ; –0.762 V
NaBr + H3AsO4 + NaCl
Cr3+ (aq) + 3e ƒ Cr (s); –0.740 V The values of X, Y and Z in the above redox
2H+ (aq) + 2e ƒ H2 (g) ; 0.00 V reaction are respectively :
Fe3+ (aq) + e ƒ Fe2+ (aq) ; 0.770 V
(a) 2, 1, 2 (b) 2, 1, 3 (c) 3, 1, 6 (d) 3, 1, 4
Redox Reactions 95
25. When SO2 is passed through acidified solution 33. In the following balanced reaction,
of potassium dichromate, then chromium
sulphate is formed. The change in valency of X MnO4- + Y C2 O 42- + Z H +
chromium is Z
(a) +4 to +2 (b) +5 to +3 X Mn 2 + + 2Y CO 2 + H2O
2
(c) +6 to +3 (d) +7 to +2 values of X, Y and Z respectively are
26. In which of the following transition metal
complexes does the metal exhibit zero oxidation
state ? (a) 2, 5, 16 (b) 8, 2, 5
(c) 5, 2, 16 (d) 5, 8, 4
(a) [Co(NH3)6]Cl3 (b) [Fe(H 2 O) 6 ]SO 4
34. Consider the reaction:
(c) Ni(CO)4 (d) [Fe(H2O)6]X3
27. In which of the following pairs, there is greatest
difference in the oxidation number of the H 2SO3 (aq) + Sn 4+ (aq) + H 2O(l)
underlined elements ? ® Sn 2 + (aq) + HSO-4 (aq) + 3H + (aq)
(a) NO2 and N 2 O 4 (b) P2 O5 and P 4 O10 Which of the following statements is correct?
(c) N 2 O and NO (d) SO2 and SO3 (a) Sn 4+ is the oxidizing agent because it
undergoes oxidation
28. Which of the following statements is not
(b) Sn 4+ is the reducing agent because it
correct? undergoes oxidation
(c) H2SO3 is the reducing agent because it
(a) The oxidation number of S in (NH4)2S2O8 undergoes oxidation
is +6. (d) H2SO3 is the reducing agent because it
(b) The oxidation number of Os in OsO4 is +8. undergoes reduction
(c) The oxidation number of S in H2SO5 is +8. 35. Which of the following statements is not correct
(d) The oxidation number of O in KO2 is –1/2. about the reaction given below?
29. Among NH3, HNO3, NaN3 and Mg3N2 the number ® Fe3+ + CO2 + NO3-
Oxidation
K 4 [Fe(CN)6 ] ¾¾¾¾¾
of molecules having nitrogen in negative
(a) Fe is oxidised from Fe2+ to Fe3+.
oxidation state is
(b) Carbon is oxidised from C2+ to C4+.
(a) 1 (b) 2 (c) 3 (d) 4
(c) N is oxidised from N3– to N5+.
30. aK 2 Cr2 O 7 + bKCl + cH 2SO 4 ¾¾
® (d) Carbon is not oxidised.
36. Which of the following is not an intermolecular
xCrO2Cl 2 + yKHSO4 + z H 2 O redox reaction?
The above equation balances when
(a) a = 2, b = 4, c = 6 and x = 2, y = 6, z = 3 (a) MgCO3 ¾¾
® MgO + CO 2
(b) a = 4, b = 2, c = 6 and x = 6, y = 2, z = 3
(c) a = 6, b = 4, c = 2 and x = 6, y = 3, z = 2 (b) O2 + 2H 2 ¾¾
® 2H 2O
(d) a = 1, b = 4, c = 6 and x = 2, y = 6, z = 3
31. The number of electrons involved in the (c) ® KOH + (1/ 2) H2
K + H2O ¾¾
reduction of one nitrate ion to hydrazine is ® MnBr2 + (1/ 2 ) Br2
(d) MnBr3 ¾¾
(a) 8 (b) 5 (c) 3 (d) 7
37. Which of the following is not a
32. Which of the following do not show disproportionation reaction?
disproportionation reaction?
(a) P4 + 5OH - ¾¾
® H 2 PO 2- + PH 3
ClO–4, F2, Cl2, ClO2–, ClO2–, P4, S8, and ClO– -
(a) ClO2–, ClO4–, and ClO– (b) Cl2 + OH - ¾¾
® Cl - + ClO
(b) F2 only (c) 2H 2 O2 ¾¾
® 2H 2O + O2
(c) F2 and ClO4– (d) PbO2 + H 2O ¾¾
® PbO + H 2 O2
(d) ClO4– only
EBD_7587
96 Chemistry Objective MCQs
38. In a balance equation H2SO4 + xHI ® H2S + yI2 + 46. The brown ring complex is formulated as
zH2O, the values of x, y, z are [Fe(H2O)5 NO]SO4. The oxidation number of iron
is
(a) x = 3, y = 5, z = 2 (b) x = 4, y = 8, z = 5 (a) 1 (b) 2 (c) 3 (d) 0
(c) x = 8, y = 4, z = 4 (d) x = 5, y = 3, z = 4 47. In which of the following reactions, there is no
39. Given
Fe3+ (aq) + e– ® Fe2+ (aq); Eo = + 0.77 V change in valency ?
Al3+ (aq) + 3e– ® Al(s); Eo = – 1.66 V
Br2(aq) + 2e– ® 2Br–(aq); Eo = + 1.09 V (a) 4KClO3 ® 3KClO4 + KCl
Considering the electrode potentials, which of (b) SO2 + 2 H2S ® 2H2O + 3 S
the following represents the correct order of (c) BaO2 + H2SO4 ® BaSO4 + H2O2
reducing power? (d) 2 BaO + O2 ® 2 BaO2
(a) Fe2+ < Al < Br - (aq)(b) Br - (aq)
< Fe2+ < Al 48. The reaction,
-
(c) Al < Br < Fe 2+ (d) Al < Fe2+ < Br - (aq) 2H 2 O(l) ® 4H + (aq) + O 2 (g) + 4e - is
40. For the reaction : (a) an oxidation reaction
NH 3 + OCl - ¾ ¾® N 2 H 4 + Cl - in basic (b) a reduction reaction
medium, the coefficients of NH3, OCl– and N2H4 (c) a redox reaction
for the balanced equation are respectively (d) a hydrolysis reaction
(a) 2, 2, 2 (b) 2, 2, 1 (c) 2, 1, 1 (d) 4, 4, 2 49. In the reaction
41. Which of the following act as reducing agents ? 2FeCl 3 + H 2S ® 2 FeCl 2 + 2 HCl + S
(i) PO34- (ii) SO3 (iii) PO32 - (iv) NH3 (a) FeCl3 acts as an oxidising agent
(a) (i), (ii) and (iii) (b) Only (iii) (b) Both H2S are FeCl3 are oxidised
(c) (i), (iii) and (iv) (d) (iii) and (iv) (c) FeCl3 is oxidised while H2S is reduced
42. In the reaction (d) H2S acts as an oxidising agent.
50. The oxidation number of S in Na2S4O6 is
Cr2 O72 - + 14H + + 6I - ¾¾
® (a) +0.5 (b) 2.5 (c) + 4 (d) + 6
2Cr3+ + 7H 2 O + 3I2
51. In the following reaction
Which element is reduced ? 4P + 3KOH + 3H 2 O ¾
¾® 3KH 2 PO 2 + PH 3
(a) I (b) O (c) H (d) Cr
(a) phosphorus is both oxidised and reduced.
43. Which of the following substances acts as an
(b) only phosphorus is reduced.
oxidising as well as a reducing agent?
(c) phosphorus is not oxidised
(d) None of these
(a) Na 2O (b) SnCl2 52. The standard electrode potentials of four
elements A, B, C and D are –3.05, –1.66, –0.40
(c) Na 2 O 2 (d) NaNO 2 and +0.80. The highest chemical reactivity will
44. Stronger is oxidising agent, more is be exhibited by :
(a) standard reduction potential of that species
(b) the tendency to get itself oxidised (a) A (b) B (c) C (d) D
(c) the tendency to lose electrons by that 53. The oxidation number of Fe in Na2 [Fe(CN)5NO]
species is
(d) standard oxidation potential of that species (a) +2 (b) +1 (c) +3 (d) –2
45. Which of the following involves transfer of five 54. Which of the following is the strongest reducing
electrons ? agent in aqueous medium?
(a) Mg (b) Na (c) Li (d) Ca
(a) MnO -4 ® Mn 2+ (b) CrO 24- ® Cr 3+
55. The oxidation state of Fe in Fe3O8 is
(c) MnO 24- ® MnO 2 (d) Cr2O27- 3+
® 2Cr (a) 3/2 (b) 4/5 (c) 5/4 (d) 16/3
Redox Reactions 97
56. Which substance serves as reducing agent in 58. Which one of the following cannot function as
the following reaction? an oxidising agent ?
(a) I– (b) S(s)
14H + + Cr2 O 72– + 3Ni ¾¾
®
(c) NO3- (aq) (d) Cr2 O72 -
2Cr 3+ + 7H 2 O + 3Ni 2 +
-
(a) H 2O (b) Ni 59. The oxidation number of Pt in [Pt(C2H 4 ) Cl3 ]
(c) H+ (d) Cr2O 72- is
(a) + 1 (b) + 2 (c) + 3 (d) + 4
57. In the reaction 60. Standard electrode potentials of redox couples
A2+/A, B2+/B, C2+/C and D2+/D are 0.3V, – 0.5V, –
3Br2 + 6CO32- + 3H 2 O ¾¾
®
0.75V and 0.9V respectively. Which of these is
5Br - + BrO3– + 6HCO3- best oxidising agent and reducing agent
(a) bromine is oxidised and carbonate is respectively ?
reduced.
(b) bromine is reduced and water is oxidised (a) D2+/D and B2+/B (b) B2+/B and D2+/D
(c) bromine is neither reduced nor oxidised
(c) D2+/D and C2+/C (d) C2+/C and D2+/D
(d) bromine is both reduced and oxidised

Answer KEY
1 (d) 7 (b) 13 (b) 19 (a) 25 (c) 31 (d) 37 (d) 43 (d) 49 (a) 55 (d)
2 (a) 8 (c) 14 (b) 20 (c) 26 (c) 32 (c) 38 (c) 44 (a) 50 (b) 56 (b)
3 (d) 9 (d) 15 (a) 21 (a) 27 (d) 33 (a) 39 (d) 45 (a) 51 (a) 57 (d)
4 (d) 10 (c) 16 (d) 22 (c) 28 (c) 34 (c) 40 (c) 46 (a) 52 (a) 58 (a)
5 (c) 11 (d) 17 (a) 23 (a) 29 (c) 35 (d) 41 (d) 47 (c) 53 (a) 59 (b)
6 (c) 12 (c) 18 (b) 24 (c) 30 (d) 36 (a) 42 (d) 48 (d) 54 (c) 60 (c)

1. (d) (a) and (b) are neutralisation reactions. The is reduced and give NH4NO3, N2O, NO and
oxidation state of Cu is +2 in both reactant NO2 (based upon the concentration of
HNO3)
and product, and SO24- ion does not
[ Zn + 2HNO 3 ¾
¾® Zn ( NO3 ) 2 + 2H] ´ 4
change. In option (d) is a redox reaction. ( nearly 6%)

Zn ® Zn 2+ + 2e- (Oxidation) HNO 3 + 8H ¾


¾® NH 3 + 3H 2 O
+ -
2H + 2e ® H 2 (Reduction) NH 3 + HNO 3 ¾
¾® NH 4 NO 3
2. (a) O. N. of Cr on both sides is + 6.
4Zn+10HNO3 ¾¾
®
3. (d) Zinc gives H2 gas with dil H2SO4/HCl but
not with HNO3 because in HNO3, NO3– ion 4Zn(NO3 ) 2 +NH 4 NO3 +3H 2 O
EBD_7587
98 Chemistry Objective MCQs
Zn is on the top position of hydrogen in 10. (c) SO2 is an oxidising agent
electrochemical series. So Zn displaces H2 4e– + SO2 ¾¾ ® S
from dilute H2SO4 and HCl with liberation
64
of H2. Equivalent of SO2 = = 16.
Zn + H2SO4 ® ZnSO4 + H2 4
11. (d) CrO2 Cl2
4. (d) Xe = 53.5 % \ F = 46.5%
Let O. No. of Cr = x
Relative number of atoms Xe
\ x + 2 (–2) + 2 (–1) = 0
53.5 46.5 x–4–2=0
= = 0.4 and F = = 2.4
131.2 19 \ x= +6
Simple ratio Xe = 1 and F = 6 ; Molecular 12. (c) In SO32 –
formula is XeF6 x + 3(– 2) = – 2; x = + 4
O.N.of Xe is +6 In S2O24 –
5. (c) (a) and (b) are disproportionation reactions. 2x + 4(– 2) = – 2
(c) is an oxidation reaction. 2x – 8 = – 2
2H - ¾¾ ® H 2 + 2e - 2x = 6; x = + 3
6. (c) Given Fe3+/Fe2+ = + 0.77 V In S2 O62–
and I2/2I– = 0.536V
– 3+
2 (e + Fe ¾¾ 2+ 2x + 6(– 2) = – 2
® Fe ) E° = 0.77 V
– ¾¾ ® – 2x = 10; x = + 5
2I I2 + 2e E° = – 0.536 V
hence the correct order is
2Fe3+ + 2I– ¾¾ 2+
® 2Fe + I2 S2O24 – < SO2–3 < S2O6
2–

E° = E°ox + E°red 13. (b) In the given reaction, oxidation state of Mg


= 0.77 – 0.536 is changing from 0 to +2, while in nitrogen it
= 0.164 V is changing from 0 to – 3. So oxidation of
\ Since value of E° is + ve reaction will take Mg and reduction of nitrogen takes place.
place. 14. (b) SO2 bleaches by reduction, while chlorine
7. (b) Oxidation reaction (loss of 2e–) bleaches colour of flowers by oxidation.
15. (a) O.N. of C changes form 0 to + 4 by oxidation.
2Na(s) + S(s) ¾® Na2S(s) Hence HNO3 is oxidising agent.
16. (d) II, III and IV are redox reactions.
Reduction (gain of2e–) 17. (a) Since oxidation potential of Zn is highest
reaction hence strongest reducing agent.
-4 +4 +6 18. (b) Oxidation no. of O are + 2, 0, – 1/2 and – 1
8. (c) loss of 10e-
N2 H4 ¾¾¾¾¾® N2Y; respectively
N
O.N.of N changes from – 2 to +3 19. (a) The balanced equation is
9. (d) 12H + + 10e - + 2IO3- ¾¾ ® I 2 + 6H 2O

( )
+2
(a) é +3 0 -1 ù -1 ´ 2 10I- ¾¾
® 5I 2 + 10e -
ëê Co N H3 5 Cl ûú C l2
+3 +5x + 15 –1 = +2 10I- + 12H + + 2IO3- ¾¾
® 6H 2O + 6I 2
5x + 17 = +2 or 5I - + 6H + + IO3- ¾¾
® 3H 2O + 3I 2
5x = –15
x = –3; Oxidation State of N = –3 a b c d
(b) NH2OH : x + 3 – 2 = 0 Þ x = –1 20. (c) The redox reaction involve loss or gain of
electron(s) i.e. change in oxidation state.
Oxidation state of N = –1 Given reaction is not a redox reaction as
(c) (N2H5)+1 SO42– : this reaction involves no change in
N2H5+ : 2x + 5 = + 1 Þ x = – 2 oxidation state of reactant or product.
Oxidation number of N = –2 21. (a) ClO3- ¾¾ ® Cl02
(d) Mg3 N 2 : 2 × 3 + 2x = 0 x - 6 = -1 x =0
2x = –6 x = +5 x = 0 ( x = oxidation number )
x = –3 Oxidation state of N = –3
Redox Reactions 99
Molecular mass (Since it has one peroxide bond)
Equivalent mass = (d) K1+ O21–, oxidation state of O = –1/2
Oxidation number
29. (c) Calculating the oxidation state of nitrogen
84.45 in given molecules;
= = 16.89 Oxidation state of N in NH3 is
5
-1´2 x + 3 × (+ 1) = 0 or x = – 3
22. (c) Ba 2 + ( H 2 PO2 )2 : Oxidation state on N in HNO3 is
1 + x + 3 × (– 2) = 0 or x = + 5
H 2 PO2- : 2 + x - 4 = -1 Oxidation state of N in NaN3 is
Þ x = +1 1
\ Oxidation state of P = + 1 + 1 + 3x = 0 or x = –
23. (a) The reaction given is 3
Oxidation state of N in Mg3N2 is
Cr2O72– + Fe2+ + C2O42– ¾¾ ® 3 × 2 + 2x = 0 or x = –3
Cr3+ + Fe3+ + CO2 Thus 3 molecules (i.e. NH3, NaN3 and
Cr2O72– ¾¾ ® 2Cr3+ Mg3N2 have nitrogen in negative oxidation
On balancing states.
30. (d) The balanced equation is
14H+ + Cr2O72– + 6e– ¾¾ ®
2Cr3+ + 7H2O ...(i) K 2 Cr2 O 7 + 4KCl + 6H 2SO 4 ¾¾
®
Fe ¾¾
2+ ® Fe + e
3+ – ...(ii) 2CrO2 Cl2 + 6K 2SO4 + 3H 2 O
C2O4 ¾¾
2–
® 2CO2 + 2e – ...(iii) +5 -2
On adding all the three equations. 31. (d) N O3- ¾¾
® N 2 H4 So, for reduction of 1
Cr2O72– + Fe2+ + C2O2– +
4 + 14H + 3e

mole of NO3– number of electrons required
¾¾ ® 2Cr + Fe + 2CO2 + 7H2O
3+ 3+
is 7.
Hence the total no. of electrons involved 32. (c) F2 being most electronegative element
in the reaction = 3 cannot exhibit any positive oxidation state.
24. (c) On balancing the given reaction, we find In ClO –4 chlorine is present in its highest
3Na2HAsO3 + NaBrO3 + 6HCl oxidation state i.e +7. Therefore it does
¾¾® 6NaCl + 3H3AsO4 + NaBr not show disproportionation reaction.
25. (c) K2Cr2O7 + 3SO2 + 4H2SO4 ® 33. (a) X MnO4– + Y C2O 42– + ZH +
K 2SO 4 + Cr2 (SO4 )3 + 3SO3 + 4H 2 O Z
X Mn +2 + 2Y CO2 + H 2O
O.N.of chrominum changes from +6 to +3 2
26. (c) In metal carbonyls, metal always has zero
O.N . ® Mn +2 ....
First half reaction MnO 4– ¾¾
27. (d) O.N. of N in NO2 and N2O4 is +4 (i)
\ difference is zero. On balancing
O.N. of P in P2O5 and P4O10 is +5
\ difference is zero MnO 4– + 8H + + 5e – ¾¾
® Mn +2 + 4H 2 O
O.N. of N in N2O is +1 and in NO is +2. The .... (ii)
difference is 1 Second half reaction
O.N. of S in SO2 is +4 and in SO3 is +6. The
C 2 O 2–
4 ¾¾
® 2CO 2 .... (iii)
difference is +2.
28. (c)

( NH ) S O
On balancing
+1
(a) 2-
4 2 2 8 C 2 O 2–
4 ¾¾ ® 2CO 2 + 2e – .... (iv)
\ Oxidation state of S = +6 On multiplying eqn. (ii) by 5 and (iv) by
(Since S2 O82 - has one peroxide bond) 2 and then adding we get
(b) Oxidation state of Os = +8 2MnO 4– + 5C 2 O 42– + 16H + ¾¾
®
(c) Oxidation state of S in H 2SO5 = +6 2Mn +2 + 10CO2 + 8H 2O
EBD_7587
100 Chemistry Objective MCQs
+4 48. (d) Since there is loss of electrons, hence it is
34. (c) H 2SO 3 (aq) + Sn 4 + (aq) + H 2O (l) ¾¾
®
oxidation reaction.
+6
Sn 2 +(aq) + HSO -4 (aq) + 3H + 49. (a) In 2FeCl 3 + H 2S ® 2FeCl 2 + 2HCl + S
Hence H 2 SO 3 is the reducing agent O.N. of S changes from –2 to 0 (hence
because it undergoes oxidation. oxidised) O.N. of Fe changes from +3 to +2
35. (d) In CN –, oxidation of C is +2, and it changes hence reduced.
to +4 oxidation state in CO2. So C is also
50. (b) Na 2S4O6 : 2 + 4x - 12 = 0
oxidised.
36. (a) Intermolecular redox reaction and redox 10
reactions are same. \ x= = 2.5
4
37. (d) Disproportionation involves simultaneous
Oxidation number of S = 2.5
oxidation and reduction of the same atom
in a molecule. 51. (a) 4P + 3KOH + 3H 2 O ® 3KH 2 PO2 + PH3
38. (c) The value of x, y, z are 8, 4, 4 respectively O.N of P = 0, In KH2PO2 it is + 1, In PH3
hence the reaction is it is –3.
Hence P is oxidised and reduced.
H 2SO4 + 8HI ® H 2S + 4I2 + 4H 2O 52. (a) Standard electrode potential i.e. reduction
39. (d) Reducing character decreases down the potential of A is minimum (–3.05V) i.e. its
series. Hence the correct order is oxidation potential is maximum which
Al < Fe2+ < Br– implies ‘A’ is most reactive chemically.
40. (c) The balanced equation :
2-
2 NH 3 + OCl - ¾
¾® N 2 H 4 + Cl - + H 2 O 53. (a)
+2 é +2 -5 +1 ù
41. (d) In (i) and (ii) both P and S are in highest Na 2 ëê Fe (CN)5 NO ûú
oxidation state. In (iii) and (iv) ; P has Oxidation state of Fe = +2
oxidation state of +4 which can be oxidized 54. (c) In an aqueous medium, Li is the strongest
to +5 state, while in case of NH3 nitrogen reducing agent, since the high negative
has oxidation state of –3 which can be enthalpy of hydration compensates high
oxidised. IE1.
42. (d) 2I- ® I2 is oxidation (loss of electrons) ; 55. (d) Fe3O8
Cr (+6) changes to Cr (+3) by gain of 16
3x – 16 = 0 Þ x =
electrons. Hence Cr is reduced. 3
43. (d) In Na2O, SnCl2 and Na2O2 central atom is 56. (b) (Ni ® Ni2+) Ni loses electrons hence act as
either in lowest or highest oxidation state, reducing agent.
so they can function either as an oxidising 57. (d) 3Br2 + 6CO32 – + 3H 2 O ®
or a reducing agent but not both. However,
the oxidation state of N in NaNO2 is +3 5Br – + BrO3– + 6HCO 3–
which lies between its highest (+5) and O.N. of Br2 changes from 0 to –1and +5 hence
lowest (–3) values. it is reduced as well as oxidised.
44. (a) More is E°RP , more is the tendency to get 58. (a) If an electronegative element is in its lowest
possible oxidation state in a compound or
itself reduced or more is oxidising power.
in free state. It can function as a powerful
45. (a) O.N. of Mn in MnO –4 is +7 and in Mn 2+ it reducing agent. e.g. I –
is +2. The difference is of 5 electrons. –
46. (a) [Fe(H2O)5NO] SO4 Let O.N. of Fe be x then, é +2 0 3– ù
59. (b) ê Pt (C2H 4 ) Cl3 ú
1× (x) + 5 × (0) + 1× (+1) + 1× (–2) = 0 ë û
\ x = +1 Oxidation number of Pt = +2
47. (c) In BaO 2 + H 2SO 4 ® Ba SO 4 + H 2 O 2 all 60. (c) The redox couple with maximum reduction
atoms are present in the same O.S. in potential will be best oxidising agent and
reactants and products. with minimum reduction potential will be best
reducing agent.
9 Hydrogen
1. When electric current is passed through an ionic (b) It has very low dielectric constant
hydride in molten state : (c) It has high liquid range
(a) hydrogen is liberated at anode (d) None of these
(b) hydrogen is liberated at cathode 8. A, 6 volume sample of H2O2
(c) hydride ion migrates towards cathode
(d) hydride ion remains is solution
2. Among CaH2, NH3, NaH and B2H6 which are (a) will contain 6% V/V of H2O2
covalent hydrides? (b) will contain 6% W/V of H2O2
(a) NH3 and B2H6 (b) NaH and CaH2 (c) would give 6 volumes of oxygen per unit
(c) NaH and NH3 (d) CaH2 and B2H6 volume of H2O2 sample at STP
3. Polyphosphates are used as water softening (d) would give 6 volumes of oxygen per unit
agents because they weight of H2O2 sample at STP
(a) form soluble complexes with anionic species 9. 34 g of H2O2 is present in 1120 mL of solution.
(b) precipitate anionic species
This solution is called
(c) form soluble complexes with cationic species
(d) precipitate cationic species (a) 10 vol solution (b) 20 vol solution
4. When two ice cubes are pressed over each other, (c) 34 vol solution (d) 32 vol solution
they unite to form one cube. Which of the following 10. Hydrogen can be fused to form helium at
forces is responsible to hold them together ? (a) high temperature and high pressure
(a) Hydrogen bond formation (b) high temperature and low pressure
(b) van der Waals forces (c) low temperature and high pressure
(c) Covalent attraction (d) low temperature and low pressure
(d) Ionic interaction 11. The oxidation states of most electronegative
5. Hydrogen molecules differs from chlorine element in the products of reaction BaO2 with
molecule in the following respect
dil. H2SO4 are
(a) Hydrogen molecule is non-polar but chlorine
molecule is polar (a) 0 and – 1 (b) –1 and – 2
(b) Hydrogen molecule is polar while chlorine (c) – 2 and 0 (d) – 2 and + 1
molecule is non-polar 12. A 5.0 mL solution of H2O2 liberates 1.27 g of
(c) Hydrogen molecule can form intermolecular iodine from an acidified KI solution. The
hydrogen bonds but chlorine molecule does percentage strength of H2O2 is
not (a) 11.2 (b) 5.6 (c) 1.7 (d) 3.4
(d) Hydrogen molecule cannot participate in 13. 3.4 g sample of H2O2 solution containing x%
coordination bond formation but chlorine H2O2 by weight requires x mL of KMnO4 solution
molecule can for complete oxidation under acidic condition.
6. The adsorption of hydrogen by metals is called The normality of KMnO4 solution is
(a) Dehydrogenation (b) Hydrogenation
(c) Occlusion (d) Adsorption
7. Which of the following statements do not define (a) 1 N (b) 2 N (c) 3 N (d) 0.5 N
the characteristic property of water “Water is a 14. H2O2 ® 2H+ + O2 + 2e– ; E° = –0.68 V. This
universal solvent” equation represents which of the following
(a) It can dissolve maximum number of behaviour of H2O2?
compounds (a) Reducing (b) Oxidising
(c) Acidic (d) Catalytic
EBD_7587
102 Chemistry Objective MCQs
15. When a substance A reacts with water it produces (a) H - (aq ) + H 2 O ® H 3O -
a combustible gas B and a solution of substance (b) H - (aq) + H 2 O (l) ® OH - + H 2
C in water. When another substance D reacts
(c) H - + H 2 O ® No reaction
with this solution of C, it also produces the same
gas B on warming but D can produce gas B on (d) None of these
24. In context with the industrial preparation of
reaction with dilute sulphuric acid at room
hydrogen from water gas (CO + H2), which of the
temperature. A imparts a deep golden yellow
following is the correct statement?
colour to a smokeless flame of Bunsen burner. A,
B, C and D respectively are
(a) CO and H2, are fractionally separated using
(a) Na , H2, NaOH, Zn differences in their densities
(b) K, H2, KOH, Al (b) CO is removed by absorption in aqueous
(c) Ca, H2, Ca(OH)2, Sn Cu2Cl2 solution
(d) CaC2, C2H2, Ca(OH)2, Fe (c) H2 is removed through occlusion with Pd
16. HCl is added to following oxides. Which one (d) CO is oxidised to CO2 with steam in the
would give H2O2 presen ce of a catalyst followed by
(a) MnO2 (b) PbO2 (c) BaO (d) None absorption of of CO2 in alkali
17. Chemical A is used for water softening to remove 25. The dielectric constant of H2O is 80. The
electrostatic force of attraction between Na + and
temporary hardness. A reacts with sodium
Cl– will be
carbonate to generate caustic soda. When CO2
(a) reduced to 1/40 in water than in air
is bubbled through a solution of A, it turns cloudy. (b) reduced to 1/80 in water than in air
What is the chemical formula of A? (c) will be increased to 80 in water than in air
(a) CaCO3 (b) CaO (d) will remain unchanged.
(c) Ca(OH)2 (d) Ca(HCO3)2 26. Which of the following is not true?
18. Which physical property of dihydrogen is (a) D2O freezes at lower temperature than H2O
wrong? (b) Reaction between H2 and Cl2 is much faster
(a) Odourless gas than D2 and Cl2
(b) Tasteless gas (c) Ordinary water gets electrolysed more
(c) Colourless gas rapidly than D2O
(d) Non–inflammable gas (d) Bond dissociation energy of D2 is greater
19. Permanent hardness in water cannot be cured than H2
by : 27. H2O2 is always stored in black bottles because
(a) Treatment with washing soda (a) It is highly unstable
(b) Boiling (b) Its enthalpy of decomposition is high
(c) Calgon’s method (c) It undergoes auto-oxidation on prolonged
(d) Ion exchange method standing
20. Which of the following metal evolves hydrogen (d) None of these
on reaction with cold dilute HNO3 ? 28. Which of the following is wrong about H2O2?
(a) Mg (b) Al (c) Fe (d) Cu It is used
21. The alum used for purifying water is (a) as an aerating agent in production of sponge
(a) Ferric alum (b) Chrome alum rubber
(c) Potash alum (d) Ammonium alum (b) as an antichlor
22. What is formed when calcium carbide reacts with (c) for restoring white colour of blackened lead
heavy water? painting
(a) C2D2 (b) CaD2 (d) None of these
(c) Ca2D2O (d) CD2 29. 100 mL of 0.01 M KMnO4 oxidises 100 mL H2O2
23. The hydride ion H– is stronger base than its in acidic medium. Volume of the same KMnO4
hydroxide ion OH– . Which of the following required in alkaline medium to oxidise 100 mL of
reactions will occur if sodium hydride (NaH) is the same H2O2 will be (MnO4– changes to Mn2+
dissolved in water ?
Hydrogen 103
in acidic medium and to MnO2 in alkaline (b) It gives precipitate
medium) (c) it contains impurities
(d) it is acidic in nature
38. The amount of H2O2 present in 1 litre of 1.5 N H2O2
100 500 solution, is :
(a) mL (b) mL (a) 25.5 g (b) 3.0 g (c) 8.0 g (d) 2.5 g
3 3
39. 20 volume H2O2 solution has a strength of about
300
(c) mL (d) None
5
30. The purity of H2O2 in a given sample is 85%.
Calculate the weight of impure sample of H2O2 (a) 30% (b) 6% (c) 3% (d) 10%
which requires 10 mL of M/5 KMnO4 solution in 40. When same amount of zinc is treated separately
titration in acidic medium with excess of sulphuric acid and excess of
sodium hydroxide solution the ratio of volumes
(a) 2 g (b) 0.2 g (c) 0.17 g (d) 0.15 g of hydrogen evolved is
31. Barium peroxide reacts with phosphoric acid to
produce barium phosphate along with (a) 1 : 1 (b) 1 : 2 (c) 2 : 1 (d) 9 : 4
(a) water (b) hydrogen peroxide 41. Hydrogen at room temperature contains :
(c) dioxygen (d) phosphine (a) 25% ortho + 75% para
32. The normality and volume strength of a solution (b) 25% para + 75% ortho
made by mixing 1.0 L each of 5.6 volume and 11.2 (c) 50% ortho + 50% para
volume H2O2 solution are : (d) 60% ortho + 34% para
42. In Bosch’s process, which gas is utilised for the
(a) 1 N, 5.6 vol (b) 1.5 N. 5.6 vol production of hydrogen gas ?
(c) 1.5 N, 8.4 vol (d) 1 N, 8.4 vol (a) Producer gas (b) Water gas
33. The numbers of protons, electrons and neutrons (c) Coal gas (d) None of these
in a molecule of heavy water are respectively : 43. D2O is preferred to H2O, as a moderator, in nuclear
(a) 8, 10, 11 (b) 10, 10, 10 reactors because
(c) 10, 11, 10 (d) 11, 10, 10 (a) D2O slows down fast neutrons better
34. 2 g of aluminium is treated separately with excess (b) D2O has high specific heat
of dilute H2SO4 and excess of NaOH. The ratio of (c) D2O is cheaper
the volumes of hydrogen evolved is (d) None of these
(a) 2 : 3 (b) 1 : 1 (c) 2 : 1 (d) 1 : 2 44. Consider the following statements :
35. Which of the following statements is correct? 1. Atomic hydrogen is obtained by passing
(a) Hydrogen has same IP as alkali metals hydrogen through an electric arc.
(b) Hydrogen has same electronegativity as 2. Hydrogen gas will not reduce heated
halogens aluminium oxide.
(c) Hydrogen has oxidation number of –1 and 3. Finely divided palladium adsorbs large
+1 volume of hydrogen gas
(d) Hydrogen will not be liberated at anode. 4. Pure nascent hydrogen is best obtained by
36. Which of the following terms is not correct for reacting Na with C2H5OH
hydrogen? Which of the above statements is/are correct?
(a) Its molecule is diatomic (a) 1 alone (b) 2 alone
(b) It exists both as H+ and H– in different (c) 1, 2 and 3 (d) 2, 3 and 4
chemical compounds 45. Water is :
(c) It is the only species which has no neu- (a) more polar than H2S
trons in the nucleus (b) more or less identical in polarity with H2S
(d) Heavy water is unstable because hydrogen (c) less polar than H2S
is substituted by its isotope deuterium (d) None of these
37. Hard water is not fit for washing clothes because 46. Calgon used as a water softener is
(a) it contains Na2SO4 and KCl (a) Na2[Na4(PO3)6] (b) Na4[Na2(PO3)6]
(c) Na4[Na4(PO4)5] (d) Na4[Na2(PO4)6]
EBD_7587
104 Chemistry Objective MCQs
47. In which of the following reactions, H2O2 is 53. Pick out the correct statement
acting as a reducing agent (a) By decreasing the temperature pure para-
(a) H 2 O 2 + SO 2 ® H 2SO 4 hydrogen can be obtained
(b) By increasing the temperature pure ortho-
(b) 2KI + H 2O 2 ® 2KOH + I 2 hydrogen can be obtained
(c) PbS + 4H 2 O 2 ® PbSO 4 + 4H 2O (c) By decreasing the temperature pure ortho-
hydrogen can be obtained
(d) Ag 2 O + H 2 O 2 ® 2Ag + H 2O + O 2
(d) By increasing the temperature pure para-
48. Which of the following statements are correct? hydrogen can be obtained
(i) Hydrogen peroxide is industrially 54. An inorganic compound gives off O2 when
prepared by the auto-oxidation of 2-alkyl heated, turns an acidic solution of KI violet and
anthraquinols reduces acidified KMnO4. The compound is
(ii) One millilitre of 30% H2O2 means that
solution will give 100 V of oxygen at STP
(iii) Dihedral angle of H2O2 in gas phase is
90.2° and in solid phase dihedral angle is (a) SO3 (b) KNO3
111.5°
(a) (i), (ii) and (iii) (b) (i) and (iii) (c) H2O2 (d) All of these
(c) (ii) and (iii) (d) (i) and (ii) 55. The reaction of H2O2 with sulphur is an example
49. When a sample of hard water is passed through of ........ reaction
the layer of sodium zeolite resulting which of (a) Addition (b) Oxidation
the following ions will not be present in the (c) Reduction (d) Redox
resulting sample of water obtained? 56. Which substance does not speed up
(a) Mg2+ and Ca2+ (b) Ca2+ and Na+ decomposition of H2O2?
(a) Glycerol (b) Pt
(c) Mg2+ and CO32 - (d) CO32 - and Cl–
(c) Gold (d) MnO2
50. Which statement is wrong? 57. When zeolite (hydrated sodium aluminium
(a) Ordinary hydrogen is an equilibrium mixture silicate) is treated with hard water, the sodium
of ortho and para hydrogen ions are exchanged with
(b) In ortho hydrogen spin of two nuclei is in (a) H+ ions (b) Ca2+ ions
same direction
(c) Ortho and para forms do not resemble in (c) SO 4 2- ions (d) OH– ions
their chemical properties 58. Saline hydrides react explosively with water, such
(d) In para hydrogen spin of two nuclei is in fires can be extinguished by
opposite direction. (a) water (b) carbon dioxide
51. An orange coloured solution acidified with (c) sand (d) None of these
H2SO4 and treated with a substance 'X' gives a 59. 30 volume hydrogen peroxide means
blue coloured solution of CrO5. The substance
(a) 30% of H 2O 2 solution
'X' is
(b) 30 cm3 solution contains 1g of H 2O 2
(a) H2O2 (b) H2O
(c) 1 cm3 of solution liberates 30 cm3 of O2 at
(c) dil HCl (d) conc HCl
STP
52. Hydrogen peroxide acts both as an oxidising and
as a reducing agent depending upon the nature (d) 30 cm3 of solution contains 1 mole of H2O2
of the reacting species. In which of the following 60. Hydrogen peroxide in its reaction with KIO4 and
cases H2O2 acts as a reducing agent in acid NH2OH respectively, is acting as a
medium? (a) Reducing agent, oxidising agent
(b) Reducing agent, reducing agent
(a) MnO-4 (b) Cr2 O72- (c) Oxidising agent, oxidising agent
(c) SO32- (d) KI (d) Oxidising agent, reducing agent
Hydrogen 105

Answer KEY
1 (a) 7 (b) 13 (b) 19 (b) 25 (b) 31 (b) 37 (b) 43 (d) 49 (a) 55 (d)
2 (a) 8 (c) 14 (a) 20 (a) 26 (a) 32 (c) 38 (a) 44 (c) 50 (c) 56 (a)
3 (c) 9 (a) 15 (a) 21 (c) 27 (c) 33 (b) 39 (b) 45 (a) 51 (a) 57 (b)
4 (a) 10 (a) 16 (d) 22 (a) 28 (d) 34 (b) 40 (a) 46 (a) 52 (a) 58 (c)
5 (d) 11 (b) 17 (c) 23 (b) 29 (b) 35 (c) 41 (b) 47 (d) 53 (a) 59 (c)
6 (c) 12 (d) 18 (d) 24 (d) 30 (b) 36 (d) 42 (b) 48 (d) 54 (c) 60 (a)

1. ® H 2­ + 2e -
(a) At anode : 2H - ¾¾ 200
N of H 2 O2 = 5.6 ´
200
= 10
2. (a) N and B belong to p-block and since the 12 12
electronegativity difference between these vol strength
elements and H is very less, they form 10. (a) A fusion reaction between hydrogen nuclei
covalent hydrides. is difficult because positively charged
3. (c) Polyphosphates form soluble complexes nuclei repel each other. However, at very
with Ca2+ and Mg2+ and are removed. high temperatures of the order of 106 to 107
4. (a) Two ice cubes stick to each other due to K, the nuclei may have sufficient energy to
H-bonding overcome the repulsive forces and thus
5. (d) Chlorine has lone pair which it can donate fuse. This is why, fusion reactions are also
to form coordinate bond, while hydrogen called thermonuclear reactions.
cannot. 11. (b) BaO 2 + H 2SO 4 ® BaSO 4 + H 2 O 2 .
6. (c) Occlusion is the phenomenon of adsorption oxygen has common O.S. as –2 and in
of hydrogen by metal. peroxides as –1.
7. (b) Water has high dielectric constant i.e., 78.39 12. (d) H2O2 º KI º I2
C2/Nm2, high liquid range and can dissolve mEq of H2O2 = 5 × N
maximum number of compounds. That is
why it is used as universal solvent. Weight
mEq of I2 = ´1000
8. (c) 6 volume H2O2 would give 6 volumes of O2 E.w
per unit volume of H2O2 1.27
´ 1000 = 10 mEq
W2 ´1000 127
9. (a) N= I2 = M.w = 254
E.w2´ VmL
M.w 254
E.w = = = 127
2 2
34 ´ 1000 200
= = \ mEq of H2O2 = mEq of I2
17 ´1120 12
5 × N = 10 \N=2
1 N of H2O2 = 5.6 vol strength
EBD_7587
106 Chemistry Objective MCQs
1 N of H2O2 = 1.7% of H2O2 21. (c) Potash alum is used for purifying water.
2 N of H2O2 = 3.4% H2O2 22. (a) CaC2 + 2D 2 O ® C 2 D 2 + Ca (OD) 2
13. (b) 100 g of H2O2 sample solution contains = x
g of H2O2 23. (b) H - (aq) + H 2 O(l) ® OH - + H 2 . Since H–
3.4 g of H2O2 sample solution contains = is a strong base it will abstract H+ to form
x H2O.
´ 3.4 24. (d) On the industrial scale hydrogen is prepared
100
from water gas according to following
3.4 x
Weight of H2O2 = reaction sequence
100
catalyst
3.4 x 1 CO + H 2 + H 2 O ¾¾¾¾
® CO2 + 2H 2
Eq. of H2O2 = ´ 1424 3 {
100 17 water gas (steam)
mEq of H2O2
2NaOH
3.4x 34 x ¾¾¾¾
® Na 2 CO3 + H 2 O
(alkali)
= ´ 1000 = = 2x
100 ´ 17 17 From the above it is clear that CO is first
mEq of KMnO4 = x × N oxidised to CO2 which is then absorbed in
x × N = 2x NaOH.
N=2 25. (b) Electrostatic forces of attraction are reduced
14. (a) As H2O2 is loosing electrons, so it is acting to 1/80th in water.
as reducing agent. 26. (a) D2O actually has higher freezing point
(3.8°C) than water H2O (0°C)
15. (a) 2 Na + 2H 2 O ® 2 NaOH + H 2 ­
'A ' 'C' 'B' 27. (c) H2O2 is unstable liquid and decomposes
into water and oxygen either on standing or
Zn + 2 NaOH ® Na 2 ZnO 2 + H 2 ­ on heating.
'D ' 'C' 'B'
28. (d) H2O2 show all these properties.
Zn + dil. H 2SO 4 ® ZnSO 4 + H 2 ­ 29. (b) MnO4– + 5e– ® Mn2+ (acidic)
'D ' 'B'
MnO4– + 3e– ® MnO2 (basic)
Na produces golden yellow colour with
smokeless flame of Bunsen burner. 100 mL H2O2 º 100 × 5N
16. (d) MnO2 , PbO2 and BaO will not give H2O2 MnO4– º V × 3N MnO2
with HCl. MnO2 and PbO2 will give Cl2 and
500
BaO will react with HCl to give BaCl2 and N= mL
water. 3
17. (c) Ca(OH) 2 is used for the softening of 30. (b) mEq of MnO4– º mEq of H2O2
temporary hard water.
M
10 ´ ´ 5 º mEq of H O
CO2 5 2 2
Ca(OH) 2 (aq) ¾¾¾® CaCO3 (s) + H 2 O(l)
cloudiness Þ 10 mEq º mEq of H2O2
18. (d) H2 is a highly inflammable gas.
34
19. (b) Only temporary hardness which is due to Weight of H2O2 = 10 × 10–3 × × 0.17
2
HCO3- (bicarbonate) ions is removed by = 0.2 g
boiling. Weight of impure H2O2
20. (a) Mg + 2HNO3 (dil.) ® Mg(NO3 ) 2 + H 2
100
(Mg and Mn give H2 with dil HNO3) = ´ 0.17 = 0.2 g
85
Hydrogen 107
\ 20 litre O2 at N.T.P. obtained by
31. (b) 3BaO2 + 2H 3PO 4 ® Ba 3 ( PO 4 )2 + 3H 2O 2
Barium Phosphoric Barium Hydrogen 68
peroxide acid phosphate peroxide ´ 20 gm of H2O2 = 60.71 gm of H2O2
22.4
32. (c) 1 N = 5.6 vol \ 1000 ml O2 at N.T.P. obtained by
Þ Normalities of two solutions are 1 N and = 60.71 gm of H2O2
2N
\ 100 ml O2 at N.T.P. obtained by
Normality of mixture
60.71
1´ 1 + 1´ 2 3 = ´ 100 = 6.071%
= = = 1.5 N 1000
1 +1 2
40. (a) Zn + H2SO4 ® ZnSO4 + H2
Volume strength Zn + 2NaOH ® Na2ZnO2 + H2
3 16.8 \ Ratio of volumes of H2 evolved is 1 : 1
= ´ 5.6 = = 8.4 vol
2 2 41. (b) It is a fact.
33. (b) Heavy water is D2O hence catalyst
42. (b) CO + H 2 + H 2 O ¾¾¾¾
® CO2 + 2H 2
number of electrons = 2 + 8 = 10 14243
water gas
number of protons = 10
Atomic mass of D2O = 4 + 16 = 20 43. (d) H2O absorbs neutrons more than D2O and
hence number of neutron this decreases the number of neutrons for
= Atomic mass – number of protons the fission process.
= 20 – 10 = 10 44. (c) Pure hydrogen is evolved by reacting
34. (b) 2Al + 3H 2SO 4 ® Al 2 (SO4 )3 + 3H 2 absolute alcohol and Na
2Al + 2NaOH + 2H 2 O ® 2NaAlO 2 + 3H 2 1
C 2 H 5OH + Na ® C 2 H 5ONa + H 2
2
The ratio of volumes of hydrogen evolved
is 1:1. other statements are correct.
35. (c) In metal hydrides the O.S. of hydrogen –1 45. (a) Polarity of bond depends on difference in
otherwise it is +1. electronegativity of the two concerned
36. (d) Heavy water is stable. atoms. H2O is more polar than H2S because
oxygen (in O–H) is more electronegative
37. (b) 2RCOONa + Mg 2+ ® ( RCOO )2 Mg + 2Na + than sulphur (in S–H).
Insoluble
46. (a) The complex salt of metaphosphoric acid
2RCOONa + Ca 2 + ® ( RCOO )2 Ca + 2Na + sodium hexametaphosphate (NaPO3)6, is
Insoluble known as calgon. It is represented as
38. (a) Molecular weight of H2O2 = 34 Na2[Na4(PO3)6]
Equivalent weight of H2O2 = 17 47. (d) SO2 changes to H2SO4 (O.N. changes from
\ 1 L of 1 N H2O2 has = 17 g of H2O +4 to +6 oxidation)
\ 1 L of 1.5 N H2O2 has 2KI ® I2 (O.S. changes from –1 to 0
= 1.5 × 17 = 25.5 g of H2O2 oxidation)
39. (b) Q 22.4 litre O2 at N.T.P. obtained by 68 gm PbS ® PbSO 4 (O.S. changes from –2 to +6
of H2O2 oxidation)
\ 1 litre O2 at N.T.P. obtained by
Ag 2 O ® 2Ag (O.S. changes from +1 to 0
68 reduction)
gm of H2O2
22.4
EBD_7587
108 Chemistry Objective MCQs
48. (d) Dihedral angle of H2O2 in gas phase is 2KMnO4 + 3H 2SO4 + 5H 2O2 ®
111.5° and in solid phase it is 90.2°
K 2SO 4 + 2MnSO 4 + 8H 2 O + 5O 2
49. (a) As Na+ ions in sodium zeolite are replaced
by Ca 2+ and Mg2+ ions present in hard oxidation
55. (d)
water, these two ions will not be present.
50. (c) Ortho and para forms of hydrogen resemble H2S + H2O2 2H2O + S
in their chemical properties. reduction
51. (a) K2Cr2O7 an orange coloured solution, on
In this reaction H2S is oxidised to sulphur
acidification with dil H2SO4 is oxidised by
and H2O2 is reduced to H2O, hence this
H2O2 to blue peroxide of chromium, CrO5,
reaction show oxidation-reduction both i.e.,
which is unstable.
redox reaction.
K 2 Cr2 O7 + H 2SO 4 ® K 2SO 4 + H 2 Cr2 O7
56. (a) Glycerol, phosphoric acid or acetanilide is
éë H 2 O 2 ® H 2 O + [O ]ùû ´ 4
added to H2O2 to check its decomposition.
H 2Cr2 O7 + 4 [O ] ® 2CrO5 + H 2O 57. (b) Na zeolite + CaCl 2 ® Ca zeolite + 2NaCl
K 2Cr2O7 + H2SO4 + 4H2O 2 ® 2CrO5 + K 2SO4 + 5H 2O 58. (c) Fire due to action of water on saline hydrides
52. (a) H2O2 acts as a reducing agent only in cannot be extinguished with water or CO2.
presence of strong oxidising agents (i.e., These hydrides can reduce CO2 at high
MnO4–) in acidic as well as alkaline medium. temperature to produce O2.
2KMnO 4 + 3H 2SO 4 + 5H 2O 2 ¾¾
® 59. (c) 30 vol of H2O2 means one volume of H2O2
K 2SO 4 + 2MnSO 4 + 8H 2 O + 5O 2 on decom-position will give 30 volume of
oxygen.
53. (a) Pure para form can be obtained at low
temperature (20K) but pure ortho form is 60. (a) KIO4 + H2O2 ® KIO3 + H2O + O2
very difficult to obtain. Thus H2O2 is acting as a reducing agent
54. (c) 2H 2 O 2 ¾¾®D 2NH2OH + H2O2 ® N2 + 4H2O
2H 2 O + O 2
2KI + H 2O2 ® 2KOH + I 2 Here H2O2 is acting as an oxidising agent
10 The s-Block Elements
1. In the case of alkali metals, the covalent character (iii) Stability of LiHCO3 < NaHCO3 < KHCO3
decreases in the order : < RbHCO3 < CsHCO3.
(a) MF > MCl > MBr > MI (iv) Melting point NaF < NaCl < NaBr < NaI
(b) MF > MCl > MI > MBr (a) (i) and (iv) (b) (i) and (iii)
(c) MI > MBr > MCl > MF (c) (i) and (ii) (d) (ii) and (iii)
(d) MCl > MI > MBr > MF 8. The solubility order for alkali metal fluoride in
2. The mobility of metal ions in aqueous medium water is :
(Li+, Na+, K+, Rb+) in the electric field, follows (a) LiF < RbF < KF < NaF
the order (b) RbF < KF < NaF < LiF
(a) Li+ > Na+ > K+ > Rb+ (c) LiF > NaF > KF > RbF
(b) Rb+ > Na+ = K+ > Li+ (d) LiF < NaF < KF < RbF
(c) Li+ < Na+ < K+ < Rb+ 9. Based on lattice energy and other considerations,
(d) Na+ = K+ > Rb+ > Li+ which one of the following alkali metal chloride
3. The main oxides formed on combustion of Li, Na is expected to have the highest melting point?
and K in excess of air are, respectively: (a) NaCl (b) KCl (c) LiCl (d) RbCl
10. In crystals which one of the following ionic
compounds would you expect maximum distance
(a) Li2O2, Na2O2 and KO2 between centres of cations and anions?
(b) Li2O, Na2O2 and KO2 (a) LiF (b) CsF (c) CsI (d) LiI
(c) Li2O, Na2O and KO2 11. Which of the following statements about Na2O2
(d) LiO2, Na2O2 and K2O is not correct?
4. Which of the following oxides of potassium is
not known ? (a) It is diamagnetic in nature
(a) K2O (b) K2O4 (c) KO3 (d) K2O3
(b) It is derivative of H2O2
5. Nitrogen dioxide cannot be prepared by heating
(a) KNO3 (b) Pb(NO3)2 (c) Na 2O2 oxidises Cr 3+ to CrO24 - in acid
(c) Cu(NO3)2 (d) AgNO3 medium.
6. Incorrect statement is: (d) It is the super oxide of sodium
(a) NaHCO3 and KHCO3 have same crystal 12. The correct order of thermal stability of
structure hydroxides is:
(b) On heating LiNO3 decomposes into Li2O (a) Ba(OH)2 < Ca(OH)2 < Sr(OH)2 < Mg(OH)2
and NO2 (b) Mg(OH)2 < Sr(OH)2 < Ca(OH)2 < Ba(OH)2
(c) Among alkali metals, Li metal impart red (c) Mg(OH)2 < Ca(OH)2 < Sr(OH)2 < Ba(OH)2
(d) Ba(OH)2 < Sr(OH)2 < Ca(OH)2 < Mg(OH)2
colour to flame
13. The reducing power of a metal depends on
(d) Li2SO4 does not form alum various factors. Suggest the factor which makes
7. Which is not correctly matched Li metal the strongest reducing agent in aqueous
solution:
(i) Basic strength Cs2O < Rb2O < K2O < Na2O (a) Sublimation enthalpy
< Li2O of oxides. (b) Ionisation enthalpy
(ii) Stability of Na2O2 < K2O2 < Rb2O2 < Cs2O2
(c) Hydration enthalpy
peroxides.
(d) Electron-gain enthalpy
EBD_7587
110 Chemistry Objective MCQs
14. Select the correct statements : (a) It gives a white turbidity with dilute HCl
I. Cs+ is more highly hydrated than the other solution
alkali metal ions (b) It decolourises a solution of iodine in
II. Among the alkali metals Li, Na, K and Rb, potassium iodide
lithium has the highest melting point (c) It gives a white precipitate with silver nitrate
III. Among the alkali metals, only lithium forms solution which turns black on standing.
a stable nitride by direct combination with
Identify the compound (X)
nitrogen
(a) I, II and III (b) I and II (a) Na2CO3.10H2O (b) Na2S2O3.5H2O
(c) I and III (d) II and III (c) Na2SO4.10H2O (d) None of these
15. In the following sequence of reaction, identify the 20. A metal M readily forms its sulphate MSO4 which
compounds (A), (B) (C) and (D) : is water-soluble. It forms its oxide MO which
SO Na CO becomes inert on heating. It forms an insoluble
Na 2 CO3 ¾¾¾
2 ®(A) ¾¾¾¾®
2 3
hyroxide M(OH)2 which is soluble in NaOH
Solution S AgNO
(B) ¾¾¾
®(C) ¾¾¾¾
3 ®(D) solution. Then M is
Heat
(a) Mg (b) Ba (c) Ca (d) Be
(a) Na2SO3, NaHSO3, Na2S, Ag2S 21. Melting point of calcium halides decreases in the
(b) NaHSO3, Na2SO3, Na2S2O3, Ag2S order
(a) CaF2 > CaCl2 > CaBr2 > CaI2
(c) NaHSO3, Na2SO4, Na2S, Ag2O
(b) CaI2 > CaBr2 > CaCl2 > CaF2
(d) Na2SO3, Na2SO4, Na2S2O3, Ag (c) CaBr2 > CaI2 > CaF2 > CaCl2
16. Which of the following does not illustrate the (d) CaCl2 > CaBr2 > CaI2 > CaF2
anomalous properties of lithium? 22. Which of the following is not correct ?
(a) The melting point and boiling point of Li are
comparatively high
heat
(b) Li is much softer than the other group I (a) 2Li 2 O ¾¾¾® Li 2 O 2 + 2Li
673K
metals
heat
(c) Li forms a nitride Li3N unlike group I metals (b) 2K 2O ¾¾¾® K 2 O 2 + 2K
673K
(d) The ion of Li and its compounds are more
heat
heavily hydrated than those of the rest of the (c) 2Na 2 O ¾¾¾® Na 2O 2 + 2Na
673K
group
heat
17. The order solubility in water of alkaline earth metal (d) 2Rb 2O ¾¾¾® Rb 2 O 2 + 2Rb
673K
sulphates down the group is Be > Mg > Ca > Sr >
Ba. This is due to increases in 23. The alkali metals form salt-like hydrides by the
(a) ionization energy direct synthesis at elevated temperature. The
thermal stability of these hydrides decreases in
(b) melting point which of the following orders ?
(c) coordination number (a) CsH > RbH > KH > NaH > LiH
(d) All of these (b) KH > NaH > LiH > CsH > RbH
18. A chloride dissolves appreciably in cold water. (c) NaH > LiH > KH > RbH > CsH
When placed on a Pt wire in bunsen flame, no
distinctive colour is noted. Which cation could (d) LiH > NaH > KH > RbH > CsH
be present? 24. Which of the following is not correct about the
solution when moderate amount of sodium metal
(a) Be2+ (b) Ba2+ (c) Pb2+ (d) Ca2+ is dissolved in liquid ammonia at low temperature?
19. An unknown inorganic compound (X) loses its (a) Na+ ions are produced in solution
water of crystallization on heating and its (b) We get a blue coloured solution
aqueous solution gives the following reactions : (c) the solution acts as a good conductor of
electricity
(d) Liquid ammonia remains diamagnetic
The s-Block Elements 111
25. The order of solubility of lithium halides in non 32. Which of the following are found in biological
polar solvents follows the order fluids Na+, Mg2+, Ca2+, K+, Sr2+, Li+ and Ba2+?
(a) LiI > LiBr > LiCl > LiF (a) Mg2+, Ca2+, and Sr2+
(b) LiF > LiI > LiBr > LiCl
(b) Na2 +and K+
(c) Na+, K+, Mg2+and Ca2+
(c) LiCl > LiF > LiI > LiBr
(d) Sr+, Li and Ba2+
(d) LiBr > LiCl > LiF > LiI
33. Metallic magnesium is prepared by
26. Which of the following is most stable? (a) reduction of MgO by coke.
(a) Na3N (b) Li3N (b) electrolysis of aqueous solution of
(c) K3N (d) Rb3N Mg(NO3)2.
Heat Heat (c) displacement of Mg by iron from MgSO4
27. ZnCl 2 + NaHCO3 ¾¾¾®(A) ¾¾¾® solution.
(B) + (C) H 2O (B) + NaOH ® (D) (d) electrolysis of molten MgCl2.
34. A compound (A) is used in preparation of
Identify the compound (D) present in the
washing soda to recover ammonia in Solvay's
solution. process. When CO2 is bubbled through an
aqueous solution of (A), the solution turns milky.
(a) ZnCO3 (b) Zn(OH)2 It is used in white washing due to disinfectant
(c) ZnO (d) Na2ZnO2 nature. What is the chemical formula of A?
28. Which of the following has correct increasing
basic strength?
(a) MgO < BeO < CaO < BaO
(a) Ca(HCO3)2 (b) CaO
(b) BeO < MgO < CaO < BaO
(c) Ca(OH)2 (d) CaCO3
(c) BaO < CaO < MgO < BeO
35. BeF2 is soluble in water whereas fluorides of
(d) CaO < BaO < BeO < MgO other alkaline earth metals are insoluble because
29. Which of the following statement is false ? of
(a) Strontium decomposes water readily than (a) ionic nature of BeF2.
beryllium. (b) covalent nature of BeF2.
(b) Barium carbonate melts at a higher (c) greater hydration energy of Be2+ ion as
temperature than calcium carbonate. compared to its lattice energy.
(c) Barium hydroxide is more soluble in water (d) none of these.
than magnesium hydroxide. 36. Which category of salts of alkaline earth metals
is not found in solid sate, but found in solution
(d) Beryllium hydroxide is more basic than
state?
barium hydroxide. (a) Carbonates (b) Bicarbonates
30. Which of the following is the best method for (c) Hydroxides (d) Sulphates
preparation of BeF2? 37. Sodium carbonate solution in water is alkaline
(a) Reaction of Be with F2 due to
(b) Thermal decomposition of (NH4)2BeF4 (a) hydrolysis of Na+
(c) Reaction of Be with HF (b) hydrolysis of CO 32 -
(d) All of the above are equally effective (c) hydrolysis of both Na+ and CO 32 - ions
(d) None of these
31. Aqueous solution of a group 2 element is
38. When a crystal of caustic soda is exposed to air,
precipitated by adding Na 2CO 3, then this
a liquid layer is deposited because :
precipitate is tested on flame, no light in visible
(a) Crystal loses water
region is observed , this element can be
(b) Crystal absorbs moisture and CO2
(c) Crystal melts
(a) Ba (b) Mg (c) Ca (d) Sr (d) Crystal sublimes
EBD_7587
112 Chemistry Objective MCQs
39. Which of the following statements is correct for metals (A) and (B) form hydroxides which are
CsBr3? soluble in alkalis. (A) and (B) are
(a) It is a covalent compound. (a) Ba and Mg (b) Na and K
(b) It contains Cs3+ and Br – ions. (c) Mg and Ba (d) K and Rb
47. Among the given statements, the incorrect one
(c) It contains Cs+ and Br 3– ions
is
(d) It contains Cs+, and Br – and lattice Br2 (a) Be differs much from other alkali metals than
molecule. Li does from other alkali metals.
40. The pair of compounds which cannot exits (b) Be generally forms covalent compounds.
together in solution is (c) Be forms a ver y strong complex,
[Be(H2O)4]2+.
(a) NaHCO3 and NaOH (d) Be usually has more than four water of
(b) Na2CO3 and NaHCO3 crystallisation associated with it.
(c) Na2CO3 and NaOH 48. Calcium imide on hydrolysis gives gas (B) which
(d) NaHCO3 and NaCl on oxidation by bleaching powder gives gas (C).
Gas (C) on reaction with magnesium give
41. Na 2SO 3 and NaHCO 3 may be distinguished
compound (D) which on hydrolysis gives again
by treating their aqueous solution with : gas (B). Identify (B), (C) and (D).
(a) MgO (b) MgSO4 (a) NH3, N2, Mg3N2
(c) litmus solution. (d) dil. acid (b) N2,NH3, MgNH
42. Substance which absorbs CO2 and violently (c) N2, N2O5, Mg(NO3)2
reacts with H2O with sound is : (d) NH3, NO2, Mg(NO2)2
(a) H 2SO 4 (b) CaCO3 49. Which one is the correct statement with reference
(c) ZnO (d) CaO to solubility of MgSO4 in water?
43. When sulphur is heated with NaOH (aq). The (a) SO 42– ion mainly contributes towards
compounds formed are hydration energy.
(a) Na 2S + H 2O (b) Sizes of Mg2+ and SO42– are similar.
(b) Na 2SO 3 + H 2O (c) Hydration energy of MgSO4 is higher in
(c) Na 2S + Na 2S2 O3 + H 2 O comparison to its lattice energy.
(d) Ionic potential (charge/radius ratio) of Mg2+
(d) Na 2S2 O3 + H 2O is very low.
44. The metal X is prepared by the electrolysis of 50. A solid compound ‘X’ on heating gives CO2 gas
fused chloride. It reacts with hydrogen to form a and a residue. The residue mixed with water forms
colourless solid from which hydrogen is released ‘Y’. On passing an excess of CO2 through ‘Y’ in
on treatment with water. The metal is : water, a clear solution ‘Z’, is obtained. On boiling
(a) Ca (b) Al (c) Zn (d) Cu ‘Z’, compound ‘X’ is reformed. The compound
45. Acidified solution of sodium thiosulphate is ‘X’ is
unstable because in thiosulphate

(a) Ca(HCO3)2 (b) CaCO3


(c) Na2CO3 (d) K2CO3
(a) the sulphur atoms are at unstable oxidation 51. Amongst the following hydroxides, the one which
state of +2 has the lowest value of K sp is :
(b) the two sulphur atoms are at different (a) Ca(OH)2 (b) Mg(OH)2
oxidation states of + 6 and – 2 (c) Be(OH)2 (d) Ba(OH)2
(c) the S-S bond are unstable bonds. 52. For which one of the following minerals, the
(d) sulphur is in zero oxidation state. composition given is incorrect?
46. Two metals (A) and (B) belong to the same group (a) Soda ash — (Na2CO3)
of the periodic table. Metal (A) forms an insoluble (b) Carnallite — (KCl . MgCl2 . 6H2O)
oxide but a soluble sulphate, metal (B) forms a (c) Borax — (Na2B4O7 . 7H2O)
soluble oxide but an insoluble sulphate. Both (d) Glauber's salt — (Na2SO4 . 10H2O)
The s-Block Elements 113
53. The difference of number of water molecules in 57. Mg2C3 reacts with water forming propyne gas.
gypsum and plaster of Paris is C34 - ions has:
(a) two sigma and two pi bonds
5 1
(b) three sigma and one pi bond
(a) (b) 2 (c) 1/2 (d) 1 (c) two sigma and one pi bond
1 2
(d) two sigma and three pi bonds
54. Which is not obtained when metal carbides react
with H2O? 58. If NaOH is added to an aqueous solution of
Zn 2 + ions, a white precipitate appears and on
(a) Al 4C3 + H 2O ¾¾ ® CH º CH
adding excess NaOH, the precipitate dissolves.
(b) CaC2 + H 2O ¾¾ ® CH º CH In this solution zinc exists in the :
(c) Mg 4C3 + H 2O ¾¾ ® CH3C º CH (a) both in cationic and anionic parts
(d) Be 2C + H 2 O ¾¾ ® CH 4 (b) there is no zinc left in the solution
55. CaCl2 is preferred over NaCl for clearing ice on (c) cationic part
roads particularly in very cold countries. This is (d) anionic part.
because:
(a) CaCl2 is less soluble in H2O than NaCl 59. Which is incorrect statement?
(b) CaCl2 is hygroscopic but NaCl is not
(c) Eutectic mixture of CaCl2/H2O freezes at (a) The heats of hydration of the dipositive
–55°C while that of NaCl/H2O freezes at alkaline earth metal decrease with an
–18°C increase in their ionic size
(d) NaCl makes the road slipperty but CaCl2 (b) NaNO3 forms Na2O2 on heating
does not (c) Hydration of alkali metal ion is less than
56. Which of the following statement about the that of IIA group
sulphate of alkali metal is correct? (d) Alkaline earth metal ions, because of their
(a) Except Li2SO4 all sulphate of other alkali much larger charge to size ratio, exert a much
metals are soluble in water stronger electrostatic attraction on the
(b) All sulphate salts of alkali metals except oxygen of water molecule surrounding
lithium sulphate forms alum. them
(c) All sulphate salts of alkali metals except 60. Which of the following does not form an oxide
lithium sulphate do not decompose at high on heating?
temperature (a) ZnCO3 (b) CaCO3
(d) All of the above (c) Li2CO3 (d) Na2CO3

Answer KEY
1 (c) 7 (a) 13 (c) 19 (b) 25 (a) 31 (b) 37 (b) 43 (c) 49 (c) 55 (c)
2 (c) 8 (d) 14 (d) 20 (d) 26 (b) 32 (c) 38 (b) 44 (a) 50 (b) 56 (d)
3 (b) 9 (a) 15 (b) 21 (a) 27 (d) 33 (d) 39 (c) 45 (b) 51 (c) 57 (a)
4 (b) 10 (c) 16 (b) 22 (a) 28 (b) 34 (c) 40 (a) 46 (c) 52 (c) 58 (d)
5 (a) 11 (d) 17 (c) 23 (d) 29 (d) 35 (c) 41 (b) 47 (d) 53 (d) 59 (b)
6 (a) 12 (d) 18 (a) 24 (d) 30 (b) 36 (b) 42 (d) 48 (a) 54 (a) 60 (d)
EBD_7587
114 Chemistry Objective MCQs

1. (c) As the size of halide ion decreases, I– > Br– > Since the lattice enthalpy of alkali metals
Cl– > F–, tendency to get polarised by cation follow the order
M+ decreases and hence covalent character Li > Na > K > Rb
decreases. Hence the correct order of solubility is
2. (c) Down the group with decrease in positive LiF < NaF < KF < RbF
charge tendency to get hydrated decreases 9. (a) NaCl due it to its largest lattice energy
and hence the size of hydrated ion has the highest melting point.
decreases. Thus mobility increases. 10. (c) As Cs+ ion has larger size than Li+ and I–
3. (b) On heating with excess of air Li, Na and K has larger size than F–, therefore maximum
forms following oxides distance between centres of cations and
4Li + O 2 ¾¾
® 2Li 2O anions is in CsI.
Lithium monoxide 11. (d) Na 2O2 is peroxide of sodium not super
575K oxide. The formula of sodium superoxide is
4Na + O2 ¾¾¾® Na 2O 2 NaO2.
Sodium peroxide
12. (d) Stability of ionic compounds decreases
K + O2 ¾¾
® KO2 with decrease in lattice enthalpy. Thus
Potassium superoxide stability of alkaline earth metal hydroxides
decreases with decrease in lattice enthalpy
4. (b) O 24 - ion is not possible and K 2O4 is
as the size of alkali earth metal cations
unknown. increases down the group.
5. (a) Only nitrates of heavy metals and lithium 13. (c) Due to high hydration energy of Li+ cation,
decompose on heating to produce NO 2 . the standard reduction potential of Li+ is
6. (a) The crystal structures of NaHCO3 and more negative among all alkali metal cations
KHCO3 both show hydrogen bonding but hence Li act as strong reducing agent in
are different. In NaHCO3, the HCO3– ions water.
are linked into an infinite chain, while in 14. (d) Amongst alkali metal ions, Li ions are highly
KHCO3, a dimeric anion is formed. hydrated.
7. (a) Basic strength of the oxides increases in 15. (b) Na 2 CO 3 + 2SO 2 + H 2 O ® 2NaHSO 3 + CO 2
the order (A)

Li2O < Na2O < K2O < Rb2O < Cs2O. The 2NaHSO3 + Na 2 CO3 ® 2Na 2SO 3 + H 2O + CO 2
increase in basic strength is due to the (A) (B)
decrease in I.E. and increase in
electropositive character. Heat
2Na 2SO3 + S ¾¾¾
® 2Na 2S2 O3
The melting points of the halides decrease (B) (C)
in the order NaF > NaCl > NaBr > NaI, as the Na 2S2 O 3 + AgNO3 ® Ag 2S+ H 2SO 4
size of the halide ion increases. The decrease (D)
in melting point is due to increase in the
16. (b) Actually Li is harder than other alkali metals.
covalent character with increase in the size
of anion according to Fajan's rule. 17. (c) In crystalline sulphates, the coordination
8. (d) Higher the lattice enthalpy lower will be number of the ions increases from Be to Ba
solubility i.e., and so also the lattice energy hence
solubility in H2O decreases.
1
lattice enthalpy µ
Solubility
The s-Block Elements 115
18. (a) Due to very high I.E. of Be, its valence 29. (d) Be(OH)2 is amphoteric, but the hydroxides
electrons require high energy to remove of other alkaline earth metals are basic. The
(excite) which is not available in bunsen basic strength increases gradually.
flame hence Be2+ ions do not impart colour 30. (b) Thermal decomposition of (NH4)2BeF4 is
to the flame. the best method for preparation of BeF2.
19. (b) X = Na2S2O3.5H2O 31. (b) Electrons in Mg due to its small size are
Na 2S2 O 3 + 2HCl ¾¾
® tightly bound so they cannot be excited by
the flame.
2NaCl + H 2 O + S + SO 2
32. (c) Monovalent sodium and potassium ions
KI3 + 2Na 2S2 O 3 ¾¾
®
and divalent magnesium and calcium ions
KI + 2NaI + Na 2S4O6 are found in large proportions in biological
2AgNO 3 + Na 2S2 O 3 ¾¾® fluids.
Ag 2S2 O3 + 2NaNO3 Electrolysis
33. (d) MgCl2 ¾¾¾¾¾® Mg +2 + 2Cl-
Ag 2 S 2 O 3 + H 2 O ¾
¾® Ag 2 S + H 2 SO 4 (Molten) Cation Anion
20. (d) Beryllium shows anomalous properties due Anode ® 2Cl–
® 2Cl + 2e–, Cl + Cl ® Cl2
to its small size.
Cathode ® Mg + 2e– ® Mg
+2
21. (a) As the size of the anion increases, the
covalent character increases and hence the NH Cl
34. (c) NH3­ ¾¾¾¾
4 ® Ca(OH)
2
Solvay 's
m.p. decrease. process (A)
22. (a) Lithium does not form peroxide. CO
2 ® CaCO
¾¾¾ 3
23. (d) The stability of alkali metal hydrides
decreases from Li to Cs. It is due to the fact 35. (c) Be2+ being small in size is heavily hydrated
that M–H bonds become weaker with and heat of hydration exceeds the lattice
increase in size of alkali metals as we move energy. Hence BeF2 is soluble in water.
down the group from Li to Cs. Thus the 36. (b) The bicarbonates of alkaline earth metals
order of stability of hydrides is exist in solution only.
LiH > NaH > KH > RbH > CsH 37. (b) Due to anionic hydrolysis
24. (d) Liquid ammonia remains paramagnetic CO23 - + H 2 O ƒ HCO3– + -OH
25. (a) As the size of the anion increases from F– to 38. (b) It is hygroscopic and deliquescent. It
I–, the covalent character increase and hence absorbs moisture and CO 2 to form
the solubility in non -polar solvent increases Na 2CO 3
LiI > LiBr > LiCl > LiF 2NaOH + CO 2 ¾ ¾® Na 2 CO 3 + H 2 O
26. (b) Only Li3N is stable, others are not formed at 39. (c) CsBr3 may be represented as Cs+Br3–
all.
40. (a) NaHCO3 + NaOH ® Na 2 CO3 + H 2 O
27. (d) D
ZnCl2 + 2NaHCO3 ¾¾® (NaHCO3 is an acidic salt)
Zn ( HCO3 )2 + 2NaCl 41. (b) 2Mg SO 4 + 2 Na 2CO 3 + H 2 O ¾ ¾®
(A) Mg CO 3 .Mg (OH) 2 + 2 Na 2SO 4 + CO 2
D
Zn ( HCO3 )2 ¾¾® Basic magnesium carbonate
(A) 42. (d) CaO + CO 2 ¾ ¾® CaCO3
ZnCO3 + H 2O + CO2 ­ CaO + H 2 O ¾ ¾® Ca(OH)2
(B) (C)
hissing sound and DH = – ve
ZnCO3 + 2NaOH ® Na 2 ZnO2 + 2H 2 O
(D)
43. (c) 4S + 6NaOH ¾¾
®
(B)
28. (b) The basic character of oxides increases Na 2S2O3 + 2 Na 2S + 3H 2O
down the group.
EBD_7587
116 Chemistry Objective MCQs
heat
44. (a) Ca + H 2 ¾¾
® CaH 2 + 2H 2O CaCO3 ¾¾¾®CO2 + CaO
'X ' Residue
¾¾
® Ca(OH)2 + H 2 boil

45. (b) Na 2S2 O3 + 2HCl ¾¾


® H 2O
(Disproportionation)
excess
Ca(HCO3 ) 2 ¬¾¾ ¾ Ca(OH)2
2NaCl + SO 2 + S + H 2O CO 2
'Z' 'Y '
46. (c) MgO is insoluble, whereas MgSO 4 is 51. (c) Be(OH)2 is insoluble in water and thus has
soluble, whereas BaO is soluble, but BaSO4 lower value of Ksp.
is insoluble.
47. (d) Hydrated Be2+ ion exists as [Be (H2O)4]2+. 52. (c) Barax is Na2B4O7. 1OH2O.
53. (d) Gypsum is CaSO4. 2H2O, where as plaster
48. (a) Ca(NH) + 2H 2O ¾¾
®
1
of paris is CaSO4 2 H2O.
Ca(OH) 2 + NH3 (g)
(B)
æ 1ö 3 1
2NH3 + 3CaOCl 2 ¾¾
® No. of water = ç 2 - ÷ ¾¾
® or 1
(B) è 2ø 2 2
N 2 (g) + 3CaCl 2 + 3H 2 O molecules
(C) 54. (a) Al4C3 + 12H2O ¾® 4Al(OH)3 + 3CH4
N 2 (g)+ 3Mg ¾¾
® Mg3 N 2 55. (c) Due to much lower freezing point of eutectic
(C) (D)
mixture of CaCl2/H2O.
Mg3 N 2 + 6H 2O ¾¾
® 3Mg(OH) 2 + NH3 56. (d) All are facts
(D) (B)
–2 –2
49. (c) MgSO4 is the only alkaline earth metal 57. (a) The structure of C34 - ion is C = C = C
sulphate which is soluble in water and for
solubility hydration energy should be i.e, 2s 2p bonds.
greater than lattice energy. 58. (d) Zn 2 + + 2NaOH ¾ ¾®
+
Zn (OH) 2 + 2 Na
50. (b) heat
X ¾¾® CO2 + Residue Zn (OH ) 2 + 2 NaOH ¾ ¾®
Solid
H2O
Na 2 ZnO 2 + 2H 2 O
boil
excess CO2 Na 2 ZnO2 ¾ ¾® 2 Na + + ( ZnO 2 ) 2 -
Z ¬¾¾¾¾¾ Y
Clear solution D
59. (b) NaNO3 ¾¾
® NaNO2 + O 2
The given properties coincide with CaCO3 60. (d) Na2CO3 does not decompose to form Na2O.
The p-Block Elements
11 (Group 13 and Group 14)
1. Which of the following hydroxide is acidic? HCl(aq.)
(a) Al(OH)3 (b) Ca(OH)3 'X' + Gas 'P'
(c) Tl(OH)3 (d) B(OH)3 6. Al
Metal
2. Al is more reactive than Fe but Al is less easily NaOH(aq.)
'Y' + Gas 'Q'
corroded than Fe because +H 2O
(a) it is a noble metal The incorrect statement regarding above
(b) oxygen forms a protective oxide layer reactions is:
(c) iron undergoes reaction easily with water
(d) Fe form mono and divalent ions. (a) Al shows amphoteric character
3. Often a ground glass stopper gets stuck in the (b) Gas 'P' and 'Q' are different
neck of a glass bottle containing NaOH solution. (c) Both X and Y are water soluble
This is due to : (d) Gas Q is inflammable
(a) The presence of dirt particles in between. 7. Ge(II)compounds are powerful reducing agents
(b) The formation of solid silicate in between whereas Pb(IV)compounds are strong oxidants.
by the reaction of SiO2 of glass with NaOH. It is because
(c) The formation of Na2CO3 in between by (a) Pb is more electropositive than Ge
the reaction of CO2 of air and NaOH. (b) ionization potential of lead is less than that
(d) Glass contains a boron compound which of Ge
forms a precipitate with the NaOH solution. (c) ionic radii of Pb2+ and Pb4+ are larger than
those of Ge2+ and Ge4+
4. (Si2O5 )2n–
n
anion is obtained when: (d) of more pronounced inert pair effect in lead
than in Ge
8. Which of the following conceivable structures
for CCl4 will have a zero dipole moment?
(a) no oxygen of a SiO44-tetrahedron is shared (a) Square planar
with another SiO44- tetrahedron (b) Square pyramid (carbon at apex)
(c) Irregular tetrahedron
(b) one oxygen of a SiO44- tetrahedron is (d) None of these
shared with another SiO44- tetrahedron 9. Least thermally stable is –
(c) two oxygen of a SiO44-tetrahedron are (a) CCl4 (b) SiCl4 (c) GeCl4 (d) GeBr4
shared with another SiO44-tetrahedron 10. To a piece of charcoal, sulphuric acid is added.
Then:
(d) three oxygen of a SiO44- tetrahedron are
(a) there is no reaction
shared with another SiO44- tetrahedron (b) water gas is formed
5. Boric acid is a weak monobasic acid and acts as (c) SO2 and CO2 are evolved
Lewis acid (d) CO and SO2 are evolved
(a) By donating H+ 11. The correct statement with respect to CO is
(b) By accepting OH– (a) it combines with H2O to give carbonic acid
(c) By donating lone pair of electrons (b) it reacts with haemoglobin in RBC
(d) By accepting lone pair of electrons. (c) it is powerful oxidising agent
(d) it is used to prepare aerated drinks
EBD_7587
118 Chemistry Objective MCQs

12. CO2 and N2 are non-supporters of combustion. (iii) d-orbitals contribute more to the overall
However for putting out fires CO2 is preferred stability of molecules than pp-pp bonding
of second row elements.
over N2 because CO2
(a) (i) (ii) (iii) (b) (i) (iii)
(a) does not burn (c) (i) (ii) (d) (ii) (iii)
(b) forms non-combustible products with
19. An aqueous solution of FeSO4, Al2(SO4)3 and
burning substances chrome alum is heated with excess of Na2O2 and
(c) is denser than nitrogen
filtered. The materials obtained are :
(d) is a more reactive gas
13. Which of the following is not correct?
(a) Ge(OH)2 is amphoteric
(b) GeCl2 is more stable than GeCl4 (a) a colourless filtrate and a green residue
(c) GeO2 is weakly acidic (b) a yellow filtrate and a green residue
(d) GeCl4 in HCl forms [ GeCl2]2– ion (c) a yellow filtrate and a brown residue
14. Which of the following is similar to graphite? (d) a green filtrate and a brown residue
(a) B (b) BN (c) B2 H 6 (d) B4C 20. Which statement is not true about potash alum?
15. In the following sets of reactants which two sets (a) On heating it melts and loses its water of
best exhibit the amphoteric characters of crystallization.
(b) It’s aqueous solution is basic in nature.
Al2O3. xH2O?
(c) It is used in dyeing industries.
Set 1: Al2O3.xH2O (s) and OH– (aq) (d) It’s empirical formula is KAl(SO4 )2 .12H 2O.
Set 2: Al2O3.xH2O (s) and H2O (l) 21. Anhydrous aluminium chloride (Al 2Cl 6) is
Set 3: Al2O3.xH2O (s) and H+ (aq) covalent compound and soluble in water giving:
Set 4: Al2O3.xH2O (s) and NH3 (aq) (a) Al3+ and Cl– ions
(a) 1 and 2 (b) 1 and 3 (b) [Al(H2O)6]3+ and Cl– ions
(c) 2 and 4 (d) 3 and 4 (c) [AlCl2 (H2O)4]+ and [AlCl4 (H2O)2]– ions
(d) none of the above
16. The gas evolved on heating CaF2 and SiO2 with
22. Borax is converted into crystalline boron by the
concentrated H2SO4, on hydrolysis gives a white
gelatinous precipitate. The precipitate is: following steps:
(a) hydrofluosilicic acid
(b) silica gel X D Y
(c) silicic acid Borax ¾¾® H3BO3 ¾¾® B2 O3 ¾¾® B
D
(d) calciumfluorosilicate X and Y are respectively:
17. On adding ammonium hydroxide solution to (a) HCl, Mg (b) HCl, C
Al2(SO4)3 (aq): (c) C, Al (d) HCl, Al
23. On controlled hydrolysis and condensation,
(a) A precipitate is formed which does not R3SiCl yields
dissolve in excess of ammonium hydroxide (a) R3Si – O – SiR3
(b) A precipitate is formed which dissolves in
excess of ammonia solution (b) -(R3Si – O – SiR3)- n
(c) No precipitate is formed (c) R3SiOH
(d) None of these
18. Which of the following statements are correct? R R
(i) Aluminium forms [AlF6]3– ion while boron | |
forms only [BF4]– ion due to presence of – Si – O – Si –
| |
d-orbitals in aluminium. (d) O O
(ii) The first member of a group differs from | |
the heavier members in its ability to form – Si – O – Si –
pp-pp multiple bonds to itself and to other | |
second row elements. While heavier R R
member forms dp-pp bonds.
The p-Block Elements (Group 13 and Group 14) 119
24. B(OH)3 + NaOH ® NaBO2 + Na[B(OH)4] + H2O (b) Trialkylchlorosilane on hydrolysis gives
How can this reaction is made to proceed in R3SiOH.
forward direction? (c) SiCl4 undergoes hydrolysis to give H4SiO4.
(a) addition of cis 1, 2-diol (d) (Si3O9)6– has cyclic structure.
(b) addition of borax 31. Aluminium chloride exists as dimer, Al2Cl6 in solid
(c) addition of trans 1, 2-diol state as well as in solution of non-polar solvents
(d) addition of Na2HPO4 such as benzene. When dissolved in water, it
25. Anhydrous AlCl3 cannot be obtained from which gives
of the following reactions?
(a) [Al(OH) 6 ]3- + 3HCl
(a) Heating AlCl3.6H2O (b) [Al(H 2 O) 6 ]3+ + 3Cl -
(b) By passing dry HCl over hot aluminium
powder (c) Al 3+ + 3Cl -
(c) By passing dry Cl2 over hot aluminium
powder (d) Al 2 O3 + 6HCl
(d) By passing dry Cl2 over a hot mixture of 32. When Al is added to NaOH solution
alumina and coke (a) No action takes place
26. The role of fluorspar ( CaF2) which is added in (b) NaAlO2 is formed and H2 is evolved
small quantities in the electrolytic reduction of (c) Al(OH)3 is formed and H2 is evolved
alumina dissolved in fused cryolite (Na3AlF6) is (d) Na2AlO2 is formed and H2 is evolved
(a) as a catalyst 33. Which of the following statement(s) is / are
(b) to make the fused mixture very conducting incorrect for CO2?
(c) to increase the temperature of the melt. (i) In laboratory CO2 is prepared by the
(d) to decrease the rate of oxidation of carbon action of dilute HCl on calcium carbonate
at the anode. (ii) Carbon dioxide is a poisonous gas
27. The correct order of increasing C – O bond length
(iii) Increase in carbon dioxide content in
of CO, CO2 and CO32 - is: atmosphere lead to increase in green
(a) CO32 -< CO2< CO (b) CO2 < CO32 -< CO house effect.
(iv) CO2 as dry ice is used as a refrigerant for
(c) CO < CO32 -< CO2 (d) CO < CO2 < CO32 - ice cream and frozen food.
28. In silicon dioxide (a) (i) and (ii) (b) Only (ii)
(a) there are double bonds between silicon and
(c) (i), (ii) and (iii) (d) (ii) and (iii)
oxygen atoms
(b) silicon atom is bonded to two oxygen atoms 34. A group 14 element is oxidised to form
(c) each silicon atom is surrounded by two corresponding oxide which is gaseous in nature,
oxygen atoms and each oxygen atom is when dissolved in water pH of the water
bonded to two silicon atoms decreases further addition of group 2 hydroxides
(d) each silicon atom is surrounded by four leads to precipitation. This oxide can be
oxygen atoms and each oxygen atom is
bonded to two silicon atoms.
29. Example of a three-dimensional silicate is: (a) GeO2 (b) CO (c) CO2 (d) SnO2
(a) Zeolites (b) Ultramarines 35. PbF4, PbCl4 exist but PbBr 4 and PbI4 do not exist
(c) Feldspars (d) Beryls because of
(a) large size of Br– and I–
30. Identify the incorrect statement :
(b) strong oxidising character of Pb4+
(a) In (Si3O9)6–, tetrahedral SiO4 units share two (c) strong reducing character of Pb4+
oxygen atoms. (d) low electronegativity of Br – and I–.
EBD_7587
120 Chemistry Objective MCQs

36. Which one of the following is a correct set for 43. AlI3, when reacts with CCl4, gives
SiO2? (a) AlCl3 (b) CI4
(a) Linear acidic (c) Al4C3 (d) both (a) and (b)
(b) Linear,basic 44. Which of the following properties of aluminium
(c) Tetrahedral, acidic makes it useful for food packaging ?
(d) Angular, basic. (a) Good electrical conductivity
37. Which is not the use of orthoboric acid? (b) Good thermal conductivity
(a) As an antiseptic and eye wash. (c) Low density
(b) In glass industry. (d) Non toxicity
(c) In glazes for pottery. 45. A metal, M forms chlorides in its +2 and +4
(d) In borax - bead test. oxidation states. Which of the following
statements about these chlorides is correct?
38. The factor responsible for weak acidic nature of
(a) MCl2 is more ionic than MCl4
B–F bonds in BF3 is
(b) MCl2 is more easily hydrolysed than MCl4
(a) large electronegativity of fluorine (c) MCl2 is more volatile than MCl 4
(b) three centred two electron bonds in BF3 (d) MCl2 is more soluble in anhydrous ethanol
(c) pp - dp back bonding than MCl4
(d) pp - pp back bonding 46. Among the following substituted silanes the one
39. Which of the following has the minimum heat of which will give rise to cross linked silicone
dissociation: polymer on hydrolysis is
(a) (CH3 )3 N : ® BF3
(b) (CH3 )3 N : ® B(CH3 )2 F (a) R4Si (b) R2SiCl2
(c) RSiCl3 (d) R3SiCl
(c) (CH3 )3 N : ® B(CH 3 ) 3 47. Which of the following statements is false?
(d) (CH3 )3 N : ® B(CH 3 )F2 (a) Water gas is a mixture of hydrogen and
carbon monoxide
40. Lead is not affected by dil. HCl in cold because
(b) Producer gas is a mixture of CO and nitrogen
(a) Pb is less electronegative than H
(c) Water gas is a mixture of water vapour and
(b) PbO film is formed which resists chemical
hydrogen
attack by acid
(d) Natural gas consists of methane, ethane
(c) PbCl2 protective coating gets formed on Pb and gaseous hydrocarbons.
surface 48. Lead pipes are not suitable for drinking water
(d) PbO2 film is always present on Pb surface, because
which resist chemical attack (a) lead forms basic lead carbonate
41. The catenation tendency of C, Si and Ge is in (b) lead reacts with water containing air to form
the order Ge < Si < C. The bond energies Pb(OH)2
(in kJ mol– l) of C – C, Si – Si and Ge –Ge bonds
(c) a layer of lead dioxide is deposited over
are respectively pipes
(d) lead reacts with air to form litharge
(a) 348, 297, 260 (b) 297, 348, 260 49. Choose the correct sequence for the geometry
of the given molecules
(c) 348, 260, 297 (d) 260, 297, 348
42. The melting pt. of group 13 follows the order Borazone, Borazole, B3O36-
['P' stands for planar and 'N' stands for non-
(a) B > Al > Ga > In > Tl
(b) B > Al < Ga > In > Tl planar]
(c) B > Al > Tl > In > Ga
(a) NP, NP, NP (b) P, P, NP
(d) B > Al < Ga < In < Tl (c) NP, P, NP (d) NP, P, P
The p-Block Elements (Group 13 and Group 14) 121
50. Which out of the following compounds does (a) X is a non-metal and belongs to the third
not exist? period while Y is a metal and belongs to the
(a) BF3 (b) TlCl3 fourth period.
(c) TlCl5 (d) Both (b) and (c) (b) One of two oxides has three-dimensional
51. Anhydrous AlCl3 is prepared from polymeric structure.
(c) Both (a) and (b) are correct.
(d) None of the above.
(a) conc. HCl and Al metal 56. A solid element (symbol Y) conducts electricity
(b) aluminium and Cl2 and forms two chlorides YCln (colourless
(c) dry HCl gas + heated Al metal
volatile liquid) and YCl n - 2 (a colourless solid).
(d) dil. HCl and Al metal
52. Boric acid is polymeric due to To which one of the following groups of the
(a) its acidic nature periodic table does Y belong?
(b) the presence of hydrogen bonds (a) 13 (b) 14 (c) 15 (d) 16
(c) its monobasic nature 57. The structure and hybridization of Si(CH3)4 is
(d) its geometry (a) Bent, sp (b) Trigonal, sp2
53. Al2O3 can be converted to anhydrous AlCl3 by (c) Octahedral, d sp (d) Tetrahedral, sp3
2 3

58. When steam reacts with red hot coke to form


heating CO2 and hydrogen :
(a) Water acts as an oxidising agent.
(a) Al2O3 with NaCl in solid state
(b) Water acts as a reducing agent.
(b) a mixture of Al2O3 and carbon in dry Cl2 gas (c) Carbon acts as an oxidising agent.
(c) Al2O3 with Cl2 gas (d) There is no oxidation or reduction.
(d) Al2O3 with HCl gas 59. Which of the following is/are not correctly
54. In which of the following, a salt of the type matched?
KMO2 is obtained? (i) GeO2 – Acidic
(a) B2H6 + KOH(aq) ® (ii) PbO2– Amphoteric
(b) Al + KOH(aq) ® (iii) CO – Neutral
(c) Both
(d) None (iv) SiO2 – Amphoteric
55. Bauxite ore is generally contaminated with (a) (i) and (iv) (b) (iv) only
impurity of oxides of two elements X and Y. (c) (ii) only (d) (iii) only
Which of the following statement is correct? 60. The shape of gaseous SnCl2 is
(a) tetrahedral (b) linear
(c) angular (d) T-shaped

Answer KEY
1 (d) 7 (d) 13 (b) 19 (c) 25 (a) 31 (b ) 37 (d) 43 (d) 49 (d) 55 (c)
2 (b) 8 (d) 14 (b) 20 (b) 26 (b ) 32 (b ) 38 (d) 44 (c) 50 (c) 56 (b)
3 (b) 9 (d) 15 (b) 21 (c) 27 (d ) 33 (b ) 39 (c) 45 (a) 51 (c) 57 (d)
4 (d) 10 (c) 16 (d) 22 (d) 28 (d ) 34 (c) 40 (c) 46 (c) 52 (b) 58 (a)
5 (b) 11 (b) 17 (a) 23 (a) 29 (c) 35 (b ) 41 (a) 47 (a) 53 (b) 59 (b)
6 (b) 12 (c) 18 (c) 24 (a) 30 (b ) 36 (c) 42 (c) 48 (b) 54 (c) 60 (c)
EBD_7587
122 Chemistry Objective MCQs

1. (d) B(OH)3 is acid because it can take OH– ions. 9. (d) The thermal stability of tetrahalides
decreases in order
H3BO3 or B(OH)3 + OH– ® B(OH)4–
CX4 > SiX4 > GeX4 > SnX4 and in terms of
2. (b) The protective oxide layer prevents Al from same metal with different halides is in order
further corrosion. of MF4 > MCl4 > MBr4 > MI4.
3. (b) SiO 2 + 2NaOH ® Na 2SiO3 + H 2O 10. (c) Charcoal is a pure form of carbon, its
Sodium reaction with hot conc. H2SO4 is as follows:
silicate C + 2H2SO4 ¾¾ ↑ 2H2O + 2SO2 + CO2
11. (b) CO react with haemoglobin, forms carboxy
4. (d) haemoglobin and stopes the supply of O2
12. (c) CO2 being more dense covers the igniting
material more effectively than N2.
13. (b) Ge4 + is more stable than Ge2+. Hence
GeCl 4 is more stable than GeCl 2
14. (b) Graphite and boron nitride have similar
structure.
15. (b) Aluminium oxide is amphoteric oxide
because it shows the properties of the both
acidic and basic oxides. It reacts with both
acids and bases to form salt and water.
Al2O3. xH2O + 2NaOH ¾¾ ®
NaAlO2 + H2O
Total No. oxygen atoms per silicon atom Al2O3. xH2O + HCl ¾¾ ® AlCl3 + H2O
1 1 1 16. (d)
= + + + 1 = 2.5 2CaF2 + SiO 2 + H 2 SO 4 ¾¾
®
2 2 2
hydrolysis
SiF4 + H 2 O + CaSO 4 ¾¾¾¾¾
®
\ Formula Si 2 O52 - .
CaSiF4
-
5. ˆˆ† é B( OH ) ù + H3O +
(b) B (OH )3 + 2H 2O ‡ˆˆ ë 4û 17. (a) Al 2 (SO 4 )3 + 6NH 4 OH ¾¾
®
6. (b) 2Al(OH)3 + 3(NH 4 )2 SO 4
HCl(aq.)
AlCl3 + H2(g)
¯ ® Na + [Al(OH) 4 ]-
Al(OH)3 + NaOH ¾¾
Al (X) (P) Soluble complex
Metal
NaOH(aq.)
Na[Al(OH)4] + H2(g)
¯ It is insoluble in NH4OH.
+H2O
(Y) (Q) 18. (c) d-orbitals are of higher energy than the p
-orbitals, they contribute less to the overall
7. (d) Ge(II) tends to acquire Ge (IV) state by loss
stability of molecules than pp-pp bonding
of electrons. Hence it is reducing in nature.
of the second row elements.
Pb (IV) tends to acquire Pb (II) O.S. by gain
of electrons. Hence it is oxidising in nature. 19. (c) The filtrate is yellow due to CrO24 - ion and
This is due to inert pair effect. residue is brown due to Fe(OH)3.
8. (d) CCl 4 is tetrahedral in nature.
The p-Block Elements (Group 13 and Group 14) 123
20. (b) Alum form acidic solution due to hydrolysis 28. (d) In SiO2 (quartz), each of O-atom is shared
of Al 3+ . between two SiO44– tetrahedra.
21. (c) dissociation
AlCl3 .6H 2 O ¾¾¾¾¾®
+ -
éë AlCl2 ( H 2 O )4 ùû + éë AlCl4 ( H 2O )2 ùû
HCl
22. (d) Borax ¾¾¾
® H 3 BO 3
Al
B2O3 ¾¾® B(crystalline)
D 29. (c) The feldspars are most abundant
Mg aluminosilicate minerals in the Earth surface.
B2O3 ¾¾¾ ® B(Amorphous) The silicon atoms and aluminium atoms
D
23. (a) R3SiCl on hydrolysis forms only a dimer. occupy the centres of interlinked tetrahedra
R3SiOH + HOSiR3 ® R3Si – O – SiR3. of SiO44 - and AlO54- . These tetrahedra
24. (a) H3BO3 acts as weak monobasic Lewis acid. connect at each corner to other tetrahedra
B(OH)3 + NaOH ® Na[B(OH)4] forming an intricate, three dimesional,
On addition of cis-1, 2-diol in H3BO3 negatively charged framework. The sodium
solution, acidic strength of H 3 BO 3 cations sit within the voids in this structure.
increases due to chelation effect. 30. (b) The hydrolysis of Trialkylchlorosilane
25. (a) D , air R3 SiCl yields dimer :
2Al + 6HCl ¾¾¾® 2AlCl3 + 3H 2
R R
2Al + 3Cl 2 ¾¾
® 2AlCl3
1000 °C
R Si O Si R
Al 2O3 + 3C + 3Cl2 ¾¾¾¾
® 2AlCl3 + 3CO
vapours R R
31. (b) ˆˆ†
Al2 Cl6 + 12H 2O ‡ˆˆ
Cooling
3+ -
Anhydrous AlCl3 2[Al(H 2O)6 ] + 6Cl

D 32. (b) Al + NaOH + H 2 O ® NaAlO 2 + H 2 ­ .


AlCl3 .6H 2O ¾¾® Al(OH)3 + 3HCl + 3H 2O
Thus AlCl 3 cannot be obtained by this 33. (b) Carbon dioxide is not a poisonous gas.
method. 34. (c) CO2 forms carbonic acid H2CO3, when
26. (b) CaF2 when added to fused cryolite, lowers dissolved in water, CO is neutral, whereas
the m.p. and increases the conductivity. other two GeO2 and SnO2 are solids.
27. (d) Structures of CO, CO2 and CO32 - are : 35. (b) F and Cl are more oxidising in nature and
.. can achieve Pb in (IV) O.S. but Br2 and I 2
C O O=C=O
can not achieve Pb in (IV) O.S. secondly
O O O Pb 4 + is strong in oxidising nature and in
C C C its presence, Br - and I - can not exist.
O O O O O O 36. (c) SiO2 being oxide of non metal is acidic in
nature and silicon is bonded to O atoms
Compound CO CO2 CO3 tetrahedrally. It has giant structure.
Bond order 3 2 1.33 37. (d) Borax on heating gives B2O3 and NaBO2
which is glassy mass and used for borax-
1 bead test.
Bond order µ
Bond length 38. (d) It is pp - pp back bonding involving B and
Hence, the decreasing (C — O) bond length is : F. The smaller atoms show more back
bonding.
CO < CO2 < CO32 -
EBD_7587
124 Chemistry Objective MCQs

39. (c) Due to + I effect of methyl groups the Lewis –


character of B(CH 3 ) 3 decreases and O
coordination becomes weaker. B
40. (c) Pb with dil. HCl forms protective coating of O O
PbCl2. B3O63– : planar
41. (a) The linking of identical atoms with each –
B –
B
O O O
other to form long chains is called
catenation. However, this property 50. (c) Because TI+5 does not exist
D
decreases from carbon to lead. Decrease of 51. (c) 2Al + 3Cl2 ¾¾® 2AlCl3 (anhydrous)
this property is associated with M-M bond 52. (b) In Boric acid each B atom is sp2 hybridized
energy which decreases from carbon to lead.
42. (c) Due to structural changes, melting point, and contains BO 33- units which are held
increases from Ga to Tl and Ga has the together by hydrogen bonds.
lowest melting point. 53. (b) Al2O3 can be converted to anhydrous AlCl3
43. (d) AlI3, on reaction with CCl4, gives the AlCl3 by heating a mixture of Al2O3 and carbon in
dry Cl2
4AlI3 + 3CCl4 ¾¾ ® 4AlCl3 + 3CI 4
44. (c) Due to the low density of aluminium, it is 1000°C
Al2O3 + 3C + 3Cl2 ¾¾¾®
useful for food packaging.
2AlCl3 + 3CO
45. (a) Metal atom in the lower oxidation state forms vapours
the ionic bond and in the higher oxidation cooled Solid anhydrous
state the covalent bond because higher ¾¾¾¾® aluminium
chloride
oxidation state means small size and great
polarizing power and hence greater the
54. (c) B2 H 6 + 2KOH + 2H 2O ® 2KBO 2 + 6H 2
covalent character. Hence MCl2 is more Potassium
ionic than MCl4. metaborate
46. (c) The cross linked polymers will be formed 2Al + 2KOH + 2H 2 O ® 2KAlO2 + 3H 2
by RSiCl3 Potassium
meta aluminate
OH 55. (c) Two oxides present in bauxite as an impurity
| are SiO2 and Fe2O3. Si belongs to the third
3nH 2O
nRSiCl3 ¾¾¾¾ ® nR - Si - OH ¾¾
® period and Fe to the fourth period. SiO2
-3nHCl |
has a three dimensional structure.
| | OH
O O 56. (b) SnCl4 is colourless volatile liquid and
| |
R - Si - O - Si
SnCl2 is colourless solid Sn conducts
| | electricity and it belongs to 14 group.
O O
| | 1
57. (d) Hybridisation = (4 + 4 + 0 - 0) = 4 ,
R - Si - O - Si - R 2
| |
O O sp3 tetrahedral.
| | 58. (a) H 2O + C ¾¾ ® CO2 + H 2
(Cross linked polymer) Steam
H2O oxidises C ¾¾ ® CO2,
47. (a) Water gas is CO + H 2
hence H2O acts as oxidising agent.
48. (b) 2Pb + 2H 2O + O 2 ® 2Pb(OH) 2 59. (b) SiO2 is acidic oxide.
49. (d) Borazone : A crystalline form of boron 60. (c) Shape of SnCl 2 is angular due to sp 2
nitride which has diamond like structure. hybridisation and having the following
(Non-planar) structure
Borazole : Inorganic benzene : B3N3H6 Cl
(planar) Sn ,
Cl
Organic Chemistry –
12 Some Basic Principles
& Techniques
1. The correct statement about the compounds A, 5. Arrange in the order of increasing acidity.
B, and C
COOCH3 COOH COOH (I) OH (II) NH2 (III) OH
H OH H OH H OH
H OH H OH HO H (a) III < I < II (b) I < III < II
is (c) III < II < I (d) II < I < III
COOH COOCH3 COOCH3
(A) (B) (C) 6. Which of the following is optically active ?
H H

(a) A and B are identical (a) CH 3 C C C2 H5


(b) A and B are diastereomers
(c) A and C are enantiomers OH OH
(d) A and B are enantiomers
H (b) CH3 — CH(OH) — CH3
|
2. Br
C6 H5CHO + HCN ¾¾
® C6H5 - C - CN
|
The product would be OH (c) CH3 C C COOH
(a) a racemate
(b) optically active O Br
(c) a meso compound O
(d) a mixture of diastereomers êê
(d) CH3 — CH 2 — C — OH
3. Arrange the following (w, x, y, z) in decreasing
order of their boiling points: 7. Keto-enol tautomerism is observed in :

O O
N || ||
N (a) H5C6 - C - CH2 - C - C6 H5
OH O CH3 H O
(w) (x) (y) (z) ||
(a) w > x > z > y (b) w > x > y > z (b) H5C6 - C - CH3
(c) w > z > y > x (d) w > z > x > y O
4. How many degrees of unsaturation are there in ||
(c) H5C6 - C - H
the following compound?
(d) Both (a) and (b)
OH 8. The change in the state of hybridization of the
asterisked carbon in the following reaction.
O
* H O *
OH CH 3CN ¾¾¾
2 ® CH CONH is
3 2
(a) sp3 to sp2 (b) sp3 to sp
(a) 6 (b) 7 (c) 10 (d) 11 (c) sp to sp2 (d) sp2 to sp3
EBD_7587
126 Chemistry Objective MCQs
9. In the reaction shown below the six membered 13. Which molecule will be most reactive for SN1
ring is generated by shifting which bond reaction?
Cl
B CH3 CH3
C D HBr
A
¾¾¾® Br Br
(a) (b)
OH H CH3

CH3
Cl CH3
(a) A (b) B (c) C (d) D CH3
(c) (d) I
10. Which among the following is substitution
reaction ? CH3

(a) C = O + RMg X ¾
¾® 14. Arrange the following in decreasing order of
solubility in water
OMgX
C
R
O O O O
|| I II III
| O ¾
¾®
||

(b) +
||
O
O
(a) I > III > II
(b) III > II > I
O (c) II > III > I
(d) All are equally soluble
O
Anhy . ZnCl 15. The correct priorities for the substituents shown
(c) R 3COH ¾¾ ¾ ¾ ¾
¾2®
Turbidity below, according to the E-Z sequence rule is
Conc . HCl
I. – CN II. – CBr (CH 3 ) 2
(d) All of these
11. In allene (C3H4), the type(s) of hybridization of O
||
the carbon atoms is (are): III. – COOH IV. – CH 2 - C - OCH3
(a) sp and sp3 (b) sp2 and sp
O
(c) only sp2 (d) sp2 and sp3 ||
12. The correct IUPAC name of the following V. – C-H
compound (a) II, III, V, I, IV (b) V, II, I, IV, III
(c) III, IV, I, II, V (d) II, V, I, IV, III
16. The alcohol product(s) of the reduction of
2-methyl-3-pentanone with LiAlH4 is (are)
is: O
(i) LiAlH diethyl ether
¾¾ ¾¾4¾ ¾¾¾¾®
(ii ) H 2O
(a) 4 - methyl - 3 - ethylhexane
(b) 3 - ethyl - 4 - methylhexane (a) a single enantiomer
(c) 3, 4 - ethylmethylhexane (b) racemic mixture
(d) 4 - ethyl - 3 - methylhexane (c) two diastereoisomers
(d) two structural isomers
Organic Chemistry – Some Basic Principles & Techniques 127
17. Identify which of the strucutre below are meso 20. In the following groups
structures?
O O -OAc -OMe
(1) (2) (I) (II)
O O
-OSO 2 Me -OSO 2CF3
O (III) (IV)
O
O O the order of leaving group ability is
(3) O (4) O (a) I > II > III > IV (b) IV > III > I > II
O O (c) III > II > I > IV (d) II > III > IV > I
21. A mixture containing the following four
O compounds is extracted with 1M HCl. The
O O compound that goes to aqueous layer is : H
(5) |
S
N
(a) 1 and 3 (b) 1, 3 and 5 (I) (II)
(c) 1, 3 and 4 (d) 2 and 5
18. Select the most stable carbocation amongst the O
following O
+ (III) (IV)
(a)
(a) (I) (b) (II) (c) (III) (d) (IV)
(b)
+ 22. Most stable carbocation among the following is:

(c) +
CH3
(a) +

(d) +
+
19. Which of the following reactions is elimination (b)
reaction?
+
(a) O + Ph 3P = CH 2
||

(c) CH2

¾
¾® CH 2 + Ph 3 PO
||

+
(d) CH3
(b) C 2 H 5 Cl + KOH ® C 2 H 5 OH + KCl 23. Which of the following cannot be represented
by resonance structures?
H (a) Dimethyl ether (b) Nitrate anion
| (c) Carboxylate anion (d) Toluene
Alc KOH
(c) C 2 H 5 - C - CH 3 ¾¾ ¾ ¾®
| 24. The IUPAC name of the compound
N(CH 3 )3 O
+
H2C — CH — CH3 is
:

CH3CH = CH - CH3 + N(CH3 ) 3 (a) 1, 2-Propoxide (b) Propylene oxide


(d) None of them (c) 1, 2-Oxo propane (d) 1, 2-Epoxy propane
EBD_7587
128 Chemistry Objective MCQs

O 30. Which of the following is the IUPAC name of the


compound
25. CH3 — C— O — CH3 ; CHO
a b
The correct relation between the bond lengths a Br
and b is:
(a) a = b Cl
(b) b > a (a) o – Bromo – m – chlorobenzaldehyde
(c) b < a (b) 2 – Bromo – 5 – chlorobenzaldehyde
(d) Impossible to predict (c) 6 – Bromo – 3 – chlorobenzaldehyde
26. Which of the following compounds are meso (d) 1 – Bromo – 4 – chlorobenzaldehyde
forms? 31. The IUPAC name of the following compound is
CH3 CH3 CH3 H
CH3
C=C
H OH H Cl H C º C – CH 2CH 3
(a) (E)-2-hepten-4-yne
H OH Cl H
(b) (Z)-5-hepten-3-yne
CH2CH3 CH3 CH3 (c) (E)-5-hepten-3-yne
1 2 3 (d) (Z)-2-hepten-4-yne
(a) 1 only (b) 3 only 32. Dipole moment is shown by
(c) 1 and 2 (d) 2 and 3 (a) 1, 2-dichlorobenzene
27. Allyl isocyanide has (b) trans 2, 3-dichloro-2-butene
(a) 9 s and 4 p - bonds (c) 1, 4-chlorobenzene
(b) 8 s and 5 p - bonds (d) trans-1, 2-dinitroethene
(c) 9 s , 3 p and 2 non- bonded electrons 33. The optically inactive compound from the
(d) 8 s , 3 p and 4 non - bonded electrons following is :
28. Which one of the following is a free-radical (a) 2 - chloropropanal
substitution reaction? (b) 2 - chlorobutane
(c) 2 - chloropentane
(a) CH 3CHO + HCN ¾
¾® CH 3CH ( OH ) CN (d) 2 - chloro - 2- methylbutane
CH3 CH2Cl 34. Tautomerism is exhibited by –
(b) Boiling
+ Cl2 ¾¾¾®

Anhy. AlCl 3
CH3
(c) + CH3Cl
(1) CH = CH – OH

CH2Cl CH2NO2
(d) + AgNO2 ¾¾®
(2) O O
+
O O O
+
29. (3) O
+
(I) (II) (III) O
The most stable canonical structure among the
given structure is : (4) O
(a) I (b) II
(c) III (d) all are equally stable
O
Organic Chemistry – Some Basic Principles & Techniques 129
(a) (1), (3) and (4) (b) (2), (3), and (4) (c) II is not acceptable because here nitrogen
(c) (1), (2) and (4) (d) None of these has 10 valence electrons
35. Which of the following represents the correct (d) II is an acceptable canonical structure.
order of stability of the given carbocations ? 39. Which of the following numberings is correct ?
+ |+ +
(a) F3 C > F3 C - C > CH3
| FF 1 2 FF 3 2
A 3 B 1
+ |+ + Br Br
5 4 4 5
(b) H3C > F3C - C > F3C F 5 4 FF 5 1
|
C 3 D 2
1 Br Br
|+ + + 2 4 3
(c) F3C - C > F3C > H3C (a) A (b) B (c) C (d) D
| 40. The two compounds given below are

|+ + + HO COOH
(d) F3C - C > H3C > F3C
|
and HO COOH
36. Which of the following behaves both as a
nucleophile and as an electrophile? (a) identical (b) enantiomers
(c) diastereo isomers (d) regiomers
(a) CH 3C º N (b) CH 3OH 41. The number of structural isomers for C6H14 is:
(c) CH 2 = CHCH 3 (d) CH 3 NH 2
37. What is the decreasing order of strength of the (a) 4 (b) 3 (c) 6 (d) 5
- 42. Which of the following pairs of compounds are
bases OH , NH -2 , HC º C - and CH 3 CH -2 ? positional isomers?
. (a) CH3 – CH 2 –CH 2 –C–CH3
||
O
(a) CH 3CH -2 > NH -2 > HC º C - > OH -
and CH3 – CH 2 – C– CH 2 – CH3
(b) HC º C - > CH 3CH -2 > NH -2 > OH -
||
(c) OH - > NH -2 > HC º C - > CH 3 CH -2 O
(d) NH -2 > HC º C - > OH - > CH 3 CH -2 (b) CH3 – CH2 – CH2 – CH 2 – CHO
38. Pick up the correct statement regarding the O
following resonating structures of the anilinium ||
ion and CH3 – CH 2 –CH 2 – C– CH 3
+ CH3 – CH 2 –CH 2 – C –CH 3
(c)
NH 3 NH 3 ||
O
+
and CH3 – CH–CH 2 – CHO
|
CH3
I II (d) CH3 – CH 2 – C –CH 2 – CH3
(a) Structure II is not acceptable because ||
carbonium ions are less stable than O
ammonium ions
CH3
(b) II is not acceptable because it is non- and CH – CH 2 –CHO
aromatic CH3
EBD_7587
130 Chemistry Objective MCQs
43. The correct nucleophilicity order is (a) optically active
(a) CH 3- < NH -2 < HO - < F - (b) optically inactive and racemic form
(b) CH 3- ~
- NH -2 > HO - ~- F- (c) optically inactive
(d) None of these
(c) CH 3- > NH -2 > HO - > F -
48. Which of the following is correct set of physical
(d) NH 2- > F - > HO - > CH 3- properties of the geometrical isomers ?

44.
Cyclopropane Cyclobutane Cyclopentane
(I) (II) (III)
CH3 CH3
The correct order of heats of combustion of
above compounds is:
(a) I > II > III (b) II > I > III
(c) III > II > I (d) III > I > II Dipole B.P. M.P. Stability
45. Which of the following optically active moment
compounds racemizes in dil. KOH/ CH 3OH (a) I > II I > II II > I I > II
solution? (b) II > I II > I II > I II > I
O (c) I > II I > II I > II I > II
(a) (d) II > I II > I I > II I > II

CH3 C3H7 49. Which of the following resonance structure is


lowest in energy?
H CH3 O
(b) H O : :
| ||
(A) H – C – C
H3C C6H5 O |
: :
O– H
(c) HB

O –
H O
:

: :
|
(B) H – C – C
(d) +
| O– H
H CH3 HB
:

46. The correct number of chiral centres in


H O–
:

: :
C 2 H5 C 2 H 5 | |
(C) H – C – C
| + O– H
: :

HB
(a) A
(a) 1 (b) 3 (c) 4 (d) 2
47. An optically active compound (A) is treated with (b) B
NaI/acetone (B) is obtained (c) C
CH 3 CH 3 (d) All have same energy
NaI 50. A solution of (+) - 2 - chloro - 2 - phenylethane in
H I ¾¾ ¾ ¾® I H
acetone toluene racemises slowly in the presence of small
C2 H 5 C2 H 5 amounts of SbCl5 due to the formation of
(a) Carbanion (b) Carbene
(A) (B)
What is true about (B)? (c) Free radical (d) Carbocation
Organic Chemistry – Some Basic Principles & Techniques 131
51. In the following compounds (a) I > II > III (b) III > II > I
(c) II > III > I (d) III > I > II
(I) (II) 55. Which one of the following acids does not exhibit
N N optical isomerism?
O
(a) Lactic acid (b) Tartaric acid
H
(III) (IV) (c) Maleic acid (d) a-amino acids
56. How many cyclic structures are possible for
N N
C4H6?
H H
the order of basicity is :
(a) IV > I > III > II (b) III > I > IV > II (a) 3 (b) 5 (c) 6 (d) 4
57. Maleic acid and fumaric acids are
(c) II > I > III > IV (d) I > III > II > IV
(a) Chain isomers
52. Which one of the following compounds has non (b) Functional isomers
zero dipole moment?
(c) Tautomers
OH
(d) Geometrical isomers
58. One among the following compounds will not
(a) give effervescence with sodium carbonate:

HO
CN (a) C6H5CO2H
SO3H
(b) (b)
NC CN
ClCH2 CH2Cl (c) C6H5OH
(c) OH
ClCH2 CH2Cl N N
2 2

NH (d)
(d)
HN
53. Dipole moment of which ketone is maximum? N 2
59. Which of the following alkenes is the most stable?

(a) (b)
(a) (b)
O
O
(c) (d)
(c) (d)
O 60. Which of the following statements is not correct?
(a) Carbocation posses sextet of electrons.
54. Which of the following orders is correct for heat (b) The order of carbocation stability is :
of hydrogenation of these compounds? + + +
CH3 CH2 CH3 CH3 > (CH3 )2 CH > (CH3 )3 C
(c) Carbocations have trigonal planar shape
CH3 CH3 CH3 (d) Carbocations are formed by heterolytic
(I) (II) (III) cleavage
EBD_7587
132 Chemistry Objective MCQs
61. The enol form of acetone after treatment with
R
D2O gives:

OD O (c) S S (d) None of these


| ||
(a) CH 3 - C = CH 2 (b) CH 3 - C - CD 3
R
OH OD
| |
(c) CH 2 = C - CH 2 D (d) CD 2= C - CD 3 64. Which of the following is not a resonance form
of the enolate ion formed in the following acid
62. Indicate whether each of the following pairs are base equilibrium
identical or not?
O
CH 3
F
+ NaOH
Br H H O
C — CH2Br
(I) H3C O O–
H CH3
F CH 2
H CH 3

:
(a) (b)
Cl O
H H O
CH2CH3 H
(II) Cl Cl
CH3 O–
O
CH3 H CH2CH3
Cl CH3
CH 3
CH3 CH3 (c) (d)
O O–
H Br Br H
(III) CH 65. The order of stability of the following
3 H H CH3
carbocations : +
Br Br (I) CH2 = CH – CH 2 ; CH 3 – CH 2 – CH 2 ;
+ +
(a) I-enantiomers; II-diastereomers; CH 2 = CH – CH
(II)2 ; CH 3 – CH 2 – CH 2 ;
+
III-enantiomers CH2
(b) I-identical; II-enantiomers; III-enantiomers
(c) I-enantiomers; II-diastereomers;III-identical
(d) I-enantiomers; II-identical; III-identical (III)
63. Which of the following compound has plane of + III
symmetry? (IV) CH3 is :
(a) IV > III > II > I (b) II > III > I > IV
(c) III > I > II > IV (d) III > I > IV > II
R = - CH - Cl , S = - CH - Cl 66. 1.4 kg of an organic compound was digested
| |
CH3 Br according to Kjeldahl’s method and the
ammonia evolved was absorbed in 60 mL of M/
R S 10 H2SO4 solution. The excess sulphuric acid
required 20 mL of M/10 NaOH solution for
neutralization. The percentage of nitrogen in the
(a) R R (b) S S
compound is:
R S (a) 10 (b) 3 (c) 24 (d) 5
Organic Chemistry – Some Basic Principles & Techniques 133
67. Which of the following would react most readily (c) Trigonal bipyramidal
with nucleophiles? (d) Tetrahedral
Cl 69. Which one of the following is most stable?
Cl

(a) (b)
+ +
Cl
OCH3
(a) (b)
Cl Cl
Ph
CH3
+ +
(c) (d) H
(c) (d)
NO2
68. The shape of transition state is – Ph

R 70. In nucleophilic substitution reaction, order of


Nu– +H C X [Transition state] halogens as incoming (attacking) nucleophile is:
I– > Br – > Cl–
R
R The order of halogens as departing nucleophile
–––––® Nu C H + X– should be :
R (a) Br– > I – > Cl– (b) I– > Br– > Cl–
(a) Triangular planar
(c) Cl– > Br– > I– (d) Cl– > I– > Br–
(b) Square pyramidal

Answer KEYs
1 (d) 8 (c) 15 (a) 22 (a) 29 (c) 36 (a) 43 (c) 50 (d) 57 (d) 64 (b)
2 (a) 9 (a) 16 (b) 23 (a) 30 (b) 37 (a) 44 (c) 51 (d) 58 (c) 65 (c)
3 (d) 10 (c) 17 (a) 24 (d) 31 (a) 38 (c) 45 (d) 52 (a) 59 (d) 66 (a)
4 (d) 11 (b) 18 (b) 25 (b) 32 (a) 39 (d) 46 (d) 53 (c) 60 (b) 67 (c)
5 (d) 12 (b) 19 (c) 26 (b) 33 (d) 40 (a) 47 (b) 54 (c) 61 (a) 68 (c)
6 (a) 13 (d) 20 (b) 27 (c) 34 (a) 41 (d) 48 (c) 55 (c) 62 (a) 69 (a)
7 (d) 14 (a) 21 (b) 28 (b) 35 (b) 42 (a) 49 (a) 56 (b) 63 (d) 70 (b)
EBD_7587
134 Chemistry Objective MCQs

1. (d) Rotation of B through 180º within the plane of the paper gives D which is an enantiomer of A, hence
A and B are enantiomers

COOH COOCH3 COOCH3


H OH OH H H OH
Rotate through 180º
H OH ¾¾¾¾¾¾¾¾¾ ® HO H H OH
with the plane of the paper
COOCH3 COOH COOH
(B) (D) (A)

2. (a) Since during the reaction, a chiral carbon is created and further since the CN – ion can attack the
planar aldehyde group both from the top and the bottom face of the aldehyde group with equal
ease, therefore, a 50:50 mixture of the two enantiomers, i.e. a racemic mixture is obtained.
3. (d) B.P. µ extent of H bonding µ surface area of molecule.
4. (d) Degree of unsaturation = 11
5. (d) Order of Acidic strength µ stability of conjugate base.

NH2 < OH < OH

– – -
i.e., NH < O < O
Resonance stabilized
{phenoxide ion}

H H
| |
6. (a) CH3 - C* - C* - C2 H5
| |
OH OH
7. (d) Keto-enol tautomerism is shown by carbonyl compounds having a-hydrogen atom.

O O O OH
|| a || || |
(a) ˆˆ† H5C6 - C - C = C - C6 H5
H5 C6 - C - C H 2 - C - C6 H5 ‡ˆˆ
|
H
enol form

O OH
|| a |
(b) ˆˆ† C6 H 5 - C = CH 2
H5C 6 - C - C H3 ‡ˆˆ
Keto form enol form

O
||
(c) C6 H 5 - C - H
no a- H atom
Organic Chemistry – Some Basic Principles & Techniques 135

sp sp 2 H O -C
H O OH
8. (c) H3 C - C º N ¾¾¾
2 ® H C - C - NH
3 2
||
| | |
O
-C-O, - C - C - OCH3
9. (a) Five membered ring is being converted into | |
| |
six membered ring. Hence the sixth bond
C -O C H O-C
must come from within the ring.

10. (c) R 3COH ¾¾ ¾ ¾ ¾


¾2® Anhy . ZnCl
R 3C.Cl (iii) (iv)
Conc . HCl
O-C
It is substitution reaction. R3C.Cl being |
insoluble in H2O give turbid solution.
-C-O
H sp 2 sp sp2 H | |
11. (b) C=C=C
H C
H H
(v)
Arrange (NNN), (BrCC), (OOO), (CHH),
CH3
| (OOH) in increasing atomic number. The
12. (b) CH3CH 2 - C H - C H - CH 2CH 3 order is ii, iii, v, i, iv.
|
CH2 16. (b) The resulting compound is C3H7CH(OH)C2H5
| which is optically inactive and reaction leads
CH 3 the racemisation
17. (a) Meso compounds are those having 2 or
3-Ethyl-4-methyl hexa ne
more chiral centers along with symmetry.
13. (d) Order of stability of carbocations is
3° > 2° > 1° and I is the best leaving group ’
O POS
among halogens. (1) (3) O
* *
14. (a) Higher the electron density on O, stronger O O ’
is the H-bond with water and thus more is
POS O
the solubility. Thus solubility of the three Meso
ethers follow the order
18. (b) Structure (b) is a 3º carbocation, while (a) is
2º and (c) and (d) are 1º carbocations; thus
(b) is the most stable.
.. .. .. 19. (c) The reaction (c) is Hoffmann elimination
.O. .O. .O. 20. (b) The more the electrons withdrawing groups
I III II attached to - O atom, the more is the ease of
electron pair electron pair electron pair more
localised delocalised delocalised leaving.
21. (b) When the given mixture is shaken with 1 M
HCl, amine get protonated and becomes
N C CH 3
| | | cation (RNHÅ 2 ) , which does not dissolve
15. (a) - C - N, - C - CH 3 ,
| | | in organic solvent but usually dissolve in
N C Br H2O due to its charge. So, shaking with
aqueous HCl will pull amines into the
(i) (ii) aqueous phase and leave all other
compounds in organic layer.
EBD_7587
136 Chemistry Objective MCQs
22. (a) Stability of carbocation µ no. of a–H present ··+
O
on carbocation. 29. (c) It has complete octet for all atoms.
23. (a) Ethers due to absence of delocalized pair
of electrons do not show resonance. 30. (b) The compound is a derivative of
24. (d) 1, 2-Epoxy propane is the correct IUPAC benzaldehyde. Start numbering C-atoms of
name of given compound. benzene nucleus from C-atom bearing –
CHO group.
O 1
25. (b) 31. (a) CH3 2 3 H
CH 3 — C — O — CH 3 C=C 4 5 6 7
H C º C – CH2CH3
due to resonance this bond has partial
double bond characters. (E) – 2 – hepten – 4 – yne

Me 32. (a) In 1, 2-dichlorobenzene the two dipoles


are at 60° (i.e. unsymmetric).
* Thus possesses dipole moment.
26. (b) 33. (d) The optically inactive compound must
* contains achiral carbon atom(s). Option (d)
Me contains achiral carbon atom
Meso ® Compounds having chiral centres
and have plane or centre of symmetry are Cl Cl
| |
known as meso compound. CH3 - C - CHO CH3 - CH 2 - C - CH 3
27. (c) Allyl isocyanide has the structure | |
® H H
H 2 C = CH - CH 2 - N = C
it has 9s and 3p bonds along with 2 non- 2- chloropropanol 2-chlorobutane (chiral)
bonded electrons
Cl
28. (b) In the presence of UV rays or energy, by |
boiling chlorine, free radical is generated CH3 - CH 2 - CH 2 - C - CH 3
|
which attacks the methyl carbon atom of the H
toluene.
2 -chlor openta ne
hn
Cl 2 ¾¾® 2Cl• (chiral)

CH3
CH3 CH2 |
l
+ Cl
¾¾® +H CH3 - CH 2 - C - CH3
|
benzyl free radical Cl

•CH 2-chloro-2-methylbutane
2
CH2Cl
(achiral)
+ Cl2 ¾¾® + •Cl

34. (a)

(a) CH = CH – OH CH2 – CH
enol form keto form
Organic Chemistry – Some Basic Principles & Techniques 137

(b) O O cannot tautomerise

O OH OH
(c)
O keto form O OH
enol form

(d) O OH OH

O O OH
keto form enol forms

35. (b) – I group destablises carbocation and since ** *


inductive effect decreases with increasing C . But numbering from C give minimum
length of carbon chain. Therefore (b) is the locant (2) to Br which is correct.
correct option. 40. (a) The given compounds are identical.
36. (a) Due to the presence of a lone pair of 41. (d)
electrons on N, CH3C º N: acts as a
nucleoph ile. Further due to gr eater
electronegativity of N than C, the C atom =
of – C º N carries a positive charge and are same
hence behaves as an electrophile. (1) (2)

37. (a) Stronger the acid, weaker the conjugate


base. Since acid character follows the order
H2O > HC º CH > NH3 > CH3 – CH3 (3) (4) (5)
(Acid character),
the basic character of their conjugate bases Total 5 structural isomers.
decreases in the reverse order, i.e., 42. (a) Pentan-2-one and pentan-3-one are
position isomers.
CH3 CH 2– > NH 2 - > HC º C - > OH - (b), (c), (d) contain different compounds
(Basic character) aldehyde and ketones.
38. (c) N has no d orbital, hence it can’t 43. (c) Nucleophilicity increases with the decrease
accommodate more than 8 electrons in its in electronegativity of the central atom.
valence shell. Since electronegativity follows the order:
F > O > N > C; nucleophilicity of the
39. (d) F *1 concerned group will follow the reverse
5
Br order i.e.,
2
**
4 3 CH 3 - > NH 2 - > OH - > F -

* 44. (c) More the no. of carbons more will be the


The numbering of C-atom starts from C or heat of combustion.
EBD_7587
138 Chemistry Objective MCQs
depends on symmetry, therefore I has higher
O
|| melting point than II. Steric crowding in II is
45. (d) The compounds containing - C - group more than in I therefore I is more stable than
adjacent to asymmetric carbon atom II.
carrying a hydrogen easily racemise. 49. (a) The order of stability of resonating structures
46. (d) The molecule contains two similar chiral carrying no charge > carrying minimum charge
centres. and each atom having octet complete.
47. (b) The reaction proceeds through planar 50. (d) SbCl5 pulls Cl– to form SbCl 6- leaving behind
CH3 +
|+ planar C6 H 5 - CH - CH 3 carbonium ion. It
C leading to racemisation. can be attacked from either side leading to
H C2 H 5 racemic mixture.
48. (c) In compounds 51. (d) In I the unshared pair of electrons on N always
available for protonation in III due to presence
CH3 CH3 of electronegative O atom the electron density
on N is decreased . In II and IV resonance
suppresses the basic character.
52. (a) Cis molecule has non-zero dipole moment.
I has more dipole moment than II, hence its
boiling point will be higher. Melting point
O O– O–
+

¬¾® ¾®
53. (c) +
Stable R.S. Resonance
because aromatic hybrid

so molecule has high dipole moment.


1
54. (c) Heat of hydrogenation µ .
Stability of alkene
55. (c) Optical isomers contain an asymmetric (chiral) carbon atom (a carbon atom attached to four different
atoms or groups). Therefore all except maleic acid exhibit optical isomerism.
56. (b) Five cyclic structures are possible for C4H6. These are as following:
CH3 CH3 CH2

Cyclobutene 2-methyl 1-methyl methylene bicyclo


cyclopropene cyclopropene cyclopropane [1, 1.0] butane

57. (d) Maleic acid and fumaric acids are geometrical isomers.
H – C – COOH H – C – COOH
|| ||
H – C – COOH HOOC – C – H
Maleic acid Fumaric acid
Organic Chemistry – Some Basic Principles & Techniques 139
58. (c) Phenol is less acidic than H2CO3 so reaction 61. (a) The enol form of acetone is
does not favours in forward direction.
OH
59. (d) Stability of Alkene µ No. of H-atoms or more |
hyperconjugation. CH3 - C = CH 2 with D2O it gives
60. (b) The order of stability of carbocations is :
OD
+ + + |
(CH 3 )3 C > (CH3 )2 CH > CH3 CH 3 - C = CH 2

F a
F
H
c CH2Br
Br H c
62. (a) (I) º H Me & b
Me
H CH3 Fa
H CH2Br
b

Enantiomer

H C2H5 H Cl H
Cl
a b
(II) &
c Cl H Me Et Cl
Me
Configuration-R S S S-Configuration

are diastereoisomers.

Me Me
(III)
H Br Br H

Me H H Me
Br Br
Enantiomer
R Cl CH3
H3C CH3
63. (d) Consider R R
Cl
Cl
R Cl CH3

no plane can pass in such a way that molecule has symmetry.

S R

Similarly for S S and S S

S R
EBD_7587
140 Chemistry Objective MCQs
64. (b) (a),(c) and (d) are resonating structures.
O O
–O
CH 3 CH 3 CH 3

O O –O
65. (c) Higher stability of allyl and aryl substituted methyl carbocation is due to dispersal of positive charge
due to resonance
+ +
CH 2 = CH - C H 2 ¬¾® CH 2 - CH = CH 2
Resonating structures of allyl carbocation

+
CH2 CH2 CH2 CH2
+ +

Resonating structures of benzyl carbocation

Hence the correct order of stability will be


+
CH2

+ + +
> CH2 = CH – CH2 > CH3 – CH2 – CH2 > CH3
Benzyl, III Allyl, I Propyl, II Methly, IV

66. (a) Milli equivalents of H2SO4 in the o- and p- position to the nuclear
M´2 halogen. Hence, p-nitrochlorobenzene
= 60 ´ = 12 would react most readily with nucleophiles.
10 68. (c) Shape of transition state is as follows:
M
Milli equivalents of NaOH = 20 × =2
10
R
Milli equivalent of NH3 = 12 – 2 = 10
% of nitrogen Nu C X
1.4 ´ ( N ´ V ) NH 3 H R
=
(Wt.of organic compound)
Trigonal bipyramidal
1.4 ´ 10 69. (a) 3° carbocations are most stable.
= 10
1.4 70. (b) Since the leaving group breaks away as a
67. (c) Aryl halides do not undergo nucleophilic base, it is easier to displace weaker bases
substitution under ordinary conditions. The as compared to stronger bases. Thus less
low reactivity of halogen atom in aryl halides basic the substituent, the more easily it is
is due to resonance. However, aryl halides displaced.
can be made to undergo nucleophilic Since the basic strength of the given groups
substitution either under drastic condition is in order.
(high temperature, pressure or very strong I – < Br– < Cl–
nucleophiles) or by activating the nuclear
Thus the order of halogen leaving groups
halogen by introducing electron
is
withdrawing group e.g. NO2, –CHO, CN etc.
I– > Br– > Cl–
13 Hydrocarbons
1. Which of th e following would not give The IUPAC name of the alkene is
2-phenylbutane as the major product in a Friedel- (a) 3, 4-dimethyl-3-pentene
Crafts alkylation reaction on benzene ring?
(b) 3, 4-dimethyl-2-pentene
(a) 1-butene + HF
(c) 2, 3-dimethyl-3-pentene
(b) 2-butanol + H2SO4
(d) 2, 3-dimethyl-2-pentene
(c) Butanoyl chloride + AlCl3 then Zn, HCl
(d) Butyl chloride + AlCl3 6. Which of the following will yield a mixture of
2. Which of the following compounds are 2-chlorobutene and 3-chlorobutene on treatment
antiaromatic with HCl ?
(a) CH 2 = C = CH – CH3

O (b) H 2C = C - CH = CH 2
– + |
CH3
(I) (II) (III) (IV) (V) (VI)
(a) (I) and (V) (b) (II) and (V) (c) CH 2 = CH - CH = CH 2
(c) (I) and (IV) (d) (V) and (VI)
(d) HC º C - CH = CH 2
3. Which one of the following reactions is expected
to readily give a hydrocarbon product in good 7. Which of the following statements is incorrect
yields ? regarding dehydrohalogenation of alkenes?
(a) During the reaction hydrogen atom is
Electrolyt ic eliminated from the b - carbon atom.
(a) RCOOK ¾ ¾ ¾ ¾¾®
oxidation (b) Rate of reaction for same alkyl group;
Iodine > Bromine > Chlorine
RCOO - Ag + ¾¾
(b) Br
¾2®
(c) Rate of reaction;
CH 3 CH 3 ¾¾
¾2® Cl (CH3)3C – > (CH3)2CH – > CH3CH2 –
(c)
hu (d) Only nature of halogen atom determine rate
of the reaction.
(d) (CH3 )3 CCl C2 H5OH
¾¾¾¾ ®
8. Which of the following will form alkynide?
H (1 mole)/Pt C º C - CH 3
(A) ¾¾¾¾¾¾®
2
4. (a)
Double bond equivalent (degree of unsaturation)
of (A) is: C º C -H
(a) 1 (b) 2 (c) 3 (d) 4 (b)
5. An alkene having molecular formula C7H14 was
subjected to ozonolysis in the presence of zinc C ºC
dust. An equimolar amount of the following two
(c)
compounds was obtained
CH3 CH3
C = O and (d) CH 3 - C º C - CH 3
CH3 C=O
CH3CH2
EBD_7587
142 Chemistry Objective MCQs

CH3 ·
(b) H3C - CH- C - CH 3
excess HBr | |
9. CH3 CH C º CH ¾¾¾¾¾ ®
D CH3
The product of the above reaction is:
CH3 Br Br
·
(c) H 3 C - C -CH - CH 3
(a) | |
D CH 3
CH 3 Br
·
(b) CH3 CH C CH 2 (d) H 3C - CH - CH - CH 3
|
CH3 Br CH3
(c) CH3 CH C CH3 13. Predict the product (A) of the following reaction
Br
CH3 Br
H+
A
(d) CH3 CH CH2 CH D

Br
10. What is the end product of the following (a) (b)
sequences of operations?

H O dil.H SO Ni (c) (d)


CaC 2 ¾¾¾
2 ® A ¾¾¾¾¾
2 + ® B ¾¾¾
2 4 ®C
Hg H2
(a) Methyl alcohol (b) Acetaldehyde 14. Consider the following sequence of reactions
(c) C2H5OH (d) C2H4
Cl Na CO
CH3 CH3CH = CH 2 ¾¾¾
2 ® A ¾¾¾¾®
2 3
11. Ha 700K 420K,12atm
Hb
(i ) HOCl
B ¾¾¾¾® C
+ Br• ® (ii) NaOH
H d OH Compound ‘C’ is
CH2 ¾ H c
(a) CH2OH
Br• will abstract which of the hydrogen most
readily? CHOH

(a) a (b) b (c) c (d) d CH2OH


12. Consider the following reaction
·
H 3C - CH - CH - CH 3 + Br ¾¾ ® ' X '+ HBr (b) CH3CHCOONa
| |
D CH3 OH
Identify the structure of the major product 'X' (c) HOCH2 – CH = CH2
·
(a) H3C - CH - CH - CH 2 (d) CH 3CHCOCl
| |
D CH3 OH
Hydrocarbons 143
15. Choose the correct alkyne and reagents for the (a) CH2 = CH – CH = CH – CH3
(b) CH3 – CH = CH – CH2 – CH3
preparation of (c) (CH3)2CH – C º CH
(d) (CH3)2C = C = CH2
18. The major product obtained in the following
(a) , HgSO 4 , H 2SO 4 , H 2O reaction
O
(b) , HgSO 4 , H 2SO 4 , H 2O || Br
¾¾
¾2®
?
FeBr3

(c) , BH 3 , H 2O 2 , NaOH O
||
(a) Br

(d) , BH 3 , H 2O 2 , NaOH
O
||
16. The major product of the following reaction (b) Br
O
CH3CH 2 MgCl O
H 3C - C º CH ¾¾¾¾¾¾ ® ¾¾¾¾¾® ||
ether H3 O+
(c) Br Br
HO CH 2 CH 3

(a) O
||
(d)
CHCH 3
|| Br

(b) O
19. ¾¾®
3

C º CH
Which of the following products cannot be
HO CH 2 obtained in ozonolysis of o-xylene?
CHO
(c) (a)
CHO
C CH 3 O O
HO C
(b) CH3 C C H

(d) O O

17. A compound (X) (C5H8) reacts with ammonical (c) CH3 C C CH3
AgNO3 to give a white precipitate, and on oxida-
O O
tion with hot alkaline KMnO4 gives the acid,
(CH3)2CHCOOH, therefore X is – (d) CH3 C C CHO
EBD_7587
144 Chemistry Objective MCQs
20. Compare rate of dehydration of (I), (II) and (III) O
by conc. H2SO4.
(a) C O
OH
I. O2N

O
OH
(b) O2N C O
II.
O

(c) C O
III.
OH O2N
(a) (I) > (III) > (II) (b) (I) > (II) > (III)
O
(c) (II) > (I) > (III) (d) (II) > (III) > (I)
21. In which step of the following reaction, sp3 (d) C O NO2
carbon changes to sp2 carbon ?
+ H 24. The major product obtained in the photo
+ Step 1
+ E ¾¾® E catalysed bromination of 2-methylbutane is:
(a) 1-bromo-2-methylbutane
(b) 1-bromo-3-methylbutane
E
Step 2 (c) 2-bromo-3-methylbutane
¾¾® + H+
(d) 2-bromo-2-methylbutane
(a) Step 1 (b) Step 2 25. In the hydroboration - oxidation reaction of
(c) Both (d) None propene with diborane, H2O2 and NaOH, the
22. Toluene reacts with excess of Cl2 in presence of organic compound formed is:
sun light to give a product which on hydrolysis (a) CH3CH2OH (b) CH3CHOHCH3
by reaction with NaOH gives (c) CH3CH2CH2OH (d) (CH3)3COH
26. In the given reaction,
C CCH3
COOH COONa
(a) (b) H +/Hg 2+
¾¾¾¾®A the product ‘A’ is

COCl
OH
(c) (d) None of these
HC = C – CH3
Na
23. The major product formed in the reaction is: (a)
O

C O HO – C = CHCH3

Conc. HNO /conc. H SO


¾¾¾¾¾¾¾¾¾
3 2 4
® (b)
(mononitration)
Hydrocarbons 145

O = C – CH2CH3 (a) CH 3 –– CH = C –– CH( OH) –– CH3

––
Cl
(c)
OH CH3

––

–– ––
(b) CH3–– CH –– C –– CH2 –– CH3
O
Cl
CH2 – C – CH3
Cl CH3
(d)

––

–– ––
(c) CH3 –– CH –– C –– CH2 –– CH3
OH
27. The product of the reaction between ethyl
benzene and N-bromosuccinamide is (d) None of these
32. The major product of the following reaction
CH2 – CH2 – Br CH2 – CH3 sequence is:

1. B H
(a) (b) ¾¾¾¾¾¾
2 6

®?
2. H2 O2 , HO
Br
OH OH
CH2 – CH3 Br (a) (b)

CH – CH3 O H HO
(c) (d)
(c) (d)
33. All the hydrocarbons shown are very weak acids.
Br One, however, is far more acidic than the others.
Which one is the strongest acid?
28. In preparation of alkene from alcohol using Al2O3
which is effective factor ?
(a) Porosity of Al2O3 (a) (b)
(b) Temperature
(c) Concentration
(d) Surface area of Al2O3 (c) (d)
29. Which one of the following compounds would
have the highest heat of hydrogenation? 34. Which one amongst the following carbocations
(a) CH 2 = CH 2 is most stable?
+
(b) CH 3 - CH 2 - CH = CH 2 (a) C 6 H 5 — C H — C6 H 5
(c) CH 3CH = CHCH 3 +
(d) (CH3 )2 C = C(CH3 )2 (b)
30. Which of the following types of reaction occur
when a reactant has got a double bond ? +
(i) Addition (c) CH3 C CH3
(ii) Photolysis
CH3
(iii) Nucleophilic substitution
(iv) Polymerization +
(d) C
(a) (i) and (iv) (b) (i) and (ii)
(c) (i), (ii) and (iv) (d) (i), (ii), (iii) and (iv)
31. HOCl reacts on 3-methyl-2-pentene, the main
product will be :
EBD_7587
146 Chemistry Objective MCQs
35. Toluene on treatment with CrO3 and (CH3CO)2O H
followed by hydrolysis with dil.HCl gives (c) C H
(a) benzaldehyde C
(b) benzoic acid CH3
(c) phenol
(d) phenylacetaldehyde H H
36. What is the correct product of reaction? C H
C
CH3
1. BH , THF
¾¾¾¾¾¾¾
3
-® ? (d)
2. H 2 O2 , OH

39. Among the following free radical bromination


reactions, select those in which 2° halide is the
CH3 major product
CH 3
H
(a) (b) Br /hn
H (P)(P) ¾¾®
2
H

OH
CH3 (Q) Br /hn
CH 3 ¾¾®
2

(c) (d) H
H
OH CH2¾ CH3

H Br /hn
(R) ¾¾®
2
37. Of the three isomeric C3H4 hydrocarbons shown
below how many can exist with all carbon and
hydrogen nuclei located in a single plane Br /hn
(S)
(S) ¾¾®
2

CHO
H 2C = C = CH 2 , CH 2 , H3C - C º CH
Br /hn
CHO (T)
(T) ¾¾¾®
2

(a) 0 (b) 1 (c) 2 (d) 3


Br2/hn
(U)
(U) ¾¾®
38. C CH (a) P, Q, R, S (b) P, R, U
(c) P, R, S, T (d) P, Q, R, S, T
(i) NaNH , NH
¾¾¾¾¾¾¾
2 3 ® (A) ¾¾¾¾¾¾®
2 (B) ;
H 40. The gas liberated by the electrolysis of
(ii) CH3Br Lindlar catalyst dipotassium succinate solution is:
Product (B) is: (a) Ethane (b) Ethyne
C C CH3 (c) Ethene (d) Propene
41. In the presence of peroxide, HCl and HI do not
give anti-Markownikoff’s addition of alkenes
(a) because:

H CH3 (a) One of the steps is endothermic in HCl and


HI
C C
(b) Both HCl and HI are strong acids
(b)
H (c) HCl is oxidizing and the HI is reducing
(d) All the steps are exothermic is HCl and HI
Hydrocarbons 147
42. Consider the following sequence of reactions (a) H2O/H+
Cl Na CO (b) BH3, THF/H2O2—OH–
CH3 CH = CH 2 ¾¾¾
2 ® A ¾¾¾¾¾¾
2 3 ®B
700K 420K,12 atm (c) Hg(OCOCH3)2, H2O/NaBH4 . NaOH
Compound ‘B’ is (d) All are possible
(a) 48. In reaction sequence
CH2OH CH 2OH
Hypochlorous R
CH 2 = CH 2 ¾¾ ¾¾ ¾ ¾® M ¾¾® |
CH acid CH 2OH
molecule 'M' and reagent 'R' respectively are
CH2
(a) CH3CH2Cl and NaOH
(b) CH3CHCOONa (b) CH3CH2OH and H2SO4
OH (c) CH2(Cl)CH2OH and aqueous NaHCO3
(c) HOCH2 – CH = CH2 (d) H2C — CH2 and heat
(d) CH 3CHCOCl O
49. What is the major product expected from the
OH following reaction?
43. How many monochlorobutanes will be obtained CH3
on chlorination of n-butane?
(a) 5 (b) 2 (c) 3 (d) 4 D – Cl
¾¾®
44. The cycloalkane having the lowest heat of
combustion per CH 2 group Where D is an isotope of hydrogen
CH3
D
D
(a) Cl (b) Cl
(a) (b)
CH3
H
H
(c) (d) CH3 D

45. Which one of the following conformations of (c) H (d) H


cyclohexane is chiral? Cl CH3
(a) Boat (b) Twist boat D Cl
(c) Rigid (d) Chair 50. The reagent needed for converting
46. Which one of the following compounds gives
acetone (CH3)2C O as one of the product of Ph H
Ph – C C – Ph ¾¾® C=C
its ozonolysis? H Ph
(a) Cat. Hydrogenation
(a) (b) (b) H2/ Lindlar Cat.
(c) Li/NH3
(d) LiAlH4
(c) (d)
51. Addition of HI to double bond of propene yields
CH3CHCH CH2 isopropyl iodide and not n-propyl iodide as the
CH3CH CHCH3 major product, because addition proceeds
through
47. A OH ;
¾¾® (a) a more stable carbonium ion
(b) a more stable carbanion
Reagent A may be: (c) a more stable free radical
(d) homolysis
EBD_7587
148 Chemistry Objective MCQs
52. The principal organic product formed in the
CH2 – CH = CH2
reaction, 57.
peroxide
CH 2 = CH(CH 2 )8 COOH + HBr ¾¾¾¾® is
On mercuration-demercuration produces the
(a) CH3CHBr(CH2)8COOH major product:
(b) CH2 = CH(CH2)8COBr
(c) CH2BrCH2(CH2)8COOH CH2 – CH – CH 3
(a)
(d) CH2 = CH(CH2)7CHBrCOOH OH
53. Acid catalyzed hydration of alkenes except ethene
leads to the formation of CH2 – CH 2 – CH 2 – OH
(b)
(a) mixture of secondary and tertiary alcohols
(b) mixture of primary and secondary alcohols
(c) secondary or tertiary alcohol CH2 – CH – CH 3
(d) primary alcohol (c)
54. Identify the reagent from the following list which OH OH
can easily distinguish between 1-butyne and CH 2 – COOH
2-butyne (d)
(a) bromine, CCl4
(b) H2, Lindlar catalyst 58. Which one of the following class of compounds
(c) dilute H2SO4, HgSO4 is obtained by polymerization of acetylene?
(d) ammonical Cu2Cl2 solution (a) Poly-yne (b) Poly-ene
55. An unknown compound A has a molecular (c) Poly-ester (d) Poly-amine
formula C4H6, when A is treated with an excess
59. The hydration of propyne results in formation of
of Br2, a new substance B with formula C4H6Br2
is formed. A forms a white precipitate with (a) Acetone (b) Propanol-1
ammonical silver nitrate solution. A may be (c) Propene (d) Propanal
CH 2 - CH = CH 2

(a) Butyne-1 (b) Butyne-2


(c) Butene-1 (d) Butene-2 60. + HCl ¾¾
® X, X is :
56. The major product formed in the reaction is:

CH 2 - CH - CH3
|
conc. H2SO4 Cl
+ HNO3 ¾¾¾¾¾®
(a)

CH 2 - CH 2 - CH 2 Cl

NO2
(a) (b)
(b)
NO2
Cl
NO2 |
CH - CH 2 - CH3

(c) (d)
(c)

NO2
Hydrocarbons 149

CH 2 - CH = CH 2
KMnO4
Cl ¾¾¾¾® OH
(c)
(d) H
H H OH H
O O
Alk.
¾¾¾¾® HO C (CH2)3 C O H
(d) KMnO 4
Br2 /hn Alcoholic
61. ® Major (X)
¾¾¾¾ ¾¾¾¾ ® H H
KOH/D
64. Which of the following change is correct
H—Br
Major (Y) ¾¾¾¾
Peroxide
® Major (Z) z is: (a) C =C I , CH COOAg
¾¾¾¾¾¾¾
2 3 ®
wet CH3COOH
Br
(a) (b) C –– C

––

––
Br OH OH
Br I , CH COOAg
(b) C =C ¾¾¾¾¾¾¾
2 3 ®
dry CH3COOH
Br
(c) (d) OH

––
C –– C

––
OH
O H Ni H+ (c) Both (a) and (b) one correct
62. ¾¾®
3
A ¾¾¾¾®3
B ¾¾® (C);
Zn Or LiAlH D 4 (d) Neither (a) nor (b)
65. Choose the correct product of the following
reaction
Product (C) of the reaction is:
O
OH O
OOH
(1)
CH3
(a) (b) ¾¾¾¾¾¾¾¾® ?
(2) H O+ 3
CH 3
(c) (d)
CH3
OH
63. Which of the following is correct? (a) CH3 OH
(b)
CH3 OH
H CH3

C6H5COOO H
¾¾¾¾¾¾® H H CH3
(a) OH
H H O OH
(c) (d) CH3
CH3 H
(i) O3
¾¾¾¾® OHC.CH2.CH2.CHO
(b) (ii) H2O 2 OH CH3
H H
EBD_7587
150 Chemistry Objective MCQs
66. The two compounds A and B obtained from
CH3 CD3 CT3
1-butyne can be distinguished by

(i) BH
B ¬¾¾¾¾¾¾
3
-¾ CH3CH 2 C º CH ¾¾¾¾¾
®A
(ii) H 2O 2 /OH (a) (b) (c)
H+ /Hg 2+
B ¬¾¾¾¾¾¾
¾ CH CH C º CH ¾¾¾¾¾
®A
(a) a > b > c (b) c > b > a
(a) NaHSO3 (b) litmus solution
(c) a > c > b (d) c > a > b
(c) iodoform test (d) 2, 4-DNP
69. Which of the following is an aromatic species?
O NO2 CH3

NH C CH3 (a) H (b) H


+ +
CH3
67.
C A (c) (d) All of the three
N
B H
Na/Dry ether
Identify the position where electrophilic aromatic 70. Cl Br ¾¾® (A)
substitution (EAS) is most favourable. 97%
(a) A (b) B Product (A) of above reaction is:
(c) C (d) A and C
68. Arrange the following in decreasing order of
reactivity towards EAS (electrophilic aromatic ¾
(a) (b)
substitution) ¾

(c) (d)

Answer KEYs
1 (c) 8 (b) 15 (b) 22 (b) 29 (a) 36 (d) 43 (b) 50 (c) 57 (a) 64 (c)
2 (d) 9 (c) 16 (d) 23 (d) 30 (a) 37 (a) 44 (a) 51 (a) 58 (b) 65 (c)
3 (a) 10 (c) 17 (c) 24 (d) 31 (c) 38 (c) 45 (b) 52 (c) 59 (a) 66 (c)
4 (c) 11 (a) 18 (a) 25 (c) 32 (b) 39 (b) 46 (d) 53 (c) 60 (c) 67 (b)
5 (d) 12 (b) 19 (d) 26 (c) 33 (c) 40 (c) 47 (c) 54 (d) 61 (c) 68 (a)
6 (a) 13 (d) 20 (b) 27 (d) 34 (d) 41 (a) 48 (c) 55 (a) 62 (c) 69 (c)
7 (d) 14 (a) 21 (b) 28 (d) 35 (a) 42 (a) 49 (b, c) 56 (b) 63 (a) 70 (b)
Hydrocarbons 151

1. (c) The Friedal-crafts acylation reaction will give propyl phenyl ketone which further on Clemmenson’s
reduction will give butyl benzene
AlCl Zn-Hg/HCl
C6H6 + CH3CH2CH2COCl ¾¾¾®
3
C6H5COCH2CH2CH3 ————® C6 H5 CH 2 CH 2CH 2CH3
Butyl benzene
2. (d) For a compound to be aromatic it must have (4n + 2) pe– and system should be planar where
n is an integer. (V) and (VI) both have 4 e– and planar structure. Hence are Antiaromatic.

2RCOOK ¾¾¾¾® 2RCOO - + 2K +


Electrolytic
3. (a)
oxidation Anode Cathode
- · –
At anode 2RCOO ® 2RCOO + 2e
2R COO· ® R —R + 2CO 2

At cathode 2K + + 2e – ® 2K
2K + H 2O ® 2KOH + H 2 ­
4. (c) Molecular formula
H
¾¾®
2
Pt Þ DBE of (A) = 3
(A)

CH3 CH3 CH CH3


5. (d) C=O+O=C ¬¾¾ 3 C=C
CH3 C2H5 CH3 CH2CH3
2,3-Dimethyl-2-pentene

é + + ù
H+
6. (a) CH 2 = C = CH - CH3 ¾¾¾ ® êCH 2 = C –– CH 2 –– CH 3 + CH 2 = CH –– CH –– CH 3 ú
ë û
Cl-
¾¾¾
® CH 2 = C - CH 2CH3 + CH 2 = CH - CHCH3
| |
Cl Cl
2 -Chlorobutene 3-Chlorobutene
7. (d) Nature of halogen atom and the alkyl group both determine rate of reaction.
8. (b) Only 1-Alkynes form alkynides

CH3 CH3 CH3 Br


H Br + – H Br+ –
9. (c) CH3 CH C CH ¾¾® CH3 CH C CH2 ¾¾® CH3 CH C CH3

Br Br
H 2O Dil.H 2SO 4 Ni
10. (c) CaC 2 ¾¾¾ ® HC º CH ¾¾¾¾¾ + 2
® CH 3CHO ¾¾ H
¾® C2 H5OH
Acetylen e Hg 2 (C)
Acetaldehyde
(A) (B)
EBD_7587
152 Chemistry Objective MCQs
11. (a) Bromine is more selective
\ abstract that hydrogen which forms stable free- radical
CH3
.

\ (3° free-radical) is most stable.

CH3

12. (b) Br · is less reactive and more selective and so the most stable free radical (3°) will be the major
product.

+
+
Ring 1, 2 – CH3
H+ expansion shift +
13. (d) ¾¾¾® ¾¾¾® ¾¾¾¾ ® ¾¾®
(3° Carbocation stable)

14. (a) CH3 CH2Cl CH2OH CH2OH


| | | |
Cl2 aq. Na2CO3 HOCl
CH ¾¾¾® CH ¾¾¾¾® CH ¾¾¾¾¾¾¾® CHCl
500 °C Markownikoff |
addition
CH2 CH2 CH2 CH2OH
Propene Allyl chloride

NaOH
CH2OH
|
CHOH
|
CH2OH
Glycerol

Tautomerisation
15. (b) HgSO H SO
¾¾ ¾4¾ ¾
2 ¾4® ¾¾¾¾¾¾
¾®
H 2O OH

16. (d) H 3C - C º C – H + CH 3CH2 MgCl ¾


¾® H 3C - C º C -MgCl + CH 3CH 3

OH

C º C - CH3
17. (c) Compound X reacts with ammonical AgNO3 solution, so it must be a terminal alkyne.
Formation of (CH3)2CH COOH on oxidation of X with hot alkaline KMnO4 further confirms that X is
(CH3)2 CHC º CH.
O
||
18. (a) A - C - CH 2 - CH2 - B

The ring B has o/p directing and (A) has m-directing group. Hence the product will be
Hydrocarbons 153

O
||
- C - CH 2 - CH2 - - Br

19. (d) (o-xylene) which on ozonolysis give a, b and c.

20. (b) Formation of carbocation is r.d.s.


+
+
> > (stability order)
+

7 R.S. 8 R.S. Vinylic


R.S. = Resonating (least stable)
Structure
21. (b) Step 1 involves the conversion of sp2 carbon to sp3 carbon (carbon having electrophile E) while
step 2 involves the conversion of sp3 carbon to sp2 carbon.
22. (b)
CH3 CCl3 COOH COONa
3Cl2 NaOH
¾¾¾® HOH ¾¾¾¾ ®
hn ¾¾¾® - H 2O

O
||
23. (d) C O ¬¾
¾ because of steric factor para is major..
(– M) (+ M)
24. (d) The order of substitution in different alkanes is
3° > 2° > 1°
Thus the bromination of 2-methyl butane mainly gives 2-Bromo - 2 - methyl butane
CH3 CH3 CH3
| Br2 | |
CH3 - CH 2 - CH - CH3 ¾¾¾ ® CH3 - CH 2 - C H - CH 2 Br + CH3 - CH 2 - C - CH3
2-methyl butane 1-Bromo-2methyl butane |
(minor) Br
2-Bromo-2-methyl butane
(major)

B H
25. (c) CH3CH = CH 2 ¾¾¾¾¾¾
2 6 ® CH3CH 2CH 2OH
H 2O2 ,NaOH

C º C – CH3 C(OH) = C – CH3 O = C – CH 2CH 3


+ 2+
26. (c) H /Hg
¾¾¾¾ ¾
® tautomerisation
¾¾¾¾¾¾ ®

Br
CH2CH3 Br CH – CH3
N
O O
¾®
||

27. (d)
||

N – bromosuccinamide
EBD_7587
154 Chemistry Objective MCQs
28. (d) The amount of alcohol absorbed depends upon the surface area of Al 2O3
29. (a) The heat of hydrogenation of an alkene depends upon its stability. Higher the stability, lower the heat
of hydrogenation. Since CH2=CH2 has no substituent, it is the least stable alkene and hence has the
highest heat of hydrogenation
30. (a) Addition reaction occurs on a double bond.
The compound containing double bonds are also undergo polymerisation. So, the correct option are
both (i) and (iv) e.g.
CH2 = CH 2 + HOCl ¾¾
® CH 2OH - CH 2 Cl
(addition reaction)

nCH 2 = CH 2 ¾¾¾¾¾¾® -[CH 2 - CH 2-]n


Polymerisation

Polyethene
31. (c) The reaction follows Markownikoff rule
CH3 H CH3 H
d- d+
CH3CH2 – C C – CH3 + HO Cl ¾
¾® CH3CH2 – C C – CH3

OH Cl
3 -Methyl-2 -pentene
32. (b) Hydroboration oxidation reaction.
H
(1) B2H6 BH2 (2) H O , OH
2 2
OH
¾¾¾® ¾¾¾¾¾®

33. (c) ¾¾
®
(Aromatic)
34. (d) very stable carbocation.
OCOCH3
CH3 CH CHO
OCOCH3
H+
35. (a) ¾¾¾¾®
3 CrO ¾¾¾¾¾¾®
(CH 3CO) 2O - (CH 3CO) 2 O
Benzaldehyde
36. (d) The reaction is hydroboration which is addition of H2O to alkene (anti Makownikoff's's rule).
37. (a) In H 2 C = C = CH 2 hybridisation of C atoms is sp2 and sp. The molecule is monoplanar but H–
atoms lie in different planes. See allene structure.

+ –
38. (c) NaNH – + CH Br
C H ¾¾¾® CNa ¾¾¾®
2 3
C C C C CH3
H2/Pd/BaSO4

H
C

C
H3C H
cis
Hydrocarbons 155
39. (b) Bromine is more selective
\ it will form 3° halide if there is presence of 3° hydrogen.
\ Q, S, T, form 3° halide as major and P, R, U form 2° halide as major.
40. (c) Ethene is obtained by electrolysis of dipotassium succinate as follows

CH2COOK CH2COO +
¾¾¾¾® + 2K
ionization

CH2COOK CH2COO
Pot. Succinate

-
ionization
ˆˆˆˆˆˆ† +
2H 2O ‡ˆˆˆˆˆˆ 2OH + 2H
At anode :

CH2COO CH2COO CH2

– 2e ¾¾® ¾® + 2CO2
– CH2
CH2COO CH2COO
Unstable
At cathode :
2H + + 2e - ¾¾ ® [2H ] ¾¾® H2
41. (a) Anti-Markownikoff addition is possible only in case of HBr and not in HCl and HI. In HBr both the
chain initiation and propagation steps are exothermic, while in HCl, first step is exothermic, and
second step is endothermic and in HI, no step is exothermic. Hence HCl and HI do not undergo anti-
Markownikoff’s addition.
42. (a) CH3 CH2Cl CH2OH
| Cl2
| aqNa2CO 3
|
CH ¾¾¾® CH ¾¾¾¾® CH
500 °C
CH2 CH2 CH2
Propene Allyl chloride

1 2 2 1
43. (b) n-Butane, CH3 CH 2 CH 2 C H3 has two different hydrogen atoms, marked by1 and 2.
44. (a) More is the stability, lower is the heat of combustion (See Baeyer’s strain theory). Cyclohexane is
more stable.
45. (b) Chiral conformation will not have plane of symmetry. Since twist boat does not have plane of symmetry
it is chiral.
4 1
3 6

5 2
O

O
46. (d) ¾¾®
3 + O

47. (c) Oxymercuration - demercuration reaction addition of H2O molecule is according to Markovnikoff's
rule.
EBD_7587
156 Chemistry Objective MCQs

CH2OH NaHCO3 CH2OH


48. (c) CH2 = CH2 + HOCl ¾¾® ¾¾¾¾®
CH2Cl R CH2OH
(M)

CH3 CH3
+
49. (b, c) D
D – Cl
¾¾®
Formation of above species is more favourable. The stability order of carbocations is
| |
- C + < - C + < - C+
1° 2° |

Now, carbocation formed is sp2 hybridised that is triangular planar as shown below
H
H
H H Cl–
H +
H H CH 3
D Cl –

Either the Cl attack from above or below formation of the plane of the molecule, so there is
probability of resultant product as shown below i.e., both product will form.

H Cl + H CH3
CH3 Cl
D D

D CH3
Cl
H
50. (c) With Lithium in liquid ammonia, trans-alkene is almost an exclusive product.
Ph H
Li in liq. NH3
Ph – C º C – Ph ¾¾¾¾¾¾® C=C
Birch reduction
H Ph
51. (a) All are examples of electrophilic addition reactions and involve the formation of stable carbocation
leading to the formation of addition product according to Markovnikov’s rule
52. (c) Anti Markovnikov's addition
H SO
53. (c) CH3 - CH = CH 2 + H 2 O ¾¾¾¾
2 4 ® CH - CH - CH
3 3
ô
OH
2° alcohol

CH3 CH3
ô ô
H 2SO4
CH3 - C = CH 2 + H 2 O ¾¾¾¾
® CH3 - C - CH3
ô
OH
3° alcohol
Addition follows Markovnikov’s rule.
Hydrocarbons 157
54. (d) Ammonical cuprous chloride gives precipitate (red) only with terminal alkynes, i.e. alkynes having
acidic hydrogen atom, such as butyne-1 ( CH º CCH 2 CH 3 )
55. (a) Conversion of C4H6 to C4H6Br2 indicates that the compound is either butyne-1 or butyne-2. However,
white precipitate with ammonical silver nitrate solution indicates that it is a terminal alkyne, i.e.
butyne-1 and not butyne-2
56. (b) Hyperconjugation ® more a H, more reactive o, p site.
CH2–CH–CH3
CH2-CH=CH2
57. (a) (i) Hg(OAC)
¾¾¾¾¾®
2
OH
(ii) NaBH 4

58. (b) Poly-ene. e.g

3 CH º CH ¾¾
® C6 H 6 3CH3 – C º CH ¾¾¾®
acetylene Benzene Propyne

1, 3, 5-Trimethyl benzene

OH O
H2SO4 |
59. (a) CH 3 – C º CH + H 2O ¾¾¾®

=
HgSO 4 CH3 РC = CH2 ¨ CH3 C РCH3

Propyne 2-Propanone
2-Propene
(unstable) (acetone)

H
+
CH2 CH CH2 CH CH CH3

+
60. (c) H
¾¾®

3° allylic carbocation

Cl CH3
+
CH CH2 CH3 CH CH2

+
Cl
¬¾¾

(Since tertiary carbocation is more stable)


Br
Br
61. (c) X = Y= Z=
EBD_7587
158 Chemistry Objective MCQs

O OH
O LiAlH4 H+
62. (c) ¾¾®
3
Zn
¾¾¾® ¾¾¾®
D, H O 2

63. (a) Peracids give epoxides (alkene oxide) with alkene. In (b), (c), and (d) the products are wrong.
64. (c) Both (a) and (b) are correct. Wet CH3COOH gives cis addition and dry CH 3COOH gives trans
addition products.
65. (c) Peracids gives trans hydroxylated products with alkenes.
(i) BH H + /Hg 2 +
66. (c) CH3CH 2CH 2 CHO ¬¾¾¾
3
¾ CH3CH 2 C º CH ¾¾¾¾¾
® CH3CH 2 COCH3
(an aldehyde) (ii) H 2O2 /OH - (A ketone with –COCH3 group)

O
||
67. (b) – NH – C – CH3 (+ M group) (More activating group)
68. (a) – CH3 > – CD3 > – CT3
Order of hyperconjugation on the basis of bond energy. Strong bond means less hyperconjugation.
69. (c) An aromatic compound have cyclic clouds of delocalised (4n + 2) p electrons above and below the
plane of the molecule. Among the given three compounds, only compound (c) satisfies these
conditions.

sp 3

NO2 CH3
H H
+ N
+
H
Carbon bearing –NO2 (or –CH3) group is sp 3 4p electrons and 2 non-bonding
hybridised and does not have a p orbital with the electrons (present in p orbital of N)
result delocalisation of p electrons over the complete form a cyclic cloud of 6p electrons
ring is interrupted.

70. (b) It is an example of intramolecular Wurtz reaction


® Na + + e –
Na ¾¾

FR
e –
e – + Cl ¾¾ ¾ Br ¾¾® Cl ¾ ¾¾® ¾¾¾®
Combination
– Br – +
– Br
Environmental
14
Chemistry
1. Which of the following statement is false? 6. Soil is polluted by
(a) London smog is oxidising in nature I. pesticides
(b) Photochemical smog causes irritation in II. synthetic fertilizers
eyes III. green manure
(c) London smog is a mixture of smoke and fog Choose the correct option.
(d) Photochemical smog results in the
(a) I and III (b) I and II
formation of PAN
(c) II and III (d) I, II and III
2. Which of the following acts as a sink for CO?
(a) Plants 7. Identify the pollutant gases largely responsible
(b) Haemoglobin for the discoloured and lustreless nature of
(c) Microorganisms present in the soil marble of the Taj Mahal?
(d) Oceans (a) O3 and CO2 (b) CO2 and NO2
3. Global warming is due to increase of: (c) SO2 and NO2 (d) SO2 and O3
(a) methane and nitrous oxide in atmosphere. 8. Which of the following is/are the hazardous
(b) methane and CO2 in atmosphere. pollutant(s) present in automobile exhaust
(c) methane and O3 in atmosphere. gases?
(d) methane and CO in atmosphere. (i) N2 (ii) CO
4. Which of the following statements about the (iii) CH4 (iv) Oxides of nitrogen
depletion of ozone layer is correct?
(a) (ii) and (iii) (b) (i) and (ii)
(c) (ii) and (iv) (d) (i) and (iii)
9. Which of the following is the major cause of
(a) The problem of ozone depletion is less global warming?
serious at poles because NO2 solidifies and (a) re-radiation of U.V. rays by CO2 and H2O.
is not available for consuming CIO· radicals. (b) re-radiation of I.R. rays by CO2 and H2O.
(b) The problem of ozone depletion is more (c) re-radiation of I.R. rays by O2 and N2.
serious at poles because ice crystals in the (d) re-radiation of U.V. rays by O2 and N2.
clouds over poles act as catalyst for
10. The false statement among the followings :
photochemical reactions involving the
(a) The average residence time of NO is one
decomposition of ozone of Cl· and CIO·
radicals. month.
(c) Freons, chlorofluorocarbons, are inert. (b) Limestone acts as a sink for SOx.
Chemically, they do not react with ozone in (c) SOx can be removed from flue gases by
stratosphere. passing through a solution of citrate ions.
(d) Oxides of nitrogen also do not react with (d) Ammonia acts as a sink for NOx.
ozone in stratosphere. 11. Phosphate fertilizers when added to water leads
5. Excess nitrate in drinking water can cause to
(a) methemoglobinemia (a) increased growth of decomposers.
(b) kidney damage (b) reduced algal growth.
(c) liver damage (c) increased algal growth.
(d) laxative effect
(d) nutrient enrichment (eutrophication).
EBD_7587
160 Chemistry Objective MCQs
12. Phosphate pollution is caused by 20. The aromatic compounds present as particulates
(a) sewage and agricultural fertilizers are
(b) weathering of phosphate rocks only (a) polycyclic aromatic hydrocarbons
(c) agricultural fertilizers only (b) benzene
(d) phosphate rocks and sewage (c) toluene
13. When rain is accompanied by a thunderstorm, (d) nitrobenzene
the collected rain water will have a pH value 21. The substance having the largest concentration
in acid rain?
(a) H2CO3 (b) HNO3
(c) HCl (d) H2SO4
(a) slightly lower than that of rain water without 22. BOD is connected with
thunderstorm. (a) microbes and organic matter.
(b) slightly higher than that when the (b) organic matter.
thunderstorm is not there. (c) microbes.
(c) uninfluenced by occurrence of thunderstorm. (d) None of the above
(d) which depends upon the amount of dust in 23. What is the concentration of dissolved oxygen
air. in cold water?
(a) 5 ppm (b) 10 ppm
14. Minamata disease of Japan is due to pollution
(c) 200, 000 ppm (d) 100 ppm
of
24. Green chemistry means such reactions which :
(a) Aresenic (b) Lead
(a) Produce colour during reactions.
(c) Cyanide (d) Mercury
(b) Reduce th e use an d production of
15. Which causes death of fishes in water bodies
hazardous chemicals.
polluted by sewage? (c) Are related to the depletion of ozone layer.
(d) Study the reactions in plants.
(a) Foul smell
25. Use of which of the following solvent in dry
(b) Pathogens
cleaning will result in less harm to ground water?
(c) Clogging of gills by silt (a) Cl2C = CCl2 (b) Liquid CO2
(d) Decrease in D.O. (c) H2O2 (d) None of these
16. Photochemical smog consists of excessive 26. The statement which is not true
amount of X, in addition to aldehydes, ketones, (a) NO 2 does not play any role in
peroxyacetyl nitrate (PAN), and so forth X is : photochemical smog
(a) CO (b) CH4 (c) O3 (d) CO2 (b) SO3 is more harmful air polluntant than SO2
17. Water sample is reported to be highly polluted if (c) SO2 dos not affect larynx (voice box)
BOD (Biological Oxygen Demand) value of (d) NO is more toxic to living tissues than NO2
sample becomes 27. Synthesis of ethanal commercially from which
(a) more than 17 ppm. (b) equal to 10 ppm. of the following reagent is the part of green
(c) equal to 5 ppm. (d) less than 5 ppm. chemistry?
18. The region which is greatly affected by air
pollution is (a) CH3 CH2OH (b) CH2 = CH2
(a) thermosphere (b) stratosphere (c) HC º CH (d) All of these
(c) troposphere (d) mesosphere 28. Green house gases can be arranged in ‘Global
19. Which of the following is most abundant Warming Potential’ sequence as
hydrocarbon pollutant? (a) N2O > CFC > CH4 > CO2
(a) Butane (b) Ethane (b) CFC > N2O > CH4 > CO2
(c) Methane (d) Propane (c) CFC > CO2 > N2O > CH4
(d) CO2 > CFC > N2O > CH4
Environmental Chemistry 161
29. Addition of phosphate fertilisers to water atmosphere while doesn't allow longer wave-
bodies causes: length to leave the earth's atmosphere.
(a) Increase in amount of dissolved oxygen in (b) allow longer wavelength to enter earth at-
water mosphere while doesn't allow shorter wave-
(b) Deposition of calcium phosphate length to leave the surface
(c) Increase in fish population (c) do not have wavelength specific character.
(d) Enhanced growth of algae (d) show wavelength specific behaviour near
30. Which forms th e part of hazy fumes of the earth while far from earth these have
photochemical smog? wavelength independent behaviour.
(a) SO2 (b) Nitrogen dioxide 37. The greenhouse effect is because of the
(c) PAN formation (d) Aldehydes (a) presence of gases, which in general are
31. Which one of the following depletes ozone layer? strong infrared absorbers, in the atmosphere
(a) CO (b) NO and freons (b) presence of CO2 only in the atmosphere
(c) SO2 (d) CO2 (c) pressure of O3 and CH4 in the atmosphere
32. Which of the following chemical, harmful to
(d) N2O and chlorofluorohydrocarbons in the
ozone, is released by chlorofluorocarbon?
(a) Sulphur dioxide (b) Fluorine atmosphere
(c) Chlorine (d) Nitrogen dioxide 38. Carbon monoxide (CO) is harmful to man because
33. In Antarctica ozone depletion is due to the (a) it forms carbolic acid.
formation of following compound (b) it generates excess CO2.
(a) acrolein (c) it is carcinogenic.
(b) peroxyacetyl nitrate
(d) it competes with O2 for haemoglobin.
(c) SO2 and SO3
(d) chlorine nitrate 39. Which of the following can control the
34. Ozone is an important constituent of stratosphere photochemical smog?
because it (A) Use of catalytic converters in automobiles.
(a) destroys bacteria which are harmful to (B) Plantation of trees like pinus, pyrus vitis
human life. etc.
(b) prevents the formation of smog over large
(C) Using less sulphur containing fossil fuels.
cities.
(c) removes poisonous gases of the (a) A and C (b) B
atmosphere by reacting with them. (c) A and B (d) A, B and C
(d) absorbs ultraviolet radiation which is 40. High concentration of which of the following in
harmful to human life. atmosphere leads to stiffness of flower buds
35. The statement which is not correct about control which eventually fall off from plants?
of particulate pollution: (a) NO2 (b) SO2 (c) CFC (d) Smog
(a) In electrostatic precipitator, the particulates 41. Identify the wrong statement in the following:
are made to acquire positive charge which
are then attracted by the negative electrode
and removed.
(b) Gravity settling chamber removes larger (a) Chlorofluorocarbons are responsible for
particles from the air. ozone layer depletion.
(c) Cyclone collector removes fine particles in
(b) Greenhouse effect is responsible for global
the diameter range 5-20 microns.
(d) Wet scrubbers are used to wash away all warming.
types of particulates. (c) Acid rain is mostly because of oxides of
36. Green house gases nitrogen and sulphur.
(d) Ozone layer does not permit infrared
(a) allow shorter wavelength to enter earth's radiation from the sun to reach the earth.
EBD_7587
162 Chemistry Objective MCQs
42. Ozone hole refers to (iv) Mists, smoke and dust
(a) increase in concentration of ozone. (v) Smog
(b) hole in ozone layer. (a) (i), (iv) and (v) (b) (iii) only
(c) reduction in thickness of ozone layer in (c) (ii) only (d) (ii) and (v)
troposphere. 48. Which one of the following substances used
(d) reduction in thickness of ozone layer in in dry cleaning is a better strategy to control
stratsophere. environmental pollution?
43. Which one of the following statements is not (a) Sulphur dioxide
true? (b) Carbon dioxide
(c) Nitrogen dioxide
(a) Dissolved oxygen (DO) in cold water can (d) Tetrachloroethylene
reach a concentration upto 10 ppm. 49. In a coal fired power plant, electrostatic
(b) Clean water would have a BOD value of precipitators are installed to control emission of
5 ppm. (a) SO2 (b) NOx (c) SPM (d) CO
(c) Fluoride deficiency in drinking water is 50. Which among the following statements is false?
harmful. Soluble fluoride is often used to (a) Oil slick in sea water increases D.O. value
bring its concentration upto 1 ppm. (b) The main reason for river water pollution is
(d) When the pH of rain water is higher than industrial and domestic sewage discharge
6.5, it is called acid rain. (c) Surface water contains a lot of organic
44. The smog is essentially caused by the presence of matter mineral nutrients and radioactive
(a) oxides of sulphur and nitrogen materials
(b) O2 and N2 (d) Oil spill in sea water causes heavy damage
(c) O2 and O3 to fishery
(d) O3 and N2 51. Lichens do not like to grow in cities
45. Presence of which of the following fuel gas in (a) because of absence of the right type of
the exhaust fumes shows incomplete algae and fungi
combustion of fuel? (b) because of lack of moisture
(a) Sulphur dioxide (c) because of SO2 pollution
(b) Carbon monoxide and water vapour (d) because natural habitat is missing
(c) Carbon monoxide 52. Which of the following statement(s) is/are
(d) Nitrogen dioxide correct?
46. Which of the following statements about polar (i) Classical smog is a mixture of smoke, fog
stratosphere clouds (PSCs) is not correct? and sulphur dioxide.
(ii) Classical smog is also called oxidising smog
(iii) Hydrocarbons, NO 2 and PAN are
components of photochemical smog.
(a) PSCs do not react with chlorine nitrate and
(a) (i) and (iii) (b) (i) and (ii)
HCl.
(c) (iii) only (d) (i), (ii) and (iii)
(b) Type I clouds are formed at about –77ºC
53. Which one of the following pairs is mismatched
and contain solid HNO3 . 3H2O.
(a) Fossil fuel burning - release of CO2
(c) Type II clouds are formed at about –85ºC (b) Nuclear power - radioactive wastes
and contain some ice. (c) Solar energy - Greenhouse effect
(d) A tight whirlpool of wind called Polar Vortex (d) Biomass burning - release of CO2
is formed which surrounds Antarctica. 54. In almost all Indian metropolitan cities like Delhi,
47. The irritant red haze in the traffic and congested the major atmospheric pollutant(s) is/are
places is due to presence of which of the (a) suspended particulate matter (SPM)
following? (b) oxides of sulphur
(i) Oxides of sulphur (c) carbon dioxide and carbon monoxide
(ii) Oxides of nitrogen (d) oxides of nitrogen
(iii) Carbon dioxide
Environmental Chemistry 163
55. Which of the following statements are not 58. Which of the following statement(s) is/are true
correct? about waste recycling?

(i) Clothes can be made from recycled plastic


(i) F– ion concentration above 2 ppm causes waste.
brown mottling in teeth. (ii) Fuel that has high octane rating and
(ii) Excessive F– (over 10 ppm) causes harmful contains no lead can be obtained from
effect to bones and teeth. plastic waste.
(iii) Excessive lead in drinking water causes (iii) Technology has now been developed to
disease methemoglobinemia produce electricity from the garbage.
(iv) Deficiency of sulphate in drinking water (a) (ii) only (b) (ii) and (iii)
causes laxative effect. (c) (iii) only (d) All of these
(a) (ii) and (iv) (b) (ii) and (iii) 59. Which of the following is/are formed when
(c) (ii), (iii) and (iv) (d) (iii) and (iv) ozone reacts with the unburnt hydrocarbons in
56. Which of the following statements is incorrect?
(a) Smoke particulates consist of solid or polluted air?
mixture of solid and liquid particles formed
during combustion of organic matter. (i) Formaldehyde
(b) Herbicides and insecticides that miss their (ii) Acrolein
target and travel through air form mists. (iii) Peroxyacetyl nitrate
(c) Organic solvents, metals and metallic oxides (iv) Formic acid
form fume particles (a) (i) and (iv) (b) (ii) only
(d) None of these (c) (iii) only (d) (i), (ii) and (iii)
57. Which of the following practices will not come 60. Which one of the following statement is not true?
under green chemistry? (a) pH of drinking water should be between
(a) If possible, making use of soap made of 5.5 – 9.5.
vegetable oils instead of using synthetic (b) Concentration of DO below 6 ppm is good
detergents for the growth of fish.
(b) Using H2O2 for bleaching purpose instead (c) Clean water would have a BOD value of less
of using chlorine based bleaching agents than 5 ppm.
(c) Using bicycle for travelling small distances (d) Oxides of sulphur, nitrogen and carbon are
instead of using petrol/diesel based vehicles
the most widespread air pollutant.
(d) Using plastic cans for storing substances

Answer KEYs
1 (a) 7 (c) 13 (a) 19 (c) 25 (b) 31 (b) 37 (a) 43 (d) 49 (c) 55 (d)
2 (c) 8 (c) 14 (d) 20 (a) 26 (b) 32 (c) 38 (d) 44 (a) 50 (a) 56 (d)
3 (b) 9 (b) 15 (d) 21 (d) 27 (b) 33 (a) 39 (c) 45 (c) 51 (c) 57 (d)
4 (c) 10 (a) 16 (c) 22 (a) 28 (b) 34 (d) 40 (b) 46 (a) 52 (a) 58 (d)
5 (a) 11 (d) 17 (a) 23 (b) 29 (d) 35 (a) 41 (d) 47 (c) 53 (c) 59 (d)
6 (b) 12 (a) 18 (c) 24 (b) 30 (b) 36 (a) 42 (d) 48 (d) 54 (a) 60 (b)
EBD_7587
164 Chemistry Objective MCQs

1. (a) London smog is reducing in nature. 14. (d) Minamata is caused by Hg poisoning.
2. (c) CO is converted into CO2 by microorganism 15. (d) Decrease in D.O causes death of fish
present in soil. 16. (c) Chemical pollutants in photochemical smog
3. (b) The main gas responsible for the global
are nitrogen oxides (NO and NO2), volatile
warming is carbon dioxide. Other
organ ic compoun ds, ozone (O 3) and
contributors include methane released from
peroxyacetyl nitrate.
landfills and agriculture.
4. (c) Freons and chlorofluorocarbon find their 17. (a) Water sample is reported to be highly
way to stratosphere through without being polluted if BOD value of sample is more
destroyed in the troposphere due to their than 17 ppm.
low reactivity. In the stratosphere the Cl and 18. (c) Air pollution greatly affect the troposphere.
Br atoms are liberated from the parent 19. (c) Most abundant hydrocarbon pollutant is
compounds by the action of ultraviolet light methane.
e.g. 20. (a) PAH (Poly Aromatic Hydrocarbon)
CF2 Cl2 + hn ¾¾
® CF2Cl + Cl 21. (d) Acid rain contains H2SO4 > HNO3> HCl.
CFCl3 + h n ¾¾
® CFCl 2 + Cl 22. (a) BOD is connected with microbes and
The Cl atoms can destroy ozone molecules organic matter
through a variety of catalytic cycles. 23. (b) In cold water, dissolved oxygen can reach
5. (a) Excessive concentration of nitrate in a concentration upto 10 ppm, whereas
drinking water is harmful and can cause oxygen in air is about 200, 000 ppm.
methemoglobinemia (blue baby syndrome). 24. (b) Green chemistry may be defined as the
6. (b) Pesticides and synthetic fertilizers pollute programme of developing new chemical
the soil.
products and chemical processes or making
7. (c) SO 2 and NO2 are responsible for the
improvements in the already existing
discoloured and lustureless nature of
compounds and processes so as to make
marble.
8. (c) CO and oxides of Nitrogen are poisnous less harmful to human h ealth and
gases present in automobile exhaust gases. environment. This means the same as to
9. (b) reduce the use and production of hazardous
10. (a) The average residence time of NO is 4 days. chemicals.
11. (d) Addition of phosphate fertilizers to water i.e. correct answer is option (b).
leads to nutrient enrichment (eutrophication) 25. (b) Replacement of earlier used tetra-
12. (a) Phosphate pollution is caused by sewage chloroethene as solvent for dry cleaning
and agricultural fertilizers.
by liquid CO2 results in less harm to
13. (a) Normal rain water has pH 5.6. Thunderstorm
ground water.
results in the formation of NO and HNO3
which lowers the pH. 26. (b) SO3 is more harmful pollutant than SO2.
Environmental Chemistry 165
27. (b) Ethanal is commerically prepared by one Juniparus. Quercus, Pyrus and Vitis can
step oxidation of ethene in the presence metabolise nitrogen oxide and therefore,
of ionic catalyst in aqueous medium with their plantation could help in this matter.
an yield of 90%. 40. (b) High concentration of SO2 leads to
Catalyst
stiffness of flower buds.
CH2 = CH2 + O2 ¾¾¾¾¾¾
Pb(II)|Cu(II)
® CH3CHO 41. (d) Ozone layer acts as a shield and does not
(in water) allow ultraviolet radiation from sun to reach
28. (b) CFC > N2O > CH4 > CO2 earth. It does not prevent infra-red radiation
29. (d) from sun to reach earth, thus option (d) is
30. (b) NO2 forms a part of photochemical smog. wrong statement and so it is the correct
31. (b) NO and freons are responsible for ozone answer.
layer depletion. 42. (d) Ozone hole is reduction in ozone layer in
stratosphere.
hn •
32. (c) CF2 Cl 2 ¾¾® CF2 Cl + Cl 43. (d) Acid rain is the rain water containing
• • sulphuric acid and nitric acid which are
Cl + O3 ¾
¾® Cl O + O 2 formed from the oxides of sulphur and
33. (a) In antarctica, ozone depletion is due to nitrogen present in the air as pollutants and
formation of acrolein. rain water has a pH range of 4-5.
34. (d) Ozone absorbs U.V. radiations harmful to 44. (a) Smog is caused by oxides of sulphur and
human life. nitrogen.
35. (a) Particulates acquire negative charge and 45. (c) Presence of CO in the exhaust fumes shows
are attracted by the positive electrode. incomplete combustion.
36. (a) Radiation coming from sun or outerspace 46. (a) PSCs react with chlorine nitrate and HCl to
have high energy or short wavelength, give HOCl and Cl2.
which are allowed to enter by green house 47. (c) The irritant red haze in the traffic and
gases. However, radiation emitted by earth congested places is due to presence of
is in infrared region, having long oxides of nitrogen.
wavelength, are reflected back by the 48. (d) Tetrachloroethylene is widely used for dry-
envelope of green house gases.
cleaning fabrics and metal degreasing
37. (a) Green house gases such as CO2, ozone,
operations.
methane, the chlorofluorocarbon compounds
49. (c) SPM ® Suspended particulates matter.
and water vapour form a thick cover around
50. (a) Oil slick in sea water decreases D.O value.
the earth which prevents the IR rays emitted
51. (c) Because lichens are very sensitive to
by the earth to escape. It gradually leads to
sulphur dioxide and in cities the amount of
increase in temperature of atmosphere.
SO2 is high, so they do not grow in cities.
38. (d) CO is highly toxic and impairs respiration.
52. (a) Classical smog is also called reducing
CO combines with haemoglobin of blood
smog.
and reduces its O2 carrying capacity.
53. (c) Solar energy is not responsible for green
39. (c) Usually catalytic converters are used in
the automobiles, which prevent the release house effect instead it is a source of energy
of nitrogen oxide and hydrocarbons to the for the plants and animals.
atmosphere. Certain plants e.g., Pinus, 54. (a)
EBD_7587
166 Chemistry Objective MCQs

55. (d) For statement (iii), Methemoglobinemia 59. (d) 3CH4 + 2O3 ® 3CH2= O + 3H2O
(blue baby syndrome) is caused due to Formaldehyde
excess of nitrate in drinking water. CH2 == CHCH == O; CH3COONO2
For statement (iv), Excessive sulphate Acrolein Peroxyacetyl
(> 500 ppm) in drinking water causes nitrate (PAN)

laxative effect, otherwise at moderate levels 60. (b) The ideal value of DO for growth of fishes
it is harmless. is 8 mg/L. 7 mg/L is desirable range, below
56. (d) 57. (d) this value fishes get susceptible to disease.
58. (d) All the given statements are true for about A value of 2 mg/L or below is lethal for fishes.
waste recycling.
15 The Solid State
1. How many unit cells are present in a cube-shaped 8. First three nearest neighbour distances for body
ideal crystal of NaCl of mass 1.00 g ? centred cubic lattice are respectively:
[Atomic masses : Na = 23, Cl = 35.5]
(a) 5.14 × 1021 unit cells
(b) 1.28 × 1021 unit cells a
(c) 1.71 × 1021 unit cells (a) 2a,a, 3a (b) , a, 3a
2
(d) 2.57 × 1021 unit cells
2. In a solid ‘AB’ having the NaCl structure, ‘A’ 3a 3a
(c) , a, 2a (d) , a, 3a
atoms occupy the corners of the cubic unit cell. 2 2
If all the face-centered atoms along one of the 9. A crystalline solid
axes are removed, then the resultant stoichiometry (a) changes abruptly from solid to liquid when
of the solid is heated.
(b) has no definite melting point.
(c) undergoes deformation of its geometry
(a) AB2 (b) A2B (c) A4B3 (d) A3B4
easily.
3. Edge length of a cube is 400 pm. Its body diagonal
(d) has an irregular 3-dimensional arrangements.
would be
10. Which of the following is not a crystalline solid?
(a) 500 pm (b) 566 pm
(a) KCl (b) CsCl
(c) 600 pm (d) 693 pm
(c) Glass (d) Rhombic S
4. The second order Bragg diffraction of X-rays with 11. Which one of the following forms a molecular
= 1.00 Å from a set of parallel planes in a metal solid when solidified?
occurs at an angle 60º. The distance between the
(a) Silicon carbide (b) Calcium fluoride
scattering planes in the crystal is
(c) Rock salt (d) Methane
(a) 0.575 Å (b) 1.00 Å
12. The intermetallic compound LiAg crystallizes in
(c) 2.00 Å (d) 1.15 Å a cubic lattice in which both lithium and silver
5. + –
The radii of Na and Cl ions are 95 pm and 181 atoms have coordination number of eight. To
pm respectively. The edge length of NaCl unit what crystal class does the unit cell belong?
cell is (a) Simple cubic (b) Face-centred cubic
(c) Body-centred cubic(d) None of these
(a) 276 pm (b) 138 pm
(c) 552 pm (d) 415 pm 13. Which set of following characteristics for ZnS
crystal is correct?
6. CsBr has bcc structure with edge length 4.3. The
shortest interionic distance in between Cs+ and (a) Coordination number (4 : 4); ccp; Zn2+ ion
Br– is in the alternate tetrahedral voids.
(a) 3.72 (b) 1.86 (c) 7.44 (d) 4.3 (b) Coordination number (6 : 6); hcp; Zn2+ ion
in all tetrahedral voids.
7. Potassium crystallizes with a
(c) Coordination number (6 : 4); hcp; Zn2+ ion
(a) body-centred cubic lattice
in all octahedral voids.
(b) face-centred cubic lattice
(d) Coordination number (4 : 4); ccp; Zn2+ ion
(c) simple cubic lattice in all tetrahedral voids.
(d) orthorhombic lattice
EBD_7587
168 Chemistry Objective MCQs
14. The ionic radii of X+ and Y– ions are 146 and 216 22. The appearance of colour in solid alkali meta
pm respectively. The probable type of structure halides is generally due to:
shown by it is (a) Schottky defect (b) Frenkel defect
(a) CsCl type (b) Rock salt type (c) Interstitial position(d) F-centres
(c) Zinc blende type (d) CaF2 type 23. A solid AB crystallises as NaCl structure and the
15. In CsCl type structure, the co-ordination number radius of the cation is 0.100nm. The maximum
of Cs+ and Cl– respectively are radius of the anion can be:
(a) 6, 6 (b) 6, 8 (c) 8, 8 (d) 8, 6 (a) 0.137 nm (b) 0.241 nm
16. Copper crystallises in fcc with a unit length of (c) 0.274 nm (d) 0.482 nm
361pm. What is the radius of copper atom ? 24. In a monoclinic unit cell, the relation of sides and
(a) 157pm (b) 128pm angles are respectively:
(c) 108pm (d) 181pm (a) a = b ¹ c and a = b = g = 90°
17. In a face centred cubic lattice, atoms of A form (b) a ¹ b ¹ c and a = b = g = 90°
the corner points and atoms of B form the face (c) a ¹ b ¹ c and b = g = 90° ¹ a
centred points. If two atoms of A are missing (d) a ¹ b ¹ c and a ¹ b ¹ g ¹ 90°
from the corner points, the formula of the ionic 25. A solid is made of two elements X and Z. The
compound is : atoms Z are in ccp arrangement while the atoms
(a) AB3 (b) AB4 (c) A2B5 (d) AB2 X occupy all the tetrahedral sites. What is the
18. Which one of the following statements about
formula of the compound?
packing in solids is incorrect ?
(a) Coordination number in bcc mode of
(a) XZ (b) XZ 2
packing is 8.
(b) Coordination number in hcp mode of (c) X 2 Z (d) X 2 Z 3
packing is 12. 26. Among the following which is the best
(c) Void space in hcp mode of packing is 32%. description of water in the solid phase?
(d) Void space is ccp mode of packing is 26%. (a) Covalent solid (b) Molecular solid
19. Among the following the incorrect statement is (c) Ionic solid (d) Network solid
27. CsBr crystallises in a body centered cubic lattice.
The unit cell length is 436.6 pm. Given that the
(a) Density of crystals remains unaffected due atomic mass of Cs = 133 and that of Br = 80 amu
to Frenkel defect. and Avogadro number being 6.02 × 1023 mol–1,
(b) In BCC unit cell the void space is 32%. the density of CsBr is
(c) Density of crystals decreases due to (a) 0.425 g/cm3 (b) 8.5 g/cm3
(c) 4.25 g/cm 3 (d) 82.5 g/cm3
Schottky defect.
(d) Electrical conductivity of semiconductors 28. Compound that will show the highest lattice
and metals increases with increase in energy
temperature. (a) KF (b) NaF (c) CsF (d) RbF
20. In a face centered cubic lattice atoms A are at the 29. The edge length of unit cell of a metal having
corner points and atoms B at the face centered molecular weight 75 g/mol is 5Å which crystallizes
points. If atom B is missing from one of the face in cubic lattice. If the density is 2g/cc then find
centered points, the formula of the ionic the radius of metal atom. (NA = 6 × 1023). Give the
compound is: answer in pm.
(a) AB2 (b) A5B2 (c) A2B3 (d) A2B5
21. A cubic crystal has (a) 217 pm (b) 210 pm
(a) thirteen planes of symmetry and nine axis (c) 220 pm (d) 205 pm
of symmetry. 30. Ammonium chloride crystallizes in a body centred
(b) thirteen axis of symmetry and nine planes cubic lattice with edge length of unit cell of 390
of symmetry. pm. If the size of chloride ion is 180 pm, the size
(c) thirteen axis and thirteen planes of of ammonium ion would be
symmetry. (a) 174 pm (b) 158 pm
(d) nine planes and nine axis of symmetry. (c) 142 pm (d) 126 pm
The Solid State 169
31. The cubic unit cell of a metal (molar mass 39. AB; crystallizes in a body centred cubic lattice
= 63.55g mol–1) has an edge length of 362 pm. Its with edge length ‘a’ equal to 387 pm. The distance
density is 8.92g cm–3. between two oppositely charged ions in the lattice
is
The type of unit cell is
(a) 335 pm (b) 250 pm
(a) primitive (b) face centered (c) 200 pm (d) 300 pm
(c) body centered (d) end centered 40. The number of unit cells in 58.8 g of NaCl is nearly
32. A solid has a ‘bcc’ structure. If the distance of (a) 6 × 1020 (b) 3 × 1022
(c) 1.5 × 10 23 (d) 0.5 × 1024
nearest approach between two atoms is 1.73Å,
the edge length of the cell is 41. An element having an atomic radius of 0.14 nm
crystallizes in an fcc unit cell. What is the length
(a) 314.20 pm (b) 1.41 pm
of a side of the cell ?
(c) 200 pm (d) 216 pm (a) 0.56 nm (b) 0.24 nm
33. When molten zinc is converted into solid state, it (c) 0.96 nm (d) 0.4 nm
acquires hcp structure. The number of nearest 42. Which of the following has maximum value of cation/
neighbours of Zn will be
anion ratio?
(a) 6 (b) 12 (c) 8 (d) 4
34. The radius of a calcium ion is 94 pm and of the
(a) KCl (b) NaCl
oxide ion is 146 pm. The possible crystal structure
(c) CaF2 (d) MgCl2
of calcium oxide will be
(a) tetrahedral (b) trigonal 43. Which of the following statements is correct for
ionic solids ?
(c) octahedral (d) pyramidal
(a) Ionic solids are soluble in CCl4, C6H6, etc.
35. An alloy of copper, silver and gold is found to
have cubic lattice in which Cu atoms constitute (b) Under the electric field cations and anions
acquire translatory motion in opposite
ccp. If Ag atoms are located at the edge centres directions.
and Au atom is present at body centre, the alloy
(c) Structural units have strong electrostatic
will have the formula force of attraction.
(d) Structural units have dipole-dipole
(a) CuAgAu (b) Cu4Ag4Au interactions.
(c) Cu4Ag3Au (d) Cu4Ag6Au 44. Total volume of atoms present in a face-centred
36. A solid has a structure in which ‘W’ atoms are cubic unit cell of a metal is (r is atomic radius)
located at the corners of a cubic lattice ‘O’ atoms 12 3 16 3
at the centre of edges and Na atoms at the centre (a) pr (b) pr
3 3
of the cube. The formula for the compound is
20 3 24 3
(a) Na2WO3 (b) Na2WO2 (c) pr (d) pr
3 3
(c) NaWO2 (d) NaWO3 45. An element occuring in the bcc structure has
37. If calcium crystallizes in bcc arrangement and 12.08 × 1023 unit cells. The total number of atoms
the radius of Ca atom is 96 pm, then the volume of the element in these cells will be
of unit cell of Ca is (a) 24.16 × 1023 (b) 36.18 × 1023
(a) 10.9 × 10–36 m3 (b) 10.9 × 10–30 m3 (c) 6.04 × 1023 (d) 12.08 × 1023
(c) 21.8 × 10 m –30 3 (d) 21.8 × 10–36 m3 46. In an antifluorite structure, cations occupy
38. How many unit cells are present in 5.0 gm of (a) octahedral voids (b) centre of cube
crystal AB (formula mass of AB = 40) having (c) tetrahedral voids (d) corners of cube
rock salt type structure? (NA = Avogadro's no.) 47. The number of atoms in 100 g of an fcc crystal
with density, d = 10 g/cm3 and cell edge equal to
NA
(a) NA (b) 100 pm, is equal to
10
(c) 4 NA (d) none of these (a) 1 × 1025 (b) 2 × 1025
(c) 3 × 1025 (d) 4 × 1025
EBD_7587
170 Chemistry Objective MCQs
48. Crystal defect indicated in the diagram below is 54. The r+ / r– ratio of ZnS is 0.402. Pick out the
incorrect statements from the following?
Na + Cl - Na + Cl - Na + Cl -
Cl– c Cl– Na+ c Na+
Na+Cl– c Cl– Na+ Cl–
(a) ZnS is 4 : 4 coordination compound.
Cl– Na+ Cl–Na+ c Na+ (b) ZnS does not crystallize in rock salt type
(a) Interstitial defect lattice because r+ / r– is too small to avoid
(b) Schottky defect overlapping of S2– ions.
(c) Frenkel defect (c) Zn2+ ion is too small to fit precisely into the
octahedral voids of S2– ions.
(d) Frenkel and Schottky defects
(d) Zn2+ ion is too large to fit into the octahedral
49. In the laboratory, sodium chloride is made by voids of S2– ions.
burning sodium in the atmosphere of chlorine. 55. For fcc, if AB is just like the rock salt like structure
The salt obtained is yellow in colour. The cause then, A+ and B– are located at –
of yellow colour is: (a) A+ - Tetrahedral voids ; B– - Corner
(b) A+ - Corner and faces ; B– - Octahedral voids
(a) Presence of Na+ ions in the crystal lattice (c) A+ - Octahedral voids ; B– - Corner and faces
(b) Presence of Cl– ions in the crystal lattice (d) A+ - Corner and faces ; B– - Octahedral and
(c) Presence of electrons in the crystal lattice tetrahedral voids
(d) Presence of face centred cubic crystal lattice. 56. A metal has a fcc lattice. The edge length of the
50. Each of the following solids show, the Frenkel unit cell is 404 pm. The density of the metal is
2.72 g cm-3. The molar mass of the metal is : (NA
defect except
Avogadro’s constant = 6.02 × 1023 mol–1)
(a) ZnS (b) AgBr (c) AgI (d) KCl (a) 30 g mol–1 (b) 27 g mol–1
51. Schottky defect in crystals is observed when (c) 20 g mol –1 (d) 40 g mol–1
(a) an ion leaves its normal site and occupies 57. What is the energy gap between valence band
an interstitial site. and conduction band in crystal of insulators ?
(b) unequal number of cations and anions are (a) Both the bands are overlapped with each
other
missing from the lattice.
(b) Very small
(c) density of the crystal increases. (c) Infinite
(d) equal number of cations and anions are (d) Very large
missing from the lattice. 58. The addition of arsenic to germanium makes the
52. In the solid state, MgO has the same structure as latter a
that of sodium chloride. The number of oxygens (a) metallic conductor
surrounding each magnesium in MgO is (b) intrinsic semiconductor
(c) mixed conductor
(a) 6 (b) 1 (c) 2 (d) 4 (d) extrinsic semiconductor
53. Which is incorrect statement ? 59. A mineral having the formula AB2 crystallizes in
ccp lattice with A atoms occupying the lattice
points. Pick out the correct statement of the
following
(a) In NaCl structure, tetrahedral voids are (a) 100% occupancy of tetrahedral voids, C.N.
unoccupied. of B = 4
(b) In ZnS structure, octahedral voids are (b) 100% occupancy of octahedral voids, C.N.
unoccupied. of B = 4
(c) In CaF2 structure, all tetrahedral voids are (c) 50% occupancy of tetrahedral voids, C.N.
occupied. of A = 4
(d) In Na2O structure, all tetrahedral voids are (d) 100% occupancy of octahedral voids, C.N.
of A = 4
unoccupied.
The Solid State 171
60. Superconductors are derived from compounds 69. A NaCl crystal is changed into CsCl type structure
of by:
(a) p-block elements (b) lanthanides
(c) actinides (d) transition elements
61. If NaCl is doped with 10– 4 mol % of SrCl2, the (a) Increasing temperature and decreasing
concentration of cation vacancies will be pressure.
(NA = 6.02 × 1023 mol–1) (b) Increasing pressure and decreasin g
(a) 6.02 × 1016 mol–1 (b) 6.02 × 1017 mol–1 temperature.
(c) 6.02 × 1014 mol–1 (d) 6.02 × 1015 mol–1 (c) Increasing both temperature and pressure
62. Predict coordination number of the cation in (d) None of these.
crystals of the following compounds: 70. To get a n- type semiconductor, the impurity to
be added to silicon should have which of the
following number of valence electrons
1. MgO : rc = 0.65 Å; r a = 1.40 Å, (a) 1 (b) 2 (c) 3 (d) 5
2. MgS : r c = 0.65 Å; r a = 1.84 Å, 71. The empty space in the body centred cubic lattice
(a) 6, 4 (b) 4, 6 (c) 3, 4 (d) 6, 8 is
63. An element (atomic mass = 100 g / mol) having (a) 68% (b) 52.4% (c) 47.6% (d) 32%
bcc structure has unit cell edge 400 pm. Then, 72. A solid with high electrical and thermal
density of the element is conductivity is
(a) 10.376 g/cm3 (b) 5.188 g/cm3 (a) Si (b) Li (c) NaCl (d) Ice
(c) 7.289 g/cm3 (d) 2.144 g/cm36 73. How many effective Na+ and Cl– ions are present
64. MgO crystallizes in a cubic type crystal system. respectively in a unit cell of NaCl solid (Rock
The ionic radii for Mg2+ and O2– are 0.066 and salt structure) if all ions along line connecting
0.140 nm respectively. One can conclude that opposite face centres are absent?
the Mg2+ ions occupy
(a) a cubic hole in a simple cubic structure. 7 7 7 7
(a) 3, 3 (b) , 4 (c) , (d) 4,
(b) every tetrahedral hole in a close packed 2 2 2 2
structure. 74. In a normal spinel type structure , the oxide ions
(c) an octahedral hole in a cubic close packed are arranged in ccp whereas 1/8 tetrahedral holes
structure. are occupied by Zn 2+ ions and 50% of octahedral
(d) every other tetrahedral hole in a close holes are occupied by Fe3+ ions .The formula of
packed structure. the compound is
65. A crystal lattice with alternate positive and
negative ions has radius ratio 0.524. Its co- (a) Zn 2 Fe 2O 4 (b) ZnFe 2 O 3
ordination number is
(c) ZnFe 2O 4 (d) ZnFe 2O 2
(a) 4 (b) 3 (c) 6 (d) 12
66. The correct statement regarding F– centre is 75. Non stoichiometric defects are formed by
(a) electrons are held in the voids of crystals. (a) s- block elements
(b) p-block elements
(b) F – centre produces colour to the crystals.
(c) either s-block elements or d-block elements
(c) conductivity of the crystal increases due to
F – centre. (d) only d-block elements.
(d) All of these 76. Which of the following metal oxides is anti-
67. The number of octahedral void(s) per atom ferromagnetic in nature?
present in a cubic close-packed structure is (a) MnO2 (b) TiO2 (c) VO2 (d) CrO2
(a) 1 (b) 3 (c) 2 (d) 4 77. A molecule A2B (Mwt. = 166.4) occupies
triclinic lattice with a 5 Å, b = 8 Å, and c = 4 Å.
68. If we mix a pentavalent impurity in a crystal lattice
If the density of AB2 is 5.2 g cm– 3, the number
of germanium, what type of semiconductor
of molecules present in one unit cell is
formation will occur?
(a) p-type (b) n-type
(c) Both (a) and (b) (d) Neither (a) nor (b) (a) 2 (b) 3 (c) 4 (d) 5
EBD_7587
172 Chemistry Objective MCQs
78. Doping of silicon (Si) with boron (B) leads to : 80. Which one of the following statements is correct?
(a) n-type semiconductor
(b) p-type semiconductor
(c) Metal
(d) Insulator (a) NaCl is a paramagnetic salt
79. Non-stoichiometr ic compound have the (b) CuSO4 is a diamagnetic salt
properties of: (c) MnO is an example of ferromagnetic
(a) Electrical conductance substance
(b) Isolation (d) Ferrimagnetic substance like ZnFe 2O 4
(c) Insulation becomes paramagnetic on heating
(d) None of these

Answer KEYs
1 (d) 9 (a) 17 (b) 25 (c) 33 (b) 41 (d) 49 (c) 57 (d) 65 (c) 73 (a)
2 (d) 10 (c) 18 (c) 26 (b) 34 (c) 42 (c) 50 (d) 58 (d) 66 (d) 74 (c)
3 (d) 11 (d) 19 (d) 27 (b) 35 (c) 43 (c) 51 (d) 59 (a) 67 (a) 75 (d)
4 (d) 12 (c) 20 (d) 28 (b) 36 (d) 44 (b) 52 (a) 60 (a) 68 (b) 76 (a)
5 (c) 13 (a) 21 (b) 29 (a) 37 (b) 45 (a) 53 (d) 61 (b) 69 (b) 77 (b)
6 (a) 14 (b) 22 (d) 30 (b) 38 (d) 46 (c) 54 (d) 62 (a) 70 (d) 78 (b)
7 (a) 15 (c) 23 (b) 31 (b) 39 (a) 47 (d) 55 (c) 63 (b) 71 (d) 79 (d)
8 (c) 16 (b) 24 (c) 32 (c) 40 (c) 48 (b) 56 (b) 64 (c) 72 (b) 80 (d)

1. (d) Since in NaCl type of structure 4 formula units 3. (d) Body diagonal (d) of a cubic crystal of edge
form a cell. length (a) is given by,
58.5 gm. of NaCl = 6.023 × 1023 atoms
d= a 3
6.023 ´ 1023 putting a = 400 pm, we get
1 gm of NaCl = atoms
58.5 d = 3 ´ 400 pm = 692.8 pm » 693 pm.
4 atoms constitute 1 unit cell 4. (d) Order of Bragg diffraction (n) = 2 ;
6.023 ´ 1023 Wavelength (l) = 1Å and angle (q) = 60º.
\ atoms constitute We know from the Bragg’s equation
58.5 nl = 2d sin q
6.023 ´ 1023 or 2 × 1 = 2d sin 60º
=
58.5 ´ 4 3 2
= 2.57 × 1021 unit cells. Þ 2 ´ 1 = 2.d. Þ d= = 1.15Å
2 3
2. (d) Effective number of ‘A’ atoms (where d = Distance between the scattering
æ 1ö æ 1ö planes)
= ç8 ´ ÷ + ç 4 ´ ÷ = 3 5. (c) In a fcc lattice, the distance between the
è 8ø è 2ø
Effective number of ‘B’ atoms cation and anion is equal to the sum of their
radii, which is equal to half of the edge
æ 1ö length of unit cell,
= ç12 ´ ÷ + 1 = 4
è 4ø a
\ Formula of the solid = A3B4. i.e. r + + r - = (where a = edge length)
2
The Solid State 173
r+ = 95 pm, r– = 181 pm 17. (b) A form corner points and two atoms of A
Edge length are missing from corner
= 2r+ + 2r– = (2 × 95 + 2 × 181) pm 1 3
= (190 + 362) pm = 552 pm. \ Atoms at corner (A) = 6 ´ =
8 4
3
6. (a) For bcc structure, atomic radius, r = a 1
4 Atoms at face centre (B) = 6 ´ = 3
2
3 \ A3/4 B3 i.e., AB4
= × 4.3 = 1.86
4 18. (c) The hcp arrangement of atoms occupies
Since, r = half the distance between two 74% of the available space and thus has
nearest neighbouring atoms. 26% vacant space.
\ Shortest interionic distance 19. (d) The electrical resistance of metals
= 2 × 1.86 = 3.72 depends upon temperature. Electrical
7. (a) Potassium crystallises in bcc lattice. resistance decreases with decrease in
8. (c) A temperature and becomes zero near the
D
absolute temperature. Material in this state
is said to possess super conductivity.
C
B 1
20. (d) Number of atoms A = ´ 8 = 1
8
1 5
Number of atoms B = ´ 5 =
2 2
3a (Since atom B is missing from one of the
AB = (nearest) face centered point)
2
AC = a (next - nearest) \ Formula of ionic compound
= AB5/2 = A2B5
CD = 2a (next - next - nearest) 21. (b)
9. (a) In crystalline solid there is perfect 22. (d) The appearance of colour in alkali metal
arrangement of the constituent particles halides is generally due to F-centres. F
only at 0 K. They have sharp M.P. centers in NaCl produce a band of optical
10. (c) Glass is an amorphous solid. absorption towards the blue end of visible
11. (d) spectrum, thus the colour appears yellow
12. (c) A body-centred cubic system consists of under transmitted light. On the other hand,
all eight corners plus one atom at the centre KCl with F-centers appears magenta, and
of cube. KBr appears blue.
13. (a) ZnS has cubic close packed (ccp) structure. 23. (b) Solid AB crystallizes as NaCl structure, so
The S2– ions are present at the corners of it has coordination number 6 and its r+/r–
the cube and at the centre of each face. Zinc ranges from 0.414 – 0.732.
ions occupy half of the tetrahedral sites. For maximum radius of anion, we have to
Each zinc ion is surrounded by four sulphide take the lower limit of the range 0.414– 0.732.
ions which are disposed towards the corners
of a regular tetrahedron. Similarly, S2– ion is r+
So, = 0.414
surrounded by four Zn2+ ions. r-
r 0.100
14. (b) The radius ratio + = 146 = 0.675 Þ r– = nm = 0.241 nm
r- 216 0.414
24. (c)
It lies between 0.414 – 0.732. Hence, it 25. (c) Let the number of Z atoms in the ccp
exhibits rock salt type structure. arrangement = 100
15. (c) The coordination number is 8 : 8 for Cs+ and Thus the number of tetrahedral sites = 200
Cl–. Since all the tetrahedral sites are occupied
16. (b) For fcc, by X atoms, the number of X atoms = 200
2a a Hence ratio of X : Z = 2 : 1
r= = = 0.3535a Thus the formula is X2Z
4 2 2 26. (b) Molecular solid is the best description of
given a = 361 pm water in the solid phase. For example, ice
r = 0.3535 × 361 = 128 pm is hydrogen bonded molecular solid.
EBD_7587
174 Chemistry Objective MCQs
27. (b) For body centred cubic lattice Z = 2 where d = distance between two atoms
Atomic mass of unit cell = 133 + 80 a = edge length
= 213 a.m.u
Volume of cell = (436.6 × 10–10)3 cm3 3
1.73 = a
Density, 2
ZM 2 ´ 1.73
r= a= = 2Å = 200pm
3 3
a NA
33. (b) hcp is a closed packed arrangement in which
2 ´ 213 the unit cell is hexagonal and coordination
=
(436.6 ´ 10-10 )3 ´ 6.02 ´ 1023 number is 12.
= 8.50 g/cm3 34. (c) As per formula,
28. (b) The amount of energy released when cation radius of cation
and anions are brought from infinity to their radius ratio =
radius of anion
respective sites in the crystal lattice to form
one mole of the ionic compounds is called 94
= = 0.643
the lattice energy. Na + being smallest in size 146
have highest lattice energy. Since the value is between 0.414 – 0.732
ZM hence the coordination no. will be 6 and
29. (a) r= geometry will be octahedral.
NA V 35. (c) Number of Cu-atoms per unit cell
23 -8 3
rN A V 2 ´ 6 ´10 ´ (5 ´10 ) 1 1
Z= = = ´8 + ´6 = 4 .
M 75 8 2
Z = 2, which represents bcc structure Number of Ag-atoms per unit cell
3 3 1
\ r= a= ´ 5 = 2.165Å = 216.5 pm = ´ 12 = 3
4 4 4
» 217 pm Number of Au-atoms per unit cell = 1 (at
30. (b) For bcc structure body centre)
Formula : Cu4 Ag3 Au
3
Interionic distance = r + + r – = a 36. (d) In a unit cell, W atoms at the corner
2
(a = edge length) 1
= ´8 = 1
3 8
r+ + r– – = a 1
NH +4 Cl 2 O-atoms at the centre of edges = ´12 = 3
3 4
r+ + 180 = ´ 390 Na-atoms at the centre of the cube = 1
NH+4 2 W : O : Na = 1 : 3 : 1
r+ + = 338pm –180pm = 158 pm Hence, formula = NaWO3
NH 4 37. (b) For bcc lattice,
ZM 4 ´ 96
31. (b) r= 3a = 4R Þ a = pm = 221.7 pm
NA V 3
rN A V (where R is the radius of Ca atom)
Z= Colume of unit
M cell = a3 = (211.7 × 10–12)3 m3
8.92 ´ 6.02 ´10 23 ´ (362)3 ´10 -30 = 10.9 × 10–30 m3
= =4 38. (d) 4 formula units of AB are present in a unit
63.55 cell
\ It has fcc unit cell
32. (c) For bcc structure 4 N
\ no. of unit cell = ´ A
40 4
3a
d= Þ 0.025 NA
2
The Solid State 175

39. (a) For bcc lattice body diagonal = a 3 . 48. (b) When equal number of cations (Na+) and
The distance between the two oppositely anions (Cl–) are missing from their regular
charged ions lattice positions, we have schottky defect.
49. (c) Due to presence of F-centres.
a 387 ´1.732 50. (d) In KCl, co-ordination number of cation and
= 3 = = 335 pm
2 2 anion is 6 and 6 respectively. KCl is highly
40. (c) Number of unit cells ionic so Schottky defect is common.
N A ´ Wt. of substance Note : Schottky defect is common in
= compounds having high coordination
M. Wt. ´ Z number while Frenkel defect is common in
compounds with low coordination number.
6.023 ´1023 ´ 58.8
= = 1.5 ´1023 51. (d) If in an ionic crystal of the type A+, B–, equal
58.8 ´ 4 number of cations and anions are missing
41. (d) For a fcc unit cell from their lattice sites so that the electrical
2a neutrality is maintained. The defect is called
r= Schottky defect.
4 52. (a) Since MgO has a rock salt structure. In this
4r structure each cation is surrounded by six
a= = 2 2 ´ 0.14 = 0.39 » 0.4 nm. anions and vice versa.
2
53. (d) In Na2O all tetrahedral void are occupied.
42. (c) Among the three options KCl, NaCl and
54. (d) Zn2+ ion is too small to fit into the octahedral
MgCl2 , the size of anion is same. So larger
voids of S2– ions.
the cation, larger will be the cation/anion 55. (c) AB is just like NaCl. Thus twelve A+ are at
ratio i.e., KCl will have larger cation/anion edges and 1 within body of fcc i.e. in
ratio among the three. So, we left with two octahedral voids and six B– at faces and 8 at
options KCl and CaF2. Among these two corner.
CaF2 will have maximum value of cation/ 56. (b) Density is given by
anion ratio because decrease in ionic radii
of anion from Cl– to F– does not overcome Z´ M
the effect of decrease in ionic radii of cation d= ; where Z = number of formula
NA a 3
from K+ to Ca2+.
43. (c) units present in unit cell, which is 4 for fcc
a = edge length of unit cell. M = Molecular
44. (b) The face centered cubic unit cell contains 4 mass
atom 4´ M
\ Total volume of atoms 2.72 =
6.02 ´ 1023 ´ (404 ´ 10-10 )3
4 16
= 4 ´ pr3 = pr 3
3 3 (Q 1pm = 10-10 cm)
45. (a) There are two atoms in a bcc unit cell.
So, number of atoms in 12.08 × 1023 unit 2.72 ´ 6.02 ´ (404)3
M=
cells 4 ´ 107
= 2 × 12.08 × 1023 = 24.16 × 1023 atoms.
= 26.99 = 27 gm mole–1
46. (c) In antifluorite structure the oxide ions
(anions) form a face-centered cubic array 57. (d) When insulators (non metal atoms) interact
and the metal ions (cations) occupy all the to form a solid, their atomic orbitals mix to
tetrahedral voids. Example : Na2O form two bunch of orbitals, separated by a
large band gap. Electrons cannot therefore
r´ a 3 ´ N A ´ 10-30 be promoted to an empty level, where they
47. (d) M= could move freely.
Z
58. (d) Extrinsic semiconductor.
10 ´ (100)3 ´ 6.02 ´ 1023 ´10-30 59. (a) Number of A cations per unit cell = 4
= = 15.05
4 Number of B anions per unit cell must be
\ Number of atoms in 100 g =2× 4=8
Number of tetrahedral voids per unit cell
6.02 ´1023 =2× 4=8
= ´100 = 4 × 1025
15.05
EBD_7587
176 Chemistry Objective MCQs
Number of octahedral voids per unit all 70. (d) For n-type, impurity added to silicon should
= 1 × 4 = 4. have more than 4 valence electrons.
Hence, occupancy of tetrahedral voids 71. (d) Packing fraction of bcc = 68%
= 100% Empty space = 100 – 68 = 32%
60. (a) Super conductors are derived from 72. (b) Out of the given substances, only Li has
compound of p block elements. high electrical and thermal conductivity as
61. (b) Since each Sr ++ ion provides one cation Li is a metallic solid.
vacancy, hence 73. (a) No. of Na+ = 4 – 1 = 3
Concentration of cation vacancies
= mole % of SrCl2 added æ 1ö
No. of Cl– = 4 - ç 2 ´ ÷ = 3
= 10–4 mole % è 2ø
10-4 74. (c) Number of O-atoms per unit cell
= ´ 6.023 ´1023 = 6.023 ´ 1017 1 1
100 = ´8 + ´6 = 4 .
r 0.65 8 2
62. (a) For MgO, c = = 0.47 Number of octahedral holes per unit cell
ra 1.4 = 1× 4 = 4
Mg2+ is in octahedral void so C.N. = 6 Number of Fe3+ ions per unit cell
For MgS, rc = 0.65 = 0.353 =
50 ´ 4
=2
ra 1.84 100
2+
Mg is in tetrahedral void so C.N. = 4 Number of tetrahedral voids per unit cell
= 2 × 4 = 8.
Z´M 1
63. (b) r= Number of Zn2+ ions per unit cell = ´8 =1
NA ´ a 3 8
2 ´ 100 Hence, formula : ZnFe2O4
= 75. (d) In non stoichiometric defects ratio of cation
6.023 ´ 10 23 ´ (400 ´ 10-10 )3 and anion is not same that is represented
= 5.188 g / cm3 by chemical ideal formula, this is due to
variable oxidation nos of cation. Transition
64. (c) rc 0.066 elements show variable oxidation nos.
= = 0.47
ra 0.140 76. (a) MnO2
So, cation (Mg2+) fitted in octahedral holes 77. (b) Volume of unit cell = a × b × c
of O2– are present at fcc lattice point. = 5 × 10– 8 × 8 × 10– 8 × 4 × 10– 8
= 1.6 × 10– 22 cm3
r+ Mass of unit cell = 1.6 × 10– 22 × 5.2
65. (c) = 0.524 . It is in between 0.414 – 0.732. = 8.32 × 10– 22 g
r-
Number of molecules in one unit cell
Hence C. No. = 6. 8.32 ´10 – 22 g
66. (d) All the given statements are correct about = =3
F-centres. 166.4 g mol –1
67. (a) Number of octahedral voids in ccp, is equal 78. (b)
to effective number of atoms. 79. (d) Non-stoichiometr ic compounds are
In ccp, effective number of atoms are 4 so, 4 chemical compounds with an elemental
octahedral voids. composition that cannot be represented by
So, 1 octahedral voids per atom. a ratio of well-defined natural numbers, and
68. (b) n-type, since electron is set free. are therefore in violation of the law of
69. (b) Low co-ordination compound can be definite proportions. They are most often
changed into high co-ordination compound solids that contain random defects, resulting
by increasing pressure and decreasing in the deficiency of one element. So, none
temperature. of given option is correct.
Pr.
ˆˆˆˆˆ † 80. (d) Ferrimagnetic substance become paramagnetic
NaCl ‡ˆˆˆˆ ˆ CsCl
(6,6) Temp. (8,8) on heating. This is due to randomisation of
spins on heating.
16 Solutions
1. For preparing 0.1 N solution of a compound from 6. Which will form maximum boiling point
its impure sample (percentage purity of which is azeotrope?
known) weight of the substance required will be (a) HNO3 + H2O solution
(a) less than the theoretical weight. (b) C2H5OH + H2O solution
(b) more than the theoretical weight. (c) C6H6 + C6H5CH3 solution
(c) same as the theoretical weight. (d) None of these
(d) None of these. 7. A solution of acetone in ethanol
2. A solution is prepared by dissolving 10 g NaOH (a) shows a positive deviation from Raoult’s
in 1250 mL of a solvent of density 0.8 g/mL. The law
molality of the solution in mol kg–1 is (b) behaves like a non ideal solution
(a) 0.25 (b) 0.2 (c) 0.008 (d) 0.0064 (c) obeys Raoult’s law
3. Two solutions of a substance (non electrolyte) (d) shows a negative deviation from Raoult’s
are mixed in the following manner. 480 mLof 1.5 M law
first solution + 520 mL of 1.2 M second solution. 8. An aqueous solution is 1.00 molal in KI. Which
What is the molarity of the final mixture? change will cause the vapour pressure of the
(a) 2.70 M (b) 1.344 M solution to increase?
(c) 1.50 M (d) 1.20 M
4. Formation of a solution from two components (a) Addition of NaCl
can be considered as (b) Addition of Na2SO4
(c) Addition of 1.00 molal KI
(i) Pure solvent ® separated solvent
(d) Addition of water
molecules, DH1
9. A binary liquid solution is prepared by mixing
(ii) Pure solute ® separated solute molecules, n-heptane and ethanol. Which one of the
DH2 following statements is correct regarding the
(iii) Separated solvent and solute molecules ® behaviour of the solution?
Solution, DH3 (a) The solution is non-ideal, showing -ve
Solution so formed will be ideal if deviation from Raoult’s Law.
(b) The solution is non-ideal, showing +ve
(a) DHsoln = DH3 -DH1 -DH 2 deviation from Raoult’s Law.
(c) n-heptane shows +ve deviation while
(b) DHsoln = DH1 + DH 2 + DH3
ethanol shows –ve deviation from Raoult’s
(c) DHsoln = DH1 + DH 2 -DH3 Law.
(d) The solution formed is an ideal solution.
(d) DHsoln = DH1 -DH 2 -DH3 10. PA and PB are the vapour pressure of pure liquid
5. Which of the following liquid pairs shows a components, A and B, respectively of an ideal
positive deviation from Raoult’s law ? binary solution. If XA represents the mole fraction
(a) Water - Nitric acid of component A, the total pressure of the solution
will be.
(b) Benzene - Methanol
(c) Water - Hydrochloric acid (a) PA + XA (PB – PA) (b) PA + XA (PA– PB)
(d) Acetone - Chloroform (c) PB + XA (PB – PA) (d) PB + XA (PA – PB)
EBD_7587
178 Chemistry Objective MCQs
11. Vapour pressure of chloroform (CHCl 3) and 17. At 20°C and 1.00 atm partial pressure of H2, 18
dichloromethane (CH2Cl2) at 25ºC are 200 mm Hg mL of H2 (STP) dissolves in 1 L of water. If 2 L of
and 41.5 mm Hg respectively. Vapour pressure of water is exposed to a gaseous mixture having a
the solution obtained by mixing 25.5 g of CHCl3 total pressure of 1425 torr (excluding the vapour
and 40 g of CH2Cl2 at the same temperature will pressure of water) and containing 80% H2 by
be: (Molecular mass of CHCl3 = 119.5 u and volume, the volume of H2(STP) dissolved is
molecular mass of CH2Cl2 = 85 u).
(a) 90.92 mm Hg (b) 115.0 mm Hg
(c) 147.9 mm Hg (d) 285.5 mm Hg (a) 27 mL (b) 54 mL
12. The vapour pressure of a solution of the liquids (c) 33.75 mL (d) 67.50 mL
A (P° = 80 mm Hg and xA = 0.4) and B (P°= 120 18. At a particular temperature, the vapour pressures
mm Hg and xB = 0.6) is found to be 100 mm Hg. It of two liquids A and B are respectively 120 and
shows that the solution exhibits 180 mm of mercury. If 2 moles of A and 3 moles of
(a) positive deviation from ideal behaviour B are mixed to form an ideal solution, the vapour
pressure of the solution at the same temperature
(b) negative deviation from ideal behaviour will be (in mm of mercury)
(c) ideal behaviour (a) 156 (b) 145 (c) 150 (d) 108
(d) positive deviation for lower conc. and 19. Which one of the following gases has the lowest
negative for higher conc. value of Henry’s law constant?
13. Vapour pressure (in torr) of an ideal solution of (a) N2 (b) He (c) H2 (d) CO2
two liquids A and B is given by : P = 52XA + 114 20. Which one of the following binary mixtures forms
where XA is the mole fraction of A in the mixture. an azeotrope with minimum boiling point type?
The vapour pressure (in torr) of equimolar mixture (a) acetone-ethanol
of the two liquids will be : (b) H2O-HNO3
(c) benzene-toluene
(a) 166 (b) 83 (c) 140 (d) 280 (d) n-hexane-n-heptane
14. The vapour pressure (at the standard boiling 21. The vapour pressure of pure benzene and toluene
point of water) of an aqueous solution containing at a particular temperature are 100 mm and 50 mm
28% by mass of a non-volatile normal solute respectively. Then the mole fraction of benzene
(molecular mass = 28) will be in vapour phase in contact with equimolar
(a) 152 torr (b) 608 torr solution of benzene and toluene is
(c) 760 torr (d) 547 torr
15. Lowering of vapour pressure of 1.00 m solution (a) 0.67 (b) 0.75 (c) 0.33 (d) 0.50
of a non-volatile solute in a hypothetical solvent
of molar mass 40 g at its normal boiling point, is : 22. 12 g of urea is dissolved in 1 litre of water and
68.4 g of sucrose is dissolved in 1 litre of water.
(a) 29.23 torr (b) 30.4 torr The lowering of vapour pressure of first case is
(c) 35.00 torr (d) 40.00 torr (a) equal to second
16. Equal masses of a solute are dissolved in equal (b) greater than second
amount of two solvents A and B, respective (c) less than second
molecular masses being MA and MB. The relative (d) double that of second
lowering of vapour pressure of solution in solvent
23. Which of the following statement is correct if the
A is twice that of the solution in solvent B. If the
intermolecular forces in liquids A, B and C are in
solutions are dilute, MA and MB are related as the order A < B < C?
(a) B evaporates more readily than A
(b) B evaporates less readily than C
(a) M A = MB (b) 2M A = M B (c) A and B evaporates at the same rate
(c) M A = 2M B (d) M A = 4M B (d) A evaporates more readily than C
Solutions 179
24. At 80° C, the vapour pressure of pure liquid ‘A’ 30. Henry’s law constant K of CO2 in water at 25°C is
is 520 mm Hg and that of pure liquid ‘B’ is 1000 3.0 × 10–2 mol L–1atm–1. The mass of CO2 present
mm Hg. If a mixture solution of ‘A’ and ‘B’ boils in 100 L of soft drink bottled with a partial pressure
at 80° C and 1 atm pressure, the amount of ‘A’ in of CO2 of 4 atm at the same temperature is
the mixture is (1 atm = 760 mm Hg) (a) 5.28 g (b) 12.0 g (c) 428 g (d) 528 g
31. Vapour pressure of solution containing 2 mol of
liquid A ( PAo = 80 torr ) and 3 mol of liquid B
(a) 52 mol percent (b) 34 mol percent
(c) 48 mol percent (d) 50 mol percent ( PBo = 100 torr ) is 87 torr. We can conclude that
25. A solution at 20°C is composed of 1.5 mol of
benzene and 3.5 mol of toluene. If the vapour
pressure of pure benzene and pure toluene at
this temperature are 74.7 torr and 22.3 torr, (a) there is negative deviation from Raoult’s law
respectively, then the total vapour pressure of (b) boiling point is higher than that expected
the solution and the benzene mole fraction in for ideal solution
equilibrium with it will be, respectively :
(c) molecular attractions between unlike
molecules are stronger than those between
like molecules
(a) 35.8 torr and 0.280 (b) 38.0 torr and 0.589
(c) 30.5 torr and 0.389 (d) 30.5 torr and 0.480 (d) All of these statements are correct
26. Choose the correct statement with respect to the 32. At 15°C and 1 atmosphere partial pressure of
vapour pressure of a liquid among the following: hydrogen, 20 mL of hydrogen measured at STP
(a) Increases linearly with increasing dissolves in 1 L of water. If water at 15°C is
temperature exposed to a gaseous mixture having a total
(b) Increases non-linearly with increasing pressure of 1500 mm of Hg (excluding the vapour
temperature pressure of water) and containing 80% hydrogen
by volumne, then the volume of hydrogen
(c) Decreases linearly with increasing measured at STP that will dissolve in 1 L of water
temperature is
(d) Decreases non-linearly with increasing
(a) 20.0 mL (b) 31.6 mL
temperature
27. For an ideal solution of two components A and (c) 36.1 mL (d) 26.3 mL
B, which of the following is true? 33. The vapour pressure of two pure liquids A and B
(a) DHmixing < 0 (zero) that form an ideal solution, are 400 and 800 mm of
Hg respectively at a temperature t°C. The mole
(b) DHmixing > 0 (zero)
fraction of A in a solution of A and B whose boiling
(c) A – B interaction is stronger than A – A point is t°C will be
and B – B interactions
(a) 0.4 (b) 0.8 (c) 0.1 (d) 0.2
(d) A– A, B – B and A – B interactions are
identical. 34. The total vapour pressure of a 4 mole % solution
of NH3 in water at 293 K is 50.0 torr. The vapour
28. Vapour pressure of pure benzene is 119 torr and
pressure of pure water is 17.0 torr at this
that of toluene is 37.0 torr at the same temperature.
temperature. Applying Henry’s and Raoult’s
Mole fraction of toluene in vapour phase which laws, the total vapour pressure for a 5 mole %
is in equilibrium with a solution of benzene and
toluene having a mole fraction of toluene 0.50, solution is
will be :
(a) 58.25 torr (b) 33 torr
(a) 0.137 (b) 0.237 (c) 0.435 (d) 0.205
29. How many grams of methyl alcohol should be (c) 42.1 torr (d) 52.25 torr
added to 10 litre tank of water to prevent its freezing 35. Which one of the following aqueous solutions
at 268 K ? (Kf for water is 1.86 K kg mol–1) will exihibit highest boiling point ?
(a) 880.07 g (b) 899.04 g (a) 0.015 M urea (b) 0.01 M KNO3
(c) 886.02 g (d) 868.06 g (c) 0.01 M Na2SO4 (d) 0.015 M glucose
EBD_7587
180 Chemistry Objective MCQs
36. The elevation in boiling point of a solution of 43. Which of the following has the lowest freezing
13.44 g of CuCl2 in 1 kg of water using the point?
following information will be (Molecular weight (a) 0.1 m sucrose (b) 0.1 m urea
of CuCl2= 134.4 g and Kb= 0.52 K kg mol-1)
(c) 0.1 m ethanol (d) 0.1 m glucose
(a) 0.16 (b) 0.05 (c) 0.1 (d) 0.2
44. The van’t Hoff factor i for a compound which
37. The temperature at which 10% aqueous solution undergoes dissociation in one solvent and
(w/v) of glucose will exhibit the osmotic pressure
association in other solvent is respectively :
of 14 bar is –
(a) less than one and greater than one.
(a) 307.3°C (b) 307.3 K
(c) 34 K (d) – 34°C (b) less than one and less than one.
38. KBr is 80% ionized in solution. The freezing point (c) greater than one and less than one.
of 0.4 molal solution of KBr is : (d) greater than one and greater than one.
Kf (H2O) = 1.86 K kg/mole 45. If 0.1 M solution of glucose and 0.1 M solution
(a) 274.339 K (b) – 1.339 K of urea are placed on two sides of the
(c) 257.3 K (d) – 1.339°C semipermeable membrane to equal heights, then
it will be correct to say that
39. A solution of urea (mol. mass 56 g mol-1) boils
at 100.18°C at the atmospheric pressure. If Kf (a) there will be no net movement across the
and Kb for water are 1.86 and 0.512 K kg mol-1 membrane
respectively, the above solution will freeze at (b) glucose will flow towards urea solution
(c) urea will flow towards glucose solution
(d) water will flow from urea solution to glucose
(a) 0.654 °C (b) - 0.654 °C
(c) 6.54 °C (d) - 6.54 °C 46. Determination of the molar mass of acetic acid
40. The solution containing 4.0 g of a polyvinyl in benzene using freezing point depression is
chloride polymer in 1 litre of dioxane was found affected by :
to have an osmotic pressure 6.0 × 10–4 atmosphere (a) Partial ionization (b) Dissociation
at 300K, the value of R used is 0.082 litre (c) Complex formation(d) Association
atmosphere mole–1K–1. The molecular mass of 47. The observed osmotic pressure for a 0.10 M
solution of Fe(NH4)2(SO4)2 at 25°C is 10.8 atm.
the polymer was found to be
The experimental (observed) value of van’t Hoff
(a) 3.0 × 102 (b) 1.6 × 105 factor will be :
(c) 5.6 × 10 4 (d) 6.4 × 102 (R = 0.082 L atm k–1 mol–1)
41. If the various terms in the given below
expressions have usual meanings, the van’t Hoff (a) 4.42 (b) 4.00
factor (i) cannot be calculated by which one of (c) 3.42 (d) 5.42
the following expressions 48. 12 g of a nonvolatile solute dissolved in 108 g of
water produces the relative lowering of vapour
(a) pV = inRT
pressure of 0.1. The molecular mass of the solute
(b) DT f = iK f .m
is :
(c) DTb = iKb .m
(a) 80 (b) 60 (c) 20 (d) 40
o
Psolvent - Psolution æ n ö 49. A molecule M associates in a given solvent
(d) = iç
o
Psolvent è N + n ÷ø according to the equation M ‡ˆˆˆˆ† (M)n. For a
42. A 0.001 molal solution of [Pt(NH3)4Cl4] in water certain concentration of M, the van't Hoff factor
had a freezing point depression of 0.0054°C. If was found to be 0.9 and the fraction of associated
K f for water is 1.80, the correct formulation for molecules was 0.2. The value of n is:
the above molecule is
(a) [Pt(NH3)4Cl3]Cl (b) [Pt(NH3)4Cl2]Cl2
(c) [Pt(NH3)4Cl]Cl3 (d) [Pt(NH3)4Cl4] (a) 3 (b) 5 (c) 2 (d) 4
Solutions 181
50. A solution containing 0.85 g of ZnCl2 in 125.0 g 56. The difference between the boiling point and
of water freezes at –0.23°C. The apparent degree freezing point of an aqueous solution containing
of dissociation of the salt is (Kf for water = 1.86K sucrose (molecular wt = 342 g mole–1) in 100 g of
kg mol–1, atomic mass: Zn = 65.3 and Cl = 35.5) water is 105°C. If Kf and Kb of water are 1.86 and
0.51 K kg mol –1 respectively, the weight of
sucrose in the solution is about
(a) 1.36% (b) 73.5% (c) 7.35% (d) 2.47%
(a) 34.2 g (b) 342 g (c) 7.2 g (d) 72 g
51. 0.5 molal aqueous solution of a weak acid (HX) is
20% ionised. If Kf for water is 1.86 K kg mol–1, the 57. A solution containing 1.8 g of a compound
(empirical formula CH2O) in 40 g of water is
lowering in freezing point of the solution is
observed to freeze at –0.465° C. The molecular
(a) 0.56 K (b) 1.12 K formula of the compound is
(c) – 0.56 K (d) – 1.12 K (Kf of water = 1.86 kg K mol–1)
52. 200 mL of an aqueous solution of a protein (a) C2H4O2 (b) C3H6O3
contains its 1.26 g. The osmotic pressure of this (c) C4H8O4 (d) C6H12O6
solution at 300 K is found to be 2.57 × 10–3 bar. 58. Freezing point of an aqueous solution is – 0.186°C.
The molar mass of protein will be If the values of Kb and Kf of water are respectively
(R = 0.083 L bar mol–1 K–1) 0.52 K kg mol–1 and 1.86 K kg mol–1, then the
(a) 51022 g mol–1 (b) 122044 g mol–1 elevation of boiling point of the solution in K is
(c) 31011 g mol–1 (d) 61038 g mol–1 (a) 0.52 (b) 1.04 (c) 1.34 (d) 0.052
53. Freezing point of an aqueous solution is (–0.186)°C. 59. If the elevation in boiling point of a solution of
Elevation of boiling point of the same solution is 10 g of solute (mol. wt. = 100) in 100 g of water is
Kb = 0.512°C, Kf = 1.86°C, find the increase in DTb, the ebullioscopic constant of water is

boiling point. DTb


(a) 10 (b) 10 DTb (c) DTb (d)
10
(a) 0.186 °C (b) 0.0512 °C 60. Which one of the following statements is
(c) 0.092 °C (d) 0.2372 °C incorrect?
54. The relationship between osmotic pressure at
273 K when 10 g glucose (P1), 10 g urea (P2), and
10 g sucrose (P3) are dissolved in 250 mL of water is (a) The correct order of osmotic pressure for
0.01 M aqueous solution of each compound
(a) P1 > P2 > P3 (b) P3 > P1 > P2
is BaCl 2 > KCl > CH3COOH > Sucrose
(c) P2 > P1 > P3 (d) P2 > P3 > P1
(b) The osmotic pressure (p) of a solution is
55. Which observation(s) reflect(s) colligative
given by the equation p = MRT, where M is
properties? the molarity of the solution
(c) Raoult’s law states that the vapour pressure
(i) A 0.5 m NaBr solution has a higher vapour of a component over a solution is
proportional to its mole fraction
pressure than a 0.5 m BaCl2 solution at the
same temperature (d) Two sucrose solutions of same molality
prepared in different solvents will have the
(ii) Pure water freezes at the higher temperature same freezing point depression
than pure methanol
61. p1, p2, p3 and p4 atm are the osmotic pressures of
(iii) A 0.1 m NaOH solution freezes at a lower 5% (mass/volume) solutions of urea, fructose,
temperature than pure water sucrose and KCl respectively at certain
Choose the correct answer from the codes given temperature. The correct order of their magnitudes
below is :
(a) (i), (ii) and (iii) (b) (i) and (ii) (a) p1 > p4 > p2 > p3 (b) p1 > p4 > p2 > p3
(c) (ii) and (iii) (d) (i) and (iii) (c) p4 > p1 > p2 > p3 (d) p4 > p1 > p3 > p2
EBD_7587
182 Chemistry Objective MCQs
62. Which of the following has been arranged in the (a) 0.008, 0.016 (b) 0.016, 0.008
increasing order of freezing point? (c) 0.16, 0.08 (d) 0.04, 0.02
(a) 0.025M KNO3 < 0.1M NH2CSNH2 68. If sodium sulphate is considered to be completely
< 0.05M BaCl2 < 0.1M NaCl dissociated into cations and anions in aqueous
(b) 0.1M NaCl < 0.05M BaCl2 solution, the change in freezing point of water
(DTf), when 0.01 mol of sodium sulphate is
< 0.1M NH2CSNH2 < 0.025M KNO3
dissolved in 1 kg of water, is (Kf = 1.86 K kg mol–1)
(c) 0.1M NH2CSNH2 < 0.1 M NaCl
(a) 0.372 K (b) 0.0558 K
< 0.05M BaCl2 < 0.025M KNO3 (c) 0.0744 K (d) 0.0186 K
(d) 0.025M KNO3 < 0.05M BaCl2 69. The osmotic pressure (at 27° C) of an aqueous
< 0.1M NaCl < 0.1M NH2CSNH2 solution (200mL) containing 6 g of a protein is
63. 0.010M solution an acid HA freezes at –0.0205ºC. If 2 × 10–3 atm. If R = 0.080 L atm mol–1K–1, the
Kf for water is 1.860 K kg mol–1, the ionization molecular weight of protein is
constant of the conjugate base of the acid will be (a) 7.2 × 105 (b) 3.6 × 105
(assume 0.010 M = 0.010 m) (c) 1.8 ×10 5 (d) 1.0 × 105
70. At temperature 327 °C and concentration C,
(a) 1.1 × 10–4 (b) 1.1 × 10–3 osmotic pressure of a solution is P, the same
(c) 9.0 × 10–11 (d) 9.0 × 10–12 solution at concentration C/2 and at temperature
64. 1.0 M aqueous solution of CrCl3.6H2O freezes at 427 °C shows osmotic pressure of 2 atm, value of
–5.58ºC. Assuming complete ionization of the P will be
hydrated complex, which of the following isomers
conforms to the observation (Kf for water = 1.86 12 24 6 5
K kg mol–1) ? (a) (b) (c) (d)
7 7 5 6
(a) [Cr(H2O)3Cl3].3H2O
71. In countries nearer to polar region, the roads are
(b) [Cr(H2O)4Cl4]Cl.2H2O sprinkled with CaCl2. This is
(c) [Cr(H2O)5Cl]Cl2.H2O (a) to minimise the snow present on roads
(d) [Cr(H2O)6]Cl3 (b) to minimise pollution
65. Assuming degree of ionization to be unity in each (c) to minimise the accumulation of dust on the
case, which of the following equimolal solutions road
would freeze at the lowest temperature ? (d) to minimise the wear and tear of the roads
(a) [Pt(NH3)6]Cl4 (b) [Pt(NH3)5Cl]Cl3 72. If a is the degree of dissociation of Na 2SO4, the
(c) [Pt(NH3)4Cl2]Cl2 (d) [Pt(NH3)3Cl3]Cl vant Hoff’s factor (i ) used for calculating the
66. Which of the following aqueous solutions molecular mass is
should have the highest osmotic pressure? (a) 1 + a (b) 1 – a (c) 1 + 2a (d) 1 – 2a
(a) 0.011 M AlCl3 at 50 °C 73. Which has the minimum freezing point ?
(b) 0.03 M NaCl at 25 °C (a) One molal NaCl aq. solution
(c) 0.012 M (NH4)2SO4 at 25 °C (b) One molal CaCl2 aq. solution
(d) 0.03 M NaCl at 50 °C (c) One molal KCl aq. solution
67. At 760 torr pressure and 20°C temperature, 1 L (d) One molal urea aq. solution
of water dissolves 0.04 gm of pure oxygen or 74. On mixing 3 g of non - volatile solute in 200 mL of
0.02 gm of pure nitrogen. Assuming that dry air water, its boiling point (100°) becomes 100.52°C.
is composed of 20% oxygen and 80% nitrogen If Kb for water is 0.6 K/m then molecular wt. of
(by volume), the masses (in g/L) of oxygen and
solute is :
nitrogen dissolved by 1 L of water at 20°C
exposed to air at a total pressure of 760 torr are (a) 10.5 g mol–1 (b) 12.6 g mol–1
respectively: (c) 15.7 g mol–1 (d) 17.3 g mol–1
Solutions 183
75. Benzene freezes at 5.50ºC. If the freezing point of 78. Which of the following terms has units of
2.48 g of phosphorous in 100 g benzene is 4.48ºC,
temperature?
the atomicity of phosphorus in benzene is (Kf
(benzene) = 5.12 K kg mol–1):
mKf
(a) 1 (b) 3 (c) 4 (d) 8 I : Kf II : mK f III : iK f IV :
i
76. A solute X has a solubility of 10.0 g in 100 g of (a) I, II (b) I, III (c) II, III (d) II, IV
water at 25°C. If a solution containing 4.0 g of
solute in 50 g of water is subjected to evaporation 79. 0.5 m solution of a complex of iron and cyanide
at constant temperature of 25°C, the osmotic ions has the depression of f.pt. to be 3.72 K (Kf for
pressure of the solution water = 1.86 K molal–1). The formula of the complex
(a) remains unchanged throughout is:
(b) increases continuously (a) K4[Fe(CN)6] (b) K2[Fe (CN)4]
(c) increases to constant value (c) K3[Fe(CN)6] (d) Fe(CN)4
(d) decreases to constant value 80. A living cell contains a solution which is isotonic
77. At 17°C, the osmotic pressure of sugar solution with 0.2 M glucose solution. What osmotic
is 580 torr. The solution is diluted and the pressure develops when the cell is placed in
temperature is raised to 57°C, when the osmotic
0.05 M BaCl2 solution at 300 K?
pressure is found to be 165 torr. The extent of
dilution is:
(a) 1.23 atm (b) 3.69 atm
(a) 2 times (b) 3 times
(c) 6.15 atm (d) None of these
(c) 4 times (d) 5 times

Answer KEYs
1 (b ) 9 (b ) 17 (b ) 25 (b ) 33 (c ) 41 (a ) 49 (c) 57 (d ) 65 (a ) 73 (b )
2 (a ) 10 (d ) 18 (a ) 26 (b ) 34 (a ) 42 (b ) 50 (b ) 58 (d ) 66 (d ) 74 (d )
3 (b ) 11 (a ) 19 (d ) 27 (d ) 35 (c ) 43 (c ) 51 (b ) 59 (c) 67 (a ) 75 (c )
4 (b ) 12 (b ) 20 (a ) 28 (b ) 36 (a ) 44 (c ) 52 (d ) 60 (d ) 68 (b ) 76 (c )
5 (b ) 13 (c ) 21 (a ) 29 (d ) 37 (b ) 45 (a ) 53 (b ) 61 (c) 69 (b ) 77 (c )
6 (a ) 14 (b ) 22 (a ) 30 (d ) 38 (d ) 46 (d ) 54 (c) 62 (b ) 70 (b ) 78 (d )
7 (a ) 15 (a ) 23 (d ) 31 (d ) 39 (b ) 47 (a ) 55 (d ) 63 (c) 71 (a ) 79 (c )
8 (d ) 16 (c ) 24 (d ) 32 (b ) 40 (b ) 48 (c ) 56 (d ) 64 (c) 72 (c ) 80 (a )

1. (b) More than theoretical weight since impurity 3. (b) From the molarity equation.
will not contribute. M1V1 + M2V2 = MV
2. (a) Given w = 10 g, Mol. mass = 40 g
M1V1 +M 2V2
Weight of solvent = 1250 × 0.8 g = 10000 g M= where V = V1 + V2
V
= 1 kg
480 ´ 1.5 + 520 ´ 1.2
10 M= = 1.344 M
\ molality = = 0.25 m 480 + 520
40 ´ 1
EBD_7587
184 Chemistry Objective MCQs
4. (b) For an ideal solution, DHmixing = 0 9. (b) For this solution intermolecular interactions
DH = DH1 + DH2 + DH3 between heptane and ethanol are weaker
than heptane -heptane and ethanol -ethanol
(According to Hess's law)
interactions hence the solution of heptane
i.e., for ideal solutions there is no change in and ethanol is non-ideal and shows positive
magnitude of the attractive forces in the two deviation from Raoult’s law.
components present.
10. (d) P = PAXA + PBXB
5. (b) Positive deviations are shown by such = PAXA+ PB (1 – XA)
solutions in which solvent-solvent and
solute-solute interactions are stronger than Þ PAXA + PB – PBXA
the solute-solvent interactions. In such Þ PB + XA (PA – PB)
solution, the interactions among molecules
becomes weaker. Therefore their escaping 11. (a) n CHCl3 = 25.5 = 0.213
119.5
tendency increases which results in the
increase in their partial vapour pressures. n CH2Cl2 = 40 = 0.47
In pure methanol there exists intermolecular 85
H–bonding. PT = PAo XA + PBo XB
---O – H--- O—H --- O—H---
| | | 0.213 0.47
= 200 ´ + 41.5 ´
CH3 CH3 CH3 0.683 0.683
On adding benzene, its molecules come = 62.37 + 28.55 = 90.92
between methanol molecules there by 12. (b) Ptotal = PAo ´ XA + PBo ´ X B
breaking H-bonds which weaken = 80.0 × 0.4 + 120.0 × 0.6 = 104 mm Hg
intermolecular forces. This results in The observed Ptotal is 100 mm Hg which is
increase in vapour pressure. less than 104 mm Hg. Hence the solution
6. (a) The solutions (liquid mixture) which boils shows negative deviation.
at constant temperature and can distil as 13. (c) Total V.P.,
such without any change in composition P = PAo X A + PBo XB = PAo X A + PBo (1 - XA )
are called azeotropes.
Solution of HNO3 and H2O will form = (PAo - PBo )X A + PBo
maximum boiling point azeotrope. Maximum
boiling azeotropes show negative deviation Thus, PB° = 114 torr ; PAo - PBo = 52
from Raoult’s law. or PAo = 166 torr
Composition (%)Boiling Point 1 1
Hence P = 166 ´ + 114 ´ = 140 torr
HNO3 68.0 359 K 2 2
H2O 32.0 373 K 28
14. (b) Moles of solute = =1;
Boiling point of the azeotrope of these two 28
solutions is 393.5 K. 100 - 28
7. (a) A solution of acetone in ethanol shows moles of water = =4
18
positive deviation from Raoult's law. It is V.P. of solution
because ethanol molecules are strongly
4
hydrogen bonded. When acetone is added, = PHo 2O ´ X H2O = 760 ´ = 608 torr
these molecules break the hydrogen bonds 5
and ethanol becomes more volatile. 15. (a) Posolvent = 760 torr (at the b. pt.);
Therefore its vapour pressure is increased. nsolute = 1
8. (d) When the aqueous solution of one molal 1000
nsolvent = = 25 ;
KI is diluted with water, concentration de- 40
creases, therefore the vapour pressure of 760
the resulting solution increases. DP = 760 × Xsolute = torr = 29.23 torr
26
Solutions 185
16. (c) For dilute solution, 12
22. (a) Moles of urea = = 0.2
ΔP n 60
Þ solute For solution in A,
P° n solvent
68.4
Moles of sucrose = = 0.2
ΔPA W/M W MA 342
= = × ....(i)
P°A WA /MA M WA Both are non electrolyte hence lowering of
ΔPB W MB V.P. will be same.
For solution in B, = × ....(ii)
P°A M WB 23. (d) Lesser the intermolecular forces, the more
From (i) and (ii), the volatile character.
24. (d) At 1 atmospheric pressure the boiling point
ΔPA /P°A M W M
= 2 = A B = A (WA = WB) of mixture is 80°C.
ΔPB /P°B M BWA MB
At boiling point the vapour pressure of
MA = 2MB mixture, PT = 1 atmosphere = 760 mm Hg.
17. (b) Vdissolved = KPH2 Using the relation,
80 ´ 1425 PT = PA° x A + PB°x B , we get
PH2 in the mixture = XH2P = = 1140
100
PT = 520x A + 1000(1 - x A )
1140
torr = = 1.5 atm
760
{Q PAo = 520 mm Hg ,
Hence, Vdissolved in 2L of water = 2 × K × PH2
= 2 × 18 × 1.5 = 54 mL PB° = 1000 mm Hg; x A + x B = 1
18. (a) Vapour pressure of solution = PA + PB
or 760 = 520x A + 1000 - 1000x A or 480x A = 240
120 ´ 2 180 ´ 3
= PAo X A + PBo X B = + = 156 mm Hg 240 1
5 5 or x A = = = or 50 mol percent
120 ´ 2 180 ´ 3 480 2
P X +P X = + = 156 mm Hg
5 5
19. (d) According to Henry’s law the mass of a gas 25. (b) Total V.P. of solution = PA° x A + PB° x B
dissolved per unit volume of solvent is
proportional to the pressure of the gas at Given, PA° = 74.7 torr, PB° = 22.3 torr
constant temperature m = Kp i.e. as the nbenzene = 1.5 mol, ntoluene = 3.5 mol
solubility increases, value of Henry’s law nsolution = 1.5 + 3.5 = 5 mol
constant decreases. Since CO 2 is most
n 1.5
soluble in water among the given set of x A = benzene =
gases. Therefore CO2 has the lowest value nsolution 5
of Henry’s law constant.
Total V.P. of solution
20. (a) Minimum boiling azeotrope is formed by
solution showing positive deviation. e.g. æ 1.5 3.5 ö
=ç ´ 74.7 + ´ 22.3 ÷ torr
acetone – ethanol. è 5 5 ø
21. (a) Total vapour pressure = vapour pressure = (22.4 + 15.6) torr = 38 torr
of pure benzene + vapour pressure of
Mole fraction of benzene in vapour form =
toluene
= 100 + 50 = 150 mm 22.4
= 0.589
We know, 38
PCo 6H 6 = P × XC6H6 26. (b) If we plot a graph between the vapour
pressure and the temperature. We would
100 = 150 × XC6H6
get a curve that rises faster as T increases,
100 giving a curved line.
XC6H6 = = 0.67
150
EBD_7587
186 Chemistry Objective MCQs
Since observed vapour pressure of solution
< ideal vapour pressure, the solution shows
Vapour Pressure/mm Hg negative deviation.
32. (b) From the given data
PH2 = 1500 × 0.80
1200
= 1200 mm of Hg = atmosphere = 1.58
760
atmosphere
If 's are volumes of gas dissolved by same
Temp (°C) volume of liquid, then from Henry's law
V2 P V 1.58
27. (d) Solutions in which solute - solute and = 2 ; or 2 =
V1 P1 20 1.0
solvent-solvent interactions are almost
similar to solute solvent interactions are or V2 = 31.60 mL
known as ideal solution. 33. (c) V.P. of solution at t°C = 760 mm
28. (b) PA = PAo × xA = total pressure × yA [at b.p., V.P. of solution =atompheric
pressure]
PB = PBo × xB = total pressure × yB Thus = P° A.xA + P°B.xB
where x and y represents mole fraction in or P = P° .x
A A + P°
B.(1 – xA) [QxA + xB = 1]
liquid and vapour phase respectively. or 760 = 400XA + 800(1 – XA) [Q P = 760
mm of Hg]
PBo x B yB PBo (1 - xA ) 1 - y A
= ; = or – 800 + 760 = – 400 xA
PAo x A yA PAo xA yA
or – 40 = – 400 xA
119(1 - 0.50) 1 - yA 40
on putting values = or xA = = 0.1
37 ´ 0.50 yA 400
on solving yA = 0.237 Thus mole fraction of in solution is 0.1
29. (d) DTf = Kf m 34. (a) The given data are
where m = molality Pwater = 17.0 torr;
w Ptotal (4 mole % solution)
273 – 268 = 1.86 ×
M ´v = PNH3 + Pwater = 50.0 torr
w
5 = 1.86 × X NH3 = 0.04 and Xwater = 0.96
32 ´ 10
5 ´ 32 ´10 Now according to Raoult’s law;
w=
1.86 °
Pwater = X water Pwater
= 860.2 » 868.06 g
30. (d) Moles of CO2 dissolved = 0.96 × 17.0 torr = 16.32 torr
Now Henry’s law constant for ammonia is
= K × PCO 2 × Vbottle = 3 ´ 10 -2 ´ 4 ´ 100 = 12 ;
PNH3 33.68 torr
Mass of CO2 = 12 × 44 = 528 g K H ( NH3 ) = = = 842 torr
X NH3 0.04
31. (d) For ideal solution vapour pressure of
solution Hence, for 5 mole % solution, we have
= P°
AXA + P°
BXB PNH3 = K H ( NH3 ) X NH3
2 3
= 80 ´ + 100 ´ = 92 torr = (842 torr) (0.05) = 16.15 torr
5 5
Solutions 187
Thus, Ptotal (5 mole % solution) 39. (b) As DTf = Kf.m
= PNH 3 + Pwater = 42.1 + 16.15 = 58.25 torr DTb = Kb.m
DTf DTb
35. (c) DTb = Kb × m Hence, we have m = =
Kf Kb
Elevation in boiling point is a colligative
Kf
property, which depends upon the no. of or DTf = DTb
particles (concentration of solution). Thus Kb
greater the number of particles, greater is Þ [DTb = 100.18 - 100 = 0.18°C]
the elevation in boiling point and hence
greater will be its boiling point. = 0.18 ×
1.86
= 0.654°C
ˆˆ†
Na2SO4 ‡ˆˆ 2Na+ + SO42– 0.512
As the freezing point of pure water is 0°C,
Since Na2SO4 has maximum number of
particles hence it has maximum boiling DTf = 0 –Tf
point. 0.654 = 0 – Tf
\ Tf = – 0.654
No. of particles after ionisation
36. (a) (i) i = Thus the freezing point of solution will
No. of particles before ionisation
be – 0.654°C.
(ii) DTb = i × Kb × m
w
CuCl2 ¾® Cu2+ + 2Cl– 40. (b) pV = RT
m
1 0 0 4
(1 – a) a 2 \ 6 ´ 10 -4 ´ 1 = ´ 0.0821 ´ 300;
m
1 + 2a
m = 1.64 × 105.
i= , = 1 + 2a 41. (a) Van’t Hoff equation is
1
Assuming 100% ionization pV = inRT
So, i = 1 + 2 = 3 For depression in freezing point.
DTb = 3 ´ 0.52 ´ 0.1 = 0.156 » 0.16 D Tf = i × Kf × m
For elevation in boiling point.
æ 13.44 ö
çm = = 0.1÷ D Tb = i × Kb × m
è 134.4 ø
For lowering of vapour pressure,
nRT pV
37. (b) p= or T =
V nR °
Psolvent - Psolution æ n ö
= iç ÷.
10% (w/v) aqueous solution of glucose °
Psolvent è N+n ø
means 10 g glucose is present in 100 mL of
H2O 42. (b) DTf = i.Kf .m ; 0.0054 = i × 1.8 × 0.001
14 ´ 100 ´ 10-3 ´ 180
i = 3 so it is [Pt(NH3)4Cl2]Cl2.
= = 307.3 K 43. (c) From the relation, DTf = iKf m, it is evident
0.082 ´ 10
that DTf µ i. Since i is maximum for ethanol
38. (d) DTf = iK f m (oth ers are non electrolytes), DT f
KBr ¾® K+ + Br– (depression in freezing point) will be
1 0 0 maximum for C2H5OH and hence freezing
point will be minimum for ethanol solution.
(1 – a) a a
44. (c) If compound dissociates in solvent i > 1
i=1+a
and on association i < 1.
Here a = 0.8 (80% ionization)
45. (a) As both the solutions are isotonic hence
= 1.8
there is no net movement of the solvent
DTf = (1.8) (1.86) (0.4) = 1.339 through the semipermeable membrane
Tf = – 1.339 °C between two solutions.
EBD_7587
188 Chemistry Objective MCQs
46. (d) Acetic acid contain carboxylic group – DTf = 0°C – (– 23°C) = 23°C
COOH which can form H-bonding so acetic
M normal 136.3
acid dimerises. Now, i = = = 2.47
M observed 55
O H –– O
CH3 –– C C –– CH3 ZnCl2 Zn ++ + 2Cl -
1- a a 2a
––
O H O
47. (a) Given pob = 10.8 atm Van't Hoff factor (i)
pnor = CRT = 0.10 × 0.0821 × 298 = 2.446 1 - a + a + 2a
now experimental value of (i) = = 2.47
1
Observed osmotic pressure \ a = 0.735 = 73.5%
=
Normal osmotic pressure 51. (b) As DTf = Kf . m
For,
10.8
= = 4.42
2.446 HX ˆˆ†
‡ˆˆ H+ + X-
t=0 1 0 0
P o - Ps n w M
48. (c) = = ´ t = eq (1 - 0.20) 0.20 0.20
Po N m W
Total no. of moles = 1 – 0.20 + 0.20 + 0.20
12 18
0.1 = ´ = 1 + 0.20 = 1.2
m 108
\ DTf = 1.2 × 1.86 × 0.5 = 1.1160 » 1.12 K
12 ´18 52. (d) Osmotic pressure (p) = CRT
m= = 20
0.1 ´108
wt ´ 1000
49. (c) van’t Hoff factor (i) and the degree of (p) = RT
Molecular mass ´ V
association are related as below :
æ 1ö 1.26 ´ 1000
i = 1 - a ç1 - ÷ = 2.57 × 10–3 = ´ 0.083 ´ 300
è nø Mol.mass ´ 200
Molecular mass = 61038 g
æ 1ö
0.9 = 1 - 0.2 ç 1 - ÷
è nø WB
53. (b) ΔTb = K b ×1000 ;
On solving, M B × WA

æ 1ö 1 WB
ç1 - ÷ = ΔTf = K f ×1000 ;
è nø 2 M B × WA
1 1 1
= 1- = ΔTb K b
= =
ΔTb
=
0.512
= 0.0512°C.
n 2 2 ΔTf K f – 0.186 1.86
\ n=2
10
k f ´ w ´ 1000 54. (c) Moles of glucose = 0.05,
50. (b) Mol.wt. = 180
DT f ´ W
10 10
urea = = 0.16, Sucrose = = 0.029
1.86 ´ 0.85 ´ 1000 60 342
= » 55 g
0.23 ´ 125 Hence osmotic presure P2 > P1 > P3 ( p µ C)
55. (d) Colligative properties depends upon the no.
Where
of particles. Since methanol is non
w = 0.85g
electrolyte hence cannot be considered.
W = 125 g
Solutions 189
56. (d) (100 + DTb) – (0 – DTf) = 105 1000
DTb + DTf = 5 Csucrose = 0.0146 × = 0.146
100
m (Kb + Kf) = 5 moles of KCl(effective)
5 5
m= i.e., moles in 1000 g water 5 5 0.134 ´ 1000
2.37 2.37 = 2´ = ; = 1.34
74.5 37.25 100
5 pµC
(or) moles in 100 g water
2.37 ´ 10
this order is p4 > p1 > p2 > p3
5 62. (b) Greater is the effective molarity (i × C),
\ Wt. of sucrose = ´ 342 = 72g
2.37 ´ 10 higher the DTf value and lower the freezing
57. (d) DTf = Kf × m point.
1000 × K f × w 2 (solute ) (i) 0.1 M NaCl = i × C = 2 × 0.1 = 0.2
M=
ΔTf × w1 (solvent ) (ii) 0.05 M BaCl2 = i × C = 3 × 0.05 = 0.15
1000 ´ 1.86 ´ 1.8 (iii) 0.1 M NH2CSNH2 = i × C = 1 × 0.1 = 0.1
= Þ M = 180
0.465 ´ 40 (iv) 0.025 M KNO3 = i × C = 2 × 0.025 = 0.50
Molecular formula = (empirical formula)n Thus, order is (i) < (ii) < (iii) < (iv)
Molecular mass 180 63. (c) ΔTf (normal) = K f m = 1.86 × 0.01 = 0.0186 ;
n= = =6
Empirical formula mass 30
Molecular formula = (CH2O)6 = C6H12O6. ΔTf(obs) 0.0205
i= = = 1.10 = 1 + a ;
ΔTf(nor) 0.0186
58. (d) DTf = i.K f .m; DTb = i.K b .m
a = 0. 1
DTf K f
=
DTb K b Ca 2 0.01 ´ 0.12 1
Ka = = = ´ 10 -3 ;
DTf = 0 - (-0.186°C) = 0.186°C 1- a 1 - 0.1 9

0.186 1.86 0.52 ´ 0.186 Kw


= Þ DTb = = 0.052 Kb = = 1.0 ´ 10 -14 ´ 9 ´ 103 = 9 ´ 10 -11
DTb 0.52 1.86 Ka

K b ´ w ´ 1000 5.58
59. (c) DTb = ; 64. (c) i= =3 ;
M´ W 1.86

DTb ´ 100 ´ 100 [Cr(H 2 O)5 Cl]Cl 2 .H 2 O ¾¾


®
\Kb = = DTb
10 ´ 1000
[Cr(H 2 O)5 Cl]2 + + 2Cl -
60. (d) DTf = Kf × m × i. Since Kf has different values
for different solvents, hence even if m is 65. (a) [Pt(NH3 ) 6 ]Cl4 ® [Pt(NH3 ) 6 ]4 + + 4Cl - ;
the same DTf will be different
i = 1 + 4 = 5 (maximum)
5
61. (c) Moles of urea = ; 66. (d) p µ iCT; iCT is maximum for 0.03 M NaCl at
60
50 °C.
5
moles of fructose = ;
180 20
67. (a) Mass of oxygen = ´ 0.04 = 0.008 g / L
0.08 ´ 1000 100
Curea = Þ 0.833
100 Mass of nitrogen
5 80
moles of sucrose = ; = ´ 0.02 = 0.016 g / L
342
100
EBD_7587
190 Chemistry Objective MCQs
68. (b) Sodium sulphate dissociates as 75. (c) Molecular mass of phosphorous
Na 2SO 4 (s) ¾¾® 2Na + + SO -4 - 1000 × K f × Wsolute
=
hence van’t hoff factor i = 3 ΔTf × Wbenzene
Now D T f = i k f .m
1000 ´ 5.12 ´ 2.48
= 3 × 1.86 × 0.01 = 0.0558 K = = 124.5 = (P) x
1.02 ´ 100
69. (b) Using the relation,
w.R.T 124.5
M = , or, x = =4
pV 31

6 ´ 0.080 ´ 300 76. (c) The given solution is unsaturated


we get M = = 3.6 × 105 (saturated one contains 5 g of solute in 50 g
-3
2 ´ 10 ´ 0.2 of water). On evaporation, concentration of
70. (b) pV = CRT solution in creases to saturation an d
p1 C1T1 thereafter becomes constant due to excess
=
p 2 C 2 T2 solid separating out. Hence, osmotic
pressure first increases and then becomes
C
p1 = P, p2 = 2 atm. C1 = C, C 2 = constant.
2
T1 = 600 K, T2 = 700 K nRT1 nRT2
77. (c) π1 = , π2 =
P 2 ´ C ´ 600 V1 V2
=
2 C ´ 700 p1 580 V2T1 V2 ´ 290
24 Hence, = = =
P= . p2 165 V1T2 V1 ´ 330
7
71. (a) CaCl 2 acts as a non-volatile solute and V2
Þ =4
results in depression in freezing point. Thus, V1
snow is reduced and prevents blocking of
78. (d) mKf = DTf has the units of temperature . Van’t
roads in the polar region.
Hoff factor, i, is dimensionless quantity.
72. (c) ® 2Na+ + SO 24-
Na2SO4 ¾¾
79. (c) DTf (normal) = Kf m = 1.86 × 0.5 = 0.93°
Mol. before Assuming 100% ionization , i = n = No. of
dissociation 1 0 0 ions per molecule
Mol. after
dissociation 1 – a 2a 1a Observed DTf 3.72
= = =4
i = 1 - a + 2a + a = 1 + 2a Normal DTf 0.93

73. (b) Among the given options, CaCl 2 solution K3[Fe(CN)6] ¾® 3K+ + [Fe(CN)6]3–
will produce maximum (three) ions per no. of ions = 4
molecule, so it will show minimum freezing 80. (a) For non electrolyte solute (glucose) i = 1
point.
For BaCl2 ‡ˆˆˆˆ† Ba2+ + 2Cl–
w 1000
74. (d) DTb = Kb ´ i = 3 (100 % ionized)
M W
i = (number of ions afterionization × a) + (1 – a)
3 1000
0.52 = 0 .6 ´ ´ (W = 200 ´1) ) = (3 × 1) + (1 – 1) = 3
m 200
p = (iC1 – i2C2)RT
1.8 ´ 5 Þ (1 × 0.2 – 3 × 0.05) 0.0821 × 300
m= = 17.3g mol -1
0.52 Þ 1.23 atm
17 Electrochemistry
1. Which of the following statement is correct? 5. Zn gives H2 gas with H2SO4 and HCl but not
(a) Cathode is –ve terminal in both, glavanic
with HNO3 because
and electrolytic cells.
(b) Anode is +ve terminal in both, galvanic and (a) Zn acts as oxidizing when reacts with HNO3.
electrolytic cells. (b) HNO3 is weaker acid than H2SO4 and HCl.
(c) Cathode and anode are –ve terminal in (c) In electrochemical series Zn is above
electrolytic and galvanic cell.
hydrogen.
(d) Cathode and anode are +ve terminal in
electrolytic and galvanic cell. (d) NO3– is reduced in preference to hydronium.
2. Based on the cell notation for a spontaneous 6. A smuggler could not carry gold by depositing
reaction, at the anode : iron on the gold surface since
Ag (s) | AgCl (s) | Cl– (aq) || Br – (aq) | Br2 (l) | C (s) (a) gold is denser
(a) AgCl gets reduced (b) iron rusts
(b) Ag gets oxidized (c) gold has higher reduction potential than iron
(c) Br– gets oxidized (d) gold has lower reduction potential than iron
(d) Br2 gets reduced 7. Standard electrode potential for Sn4+ / Sn2+ couple
3. For the electrochemical cell is + 0.15 V and that for the Cr3+ / Cr couple is
–0.74V. These two couples in their standard state
Pt (s)ô H 2 (g) ôH + (1M) ô, Cu (s) which one of are connected to make a cell. The cell potential will
1 atm
be:
the following statements is true?
(a) + 1.19 V (b) + 0.89 V
(a) H + ions are formed at anode and Cu is
deposited at cathode. (c) + 0.18 V (d) + 1.83 V
(b) H2 is liberated at cathode and Cu is 8. A solution contains Fe , Fe3+ and I– ions. This
2+

deposited at anode. solution was treated with iodine at 35°C. E° for


(c) Oxidation occurs at cathode. Fe3+ / Fe2+ is + 0.77 V and E° for I2/2I– = 0.536 V.
(d) Reduction occurs at anode. The favourable redox reaction is :
4. The standard reduction potentials at 298K for the (a) I2 will be reduced to I–
following half reactions are given against each (b) There will be no redox reaction
Zn2+ (aq) + 2e- Zn(s) ; –0.762 V (c) I– will be oxidised to I2
(d) Fe2+ will be oxidised to Fe3+
Cr3+ (aq) + 3e - Cr (s); –0.740 V 9. Consider the following four electrodes:
P = Cu2+ (0.0001 M)/Cu(s)
2H + (aq) + 2e - H2 (g) ; 0.00 V
Q = Cu2+ (0.1 M)/Cu(s)
R = Cu2+ (0.01 M)/Cu(s)
Fe3+ (aq) + e - Fe 2+ (aq) ; 0.770 V S = Cu2+ (0.001 M)/Cu(s)
Which is the strongest reducing agent? If the standard reduction potential of Cu2+/Cu
is +0.34 V, the reduction potentials in volts of
the above electrodes follow the order.
(a) Zn (s) (b) Cr (s) (a) P > S > R > Q (b) S > R > Q > P
(c) H2 (g) (d) Fe3+ (aq) (c) R > S > Q > P (d) Q > R > S > P
EBD_7587
192 Chemistry Objective MCQs
10. Following cell has EMF 0.7995V. 16. Which of the following will form a cell with the
Pt | H2 (1 atm) | HNO3 (1M) || AgNO3 (1M) | Ag highest voltage?
If we add enough KCl to the Ag cell so that the (a) 1 M Ag + , 1M Co 2 +
final Cl– is 1M. Now the measured emf of the cell
is 0.222V. The Ksp of AgCl would be – (b) 2 M Ag + , 2 M Co 2+

(c) 0.1 M Ag + , 2 M Co 2 +
(a) 1 × 10–9.8 (b) 1 × 10–19.6
(c) 2 × 10–10 (d) 2.64 × 10–14 (d) 2 M Ag + , 0.1M Co 2 +
11. Standard cell voltage for the cell
17. The standard EMF for the cell reaction,
Pb | Pb2+ || Sn2+ | Sn is – 0.01 V. If the cell is to
exhibit Ecell = 0, the value of [Sn2+] / [Pb2+] should Zn + Cu 2 + ¾
¾® Cu + Zn 2 + is 1.1 volt at 25°C.
be antilog of – The EMF for the cell reaction, when 0.1 M Cu 2+
(a) + 0.3 (b) 0.5 (c) 1.5 (d) – 0.5 and 0.1 M Zn2+ solutions are used, at 25°C is
12. An electrochemical cell is set up as: (a) 1.10 V (b) 0.10 V
Pt; H2 (1atm)|HCl (0.1 M) || CH3COOH (0.1 M)| H2 (c) –1.10 V (d) –0.110 V
(1atm); Pt. The e.m.f of this cell will not be zero, 18. Consider the following reactions
because
(a) the temperature is constant (i) Cd 2 + (aq ) + 2e - ¾¾
® Cd( s ) , E° = – 0.40 V
(b) e.m.f depends on molarities of acids used
(ii) Ag + (aq ) + e - ¾¾® Ag(s), E° = 0.80 V
(c) acids used in two compartments are different
For the galvanic cell involving the above
(d) pH of 0.1 M HCl and 0.1 M CH 3COOH is
reactions. Which of the following is not correct ?
not same
13. A variable, opposite external potential (Eext) is
applied to the cell
Zn|Zn2+ (1 M) || Cu2+ (1 M) | Cu, of potential 1.1 V. (a) Molar concentration of the cation in the
When Eext < 1.1 V and Eext > 1.1 V, respectively cathodic compartment changes faster than
electrons flow from :
that of the cation in the anodic compartment.
(a) anode to cathode in both cases
(b) cathode to anode and anode to cathode (b) Ecell increase when Cd2+ solution is diluted.
(c) anode to cathode and cathode to anode (c) Twice as many electrons pass through the
(d) cathode to anode in both cases cadmium electrode as through silver electrode.

14. The standard electrode potentials E


o
( +
M /M ) of (d) E cell decreases when Ag + solution is
diluted.
four metals A, B, C and D are – 1.2 V, 0.6 V, 0.85 V
and – 0.76 V, respectively. The sequence of 19. Given that : E° = 0.80V and [Ag + ] = 10-3 M ;
Ag+ / Ag
deposition of metals on applying potential is: 2+ -1
(a) A, C, B, D (b) B, D, C, A E° = 0.785 V and [Hg 2 ] = 10 M
Hg 22+ / Hg
(c) C, B, D, A (d) D, A, B, C
15. A current of 10.0 A flows for 2.00 h through an which is true for the cell reaction
electrolytic cell containing a molten salt of metal 2Hg(l ) + 2Ag + (aq) ® 2Ag( s) + Hg 22+ (aq) ?
X. This results in the decomposition of 0.250 mol
of metal X at the cathode. The oxidation state of (a) The forward reaction is spontaneous
X in the molten salt is: (F = 96,500 C) (b) The backward reaction is spontaneous
(c) Ecell = 0.163 V
(d) Ecell = 1.585 V
(a) 1 + (b) 2 + (c) 3 + (d) 4 +
Electrochemistry 193
20. Use the following standand electrode potentials, 26. When electric current is passed through acidified
calculate DG° in kJ/mol for the indicated reaction: water, 112 mL of hydrogen gas at STP collected
5Ce4 + (aq) + Mn 2 + (aq) + 4H 2 O(l) ¾¾
® at the cathode in 965 seconds. The current passed
in amperes is
5Ce3+ (aq) + MnO -4 (aq) + 8H + (aq) (a) 1.0 (b) 0.5 (c) 0.1 (d) 2.0
MnO -4 (aq) + 8H + (aq) + 5e- ¾¾
® 27. On passing current through two cells, connected
in series containing solution of AgNO3 and
Mn 2 + (aq) + 4H 2O(l); E° = +1.51 V CuSO4, 0.18 g of Ag is deposited. The amount of
the Cu deposited is:
Ce 4 + (aq) + e - ¾¾
® Ce3+ (aq) ; E° = + 1.61 V
(a) 0.529 g (b) 10.623 g
(a) – 9.65 (b) – 24.3 (c) – 48.25(d) – 35.2
21. If 0.01 M solution of an electrolyte has a (c) 0.0529 g (d) 1.2708 g
resistance of 40 ohms in a cell having a cell 28. If x is the specific resistance of the solution and
constant of 0.4 cm–1, then its molar conductance N is the normality of the solution, the equivalent
in ohm–1 cm2 mol–1 is conductivity of the solution is given by
1000 x 1000
(a) 102 (b) 104 (c) 10 (d) 103 (a) (b)
N Nx
22. When a concentrated solution of an electrolyte
is diluted 1000N Nx
(a) its specific conductance increases. (c) (d)
x 1000
(b) its equivalent conductivity decreases.
(c) its specific conductivity decreases and 29. For an electrolyte solution of 0.05 mol L– 1,
equivalent conductivity increases. the conductivity has been found to be 0.0110 S
(d) both specific and equivalent conductivity cm– 1. The molar conductivity is
increase. (a) 0.055 S cm2 mol– 1
23. The limiting equivalent conductivity of NaCl, (b) 550 S cm2 mol– 1
KCl and KBr are 126.5, 150.0 and 151.5 S cm2
(c) 0.22 S cm2 mol– 1
eq–1, respectively. The limiting equivalent ionic
conductivity for Br– is 78 S cm2eq–1. The limiting (d) 220 S cm2 mol– 1
equivalent ionic conductivity for Na+ ions would 30. The highest electrical conductivity of the
be : following aqueous solutions is of
(a) 128 (b) 125 (c) 49 (d) 50 (a) 0.1 M difluoroacetic acid
24. The ionic conductivity of Ba2+ and Cl– at infinite (b) 0.1 M fluoroacetic acid
dilution are 127 and 76 ohm –1 cm 2 eq –1
respectively. The equivalent conductivity of BaCl2 (c) 0.1 M chloroacetic acid
at infinity dilution (in ohm–1cm2 eq–1) would be : (d) 0.1 M acetic acid
(a) 203 (b) 279 (c) 101.5 (d) 139.5 31. A current of 2.0 A passed for 5 hours through
25. Aluminium oxide may be electrolysed at 1000°C a molten metal salt deposits 22.2 g of metal
to furnish aluminium metal (At. Mass = 27 amu; 1 (At wt. = 177). The oxidation state of the metal in
Faraday = 96,500 Coulombs). The cathode
the metal salt is
3+ -
reaction is– Al + 3e ® Al (a) +1 (b) +2 (c) +3 (d) +4
32. The specific conductivity of N/10 KCl solution
To prepare 5.12 kg of aluminium metal by this
at 20°C is 0.212 ohm–1 cm–1 and the resistance of
method we require electricity of
the cell containing this solution at 20°C is 55 ohm.
(a) 5.49 × 101 C (b) 5.49 × 10 4 C The cell constant is
(a) 4.616 cm–1 (b) 11.66 cm–1
(c) 1.83 × 10 7 C (d) 5.49 × 10 7 C
(c) 2.173 cm–1 (d) 3.324 cm–1
EBD_7587
194 Chemistry Objective MCQs
33. The equivalent conductance at infinite dilution 38. On passing a current of 1.0 ampere for 16 min and
of a weak acid such as HF 5 sec through one litre solution of CuCl2, all copper
(a) can be determined by extrapolation of of the solution was deposited at cathode. The
measurements of dilute solutions of HCl, strength of CuCl2 solution was (Molar mass of
HBr and HI Cu= 63.5; Faraday constant = 96500 Cmol–1)
(b) can be determined by measurement of very
dilute HF solutions
(c) can be determined from measurements of
dilute solutions of NaF, NaCl and HCl (a) 0.01 N (b) 0.01 M
(d) is an undefined quantity (c) 0.02 M (d) 0.2 N
34. Conductance of 0.1 M KCl (conductivity = X 39. Electrode potential of the half cell Pt (s) | Hg (l) |
-1 -1 ) filled in a conductivity cell is Y Hg2Cl2(s) | Cl– (aq) can be increased by :
Ohm cm
(a) Increasing [Cl–]
-1
Ohm . If the conductance of 0.1 M NaOH filled (b) Decreasing [Cl–]
(c) Increasing Hg2Cl2 (s)
in the same cell is Z Ohm-1 , the molar
(d) Decreasing Hg (l)
conductance of NaOH will be 40. Identify the correct statement :
XZ XZ (a) Corrosion of iron can be minimized by
(a) 103 (b) 104
Y Y forming a contact with another metal with a
XZ
higher reduction potential
XZ
(c) 10 (d) 0.1 (b) Iron corrodes in oxygen free water
Y Y
(c) Corrosion of iron can be minimized by
35. In electrolytic reduction of a nitroarene with 50% forming an impermeable barrier at its surface
current efficiency 20.50 g of the compound is
(d) Iron corrodes more rapidly in salt water
reduced by 2 × 96500 C of electric charge. The
molar mass of the compound is because its electrochemical potential is
higher
(a) 123.0 g (b) 61.5 g
41. During the charging of lead storage battery, the
(c) 10.2 g (d) 20.5 g
reaction at anode is represented by :
36. Electrolysis of NaCl solution with inert electrodes
for certain period of time gave 600 cm 3 of 1.0 M (a) Pb2 + + SO24 - ¾¾
® PbSO 4
NaOH in the electrolytic cell. During the same (b) ® PbO 2 + SO 42 -
PbSO 4 + 2H 2 O ¾¾
period 31.80 g of copper was deposited in a copper
voltmeter in series with the electrolytic cell. What
+ 4H + + 2e -
is the percent current efficiency in the electrolytic
cell ? (At. wt. of Cu = 63. 6) (c) ® Pb 2 + + 2e -
Pb ¾¾
(a) 40 (b) 25 (c) 60 (d) 50
(d) Pb 2+ + 2e - ¾¾ ® Pb

COO 42. Which colourless gas evolves, when NH4Cl
37. Given the ionic conductance of | , K+, and
COO – reacts with zinc in a dry cell battery
(a) NH4 (b) N2 (c) H2 (d) Cl2
Na are 74, 50, and 73 cm ohm – 1 eq – 1,
+ 2

respectively. The equivalent conductance at 43. Given


Fe3+ (aq) + e– ® Fe2+ (aq); Eo = + 0.77 V
COONa
Al3+ (aq) + 3e– ® Al(s); Eo = – 1.66 V
infinite dilution of the salt | is
COOK Br2(aq) + 2e– ® 2Br–; Eo = + 1.09 V
Considering the electrode potentials, which of
the following represents the correct order of
(a) 197 cm2 ohm– 1 eq– 1 reducing power?
(b) 172 cm2 ohm– 1 eq– 1 (a) Fe2+ < Al < Br – (b) Br– < Fe2+ < Al
(c) 135.5 cm2 ohm– 1 eq– 1 (c) Al < Br – < Fe2+ (d) Al < Fe2+ < Br–
(d) 160.5 cm2 ohm– 1 eq– 1
Electrochemistry 195
44. How many electrons would be required to deposit 50. An unknown metal M displaces nickel from nickel
6.35 g of copper at the cathode during the (II) sulphate solution but does not displace
electrolysis of an aqueous solution of copper manganese from manganese sulphate solution.
sulphate? (Atomic mass of copper = 63.5 u, NA = Which order represents the correct order of
Avogadro’s constant): reducing power?
NA NA NA NA (a) Mn > Ni > M (b) Ni > Mn > M
(a) (b) (c) (d)
20 10 5 2 (c) Mn > M > Ni (d) M > Ni > Mn
45. Given :
51. What is the e.m.f for the given cell?
Eo 1 = 1.36 V, E o = - 0.74 V,
Cl2 / Cl - Cr3+ / Cr Cr | Cr 3+ (1.0M ) || Co 2 + (1.0M ) | Co
2
Eo = 1.33V, E o = 1.51V (E° for Cr3+ / Cr = -0.74 volt and E° for
Cr2O27- / Cr3+ MnO-4 / Mn 2+
The correct order of reducing power of the Co 2+ / Co = -0.28 volt)
species (Cr, Cr3+, Mn2+ and Cl–) will be : (a) – 0.46 volt (b) – 1.02 volt
(a) Mn2+ < Cl– < Cr3+ < Cr
(c) + 0.46 volt (d) 1.66 volt
(b) Mn2+ < Cr3+ < Cl– < Cr
(c) Cr3+ < Cl– < Mn2+ < Cr 52. The standard electrode potential (E°) for
(d) Cr3+ < Cl– < Cr < Mn2+ 1
OCl– /Cl – and Cl - / Cl 2 respectively are 0.94
46. The oxidation potential of a hydrogen electrode 2
at pH = 10 and pH 2 = 1 atm is 1
(a) – 0.59 V (b) 0.00 V V and –1.36 V. The E° value for OCl - / Cl 2 will
2
(c) + 0.59 V (d) 0.059 V be
47. If the E°cell for a given reaction has a negative
value, then which of the following gives the (a) –0.42 V (b) –2.20 V
correct relationships for the values of DG° and (c) 0.52 V (d) 1.04 V
Keq? 53. For the reduction of silver ions with copper metal,
the standard cell potential was found to be + 0.46
(a) DG° > 0 ; Keq > 1 (b) DG° < 0 ; Keq > 1 V at 25°C. The value of standard Gibbs energy,
(c) DG° < 0 ; Keq < 1 (d) DG° > 0 ; Keq < 1 DG° will be (F = 96500 C mol–1 )
48. The EMF of the cell
Tl/Tl+ (0.001M) || Cu2+ (0.01M) /Cu is 0.83. The (a) – 89.0 kJ (b) – 89.0 J
cell EMF can be increased by (c) – 44.5 kJ (d) – 98.0 kJ
(a) Increasing the concentration of Tl+ ions. 54. In a cell that utilises the reaction
(b) Increasing the concentration of Cu2+ ions.
(c) Increasing the concentration of Tl + and Zn( s) + 2H + ( aq ) ® Zn 2+ ( aq ) + H 2 ( g )
Cu2+ ions. addition of H2SO4 to cathode compartment, will
(d) None of these (a) increase the E and shift equilibrium to the right
49. Small quantities of solutions of compounds TX, (b) lower the E and shift equilibrium to the right
TY and TZ are put into separate test tubes (c) lower the E and shift equlibrium to the left
containing X, Y and Z solution. TX does not (d) increase the E and shift equilibrium to the left
react with any of these. TY reacts with both X 55. For a cell reaction involving two electron change,
and Z. TZ reacts with X. The decreasing order of the standard EMF of the cell is 0.295 V at 2°C.
state of oxidation of the anions X–, Y–, Z– is
The equilibrium constant of the reaction at 25°C
will be:
(a) Y–, Z–, X– (b) Z–, X–, Y– (a) 29.5 × 10–2 (b) 10
(c) Y–, X–, Z– (d) X–, Z–, Y– (c) 1 × 10 10 (d) 2.95 × 10–10
EBD_7587
196 Chemistry Objective MCQs
56. On the basis of the information available Na 2Cr2 O 7 + Cr + H + ® Cr 3+ + H 2O + Na +
4 2 If one Faraday of electricity is passed through
from the reaction Al + O 2 ® Al 2 O 3 , DG
3 3 the battery during the charging, the number of
= – 827 kJ mol –1 of O2 the minimum e.m.f moles of Cr3+ removed from the solution is
required to carry out an electrolysis of Al2O3
is (F = 96500 C mol –1) 4 1 3 2
(a) (b) (c) (d)
3 3 3 3
(a) 8.56 V (b) 2.14 V (c) 4.28 V (d) 6.42 V 63. The standard potentials of Ag+/Ag, Hg22+/2Hg,
57. The e.m.f. of a Daniell cell at 298 K is E1. Cu2+/Cu and Mg2+/Mg electrodes are 0.80, 0.79,
0.34 and –2.37 V, respectively. An aqueous
ZnSO4 CuSO4
Zn Cu solution which contains one mole per litre of the
(0.01 M) (1.0 M) salts of each of the four metals is electrolyzed.
When the concentration of ZnSO4 is 1.0 M and With increasing voltage, the correct sequence
that of CuSO4 is 0.01 M, the e.m.f. changed to E2. of deposition of the metals at the cathode is
What is the relationship between E1 and E2? (a) Ag, Hg, Cu, Mg (b) Cu, Hg, Ag only
(a) E2 = 0 ¹ E1 (b) E1 > E2 (c) Ag, Hg, Cu only (d) Mg, Cu, Hg, Ag
(c) E1 < E2 (d) E1 = E2 64. When electric current is passed through a cell
having an electrolytic solution, the cations move
58. Mark the false statement? towards the cathode and anions towards the
(a) A salt bridge is used to eliminate liquid anode. If anode is pulled out from the solution
junction potential (a) the cations and anions will move towards
(b) The Gibbs free energy change, DG is related the cathode
with electromotive force E as DG = –nFE (b) the cations will continue to move towards
(c) Nernst equation for single electrode cathode and anions will stop moving
RT (c) both the cations and anions will stop moving
potential is E = E o - log a n+ (d) the cations and anions will start moving
nF M
(d) The efficiency of a hydrogen-oxygen fuel randomly.
cell is 23% 65. At 298K the standard free energy of formation of
H2O (l) is –237.20 kJ/mol while that of its
59. The electrode potential E 2+ of a zinc ionisation into H+ ion and hydroxyl ions is 80 kJ/
(Zn Zn)
electrode at 25°C with an aqueous solution of 0.1 mol, then the emf of the following cell at 298 K
M ZnSO4 is [ E° = – 0.76 V. Assume will be
(Zn 2+ Zn)
[Take Faraday constant F = 96500 C]
2.303RT
= 0.06 at 298 K]. H2 (g , 1 bar) | H+ (1M) | | OH– (1M) | O2(g, 1 bar)
F
(a) 0.40 V (b) 0.81 V
(a) + 0.73 (b) – 0.79 (c) – 0.82 (d) – 0.70
60. The reduction potential (in volt) of a (c) 1.23 V (d) – 0.40 V
hydrogen electrode set up with a 2 × 10–2 M 66. A concentration cell is a galvanic cell in which
aqueous solution of a weak mono basic acid
(Ka = 5 × 10–5) at one atmosphere and 25°C is
(a) + 0.09 (b) + 0.18 (c) – 0.09 (d) – 0.18
61. A solution of copper sulphate (CuSO 4 ) is (a) decrease in free energy in a spontaneous
electrolysed for 10 minutes with a current of 1.5 chemical process appears as electrical energy
amperes. The mass of copper deposited at the (b) decrease in free energy in a spontaneous
cathode (at. mass of Cu = 63u) is : physical process appears as electrical energy
(a) 0.3892g (b) 0.2938g (c) decrease in free energy in a spontaneous
(c) 0.2398g (d) 0.3928g physical or chemical process appears as
62. A battery is constructed of Cr and Na2Cr2O7. electrical energy
The unbalanced chemical equation when such a (d) a non-spontaneous physical or chemical
battery discharges is following: process produces electrical energy.
Electrochemistry 197
67. In the silver plating of copper, K[Ag(CN)2] is used

Molar conductivity
instead of AgNO3. The reason is
(a) a thin layer of Ag is formed on Cu I
(b) more voltage is required II
(c) Ag+ ions are completely removed from
III
solution
(d) less availability of Ag+ ions, as Cu cannot c
displace Ag from [Ag(CN)2]– ion
(a) I (NaCl), II (HCl), III, (NH4OH)
68. 1. 0 L each of a buffer containing 1 mole NH 3 and
(b) I (HCl), II (NaCl), III, (NH4OH)
1 mol of NH +4 were placed in the cathodic and (c) I (NH4OH), II (NaCl), III, (HCl)
anodic half-cells and 965 C of electricity was (d) I (NH4OH), II (HCl), III, (NaCl)
passed. If anodic and cathodic half cells reactions 74. HNO 3 (aq) is titrated with NaOH(aq)
involve oxidation and reduction of water only as
conductometrically, graphical representation of
2H 2 O ¾ ¾® 4 H + + O 2 - 4 e - ; the titration is :
2H 2 O + 2e - ¾¾
® H 2 + 2OH -
Then pH of

Conductance

Conductance
(a) cathodic solution will increase
(b) anodic solution will decrease (a) (b)
(c) both the solutions will remain practically
constant Vol. of NaOH Vol. of NaOH
(d) both the solutions will increase
69. The emf of a particular voltaic cell with the cell
Conductance

Conductance
reaction Hg 22 + + H 2 2Hg + 2H + is 0.65
V. The maximum electrical work of this cell when
0.5 g of H2 is consumed. (c) (d)

(a) – 3.12 × 10 4 J (b) –1.25 ´ 105 J Vol. of NaOH Vol. of NaOH

(c) 25.0 ´ 10 6 J (d) None of these 75. In the electrolysis of water, one faraday of
70. A cell reaction would be spontaneous if the cell
electrical energy would liberate
potential and DrG are respectively :
(a) positive and negative (a) one mole of oxygen
(b) negative, negative (b) one gram atom of oxygen
(c) zero, zero (c) 8 g oxygen
(d) positive, zero (d) 22.4 lit. of oxygen
71. The Nernst equation E = E° – RT/nF ln Q
indicates that the Q will be equal to equilibrium 76. Resistance of a conductivity cell filled with a
constant Kc when : solution of an electrolyte of concentration 0.1 M
(a) E = E° (b) RT / nF = 1 is 100 W. The conductivity of this solution is 1.29
(c) E = zero (d) E° = 1 S m–1. Resistance of the same cell when filled
72. Thermodynamic efficiency of a cell is given by : with 0.02 M of the same solution is 520 W. The
DH nFE -nFE molar conductivity of 0.02 M solution of
(a) (b) (c) (d) nFE° electrolyte will be
DG DG DH
73. A graph was plotted between molar conductivity (a) 1.24 × 10–4 S m2 mol–1
of various electrolytes (NaCl, HCl and NH4OH) (b) 12.4 × 10–4 S m2 mol–1
and c (in mol L-1 ) . Correct set is : (c) 124 × 10–4 S m2 mol–1
(d) 1240 × 10–4 S m2 mol–1
EBD_7587
198 Chemistry Objective MCQs

77. L ClCH 2COONa = 224 ohm -1cm 2g eq -1, Ag + /Ag = + 0.88V.


Based on the above potentials, strongest
L NaCl = 38.2 ohm -1cm 2g eq -1, oxidizing agent will be :
(a) Cu+ (b) Fe3+ (c) Ag+ (d) I2
L HCl = 203 ohm -1cm 2g eq -1,
84. Which of the following is a highly corrosive salt?
What is the value of L ClCH 2COOH (a) Hg2Cl2 (b) HgCl2
(a) 288.5 ohm–1cm2g eq–1 (c) FeCl2 (d) PbCl2
(b) 289.5 ohm–1cm2g eq–1 85. The Zn acts as sacrificial or cathodic protection
(c) 388.8 ohm–1cm2g eq–1 to prevent rusting of iron because :
(d) 59.5 ohm–1cm2g eq–1 (a) E°OP of Zn < E OP
°
of Fe
78. 1.08 g of pure silver was converted into silver
nitrate and its solution was taken in a beaker. It (b) E°OP of Zn > E OP
°
of Fe
was electrolysed using platinum cathode and
silver anode. 0.01 Faraday of electricity was (c) E°OP of Zn = E °OP of Fe
passed using 0.15 volt above the decomposition (d) Zn is cheaper than iron
potential of silver. The silver content of the beaker 86. Among the following cells:
after the above shall be (I) Leclanche cell
(a) 0 g (b) 0.108 g (II) Nickel-Cadmium cell
(c) 1.08 g (d) None of these (III) Lead storage battery
79. Specific conductance of 0.1 M sodium chloride (IV) Mercury cell
solution is 1.06 × 10–2 ohm–1 cm–1. Its molar primary cells are
conductance in ohm–1 cm2 mol–1 is (a) I and II (b) I and III
(a) 1.06 × 102 (b) 1.06 × 103 (c) II and III (d) I and IV
(c) 1.06 × 10 4 (d) 5.3 × 102 87. How long a current of 3 amperes has to be passed
80. In the electrolysis of CuCl2 solution, the mass of through a solution of AgNO3 to coat a metal
the cathode increased by 3.2 g. What occured at surface of 80 cm2 and 0.005 mm thick layer. Density
the copper anode? of Ag is 10.5 g cm–3
(a) 0.12 litre of Cl2 was liberated (a) 125.1 seconds (b) 12.5 seconds
(b) 0.56 litre of O2 was liberated (c) 155.2 seconds (d) 200 seconds
(c) 0.1 mol Cu2+ passed into the solution. 88. Al2O3 is reduced by electrolysis at low potentials
(d) 0.05 mol of Cu2+ passed into the solution. and high currents. If 4.0 × 104 amperes of current
81. Given is passed through molten Al2O3 for 6 hours, what
mass of aluminium is produced? (Assume 100%
E ° 2+ = 0.34V , E° 2+ = 0.15V current efficiency. At. mass of Al = 27 g mol–1)
Cu /Cu Cu /Cu
Standard electrode potential for the half cell (a) 8.1 × 104 g (b) 2.4 × 105 g
Cu+/Cu is
(c) 1.3 × 104 g (d) 9.0 × 103 g
(a) 0.38 V (b) 0.53 V (c) 0.19 V (d) 0.49 V
82. At 298 K, the standard reduction potentials are 89. The resistance of 1 N solution of acetic acid
is 250 ohm, when measured in a cell of cell
1.51 V for MnO4– | Mn2+, 1.36 V for Cl2 | Cl–, 1.07 constant 1.15 cm–1. The equivalent conductance
V for Br2 | Br –, and 0.54 V for I2 | I–. At pH = 3, (in ohm–1 cm2 equiv–1) of 1 N acetic acid will be
permanganate is expected to oxidize : (a) 4.6 (b) 9.2 (c) 18.4 (d) 0.023
æ RT ö 90. Specific conductance of 0.1 M HA is 3.75 × 10–4
ç = 0.059V ÷
è F ø ohm–1 cm–1. If l¥ ( HA) = 250 ohm -1 cm 2 mol -1 ,
(a) Cl–, Br– and I– (b) Br– and I– the dissociation constant Ka of HA is :
(c) Cl– and Br– (d) I– only
83. Electrode potentials (E°) are given below :
Cu+ / Cu = + 0.52 V,
Fe 3+ 2+
1 / Fe - = + 0.77 V, (a) 1.0 × 10–5 (b) 2.25 × 10–4
I2 (s) / I = +0.54 V, (c) 2.25 × 10–5 (d) 2.25 × 10–13
2
Electrochemistry 199
91. Equivalent conductance at infinite dilution, l° of Which one of the following is the strongest
NH4Cl, NaOH and NaCl are 128.0, 217.8 and 109.3 oxidising agent?
ohm–1 cm2 eq–1 respectively. The equivalent (a) Na+ (b) Li+ (c) Ba2+ (d) Mg2+
conductance of 0.01 N NH4OH is 9.30 ohm–1
96. The E º M 3+ / M 2 + values for Cr, Mn, Fe and Co
cm2 eq–1, then the degree of ionization of NH4OH
at this temperature would be are – 0.41, + 1.57, + 0.77 and + 1.97V respectively.
For which one of these metals the change in
(a) 0.04 (b) 0.1 (c) 0.39 (d) 0.62 oxidation state from +2 to +3 is easiest?
92. I2 (s) |I– (0.1 M) half cell is connected to a (a) Fe (b) Mn (c) Cr (d) Co
H+ (aq) | H2 (1 bar) | Pt half cell and e.m.f. is found
97. The solution of CuSO4 in which copper rod is
to be 0.7714 V. if E° - = 0.535 V , find the pH of immersed is diluted to 10 times. The reduction
I2 |I
H+ | H2 half-cell. electrode potential
(a) 1 (b) 3 (c) 5 (d) 7 (a) Increases by 30 mV
93. Consider the following standard electrode (b) Decreases by 30 mV
potentials and calculate the equilibrium constant (c) Increases by 59 mV
at 25° C for the indicated disproportion nation (d) Decreases by 59 mV
reaction : 98. For a given reaction: M(x+n) + ne– ¾¾ ® Mx+,

E red is known along with M (x + n) and Mx+ ion
® Mn(s) + 2Mn 3+ (aq )
3Mn 2+ (aq) ¾¾
concentrations. Then
Mn 3+ (aq) + e - ¾¾
® Mn 2+ (aq); E° = 1.51 V (a) n can be evaluated
(b) x can be evaluated
Mn 2+ (aq) + 2e - ¾¾
® Mn(s); E° = -1.185 V (c) (x + n) can be evaluated
(a) 1.2 ´ 10–43 (b) 2.4 ´ 10–73 (d) n, x, (x + n) can be evaluated
(c) 6.3 ´ 10 –92 (d) 1.5 ´ 10–62
99. The emf of the cell Pt Cl 2 (g) (P1 atm)/ Cl - (aq)
94. In a hydrogen-oxygen fuel cell, combustion of
hydrogen occurs to (1M)/ Cl 2 (g) (P2 atm) Pt will be positive when
(a) produce high purity water
(b) create potential difference between two
electrodes (a) P1 = P2 (b) P1 < P2
(c) generate heat (c) P1 > P2 (d) None of these
(d) remove adsorbed oxygen from elctrode 100. Equivalent conductivity can be expressed in terms
surfaces of specific conductance (k) and concentration (N)
95. The standard reduction potentials at 25°C of in gram equivalent per dm–3 as :
k ´ 1000
2+ +
Li+ / Li, Ba / Ba, Na / Na and Mg 2+ / Mg are (a) k × N (b)
N
– 3.03, – 2.73, – 2.71 and – 2.37 volt respectively. k´N
(c) (d) k × N × 1000
1000

Answer KEYs
1 (c) 11 (a) 21 (d) 31 (c) 41 (b) 51 (c) 61 (b) 71 (c) 81 (b) 91 (a)
2 (b) 12 (d) 22 (c) 32 (b) 42 (c) 52 (a) 62 (c) 72 (c) 82 (b) 92 (b)
3 (a) 13 (c) 23 (d) 33 (c) 43 (d) 53 (a) 63 (c) 73 (b) 83 (c) 93 (c)
4 (a) 14 (c) 24 (b) 34 (b) 44 (c) 54 (a) 64 (d) 74 (a) 84 (b) 94 (b)
5 (d) 15 (c) 25 (d) 35 (a) 45 (a) 55 (c) 65 (a) 75 (c) 85 (b) 95 (d)
6 (c) 16 (d) 26 (a) 36 (c) 46 (c) 56 (b) 66 (b) 76 (a) 86 (d) 96 (c)
7 (b) 17 (a) 27 (c) 37 (c) 47 (d) 57 (b) 67 (d) 77 (c) 87 (a) 97 (b)
8 (c) 18 (c) 28 (b) 38 (a) 48 (b) 58 (c) 68 (c) 78 (a) 88 (a) 98 (a)
9 (d) 19 (b) 29 (d) 39 (b) 49 (a) 59 (b) 69 (a) 79 (a) 89 (a) 99 (b)
10 (a) 20 (c) 30 (a) 40 (c) 50 (c) 60 (d) 70 (a) 80 (d) 90 (c) 100 (b)
EBD_7587
200 Chemistry Objective MCQs

1. (c) Cathode and anode are (–) ve terminals in 11. (a) Apply Nernst equation to the reaction
electrolytic and galvanic cell. Pb + Sn2+ ® Pb2+ + Sn
2. (b) Ag becomes oxidized and Br – becomes
reduced. 0.059 [Sn 2 + ]
Ecell = E° + log
ˆˆ† 2H + (aq ) + 2e -
2 [Pb 2 + ]
3. (a) A : H 2 (g ) ‡ˆˆ
[Sn 2 + ] 0.01 ´ 2
ˆˆ† Cu (s ) log = = 0.3 (Q Ecell = 0)
C : Cu (aq ) + 2e ‡ˆˆ
2+ - or
[Pb 2+
] 0.059
4. (a) Since oxidation potential of Zn is highest
hence strongest reducing agent. [Sn 2+ ]
or = antilog (0.3)
5. (d) The reduction potential of NO3- ion is more [Pb 2 + ]
than H+ ion. 12. (d) For a concentration cell having different
6. (c) Gold having higher E oRed and oxidises concentrations of ions.
0.0591 c
Fe ¾¾ ® Fe2+ . E <, log 1
7. (b) Given ESn 4+ Sn 2+ = + 0.15 V n c2
If all the concentrations are identical then
ECr3+ Cr = – 0.74 V obviously the cell voltage is zero. But as
the pH of 0.1 M HCl (strong acid) and pH
E ocell = E cathode
o o
– E anode of 0.1M CH3COOH is (weak acid) not same,
= 0.15 – (– 0.74) = + 0.89 V therefore the cell voltage is not zero.
13. (c) EMF of galvanic cell = 1.1 volt
8. (c) Given Fe3+/Fe2+ = + 0.77 V
If Eext < EMF then electron flows steadily
and I2/2I– = 0.536V from anode to cathode while If E ext > EMF
2 (e– + Fe3+ ¾¾ 2+
® Fe ) E° = 0.77 V then electron flows from cathode to anode
2I ¾¾
– ® I + 2e
2
– E° = – 0.536 V as polarity is changed.
2Fe3+ + 2I– ¾¾ ® 2Fe 2+ + I 14. (c) As the value of reduction potential
2
E° = E°ox + E°red decreases the reducing power increases i.e.
C < B < D < A
= 0.77 – 0.536 = 0.164 V
(0.85). (0.6). (–0.76) (– 1.2)
So, reaction will take place. 15. (c) According to Faraday’s first law of
0.591 electrolysis
9. (d) E red =E ored + log[M n+ ] E´i´t
n W=
Lower the concentration of Mn+, lower is 96500
the reduction potential. Where E = equivalent weight
Hence order of reduction potential is : mol. mass of metal (M)
=
Q> R>S >P oxidation state of metal ( x)
10. (a) 2Ag+ + H2 ¾¾ ® 2H+ + 2Ag Substituting the value in the formula
M i ´t
W= ´
0.0591 [H + ]2 x 96500
E = E° - log
2 PH 2 ´ [Ag+ ]2 M i ´t 10 ´ 2 ´ 60 ´ 60
or x = ´ = =3
0.0591 1 W 96500 96500 ´ 0.250
0.222 = 0.7995 - log
2 [Ag + ]2 é M ù
[Ag+] = 10–9.8 êë Given : no. of moles = W = 0.250 úû
Ksp = [Ag+] [Cl–] = (10–9.8) × (1) = 10–9.8 Hence oxidation state of metal is (+3)
Electrochemistry 201
16. (d) Oxidation potential of Co is more than Ag, 23. (d) L ¥m (NaBr) = L ¥m (NaCl) + L ¥m KBr - L m
¥
(KCl)
hence cell reaction will be
¥
Co + 2 Ag + ¾
¾® Co 2 + + 2 Ag lm (Na + ) + l¥ -
m (Br ) = 126.5 + 151.5 - 150

RT ln [Co 2+ ] , the lesser the l¥ + 2


m (Na ) = 50 S cm eq .
–1
o
Ecell = Ecell - 24. (b) Equivalent conductivity of BaCl2
nF [Ag + ]2
¥
2+ Lm (BaCl2 ) = l ¥ 2+ ¥ -
m (Ba ) + 2l m (Cl )
value of the factor [Co ] greater will be = 127 + 2 × 76 = 279 ohm cm2 eq–1.
–1
[ Ag + ]2 25. (d) 1 mole of e– = 1F = 96500 C
value of Ecell. 27g of Al is deposited by 3 × 96500 C
17. (a) Since concentration of ions is the same 5120 g of Al will be deposited by
o
hence Ecell = Ecell
3 ´ 96500 ´ 5120
= = 5.49 ´ 10 7 C
+ 27
18. (c) Cd(s) + 2 Ag (aq) ¾
¾®
26. (a) 2H+ + 2e– ¾ ¾ ↑ H2
Cd 2 + (aq) + Ag ( s ),
2
2+ EH (Eq. wt) = =1
° – 0.059 [Cd ] 2
Ecell = Ecell log
2 [Ag + ]2 22400
1g= < 11200 mL (STP)
º 0.0592 [Hg 22 + (aq)] 2
19. (b) Ecell = Ecell - log
2 +
[Ag (aq)] 2 96500≥112
Total charge passed = < 965
11200
0.0592 10 -1
= (0.80 - 0.785) - log = - 0.133V Q = It = 965
2 (10 -3 ) 2
965
hence backward reaction is spontaneous. I= < 1amp.
965
20. (c) E°cell = E°RP( RHS) - E °RP (LHS) 27. (c) Using Faraday’s second law of electrolysis,
= 1.61 – 1.51 Þ 0.10 V Weight of Cu deposited Equ. wt. of Cu
DG° = – nFE° Þ – 5 × 96500 × 0.10 J =
DG° = – 48.25 kJ mol–1 Weight of Ag deposited Equ. wt. of Ag
21. (d) Molarity = 0.01 M ; Resistance = 40 ohm; w Cu 63.5 1
Þ = ´
l ,1 0.18 2 108
Cell constant < 0.4cm .
A
63.5 ´ 18
Specific conductivity (k ) Þ wCu = = 0.0529 g.
2 ´108 ´ 100
cell constant 0.4 28. (b) Specific resistance = x
= = = 0.01 ohm -1 cm -1 \ Specific conductance (or conductivity)
resistance 40
1
1000k =k= .
Molar conductance ( Ù m ) = x
Molarity
k ´1000 1000
\ Leq = =
1000 ´ 0.01 N xN
= = 103 ohm -1 cm 2 mol -1
0.01 k ´1000 0.0110 ´ 1000
22. (c) On dilution number of ions decrease in unit 29. (d) Lm = =
M 0.05
volume hence specific conductance = 220 S cm2 mol– 1
decreases. But separation between ions also 30. (a) Difluoroacetic acid being strongest acid will
increase hence equivalent conductance furnish maximum number of ions showing
increases. highest electrical conductivity.
EBD_7587
202 Chemistry Objective MCQs

31. (c) E.wt ´ Q ; 1 1


m= Eq of Na+ = , Eq of K+ =
96500 2 2
m ´ 96500 22.2 ´ 96500 COONa
\ E.wt = = = 60.3 |
Q 2 ´ 5 ´ 60 ´ 60 \ l°eq
COOK
At wt. 177
Oxidation state = = =3 COO– 1 1
Eq. wt . 60.3
= l°eq | + l° Na + + l° K+
2 2
32. (b) Cell constant COO–
=k ´R 50 73
-1 -1 = 74 +
+ = 135.5 ohm–1 cm2 eq–1
= 0.212 ohm cm ´ 55 ohm 2 2
-1 38. (a) By Faraday's I st law of electrolysis,
= 11.66 cm
W Q
<
33. (c) Kohlrausch Law, Leq (NaF) + Leq (HCl) – E 96500 (where Q = it = charge of ion)
We know that no. of gram equivalent
Leq (NaCl) = Leq (HF)
W it 1 ´ 965 1
34. (b) Conductivity (X) = conductance (c) × cell = = = =
constant E 96500 96500 100
(where i = 1 A, t = 16 × 60 + 5 = 965 sec.)
X Since, we know that
\ Cell constant =
Y Normality
X 1
Conductivity of NaOH = .Z No. of gram equivalent 100 = 0.01 N
Y = =
Molar conductance of NaOH Volume (in litre) 1

; as [Cl - ] ¯, E ­
X 1000 XZ 4 0.0591 1
= .Z ´ = 10 39. (b) E = E° - log
Y 0.1 Y 2 [Cl- ]2

35. (a) Ar NO 2 + 6H + + 6e - ¾¾® ArNH 2 + 2H 2 O 40. (c) Corrosion of iron can be minimized by
forming an impermeable barrier at its surface.
E.wt ´ Q M 2 ´ 96500 50 41. (b) During charging, the lead storage battery
W= ;20.50= ´ ´
96500 6 96500 100 behaves like an electrolytic cell. So, at anode
\ M = 123.0 g the reaction is
600 ´ 1 ® PbO 2 + 4H + + SO 24 - + 2e -
PbSO 4 + 2H 2 O ¾¾
36. (c) Moles of NaOH formed = = 0.6 42. (c) 2NH 4 Cl + Zn ® 2NH 3 + ZnCl 2 + H 2 ­ .
100
or geq of NaOH = 0.6 geq of Cu deposited 43. (d) Reducing character decreases down the
31.8 series. Hence the correct order is
= = 1.0 Al < Fe2+ < Br–
63.6 / 2
44. (c) Cu ¾¾ ® Cu++ + 2e–
0.6 ´ 100
\ Current efficiency = = 60 % i.e, to deposit 1 mole of Cu at cathode from
1 Cu2+ SO42– solution = 2 moles of electrons
37. (c) Total charge = 2 are required
Number of equivalent of ion i.e, To deposit
Charge on the ion 6.35 2 1
= 6.35g = ´2 = = moles
Total charge 63.5 10 5
COO– NA
2 Thus total no. of electrons required =
\ Eq of | = =1 5
COO – 2
Electrochemistry 203
45. (a) Lower the value of reduction potential DG° = – nE°F
higher will be reducing power hence the
correct order will be – 2×0.46×96500
= kJ ; – 89 kJ
Mn2+ < Cl– < Cr3+ < Cr 1000
46. (c) Eoxidation = 0.059 pH = 0.059 × 10 = 0.59 V
47. (d) Standard Gibbs free energy is given as 54. (a) Zn(s)+2H + (aq) Zn 2+ (aq)+H 2 (g)
DG° = – nE° F
If E°cell < 0 i.e. – ve 0.059 [Zn 2+ ][H 2 ]
E cell = E°cell – log
DG° > 0 2 [H + ]2
Further DG° = – RT l n Keq Addition of H2SO4 will increase [H+]and
48. (b) The oxidation potential Ecell will also increase and the equilibrium
1 will shift towards RHS
µ and reduction
Concentration of ions 55. (c) Using the relation,
potential µ concentration of ions. The cell 2.303RT 0.0591
voltage can be increased by decreasing the E°cell = log Keq = log Keq
nF n
concentration of ions around anode or by
increasing the concentration of ions around 0.0591
\ 0.295 V = log Keq
cathode 2
1
49. (a) Oxidising tendency µ 2 ´ 0.295
Electrode potential or log Keq = = 10
0.0591
TX ¾¾
® No reaction
or Keq = 1 × 1010
TY ¾¾
® X, Z
56. (b) DG = –nEF
TZ ¾¾
®X For 1 mole of Al, n = 3
Þ order of electrode potential is
TY < TZ < TX 4 4
\ for mole of Al, n = 3 ´ = 4
Þ Order of oxidation of the anion is 3 3
Y – > Z– > X– According to question,
50. (c) Oxidation potential of M is more than Ni -827 ´ 1000 = -4 ´ E ´ 96500
and less than Mn. Hence reducing power 827 ´ 1000
Mn > M > Ni E= = 2.14V
4 ´ 96500
51. (c) E° °
= – 0.74 V, E Co 2+ /Co = – 0.28 V \ minimum e.m.f. required = 2.14 V
C r 3+ /C r
57. (b) Cell reaction is, Zn + Cu 2 + ® Zn 2 + + Cu
The given cell reaction is
2+
Cr | Cr 3+ (1.0 M ) || Co 2 + (1.0 M) | Co ° - RT ln [Zn ]
Ecell = Ecell
nF [Cu 2+ ]
\ Cr is anode and Co is cathode
E°cell =E°C –E°A = – 0.28 – (– 0.74) é ( Zn 2 + ) ù
= – 0.28 + 0.74 = + 0.46 V Greater the factor ê 2+
ú , less is the
ëê (Cu ) ûú
52. (a) - ; E° -
= 0.94 V (I)
OCl ¾
¾® Cl EMF
1 Hence E1 > E2
Cl - ¾
¾® Cl 2 + e - ; E° = 1.36 V (II)
2 58. (c) Correct Nernst equation is
1 2.303 RT
Add (I)+ (II) OCl - ¾¾® Cl 2 ; E° = 0.94 –1.36 E = Eo + log a n+ .
2 nF M
=–0.42V 59. (b) For Zn ¾® Zn
2+

53. ® Cu 2+ ( aq ) + 2Ag(s)
(a) Cu + 2Ag + ( aq ) ¾¾
E = E° -
2.303RT
log
[ Zn ]
Zn 2+ / Zn Zn 2 + / Zn nF é Zn 2 + ù
Here, n = 2 , E° = + 0.46 V ë û
cell
EBD_7587
204 Chemistry Objective MCQs

0.06 1 1
= -0.76 - log = -0.76 - 0.03 ® H 2 O(l) ;
H 2 (g) + O 2 (g) ¾¾
2 [ 0.1] 2

E = -0.79V DG°f = -237.2 kJ / mole


Zn 2+ / Zn
® H + (aq) + OH - (aq);
H 2 O(l) ¾¾
60. (d) 2H + + 2e - ¾¾® H 2
DG ° = 80 kJ / mol
Ecell = 0.06 log [H+] = 0.06 ´ log K a C
Hence for cell reaction
= 0.06 × log 10–3 = –0.18 V DG° = -237.2 + (2 ´ 80) = -77.20 kJ / mol
61. (b) W = Zit
DG ° 77200
where Z = Electrochemical equivalent \ E° = - = = -0.40V
63 nF 2 ´ 96500
Eq. wt. of copper = = 31.5 66. (b) In concentration cell the spontaneous
2 process is physical in nature involving
31.5
Z= transfer of matter from higher concentration
96500 to lower concentration in indirect manner.
31.5 67. (d) In the silver plating of copper, K[Ag(CN)2]
W = Zit = ´ 1.5 ´ 10 ´ 60 = 0.2938g
96500 is used instead of AgNO3. Copper being
62. (c) Reduction half reaction : more electropositive readily precipitate
silver from their salt solution
Cr2 O 72– + 6e – + 14H + ¾¾ ® 2Cr 3+ + 7H 2 O
Cu ∗ 2AgNO 3 ¾ ¾
↑ Cu(NO 3 ) 2 ∗ Ag
Oxidation half reaction :
® Cr 3+ + 3e –
Cr ¾¾
whereas in K[Ag(CN)2] solution a complex
Overall reaction : anion [Ag(CN)2]– is formed and hence Ag+
are less available in the solution and
Cr2 O 72– + Cr + 14H + + 3e – ¾¾
® 3Cr 3+ + 7H 2O therefore copper cannot displace Ag from
3F of electricity = 3 moles of Cr3+ its complex ion.
3 68. (c) Due to buffer action the pH will remain
1F of electricity = moles of Cr3+
3 practically constant.
63. (c) More the reduction potential, more is the
deposition of metals at cathode. Cation 69. (a) Wmax = – n.FE;
having E 0 value less than – 0.83V Wmax = – 2 × 96500 × 0.65 = – 1.25 ´ 10 5 J
(reduction potential of H2O) will not
deposit from aqueous solution. Hence 0.5g H 2 = 0.25 mole
correct order of A deposition of the metal
at the cathode is Hence Wmax
Ag > Hg > Cu
= – 1.25 ´ 10 5 ´ 0.25 = –3.12 ´ 10 4 J
64. (d) On pulling out one of the electrodes the
electric field applied to the solution 70. (a) For spontaneous
disappears and hence the ions start moving DrG° = –nFE° < 0
randomly. \ E° > 0
71. (c) When Q = KC, E = 0
65. (a) Cell reaction
72. (c) Thermodynamic efficiency of a cell is given
cathode : by the ratio of Gibbs function change to
1 enthalpy change in the overall cell reaction.
H 2O(l) + O 2 (g) + 2e - ¾¾
® 2OH - (aq) 73. (b) Ionic molar conductivity of H+ is very high
2
and NH4OH is a weak electrolyte.
® 2H + (aq) + 2e -
anode : H 2 (g) ¾¾ 74. (a) Molar conductivity of H+ and OH– are very
high as compare to other ions.
––––––––––––––––––––––––––––––––––
Initially conductance of solution sharply
1 decreases due to consumption of free H+.
® 2H + (aq) + 2OH - (aq)
H 2O(l) + O2 (g) + H2 (g) ¾¾
2 After complete neutralization further
Also we have slightly increases due to presence of OH–.
Electrochemistry 205
75. (c) According to the definition 1 F or 96500 C is 81. (b) Cu 2 + + e – ¾¾
® Cu + ;
the charge carried by 1 mol of electrons E1o = 0.15V; DG1o = – n1E1o F
when water is electrolysed
Cu 2 + + 2e ¾¾® Cu ;
2H2O ¾¾
® 4H+ + O2 + 4e– E2o = 0.34V; DG2o = – n2 E2o F
So, 4 Faraday of electricity liberate = 32 g of O2. On subracting eq.(i) from eq. (ii) we get
Thus 1 Faraday of electricity liberate Cu + + e – ¾¾
® Cu ; DG o = DG2o – DG1o
32
= g of O2 = 8 g of O2 - nE o F = -(n2 E o F - n1E1o F )
4
l n2 E o2 F - n1E1o F
76. (a) Conductivity (k) = Eo =
R.a. nF
l æ lö 2 ´ 0.34 - 0.15
k= ; cell constant. ç ÷ = 1.29 ´ 100 = 129
R.a è aø = = 0.53V
1
Again conductivity of 0.02M solution
1 82. (b) MnO4– + 8H+ + 5e– ¾¾
® Mn2+ + 4H2O
k= ´ 129
520
0.059 [Mn 2+ ]
E=1.51– log
k ´ 1000 129 1000 5 [MnO4– ][H + ]8
Lm = = ´
M 520 0.02
Taking Mn2+ and MnO4– in standard
= 1.24 ´ 10 -4 S m 2 mol-1
state i.e. 1 M,
77. (c) ClCH 2COONa + HCl ® ClCH 2COOH + NaCl
ΚClCH2COONa ∗ ΚHCl < ΚClCH 2COOH ∗ Κ NaCl 0.059 1
E=1.51 – ×8log
5 [H + ]
224 ∗ 203 < ΚClCH 2COOH ∗ 38.2

ΚClCH 2COOH < 427 , 38.2 0.059


= 1.51 – ´ 8 ´ 3 = 1.2268 V
5
< 388.8 ohm,1cm 2 g eq,1
Hence at this pH, MnO4– will oxidise only
78. (a) Ag + + e - ¾
¾® Ag Br– and I– as SRP of Cl2/Cl– is 1.36 V which
1F 108 g
is greater than that for MnO4– /Mn2+.
1 F = 1 mole of electrons = 96500 C
83. (c) Higher the value of reduction potential
0.01F = 1.08 g Ag; Ag left = 1.08 – 1.08 = 0 g stronger will be the oxidising hence based
on the given values Ag+ will be strongest
k ´1000 1.06 ´10 -2 ´1000
79. (a) Lm = = oxidizing agent.
M 0.1
84. (b) E° > E° > E° > E° .
2 Hg 2+ |Hg Hg 22+ |Hg Fe2+ |Fe Pb2+ |Pb
= 1.06 ´ 10
80. (d) The amount. of copper deposited at cathode 85. (b) E°OP of Zn > E OP
°
of Fe.
3.2 86. (d) Primary cells are those cells, in which the
by reduction of Cu 2 + ions is = 0.05 moles.
63 reaction occurs only once and after use over
a period of time, it becomes dead and cannot
The same amount 0.05 mole of Cu 2 + must be reused again. e.g., Leclanche cell and
pass into solution from anode by oxidation mercury cell.
EBD_7587
206 Chemistry Objective MCQs
87. (a) Volume = Area × thickness
93. (c) 2Mn 2+ ¾¾
® 2Mn 3+ + 2e - , DG1°
Mass = Volume × density
\ Mass of Ag to be deposited Mn 2+ + 2e - ® Mn, DG°2
80 ´ 0.005 3Mn 2+ (aq ) ® Mn (s ) + 2Mn 3+ (aq )
= ´ 10.5 = 0.42 g
10 – 2 × F × E°3 = – 2 ×F × [–1.51] – 2 × F × (–1.185)

Amount deposited =
i×t× E.wt E3° = -2.695
96500
0.0591
108 ´ 3 ´ t E °3 = + log K eq ; K eq ; 6.3 ´ 10-92 .
\ 0.42 = \ t = 125.1 seconds 2
98500 94. (b) In H 2 - O 2 fuel cell, the combustion of H2
88. (a) Q Q = i × t occurs to create potential difference
\ Q = 4.0 × 104 × 6 × 60 × 60 C = 8.64 × 10 8 C between the two electrodes
Now since 96500 C liberates 9 g of Al 95. (d) Higher the reduction potential, stronger is
8.64 × 108 C liberates the oxidising agent.
9 96. (c) The given values show that Cr has highest
´ 8.64 ´ 108 g Al = 8.1 × 104 g of Al negative value of E↓ 3∗ 2∗ i.e., Cr is the
96500 M /M
strongest reducing agent among given
Cell constant 1.15
89. (a) k= = metals therefore its oxidation will be easiest.
R 250
1.15 1000 0.059
L eq = ´ = 4.6 97. (b) (E ) = E0 +
Cu 2 + | Cu 1 Cu 2 + | Cu 2
250 1
1000k 1000 ´ 3.75 ´10 -4 log[Cu 2+ ]1
90. (c) lm = = = 3.75;
0.1 0.1
[Cu2+(aq) + 2e– ¾¾
® Cu(s)] and
lm 3.75
a= = = 1.5 ´ 10-2 ; 0 0.059
l¥ 250 (E ) =E + log [Cu 2 + ]
m Cu 2 + | Cu 2 Cu 2 + | Cu 2 2
Ka = Ca2 = 0.1 ×(1.5 ×10 –2)2 = 2.25 ×10–5 æC ö
0.059
¥ ¥ ¥ Þ DE Cu 2+ | Cu = – log ç 2 ÷
91. (a) Λeq (NH4 OH) = Λeq (NH4 Cl) + Λeq (NaOH) 2 è C1 ø
0.059
- Λ¥
eq (NaCl) =- log10 = -0.030 V = -30 mV
2
= 129.8 + 217.8 - 109.3 = 238.3 ohm -1cm 2 eq -1 0.059 [M (x + n) ]
98. (a) E red = E°red + log
L eq 9.30
n [M x + ]
a= = = 0.04 99. (b) The cell reaction is
¥ 238.3
L eq
92. (b) The cell reaction is Cl 2 (g) ( P2 atm) ¾
¾® Cl 2 (aq) (P1atm)

H 2 ( g ) + I2 (s ) ƒ 2H + (aq ) + 2I - (aq ) E cell = –


0.0592 P
log 1 =
0.0592 P
log 2
2 P2 2 p1
0.0591 [H + ]2 [I - ]2
0.7714 = 0.535 - log E cell will be positive when P2 > P1 .
2 PH2
pH = 3 k ´ 1000
100. (b) L eq =
N
@unacademyplusdiscounts_link

https://telegram.me/Unacademyplusdiscounts_link

https://telegram.me/Unacademyplusdiscounts_link

https://telegram.me/Unacademyplusdiscounts_link

Join Us Now For all study Materials for free


18 Chemical Kinetics
1. For the reaction 4. For the reaction H2(g) + Br2 (g) ® 2HBr (g), the
N 2O5 (g) ¾¾
® 2NO2 (g) + 1/2 O 2(g) the value experimental data suggest, rate = k[H2][Br2]1/2.
The molecularity and order of the reaction are
of rate of disappearance of N 2 O5 is given as respectively
6.25 × 10–3 mol L–1s–1. The rate of formation of
3 3 3 1
NO 2 and O2 is given respectively as : (a) 2, (b) , (c) 1, 1 (d) 1,
2 2 2 2
(a) 6.25 × 10–3 mol L–1s–1 and
5. For a chemical reaction t1/2 is 2.5 hours at room
6.25 × 10–3 mol L–1s–1 temperature. How much of the reactant will be
(b) 1.25 × 10–2 mol L–1s–1 and left after 7.5 hours if initial weight of reactant was
3.125 × 10–3 mol L–1s–1 160 g?
(c) 6.25 × 10–3 mol L–1s–1 and
3.125 × 10–3 mol L–1s–1 (a) 10 g (b) 40 g (c) 80 g (d) 20 g
(d) 1.25 × 10–2 mol L–1s–1 and 6. The rate law for a reaction between the
6.25 × 10–3 mol L–1s–1 substances A and B is given by rate = k[A]n[B]m.
2. The reactions rate N 2 (g) + 3H 2 (g) ® 2 NH3 (g) On doubling the concentration of A and halving
the concentration of B, the ratio of the new rate
was measured d[ NH 3 ] = 2 ´10 - 4 mol sec–1. to the earlier rate of the reaction will be as
dt
The rates of reactions expressed in terms of N2
(a) (m + n) (b) (n – m)
and H2 are
Rate in terms of N2. Rate in terms of H2 1
(mol L–1sec–1 ) (mol L–1sec–1 ) (c) 2(n + m) (d) ( m+ n)
–4 2
(a) 2×10 2×10–4 7. The order of a reaction, with respect to one of
(b) 3×10–4 1×10–4 the reacting component Y, is zero. It implies that:
(c) 1×10 –4 3×10–4 (a) the reaction is going on at a constant rate
(d) 2×10 –1 2×10–3 (b) the rate of reaction does not vary with
3. For the reaction A + B ¾¾® C + D. The variation temperature
of the concentration of the products is given by (c) the reaction rate is independent of the
the curve concentration of Y
(d) the rate of formation of the activated
Y complex is zero
Z 8. For a first order reaction, a plot of log (a – x)
against time is a straight line with a negative slope
Conc equal to
W -k
(a) (b) – 2.303 k
2.303
X
Time 2.303 Ea
(c) (d) -
(a) Z (b) Y (c) W (d) X k 2.303 R
EBD_7587
208 Chemistry Objective MCQs
9. A following mechanism has been proposed for a 16. The activation energy of the reaction,
reaction A + B ® C + D + 38 kcal is 20 kcal. What would
2A + B ® D + E be the activation energy of the following reaction.
A + B ® C + D (slow) C + D ®A+ B
A + C ® E (fast) (a) 20 kcal (b) –20 kcal
(c) 18 kcal (d) 58 kcal
The rate law expression for the reaction is:
17. The half-life of a reaction is inversely proportional
(a) r = k [A]2 [B] (b) r = k [A] [B]
to the square of the initial concentration of the
(c) r = k [A]2 (d) r = k [A] [C] reactant. Then the order of the reaction is
10. The rate law for the reaction below is given by (a) 0 (b) 1 (c) 2 (d) 3
the expression k [A][B]
18. For a reaction A + 2B ® C, the amount of C
A + B ® Product formed by starting the reaction with 5 moles of
If the concentration of B is increased from 0.1 to A and 8 moles of B is
0.3 mole, keeping the value of A at 0.1 mole, the (a) 5 moles (b) 8 moles
rate constant will be: (c) 16 moles (d) 4 moles
(a) 3k (b) 9k (c) k/3 (d) k 19. The rate law for the reaction 2X + Y ® Z is
11. The half life of a radioactive element is 20 min. Rate = k[X][Y]. The correct statement with
The time interval between the stages of its 33% regard to this relation is
and 67% decay is
(a) the unit of k is s–1
(a) 40 min (b) 20 min (c) 30 min (d) 25 min
(b) the rate of the reaction is independent of
12. Which of the following reactions is not of the
[X] and [Y]
first order?
(c) for this reaction t1/2 is independent of
(a) Inversion of sucrose in the presence of acid initial concentrations of reactant
(b) Acid-catalyzed hydrolysis of ethyl acetate (d) the rate of formation of Z is half of the rate
(c) Hydrolysis of tertiary butyl halide using of disappearance of X
alkali 20. In a reaction, 2A ® products, the concentration
(d) Oxidation of I– ion by S2O82– ion of A decreases from 0.50 M to 0.38 M in
13. Rate constant k = 1.2 × 103 mol– 1 L s– 1 and 10 min. What is the rate of reaction (in M s–1)
Ea = 2.0 × 102 kJ mol– 1. When T ® ¥: during this interval?
(a) A = 2.0 × 102 kJ mol– 1 (a) 0.012 (b) 0.024
(c) 2 × 10 –3 (d) 2 × 10–4
(b) A = 1.2 × 103 mol– 1 L s– 1
(c) A = 1.2 × 103 mol L– 1 s– 1 21. A chemical reaction is catalyzed by a catalyst X.
Hence X
(d) A = 2.4 × 103 kJ mol– 1 s– 1
(a) reduces enthalpy of the reaction
14. A catalyst lowers the activation enegy of a (b) decreases rate constant of the reaction
reaction from 20 kJ mol– 1 to 10 kJ mol– 1. The (c) increases activation energy of the reaction
temperature at which the uncatalyzed reaction (d) does not affect equilibrium constant of the
will have the same rate as that of the catalyzed at reaction
27 ° C is
22. In respect of the equation k = Ae- Ea / RT in
(a) – 123 °C (b) 327 °C chemical kinetics, which one of the following
(c) 32.7 °C (d) + 23 °C statements is correct ?
15. For a reaction, the rate constant is expressed as (a) A is adsorption factor
k = Ae–40000/T. The energy of the activation is (b) Ea is energy of activation
(a) 40000 cal (b) 88000 cal (c) R is Rydberg’s constant
(c) 80000 cal (d) 8000 cal (d) k is equilibrium constant
Chemical Kinetics 209
23. The minimum energy a molecule should possess 29. Consider the two hypothetical reactions given
in order to enter into a fruitful collision is known below :
as I a A ® Products, k = x mol–1 L min–1
(a) reaction energy (b) collision energy II b B ® Products, k = y min–1
(c) activation energy (d) threshold energy The half-lives of both the reactions are the same,
24. For a reaction, activation energy (Ea) = 0 and rate equal to 1 hr when molar concentration of the
constant reactant is 1.0 M in each case. If these reactions
( k ) = 3.2 ´106 s -1 at 300 K. What is the value of are started at the same time taking 1M of the
the rate constant at 310 K reactant in each case, the ratio [A]/[B] after 3 hr
will be :
(a) 3.2 ´ 10-12 s -1 (b) 3.2 ´ 106 s -1 (a) 0.5 (b) 4 (c) 1 (d) 2
(c) 6.4 ´ 1012 s -1 (d) 6.4 ´ 106 s -1 30. Consider the consecutive reactions :
1
25. In the Arrhenius plot of ln k Vs , a linear plot
T k = 2´10-5 s -1 k =8´10-6 s -1 k = 3´10-3 s -1
is obtained with a slope of –2 × 104 K. The energy A ¾¾¾¾¾¾
® B ¾¾¾¾¾¾
®.C ¾¾¾¾¾¾
®D
of activation of the reaction (in kJ mole–1) is (R k = 2 ´10 -5 s -1 k = 8´10 -6 s - 1 k = 3´10 -3 s -1
A ¾¾ ¾¾ ¾ ¾
® B ¾¾ ¾ ¾ ¾¾
® C ¾¾¾ ¾ ¾¾
®D
value is 8.3 J K–1 mol–1) The rate determining step of the reaction is :
(a) A ® B (b) C ® D
(a) 83 (b) 166 (c) 249 (d) 332 (c) B ® C (d) A ® D
26. The rate of reaction is doubled for every 10°C 31. A + 2B ® C, the rate equation for this reaction
rise in temperature. The increase in reaction rate is given as Rate = K[A][B].
as a result of temperature rise from 10°C to 100°C If the concentration of A is kept the same but
is that of B is doubled what will happen to the
rate itself ?
(a) 112 (b) 512 (c) 400 (d) 614
(a) halved (b) the same
27. Consider the reaction A ® 2 B + C, DH = – 15 (c) doubled (d) quadrupled
kcal. The energy of activation of backward 32. In the reaction of formation of sulphur trioxide
reaction is 20 kcal mol–1. In presence of catalyst
by contact process 2SO2 + O2 ƒ 2SO3 the rate
the energy of activation of forward reaction is 3
kcal mol–1. At 400 K the catalyst causes the rate of reaction was measured as
of the reaction to increase by the number of times d [O2 ]
= -2.5 ´10-4 mol L-1s -1 . The rate of
equal to dt
reaction is terms of [SO2] in mol L–1 s–1 will be:
(a) – 1.25 × 10–4 (b) – 2.50 × 10–4
(a) e3.5 (b) e2.5 –4
(c) – 3.75 × 10 (d) – 5.00 × 10–4
(c) e –2.5 (d) e 2.303
33. For the reaction, 2N2O5 ® 4NO2 + O2, the rate
28. A reaction takes place in various steps. The rate equation can be expressed in two ways
constant for first, second, third and fifth steps
d [ N 2O5 ] d [ NO 2 ]
are k1, k2, k3 and k5 respectively. The overall rate - = k [N 2 O5 ] and + = k ¢ [ N 2 O5 ]
dt dt
1/ 2
k 2 æ k1 ö k and k¢ are related as:
constant is given by k = ç ÷
k3 è k5 ø (a) k = k¢ (b) 2k = k¢
If activation energy are 40, 60, 50 and 10 kJ/mol (c) k = 2k¢ (d) k = 4k¢
respectively, the overall energy of activation (kJ/ 34. The reaction 2N2O5 2N2O4 + O2 is
mol) is : (a) bimolecular and of second order
(b) unimolecular and of first order
(a) 10 (b) 20 (c) bimolecular and of first order
(c) 25 (d) none of these (d) bimolecular and of zero order
EBD_7587
210 Chemistry Objective MCQs
35. For the reaction system : 41. The reaction L ¾¾® M is started with 10.0 g of
2NO (g) + O2 (g) ® 2 NO2 (g) volume is L. After 30 and 90 minutes 5.0 g and 1.25 g of L
suddenly reduced to half its value by increasing respectively are left. The order of the reaction is
the pressure on it. If the reaction is of first order (a) 0 (b) 1 (c) 2 (d) 3
with respect to O2 and second order with respect 42. A first order reaction is half-completed in
to NO, the rate of reaction will 45 minutes. How long does it need for 99.9% of
(a) diminish to one-eighth of its initial value the reaction to be completed?
(b) increase to eight times of its initial value (a) 20 hours (b) 10 hours
(c) increase to four times of its initial value 1
(d) diminish to one-fourth of its initial value (c) 7 hours (d) 5 hours
2
36. Units of rate constant of first and zero order 43. A reaction which is of first order w.r.t. reactant A,
reactions in terms of molarity M unit are has a rate constant 6 min –1. If we start with [A] =
respectively 0.5 mol L–1, when would [A] reach the value of
(a) sec–1, M.sec–1 (b) sec–1, M 0.05 mol L–1
–1
(c) M.sec , sec –1 (d) M, sec–1 (a) 0.384 min (b) 0.15 min
37. The plot of concentration of the reactant Vs time (c) 3 min (d) 3.84 min
for a reaction is a straight line with a negative 44. The time taken for 90% of a first order reaction to
slope. The reaction follows a rate equation of complete is approximately
(a) 1.1 times that of half-life
(b) 2.2 times that of half-life
(a) zero order (b) first order
(c) 3.3 times that of half-life
(c) second order (d) third order
38. The differential rate law for the reaction (d) 4.4 times that of half-life
45. The rate constant of a reaction is 0.0693 min– 1.
H2 (g) + I2 (g) ® 2HI (g) is
Starting with 10 mol, the rate of the reaction after
d[ H 2 ] d[ I 2 ] d[ HI ] 10 min is
(a) - =- =-
dt dt dt (a) 0.0693 mol min– 1
d[H2 ] d[I2 ] 1 d[HI] (b) 0.0693 × 2 mol min– 1
(b) = = (c) 0.0693 × 5 mol min– 1
dt dt 2 dt
(d) 0.0693 × (5)2 mol min– 1
1 d[H 2 ] 1 d[I 2 ] d[HI]
(c) = =- 46. The plot that represents the zero order reaction
2 dt 2 dt dt is :
d[H 2 ] d[I ] d[HI]
(d) -2 = -2 2 =
dt dt dt [R] [R]
(a) (b)
39. Which of the following will react at the highest
t t
rate?

[R]
(a) 1 mole of A and 1 mole of B in a 1-L vessel (c) (d) [R]
(b) 2 mole of A and 2 mole of B in a 2-L vessel t t
(c) 3 mole of A and 3 mole of B in a 3-L vessel 47. Which of the following reactions is not of the
(d) All would react at the same rate first order?
40. In the reaction, A + 2B ¾¾ ® 6C + 2D, If the initial
d[A] (a) Inversion of sucrose in the presence of acid
rate - at t = 0 is 2.6 × 10–2 M sec–1, what
dt (b) Acid-catalyzed hydrolysis of ethyl acetate
d [B]
will be the value of at t = 0? (c) Hydrolysis of tertiary butyl halide using
dt alkali
(a) 8.5 × 10 M sec (b) 2.5 × 10–2 M sec–1
–2 –1
(d) Oxidation of I– ion by S2O82– ion
(c) 5.2 × 10–2 M sec–1 (d) 7.5 × 10–2 M sec–1
Chemical Kinetics 211
48. A first order reaction is 50% completed in 20 (a) log k versus log T will give a straight line
minutes at 27°C and in 5 minutes at 47°C. The with a slope as –25000
energy of activation of the reaction is : (b) log k versus T will give a straight line with
slope as 25000
(c) log k versus 1/T will give a straight line with
(a) 43.85 kJ/mol (b) 55.14 kJ/mol slope as –25000/R
(c) 11.97 kJ/mol (d) 6.65 kJ/mol (d) log k versus 1/T will give a straight line
49. For the first order reaction A ® B + C is carried 55. The velocity of a reaction is doubled for every
out at 27°C. If 3.8 × 10–16 % of the reactant 10°C rise in temp. If the temp. is raised to 50°C
molecules exists in the activated state, the Ea from 0 °C the reaction velocity increases by about
(activation energy) of the reaction is: (a) 12 times (b) 16 times
(a) 12 kJ/mol (b) 831.4 kJ/mol (c) 32 times (d) 50 times
(c) 100 kJ/mol (d) 88.57 kJ/mol 56. Which of the following statements is incorrect?
50. A catalyst lowers the activation energy of a (a) Activation energy for the forward reaction
certain reaction from 83.314 to 75 kJ mol–1 at is equals to activation energy for the reverse
500 K. What will be the rate of reaction as reaction
compared to uncatalysed reaction? Assume (b) For a reversible reaction, an increase in
other things being equal. temperature increases the reaction rate for
(a) Double (b) 28 times both the forward and the backward reaction
(c) 7.38 times (d) 7.38 × 103 times (c) The larger the initial reactant concentration
51. Rate of a reaction can be expressed by Arrhenius for a second order reaction, the shorter is
equation as: k = Ae - Ea / RT its half-life.
(d) When Dt is infinitesimally small, the average
In this equation, Ea represents
rate equals the instantaneous rate
(a) the total energy of the reacting molecules
57. Consider the following statements:
at a temperature, T
(b) the fraction of molecules with energy greater
than the activation energy of the reaction
(c) the energy below which all the colliding I. Increase in concentration of reactant
molecules will react increases the rate of a zero order reaction.
(d) the energy below which colliding molecules
II. Rate constant k is equal to collision
will not react
frequency A if Ea = 0.
52. The reason for almost doubling the rate of
III. Rate constant k is equal to collision
reaction on increasing the temperature of the
frequency A if Ea = ¥.
reaction system by 10°C is
(a) The value of threshold energy increases IV. ln k Vs T is a straight line.
(b) Collision frequency increases V. In k Vs 1/T is a straight line.
(c) The fraction of the molecule having energy Correct statements are
equal to threshold energy or more increases (a) I and IV (b) II and V
(d) Activation energy decreases (c) III and IV (d) II and III
53. In Arrhenius plot, intercept is equal to 58. The time required for 10% completion of a first
-Ea order reaction at 298 K is equal to that required
(a) (b) ln A for its 25% completion at 308 K. If the pre-
R
(c) ln K (d) log10A exponential factor for the reaction is
54. A reaction rate constant is given by 3.56 × 109 s–1, the rate constant at 318 K is:

(a) 18.39 kcal mol–1 (b) 20 kcal mol–1


k = 1.2 ´ 1014 e -25000 / RTsec -1 . It means (c) 16 kcal mol–1 (d) 21.5 kcal mol–1
EBD_7587
212 Chemistry Objective MCQs
59. For the reaction, 3A + 2B ® C + D, the differential 65. A reaction proceeds by first order, 75% of this
rate law can be written as: reaction was completed in 32 min. The time

1 d [A ] d [C] required for 50% completion is


= k [A ] [B]
n m
(a) =
3 dt dt
(a) 8 min (b) 16 min
d [A ] d [C ]
= k [A ] [B]
n m (c) 20 min (d) 24 min
(b) - =
dt dt
66. Consider the reaction : N2 (g) + 3H2 (g) ® 2 NH3 (g)
1 d [A ] d [C]
= k [A ] [B]
n m
(c) + =- d[ NH 3 ]
3 dt dt The equality relationship between and
dt
d[H 2 ]
1 d [A ] d [C] -
= k [A ] [B] is
n m
(d) - = dt
3 dt dt
60. The instantaneous rate of disappearance d[ NH 3 ] 2 d[H 2 ]
(a) + =-
of MnO4– ion in the following reaction is dt 3 dt
4.56 × 10–3 Ms–1 2MnO4– + 10I– + 16H+ ® d[ NH 3 ] 3 d[H 2 ]
2Mn2+ + 5I2 + 8H2O (b) + =-
dt 2 dt
The rate of appearance I2 is :
d[ NH 3 ] d[H 2 ]
(a) 4.56 × 10–4 Ms–1 (b) 1.14 × 10–2 Ms–1 (c) =-
dt dt
(c) 1.14 × 10–3 Ms–1 (d) 5.7 × 10–3 Ms–1
61. The rate constant of a zero order reaction is d[ NH 3 ] 1 d[H 2 ]
2.0 × 10–2 mol L–1 s–1. If the concentration of the (d) =-
dt 3 dt
reactant after 25 seconds is 0.5 M. What is the
initial concentration? 67. The rate of a reaction increases four-fold when
(a) 0.5 M (b) 1.25 M the concentration of reactant is increased 16 times.
If the rate of reaction is 4 × 10– 6 mol L– 1 s– 1 when
(c) 12.5 M (d) 1.0 M
the concentration of the reactant is 4 × 10– 4 mol
62. The reaction A ® B follows first order kinetics.
L– 1. The rate constant of the reaction will be
The time taken for 0.8 mole of A to produce 0.6
mole of B is 1 hour. What is the time taken for
conversion of 0.9 mole of A to produce 0.675 mole
(a) 2×10–4 mol1/2 L–1/2 s–1
of B?
(b) 1×10–2 s–1
(a) 2 hours (b) 1 hour (c) 2×10–4 mol–1/2 L1/2 s–1
(c) 0.5 hour (d) 0.25 hour (d) 25 mol–1 L min–1
63. The rate of a first order reaction is 1.5 × 10–2 mol L–1 68. In the reaction A ® B + C, rate constant is 0.001
min–1 at 0.5 M concentration of the reactant. The Ms–1. If we start with 1 M of A then conc. of A
half life of the reaction is and B after 10 minuter are respectively.
(a) 0.383 min (b) 23.1 min
(a) 0.5 M, 0.5 M (b) 0.6 M, 0.4 M
(c) 8.73 min (d) 7.53 min
(c) 0.4 M, 0.6 M (d) 0.6 M 0.5 M
64. For a first order reaction A ¾® B the reaction
rate at reactant concentration of 0.01 M is found 69. The half life for the virus inactivation if in the
to be 2.0 ´10-5 mol L-1 s-1. The half life period of beginning 1.5% of the virus is inactivated per
the reaction is minute is (Given: The reaction is of first order)
(a) 30 s (b) 220 s (c) 300 s (d) 347 s (a) 76 min (b) 66 min (c) 56 min (d) 46 min
Chemical Kinetics 213
70. The reaction of A2 and B2 follows the equation 75. For an exothermic reaction, the energy of
A 2 (g ) + B2 (g ) ® 2AB ( g ) activation of the reactants is
(a) equal to the energy of activation of products
The following data were observed
(b) less than the energy of activation of
Initial rate of appearance of products
[ A 2 ]0 [ B2 ]0 (c) greater than the energy of activation of
AB(g) (in Ms -1) products
0.10 0.10 2.5 ´ 10 -4 (d) sometimes greater and sometimes less than
that of the products
0.20 0.10 5 ´ 10 -4 76. When a biochemical reaction is carried out in
0.20 0.20 10 ´ 10 -4 laboratory in the absence of enzyme then rate of
reaction obtained is 10–6 times, then activation
The value of rate constant for the above reaction energy of reaction in the presence of enzyme is
is:
6
(a) 2.5 × 10–4 (b) 2.5 × 10–2 (a)
(c) 1.25 × 10 –2 (d) None of these RT
71. In th e presence of an acid, the initial (b) different from Ea obtained in laboratory
concentration of cane sugar was reduced from (c) P is required
0.20 to 0.10 M in 5 hours and from 0.2 to 0.05 M (d) can't say anything
in 10 hours. The reaction is of : 77. A chemical reaction was carried out at 300 K and
280 K. The rate constants were found to be k1
(a) Zero order (b) First order and k2 respectively. then
(c) Second order (d) Third order (a) k2 = 4k1 (b) k2 = 2k1
72. The activation energies of the forward and (c) k2 = 0.25 k1 (d) k2 = 0.5 k1
backward reactions in the case of a chemical
78. In a reaction at 27ºC, 10–3% reactant molecules
reaction are 30.5 and 45.4 kJ/mol respectively.
manage to cross over the barrier of transition
The reaction is :
state. The energy of these molecules in excess of
(a) exothermic
the average value will be (R = 2 cal K–1 mol–1) :
(b) endothermic
(c) neither exothermic nor endothermic
(d) independent of temperature
73. A homogeneous catalytic reaction takes place (a) 6.91 kcal mol–1 (b) 3.00 kcal mol–1
through the three alternative plots A, B and C (c) 4.15 kcal mol –1 (d) 5.10 kcal mol–1
shown in the given figure. Which one of the 79. The activation energy for a simple chemical
following indicates the relative ease with which reaction A ® B is Ea in forward direction. The
the reaction can take place? activation energy for reverse reaction
A (a) is always double of Ea
B
C
(b) is negative of Ea
(c) is always less than Ea
Energy

(d) can be less than or more than Ea


80. The slope in Arrhenius plot, is equal to:
Ea Ea
(a) - (b)
Reaction course 2.303 R R
(a) A > B > C (b) C > B > A
R
(c) A > C > B (d) A = B = C (c) - (d) None of these
74. A catalyst is a substance which : 2.303 Ea
(a) is always in the same phase as in the reaction 81. A radioactive isotope having a half - life period
(b) alters the equilibrium in a reaction of 3 days was received after 12 days. If 3g of the
(c) does not participate in the reaction but alters isotope is left in the container, what would be the
the rate of reaction initial mass of the isotope?
(d) participates in the reaction and provides an (a) 12g (b) 36g (c) 48g (d) 24g
easier pathway for the same
EBD_7587
214 Chemistry Objective MCQs
82. In a chemical reaction A is converted into B. The 88. The rate constant for a first order reaction whose
rates of reaction, starting with initial half life is 480 sec, is :
concentrations of A as 2 × 10–3 M and 1 × 10–3 (a) 1.44 × 10–3 sec–1 (b) 1.44 × sec–1
M, are equal to 2.40 × 10–4 Ms–1 and 0.60 × 10–4 (c) 0.72 × 10–3 sec–1 (d) 2.88 × 10–3 sec–1
Ms–1 respectively. The order of reaction with 89. The hypothetical reaction
respect to reactant A will be
(a) 0 (b) 1.5 (c) 1 (d) 2 A 2 + B2 ¾
¾® 2AB ; follows the following
83. For a reaction A ® Products, a plot of log t1/2 mechanism A 2 ¾Fast
¾¾® A + A ,
versus log a0 is shown in the figure. If the initial Slow Fast
concentration of A is represented by a0, the order A + B 2 ¾¾¾® AB + B , A + B ¾¾ ¾® AB .
of the reaction is The order of the overall reaction is
(a) 0 (b) 1 (c) 2 (d) 3/2
90. If half-life of a substance is 5 yrs, then the total
amount of substance left after 15 years, when initial
amount is 64 grams is
log t1/2 45° (a) 16 g (b) 2 g (c) 32 g (d) 8 g
91. In a reversible reaction the energy of activation
of the forward reaction is 50 kcal. The energy of
activation for the reverse reaction will be
log a0 (a) < 50 kcal
(b) either greater than or less than 50 kcal
(a) one (b) zero (c) two (d) three
(c) 50 kcal
84. The rate constant of a reaction with a virus is (d) > 50 kcal
3.3 × 10– 4 s– 1. Time required for the virus to 92. Activation energy of a chemical reaction can be
become 75% inactivated is
determined by
(a) 35 min (b) 70 min
(c) 105 min (d) 17.5 min
(a) evaluating rate constant at standard
85. A Geigger Muller counter is used to study the
temperature
radioactive process. In the absence of radioactive
(b) evaluating velocities of reaction at two
substance A, it counts 3 disintegration per different temperatures
second (dps). At the start in the presence of A, it (c) evaluating rate constants at two different
records 23 dps; and after 10 min 13 dps, temperatures
(i) What does it count after 20 min (d) changing concentration of reactants
(ii) What is the half life of A? 93. For the exothermic reaction A + B ® C + D, DH
(a) 8 dps, 10 min (b) 5 dps, 10 min is the heat of reaction and Ea is the energy of
(c) 5 dps, 20 min (d) 5 dps, 5 min activation. The energy of activation for the
formation of A + B will be
86. The half life period for catalytic decomposition
(a) Ea (b) DH
of AB3 at 50 mm Hg is 4 hrs and at 100 mm Hg it (c) Ea + DH (d) DH – Ea
is 2 hrs. The order of reaction is 94. A reaction having equal energies of activation
(a) 1 (b) 3 (c) 2 (d) 0 for forward and reverse reaction has :
87. The rate equation for a reaction, (a) DG = 0
N2O ¾® N2 + 1/2O2 (b) DH = 0
is Rate = k[N2O]0 = k. If the initial concentration (c) DH = DG = DS = 0
of the reactant is a mol Lit–1, the half-life period (d) DS = 0
of the reaction is 95. The rate coefficient (k) for a particular reactions
a is 1.3 × 10–4 M–1 s–1 at 100°C, and 1.3 × 10–3 M–1 s–1
(a) t 1 = (b) - t 1 = ka at 150°C. What is the energy of activation (Ea)
2
2k 2
(in kJ) for this reaction? (R = molar gas constant
a k = 8.314 JK–1 mol–1)
(c) t1 = (d) t1 =
k 2
a
2 (a) 16 (b) 60 (c) 99 (d) 132
Chemical Kinetics 215
96. The reaction X ® Y is an exothermic reaction. initial value. If the rate constant for a first order
Activation energy of the reaction for X into Y is reaction is k, the t1/4 can be written as
150 kJ mol–1. Enthalpy of reaction is 135 kJ mol–1.
The activation energy for the reverse reaction,
Y ® X will be :
(a) 280 kJ mol–1(b) 285 kJ mol–1 (a) 0.75/k (b) 0.69/k (c) 0.29/k (d) 0.10/k
–1
(c) 270 kJ mol (d) 15 kJ mol–1 99. A ® B, DH = – 10kJ mol– 1, Ea(f) = 50 kJ mol– 1, then
97. The activation energy for a reaction which Ea of B ® A will be
doubles the rate when the temperature is raised (a) 40 kJ mol– 1 (b) 50kJ mol– 1
from 298 K to 308 K is (c) – 50kJ mol – 1 (d) 60 kJ mol– 1
100. For a certain reaction, rate = k × [H+]n. If pH of
(a) 59.2 kJ mol–1 (b) 39.2 kJ mol–1 reaction changes from two to one, the rate
(c) 52.9 kJ mol–1 (d) 29.5 kJ mol–1 becomes 100 times of its value at pH = 2, the
98. t1/4 can be taken as the time taken for the order of reaction is –
3 (a) 1 (b) 2 (c) 0 (d) 3
concentration of a reactant to drop to of its
4

Answer KEYs
1 (b) 11 (b) 21 (d) 31 (c) 41 (b) 51 (d) 61 (d) 71 (b) 81 (c) 91 (b)
2 (c) 12 (d) 22 (b) 32 (d) 42 (c) 52 (b) 62 (b) 72 (a) 82 (d) 92 (c)
3 (b) 13 (b) 23 (d) 33 (b) 43 (a) 53 (b) 63 (b) 73 (b) 83 (b) 93 (c)
4 (a) 14 (b) 24 (b) 34 (c) 44 (c) 54 (c) 64 (d) 74 (c) 84 (b) 94 (b)
5 (d) 15 (c) 25 (b) 35 (b) 45 (c) 55 (c) 65 (b) 75 (b) 85 (a) 95 (b)
6 (c) 16 (d) 26 (b) 36 (a) 46 (d) 56 (a) 66 (a) 76 (b) 86 (c) 96 (b)
7 (c) 17 (d) 27 (b) 37 (a) 47 (d) 57 (b) 67 (a) 77 (c) 87 (a) 97 (c)
8 (a) 18 (d) 28 (c) 38 (d) 48 (b) 58 (a) 68 (c) 78 (a) 88 (a) 98 (c)
9 (b) 19 (d) 29 (d) 39 (d) 49 (c) 59 (d) 69 (d) 79 (d) 89 (d) 99 (d)
10 (d) 20 (d) 30 (c) 40 (c) 50 (c) 60 (b) 70 (c) 80 (a) 90 (d) 100 (b)

1. (b) N2O5 (g) ¾¾


® 2 NO2(g) + 1/2 O2 (g) -d[ N 2 ] 1d[ H 2 ] 1 d ( NH 3 ]
=- =
d 1 d d
dt 3dt 2 dt
– [ N 2O5 ] = + [NO2 ] = 2 [O 2 ] 1
Rate of disappearance of N2 = the rate of
dt 2 dt dt
2
d formation of NH3 = 1 × 10–4
[ NO2 ] = 1.25 ´ 10 -2 mol L–1s–1 and Rate of disappearance of H2 =3/2 the rate of
dt
formation of NH3 = 3 × 10–4
d
[O2 ] = 3.125 ´ 10 -3 mol L–1s–1 3. (b) The curve Y shows the increase in
dt concentration of products with time.
2. (c) N2+3H2 2NH3; Rate is given by any of 3
4. (a) The order of reaction is and molecularity
the expressions 2
is 2.
EBD_7587
216 Chemistry Objective MCQs
5. (d) Using the relation – Ea / RT
[A] = [A]0 (1/2)n [n = number of half-lives] 15. (c) k = Ae
T = n × t1/2 – Ea
7.5 \ = – 40000
Here, n = =3 R
2.5 \ Ea = 40000 × 2 = 80000 cal
3
æ 1ö 1
\ [A] = 160 ´ çè ÷ø = 160 ´ = 20g 16. (d) E a = 58
2 8 2

Ea1 20
6. (c) Rate1 = k [A]n [B]m; Rate2 = k [2A]n [½B]m
Ea2

Energy
Rate2 k[2A]n [½B]m A+B
\ =
Rate1 k[A]n [B]m 38

= [2]n [½]m = 2n.2–m C+D


= 2n–m Reaction Co-ordinate
7. (c) Let us consider a reaction, 1
17. (d) t1/2 µ
x X + y Y ¾¾ ® aA+bB a2 1
x
rate = [X] [Y] y We know that t1/2 µ n -1
a
It is given that order of reaction w.r.t. i.e. n = 3
rd
Thus reaction is of 3 order.
component Y is zero.
Hence, rate = [X]x 18. (d) A + 2B —® C
i.e., rate becomes independent of the 1 mole of A reacts with 2 moles of B to give
concentration of Y. 1 mol of C.
2.303 a \ 5 moles of A would react with 10 moles of
8. (a) t= log B to give 5 moles of C.
k a-x
But, only 8 moles of B are available
2.303 2.303
or t = log a - log(a - x) \ B acts as a limiting reagent.
k k 2 moles of B gives 1 mole of C
-k
log (a – x) = t + log a \ 8 moles of B will give 1/2 × 8 = 4 moles of C.
2.303 19. (d) The given reaction is : 2X + Y —® Z
9. (b) From slow reaction d[X] d[Z]
Rate = k [A] [B] – =
2dt dt
10. (d) Rate constant is independent of concentration. \ Rate of formation of Z is half of the rate
11. (b) Change in 67% to 33% is almost half the of disappearance of X.
concentration change.
20. (d) Rate of reaction = d[A]
Half dt
67% ¾¾¾¾ ®
change 33.5(»33%)
Given, [A]initial = 0.50 M
So time interval between the stages of its [A]final = 0.38 M
33% and 67% decay is same as t1/2 = 20 min. dt = 10 min = 600 sec
12. (d) It is a second order reaction, first order both d[A] = 0.12
w.r.t S2O82– and I–. 0.12
\ r = k[S2O82–][I–] Rate = = 2 ´ 10 –4 M s –1.
600
All other options are of first order reaction. 21. (d) A catalyst affects equally both forward and
13. (b) k = Ae– Ea/RT backward reactions, therefore it does not
If T ¾¾ ® ¥, k = A affect equilibrium constant of reaction.

E'a E'a 10 20 22. (b) In equation k = Ae - Ea / RT ; A = Frequency


14. (b) = = = T factor
T1 T2 300 2
k = velocity constant, R = gas constant and
\ T2 = 600 K = 327°C Ea = energy of activation
Chemical Kinetics 217
23. (d) The definition of threshold energy. dSO2 dO
24. (b) When Ea = 0, rate constant is independent \- = -2 ´ 2
of temperature. dt dt
25. (b) k = Ae a
–E / RT = – 2 × 2.5 × 10–4
= – 5 × 10–4 mol L–1 s–1
lnk = ln A – Ea /RT
33. (b) Rate of disappearance of reactant = Rate of
For ln k Vs 1/T
appearance of products
ln A = intercept
– Ea/R = slope = –2 × 104 K 1 d [ N 2 O5 ] 1 d [ NO2 ]
\ Ea = 8.3 × 2 × 104 J mol–1 - =
2 dt 4 dt
= 16.6 × 104 J mol–1 or 166 kJ mol–1
26. (b) As the rate of reaction get doubled for every 1 1
k [ N 2 O 5 ] = k ¢ [ N 2 O5 ]
10°C rise in temperature. Hence the increase 2 4
in reaction rate as a result of temperature
rise from 10°C to 100°C is equal to = 29 = 512 k k¢
=
27. (b) Ea (f) – Ea (b) = DH = – 15 k cal 2 4
Þ Ea (f) = – 15 + 20 = 5 k cal \ k ¢ = 2k
Ea - Ea (catalyst) 34. (c) It is bimolecular first order reaction since
k (catalyst)
=e RT =e =e Rate µ [N2O5]
k 3 35. (b) r = k [O2][NO]2. When the volume is reduced
(5 -3) ´10
2.5 to 1/2, the conc. will double
=e =
= e 2 ´ 400 =e
\ New rate = k [2O2][2 NO]2 = 8 k [O2][NO]2
The new rate increases to eight times of its
28. (c) k = A .e - Ea /(RT) initial.
\ Effective overall energy of activation 36. (a) For a zero order reaction.
1 1 rate =k[A]º i.e., rate = k
Ea = Ea (2) - E a (3) + Ea (1) - Ea (5)
2 2 hence unit of k = M.sec–1
1 1 For a first order reaction.
= 60 - 50 + ´ 40 - ´ 10 = 25 kJ/mol
2 2 rate = k [A]
29. (d) Units of k indicate that reaction I is of k = M.sec–1/M = sec–1
second order and reaction II is first order. 37. (a) Plots of conc. [A] Vs time, t
For I reaction, t1/2 µ 1/a,
first t1/2 = 1 hr, second t1/2 = 2 hr Zero First
1hr 2 hr order order
[ A] = 1M ¾¾¾
® 0.5M ¾¾¾
® 0.25M
log [A]

[ A]
1hr 1hr 1hr
[B] = 1M ¾¾¾
® 0.5M ¾¾¾
®0.25M ¾¾¾
® 0.125M t t
1hr 1hr 1hr
= 1M ¾¾¾ ® 0.5M ¾¾¾ ® 0.25M ¾¾¾ ® 0.125M
First
[ A] 0.25M order
= =2 1 1
[ B] 0.125M [ A] Second [ A ]2 Third
30. (c) The slowest step determines the rate. order order
t t t
31. (c) Rate = k [A][B] = R
R' = k [A][2B] 1 d [ HI]
38. (d) rate of appearance of HI =
R k[A][B] k[A][B] 2 dt
= =
R ¢ k[A][2 B] 2k[A][B]
-d ëé H 2 ûù
Þ 2R = R' i.e., rate become doubles. rate of disappearance of H2 =
32. (d) From rate law dt

1 dSO2 dO 1 dSO3 -d [ I 2 ]
- =- 2 = rate of disappearance of I2 =
2 dt dt 2 dt dt
EBD_7587
218 Chemistry Objective MCQs

-d [ H 2 ] d [I 2 ] 1 d [HI] 0.693 0.693


hence =- = 45. (c) t1/2 = = = 10 min
dt dt 2 dt k 0.0693
Reactant after 10 min = 5 mol
2d [H 2 ]2d [I2 ] d [HI] æ dx ö
or – =- =
dt dt dt Rate çè ÷ø = k[A] = 0.0693 × 5 mol min– 1
dt
39. (d) Since all have same concentration of 46. (d) For a zero order, rate of reaction does not
reactants, all would react at same time. change with time.
40. (c) Rate of reaction 47. (d) It is a second order reaction, first order both
Rate of disappearance / appearance w.r.t S2O82– and I–.
= \ r = k[S2O82–][I–]
Stoichiometric coefficient
All other options are of first order reaction.
d [A ] 1 d [B] 0.693 0.693
=- = = 2.6 ´10-2 48. (b) k1 (300 ) = ; k 2 (320 ) =
dt 2 dt 20 5
1 d [B] k 2 (320 )
=
Ea é1 1 ù
=- In ê - ú
2 dt k1 (300 ) R ë T1 T2 û
d [B] 2.303 RT1T2 k
5.2 ´10-2 = - Ea = log 2
dt ( T2 - T1 ) k1
41. (b) After every 30 minutes the amount is
2.303 ´ 8.314
1 = ´ 300 ´ 320 log 4
reduced to therefore t1/2 is 30 minutes. 20 ´ 1000
2
In 90 minutes the amount is reduced to = 55.14 kJ/mol
1 1 E
i.e. n . Here n = 3. True for 1st order - a 10 -16
8 2 49. (c) e RT = 3.8 ´ -
reaction. 100
0.693 Ea
42. (c) k= further - = ln 3.8 ´10 -18
45 RT
2.303 100 Ea = 100 kJ/mol
t= ´ 45 log
0.693 100 - 99.9 -E /RT
k 2 Ae a2 (E - E ) /RT
2.303 ´ 45 ´ 3 1 50. (c) = - E
= e a1 a2
= = 7 hours . k1 Ae a1 /RT
0.693 2
k Ea - Ea
2.303 a 2.303log 2 = 1
43. (a) t= log
k a-x k1 RT

2.303 0.5 (83.314 - 75) ´ 103


= log = 0.384 min . = =2
6 0.05 8.314 ´ 500
2
2.303 100 log k 2 = = 0.868
44. (c) t90% = log (I) 2.303
k 100 - 90 Taking Antilog
2.303 100 k2 = 7.38
t50% = log (II)
k 100 - 50 51. (d) In Arrhenius equation k = Ae - Ea / RT , Ea is
t log10 the energy of activation, which is required
Dividing 90% =
t50% log 2 by the colliding molecules to react resulting
in the formation of products.
\ t90% = 3.3t50% 52. (b) When the temperature is increased, energy
in form of heat is supplied which increases
Chemical Kinetics 219
the kinetic energy of the reacting molecules. 2.303 ´ 8.314 ´ 298 ´ 308
this will increase the number of collisions = log 2.73
and ultimately the rate of reaction will be 308 - 298
enhanced. Ea = 76.62 kJ mol–1 = 18.39 kcal mol–1
53. (b) According to Arrhenius equation,
59. (d) For the reaction
k = Ae. - Ea / RT
where k = rate constant 3A + 2B ¾¾ ® C+D
Rate of disappearance of A = Rate of
E
or ln k = lnA - a appearance of C reaction
RT
A = frequency factor 1 d [ A] d [C ]
= k [ A] [ B ]
n m
Ea = Energy of activation. =- =
3 dt dt
54. (c) k = 1.2 ´ 1014 e -25000 / RT sec -1 or
dMnO-4
14 25000 1 60. (b) Given - = 4.56 × 10–3 Ms–1
log k = log 1.2 × 10 - . dt
R T
From the reaction given,
logk 1 dMnO 4 – 4.56 ´ 10 -3
- = Ms -1
2 dt 2
1/T
Equation of straight line
1 d MnO4- 1 d I2
25000 - =
slope = - 2 dt 5 dt
R
55. (c) There are 5 tens hence (2)5 = 32.
5 dMnO-4 dI2
56. (a) Ea (F.R.) ¹ Ea (B.R.) Ea can be calculated. \ - =
2 dt dt
57. (b) According to Arrhenius equation, On substituting the given value
k = Ae–Ea/RT
\ when Ea = 0, k = A dI2 4.56 ´ 10-3 ´ 5
\ = = 1.14 × 10–2 M/s
Also ln k ns 1/T is a straight line with slope dt 2
= –Ea/R. 61. (d) For a zero order reaction
\ Statements (ii) and (v) are correct.
a-x
58. (a) Let the initial concentration (A) = 100 Rate constant = k =
Final concentration at 298 K = 100 – 10 = 90 t
Final concentration at 308 K = 100 – 25 = 75 a - 0.5
2 × 10–2 =
Substituting the values in the 1st order rate 25
reaction a – 0.5 = 0.5
2.303 100 a = 1.0 M
t= log ...(i) 62. (b) A ® B For a first order reaction
k298 90
Given a = 0.8 mol, (a – x) = 0.8 – 0.6 = 0.2
2.303 100
t= log ...(ii) 2.303 0.8
k308 75 k= log or k = 2.303 log 4
1 0.2
k308 again a = 0.9, a – x = 0.9 – 0.675 = 0.225
From (i) and (ii) = 2.73
k 208 2.303 0.9
k= log
Substituting the value in the following t 0.225
relation
2.303
2.303 R ´ T1 ´ T2 k 2.303log 4 = log 4
Ea = log 2 t
T2 - T1 k1 Hence t = 1 hour
EBD_7587
220 Chemistry Objective MCQs
63. (b) For a first order reaction, A ® Products
4 ×10 – 6 4 ×10 – 6
r = =
r = k[A] or k = (4 ´ 10 – 4 )1/ 2 2 ´ 10 – 2
[A]
= 2 × 10– 4 mol1/2 L– 1/2 s– 1
1.5 ´ 10-2 68. (c) [A ]t = [A ] - kt = 1 - 0.001´10 ´ 60
Þk = = 3 × 10–2
0.5 = 0.4 M

Further, t1/ 2 =
0.693
=
0.693
= 23.1 [B]t = 0.001´ 10 ´ 60 = 0.6 M
k 3 ´ 10-2 69. (d) For the first order reaction for small finite
64. (d) Given [A] = 0.01 M change
Rate = 2.0 × 10–5 mol L–1 s–1
For a first order reaction
1 D[A] D[ A] /[ A]
k1 = Þ = 1.5% min– 1
Rate = k[A] [A] Dt Dt
= 0.015 min– 1
2.0 ´ 10 -5
k= = 2 × 10–3 0.693
[0.01] t1/2 = = 46.2 min » 46 min
0.693 0.015 min –1
t1/2 = -3 = 347 sec 70. (c) Order w.r.t. A = 1; order w.r.t. B = 1
2 ´ 10
65. (b) Given: 75% reaction gets completed in 32 1 d ( AB)
Rate = = k r [ A ][ B ] ;
min 2 dt
2.303 a
Thus, k = log 1
t (a - x ) ´ (2.5 ´104 ) = k r ( 0.1)( 0.1)
2
2.303 100 kr = 1.25 × 10–2
= log
32 (100 - 75) 1 0.2 1 0.2
71. (b) k1 = In ; k 2 = In ; k1 = k 2 .
2.303 5 0.1 10 0.05
= log 4 = 0.0433 min–1
32 72. (a) Exothermic because of activation energy
Now we can use this value of k to get the Eb > E¦
value of time required for 50% completion 73. (b) More activations energy, slow reaction rate.
of reaction
74. (c) A catalyst does not take part in chemical
2.303 a 2.303 100 reaction but it alters the rate of reaction.
t= log = log
k (a - x ) 0.0433 50 75. (b) Activation energy of reactant is less than
2.303 the energy of activation of products.
= log 2 = 16 min 76. (b) The presence of enzyme (catalyst) increases
0.0433
the speed of reaction by lowering the energy
66. (a) If we write rate of reaction in terms of
barrier, i.e., a new path is followed with lower
concentration of NH3 and H2,then
activation energy.
1 d[ NH 3 ] 1 d[H 2 ]
Rate of reaction = =- ET
2 dt 3 dt
E'T
d[ NH 3 ] 2 d[H 2 ]
So, =-
dt 3 dt Ea
Products
Energy

67. (a) Rate µ Ea


Concentration 1

= k Concentration
Rate Reactants + catalyst
k= 1/2
(Concentration) Progress of reaction
Chemical Kinetics 221
Here ET is the threshold energy. 82. (d) A B
Ea and Ea1 is energy of activation of reaction Initial concentration Rate of reaction
in absence an d presence of catalyst 2 × 10–3 M 2.40 × 10–4 Ms–1
respectively. 1 × 10–3 M 0.60 × 10–4 Ms–1
77. (c) The rate constant doubles for 10º C rise in rate of reaction
temperature. r = k[A]x
For 20º C rise, the rate constant will be 4 where x = order of reaction
times
hence
\ k1 = 4k2 or k2 = 0.25 k1
2.40 × 10–4 = k [2 × 10–3]x ......(i)
78. (a) e - Ea / RT = 10 -3 % = 10 -5 ; 0.60 × 10–4 = k [1 × 10–3]x ......(ii)
Ea = 2.303 × 2 × 300 × 5 cal On dividing eqn.(i) from eqn. (ii) we get
= 6.91 k cal mol–1 4 = (2)x
79. (d) The activation energy of reverse reaction \ x=2
will depend upon whether the forward i.e. order of reaction = 2
reaction is exothermic or endothermic. 83. (b) Plot given is for zero order reaction.
As DH = E a (forward reaction) – E a
0.693
(backward reaction) 84. (b) t1/2 = = 2100 s = 35 min
k
For exothermic reaction
t75% = 2t1/2 = 2 × 35 = 70 min
DH = –ve
85. (a) In the absence of A, 3 dps is zero error, hence
\ –DH = Ea(f) – Ea(b)
or Ea(f) = Ea(b) – DH Initial count = 23 – 3 = 20 dps
\ Ea(f) < Ea(b) After 10 min = 13 – 3 = 10 dps
for endothermic reaction After 20 min = 5 dps
DH = + ve (recorded = 5 + 3 = 8 dps)
\ DH = Ea(f) – Ea(b) or Ea(f) = DH + Ea(b) (50% fall in 10 min, T50 = 10 min)
\ Ea(f) > Ea(b). 1
86. (c) t1/ 2 µ where n is the order of
80. (a) Arrhenius equation is given by ( p ) n -1
k = Ae - Ea / RT reaction
Taking log on both sides, we get n -1 n -1
Ea
2 æ 50 ö 1 æ 1ö
log k = log A – =ç ÷ or =ç ÷
4 è 100 ø 2 è 2ø
2.303 RT
Arrhenius plot a graph between log k and n – 1 = 1; n = 2
1 - Ea 87. (a) For a zero order reaction
whose slope is .
T 2.303R a
t1/2 =
81. (c) Given t1/2 = 3 2k
Total time T = 12 0.693 0.693
12 88. (a) k= = = 1.44 ´ 10 -3 s-1
No. of half lives (n) = =4 t1/2 480
3
n
89. (d) ¾® 2AB ;
A 2 + B2 ¾
æ1ö N 4
ç ÷ = \ æç 1 ö÷ = 3 A2 ¾
¾® A + A ( Fast );
è2ø No è2ø N
3 1 A + B2 ¾
¾® AB + B (Slow )
=
N 16 Rate law = k[A][B 2 ] put value of [A] from
N = 48 g Ist reaction since A is intermediate
k[A 2 ] = A
EBD_7587
222 Chemistry Objective MCQs

\ Rate law equation = K k[A 2 ][B2 ] Ea é 1 1 ù


1= -
1 3 2.303 ´ 8.314 ë 373 423 úû
ê
\ Order = +1 =
2 2 Ea = 60 kJ / mol
90. (d) t1/2 = 5 years, T = 15 years hence total num- 96. (b) X ¾® Y ; DH = –135 kJ/mol,
Ea = 150 kJ/mol
ber of half life periods = 15 = 3
5 For an exothermic reaction
64 Ea(F.R.) = DH + E¢a(B.R.)
\ Amount left = = 8g 150 = – 135 + E¢a(B.R.)
( 2) 3
E¢a(B.R.) = 285 kJ/mol
91. (b) DH = Ea (f) - Ea (b) 97. (c) Activation energy can be calculated from
Thus energy of activation for reverse the equation.
reaction depend upon whether reaction is
log K 2 – Ea æ 1 1 ö
exothermic or endothermic. = ç – ÷
log K1 2.303 R è T2 T1 ø
If reaction is exothermic,
DH = - ve , Ea (b) > Ea (f)
log K 2
If reaction is endothermic, Given = 2 T2 = 308 K; T1 = 298 K
log K1
DH = + ve Ea (b) < Ea (f)
92. (c) We know that the activation energy of – Ea æ 1 1 ö
\ log 2 = ç – ÷
chemical reaction is given by formula 2.303 ´ 8.314 è 308 298 ø

Ea é T2 - T1 ù Ea = 52.9 kJ mol–1
k2
= = ê ú , where k1 is the
k1 2.303R ë T1T2 û 2.303 1 2.303 4
98. (c) t1/ 4 = log = log
k 3/ 4 k 3
rate constant at temperature T1 and k2 is
the rate constant at temperature T2 and Ea 2.303 2.303
is the activation energy. Therefore activation = (log 4 - log 3) = (2log 2 - log 3)
k k
energy of chemical reaction is determined
2.303 0.29
by evaluating rate constant at two different = (2 ´ 0.301 - 0.4771) =
k k
temperatures.
93. (c) For the exothermic reaction the energy of 99. ® B, DH = – 10 kJ mol– 1
(d) A ¾¾
products is always less than the reactants.
It is an exothermic reaction.
If E a is the energy of activation for the
Ea(b) = Ea(f) – (DH)
forward reaction, the energy of activation
for backward reaction is Ea + DH = 50 – (– 10) = 60 kJ
100. (b) pH = 2 ; r1 = k × (10–2)n {Q [H+] = 10–pH}
94. (b) DH = Ea (f ) - Ea (b) = 0
pH = 1 ; r2 = k × (10–1)n
95. (b) According to Arrhenius equation
Given r2 = 100 r1
k2 Ea æ 1 1 ö
log = - æ 10-1 ö
n
k1 2.303R çè T1 T2 ÷ø Þ ç ÷ = 100
ç 10-2 ÷
è ø
1.3 ´ 10 -3 Ea é 1 1 ù 10n = 100
log = -
1.3 ´ 10 -4 2.303 ´ 8.314 êë 373 423 úû \ n=2
19 Surface Chemistry
1. The volume of gases NH3, CO2 and H2 adsorbed 7. Gold number of a lyophilic sol is such a property
by one gram of charcoal at 300 K are in order of: that:
(a) H2 > CO2 > NH3 (b) NH3 > H2 > CO2 (a) The larger its value, the greater is the
(c) NH3 > CO2 > H2 (d) CO2 > NH3 > H2 peptizing power.
æ xö (b) The lower its value, the greater is the
2. A plot of log ç ÷ against log P for the peptizing power.
è mø
(c) The lower its value, the greater is the
adsorption of a gas on a solid gives a straight
protecting power.
line with slope equal to : (d) The larger its value, the greater is the
protecting power.
1 8. Among the following, correct statement is :
(a) (b) n (c) log K (d) K
n
3. The heat evolved in chemisorption lies in the
range (in kJ/mol) of :
(a) 80 to 240 (b) 20 to 40 (a) Brownian movement is more pronounced
(c) 40 to 80 (d) 20 to 100 for smaller particles than for bigger–
4. Identify the correct statement regarding particles.
enzymes_______. (b) Sols of metal sulphides are lyophilic.
(a) Enzymes are specific biological catalysts (c) Hardy Schulze law states that bigger the
that cannot be poisoned size of the ons, the greater is its coagulating
(b) Enzymes are normally heterogeneous power.
catalysts that are very specific in their (d) One would expect charcoal to adsorb
action chlorine more than hydrogen sulphide.
(c) Enzymes are specific biological catalysts 9. Under the influence of an electric field, the particles
that can normally function at very high in a sol migrate towards cathode. The coagulation
temperatures (T » 1000K) of the same sol is studied using NaCl, Na2SO4
(d) Enzymes are specific biological catalysts
and Na3PO4 solutions. Their coagulating values
that possess well-defined active sites
will be in the order
5. Catalytic poisons act by :
(a) making the products chemically inactive. (a) NaCl > Na2SO4 > Na3PO4
(b) increasing the rate of the backward reaction. (b) Na2SO4 > Na3PO4 > NaCl
(c) chemical combination with any one of the (c) Na3PO4 > Na2SO4 > NaCl
reactants. (d) Na2SO4 > NaCl > Na3PO4
(d) preferential adsorption on the catalyst
10. 3.6 gram of oxygen is adsorbed on 1.2 g of metal
surface.
6. The size of particles in suspension, true solution powder. What volume of oxygen adsorbed per
and colloidal solution varies in the order: gram of the adsorbent at 1 atm and 273 K?
(a) suspension > colloidal > true solution
(b) true solution > suspension > colloidal
(c) suspension > colloidal = true solution (a) 0.19 Lg–1 (b) 1 Lg–1
(d) none of these (c) 2.1 L g–1 (d) None of these
EBD_7587
224 Chemistry Objective MCQs
11. For the graph below, select correct order of 17. A thixotropic gel is characterised by
temperature? (a) reversible transformation of gel into sol on
heating
Ps T1
(b) irreversible transformation of gel into sol
by stirring
Extent of T2 (c) reversible transformation of gel into sol by
adsorption stirring
(x/m) T3 (d) swelling on placing in the dispersion medium
18. In an experiment, addition of 4.0 ml of 0.005 M
Pressure (P) BaCl2 to 16.0 ml of arsenius sulphide sol just
(a) T1 > T2 > T3 (b) T2 > T3 > T1 causes the complete coagulation in 2 hrs. The
(c) T3 > T2 > T1 (d) T1 = T2 = T3 flocculating value of the effective ion is
12. 100 mL of 0.6 M acetic acid is shaken with 2 g
activated carbon. The final concentration of the
solution after adsorption is 0.5 M. What is the
(a) Cl–, 1.0 (b) Cl–, 2.0
amount of acetic acid adsorbed per gram of 2+
(c) Ba , 1.0 (d) Ba2+, 0.5
carbon? 19. Choose the correct statement
(a) 0.6 g (b) 0.3 g (a) Latex is an example of solid-in-liquid sol
(c) 1.2 g (d) None of these (b) Latex is a negative sol
13. The efficiency of an enzyme in catalysing a (c) Latex is a positive sol
reaction is due to its capacity (d) None of the above is correct
(a) to form a strong enzyme-substrate complex 20. Arrange the following electrolytes in the
(b) to decrease the bond energies of substrate increasing order of coagulating power for ferric
molecule hydroxide sol
(c) to change the shape of the substrate
molecule Na 3PO 4 KCl K 2SO4 NaCl
(d) to lower the activation energy of the reaction I II III IV
14. A chemical reaction is catalyzed by a catalyst X. (a) (b)
I < II < III < IV II = IV < III < I
Hence X
(a) reduces enthalpy of the reaction (c) II = IV < I < III (d) II = III < IV = I
(b) decreases rate constant of the reaction 21. Although, nitrogen does not adsorb on a surface
(c) increases activation energy of the reaction at room temperature, it adsorbs on the surface at
(d) does not affect equilibrium constant of the 83 K. Which one of the following statements is
reaction correct ?
15. The gold numbers of protective colloids A, B, C (a) At 83 K, there is formation of monolayer
and D are 0.04, 0.004, 10 and 40 respectively. The (b) At 83 K, nitrogen is adsorbed as atoms
protective powers of A, B, C and D are in the (c) At 83 K, nitrogen molecules are held by
order : chemical bonds
(a) A > B > C > D (b ) B > A > C > D (d) At 83 K, there is formation of multimolecular
(c) D > C > A > B (d) D > C > B > A layers
16. The density of gold is 19 g/cm3. If 1.9 × 10–4g of 22. The following statements relate to the adsorption
gold is dispersed in one litre of water to give a of gases on a solid surface. Identify the incorrect
sol having spherical gold particles of radius 10 statement among them :
nm, then the number of gold particles per mm3 of (a) Enthalpy of adsorption is negative
the sol will be : (b) Energy appears as heat
(c) On adsorption, the residual forces on the
(a) 1.9 × 1012 (b) 6.3 × 1014 surface are increased
(c) 6.3 × 1010 (d) 2.4 × 106 (d) Entropy of adsorption is negative
Surface Chemistry 225
23. If x is the mass of the gas adsorbed on mass m of (a) The adsorption requires activation at 25°C
the adsorbent at pressure p, Freundlich adsorption (b) The adsorption is accompanied by a
isotherm gives a straight line on plotting decrease in enthalpy
(a) x/m vs p (b) x/m vs 1/p (c) The adsorption increases with increase of
(c) log x/m vs log p (d) log x/m vs p temperature
24. The correct statement for both the processes of
(d) The adsorption is irreversible
physisorption and chemisorption is
(a) both are endothermic 33. In an experiment, 200 mL of 0.5 M oxalic acid is
(b) chemisorption is endothermic but shaken with 10 g of activated charcoal and filtered.
physisorption is exothermic The concentration of the filtrate is reduced to 0.4
(c) both are exothermic æxö
(d) physisorption is endothermic but M. The amount of adsorption ç ÷ is
èmø
chemisorption is exothermic.
25. Which one of the sols acts as protective colloid?
(a) As2S3 (b) Gelatin (a) 0.9 (b) 1.8 (c) 0.18 (d) 0.09
(c) Au (d) Fe (OH)3 34. Equal volume each of two sols of AgI, one
26. Given below are a few electrolytes, indicate which obtained by adding AgNO3 to slight excess of
one among them will bring about the coagulation KI and another obtained by adding KI to slight
of a gold sol quickest and in the least of
excess of AgNO3, are mixed together. Then:
concentration? (a) the two sols will stabilize each other
(a) NaCl (b) MgSO4 (b) the sol particles will acquire more electric
(c) Al2 (SO4)3 (d) K4 [Fe (CN)6] charge
27. Which of the following has minimum flocculation (c) the sols will coagulate each other mutually
value for positively charged sol? (d) a true solution will be obtained
(a) Cl– (b) SO 24 - 35. Under the influence of an electric field, the
(c) PO 4 3 - (d) [Fe (CN)6]4– particles in a sol migrate towards cathode. The
28. Butter is a colloid formed when coagulation of the same sol is studied using NaCl,
(a) fat is dispersed in water Na2SO4 and Na3PO4 solutions. Their coagulating
(b) fat globules are dispersed in water values will be in maximum for:
(c) water is dispersed in fat (a) NaCl (b) Na2SO4
(d) None of the these (c) Na3PO4 (d) same for all
29. Cloud or fog is a colloidal system in which the 36. According to Langmuir adsorption isotherm, the
dispersed phase and the dispersion medium are
amount of gas adsorbed at very high pressures
(a) Gas, Liquid (b) Liquid, gas
(c) Liquid, liquid (d) Solid, Liquid (a) reaches a constant limiting value
30. Which of the following constitutes irreversible (b) goes on increasing with pressure
colloidal system in water as dispersion medium? (c) goes on decreasing with pressure
(a) Clay (b) Platinum (d) increases first and then decreases with
(c) Fe(OH)3 (d) All of these pressure
31. At the high pressure, Langmuir adsorption 37. Under ambient conditions, which among the
isotherm takes the form following surfactants will form micelles in
x ap x a aqueous solution at lowest molar
(a) = (b) =
m 1 + bp m b concentration?
x m b 1
(c) = ap (d) = + (a) CH3–(CH2)8–COO Na+
-
m x a ap
+ -
32. Methylene blue, from its aqueous solution, is (b) CH3(CH2)11 N (CH3)3 Br
adsorbed on activated charcoal at 25°C. For this
process, which of the following statement is (c) CH3–(CH2)13–OSO3– Na+
correct ? + -
(d) CH3(CH2)15 N (CH3)3 Br
EBD_7587
226 Chemistry Objective MCQs
38. The migration of dispersion medium under the (c) They carry charge
influence of an electric potential is called : (d) The particles are hydrated
(a) Cataphoresis (b) Electro-osmosis 43. Lyophilic sols are more stable than lyophobic
(c) Electrophoresis (d) Sedimentation sols because:
39. Flocculation value of BaCl2 is much less than (a) the colloidal particles have positive charge
that of KCl for sol A and flocculation value of (b) the colloidal particles have negative charge
Na2SO4 is much less than that of NaBr for sol B. (c) the colloidal particles are solvated
The correct statement among the following is : (d) there is strong electrostatic repulsion
between the colloidal particles
44. Select incorrect statement:
(a) Micelles are associated colloids
(a) Both the sols A and B are negatively (b) The electrical charge on a colloid particle is
charged. indicated by electrophoresis
(b) Sol A is positively charged and Sol B is (c) Formation of micelles takes place above Kraft
negatively charged. temperature
(c) Both the sols A and B are positively charged. (d) Formation of micelles takes place below
(d) Sol A is negatively charged and sol B is CMC
positively charged. 45. Point out the false statement
40. Which of the following statements regarding
(a) Brownian movement and Tyndall effect are
difference between adsorption and absorption shown by colloidal systems.
is incorrect?
(b) Gold number is a measure of the protective
(a) Adsorption is a surface whereas absorption
power of a lyophillic colloid
is a bulk phenomena.
(c) The colloidal solution of a liquid in liquid is
(b) Water vapours are absorbed by anhydrous
called gel
CaCl2 but adsorbed by silica gel.
(d) Hardy - Schulze rule is related with
(c) Adsorption and absorption take place
coagulation.
individually. They can not occur
46. Which gas will be adsorbed on a solid to greater
simultaneously.
extent?
(d) All of the above statements are correct.
(a) Having non - polar molecule
41. In Langmuir's model of adsorption of a gas on a
(b) Having highest critical temperature
solid surface (c) Having lowest critical temperature
(d) Having lowest critical pressure
(a) the mass of gas striking a given area of 47. Which of the following adsorption isotherms
surface is proportional to the pressure of represents the absorption of a gas by a solid
the gas involving multilayers of formation? (P s =
(b) the mass of gas striking a given area of saturation pressure)
surface is independent of the pressure of
the gas Ps
(c) the rate of dissociation of adsorbed
molecules from the surface does not depend x
(a)
on the surface covered m
(d) the adsorption at a single site on the surface P
may involve multiple molecules at the same
time.
42. Which one of the following statements is not Ps
correct in respect of lyophilic sols?
(a) There is a considerable interaction between the x
(b)
dispersed phase and dispersion medium m
(b) These are quite stable and are not easily P
coagulated
Surface Chemistry 227
53. Which property of colloids is not dependent on
Ps the charge on colloidal particles?
(a) Coagulation (b) Electrophoresis
x
(c) (c) Electro - osmosis (d) Tyndall effect
m
54. Smoke is an example of :
P
(a) Solid dispersed in solid
(b) Gas dispersed in liquid
Ps
(c) Solid dispersed in gas
x (d) Gas dispersed in solid
(d) 55. On addition of 1 mL of 10% NaCl solution to
m
P 10 mL gold sol in the presence of 0.025 g of starch,
the coagulation is just prevented. Starch has gold
48. A deter gent (C12 H 25SO 4- Na + ) sol ut ion
number
becomes a colloidal sol at a concentration of
10–3 M. On an average 1013 colloidal particles are
present in 1 mm3. What is the average number of (a) 2.5 (b) 25 (c) 0.25 (d) 0.025
ions which are contained by one colloidal particle
56. Fog is a colloidal solution of
(micelle)? [Given: NA = 6 × 1023mol–1]
(a) solid particles dispersed in gas
(b) solid particles dispersed in a liquid
(a) 6 × 107 (b) 10 (c) liquid particles dispersed in gas
(c) 60 (d) None of these (d) gaseous particles dispersed in a liquid
49. Which of the following is most powerful to 57. Select incorrect statement:
coagulate the negative colloid?
(a) Soap and detergent lower the interfacial
(a) ZnSO4 (b) Na3PO4
(c) AlCl3 (d) K4[Fe(CN)6] surface tension between oil and water
50. Coagulation value of the electrolytes AlCl3 and (b) Basic principle of peptization is reverse of
NaCl for As2S3 sol are 0.093 and 52 respectively. coagulation
How many times AlCl3 has greater coagulating (c) Soap and detergent used as emulsifiers
power than NaCl ?
(d) Lyophilic sols need stabilizing agent
(a) 930 (b) 520
58. Which of the following is most effective in
(c) 560 (d) None of these
51. The protecting power of lyophilic colloidal sol is causing the coagulation of ferric hydroxide sol?
expressed in terms of : (a) KCl (b) KNO3
(a) coagulation value (c) K2SO4 (d) K3[Fe(CN)6]
(b) gold number 59. Aluminium hydroxide forms a positively charged
(c) critical miscelle concentration
sol. Which of the following ionic substances
(d) oxidation number
52. Which of the following statements is correct for should be most effective in coagulating the sol?
the spontaneous adsorption of a gas? (a) NaCl (b) CaCl2
(a) DS is negative and, therefore, DH should be (c) Fe2(SO4)3 (d) K3PO4
highly positive 60. Colloidal gold is prepared by
(b) DS is negative and therefore, DH should be
(a) Mechanical dispersion
highly negative
(c) DS is positive and, therefore, DH should be (b) Peptisation
negative (c) Bredig’s Arc method
(d) DS is positive and, therefore, DH should also (d) Hydrolysis
be highly positive
EBD_7587
228 Chemistry Objective MCQs
61. Colloid of which one of the following can be (a) between 0 and 1 in all cases
prepared by electrical dispersion method as well (b) between 2 and 4 in all cases
as reduction method ? (c) 1 in case of physical adsorption
(d) 1 in case of chemisorption
(a) Sulphur 67. The adsorption of a gas on a solid surface varies
(b) Ferric hydroxide with pressure of the gas in which of the following
(c) Arsenious sulphide manner.
(d) Gold (a) Fast ® slow ® independent of the pressure
62. The cause of Brownian movement is (b) Slow ® fast ® independent of the pressure
(a) Heat changes in liquid state (c) Independent of the pressure ® fast ® slow
(b) Convectional currents (d) Independent of the pressure ® slow ® fast
(c) The impact of molecules of the dispersion 68. Associated colloids :
medium on the colloidal particles.
(d) Attractive forces between the colloidal (a) raise both the surface tension and viscosity
particles and molecules of dispersion of water
medium. (b) lower both the surface tension and viscosity
63. During adsorption of water
(a) TDS is positive (c) lower the surface tension and raise the
(b) DH–TDS is negative viscosity of water
(c) DH is positive (d) have greater concentration at the surface
(d) TDS and DG become zero layer than the bulk of the solution
64. Which of the following gas molecules have 69. Hydrolysis of urea is an example of
maximum value of enthalpy of physisorption? (a) homogeneous catalysis
(a) C2H6 (b) Ne (c) H2O (d) H2 (b) heterogeneous catalysis
65. Adsorption is accompanied by the evolution of (c) biochemical catalysis
heat. So according to Le-Chatelier principle the (d) zeolite catalysis
amount of substance adsorbed should 70. When KClO3 is heated, it decomposes into
(a) increase with decrease in temperature KCl + O2. If some MnO2 is added, the reaction
(b) increase with increase in temperature goes much faster because
(c) decrease with decrease in temperature (a) MnO2 decomposes to give O2
(d) decrease with increase in temperature
(b) MnO2 provides heat by reacting
66. In Freundlich adsorption isotherm, the value of
(c) better contact is provided by MnO2
1/n is : (d) MnO2 acts as a catalyst

Answer KEYs
1 (c ) 8 (a ) 15 (b ) 22 (c) 29 (b ) 36 (a ) 43 (c ) 50 (c ) 57 (d ) 64 (c )
2 (a ) 9 (a ) 16 (d ) 23 (c) 30 (d ) 37 (d ) 44 (d ) 51 (b ) 58 (d ) 65 (a )
3 (a ) 10 (c ) 17 (c ) 24 (c) 31 (b ) 38 (b ) 45 (c ) 52 (b ) 59 (d ) 66 (a )
4 (d ) 11 (c ) 18 (c ) 25 (b ) 32 (b ) 39 (b ) 46 (b ) 53 (d ) 60 (c) 67 (a )
5 (d ) 12 (b ) 19 (b ) 26 (c) 33 (c ) 40 (c ) 47 (a ) 54 (c ) 61 (d ) 68 (c )
6 (a ) 13 (d ) 20 (b ) 27 (d ) 34 (c ) 41 (a ) 48 (c ) 55 (b ) 62 (c) 69 (c )
7 (c ) 14 (d ) 21 (d ) 28 (c) 35 (a ) 42 (c ) 49 (c ) 56 (c ) 63 (b ) 70 (d )
Surface Chemistry 229

1. (c) Van der Waal force ­ adsorption ­ Volume of O2 per gram of adsorbent
1 3 0.0821 ´ 273
x = ´ = 2.10
2. (a) = KP n 32 1
m
x 1 11. (c) Temp­ Extent of adsorption ¯
log = log K + ´ log P
m n 12. (b) Mass of acetic acid adsorbed by 2 g
1 charcoal
Slope = = 100 × 10–3 × (0.6 – 0.5) × 60
n
3. (a) DH : 80 to 240 kJ mol–1 x 0.6
4. (d) Enzymes are specific biological catalysts Þ 0.6; = Þ 0.3
m 2
possessing well - defined active sites. 13. (d) Efficiency of catalysing property of a
5. (d) The catalytic poisons decrease the activity
catalyst is inversely proportional to
of the catalyst because they are activation energy.
preferentially adsorbed on the surface of
14. (d) A catalyst affects equally both forward and
catalyst.
backward reactions, therefore it does not
6. (a) Solution Particle size
affect equilibrium constant of reaction.
True solution Size < 1 nm 15. (b) Smaller is the 'gold number' of protective
Colloidal solution 1 nm < size < 1000 nm
colloid, greater is its protective power.
Suspension Size > 1000 nm 16. (d) Volume of gold dispersed in 1 L water
7. (c) The reciprocal of gold number is directly
proportional to the protecting power. Mass 1.9 ´ 10 - 4 g
= = = 1 × 10–5 cm3
8. (a) Brownian movement is the random motion Density 19gm cm -3
of particles suspended in a fluid (a liquid or
Radius of gold sol particle
gas) resulting from their collision with the
fast moving atom or molecules in the liquid = 10 nm = 10 × 10–7 cm = 10–6 cm
or gaseous state of the matter. That means 4
Volume of gol sol particle = p r 3
smaller particles are responsible for the 3
Brownian movement than for bigger 4 22
particles. = ´ ´ (10-6 )3 = 4.19 × 10–18 cm3
3 7
9. (a) Since the sol particles migrate towards
\ No. of gold sol particles in 1 × 10–5 cm3
cathode, they are positively charged. Hence,
anions would be effective in coagulation. 1 ´ 10-5
= = 2.38 ´ 1012
Greater is the valency of effective ion, 4.19 ´ 10 -18
smaller will be its coagulating value.
\ No. of gold sol particles in one mm3
10. (c) Mass of O 2 per gram of adsorbent
3.6 2.38 ´ 1012
= =3 = = 2.38 ´ 106
1.2 106
No. of moles of O 2 per gram of
17. (c) Thixotropic gel changes into sol on
3 mechanical agitation, which on standing sets
adsorbent =
32 to gel.
EBD_7587
230 Chemistry Objective MCQs
18. (c) As2S3 sol is negatively charged owing to 27. (d) For positive charge colloids coagulating
preferential adsorption of S2– ions. Cation power ­coagulation value ¯.[Fe(CN)6]4–.
would be the effective ion in coagulation. 28. (c) Butter (liquid - solid) water dispersed in fat
Flocculating value = minimum milli mol of 29. (b) Cloud and fog are colloidal system having
the effective ion per litre of sol = dispersed phase liquid and dispersion
medium gas
4 ´ 0.005 ´103
= 1.0 30. (d) All are irreversible colloidal systems.
4 + 16
31. (b) Langmuir adsorption isotherm, is
19. (b) Latex is a negative sol.
Latex is an emulsion in which the rubber x ap
=
particles forming emulsion are stabilized by m 1 + bp
various proteins. At high pressure, 1 + bp = bp
20. (b) Ferric hydroxide sol is positively charged
and hence anions would be effective in x ap a
or = =
causing coagulation. Greater the valence of m bp b
the effective ion, more will be its coagulating 32. (b) The adsorption of methylene blue on
power. activated charcoal is an example of
21. (d) At 83 K physical absorption takes place physiosorption which is exothermic,
effectively so multimolecular layers formed. multilayer and does not have energy barrier.
22. (c) Adsorption is a phenomena of attracting 33. (c) Mass of oxalic acid adsorbed by 10 g
and retaining the molecules of a substance charcoal
on the surface of a liquid or a solid resulting
into a higher concentration of the molecules = 200 ´ 10 -3 (0.5 - 0.4) ´ 90 = 1.8 g
on the surface.
x 1 .8
After adsorption there is a decrease in the = = 0 .18
m 10
residual forces due to bond formation DG,
DH & DS all are negative in the case of 34. (c) The sols obtained in the two cases will be
adsorption. oppositely charged to coagulate each other.
23. (c) Freundlich adsorption isotherm gives 35. (a) The sol particles migrate towards cathode.
straight line on plotting log x/m vs log p So they are positively charged. Hence,
as show below. anions would be effective in coagulation.
Greater is the valence of effective ion,
smaller will be its coagulating value.
1/n 36. (a) According to Langmuir adsorption
=
pe isotherm, the amount of gas adsorbed at
Slo
very high pressure reaches a constant
log x/m

limiting volume.
37. (d) Greater the surface area, greater the van der
Intercept = log k
waal forces of attraction and therefore at
O lesser concentration micelle formation will
log P
Å
take place. In case of CH3 (CH 2 )15 N
24. (c) Both are exothermic. (CH3)3 Br – due to greater chain length,
25. (b) Gelatin act as protective colloids. greater will be van der waal forces.
26. (c) Gold sol. h ave negative charge. So
Al2(SO4)3 is most effective for coagulation.
Surface Chemistry 231
38. (b) The motion of a liquid through a membrane 49. (c) According to Hardy-Schulze rule "The
under the influence of an applied electric amount of electrolyte required to coagulate
field is known as electro-osmosis. a fixed amount of a sol depends upon the
39. (b) In first case the given compounds have sign of charge and valency of the
same anion but different cations having flocculating ion."
different charge hence they will precipitate Thus, the coagulating power vary in the
negatively charged sol i.e. ‘A’. order.
In second case the given compounds have Al3+ > Zn2+ > Na+
similar cation but different anion with
different charge. Hence they will precipitate Coagulation power of AlCl3
50. (c)
positively charged sol. i.e. ‘B’. Coagulation power of NaCl
40. (c) Both adsorption and absorption can take Coagulation value of NaCl
place simultaneously. The term sorption is =
Coagulation value of AlCl3
used to describe both the processes.
52
41. (a) According to Langmuir's model of = = 559.13 » 560
adsorption of a gas on a solid surface, the 0.093
mass of gas adsorbed (x)per gram of the 51. (b) The lyophobic sols are less stable than
adsorbent (m) is directly proportional to the lyophilic sols. The lyophilic sols are thus
pressure of the gas (p) at constant used to protect the lyophobic sols. This
temperature. property of lyophilic sols is known as
42. (c) In lyophilic sol no charge present. protective action of lyophilic sols which is
43. (c) D.P. particles have great affinity towards represented by gold number.
D.M. in lyophilic colloids. 52. (b) For adsorption DS < 0 and for a
44. (d) Formation of micelles takes place below spontaneous change DG = – ve
CMC. hence DH should be highly negative which
45. (c) Liquid - liquid system is known as emulsion. is clear from the equation
46. (b) Critical temp. ­ Attractive force ­ DG = DH – TDS
Extent of adsorption ­. = – DH – T(– DS) = – DH + TDS
So if DH is highly negative DG will also be
(– ve)
x
53. (d) Tyndall effect is optical property, whereas
47. (a) other properties are electrical properties.
m
P Hence dependent on the charge on colloids
54. (c) Smoke is an example of solid particles
48. (c) No. of sodium lauryl sulphate molecules dispersed in gas.
(CH3 (CH 2 )11 SO 4- Na + ) in 1 litre solution 55. (b) Gold Number is the minimum amount of
= 10–3 × 6 × 1023 = 6 × 1020 lyophilic colloide in miligrams (starch in
No. of sodium lauryle sulphate molecules given case) which prevents the
per mm3 = 6 × 1014 flocculation of 10 ml gold sol by the
No. of colloidal particles per mm3 = 1013 addition of 1ml of 10% NaCl solution.
No. of molecules per colloidal particle \ Gold number = miligram of starch
= 0.025 × 103 mg
6 ´ 1014
= = 60 = 25
1013
EBD_7587
232 Chemistry Objective MCQs
56. (c) Fog has liquid as dispersed phase and 1
at low pressure, = 1
gas as dispersion medium. n
x
57. (d) No stabilising agent required for the \ µ p1
preparation of lyophilic sol. m
58. (d) Fe(OH)3 is positive sol. K3[Fe(CN)6] will 1
at high pressure, =0
provide [Fe(CN)6]3– for coagulation having n
highest magnitude of –ve charge among x
given options. µ p°
m
59. (d) Al(OH)3 is positively charged sol. K3PO4
i.e., the value of n varies between 0 to 1
has greater negative charge. Hence, it is
67. (a) Adsorption of a gas on solid is represented
most effective in coagulation
60. (c) Colloidal gold is prepared by Bredig's arc by following equilibria,
method. Gas (Adsorbate) + Solid (Adsorbent)
61. (d) Gold by Bredig’s method (Dispersion ˆˆ† Gas adsorbed on solid + Heat
‡ˆˆ
method) and by reduction method Initially adsorption increases with increase in
AuCl3 + Tannic acid ® Gold sol pressure at a particular temperature then got
62. (c) It is due to impact of molecules of dispersion slow. After attaining equilibrium, adsorption
medium on the colloidal particles. becomes independent of pressure.
63. (b) DG = DH – TDS. Adsorption is spontaneous 68. (c) Associate colloids decrease surface
process DG should be negative. tension and increase viscosity.
Hence DH – TDS should be negative. 69. (c) Hydrolysis of urea can be represented as
64. (c) The more the liquefiable nature of a gas, the
follows
more is the enthalpy of adsorption. Water
is more liquefiable. Urease
H 2 N - C - NH 2 + H 2O ¾¾¾¾® 2NH3 + CO 2
65. (a) The extent of adsorption increases with || (enzyme)
decrease in temperature. O
66. (a) According to Freun dlich adsorption Since it involves biological catalyst
isotherm (enzyme), so it is an example of biochemical
1 catalysis.
x
= kp n MnO
m 70. (d) 2 ® 2KCl + 3O
2KClO3 ¾¾¾¾ 2
General Principles and
20 Processes of Isolation of
Elements
1. Which one of the following is not a sulphide ore? 7. The most electropositive metals are isolated from
(a) Magnetite (b) Iron pyrites their ores by
(c) Copper glance (d) Sphalerite (a) high temperature reduction with carbon
2. Which of the following pair is incorrectly (b) self reduction
matched? (c) thermal decomposition
(d) electrolysis of fused ionic salts
(a) Magnetite – Fe3O4
8. Which of the following condition favours the
(b) Copper glance – Cu2S reduction of a metal oxide to metal?
(c) Calamine – ZnCO3
(d) Zincite – ZnS
3. The impurities associated with mineral used in (a) DH = +ve, TDS = + ve at low temperature
metallurgy are called collectively? (b) DH = +ve, TDS = – ve at any temperature
(a) Slag (b) Flux (c) Gangue (d) Ore (c) DH = –ve, TDS = – ve at high temperature
4. Galena is an ore of (d) DH = –ve, TDS = + ve at any temperature
(a) Pb (b) Hg (c) Zn (d) Cu 9. Which of the following statements regarding
electrolytic refining of copper is incorrect ?
5. The process of converting hydrated alumina into
anhydrous alumina is called
(a) roasting (b) smelting
(c) dressing (d) calcination (a) In this process anode is made up of impure
6. Which of the following statements regarding copper and pure copper strips are taken as
cathode.
metallurgy of iron is incorrect ? (b) Acidic or basic solution of copper sulphate
is used as electrolyte
(a) Reaction Fe3O 4 + 4CO ¾¾ ® 3Fe + 4CO 2 (c) Antimony, tellurium, silver and gold are
belongs to lower temperature range some of the metals deposits as anode mud
during this process
(500 – 800K) of the blast furnace.
(d) Zinc can be also refined by electrolytic
(b) Reaction FeO + CO ¾¾ ® Fe + CO 2 belongs refining method.
to higher temperature range (900 – 1500K) 10. If an impurity in a metal has a greater affinity for
of the blast furnace. oxygen and is more easily oxidised than the metal,
(c) The iron obtained from blast furnace is cast then the purification of metal may be carried out
iron with 3% carbon. by
(d) For reduction of iron oxide to occur DG of (a) Poling (b) Zone refining
the couple of following reactions should be (c) Electrolytic refining (d) Cupellation
negative 11. van Arkel method of purification of metals
involves converting the metal to a
1
FeO(s) ¾¾ ® Fe(s) + O 2 (g) (a) volatile stable compound
2 (b) volatile unstable compound
1 (c) non volatile stable compound
C(s) + O2 (g) ¾¾ ® CO(g)
2 (d) None of the above
EBD_7587
234 Chemistry Objective MCQs
12. In order to refine “blister copper” it is melted in a (c) sulphur is liberated at cathode, oxygen at
furnace and is stirred with green logs of wood. anode
The purpose is (d) oxygen is liberated at cathode, copper at
(a) to expel the dissolved gases in blister anode
copper.
18. Which one of the following is a mineral of iron ?
(b) to bring the impurities to surface and oxidize
(a) Malachite (b) Cassiterite
them.
(c) to increase the carbon content of copper. (c) Pyrolusite (d) Magnetite
(d) to reduce the metallic oxide impurities with 19. In the extraction of copper from its sulphide ore,
hydrocarbon gases liberated from the wood. the metal finally obtained by the reduction of
13. The value of Df Gº for formation of Cr 2 O3 is cuprous oxide with
– 540 kJmol–1 and that of Al2O3 is – 827 kJ mol–1
What is the value of DrG° for the reaction? (a) iron (II) sulphide (b) carbon monoxide
4 2 2 4 (c) copper (I) sulphide(d) sulphur dioxide
Al(s) + Cr2 O3 (s) ® Al2 O3 (s) + Cr(s).
3 3 3 3 20. Calamine is an ore of :
(a) zinc (b) aluminium
(c) iron (d) copper
(a) – 574 kJ mol–1 (b) –287kJ mol–1 21. Which one of the following ores is known as
(c) + 574 kJ mol –1 (d) +287kJ mol–1 Malachite:
14. Which of the following statement is not correct (a) Cu2O (b) Cu2S
about Ellingham diagram? (c) CuFeS2 (d) Cu(OH)2.CuCO3
(a) D G increases with an increase in
22. In the isolation of metals, calcination process
temperature
usually results in :
(b) It consists of plots of DfGº Vs T for formation
of oxides (a) metal hydroxide (b) metal sulphide
(c) A coupling reaction can be well expressed (c) metal oxide (d) metal carbonate
by this diagram 23. Cryolite is
(d) It express the kinetics of the reduction (a) Na 3AlF6 and used in the electrolysis of
process alumina for decreasing electrical
15. Iron is obtained on large scale from Fe2O3 by conductivity
(a) Reduction with CO(b) Reduction with Al (b) Na 3AlF6 and used in the electrolysis of
(c) Calcination (d) Passing H2 alumina for lowering the melting point of
16. Aluminothermic process is used for the extraction alumina
of metals, whose oxides are (c) Na 3AlF 6 and used in the electrolytic
(a) fusible purification of alumina
(b) not easily reduced by carbon (d) Na 3AlF6 and used in the electrolysis of
(c) not easily reduced by hydrogen
alumina
(d) strongly basic.
24. Froth floatation process is used for the metallurgy
17. When CuSO4 is electrolysed using platinum
electrodes, of
(a) chloride ores (b) amalgams
(c) oxide ores (d) sulphide ores
(a) copper is liberated at cathode, sulphur at 25. Which of the following metal is leached by
anode cyanide process
(b) copper is liberated at cathode, oxygen at
(a) Ag (b) Na (c) Al (d) Cu
anode
General Principles and Processes of Isolation of Elements 235
26. Which of the following statements, about the 32. Aluminium is extracted from alumina (Al2O3 ) by
advantage of roasting of sulphide ore before electrolysis of a molten mixture of
reduction is not true? (a) Al2O3 + HF + NaAlF4
(b) Al2O3 + CaF2 + NaAlF4
(a) The DG of of the sulphide is greater than (c) Al2O3 + Na3AlF6 + CaF2
those for CS2 and H2S. (d) Al2O3 + KF + Na3AlF6
33. Correct match is:
(b) The DG of is negative for roasting of
(a) Bayer's method - Na2CO3
sulphide ore to oxide. (b) Matte - 98% Cu2S+2% FeS
(c) Roasting of the sulphide to the oxide is (c) van Arkel method - AgI
thermodynamically feasible. (d) Thomas slag - Raw material for
(d) Carbon and hydrogen are suitable reducing cement industry
agents for reduction of metal sulphides. 34. Consider the following metallurgical processes:
27. Sulphide ores of metals are usually concentrated (I) Heating impure metal CO and distilling the
by froth flotation process. Which one of the resulting volatile carbonyl (b.p. 43°C) and
following sulphide ores offer an exception and is finally decomposition at 150°–200°C to get
concentrated by chemical leaching? the pure metal
(II) Heating the sulphide ore in air until a part
is converted to oxide and then furher
heating in the absence of air to let the oxide
(a) Galena (b) Copper pyrite
react with unchanged metal sulphide
(c) Sphalerite (d) Argentite
(III) Electrolysis of the molten electrolyte
28. Pb and Sn are extracted from their chief ore by
containing approximately equal amounts of
(a) carbon reduction and self reduction.
the metal chloride and NaCl to obtained the
(b) self reduction and carbon reduction.
metal
(c) electrolysis and self reduction. The processes used for obtaning magnesium,
(d) self reduction and electrolysis. nickel and copper are respectively:
29. Which of the following elements is present as
the impurity to the maximum extent in the pig
iron?
(a) Manganese (b) Carbon (a) (I), (II) and (III) (b) (II), (III) and (I)
(c) Silicon (d) Phosphorus (c) (III), (I) and (II) (d) (II), (I) and (III)
30. Which of the following pairs of metals is purified 35. When copper pyrites is roasted in excess of air,
by van Arkel method ? a mixture of CuO + FeO is formed. FeO is present
(a) Ga and In (b) Zr and Ti as impurity. This can be removed as slag during
(c) Ag and Au (d) Ni and Fe reduction of CuO. The flux added to form slag is
31. The following reactions take place in the blast (a) SiO2, which is an acidic flux
furnace in the preparation of impure iron. Identify (b) Limestone, which is a basic flux
(c) SiO2, which is the basic flux
the reaction pertaining to the formation of the
(d) CaO, which is a basic flux
slag. 36. One of the processes used for concentration of
(a) Fe2O3(s) + 3 CO(g) ®2 Fe (l) + 3 CO2 (g) ores is Froth floatation process. This process is
(b) CaCO3 (s) ®CaO (s) + CO2 (g) generally used for concentration of sulphide
ores. Sometimes in this process we add NaCN as
(c) CaO (s) + SiO2(s) ® CaSiO3(s)
a depressant. NaCN is generally added in case of
(d) 2C(s) + O2 (g) ®2 CO(g) ZnS and PbS minerals. What is the purpose of
EBD_7587
236 Chemistry Objective MCQs
addition of NaCN during the process of Froth (a) calcium (b) coke
floatation ? (c) Al-powder (d) sodium
(a) NaCN causes reduction by precipitation 42. Pyrolusite is a/an
(b) A soluble complex is formed by reaction (a) oxide ore (b) sulphide ore
between NaCN and ZnS while PbS forms (c) carbide ore (d) Not an ore
froth 43. Among the following statements the incorrect
(c) A soluble complex is formed by reaction one is
between NaCN and PbS while ZnS forms (a) Calamine and siderite are carbonates
froth (b) Argentite and cuprite are oxides
(d) A precipitate of Pb(CN)2 is produced while (c) Zinc blende and iron pyrites are sulphides
ZnS remain unaffected. (d) Malachite and azurite are ores of copper.
37. Which of the following statement(s) is/are 44. During smelting an additional substance is added
correct? which combines with impurities to form a fusible
product. It is known as
(i) Cast iron is used in the manufacture of
(a) slag (b) mud (c) gangue (d) flux
railway sleepers
45. Which of the following processes involve the
(ii) Wrought iron is used in the manufacture of
roasting process?
anchors, bolts, chains etc.
(a) ZnCO3 ® ZnO + CO2
(iii) Nickel steel is used in making pendulums.
(b) Fe2O3 + 3C ® 2Fe + 3CO
(a) Only (i) (b) (i) and (ii)
(c) 2PbS + 3O2 ® 2PbO + 2SO2
(c) (i), (ii) and (iii) (d) Only (iii)
(d) Al2O3 . 2H2O ® Al2O3 + 2H2O
38. A coupled reaction takes place as follow– 46. Before introducing FeO in blast furnace , it is
A + B ––® C + D DGº = + x kj
converted to Fe2O3 by roasting so that
D + E ––® F DGº = – y kj
(a) it may not be removed as slag with silica
for the spontaneity of reaction
(b) it may not evaporate in the furnace
A + B + E –––® C + F, which of the following
(c) presence of it may increase the m.pt. of charge
is correct? (d) None of these
47. In electrorefining of copper some gold is
(a) 2x = y (b) x < y
deposited as
(c) x > y (d) x = (y) × TDS
(a) anode mud (b) cathode mud
39. The form of iron obtained from blast furnace is:
(c) electrolyte (d) None of these
(a) Steel (b) Cast Iron 48. In the extraction of copper from its sulphide ore,
(c) Pig Iron (d) Wrought Iron the metal is finally obtained by the reduction of
40. Calcination is the process in which : cuprous oxide with :
(a) ore is heated above its melting point to expel (a) Copper (I) sulphide (Cu2S)
H2O or CO2 or SO2 (b) Sulphur dioxide (SO2)
(b) ore is heated below its melting point to expel (c) Iron sulphide (FeS)
volatile impurities (d) Carbon monoxide (CO)
(c) ore is heated above its melting point to 49. XCl2 (excess) + YCl2 ¾¾ ® XCl4 + Y ¯ ;
remove S, As and Sb as SO2, As2O3 and D 1
Sb2O3 respectively YO ¾¾¾® O2 + Y, Ore of Y would be:
>400° 2
(d) ore is heated below its melting point to expel
H2O or CO2
41. In Goldschmidt aluminothermic process which
(a) Siderite (b) Cinnabar
of the following reducing agents is used :
(c) Malachite (d) Hornsilver
General Principles and Processes of Isolation of Elements 237
50. Refractory materials are generally used in furnaces 57. The substance used as froth stabilisers in froth-
because floatation process is
(a) they possess great structural strength (a) Potassium ethyl xanthate
(b) they can withstand high temperature (b) Aniline
(c) they are chemically inert (c) Sodium cyanide
(d) they do not require replacement (d) Copper sulphate
51. In electro-refining of metal the impure metal is 58. When a metal is to be extracted from its ore and
made the anode and a strip of pure metal, the the gangue associated with the ore is silica, then
cathode, during the electrolysis of an aqueous (a) an acidic flux is needed
solution of a complex metal salt. This method (b) a basic flux is needed
cannot be used for refining of (c) both acidic and basic fluxes are needed
(a) silver (b) copper
(d) Neither of them is needed
(c) aluminium (d) sodium
59. Wh ich one of the following ores is not
52. When the sample of copper with zinc impurity is
concentrated by froth floatation process?
to be purified by electrolysis, the appropriate
(a) Copper pyrites (b) Pyrargyrite
electrodes are
(c) Pyrolusite (d) Zinc blende
cathode anode
(a) pure zinc pure copper 60. Which of the following reactions is an example
(b) impure sample pure copper for calcination process ?
(c) impure zinc impure sample
(d) pure copper impure sample. (a) 2Ag + 2HCl + (O ) ® 2AgCl + H 2 O
53. In the extraction of aluminium by Hall-Heroult (b) 2Zn + O 2 ® 2 ZnO
process, purified Al2O3 is mixed with CaF2 to (c) 2ZnS + 3O 2 ® 2ZnO + 2SO 2
(i) lower the melting point of Al2O3. (d) MgCO3 ® MgO + CO 2
(ii) increase the conductivity of molten mixture. 61. Heating of ore in the absence of air below its
(iii) reduce Al3+ into Al(s).
melting point is called
(iv) acts as catalyst.
(a) leaching (b) roasting
(c) smelting (d) calcination
(a) (i) and (ii) (b) (i), (ii) and (iii) 62. Heating pyrites to remove sulphur is called
(c) (iii) and (iv) (d) (ii), (iii) and (iv) (a) smelting (b) calcination
54. In Hall-Heroult process how much carbon anode (c) liquation (d) roasting
is burnt away to produce each 1kg of aluminium? 63. Complexes formed in the following methods are

(a) 0.3 kg (b) 0.5 kg (c) 1 kg (d) 0.1 kg I. Mond's process for purification of nickel.
55. Which of the oxide groups among the following II. Removal of lead poisoning from the body.
cannot be reduced by carbon? III. Cyanide process for extraction of silver.
(a) Cu2O, SnO2 (b) CaO, K2O IV. Froth flotation process for separation of
(c) PbO, Fe2O4 (b) Fe2O3, ZnO ZnS from galena ore by using depressant.
56. Extraction of zinc from zinc blende is achieved I II III IV
by
(a) Ni(CO)4 [Pb(EDTA]2– [Ag(CN)2]– [Zn(CN)2]
(a) roasting followed by self-reduction
(b) electrolytic reduction (b) Ni(CO)4 [Pb(EDTA]2– [Ag(CN)2]– [Zn(CN)4]2–
(c) roasting followed by reduction with carbon (c) Ni(CO)6 [Pb(EDTA)]4– [Ag(CN)2]– [Zn(CN)6]4–
(d) roasting followed by reduction with another (d) Ni(CO)4 [Pb(EDTA)]2– [Ag(CN)4]3– Zn(CN)4]2–
metal
EBD_7587
238 Chemistry Objective MCQs

64. The method of zone refining of metals is based 66. Purification of silicon element used in
on the principle of: semiconductors is done by
(a) Greater solubility of the impurity in the (a) zone refining (b) heating
molten state than in the solid (c) froth floatation (d) heating in vacuum
(b) Greater mobility of the pure metal than that 67. In the electrolysis of alumina to obtain aluminium
of the impurity metal, cryolite is added mainly to
(c) Higher melting point of the impurity than (a) lower the melting point of alumina
that of the pure metal (b) dissolve alumina in molten cryolite
(d) Greater noble character of the solid metal (c) remove the impurities of alumina
than that of the impurity (d) increase the electrical conductivity
65. D¦G– vs T plot in the Ellingham diagram slopes 68. Cassiterite is concentrated by
downward for the reaction (a) levigation
(b) electromagnetic separation
1
(a) Mg + O2 ® MgO (c) floatation
2
(d) liquefaction
1 69. Calcination is used in metallurgy for removal of ?
(b) 2Ag + O 2 ® Ag 2O (a) Water and sulphide
2
(b) Water and CO2
1 (c) CO2 and H 2S
(c) C + O2 ® CO
2 (d) H 2 O and H 2S
70. Which of the following metals is extracted by the
1 electrometallurgical method ?
(d) CO + O 2 ® CO 2
2 (a) Cu (b) Fe (c) Na (d) Ag

Answer KEYs
1 (a) 8 (d) 15 (a) 22 (c) 29 (b) 36 (b) 43 (b) 50 (b) 57 (b) 64 (a)
2 (d) 9 (b) 16 (b) 23 (b) 30 (b) 37 (c) 44 (d) 51 (d) 58 (b) 65 (c)
3 (c) 10 (d) 17 (b) 24 (d) 31 (c) 38 (b) 45 (c) 52 (d) 59 (c) 66 (a)
4 (a) 11 (a) 18 (d) 25 (a) 32 (c) 39 (b) 46 (a) 53 (a) 60 (d) 67 (a)
5 (d) 12 (d) 19 (a) 26 (d) 33 (b) 40 (d) 47 (a) 54 (b) 61 (d) 68 (b)
6 (c) 13 (b) 20 (a) 27 (d) 34 (c) 41 (c) 48 (a) 55 (b) 62 (d) 69 (b)
7 (d) 14 (d) 21 (d) 28 (b) 35 (a) 42 (a) 49 (b) 56 (c) 63 (b) 70 (c)
General Principles and Processes of Isolation of Elements 239

1. (a) The formula of magnetite is Fe3O4. Subtracting equation (ii) from equation (i)
2. (d) Zincite is ZnO. we have,
3. (c) Impurities associated with minerals are 4 2
called gangue or matrix. Al(s) + Cr2 O 3 (s),
3 3
4. (a) Galena is an ore of lead. It is PbS.
5. (d) Al 2 O3 .2H 2O ® Al 2 O 3 + 2H 2O 2 4
® Al 2 O3 (s) + Cr(s),
3 3
is calcination.
6. (c) The iron obtained from blast furnace is pig D r G° = -287kJ mol -1
iron with 4% carbon and impurities like S, P, 14. (d) Ellingham diagrams are based on thermo-
Mn etc., in small amount. dynamic concepts. It does not tell anything
7. (d) Most electropositive metals are obtained by about the kinetics of the reduction process.
electrolysis of their fused ionic salts.
15. (a) Fe 2 O 3 + 3CO ® 2Fe + 3CO 2
8. (d) DH = –ve, TDS = +ve at any temperature
(in blast furnace)
9. (b) During electrolytic refining of copper
16. (b) When reduction by carbon is not
electrolyte used is acidified solution of
satisfactory in case of metals having high
copper sulphate.
m.pt., aluminothermic process is used.
10. (d) Cupellation can be used in such a case.
17. (b) CuSO 4 Cu 2 + + SO 24 -
523K
11. (a) Ti + 2I2 ¾¾¾® TiI 4
Volatile H2O H + + OH -
stable compound
At cathode : Cu 2 + + 2e - ® Cu
1700K
¾¾¾¾
® Ti + 2I2
Pure metal
At anode : 4OH - ® 2H 2 O + O 2 + 4e -
12. (d) 2Cu2S + 3O2 ® 2Cu2O + 2SO2 18. (d) Magnetite – Fe3O4
Malachite – CuCO3 · Cu(OH)2
3Cu 2 O + CH 4 ® 6Cu + 2H 2 O + CO Cassiterite – SnO2.
(From green
logs of wood)
Pyrolusite – MnO2
13. (b) The two equation are: 19. (a) Highly electropositive metals like Al, K, Na
etc. are extracted by the electrolytic
4 2 reduction.
Al(s) + O 2 (g) ® Al 2 O3 (s),
3 3 zone refining method is used for obtaining
metals of high purity e.g. Ge
D f Gº = -827kJ mol-1 … (1)
Froth flotation process is suitable for
sulphide ores
4 2
Cr(s) + O 2 (g) ® Cr2O 3 (s), Cyanide process is used for the extraction
3 3
of gold.
D f Gº = -540kJ mol-1 … (2) 20. (a) ZnCO3 = calamine.
EBD_7587
240 Chemistry Objective MCQs
21. (d) Malachite is a copper carbonate hydroxide because the reduction took place by itself,
mineral with the formula Cu (OH)2.CuCO3. hence is known as self reduction.
22. (c) Calcination is the process of converting an D
2PbO + PbS ¾¾
® 3Pb + SO 2 ­
ore into its oxide by heating it strongly
below its melting point either in absence or D
PbSO 4 + PbS ¾¾
® 2Pb + 2SO 2 ­
limited supply of air. It is usually done to
convert metal carbonates and hydroxides 29. (b) Pig iron or cast iron contains 3 – 5% carbon
to their respective oxides. and varying amounts of Mn, Si, P and S
which makes the iron hard and brittle.
D 30. (b) Zr and Ti are purified by van Arkel method.
CaCO3 ¾¾® CaO + CO2 ­
Limestone Calcium 870K
oxide Zr(s) + 2I 2 (g) ¾¾¾® ZrI 4 (g)
D
CuCO3 .Cu(OH)2 ¾¾
® 2075K
Malachite
ZrI4 (g) ¾¾¾¾¾¾¾
® Zr(s) + 2I 2 (g)
Tugsten filament
2CuO + H 2 O ­ + CO2­
523K
Ti(s) + 2I2(s) ¾¾¾
23. (b) Na 3 AlF6 is cryolite and used in the ® TiI4(g)
electrolysis of alumina to lower the melting TiI4 ¾¾¾¾
1700 K
® Ti(s) + 2I (g)
point and increase electrical conductivity. 2
Pure titanium
24. (d) Froth floatation process is used for the
31. (c) In blast furnace at about 1270 K, calcium
concentration of sulphide ores.
carbonate is almost completely decomposed
25. (a) Ag is leached by cyanide process. to give CaO which acts as a flux and
26. (d) The sulphide ore is roasted to oxide before combines with SiO2 present as impurity
reduction because the DGof of most of the (gangue) in the ore to form calcium silicate
sulphides are greater than those of CS2 and (fusible slag)
H2S, therefore neither C nor H can reduce CaO(s) + SiO2 (s) ¾¾ ® CaSiO3 (s)
metal sulphide to metal. Further, the standard (basic flux) (acidic flux) (slag)
free energies of formation of oxide are much 32. (c) Fused alumina (Al2O3) is a bad conductor
less than those of SO2. Hence oxidation of of electricity. Therefore, cryolite (Na3AlF6)
m eta l s ul p h i d es t o m et a l ox i de i s and fluorspar (CaF2) are added to purified
thermodynamically favourable. alumina which not only make alumina a good
27. (d) Leaching is the selective dissolution of the conductor of electricity but also reduce the
desired mineral leaving behind the impurities melting point of the mixture to around
in a suitable dissolving agent e.g., 1140 K.
Argentitie or Silver glance, Ag2S is an ore 33. (b) Roasted mass obtained from roasting step
of silver. Silver is extracted from argentite is called matte. (98% Cu2S + 2% FeS)
by the mac-Arthur and Forest process 50°C
34. (c) (I) Ni + 4CO ¾¾¾® [ Ni(CO)4 ]
(leaching process). (impure) (volite)
Ag 2S + 4NaCN ® 2Na[Ag ( CN )2 ] + Na 2S
230°C
¾¾¾¾
® Ni + 4CO ­
4Au + 8KCN + 2H 2O + O2 ® (pure)

4K[Au ( CN )2 ] + 4KOH 3 D
(II) Cu 2S + O 2 ¾¾® Cu 2O + SO 2
2
28. (b) PbO & PbSO4 get reduced by PbS itself high
which is already present in mixture so Cu 2S + 2Cu 2O ¾¾¾® 6Cu + SO2
temp.
General Principles and Processes of Isolation of Elements 241

electrolysis 48. (a) Cuprous oxide formed during roasting of


(III) MgCl2 (s) ¾¾¾¾¾ ®
cuprous sulphide is mixed with few amount
Mg 2+ (l) + 2 Cl - (l) of cuprous sulphide and heated in a
reverberatory furnace to get metallic copper.
At cathode :
2Cu 2O + Cu 2S ® 6Cu + SO2
Mg 2+ 2e - ¾¾
® Mg(s)
At anode : 49. (b) SnCl2 + HgCl 2 ¾¾® SnCl4 + Hg
(XCl2 ) (YCl 2 ) (XCl4 ) (Y)
2Cl- (l) ¾¾
® Cl 2 (g) + 2 e- 1
D
HgO + ¾¾¾¾® Hg + O2
35. (a) FeO + SiO2 ® FeSiO3. > 400°C 2
HgS : Cinnabar
36. (b) NaCN + ZnS ¾¾
® Na 2 [Zn(CN) 4 ] 50. (b) Refractories are the substances which can
Soluble complex
withstand very high temperature without
A layer of this zinc complex is formed on the melting or becoming soft.
surface of ZnS and due to this ZnS is 51. (d) Na reacts vigorously with water (exothermic
prevented from the froth formation while process)
PbS form froth. (i.e., NaCN is added as 52. (d) Pure metal always deposits at cathode.
depressant for ZnS) 53. (a) In the extraction of aluminium by Hall-
37. (c) All three statements are correct Heroult process purified Al2O3 is mixed with
38. (b) For a spontaneous reaction , D Gº must be CaF2 to lower the melting point by Al2O3
negative and it can be possible only in this and increase the conductivity of molten
case when x < y mixture.
39. (b) The iron obtained from blast furnace is in 54. (b) For each kg of Al produced, about 0.5 kg of
the form of cast iron. carbon anode is burnt away.
40. (d) Calcination is a process of heating a 55. (b) Ca and K are strong reducing agents,
substance to a high temperature but below hence their oxides can not be reduced
the melting or fusion point, causing loss of with carbon.
moisture, reduction or oxidation and 56. (c) Extraction of Zn from zinc blends is
dissociation into simpler substances. ach ieved by roasting followed by
reduction with carbon. Reactions involved
41. (c) Reduction by powdered aluminium is
are :
known as Gold-Schmidt aluminothermic
heat
process. This process is employed in cases 2ZnS + 3O2 ¾¾¾
® 2ZnO + 2SO2 (Roasting)
where metals have very high m.p. and are to heat
ZnO + C(Coke) ¾¾¾ ® Zn + CO (Reduction)
be extracted from their oxides.
57. (b) Froth stabilizers cresols, aniline are added
42. (a) MnO2 is pyrolusite (oxide ore). to stabilize the froth and enhance the non-
43. (b) Cuprite is Cu2O and Argentite is Ag2S. wettability of the mineral particles.
44. (d) Flux is added which combines with 58. (b) Since silica is acidic impurity the flux must
impurities to form slag. be basic.
45. (c) Roasting involves heating the ore strongly CaO + SiO2 ® CaSiO3
in presence of excess of air (O2).
59. (c) Pyrolusite is MnO2. Hence not concentrated
46. (a) FeO is capable forming slag with SiO2
by froth floatation process.
SiO 2 + FeO ® FeSiO 3 60. (d) Decomposition of carbonates and hydrated
47. (a) Gold is deposited as anode mud. oxides.
EBD_7587
242 Chemistry Objective MCQs
61. (d) It is definition of calcination.
1
62. (d) Pyrites are sulphur ores and are converted 65. (c) C (s ) + O2 (g) ® CO(g); DS increases.
2
into oxide by roasting.
63. (b) Hence, as the temperature increases, TDS
(I) Mond's process increases and hence DG (DH – TDS)
decreases. In other words, the slope of the
50°C
Ni + 4CO ¾¾¾® éë Ni (CO )4 ùû curve for formation of CO decreases.
Impure
Volatile However, for all other oxides, it increases.
complex
66. (a) Silicon is purified by zone refining . Metals
200°C of high purity are always purified by zone
¾¾¾¾
® Ni + 4CO ­
Pure refining.
(II) Removal of lead poisoning 67. (a) Fused alumina (Al2O3) is a bad conductor
2,
of electricity. Therefore, cryolite (Na3AlF6)
Pb 2∗ ∗ EDTA 4, ‡ˆˆ
ˆˆ† é Pb ∋ EDTA (ù
ë û and fluorspar (CaF2) are added to purified
(III) Cyanide process for extraction of silver alumina which not only make alumina a
good conductor of electricity but also
1
ˆˆ†
2Ag ∗ 4NaCN ∗ H 2O ∗ O2 ‡ˆˆ reduce the melting point of the mixture to
2 around 1140 K.
2Na éë Ag ( CN )2 ùû + 2NaOH 68. (b) Cassiterite contains the magnetic impurities
of FeSO4 and concentrated by electromagnetic
(IV) Separation of ZnS from galena by using
separation.
depressant in froth flotation process
69. (b) Calcination is used for removal of volatile
64. (a) Zone refining is based on the difference in
impurities and decompose carbonates.
solubility of impurities in molten and solid
state of the metal. This method is used for 70. (c) Na is obtained by elctrolytic reduction being
obtaining metals of very high purity. electropositive in nature.
The p-Block Elements
21 (Group 15, 16, 17 & 18)
1. Incorrect statement about PH3 is: (b) almost pure p-orbitals are used for M-H
(a) It is produced by hydrolysis of Ca3P2. bonding.
(b) It gives black ppt. (Cu 3P2) with CuSO4 (c) the bond energies of M-H bonds increases.
solution. (d) the bond pairs of electrons become nearer
(c) Spontaneously burns in presence of P2H4. to the central atom.
(d) It does not react with B2H6. 8. Brown colour in HNO3 can be removed by
2. The number of P–OH bonds and the oxidation (a) adding Mg powder.
state of phosphorus atom in pyrophosphoric acid (b) boiling the acid.
(c) passing NH3 through acid.
(H4P2O7) respectively are : (d) passing air through warm acid.
(a) four and four (b) five and four 9. In nitroprusside ion, the iron and NO exist as
(c) five and five (d) four and five Fe2+ and NO+ rather than Fe3+ and NO. These
forms can be differentiated by
3. P2H4 can be removed from phosphine containing
(a) estimating the concentration of iron.
traces of it (b) measuring the concentration of CN - .
(a) by passing impure PH3 gas through a (c) measuring the solid state magnetic moment.
freezing mixture. (d) thermally decomposing the compound.
(b) by passing the impure PH3 gas through HI 10. A deep brown gas is formed by mixing two
and then its treatment with KOH (aq). colourless gases which are
(c) by both (a) and (b). (a) NO2 and O2 (b) N2O and NO
(d) by none of these. (c) NO and O2 (d) NH3 and HCl
11. Which of the following compounds does not
4. NH 4 ClO4 + HNO 3 (dil.) ¾¾ ® HClO 4 + [X] exist?
D
[X] ¾¾
® Y(g) (a) AsCl5 (b) SbCl3 (c) BiCl5 (d) SbCl5
[X] and [Y] are respectively – 12. The brown ring test for NO -2 and NO 3- is due
(a) NH4NO3 & N2O (b) NH4NO2 & N2 to the formation of complex ion with a formula
(c) HNO4 & O2 (d) None of these (a) [Fe(H2O)6]2+ (b) [Fe(NO)(CN)5]2+
5. For H3PO3 and H3PO4, the correct choice is (c) [Fe(H2O)5NO] 2+ (d) [Fe(H2O)(NO)5]2+
13. Which of the following is a cyclic phosphate ?

(a) H3PO3 is dibasic and reducing (a) H3P3O10 (b) H6P4O13


(b) H3PO4 is diabasic and non-reducing (c) H5P5O15 (d) H7P5O16
(c) H3PO4 is tribasic and reducing 14. An element (X) forms compounds of the formula
(d) H3PO4 is tribasic and non reducing XCl3, X2O5 and Ca3X2 but does not form XCl5.
6. One mole of calcium phosphide on reaction with Which of the following is the element X ?
excess of water gives (a) B (b) Al (c) N (d) P
(a) one mole of phosphine. 15. P4O10 is not used to dry NH3 gas because
(b) two moles of phosphoric acid.
(c) two moles of phosphine.
(d) one mole of phosphorus oxide.
7. In nitrogen family, the H-M-H bond angle in the (a) P4O10 reacts with moisture in NH3.
hydrides gradually becomes closer to 90º on (b) P4O10 is not a drying agent.
going from N to Sb. This shows that gradually (c) P4O10 is acidic and NH3 is basic.
(a) the basic strength of the hydrides increases. (d) P4O10 is basic and NH3 is acidic.
EBD_7587
244 Chemistry Objective MCQs
16. Pick out the wrong statement. 24. Consider the following sequence of reaction.
(a) Nitrogen has the ability to form pp-pp bonds 2 ® N O
with itself. ® [X ] ¾¾¾
Na + NH3 (g) ¾¾
Heat
(b) Bismuth forms metallic bonds in elemental [Y ] ¾¾¾® [Z ]
state. Gas Pure
Identify (Z) gas :
(c) Catenation tendency is higher in nitrogen
when compared with other elements of the (a) N2 (b) NH3 (c) O2 (d) H2
same group. 25. Among the following compounds, which on
(d) Nitrogen has higher first ionisation enthalpy heating do not produce N2?
when compared with other elements of the (a) (NH4)2 Cr2O7 (b) NH4Cl + NaNO2
same group. (c) NH4Cl + CaO (d) Ba(N3)2
17. Which of the following is incorrect for white and 26. A + H 2O ¾¾ ® B + HCl
red phosphorus ? B + H 2O ¾¾® C + HCl
(a) They are both soluble in CS2. Compound (A), (B) and (C) will be respectively :
(b) They can be oxidised by heating in air. (a) PCl5 , POCl3 , H3PO3
(c) They consist of the same kind of atoms.
(d) They can be converted into one another. (b) PCl5 , POCl3 , H3PO4
18. BCl 3 is a planar molecule whereas NCl 3 is (c) SOCl 2 , POCl3 , H3PO3
pyramidal because
(d) PCl3 , POCl3 , H3PO4
(a) BCl3 has no lone pair of electrons but NCl 3
has a lone pair of electrons. 27. A white precipitate is obtained on hydrolysis of
(b) B—Cl bond is more polar than N—Cl bond. (a) PCl5 (b) NCl3 (c) BiCl3 (d) AsCl3
(c) nitrogen atom is smaller than boron atom. 28. Superphosphate of lime is obtained from the
(d) N—Cl bond is more covalent than B—Cl reaction of :
bond. (a) Bones with gypsum.
(b) Calcium phosphate with sulphuric acid.
19. A metal X on heating in nitrogen gas gives Y. Y
(c) Calcium phosphate with HCl.
on treatment with H2O gives a colourless gas
(d) Calcium carbonate with phosphoric acid.
which when passed through CuSO4 solution
29. Phosphine is not obtained by which of the
gives a blue colour. Y is
following reaction?
(a) Mg(NO3)2 (b) Mg3N2
(a) White P is heated with NaOH.
(c) NH3 (d) MgO (b) Red P is heated with NaOH.
20. Which of the following oxy-acids has the (c) Ca3P2 reacts with water.
maximum number of hydrogens directly attached (d) Phosphorus trioxide is boiled with water.
to phosphorus? 30. Which of the following statements is not true ?
(a) H4P2O7 (b) H3PO2 (a) NO2 can be prepared by heating Pb(NO3)2.
(c) H3PO3 (d) H3PO4 (b) NO2 is red - brown gas.
21. Number of sigma bonds in P4O10 is (c) NO2 is diamagnetic.
(a) 6 (b) 7 (c) 17 (d) 16 (d) NO2 readily dimerises to N2O4.
22. Which of the following compound has a P–P 31. The deep blue colour produced on adding excess
bond? of ammonia to copper sulphate is due to presence
(a) H4P2O5 (b) (HPO3)3 of
(c) H4P2O6 (d) H4P2O7 (a) Cu2+ (b) [Cu(NH 3 ) 4 ] 2 +
23. Which one of the following does not have a
2+
pyramidal shape? (c) [Cu(NH3 )6 ] (d) [Cu(NH3 )2 ]2+
(a) (CH3)3N (b) (SiH3)3N 32. Conc. HNO3 is heated with P2O5 to form :
(c) P(CH3)3 (d) P(SiH3)3 (a) NO2 (b) NO (c) N2O5 (d) N2O
The p-Block Elements (Group 15, 16, 17 & 18) 245
33. From the following information 40. Which of the following is not correctly matched?
X + H2SO4 ® Y (a colourless and irritating gas) (a) SF4 – gas (b) SeF4 – liquid
Y+K2Cr2O7+H2SO4 ® (green coloured solution) (c) TeF4 – solid (d) SF6 – solid
Identify the pair X and Y. 41. Which of the following on thermal decomposition
– gives oxygen gas ?
(a) Cr, HCl (b) SO32- , SO2
(a) Ag2O (b) Pb3O4
(c) S2–, H2S (d) CO32 - , CO2 (c) PbO2 (d) All of these
42. When sugar is treated with conc. sulphuric acid,
34. H2SO4 and H2SO3 can be distinguished by the
the sugar is charred. In this process, sugar is
addition of
(a) reduced (b) oxidized
(a) magnesium powder
(b) NaHSO4 solution (c) sulphonated (d) dehydrated
(c) FeCl3 solution 43. The number of S—S bonds in SO3, S2 O32-
(d) litmus solution S2 O62- and S2O82- respectively are
35. Which of the following is the key step in the
manufacture of sulphuric acid?
(a) Burning of sulphur or sulphide ores in air to
(a) 1, 0, 0, 1 (b) 1, 0, 1, 0
generate SO2.
(b) Conversion of SO2 to SO3 by the reaction (c) 0, 1, 1, 0 (d) 0, 1, 0, 1
with oxygen in presence of catalyst. 44. Correct order of O – O bond length (increasing)
(c) Absorption of SO3 in H2SO4 to give oleum. in O2, H2O2 and O3 is
(d) Both (b) and (c). (a) H2O2 < O3 < O2 (b) O2 < O3 < H2O2
36. FeCl3 solution on reaction with SO2 changes to (c) O3 < O2 < H2O2 (d) O3 < H2O2 < O2
(a) FeCl2 (b) Fe2(SO4)3 45. Iron sulphide is heated in air to form A, an oxide
(c) Fe2(SO3)3 (d) FeSO4 of sulphur. A is dissolved in water to give an
37. Hydrolysis of one mole of peroxodisulphuric acid acid. The basicity of this acid is
produces (a) 2 (b) 3 (c) 1 (d) zero
46. The shape of the molecule SF3Cl3 is
(a) trigonal bipyramidal
(a) two moles of sulphuric acid. (b) cubic
(b) two moles of peroxomonosulphuric acid. (c) octahedral
(c) one mole of sulphuric acid and one mole of (d) tetrahedral
peroxomonosulphuric acid.
47. What are the products formed when chlorine is
(d) one mole of sulphuric acid, one mole of
passed through aqueous hypo solution?
peroxomonosulphuric acid and one mole of
(a) Na2SO3 + HCl + S
hydrogen peroxide.
38. Concentrated H2SO4 is not used to prepare HBr (b) Na2SO3 + SO3 + HCl
from KBr because it (c) Na2SO4 + HCl + S
(a) oxidizes HBr. (d) Na2SO4 + HCl + SO2
(b) reduces HBr. 48. Which of the following is incorrect?
(c) causes disproportionation of HBr.
(d) reacts too slowly with KBr.
39. Which reaction represents the oxidizing behaviour
of H2SO4? (a) M.p of monoclinic sulphur > m.p. of
rhombic sulphur.
(a) 2PCl5 ∗ H 2SO 4 ® 2POCl3 ∗ 2HCl ∗ SO 2 Cl 2
(b) Specific gravity of rhombic sulphur >
(b) 2NaOH + H 2SO 4 ® Na 2SO 4 + 2H 2O specific gravity of monoclinic sulphur.
(c) NaCl + H 2SO 4 ® NaHSO 4 + HCl (c) Monoclinic sulphur is stable below 369 K.
(d) Both rhombic sulphur and monoclinic
(d) 2HI + H 2SO 4 ® I 2 + SO 2 + 2H 2 O sulphur have S8 molecules.
EBD_7587
246 Chemistry Objective MCQs
49. Which of the following form of the sulphur shows 57. Oxidation of thiosulphate with iodine gives
paramagnetic behaviour ? (a) sulphate ion (b) sulphite ion
(a) S8 (b) S6 (c) tetrathionate ion (d) sulphide ion
(c) S2 (d) All of these 58. Which of the following is correct about the
50. Which of the following has pp- dp bonding ? reaction?
heat
3NaClO ¾¾¾ ® NaClO3 + 2NaCl
(a) NO 3- (b) SO 32 -
(a) It is disproportionation reaction
(c) BO 33- (d) CO 32- (b) Oxidation number of Cl decreases as well as
51. Sulphuric acid reacts with PCl5 to give increases in this reaction
(a) thionyl chloride (c) This reaction is used for the manufacture of
(b) sulphur monochloride halates
(c) sulphuryl chloride (d) All of these
(d) sulphur tetrachloride 59. The reaction of KMnO4 and HCl results in
52. By which of the following SO2 is formed? (a) oxidation of Mn in KMnO4 and production
of Cl2.
(a) Reaction of dil. H2SO4 with O2
(b) reduction of Mn in KMnO4 and production
(b) Hydrolysis of dil. H2SO4
of H2.
(c) Reaction of conc. H2SO4 with Cu
(c) oxidation of Mn in KMnO4 and production
(d) None of these
of H2.
53. The acid which has a peroxy linkage is
(d) reduction of Mn in KMnO4 and production
(a) Sulphurous acid (b) Pyrosulphuric acid
of Cl2.
(c) Dithionic acid (d) Caro’s acid 60. The correct order of reactivity of halogens with
54. Excess of KI reacts with CuSO4 solution and then alkalies is
Na2S2O3 solution is added to it. Which of the (a) F > Cl > Br > I (b) F < Cl > Br < I
statements is incorrect for this reaction ? (c) F < Cl < Br < I (d) F < Cl < Br > I
(a) Na2S2O3 is oxidised 61. One mole of fluorine is reacted with two moles
(b) CuI2 is formed of hot and concentrated KOH. The products
(c) Cu2I2 is formed formed are KF, H2O and O2. The molar ratio of
(d) Evolved I2 is reduced KF, H2O and O2 respectively is
55. Of the interhalogen AX3 compounds, ClF3 is
(a) 1 : 1 : 2 (b) 2 : 1 : 0.5
most reactive but BrF3 has higher conductance
(c) 1 : 2 : 1 (d) 2 : 1 : 2
in liquid state. This is because 62. Chlorine cannot displace
(a) fluorine from NaF
(a) BrF3 has higher molecular mass. (b) iodine from NaI
(b) ClF3 is more volatile. (c) bromine from NaBr
(c) BrF3 dissociates into BrF2+ and BrF–4 most (d) None of these
easily. 63. When chlorine reacts with cold and dilute
(d) Electrical conductance does not depend on solution of sodium hydroxide, the products
concentration. obtained are
56. Which of the following is the best description (a) Cl– + ClO– (b) Cl– + ClO -2
for the behaviour of bromine in the reaction given (c) Cl– + ClO 3- (d) Cl– + ClO -4
below ?
64. A greenish yellow gas reacts with an alkali metal
H 2 O + Br2 ® HOBr + HBr hydroxide to form a halate which can be used in
(a) Proton acceptor only fire works safety matches. The gas and halate
(b) Both oxidized and reduced respectively are
(c) Oxidized only (a) Br2, KBrO3 (b) Cl2, KClO3
(d) Reduced only (c) I2, NaIO3 (d) Cl2, NaClO3
The p-Block Elements (Group 15, 16, 17 & 18) 247
65. In the reaction 74. Trigonal bipyramidal geometry is shown by :
+ (a) XeO3 (b) XeO3F2
® H 2 NO 3 + F - base is
HNO 3 + HF ¾¾
(c) FXeOSO2F (d) [XeF8]2–
(a) HF (b) HNO3
75. XeO4 molecule is tetrahedral having :
(c) HF and HNO3 (d) None of these
(a) Two pp – dp bonds
66. The aqueous solution of which of the following
(b) One pp – dp bonds
has maximum pH ? (c) Four pp – dp bonds
(a) NaClO (b) NaClO2 (d) Three pp – dp bonds
(c) NaClO3 (d) NaClO4 76. The compound of xenon with zero dipole moment
67. Which one of the following oxides of chlorine is is
obtained by passing dry chlorine over silver (a) XeO3 (b) XeF4
chlorate at 90°C ? (c) XeOF4 (d) XeO2
(a) Cl2O (b) ClO3 (c) ClO2 (d) ClO4 77. XeF6 dissolves in anhydrous HF to give a good
68. Identify the incorrect statement among the conducting solution which contains:
following (a) H+ and XeF7- ion
(a) Br 2 reacts with hot and strong NaOH
solution to give NaBr and H2O. (b) HF2– and XeF5+ ions
(b) Ozone reacts with SO2 to give SO3. (c) HXeF+6 and F– ions
(c) Silicon reacts with NaOH(aq) in the presence (d) none of these
of air to give Na2SiO3 and H2O. 78. Incorrect statement regarding following
(d) Cl2 reacts with excess of NH3 to give N2
reactions is:
and HCl.
69. The least number of oxyacids are formed by : +Excess H2O
(a) Chlorine (b) Nitrogen 'X' + HF
(c) Fluorine (d) Sulphur XeF6 ¾
70. Chlorine water on standing loses its colour and +2H 2O
'Y' + HF
forms :
(a) 'X' is explosive
(a) HCl only (b) HCl and HClO2
(c) HCl and HOCl (d) HOCl and HOCl2 (b) 'Y' is an oxyacid of xenon
71. Shapes of certain interhalogen compounds are (c) Both are example of non-redox reaction
stated below. Which one of them is not correctly (d) XeF6 can undergo partial hydrolysis.
stated? 79. In the clathrates of xenon with water, the nature
(a) IF7 : pentagonal bipyramidal of bonding between xenon and water molecule
is
(b) BrF5 : trigonal bipyramidal
(a) covalent
(c) BrF3 : planar T-shaped (b) hydrogen bonding
(d) ICI3 : planar dimeric (c) co-ordinate
72. Electron gain enthalpy with negative sign of (d) dipole-induced dipole interaction
fluorine is less than that of chlorine due to : 80. XeF4 on partial hydrolysis produces
(a) High ionization enthalpy of fluorine (a) XeF4 (b) XeOF2
(b) Smaller size of chlorine atom (c) XeOF4 (d) XeO3
(c) Smaller size of fluorine atom 81. Which one of the following is correct pair with
(d) Bigger size of 2p orbital of fluorine respect to molecular formula of xenon compound
73. Which of the following xenon-oxo compounds and hybridization state of xenon in it?
may not be obtained by hydrolysis of xenon (a) XeF4, sp3 (b) XeF2, sp
(c) XeF2, sp3d (d) XeF4, sp2
fluorides?
82. Which of the following noble gas is least
(a) XeO2F2 (b) XeOF4 polarisable?
(c) XeO3 (d) XeO4 (a) He (b) Xe (c) Ar (d) Ne
EBD_7587
248 Chemistry Objective MCQs

83. What are the products formed in the reaction of 87. The reason for not storing XeF6 in a glass or a
xenon hexafluoride with silicon dioxide? quartz vessel is that
(a) XeSiO4 + HF (b) XeF2 + SiF4
(c) XeOF4 + SiF4 (d) XeO3 + SiF2 (a) it forms an explosive having the formula
84. Which statement about noble gases is not XeO2F2.
correct? (b) it forms an explosive having the formula
(a) Xe forms XeF6. XeOF4.
(c) it forms XeO2 which is explosive substance.
(b) Ar is used in electric bulbs.
(d) it forms XeO64– which is explosive in nature.
(c) Kr is obtained during radioactive
88. SbF5 reacts with XeF4 to form an adduct. The
disintegration.
shapes of cation and anion in the adduct are
(d) He has the lowest b.p. among all the noble
respectively:
gases.
85. Density of nitrogen gas prepared from air is
slightly greater than that of nitrogen prepared
(a) square planar, trigonal bipyramidal
by chemical reaction from a compound of
(b) T-shaped, octahedral
nitrogen due to the presence of
(c) square pyramidal, octahedral
(a) argon
(d) square planar, octahedral
(b) carbon dioxide
89. Out of (i) XeO3 (ii) XeOF4 and (iii) XeF6, the
(c) some N3 molecules analogous to O3.
molecules having same number of lone pairs on
(d) greater amount of N2 molecules derived from
Xe are
N-15 isotope.
86. The formation of O2+[PtF6]– is the basis for the (a) (i) and (ii) only (b) (i) and (iii) only
formation of xenon fluorides. This is because (c) (ii) and (iii) only (d) (i), (ii) and (iii)
(a) O2 and Xe have comparable sizes. 90. The geometry of XeOF4 by VSEPR theory is :
(b) both O2 and Xe are gases. (a) pentagonal planar
(c) O2 and Xe have comparable ionisation (b) octahedral
energies. (c) square pyramidal
(d) Both (a) and (c). (d) trigonal bipyramidal

Answer KEYs
1 (d) 10 (c) 19 (b) 28 (b) 37 (c) 46 (c) 55 (c) 64 (b) 73 (d) 82 (a)
2 (d) 11 (c) 20 (b) 29 (b) 38 (a) 47 (c) 56 (b) 65 (b) 74 (b) 83 (c)
3 (c) 12 (c) 21 (d) 30 (c) 39 (d) 48 (c) 57 (c) 66 (a) 75 (c) 84 (c)
4 (a) 13 (c) 22 (c) 31 (b) 40 (d) 49 (c) 58 (d) 67 (c) 76 (b) 85 (a)
5 (a) 14 (c) 23 (b) 32 (c) 41 (d) 50 (b) 59 (d) 68 (d) 77 (b) 86 (d)
6 (c) 15 (c) 24 (a) 33 (b) 42 (d) 51 (c) 60 (a) 69 (c) 78 (b) 87 (b)
7 (b) 16 (c) 25 (c) 34 (c) 43 (c) 52 (c) 61 (b) 70 (c) 79 (d) 88 (b)
8 (d) 17 (a) 26 (b) 35 (b) 44 (b) 53 (d) 62 (a) 71 (b) 80 (b) 89 (d)
9 (c) 18 (a) 27 (c) 36 (a) 45 (a) 54 (b) 63 (a) 72 (c) 81 (c) 90 (c)
The p-Block Elements (Group 15, 16, 17 & 18) 249

1. (d) PH3 (Lewis base) can react with B2H6 (Lewis


6. (c) Ca 3 P2 + 6 H 2 O ® 3Ca (OH ) 2 + 2PH 3
acid). 1 mole 2 moles

O O 7. (b) With the decrease in the electronegativity


2. (d)
of central atom the bond angle decreases
P P 8. (d) Nitric acid decomposes to give NO2 which
HO O OH
is brown
OH OH
4HNO 3 ® 4 NO 2 + 2H 2 O + O 2
Pyrophosphoric acid (H4P2O7)
When air is passed through acid the
Oxidation State : reaction proceeds towards left hand side
Each P atom is bound to one oxygen = –1 and brown colour diminishes.
2OH = –1 × 2 = – 2 9. (c) The nitroprusside ion is [Fe(CN)6NO+]2–.
1 Oxygen = – 2 The magnetic moment measurements reveal
Total = –5 the presence of 4 unpaired electrons in Fe
P = +5. which must be then in Fe2+ (3d6) and not
3. (c) Both the methods (a) and (b) can be used. Fe3+(3d5)
When passed through freezing mixture the 10. (c) 2NO + O 2 ® 2 NO 2 brown
P2H4 present condenses and pure PH3 is
11. (c) Due to inert pair effect the lower O.S. is
obtained.
more common at the bottom of group
When passed through HI, PH3 is absorbed
12. (c) [Fe(H2O)5NO]2+ ion is formed
forming PH4I.PH4I when treated with KOH
13. (c) H5P5O15 or (HPO3)5. It is metaphosphoric
(aq) yields pure phosphine.
acid which is a cyclic phosphate.
PH 3 + HI ¾¾
® PH 4 I 14. (c) Nitrogen can form NCl3, N2O5 Ca3N2 and
PH 4 I + KOH(aq) ¾¾® KI + H 2 O + PH 3 ­ not NCl5 since it has no d atomic orbitals in
4. (a) NH 4 ClO 4 +HNO3 ® HClO 4 + NH 4 NO3 valence shell
(X)
D
NH 4 NO3 ¾¾
® N 2O + 2H 2 O 15. (c) P4 O10 + 2H 2 O ¾¾
® 4HPO3
(Y) Acidic meta phosphoric
5. (a) The structure of H3PO3 is as given below. acid

H
NH
P ¾¾¾
3
® NH 4 PO3
O OH basic meta ammonium
phosphate
OH
There are only two –OH groups and hence 16. (c) Catenation tendency is higher in
it is dibasic. The oxidation number of P in phosphorus when compared with other
this acid is +3, whereas P may have +5
elements of same group.
oxidation state also. Therefore, H3PO3 can
be oxidised which means H3PO 3 is a
reducing agent.
EBD_7587
250 Chemistry Objective MCQs

17. (a) Both white and red phosphorus are not 22. (c) H4P2O6 has P–P linkage
soluble in CS2 only white phosphorus is
soluble in CS2. O O
|| ||
18. (a) In BCl3 there is sp2 hybridisation (planar) HO – P –– P – OH
whereas in NCl3 hybridisation is sp3 hence | |
OH OH
shape is pyramidal

19. (b) D 6H O 23. (b) In case of N(SiH 3 ) 3, N atom is sp 2


3Mg + N 2 ¾¾® Mg 3 N 2 ¾¾¾®
2

hybridised, the lone pair is present in 2p


3Mg(OH)2 + 2NH3 ; orbital and it is transferred to empty d orbital
CuSO 4 + 4 NH 3 ® [Cu ( NH 3 ) 4 ]SO 4 of Si forming dp – pp bond. Hence nitrogen
deep blue colour with sp2 hybridization has trigonal planar
20. (b) shape.

OH OH D
24. (a) Na + NH3 ( g ) ¾¾® NaNH 2
(a) H4P2O7 ÞO P O P O [X ]

Pyrophosphoric acid OH OH N O
2 ® NaN ¾¾ D
¾¾¾ 3 ® N2 ­
[Y ] [Z ]
25. (c)
O
P D
(a) (NH 4 )2 Cr2 O7 ¾¾® Cr2O7 + N 2 + H 2 O
(b) H3PO2 Þ HO
H H
(b) D
Hypophosphorous acid NH4 Cl + NaNO2 ¾¾® NH4 NO2 + NaCl
O (c) 2NH 4Cl + CaO ¾¾D
® CaCl 2 + 2NH3 ­ + H2 O
D
(c) H3PO3 Þ HO – P – OH (d) NH 4 NO 2 ¾¾® N 2 ­ +2H 2 O
D
H Ba ( N3 )2 ¾¾® Ba + 3N 2 ¯
Phosphorous acid
26. (b) PCl5 + H 2O ¾¾
® POCl3 + 2HCl
O (A) ( B)
(d) H3PO4 Þ HO – P – OH
POCl3 + 3H 2 O ¾¾
® H 3PO4 + 3HCl
OH (B) (C )
orthophosphoric acid
27. (c) BiCl 3 + H 2O ® BiOCl + 2HCl .BiOCl

O gives white ppt. which is used as white


|| pigment, under the name of pearl white.
P 28. (b) Ca 3 (PO 4 )2 + 2H 2SO 4 + 5H 2 O ®
O O
Ca (H 2 PO 4 ) 2 . H 2 O + 2CaSO 4 .2H 2 O
O
O P P O 29. (b) Red P does not react with NaOH to give
||

||

21. (d)
O PH3.
O P 30. (c) NO2 is paramagnetic and readily dimerises
|| O
O to N2O4 which is diamagnetic.
The p-Block Elements (Group 15, 16, 17 & 18) 251

31. (b) CuSO4 ∗ 4NH3 ® [Cu( NH 3 ) 4 ] SO 4 H SO


42. (d) C12 H 22 O11 ¾¾¾¾
2 4 ® 12C + 11H O
2
D black
Blue complex due to Cu(NH3 ) 24 +
43. (c)
32. (c) P2O 5 + 2HNO 3 ® N 2O 5 + 2HPO 3
O S O O
|| || || ||
33. (b) SO32- + 2HÅ ® SO2 + H 2 O S S S S O

||
(X ) , , ,

||
–O

||
O

||
O O O
3SO2 + K 2 Cr2O7 + H 2SO 4 ®
(Y) O– O– O–
K 2 SO4 + Cr2 (SO4 )2 + H 2O (SO3)
3)
(S2O2– (S2O62–)
Green coloured O O
solution || ||
34. (c) FeCl 3 acts as oxidant and H 2SO 3 as S S

||
O

||
O–O O
reductant. O– –
O
35. (b) The key step in the manufacture of H2SO4
is catalytic oxidation of SO2 with O2 to (S2O 82– )
Hence (c) is the correct option.
give SO3 in presence of V2O5 .
44. (b) H2O2 > O3 > O2
36. (a) 2FeCl3 + SO2 + 2H 2 O ® 2FeCl2 B. O. is 1, 1.5 and 2 respectively.
+ H 2SO4 + 2HCl O
O O
H – O – O – H; O O; O = O
37. (c) HO S O O S OH + H2O ¾®
45. (a) 4FeS + 7O 2 ® 2Fe 2 O 3 + 4SO 2
O O
peroxodisulphuric acid

O O
It is dibasic
HO S O OH + HO S OH 46. (c) Hybridisation is sp3d2 , hence octahedral
47. (c) Na 2S2 O3 + Cl 2 + H 2 O ¾¾®
O O 2HCl + S + Na 2SO 4
peroxomonosulphuric acid sulphuric acid
48. (c) Monoclinic sulphur is stable above 369 K.
38. (a) 2HBr + H 2SO 4 ® Br2 + 2H 2O + SO 2 49. (c) S2 is paramagnetic. It contains two unpaired
electrons in the antibonding p* orbital
HBr is oxidised to Br2
50. (b) p p –d p bonding is present in SO 32 - , N, B,
39. (d) 2HI + H 2SO 4 ® I 2 + SO 2 + 2H 2 O (HI is C have no vacant d atomic arbitals.
oxidised to I2) ( H 2SO 4 is reduced to SO2)
40. (d) All hexafluorides of group 16 elements are
gaseous in nature.
41. (d) 2Ag2O (s) ¾® 4Ag (s) + O2 (g)
2Pb3O4 (s) ¾® 6PbO (s) + O2 (g)
O S O–
2PbO2 (s) ¾® 2PbO (s) + O2 (g)
O–
pyramidal
EBD_7587
252 Chemistry Objective MCQs

51. (c) HO.SO2OH + 2PCl5 ¾® 61. (b) 2F2 + 4KOH ® 4KF + O 2 + 2H 2 O for 1 mole
ClSO2Cl + 2POCl3 + 2HCl of F2 the molar ratio.
Sulphuryl chloride
F2 KOH KF O2 H2O
52. (c) Cu + 2H2SO4 (conc) ¾®
CuSO4 + SO2 + 2H2O 1
1 2 2 1
2
53. (d) Caro’s acid is H 2SO 5 which contains one
62. (a) Chlorine cannot displace F from NaF. The
S – O – O – H peroxy linkage. It is also
reactivity follows the order F > Cl > Br > I
known as permonosulphuric acids.
63. (a) 2NaOH + Cl 2 ® NaCl + NaOCl + H 2 O
O
|| hence Cl- and OCl-
H – O – O – S – OH
|| 64. (b) 3Cl 2 + 6KOH ® KClO3 + 5KCl + 3H 2O
O
KClO3 is used in fire works and safety
Caro's acid
matches and Cl2 is greenish yellow gas.
-1 0 +
54. (b) 4KI+ 2CuSO4 ® I2 + Cu 2 I2 + 2K 2SO 4 65. (b) In HNO3 + HF ® H 2 NO3 + F- HNO 3
accepts H+
hence it is a base.
0 2+ -2.5 -1
I2 + 2Na 2S2O3 ® Na 2S4O6 + 2NaI 66. (a) NaClO is a salt of strong base and weak
acid hence on hydrolysis the solution will
In this CuI2 is not formed. be most basic, therefore maximum pH.
55. (c) In liquid state BrF3 dissociates into BrF2+ 67. (c) Pure ClO2 is obtained by passing dry Cl2
and BrF4– ions most easily. over AgClO3 at 90°C.
0 +1 -1 90°C
56. (b) 2AgClO 3 + Cl 2 (dry ) ¾¾¾®
H 2 O + Br2 ¾¾
® HOBr + HBr
2AgCl + 2ClO 2 + O 2
Thus here oxidation number of Br increases
from 0 to +1 and also decreases from 0 to 68. (d) Chlorine reacts with excess of ammonia to
–1. Thus it is oxidised as well as reduced. produce ammonium chloride and nitrogen.
3Cl2 + 8NH3 (excess)¾¾ ® 6NH4Cl + N2
57. (c) 2 Na 2S2 O3 + I 2 ® Na 2 S4 O 6 + 2 NaI 69. (c) Flourine is the most electronegative
Sodium
tetrathion ate element & has least tendency to form
double bonds.
+1 +5 -1 70. (c) Chlorinated water is yellow in colour on
58. (d) NaClO ® NaClO3 + 2NaCl.
standing reaction occurs
All statements are correct as evident from Cl2 + H2O ® HCl + HOCl
the reaction. 71. (b) The molecular geometry of BrF5 is square
+7 +2 pyramidal with asymmetric charge
59. (d) 2 KMnO 4 + 16HCl ® 2 MnCl2 + 2KCl
distribution on the central atom.
+ 8H 2 O + 5Cl2
O.S of Mn changes from +7 to +2 hence F F
reduction occurs and Cl2 is formed. Br

60. (a) Reactivity follows the order F > Cl > Br > I F F


F
The p-Block Elements (Group 15, 16, 17 & 18) 253
72. (c) The electron gain enthalpy order for four s bonds with oxygen atoms. They are
halogens is ssp3 – p. Four pp– dp bonds are also formed
Cl > F > Br > I with oxygen atoms by the unpaired
Due to small size of fluorine the extra electrons.
electron to be added feels more electron- 76. (b) XeF4 has zero dipole moment. It has
electron repulsion. Therefore fluorine has square planar structure due to which the
less value for electron affinity than chlorine. bond moments of Xe – F cancel each
other.
73. (d) Partial
XeF6 + H 2 O ¾¾¾¾
® XeOF4 + 2HF
F F
Partial
..
XeF6 + 2H 2 O ¾¾¾¾
® XeO2 F2 + 4HF Xe
..
Complete F
XeF6 + 3H 2 O ¾¾¾¾¾
® XeO3 + 6HF F

74. (b) The hybridization of XeO3F2 is sp3d and its 77. (b) ® XeF5+ + HF2-
HF + XeF6 ¾¾
structure is trigonal bipyramidal in which
oxygen atoms are situated on the plane and
+Excess H2O
the fluoride atoms are on the top and XeO3 + HF
78. (b) (X)
bottom. XeF6
+2H 2O
75. (c) Xenon undergo sp3 hybridization. XeO2 F2 + HF
(Y)

Y is not an oxyacid of xenon.


5s 5p
79. (d) In clathrates the forces are dipole - induced
(ground dipole interaction
state)
5d 80. (b) XeF4 + H 2O ® 2HF + XeOF2
81. (c) Hybridisation in each case is XeF4, sp3d2,
XeF2, sp3d.
5s 5p 82. (a) The smaller the size the least is the
polarisability.
(third excited
state) 83. (c) 2XeF6 + SiO 2 ® SiF4 + 2XeOF4
5d
In the fourth excited state xenon atom, has 84. (c) Helium is obtained during radioactive decay.
8 unpaired electrons 85. (a) Air contains about 1% inert gases, mainly
Ar. (At.wt 40) The atomic wt. of N2 is 28.
86. (d)
5s 5p
(i) The first ionization energy of xenon (1, 170
kJ mol–) is quite close to that of dioxygen
5d (1,180 kJ mol–1).
One s and three p orbital undergo sp3
(ii) The molecular diameters of xenon and
hybridization. Four sp3 hybrid orbitals form
dioxygen are almost identical.
EBD_7587
254 Chemistry Objective MCQs

Based on the above similarities Barlett (who


prepared O2+[PtF6]– compound) suggested XeOF4–
O


that since oxygen combines with PtF6, so F F
xenon should also form similar compound Xe

with PtF6. F F
– 1 lP
87. (b) 2XeF6 + SiO 2 ¾¾
® 2XeOF4 + SiF4
(glass) (explosive nature)
XeF6– F
2XeOF4 + SiO 2 ¾¾
® 2XeO 2 F2 + SiF4 F
F Xe F

2XeO 2 F2 + SiO2 ¾¾
® 2XeO3 + SiF4 F
(explosive) F
– 1 lP
+ -
88. (b) ® [ XeF3 ] [SbF6 ] ¾¾
XeF4 + SbF5 ¾¾ ®
90. (c) In XeOF4, Xe is sp3d 2 hybridised

+ -
éë XeF3 ùû + éSbF ù O
ë 6û
F F
sp3d sp 3d 2
Xe
bent T -shape octahedral
F F
XeOF4
89. (d) XeO3– Square pyramidal
Xe—

(sp 3d 2)


O
O

O
– 1 lP
The d-and f-Block
22
Elements
1. Which of the following is not a condition for 8. A compound of a metal ion Mx+ (Z = 24) has a
complex formation? spin only magnetic moment of 15 Bohr
(a) Small atomic size Magnetons. The number of unpaired electrons
(b) High nuclear charge in the compound are
(c) Variable oxidation states (a) 2 (b) 4 (c) 5 (d) 3
(d) Availability of vacant d orbitals 9. Cuprous ion is colourless while cupric ion is
2. Zn gives H2 gas with H2SO4 and HCl but not coloured because
with HNO3 because (a) both have half filled p-and d-orbitals.
(a) Zn acts as an oxidising agent when react (b) cuprous ion has incomplete d-orbital and
with HNO3. cupric ion has a complete d-orbital.
(c) both have unpaired electrons in the
(b) HNO3 is weaker acid than H2SO4 and HCl.
d-orbitals.
(c) In electrochemical series Zn is above (d) cuprous ion has complete d-orbital and
hydrogen. cupric ion has an incomplete d-orbital.
(d) NO3- ion is reduced in preference to 10. Among the following, the compound that is both
hydroniumion. paramagnetic and coloured, is
3. Which one of the following species is stable in (a) KMnO4 (b) CuF2
(c) K2Cr2O7 (d) All are coloured
aqueous solution ? 11. Which of the following oxides of manganese is
amphoteric?
(a) Cr2+ (b) MnO 2– 4 (a) MnO2 (b) Mn 2O3
(c) MnO 4 3– (d) Cu+ (c) Mn 2O7 (d) MnO
12. In the laboratory, manganese (II) salt is oxidised
4. Which of the following is not correct about
to permanganate ion in aqueous solution by
transition metals? (a) hydrogen peroxide (b) conc. nitric acid
(a) Their melting and boiling points are high (c) peroxy disulphate (d) dichromate
(b) Their compounds are generally coloured 13. When Cu reacts with AgNO3 solution, the
(c) They can form ionic or covalent compounds reaction takes place is
(d) They do not exhibit variable valency (a) oxidation of Cu (b) reduction of Cu
+
5. Zinc and mercury do not show variable valency (c) oxidation of Ag (d) reduction of NO3-
like d-block element because 14. CuSO4 reacts with KCN solution and forms
(a) they are soft.
(b) their d-shells are complete.
(c) they have only two electrons in the (a) Cu(CN) (b) Cu(CN)2
outermost shell. (c) K3[Cu(CN)4] (d) K4[Cu(CN)6]
(d) their d-shells are incomplete. 15. Which of the following compounds gives red
6. Which one of the following is diamagnetic ion? precipitate with AgNO3?
(a) Co2+ (b) Cu2+ (c) Mn 2+ (d) Sc3+ (a) KI (b) K2CrO4
7. Which group contains coloured ions out of (c) NaBr (d) NaNO3
16. When horn silver ore is dissolved in excess of
1. Cu2+ 2. Ti4+ 3. Co2+ 4. Fe2+ sodium cyanide solution compound formed is
(a) 1, 2, 3, 4 (b) 1, 3, 4 (a) AgCN (b) Na[Ag(CN)2 ]
(c) 2, 3 (d) 1, 2 (c) Na 2[Ag(CN)3 ] (d) Na 4[Ag(CN)5 ]
EBD_7587
256 Chemistry Objective MCQs
17. In the equation (c) [MnCl 4 ]2- > [CoCl 4 ]2 - > [Fe(CN) 6 ]4 -
4M + 8CN - + 2H 2 O + O 2 ® 4 M CN + 4OH - (d) [Fe(CN ) 6 ]4 - > [CoCl 4 ]2- > [MnCl 4 ]2 -
- (Atomic nos. : Mn = 25, Fe = 26, Co = 27)
4M + 8CN - + 2H O + O ® 4 éë M (CN )2 ùû + 4OH -
Out of [TiF6 ] ,,[CoF6 ]3- , Cu2Cl2 and [ NiCl 4 ]2-
2–
25.
Identify the metal M. It is
(a) copper (b) iron (c) gold (c) zinc (Z of Ti = 22, Co = 27, Cu = 29, Ni = 28), the
18. Mercury is the only metal which is liquid at 0°C. colourless species are:
This is due to its
(a) Cu2Cl2 and [ NiCl4 ]
2–
(a) very high ionisation energy and weak
metallic bond.
(b) low ionisation potential. (b) [TiF6 ]2– and Cu2Cl2
(c) high atomic weight.
(d) high vapour pressure. (c) [CoF6 ]3– and
, [NiCl4 ]2–
19. If KMnO4 is reduced by oxalic acid in an acidic
medium then oxidation number of Mn changes (d) [TiF6 ]2– a, nd [CoF6 ]3– ,
from 26. The basic character of the transition metal
(a) 4 to 2 (b) 6 to 4 monoxides follows the order
(c) +7 to +2 (d) 7 to 4 (Atomic Nos.,Ti = 22, V = 23, Cr = 24, Fe = 26)
20. In the preparation of KMnO4 pyrolusite (MnO2) (a) TiO > VO > CrO > FeO
is first converted to potassium manganate (b) VO > CrO > TiO > FeO
(K2MnO4). In this conversion, the oxidation state (c) CrO > VO > FeO > TiO
of manganese changes from (d) TiO > FeO > VO > CrO
(a) +1 to +3 (b) +2 to +4 27. Which of the following statement is incorrect ?
(c) +3 to +5 (d) +4 to +6
21. In an alkaline condition KMnO4 reacts as follows:
2KMnO 4 + 2KOH ® 2K 2 MnO4 + H 2O + O
(a) Across a period from Sc to Cu the densities
Its equivalent weight is
increase with increasing atomic number.
(a) 31.6 (b) 52.7 (c) 49 (d) 158
(b) The melting point of transition elements rise
22. In which of the following compounds manganese
to a maximum from Sc to Cr and then
has oxidation number equal to that of iodine in
decreases from Fe to Zn.
KIO4 ?
(c) Transition elements have high enthalpies of
(a) Potassium manganate
atomization and in 3d series increase
(b) Potassium permanganate
regularly from Sc to Cu.
(c) Manganous chloride
(d) On going down a group from 3d to 6d series
(d) Manganese chloride
the stability of higher oxidation state
23. The colour of CuCr2O7 solution in water is green
increases with increasing atomic number.
because
28. On the basis of data given below,
(a) Cr2 O72- ions are green. !
E Sc3+ /Sc 2 + = -0.37 , E!
- -
Mn 3+ /Mn 2+
= +1.57
(b) Cu 2+ ions are green.
d
- d
-
(c) Both Cr2O2– 2+ E = -0.90 , E = 0.34
7 and Cu ions are green. Cr 2 + /Cr Cu 2 + /Cu
(d) Cu2+ ions are blue and Cr2 O72- ions are Which of the following statements is incorrect?
yellow. (a) Sc3+ has good stability due of [Ar]3d04s0
24. The correct order of magnetic moments (spin only configuration.
values in B.M.) among is (b) Mn3+ is more stable than Mn 2+.
(c) Cr2+ is reducing in nature.
(a) [Fe(CN ) 6 ]4 - > [MnCl 4 ]2 - > [CoCl 4 ]2 - (d) Copper does not give H2 on reaction with
(b) [MnCl 4 ]2- > [Fe(CN ) 6 ]4 - > [CoCl 4 ]2 - dil. H2SO4.
The d-and f-Block Elements 257
29. A compound of iron exists as a dimer in vapour
state. It is hygroscopic in nature and dissolves 35. Of Cr (VI) as Cr2 O72 - and CrO24 - , which is better
in water giving brown acidic solution. The oxidising agent?
compound is (a) CrO42–, basic medium
(a) Fe3O4 (b) FeSO4 (c) FeCl3 (f) FeO2 (b) Cr2O72–, basic medium
30. A metal M and its compound can give the (c) Cr2O72–, acidic medium
following observable changes in a consequence (d) CrO3, basic medium
of reactions 36. A blue colouration is not obtained when
(a) ammonium hydroxide dissolves in copper
sulphate.
dilute é Colourless ù aqueous é White ù
M ¾¾¾¾ ®ê ú ¾¾¾¾® ê ú (b) copper sulphate solution reacts with
HNO3
ë solutions û NaOH ë precipitate û
K 4 [Fe(CN) 6 ] .
é White ù H 2S éColourless ù excess
ê precipitate ú ¬¾¾ ¾ê
solutions ú ¬¾¾¾¾
NaOH (aq.)
¾ (c) ferric chloride reacts with sod. ferrocyanide.
ë û ë û
(d) anhydrous CuSO4 is dissolved in water.
(a) Mg (b) Pb (c) Zn (d) Sn 37. Which of the statements is not true?
heat
( X ) + K 2CO3 + Air ¾¾¾
31. ® (Y)
( Y ) + Cl2 ¾¾ ( )
® Z Pink
Which of the following is correct? (a) On passing H2S through acidified K2Cr2O7
(a) X = black, MnO2, Y = Blue, solution, a milky colour is observed.
K2CrO4, Z = KMnO4 (b) Na 2Cr 2O7 is preferred over K2Cr 2O7 in
(b) X = green, Cr2O3, Y = Yellow,
volumetric analysis.
K2CrO4, Z = K2Cr2O7
(c) K2Cr 2O7 solution in acidic medium is
(c) X = black, MnO2, Y = green,
orange.
K2MnO4, Z = KMnO4
(d) K2Cr 2O 7 solution becomes yellow on
(d) X = black, Bi2O3, Y = colourless
KBiO2, Z = KBiO3 increasing the pH beyond 7.
H S­ Excess 38. The blue complex formed on addition of conc.
CuSO4 (aq.) ¾¾¾® M ¯ ¾¾¾¾ ®N+O
2
32. of KCN NH4OH solution to a Cu2+ salt solution has the
Then final products N and O are respectively. structure?
(a) [Cu(CN)4]3–, (CN)2 (b) CuCN, (CN)2 (a) [Cu(NH4)4]2+ (b) [Cu(NH3)2]2+
(c) [Cu(CN)4]2–, (CN)2 (d) Cu (CN)2, K2S (c) [Cu(NH3)4] 2+ (d) [Cu(NH4)2]2+
33. Which of the following statements is false ? 39. Identify the incorrect statement
(a) CuSO4 reacts with KCl in aqueous solution
to give Cu2Cl2.
(a) Na2Cr2O7 is less soluble than K2Cr2O7. (b) CuSO4 reacts with KI in aqueous solution
(b) Na2Cr2O7 is primary standard in volumetry. to give Cu2I2.
(c) CrO42– is tetrahedral in shape. (c) CuSO4 react with NaOH and glucose in
(d) Cr2O72– has a Cr–O–Cr bond. aqueous medium to give Cu2O.
34. A red solid is insoluble in water. However it (d) CuSO4 on strong heating gives CuO.
becomes soluble if some KI is added to water. 40. The black compound formed during the reaction
Heating the red solid in a test tube results in between sodium thiosulphate and silver nitrate
liberation of some violet coloured fumes and is
droplets of a metal appear on the cooler parts of (a) silver thiosulphate (Ag2S2O3)
the test tube. The red solid is (b) silver sulphide (Ag2S)
(a) HgI2 (b) HgO (c) silver sulphate (Ag2SO4)
(c) Pb3O4 (d) (NH4)2Cr2O7 (d) silver sulphite (Ag2SO3)
EBD_7587
258 Chemistry Objective MCQs
41. When KMnO4 is added to oxalic acid, the 49. Identify incorrect statement :
decolourisation is slow in the beginning but (a) Cu2O is colourless.
becomes instantaneous after sometime because (b) Copper (I) compounds are colourless except
(a) Mn2+ acts as autocatalyst. when colour results from charge transfer.
(b) CO2 is formed as the product. (c) Copper (I) compounds are diamagnetic.
(c) Reaction is exothermic. (d) Cu2S is black.
(d) MnO4– catalyses the reaction. 50. Which of the following compounds has colour
42. Number of electrons transfered in each case when but no unpaired electrons?
KMnO4 acts as an oxidising agent to give MnO2, (a) KMnO4 (b) K2MnO4
(c) MnSO4 (d) MnCl2
Mn2+, Mn(OH)3 and MnO 24- are respectively
51. Which one of the following exhibits the large
(a) 3,5,4 and 1 (b) 4,3,1 and 5 number of oxidation states?
(c) 1,3,4 and 5 (d) 5,4,3 and 1
(a) Ti (22) (b) V (23)
43. Chloro compound of Vanadium has only spin
(c) Cr (24) (d) Mn (25)
magnetic moment of 1.73 BM. This Vanadium
52. The element with which of the following outer
chloride has the formula: electron configuration may exhibit the largest
number of oxidation states in its compounds :
(a) VCl2 (b) VCl4 (c) VCl3 (d) VCl5
(a) 3d 54s2 (b) 3d 84s2
44. Which pair of elements with the given atomic 7
(c) 3d 4s 2 (d) 3d 64s2
numbers is expected to have similar properties?
53. Because of lanthanoid contraction, which of the
(a) 40, 72 (b) 20, 36 (c) 10, 28 (d) 11, 12 following pairs of elements have nearly same
45. When concentrated HCl is added to an aqueous atomic radii ? (Numbers in the parenthesis are
solution of CoCl2, its colour changes from atomic numbers).
reddish pink to deep blue. Which complex ion (a) Zr (40) and Nb (41) (b) Zr (40) and Hf (72)
gives blue colour in this reaction? (c) Zr (40) and Ta (73) (d) Ti (22) and Zr (40)
54. Which is not the correct statement?
(At. nos. Ce = 58, Lu = 71, La = 57, Yb = 70)
(a) [CoCl4]2– (b) [CoCl6]3– (a) Colour of Yb3+ ion is pink.
(c) [CoCl6] 4– (d) [Co(H2O)6]2+ (b) La3+ is diamagnetic.
46. Which of the following is not formed when H2S (c) Ce4+ has f 0 configuration.
reacts with acidic K2Cr2O7 solution? (d) Lu3+ had f 14 configuration.
(a) CrSO4 (b) Cr2(SO4)3 55. Magnetic moment of Gd3+ ion (Z = 64) is
(c) K2SO4 (d) S (a) 3.62 BM (b) 9.72 BM
47. Potassium dichromate when heated with (c) 7.9 BM (d) 10.60 BM
concentrated sulphuric acid and a soluble 56. Which of the following forms stable + 4 oxidation
chloride, gives brown-red vapours of : state?
(a) La (Z = 57) (b) Eu (Z = 63)
(c) Ce (Z = 58) (d) Gd (Z = 64)
(a) CrO3 (b) CrCl3 57. The number of unpaired electrons in Gadolinium
(c) CrO2Cl2 (d) Cr2O3 [Z = 64] is
48. When a small amount of KMnO4 is added to (a) 3 (b) 8 (c) 6 (d) 2
concentrated H2SO4, a green oily compound is 58. Which one of the following elements shows
obtained which is highly explosive in nature. maximum number of different oxidation states in
Compound may be :
its compounds?
(a) MnSO4 (b) Mn 2O7
(c) MnO2 (d) Mn 2O3 (a) Eu (b) La (c) Gd (d) Am
The d-and f-Block Elements 259
59. The radius of La3+ (Atomic number of La = 57) is 67. Arrange (I) Ce3+, (II) La3+, (III) Pm 3+ and
1.06Å. Which one of the following given values (IV) Yb3+ in increasing order of their ionic radii.
will be closest to the radius of Lu3+ (Atomic (a) IV < III < I < II (b) I < IV < III < II
number of Lu = 71) ? (c) IV < III < II < I (d) III < II < I < IV
(a) 1.40 Å (b) 1.06 Å (c) 0.85 Å (d) 1.60 Å 68. Which of the following statement is not correct?
60. The electronic configuration of gadolinium
(At. No. 64) is
(a) [ Xe]4 f 8 5d1 6s 2 (b) [ Xe]4 f 7 5d1 6s 2
(a) La(OH)3 is less basic than Lu(OH)3.
(c) [ Xe]4 f 3 5d 5 6s 2 (d) [ Xe]4 f 6 5d 2 6 s 2 (b) In lanthanide series ionic radius of Ln 3+
61. In which of the following lanthanoids oxidation ions decreases.
state +2 is most stable? (c) La is actually an element of transition series
(a) Ce (b) Eu (c) Tb (d) Dy rather than lanthanide series.
62. Which of the following lanthanoid ions is (d) Atomic radii of Zr and Hf are same because
diamagnetic ? of lanthanide contraction.
(At nos. Ce = 58, Sm = 62, Eu = 63, Yb = 70) 69. The reason for the stability of Gd3+ ion is
(a) Sm2+ (b) Eu2+ (c) Yb2+ (d) Ce2+
(a) 4¦ subshell — half filled.
63. Lanthanoid contraction can be observed in
(b) 4¦ subshell — completely filled.
(a) At (b) Gd (c) Ac (d) Lw
64. Lanthanoides and actinoides resemble in (c) Possesses the general electronic
(a) Electronic configuration configuration of noble gases.
(b) Oxidation state (d) 4¦-subshell empty.
(c) Ionization energy 70. Knowing that the chemistry of lanthanoids(Ln)
(d) Formation of complexes is dominated by its + 3 oxidation state, which of
65. The Ce (Z = 58) belongs to IIIrd group of peri- the following statements is incorrect?
odic table. If it furnish one a particle to form an (a) The ionic size of Ln (III) decrease in general
element 'X', then X belongs to with increasing atomic number.
(a) IIIrd group (b) IInd group (b) Ln (III) compounds are generally colourless.
st
(c) I group (d) zero group (c) Ln (III) hydroxide are mainly basic in
66. In aqueous solutions Eu2+ acts as
character.
(a) an oxidising agent.
(d) Because of the large size of the Ln (III) ions
(b) a reducing agent.
(c) can act either of these. the bonding in its compounds is
(d) can act as redox agent. predominantly ionic in character.

Answer KEYs
1 (c) 8 (d) 15 (b) 22 (b) 29 (c) 36 (b) 43 (b) 50 (a) 57 (b) 64 (b)
2 (d) 9 (d) 16 (b) 23 (d) 30 (c) 37 (b) 44 (a) 51 (d) 58 (d) 65 (b)
3 (b) 10 (b) 17 (c) 24 (c) 31 (c) 38 (c) 45 (a) 52 (a) 59 (c) 66 (b)
4 (d) 11 (a) 18 (a) 25 (b) 32 (a) 39 (a) 46 (a) 53 (b) 60 (b) 67 (a)
5 (b) 12 (c) 19 (c) 26 (a) 33 (a) 40 (a) 47 (c) 54 (a) 61 (b) 68 (a)
6 (d) 13 (a) 20 (d) 27 (c) 34 (a) 41 (a) 48 (b) 55 (c) 62 (c) 69 (a)
7 (b) 14 (c) 21 (d) 28 (b) 35 (c) 42 (a) 49 (a) 56 (c) 63 (b) 70 (b)
EBD_7587
260 Chemistry Objective MCQs

1. (c) Tendency of an element to form complexes 2+


does not depend on whether the element 13. (a) Cu 0 + 2AgNO3 ® Cu ( NO3 )2 + 2Ag .

2. (d)
shows variable O.S. or not.
NO3- ions are reduced by nascent
( 0
It is oxidation of copper Cu ® Cu
2+
)
hydrogen 14. (c) 2CuSO 4 + 10KCN ¾¾ ®
Metal + HNO 3 ® Metal nitrate + [H] 2K 3[Cu(CN)4 ] + 2K 2SO 4 + (CN)2
15. (b) 2AgNO3 + K 2CrO4 ®
HNO3 + 8[H] ® N 2 O + 5H 2O
Ag 2 CrO 4 + 2KNO 3
3. (b) In MnO 24- manganese is in + 6 oxidation Red precpitate

state which is having highest stability. 16. (b) AgCl + 2NaCN ®


Na éëAg ( CN )2 ùû + NaCl
MnO24- disproportionates in neutral or
acidic solution. 17. (c) This is cyanide process for gold (M = Au).
18. (a) Metallic bonding in mercury is weak.
3MnO 24 - + 4H + ¾¾
® 19. (c) In acid medium
2MnO 24- + MnO2 + 2H 2O MnO4- + 8H + + 5e - ® Mn 2 + + 4H 2O
4. (d) Transition metals exhibit variable valency. (O.S. of Mn changes form +7 to +2).
5. (b) Zn and Hg have d-shells completely filled 20. (d) In MnO2 (O.S. of Mn is + 4) in K 2MnO4
hence they do not exhibit variable valency. (O.S.of Mn is +6). Hence O.S. changes by 2.
6. (d) Sc3+ is diamagnetic as it does not contain 21. (d) Change in O.S. is by 1. Hence eq. wt. is
any unpaired electron while others contain.
158
7. (b) Cu 2 + [ Ar ] 3d 9 , Ti 4 + [ Ar ]3d 0 , Co 2 + [ Ar ]3d 7 , = 158 .
1
Fe2 + [ Ar ] 3d 6 1,3,4 are coloured ions 22. (b) In KIO4, O.S of I is +7 and in KMnO4, O.S.
hence the answer is b. of Mn is +7.
23. (d) Green colour of CuCr 2O7 is due to blue
8. (d) Magnetic moment m = n ( n + 2 ) where
colour of Cu 2+ ions and yellow colour of
n = number of unpaired electrons
Cr2 O7 2- ions .
15 = n ( n + 2 ) \ n = 3 24. (c) [Fe(CN)6]4–
(d) In Cu [ Ar ] 3d there is no unpaired [Fe (CN ]
+ 10 4–
9. ®
electron, no of unpaired electron = 0
Cu 2 + [ Ar ] 3d 9 contains one unpaired
2–
[MnCl4] ®
electron hence coloured. no of unpaired electron = 5
10. (b) CuF2 is both paramagnetic and coloured. 2–
11. (a) The more the oxidation state, the more is [CoCl4] ®
the acid character, MnO, Mn 2O3 are basic, no of unpaired electron = 3
MnO2 is amphoteric and Mn2O7 is acidic. The greater the number of unpaired
12. (c) In laboratory, manganese (II) ion salt is electrons, greater the magnitude of
oxidised to permagnate ion in aqueous magnetic moment. Hence the correct order
solution by peroxodi-sulphate. will be
[MnCl4]2 – > [CoCl4]2 – > [Fe(CN)6]4–
2Mn 2+ + S2O82- + 8H 2O ¾¾ ®
peroxodisulphate ion
25. (b) The colour exhibited by transition metal ions
2MnO -4 + 10 SO24 - + 16H + is due to the presence of unpaired electrons
in d-orbitals which permits the d - d
excitation of electrons.
The d-and f-Block Elements 261

In [TiF6 ] –, Ti is in + 4 O.S. ; 3d 0 – colourless Na+ is greater


than K+
2– Hydration energy of
+
because of smaller size of Na , Na2Cr2O7 is
In [CoF6 ] –,Co is in + 3 O.S ; 3d 5 – coloured
3– more soluble than K2Cr2O7.
34. (a) When KI is added to mercuric iodide it
In Cu2Cl2– Cu is in +1 O.S. ; 3d10 – colourless disssolve in it and form complex.
In [ NiCl4 ] – Ni is in + 2 O.S ; 3d8 – coloured
2–
HgI 2 ∗ KI ® K 2 [HgI 4 ]
26. (a) The order of basic character of the transition red, solid (soluble)
(insoluble)
metal monoxide is TiO > VO > CrO > FeO
because basic character of oxides decreases On heating HgI2 decomposes as
with increase in atomic number. HgI 2 ƒ Hg + I 2
27. (c) (a), (b) and (d) are correct statements. (violet vapours)
(c) There is drop in enthalpy of atomization
at manganese because of half filled stable 35. (c) CrO 24 - (aq) + 4H 2 O + 3e - ¾¾
®
electronic configuration leading to less
mobilisation of electrons. This results in Cr(OH)3 + 5OH - ,
weaker metallic bonds. Eº = -0.13V
28. (b) Mn2+ (d 5)is more stable than Mn3+ (d4), Cr2 O 72 - (aq.) + 14 H + (aq.) + 6e - ¾¾ ®
thus E - 3+ 2 + = + ve 3+
Mn /Mn 2Cr (aq.) + 7 H 2 O,
29. (c) The solution is acidic due to formation of Eº = 1.33V
HCl and brown due to Fe(OH)3. 36. (b) 2CuSO 4 + K 4 [ Fe (CN ) 6 ] ¾ ¾®
FeCl3 + 3H 2O ® Fe (OH )3 + 3HCl Cu 2 [ Fe (CN ) 6 ] + 2 K 2SO 4
Brown Chocolate ppt.
30. (c) dil. HNO3 NaOH
Zn ¾¾¾¾¾ ® Zn(NO3 ) 2 ¾¾¾¾ ® Zn(OH) 2
aq.
37. (b) Na2Cr2O7 is hygroscopic.
(M) Colourless White ppt.
solution 38. (c) Cu 2+ + 4NH 4 OH ®
Excess
¾¾¾¾ ® Na 2 [Zn (OH )4 ] ¾¾¾® H 2S
ZnS
NaOH
Soluble White ppt. [Cu(NH 3 ) 4 ]2 + + 4H 2 O
D deep blue comp
31. (c) 2MnO2 + 2K 2CO3 + O 2 ¾¾® 39. (a) 2CuSO 4 + 4KI ® Cu 2 I 2 + 2K 2SO4 + I 2
(X) (air)
(not given by KCl).
2K2 MnO4 +2CO2 ­ Fehling solution gives Cu2O with reducing
green substances
(Y)
strong
2K 2 MnO 4 + Cl 2 ¾¾
® 2KMnO 4 + 2KCl CuSO 4 ¾¾ ¾® CuO + SO 3
D
(Y) ( Z ) Pink
40. (a) 2AgNO3 + Na 2S2O3 ®
H S KCN
32. (a) CuSO 4 (aq.) ¾¾¾
2 ® 2CuS ¾¾¾¾
® Ag 2S2O3 + 2NaNO3
Black ppt. Excess
(M) 41. (a) Little bit acid is required for reaction
\ Mn2+ acts as autocatalyst.
[Cu(CN) 4 ]3- + (CN)2 ­ 42. (a)
(N) (O)
KMnO4 MnO 2 Mn2+ Mn ( OH )3 MnO 42-
O
2– O.S of Mn +7 +4 +2 +3 +6
33. (a) Cr O– , CrO4 is tetrahedral electrons 0 3 5 4 1
O needed
O–
O O
43. (b) m = n ( n + 2)
Cr Cr , Cr – O – Cr
O – O O 1.73 = n ( n + 2 )
O –
O
On calculating the value of n we find n = 1
bond in Cr2 O7 2 - No. of unpaired electrons = 1
Na2Cr2O7 is used as primary standard in hence its configuration will be
volumetry. V(23) = [Ar] 3d 34s2
EBD_7587
262 Chemistry Objective MCQs
V4+ = [Ar]3d1 57. (b) Gd(64) = [Xe]4f 7 5d1 6s2
\ Its chloride has the formula VCl4 \ No. of unpaired electrons = 8
44. (a) Zr (40), Hf (72) have similar properties as 58. (d) We know that lanthanides La, Gd shows
they belong to same group and have same +3, oxidation state, while Eu shows
valence shell electronic configuration. oxidation state of +2 and + 3. Am shows +3,
45. (a) Aqueous of solution of CoCl 2 contains +4, +5 and +6 oxidation states. Therefore
[Co(H2O)6]2+ which is pinkish in colour so Americium (Am) has maximum number of
option (d) is incorrect. oxidation states.
Reduction potential of Co3+ ¾¾ ® Co 2 + 1
59. (c) Ionic radii µ
is high so option(b) is incorrect. Co2+ does z
not oxidises easily to Co3+. z2 1.06 71
It is general case that symmetrical
Thus, Þ =
z1 3+
(Ionic radii of Lu ) 57
substituted octahedral complexes are less
deeper in colour than tetrahedral complexes. Þ Ionic radii of Lu3+ = 0.85 Å
So [CoCl4]2– is deep blue in colour. 60. (b) Electronic configuration of gadolinium is
46. (a) K2Cr2O7 + 4H2SO4 + 3H2S ¾¾ ® [Xe ] 4 f 7 5d1 6s 2
K2SO4 + Cr2 (SO4)3 + 7H2O + 3S 61. (b) Eu2+ has electronic configuration [Xe]4 f 7
47. (c) Solid potassium dichromate when heated hence stable due to half filled atomic
with concentrated sulphuric acid and a orbitals.
soluble chloride gives orange red vapours 62. (c) Sm2+(Z = 62)
of a volatile oily liquid CrO2Cl2 [Xe]4f 6 6s2 – 6 unpaired e– Eu2+(Z = 63)
K2Cr2O7 + 4NaCl + 6H2SO4 [Xe]4f 7 6s2 – 7 unpaired e– Yb2+(Z = 70)
¾® 2KHSO4 + 4NaHSO4 + 2CrO2Cl2 [Xe]4f 14 6s2 – 0 unpaired e– Ce2+(Z = 58)
chromyl chloride
[Xe]4f 1 5d1 6s2 – 2 unpaired e–
48. (b) KMnO4 reacts with H2SO4 to form Mn2O7
which is highly explosive substance. Only Yb2+ is diamagnetic.
2KMnO4 + H2SO4 ¾¾® 63. (b) Amongst the given elements, only Gd is a
lanthanide.
K2SO4 + Mn2O7 + H2O 64. (b) The oxidation state in both (lanthanide and
49. (a) Cu2O is red in colour. actinide) is +3. The property of actinide are
50. (a) Electronic configuration of Mn is very similar to those of lanthanide when both
[ Ar ]3d 5 4s 2 . Being transition metal it has 7 are in +3 state.
valence electrons and all are involved in 65. (b) 58Ce is lanthanoid. Lanthanoid are from 57
bond formation in MnO-4 . Hence it has no to 71 all present in IIIrd group. Hence 58Ce
unpaired electron. forms 56X element on emission of one a
51. (d) Manganese exhibits the large number of
particle with belongs to IInd group.
oxidation states. The most common
oxidation states of Mn are +2, +3, + 4, + 6 66. (b) Reducing agent
and +7. 67. (a) Ionic radii decrease across lanthanide series
due to lanthanide contraction. As all ions
52. (a) The element with outer electron configura-
are in +3 O.S., ionic radii will follow the
tion 3d 54s2 is Mn which exhibits oxidation
states from + 2 to + 7. trend of atomic radii.
53. (b) Due to lanthanoid contraction atomic radii \ La3+ > Ce3+ > Pm3+ > Yb 3+
of Zr and Hf is almost similar. ( Z = 57 ) ( Z = 58) ( Z = 61) ( Z = 70 )
54. (a) Option (a) is incorrect as Yb3+ is colorless. 68. (a) La(OH)3 is more basic than Lu(OH)3. This
55. (c) Gd (64) = [Xe] 4f 7 5d1 6s2 is because ionic size of La3+ ion is more
Gd3+ = [Xe] 4f 7 5d0 6s0 than Lu3+ ion (Fajan's rule).
i.e. no. of unpaired electrons = 7 69. (a) Gd (Z = 64) Þ 4¦7 5d 1 6s2
Gd3+ = 4¦7
m = n(n + 2) = 7(7 + 2)
Gd3+ is stable due to the presence of exactly
= 63 = 7.93 BM half-filled 4¦-subshell.
56. (c) Only Ce(Z = 58) shows stable (+4) oxidation 70. (b) Most of the Ln3+ compounds except La3+
state. and Lu3+ are coloured due to the presence
of f-electrons.
Co-ordination
23
Compounds
1. AgCl is soluble in NH4OH solution. The 9. The degeneracy of d-orbitals is lost under :
solubility is due to formation of (I) Strong field ligand
(a) AgOH (b) Ag2O (II) Weak field ligand
(c) [Ag(NH3)2]+ (d) NH4Cl (III) Mixed field ligand
2. In which of the following coordinate compounds (IV) Chelated ligand field
the central metal atom obeys the EAN rule. (a) I, II and IV (b) I and II
(a) K3[Fe(CN)6] (b) K4[Fe(CN)6] (c) I, II, III and IV (d) I, II and III
(c) [Cu(NH3)4]SO4 (d) All of these 10. What is the magnetic moment (spin only) and
hybridisation of the brown ring complex
3. Which of the following complex will show
geometrical as well as optical isomerism éë Fe ( H2 O )5 NO ùû SO4 ?
(en=ethylenediammine)
(a) 3BM, sp3 d 2 (b) 3BM, d 2 sp3
(a) [Pt(NH3)2Cl2] (b) [Pt(en)Cl4]
(c) [Pt(en)3]4+ (d) [Pt(en)2Cl2] (c) 15BM, sp3 d 2 (d) 15BM, d 2 sp3
4. The total number of possible isomers of the 11. Mn 2 + forms a complex with Br - ion. The
complex compound [CuII(NH3)4][PtIICl4] is magnetic moment of the complex is 5.92 B.M.
(a) 3 (b) 6 (c) 5 (d) 4 What would be the probable formula and
5. Which of the following will give maximum geometry of the complex?
number of isomers? (a) [MnBr6 ]4 - , octahedral
(a) [Co(NH3)4Cl2] (b) [Ni(en)(NH3)4]2+
(b) [MnBr4 ]2 - , square planar
(c) [Ni(C2O4)(en)2] (d) [Cr(SCN)2(NH3)4]2+
6. Which of the following will exhibit maximum ionic
(c) [MnBr4 ]2 - , tetrahedral
conductivity ? (d) [MnBr5 ]3- , trigonal bipyramidal
(a) K4[Fe(CN)6] (b) [Co(NH3)6]Cl3 12. The complex given is
(c) [Cu(NH3)4Cl2] (d) [Ni(CO)4]
en 3+
2+
and [ Fe (CN )6 ]
4-
7. éë Fe ( H 2 O)6 ùû differ in :

(a) geometry, magnetic moment


(b) geometry, hybridization Co
(c) magnetic moment, colour
(d) hybridization, number of d-electrons en
8. Which of the following complexes have a en
maximum number of unpaired electrons? (i) non-superimposable on its mirror images
(a) [Ni(CO)4] (ii) optically active
(b) [Co ( NH 3 )4 ( NO 2 )2 ]+ (iii) rotate plane polarised light
(iv) planar
(c) [Ag ( CN )2 ]- (a) (i) and (ii) (b) (i) and (iv)
(d) [CuBr4 ]2 - (c) (i), (ii) and (iii) (d) (ii) only
EBD_7587
264 Chemistry Objective MCQs
18. The pair in which both species have same
13. [Co(NH3)4 (NO2)2] Cl exhibits
magnetic moment (spin only value) is
(a) [Cr(H2O)6]2+, [CoCl4]2–
(a) linkage isomerism, ionization isomerism and
geometrical isomerism (b) [Cr(H2O)6]2+, [Fe(H2O)6]2+
(b) ionization isomerism, geometrical isomerism (c) [Mn(H2O)6]2+, [Cr(H2O)6]2+
and optical isomerism (d) [CoCl4]2–, [Fe(H2O)6]2+
(c) linkage isomerism, geometrical isomerism 19. In which complex, d 2 orbital of inner shell is
and optical isomerism z
(d) linkage isomerism, ionization isomerism and not used in the hybridization of central metal
optical isomerism cation
2+
14. Coordination number of Cr is six. A complex with (a) Fe (CO )5 (b) éë Cu ( NH3 )5 ùû
C2 O42- , en and superoxide O2– will be in the ratio éë Co ( NH3 )6 ùû
2+
- (c) (d) [ IrF6 ]3-
to make complex éë Cr(C2 O4 )x , (en) y (O2 )z ùû
20. Which of the characteristic is not common
x y z
between [Cu(en)2)2+ and [Ni(dmg)2] ?
(a) 1 1 1
(b) 1 1 2 (a) Geometry of complexes
(c) 1 2 2 (b) Hybridization of central metal cation
(d) 2 1 1 (c) Magnetic behaviour
15. For complex ion/compound formation reactions (d) Number of stereoisomers
(I) Co3+ (aq ) + EDTA 4 - ¾¾
®P 21. In which of the following complex hybridization
of central metal is not same as that of donor
(II) Ni 2 + (aq ) + dmg (excess ) ¾¾¾¾
NH 4OH
®Q atom of ligand
(III) Zn 2+ (aq ) + gly (excess ) ¾¾
®R
(a) éë Ni ( PF3 )4 ùû (b) éë Fe (dmg )2 ùû
(IV) Pt 4+
(aq ) + en (excess ) ¾¾
®S (c) [ Zn (en )2 ]2+ (d) éë Ni ( PMe3 )4 ùû
2+

which of the following complex ion/compound 22. Which of the following complex compound(s)
does not exhibit optical activity? is/are paramagnetic and low spin?
(a) P (b) Q (c) R (d) S
16. The diamagnetic species is : (I) K 3 ëé Fe (CN 6 )ûù (II) [ Ni (CO)4 ]0
3+
(I) [Cu (CN )4 ]3- (II) éë Co ( NH3 )6 ùû (III) éëCr ( NH3 )6 ùû
3+
(IV) [Mn (CN )6 ] 4–
2+ Choose the correct code:
(III) éë Ni ( NH3 )6 ùû (IV) [Fe ( CN )6 ]3-
(a) I only (b) II and III
(a) I, III (b) I, II (c) I and IV (d) IV only
(c) III, IV (d) only IV
23. An aqueous solution of titanium chloride, when
17. Which of the following order of stability of subjected to magnetic measurement, measured
complex ion is Incorrect? zero magnetic moment. Assuming the octahedral
complex in aqueous solution, the formulae of
(a) [Fe(C 2 O 4 )3 ]3- < [Fe(H 2 O) 6 ]3+ the complex is:
(b) [Fe (edta )]- > [Fe (en )3 ]3+
(a) éë Ti ( H 2O)6 ùû Cl2 (b) éë Ti (H 2O )6 ùû Cl4
(c) [ Ni (en )2 ]2+ > éë Ni (dmg )2 ùû
(c) éë TiCl3 ( H 2O )3 ùû (d) éë TiCl 2 ( H 2O )4 ùû
(d) [Fe (CN)6 ]3- > [Fe (CN)6 ]4-
Co-ordination Compounds 265
24. For octahedral complex, which of the following 29. Which of the following complex ions will exhibit
d n configurations of metal cation cannot exist optical isomerism? (en = 1, 2-diamine ethane).
in high spin and low spin forms? (a) [Cr(NH3)2Cl2]+ (b) [Co(en)2Cl2]+
3
(a) d and d 8 3
(b) d and d 5
(c) [Co(NH3)4Cl2]+ (d) [Zn(en)2]2+
3
(c) d and d 6 4
(d) d and d 8 30. The complex ion [Pt (NO2)(Py) (NH3) (NH2OH)]+
25. Which of the following statement is not true for will give
the reaction given below? (a) 2 isomers (Geometrical)
(b) 3 isomers (Geometrical)
2+
ˆˆ† Cu NH3
éë Cu ( H 2O )4 ùû + 4NH3 ‡ˆˆ + 4H 2 O(c) 6 isomers (Geometrical)
2+ (d) 4 isomers (Geometrical)
Cu H 2 O + 4NH3 ƒ ëéCu ( NH3 )4 ûù + 4H 2O
31. Which of the following complex ions has
(a) It is a ligand substitution reaction electrons that are symmetrically filled in both
(b) NH3 is relatively strong field ligand while t2g and eg orbitals ?
H2O is a weak field ligand (a) [FeF ]3– (b) [Mn(CN) ]4–
6 6
(c) During the reaction, there is a change in (c) [CoF6]3– (d) [CO(NH3)6]2+
colour form light blue to dark blue
32. In [Co(NH3)6]Cl3, the number of covalent bonds
(d) [Cu(NH3)4]2+ has tetrahedral structure and is
is paramagnetic
(a) 3 (b) 6 (c) 9 (d) 18
26. The d-electron configurations of Cr2+, Mn2+, Fe2+ 33. The unpaired electrons in Ni(CO)4 are
and Co2+ are d 4, d 5, d 6 and d 7 respectively. (a) zero (b) one (c) three (d) four
Which one of the following will exhibit the lowest
34. The shape of [Cu(NH3)4]2+ is
paramagnetic behaviour?
(a) tetrahedral (b) square planar
(Atomic no. Cr = 24, Mn = 25, Fe = 26, Co = 27). (c) pyramidal (d) octahedral
(a) [Co(H2O)6]2+ (b) [Cr(H2O)6]2+ 35. Consider the following complex
(c) [Mn(H2O)6]2+ (d) [Fe(H2O)6]2+ [Co(NH3)5CO3]ClO4. The coordination number,
27. An octahedral complex with molecular oxidation number, number of d-electrons and
composition M.5 NH3.Cl.SO4 has two isomers, number of unpaired d-electrons on the metal are
A and B. The solution of A gives a white
respectively
precipitate with AgNO3 solution and the solution
of B gives white precipitate with BaCl2 solution.
The type of isomerism exhibited by the complex (a) 6, 3, 6, 0 (b) 7, 2, 7, 1
is: (c) 7, 1, 6, 4 (d) 6, 2, 7, 3
(a) Linkage isomerism 36. Which of the following statements is incorrect?
(b) Ionisation isomerism (a) [MnCl6] 3– is more paramagnetic than
(c) Coordinate isomerism [Mn(CN)6]3–
(d) Geometrical isomerism
(b) Both [Co(C 2O 4) 3] 3– an d [CoF 6] 3– are
28. Type of isomerism which exists between
paramagnetic.
[Pd(C6H5)2(SCN)2] and [Pd(C6H5)2(NCS)2] is :
(c) [Fe(CN)6]3– forms inner orbital complex
(a) Linkage isomerism
whereas [FeF 6] 3– forms outer orbital
(b) Coordination isomerism
complex.
(c) Ionisation isomerism
(d) Both (a) and (b).
(d) Solvate isomerism
EBD_7587
266 Chemistry Objective MCQs
37. Which of the following statements related to 44. Incorrect match is :
crystal field splitting in octahedral coordination
entities is incorrect?
(a) The dx2–y2 and dz2 orbitals has more energy (a) [Rh(PPh3)3Cl] : Wilkinson's catalyst
as compared to dxy, dyz and dxz orbitals. (b) [Co(CO)4]– : Bond order of Co—CO bond
(b) Crystal field splitting energy (Do) depends is greater than one
directly on the charge of the metal ion and (c) [Zn(NH3)4] [Be(OH)4] : colorless complex
on the field produced by the ligand. (d) [Cr(CN)6]3– : Inner orbital low spin complex
(c) In the presence of Br – as a ligand the 45. Which one of the following complexes is an outer
distribution of electrons for d 4 orbital complex ?
configuration will be t 32 , eg1, (a) [Co(NH3)6]3+ (b) [Mn(CN)6]4–
g
(c) [Fe(CN)6]4– (d) [Ni(NH3)6]2+
(d) In the presence of CN– as a ligand Do < P.
(Atomic nos. : Mn = 25; Fe = 26; Co = 27, Ni = 28)
38. ˆˆ† ML 4
For the reaction of the type M + 4L ‡ˆˆ 46. Which is not p - bonded complex ?
(a) larger the stability constant, lower the (a) Zeise’s salt
proportion of ML4 that exists in solution (b) Ferrocene
(b) larger the stability constant, higher the (c) Dibenzene chromiun
proportion of ML4 that exists in solution (d) Tetraethyl lead
(c) smaller the stability constant, higher the 47. Which of the following carbonyls will have the
proportion of ML4 that exists in solution strongest C – O bond ?
(d) None of the above (a) [Mn (CO)6]+ (b) [Cr (CO)6]
39. An octahedral complex of Co3+ is diamagnetic. (c) [V (CO)6]– (d) [Fe (CO)5]
The hybridisation involved in the formation of 48. In Fe(CO)5, the Fe – C bond possesses
the complex is:
(a) ionic character
(a) sp3d 2 (b) dsp 2 (c) d2sp 3 (d) sp3d
(b) s-character only
40. The correct statement about the magnetic
properties of [Fe(CN) 6 ] 3– and [FeF 6] 3– is (c) p-character
(Z = 26) : (d) both s and p characters
(a) both are paramagnetic. 49. Which of the following does not have a metal-
(b) both are diamagnetic. carbon bond?
(c) [Fe(CN) 6]3– is diamagnetic, [FeF6]3– is (a) Al(OC 2 H 5 )3 (b) C 2 H 5MgBr
paramagnetic.
(c) K[Pt (C 2 H 4 )Cl 3 ] (d) Ni(CO) 4
(d) [Fe(CN)6]3– is paramagnetic, [FeF6]3– is
diamagnetic. 50. In the silver plating of copper, K[Ag(CN)2] is
41. Consider the coordination compound, used instead of AgNO3. The reason is
[Co(NH3)6]Cl3. In the formation of this complex, (a) a thin layer of Ag is formed on Cu
the species which acts as the Lewis acid is: (b) more voltage is required
(a) [Co(NH3)6]3+ (b) Cl– (c) Ag+ ions are completely removed from
(c) Co 3+ (d) NH3 solution
42. Among the following species the one which (d) less availability of Ag+ ions, as Cu cannot
causes the highest CFSE, Do as a ligand is: displace Ag from [Ag(CN) 2]– ion.
(a) CN– (b) NH3 (c) F– (d) CO 51. Pick a poor electrolytic conductor complex in
43. Which one of the following complexes will most solution
likely absorb visible light? (a) K 2 [PtCl 6 ]
(b) [Co ( NH 3 ) 3 ]( NO 2 ) 3
(At nos. Sc = 21, Ti = 22, V = 23, Zn = 30)
(a) [Sc(H2O)6]3+ (b) [Ti(NH3)6]4+ (c) K 4 [ Fe (CN ) 6 ]
(c) [V(NH3)6] 3+ (d) [Zn(NH3)6]2+ (d) [Co ( NH 3 ) 4 ]SO 4
Co-ordination Compounds 267
52. Nickel (Z = 28) combines with a uninegative 59. Which of the following compounds is not
monodentate ligand to form a diamagnetic coloured?
complex [NiL4]2–. The hybridisation involved and (a) Na2[CuCl6] (b) Na2[CdCl4]
the number of unpaired electrons present in the
(c) K4[Fe(CN)6] (d) K3[Fe(CN)6]
complex are respectively: 60. The geometry of Ni(CO)4 and Ni(PPh3)2Cl2 are
(a) both square planar
(a) sp3, two (b) dsp2, zero
(b) tetrahedral and square planar
(c) dsp2, one (d) sp3, zero (c) both tetrahedral
53. Which of the following is diamagnetic ? (d) None of these
(a) [Fe(CN)6]3– (b) [Co(ox)3]3– 61. Which one is the most likely structure of
(c) [FeF6] 3– (d) [CoF6]3– CrCl 3. 6H2O if 1/3 of total chlorine of the
54. The magnetic moment of the complex anion compound is precipitated by adding AgNO3
[Cr(NO) (NH3) (CN)4]2– is : (a) CrCl3. 6H2O
(a) 5.91 BM (b) 3.87 BM (b) [ Cr (H2O)3 Cl3]. (H2O)3
(c) 1.73 BM (d) 2.82 BM (c) [ CrCl2 (H2O)4] Cl . 2H2O
55. In which of the following octahedral complex (d) [ CrCl (H2O)5] Cl2 . H2O
species the magnitude of Do will be maximum ? 62. In the complexes [Fe(H2O)6]3+ , [Fe(CN)6]3–,
(a) [Co (H 2 O)6 ]2+ (b) [Co(CN)6 ]3- [Fe(C2O4)3]3– and [FeCl6]3–, more stability is
shown by
(c) [Co(C2 O4 )3 ]3- (4) [Co (NH3)6]3+
56. The structure of which of the following chloro (a) [Fe(H2O)6]3+ (b) [Fe(CN)6]3–
species can be explained on the basis of dsp2 (c) [Fe(C2O4)3]3– (d) [FeCl6]3–
hybridization ?
63. The d-electron configurations of Cr2+, Mn2+, Fe2+
(a) [PdCl 4 ]2- (b) [FeCl4] 2 - and Co2+ are d 4, d 5, d 6 and d 7 respectively.
Which one of the following will exhibit the lowest
(c) [CoCl4 ]2 - (d) [NiCl 4 ]2 - paramagnetic behaviour?
(Atomic no. Cr = 24, Mn = 25, Fe = 26, Co = 27).
57. [Ni(NH 3 ) 6 ]2++ en ¾¾
® X + 2NH 3 (a) [Co(H2O)6]2+ (b) [Cr(H2O)6]2+
(c) [Mn(H2O)6] 2+ (d) [Fe(H2O)6]2+
[Ni(NH 3 ) 6 ] + 2en ¾¾
® Y + 4NH 3
64. The correct order of ligands in the
[Ni(NH 3 )6 + 3en ¾¾ ® Z + 6NH 3 spectrochemical series is
Which of them sh ow optical as well as (a) Cl– > en > CN– > NCS–
geometrical isomerism? (b) CN– > en > NCS– > Cl–
(c) NCS– > CN– > Cl– > en
(a) X (b) Y (d) en > CN– > Cl– > NCS–
(c) Z (d) All of these 65. Which of the following paramagnetic ions would
58. 50 mL of 0.2 M solution of a compound with exhibit a magnetic moment (spin only) of the order
empirical formula CoCl3.4NH3 on treatment with of 5 BM?
excess of AgNO3 (aq) yields 1.435g of AgCl. (At. Nos. Mn = 25, Cr = 24, V = 23, Ti = 22)
Ammonia is not removed by treatment with (a) Mn 2+ (b) Ti2+ (c) V2+ (d) Cr2+
concentrated H 2SO 4 . The formula of the 66. Square-planar geometry is shown by
compound is: (a) [PtCl2(NH3)2] (b) [NiCl4]2–
(a) [Co(NH3 )4 ]Cl3
(c) MnO-4 (d) CrO24 -
(b) [Co(NH3 )4Cl 2 ]Cl
67. Which molecule/ion among the following cannot
(c) [Co( NH 3 ) 4 ]Cl 3 act as a ligand in complex compounds?
(d) [CoCl3(NH3)](NH3 )3 (a) CH4 (b) CO (c) CN– (d) Br–
EBD_7587
268 Chemistry Objective MCQs
68. Which one of the following complexes will 74. Complexes [Co( NH 3 )5 SO 4 ] Br and
consume more equivalents of aqueous solution
of AgNO3? [Co( NH 3 ) 5 Br]SO 4 can be distinguished by
(a) Na2[CrCl5(H2O)] (b) Na3[CrCl6] (a) conductance measurement
(c) [Cr(H2O)5Cl]Cl2 (d) [Cr(H2O)6]Cl3 (b) using BaCl2
69. Possible isomerism in complexes (c) using AgNO3
[Co(NH3)3(NO2)3] and [Co(NH3)5(NO2)]Cl 2, (d) Both (b) and (c)
respecitvely are: 75. The correct name for the complex ion
(a) Linkage and optical [CoCl(ONO)(en)2]+ is:
(b) Geometrical and linkage (a) chlorobis (ethylenediamine) nitrito-O-
(c) Optical and ionization cobaltate (III) ion
(d) Linkage and geometrical
(b) chlorodiethyldiaminenitrito-O-cobalt (III)
70. Which of the following statements is correct ?
ion
(Atomic number of Ni = 28)
(c) chloronitrito-O-diethyldiamine cobaltate
(a) Ni(CO)4 is diamagnetic and [NiCl4]2– and
(III) ion
[Ni(CN)4]2– are paramagnetic
(d) chlorobis (ethylenediamine) nitrito-O-
(b) Ni(CO)4and [Ni(CN)4]2– are diamagnetic and cobalt (III) ion
[NiCl4]2– is paramagnetic
76. Which of the following ions are optically active?
(c) Ni(CO)4 and [NiCl4]2–are diamagnetic and
[Ni(CN)4]2– is paramagnetic en en
Cl + Cl +
(d) [NiCl4]2– and [Ni(CN)4]2– are diamagnetic
and Ni(CO)4 is paramagnetic
71. For [Co2(CO)8], what is the total number of Co Co
metal – carbon bonds and number of metal–metal
bonds. Cl Cl
en en
I II
(a) 10 ,1 (b) 8, 2 (c) 8, 1 (d) 10, 0
72. [Cr(H2O)6]Cl3 (atomic number of Cr = 24) has a en en
3+ Cl +
magnetic moment of 3.83 BM. The correct
distribution of 3d-elections in the chromium
present in the complex is en Co Co
1 1 1
(a) 3dxy ,3dyz ,3dzx
Cl
en en
(b) 3d 1xy ,3dyz
1
,3dz12
III IV
3d1 2 2 ,3d12 ,3dzx
1 (a) I only (b) II only
(c) (x - y ) z (c) II and III (d) IV only
1 1 1 77. Which is not a p-acceptor ligand ?
(d) 3dxy ,3d(x 2 - y2 ) ,3d xz
(a) I3– (b) NO+
73. [NiCl2 {P(C2H5)2(C6H5)}2] exhibits temperature (c) (CH3)3P (d) CN–
dependent magnetic behaviour (paramagnetic/ 78. Select the complex in which secondary valency
diamagnetic). The coordination geometries of Ni2+ is satisfied before the primary valency.
in the paramagnetic and diamagnetic states are
(a) RMgX (b) [Cr(NH3)6]Cl2
respectively (c) K4[Fe(CN)6] (d) [Ni(CO)4]
(a) tetrahedral and tetrahedral 79. Select the complex that can be reduced most
easily.
(b) square planar and square planar
(a) Ni(CO)4 (b) Cr(CO)6
(c) tetrahedral and square planar (c) V(CO)6 (d) Fe(CO)5
(d) square planar and tetrahedral
Co-ordination Compounds 269
80. The correct order of ligands in the trans-directing 86. In solid CuSO4.5H2O copper is coordinated to
series is (a) 4 water molecules
(a) CN– > CH3– > NO2– > Br– (b) 5 water molecules
(c) one sulphate molecule
(b) CN– > Br– > NO2– > CH3– (d) one water molecule
(c) Br– > NO2– > CN– > CH3– 87. A compound contains 1.08 mole of Na, 0.539 mole
(d) CH3– > CN– > NO2– > Br– of Cu and 2.16 mole of F. Its aqueous solution
shows osmotic pressure which is three times that
81. Among the following complexes (K-P)
of urea having same molar concentration. The
K3[Fe(CN) 6] (K), [Co(NH3) 6]Cl 3 (L), formula of the compound is :
Na 3[Co(oxalate) 3] (M), [Ni(H2O)6]Cl 2 (N),
(a) Na 4 [CuF6 ] (b) Na[CuF4 ]
K2[Pt(CN)4] (O) and [Zn(H2O)6](NO3)2 (P) the
diamagnetic complexes are (c) Na 2 [Cu F4 ] (d) Na 2 [Cu F3 ]
88. Relative to the average energy in the spherical
crystal field, the t2g orbitals is tetrahedral field is
(a) K, L, M, N (b) K, M, O, P (a) Raised by (2/5) Dt
(c) L, M, O, P (d) L, M, N, O (b) Lowered by (2/5) Dt
82. [Co2(CO)8] displays : (c) Raised by (3/5) Dt
(d) Lowered by (1/5) Dt
(a) one Co – Co bond, six terminal CO and two
89. For which value of the x, and y, the following
bridging CO
square planar compound shows geometrical
(b) one Co – Co bond, four terminal CO and four isomers [Pt (Cl)x (Br)y]2–
bridging CO
(a) 1, 3 (b) 3, 1 (c) 2, 2 (d) 1, 1
(c) no Co – Co bond, six terminal CO and two 90. The terahedral complex [M(A)(B)(X)(Y)], where
bridging CO A,B,X and Y are different ligands and M is a
(d) no Co – Co bond, four terminal CO and four metal ion is
bridging CO (a) optically inactive
83. In isolated condition C–C bond length of C2H4 (b) rotate plane polarized light
is x, than the bond length of a C–C bond of C2H4 (c) incomplete information
in Zeise's salt is (d) can’t be said
(a) Greater than x (b) Less than x 91. Incorrect match is :
(c) Equal to x (d) None of these (a) K+ [Pt(C2H4)Cl3]– : Zeise’s salt
84. The number of s and p-bonds in Fe2(CO)9, (b) [Co(CO)4]– : Bond order of CO—CO bond
respectively, are is greater than one
(a) 22s and 15p (b) 22s and 16p (c) [V(NH3)6]3+ absorb visible light
(c) 23s and 15p (d) 15s and 8p (d) [Cr(CN)6]3– : Inner orbital low spin complex
85. Which of the following type of isomerism is 92. Which complex of Co2+ will have the weakest
shown by given complex compound? crystal field splitting –
(a) [CoCl6]4– (b) [Co(CN)6]4–
NH3
(c) [Co(NH3)6] 2+ (d) [Co(en)3]2+
H3N NO2
Co 93. The complex showing a spin-only magnetic
O2N NO2 moment of 2.82 B.M. is :
NH3 (a) Ni(CO)4 (b) [NiCl4]2–
(a) Facial (b) Meridional (c) Ni(PPh3)4 (d) [Ni(CN)4]2–
(c) Cis (d) Both (b) and (c)
EBD_7587
270 Chemistry Objective MCQs
94. Red precipitate is obtained when ethanol solution (a) it is a low spin complex
of dimethylglyoxime is added to ammoniacal (b) it is diamagnetic
Ni(II). Which of the following statements is not (c) it shows geometrical isomerism
true ? (d) (a) and (b) both
97. Which of the following is not chelating agent?
(a) Red complex has a square planar geometry. (a) thiosulphato (b) oxalato
(b) Complex has symmetrical H-bonding (c) glycinato (d) ethylene diamine
(c) Red complex has a tetrahedral geometry. 98. When HgI2 is added to excess of aqueous KI,
(d) Dimethylglyoxime functions as bidentate mercury largely exists as.
ligand. (a) Hg2I2 (b) [HgI3]–
(c) [HgI4]2– (d) none of these
OH
H3C C N 99. In nitroprusside ion, the iron and NO exist as Fe
dimethylglyoxime = +
H3C C N (II) and NO rather than Fe(III) and NO. This
OH
can be established by
95. Which of the following statements is correct for
(a) estimating the concentration of iron
the complex Ca 2 éëFe ( CN )5 O2 ùû having
t 2g6 , eg0 electronic configuration? (b) estimating the concentration of CN -
(c) thermally decomposing the compound
(d) measuring the solid state magnetic moment
(a) d 2sp3 hybridised and diamagnetic 100. The number of geometrical isomers for octahedral
(b) sp3d 2 hybridised and paramagnetic
[Co( NH 3 ) 2 Cl 4 ]- , square planar AuCl 2 Br2-
(c) sp3d 2 hybridised and diamagnetic
(d) d 2sp3 hybridised and paramagnetic and [Co (NO2) (NH3)5]2+are
96. [Fe(en)2(H2O)2]2+ + en ® complex (X). The (a) 2, 2, 2 (b) 2, 2 , no isomerism
correct statement about the complex (X) is – (c) 3, 2, 2 (d) 2, 3, no isomerism

Answer KEYs
1 (c) 11 (c) 21 (b) 31 (a) 41 (a) 51 (d) 61 (c) 71 (a) 81 (c) 91 (d)
2 (b) 12 (c) 22 (c) 32 (d) 42 (d) 52 (a) 62 (c) 72 (a) 82 (a) 92 (a)
3 (d) 13 (a) 23 (b) 33 (a) 43 (c) 53 (b) 63 (a) 73 (c) 83 (a) 93 (b)
4 (d) 14 (d) 24 (a) 34 (b) 44 (d) 54 (d) 64 (b) 74 (d) 84 (a) 94 (c)
5 (d) 15 (b) 25 (d) 35 (a) 45 (d) 55 (b) 65 (d) 75 (d) 85 (b) 95 (d)
6 (a) 16 (b) 26 (a) 36 (b) 46 (d) 56 (a) 66 (a) 76 (c) 86 (a) 96 (d)
7 (c) 17 (c) 27 (b) 37 (d) 47 (a) 57 (b) 67 (a) 77 (b) 87 (c) 97 (a)
8 (d) 18 (b) 28 (a) 38 (b) 48 (d) 58 (b) 68 (d) 78 (d) 88 (a) 98 (c)
9 (c) 19 (b) 29 (b) 39 (c) 49 (a) 59 (b) 69 (b) 79 (c) 89 (b) 99 (d)
10 (c) 20 (c) 30 (b) 40 (a) 50 (d) 60 (c) 70 (b) 80 (a) 90 (b) 100 (b)
Co-ordination Compounds 271

1. (c) AgCl form soluble complex with NH4OH 4. (d) For [CuII(NH3)4][PtIICl4] four isomers are
which is [Ag(NH3)2]+. possible which are [Cu(NH3)4] [PtCl 4],
2. (b) K4[Fe(CN)6] obeys the EAN rule. [CuCl4][Pt(NH3)4],[PtCl3(NH3)][Cu(NH3)3Cl]
and [Pt(NH3)3Cl] [Cu(NH3)Cl3]
3. (d) [Pt(en) 2Cl 2] is a complex of the type
5. (d) The complex ion [Cr(SCN)2(NH3)4]2+ can
[M(AA)2 B2] which is octahedral. Such
exhibit geometrical and linkage isomerism
compounds exhibit optical and geometrical
6. (a) K4 [Fe(CN)6] in solution furnishes 4K++
isomerism both.
[Fe(CN)6]4– five ions and exhibits maximum
ionic conductivity.
2+
7. (c) Fe 2+ in éë Fe ( H 2O )6 ùû
3d 4s 4p 4d

2
Hyb. : sp3d
Colour : Pale green µ = 4.9 B.M.; octahedral
4-
Fe2 + in éë Fe (CN 6 )ùû

3d 4s 4p

2
Hyb. : d sp3
Colour : Yellow; µ = 0; octahedral

8. (d) Ni ( CO )4 ;µeff = 0 9. (c) Degenerate d-orbitals undergo splitting


under ligand field created by strong, weak
+
éë Co ( NH3 )4 ( NO2 )2 ùû ;µeff = 0 or mixed ligands.
10. (c) Number of unpaired electrons = 3
[Ag (CN )2 ];µeff = 0 µeff = 3.9 BM
type of hybridisation = sp3d 2
[CuBr4 ]2- ;µeff = 1.732 BM

11. (c) Magnetic moment, n(n + 2) = 5.92 B.M. Þ n


(unpaired electrons) = 5

2+
Mn :
3d 5 4s 4p
.. .. ..
sp3 hybridization (tetrahedral)
EBD_7587
272 Chemistry Objective MCQs
12. (c) Complex is not superimposable on its mirror image hence optically active i.e., rotate plane polarized
light.

180°
3+ 3+
en en
en Co en
Co

en en
180°
cannot be
superimposed en 3+

non-superimposable en
Co
en

13. (a) The given compound may have linkage 15. (b) Ni2+ (aq) with DMG (dimethylglyoxime) form
isomerism due to presence of NO2 group square planar complex which is obtically
which may be in the form –NO2 or –ONO. inactive. While [Co(EDTA)]–, [Zn(gly)2]2+
[Co(NH3)4(NO2)2]Cl & [Co(NH3)4(ONO)2]Cl and [Pt(en)3]4+ are optically active as they
It may have ionisation isomerism due to do not have P.O.S.
presence of two ionisable groups –NO2 &
–Cl. 16. (b)
[Co(NH3)4Cl(NO2)]NO2 & [Co(NH3) (NO2)2]Cl
It may have geometrical isomerism in the (I) [Cu (CN )4 ]3- ®µ=0
form of cis trans form as follows : (Diamagnetic)
NO2 NO2 3+
(II) éë Co ( NH3 )6 ùû ®µ=0
H3N NH NH3 H3N NO2
(Diamagnetic)
Co Co
2+
H3N NH3 NH3 H3N NH3 (III) éë Ni ( NH3 )6 ùû ® µ= 8 BM

NO2 NH3 (Paramagnetic)

(IV) [Fe ( CN )6 ]3- ®µ= 3 BM


Cr3+ , C2 O24- , ( en ) , O2
0
14. (d)
(Paramagnetic)
C2 O42- and (en)0 are bidenate ligand.
17. (c) As number of stable rings increases
CN of Cr3+ = 6,
stability of complex compound also
A A increases due to chelation effect.

Number of rings in [ Ni (en )2 ] + = 2


B A B A 2

B A B B Number of rings in éë Ni ( dmg )2 ùû = 4

B A
facial isomer meridonal isomer
Co-ordination Compounds 273
18. (b) Same magnetic moment = Same number of 3-
unpaired electrons. 22. (c) Fe3+ in éë Fe (CN 6 )ùû

m = n (n + 2) 3d 4s 4p

where n = number of unpaired electrons


Co2+ = 3d 7, 3 unpaired electrons;
d 2sp3
Cr2+ = 3d 4, 4 unpaired electrons;
Þ low spin complex µ = 1.732 BM
Mn2+ = 3d 5, 5 unpaired electrons;
Ni in [Ni(CO)4]
Fe2+ = 3d 6, 4 unpaired electrons.
sp3
19. (b) [Cu(NH3)5]2+ ® geometry with C. N. = 5 is
sq. pyramidal, dsp 3 hybridisation and 3d 4s 4p
dx2–y2 orbital is used in hybridisation. µ=0
(a) Fe (CO )5 ® d z 2 sp3 3+
Cr3+ in éë Cr ( NH3 )6 ùû
2+
(b) éë Co ( NH3 )6 ùû ® d 2sp3 3d 4s 4p

(d) [IrF6 ]3- ® d 2sp3


d 2sp3
20. (c)
Þ low and high spin complex is applicable
(a) Geometry of complexes ® Both square for d4 to d7 configuration
planar
(b) Hybridisation of central metal cation ® dsp2 Mn2+ in [Mn ( CN )6 ]4 -
(c) Magnetic behaviour [Cu(en) 2)] 2 ® 4p
3d 4s
Paramagnetic; [Ni(dmg)2] ® Diamagnetic
(d) Number of stereoisomers = 0
21. (b)
d 2 sp3
(a) éë Ni ( PF3 )4 ùû Ni : sp3 Hybridization Þ low spin complex µ = 1.732 BM
P: sp3 Hybridization 23. (b) éë Ti ( H 2O )6 ùû Cl 4
(b) éë Fe ( dmg )2 ùû Fe : dsp2 Hybridization Coordination number 6 Þ octahedral
N-atom of dmg : complex
sp3 Hybridization Ti is in +4 oxidations state Þ no unpaired
electrons
(c) [Zn (en )2 ]2+ Zn : sp3 Hybridization
Þ magnetic moment = 0 B.M.
N-atom of en : 24. (a) In d 3 and d 8 octahedral complexes number
sp3 Hybridization of unpaired e– s at central metal atom/ion
2+ never changes, therefore for such
(d) é Ni ( PMe3 ) ù Ni : sp3 Hybridization octahedral complexes terms high spin and
ë 4û
low spin not used.
P : sp3 Hybridization
EBD_7587
274 Chemistry Objective MCQs
25. (d) [Cu(NH3)4]2+ has square planar structure and is paramagnetic.
26. (a) Electronic No. of unpaired
configuration electrons

Co2+ d7 3

Cr2+ d4 4

Mn 2+ d5 5

Fe2+ d6 4

\ Since Co3+ has lowest no. of unpaired electrons hence lowest paramagnetic behaviour is shown
by [Co(H2O)6]2+
27. (b) The two possible isomers for the given octahedral complex are [M(NH3)5 SO4] Cl and [M(NH3)5 Cl]
SO4. They respectively give chloride ion (indicated by precipitation with BaCl2) and SO4 ion (indicated
by precipitation with AgNO3). Hence the type of isomerism exhibited by the complex is ionization
isomerism.
28. (a) The compound shows linkage isomerism because the ligand in the compound is an ambidenate
ligand that can bond at more than one atomic site.
i.e., NCS and SCN
29. (b) Non superimposable mirror images are called optical isomers and may be described as “chiral’.
They are also called enantiomers and rotate plane polarised light in opposite directions.

+ +
Cl Cl
Cl Cl

en Co Co en

en en

30. (b) Complexes of the type MABCD may exist in three isomeric form. Similarily [Pt (NO2)(Py)(NH3)
(NH2OH)+] may exist in three isomeric form.

O2N Py O2N NH3 O2N NH3

Pt Pt Pt

HOH2N NH3 HOH2N Py Py HOH2N


(I) (II) (III)
Co-ordination Compounds 275
31. (a) Symmetrically filled t2g and eg are those, which contain equal distribution of electrons.

[FeF6]3– ­ ­ eg ìequal distribution of ü


ïelectrons in orbitals ï
í ý
­ ­ ­ t 2g ïî t 2g and eg ïþ

[Mn(CN)6]4– eg
unequal
t 2g distribution
­¯ ­¯ ­
of electrons
in orbitals
[CoF6]3– eg
­ ­ unequal
­¯ ­ ­ t 2g distribution
of electrons
unequal
[Co(NH3)6]2+ ­ ­ eg distribution
of electrons
­¯ ­¯ ­ t 2g in orbitals

32. (d) The number of ammonia molecules is 6. 34. (b) Shape of [Cu(NH3)4]2+ is square planar.
Each ammonia molecule contains 3 covalent 35. (a) [Co(NH3) 5CO 3]ClO 4. Six monodentate
bonds between N and H. Therefore the ligands are attached to Co hence C. N. of
number of covalent bonds is 18. Co = 6;
33. (a) Ni(CO)4. The O. S. of Ni is zero. Electronic O. N. = x + 5 × (0) + 1 × (–2) + 1× (–1) = 0 \
configuration is [Ar] 3d 8 4s 2 4p 0 . In x = + 3 ; electronic configuration of Co3+[Ar]
presence of strong ligand CO the paring of 3d64s0 hence number of d electrons is 6.
electrons take place and electronic All d electrons are paired due to strong
configuration will be [Ar] 3d 10 4s0 4p0. ligand hence unpaired electron is zero.
Hence unpaired electrons is zero .
36. (b) [Co(C2O4)3]3– is diamagnetic as oxalate is a strong ligand causing pairing of 3d electrons in
Co3+thereby leading to d 2sp3 hybridisation.

3d 4s 4p
Orbitals of
3+
Co ion
3d 4s 4p
d 2sp3 hybridised
3+
oribitals of Co
2 3
d sp hybrid

37. (d) CN– is a strong field ligand and form low 38. (b) For the reaction of the type
spin complexes thus Do > P.
ˆˆ† ML4 , larger the stability
M + 4L ‡ˆˆ
constant, the higher the proportion of ML4
that exists in solution.
EBD_7587
276 Chemistry Objective MCQs
39. (c) [Co(NH3)6]3+

2 3
(d 2sp 3)

Octahedral and Diamagnetic


Hence [ Ni( NH 3 ) 6 ]2 + is outer orbital
40. (a) Both are paramagnetic, the only difference
is that CN– is a strong field ligand whereas complex.
F – is a weak field ligand. 46. (d) Tetraethyl lead Pb(C2H5)4 is not p bonded
complex. It is s bonded organometallic
3+
41. (a) ® éëCo ( NH3 )6 ùû
Co3+ + 6NH3 ¾¾ compound.
Lewis Lewis adduct
47. (a) As positive charge on the central metal
acid base atom increases, the less readily the metal
42. (d) Following is the order of increasing value can donate electron density into the p*
of Do for the ligands. orbitals of CO ligand (donation of electron
I– < Br– < Cl– < F– < H2O < Py = NH3 < en < density into p* orbitals of CO result in
NO2– << CN– < CO weakening of C – O bond). Hence, the C – O
43. (c) The absorption of visible light and hence bond would be strongest in [Mn(CO)6]+.
coloured nature of the transition metal 48. (d) Due to some backbonding by sidewise
cation is due to the promotion of one or overlapping between d-orbitals of metal and
more unpaired - d - electron from a lower to p-orbital of carbon, the F–C bond has s and
higher level withing same d-subshell. Hence p character.
higher will be the number of unpaired 49. (a) Triethoxyaluminium has no Al – C linkage
electron higher will be the absorpion in
O - CH 2 CH 3
visible light.
Al O - CH 2 CH 3
The electronic configuration of the given
elements is O - CH 2 CH 3
Sc3+ (18) = 1s2 2s2 2p6 3s2 3p6 3d0 4s0 - no 50. (d) In silver plating K[Ag(CN)2] is used which
unpaired e–. provides constant and required supply of
Ti4+ (18) = 1s2 2s2 2p6 3s2 3p6 3d0 4s0 - no Ag+ ions as Ag(CN)2– is very stable. But if
unpaired e–. AgNO3 is used concentration of [Ag+] in
V3+ (20) = 1s2 2s2 2p6 3s2 3p6 3d2 4s0 - Two solution will be very large. In that case Ag
unpaired e–. will be deposited at faster rate without any
Zn2+ (28) = 1s2 2s2 2p6 3s2 3p6 3d10 4s0 - no uniformity.
unpaired e–. 51. (d) The complex furnishing least number of ions
hence [V(NH3)6]3+ will most likely absorb in solution will be poor electrolytic
visible light. conductor.
44. (d) Cr(CN)6]3– : Hybridization : d2sp3 High or 52. (a) [NiL4]2–
low spin is not defined because Cr3+ is an 3d 4s 4p
example of d3 system of configuration. Ni 2+ ( d 8 )
45. (d) Hybridisation

[Fe(CN)6 ]4 -,[Mn(CN)6 ]4 - ,
2 3 2 3
[NiL4 ]2–
d sp d sp
sp3
[Co(NH 3 ] 3+ ,[Ni(NH 3 ) 6 ] 2+
i.e, no. of unpaired electron = 2
d 2 sp 3 sp 3d 2
hybridization – sp3.
Co-ordination Compounds 277
53. (b) [Fe(CN)6]3– has magnetic moment of a single C2O42– < H2O < NH3 < CN –
unpaired electron whereas [FeF6]3– has a hence the crystal field splitting will be
magnetic moment of five unpaired electrons. maximum for [Co(CN)6]3–
[CoF6]3– is paramagnetic with four unpaired 56. (a) [PdCl4]2– is dsp2 hybridized and square
electr ons while [Co(C 2 O 4 ) 3 ] 3 is planar in shape.
diamagnetic. This anomaly is explained by
2+
valence bond theory in terms of formation 57. (b) X = éë Ni (en )( NH3 )4 ùû neither G.I. nor OI
of inner and outer orbital coordination
entities. [Co(C2O4)3]3– is an inner orbital 2+
Y = éë Ni (en )2 ( NH3 )2 ùû G. I as well as OI
complexes having d2sp3 hybridization.
54. (d) In [Cr (NO) (NH3) (CN)4]2–,
Z = [ Ni (en )3 ]2+ does not show G.I. while
Cr2+(d4) is given as :
show OI
1.435
58. (b) Mole of AgCl = = 0.01; 50 mL of 0.2
143.5
M complex º 0.01 mole
i.e., 2 unpaired electrons
[Co(NH3 ) 4Cl 2 ]Cl ® [CoNH3 ) 4Cl2 ]+ + Cl -
m = 2(2 + 2) = 8 = 2.82 BM
59. (b) Na2[CdCl4] does not contain any unpaired
55. (b) Crystal field splitting depends upon the electron hence colourless
nature of ligand. The nature of ligand D
decreases as shown below

60. (c) Ni(CO)4 Ni (PPh3)2 Cl2


O.S. Ni0 Ni2+
E.C. [Ar]3d 84s2 [Ar]3d 84s0
Pairing Pairing of e– No pairing of e–
Hybridization sp3 (tetrahedral) sp3 (tetrahedral)

61. (c) The ions present in the ionisation sphere are 63. (a) Electronic No. of unpaired
precipited Hence [CrCl 2 (H 2O) 4 ]Cl.2H 2 O configuration electrons
contains 1/3 Cl in ionisation sphere to be Co2+ d7 3
precipited by AgNO3 as AgCl
62. (c) [Fe(C2O4)3]3–. The iron is present in the
Cr2+ d4 4
highest oxidation state Fe3+ and C2 O 24 - is a
chelating ligand. Chelates are always form
Mn 2+ d5 5
more stable complexes

Fe2+ d6 4

\ Since Co2+ has lowest no. of unpaired


electrons hence lowest paramagnetic
behaviour is shown by [Co(H2O)6]2+
EBD_7587
278 Chemistry Objective MCQs
64. (b) Correct order is : CN– > en > NCS– > Cl– 67. (a) The donor atoms, molecules or anions
65. (d) which donate a pair of electrons to the metal
Ion Electronic Magnetic atom or ion and form a coordinate bond
Configuration moment n (n + 2) ( ) with it are called ligands. In methane there
is no electrons for donation to central metal
Cr2+ d 4 4(4 + 2) = 4.9 atom/ion it is stable with complete octet
configuration.
Mn 2+ d 5 5(5 + 2) = 5.8 68. (d) More equivalents of AgNO 3 aqueous
solution will be consumed if complex will
Ti2+ d 2 2(2 + 2) = 2.4 furnish more Cl– ions in solution. Hence
complex [Cr(H2O)6]Cl3 will consume more
V2+ d 3 3(3 + 2) = 3.8 equivalents of aqueous solution of AgNO3.
Hence Cr2+ has magnetic moment of the 69. (b) [Co(NH3)3(NO2)3]Cl 2 complex exhibits
order of 5 B.M. geometrical isomerism (G.I.)
Geometrical isomers = 2 (1 cis + 1 trans)
66. (a) [PtCl2(NH3)2] has square planar geometry.
Optical isomers = 0
H3N Cl Cl H3N Cl Space isomers = 2
[Co(NH3)5(NO2)]Cl2 complex shows linkage
and ionization isomerism.
Pt Pt

Cl NH NH3 H3N Cl
trans-Isomer cis-isomer

70. (b) No. of


Atom/Ion Configuration Magnetic
Complex unpaired
nature
electrons
3d 4s 4p
2+ 8
Ni (d ) 2
2–
[NiCl4]
2

sp3
2–
[Ni(CN)4] 0
Rearrangement dsp
2

Ni (d 8s2) 2
[Ni(CO)4]
0

Rearrangement
sp3
Co-ordination Compounds 279
71. (a) Structure of [Co2(CO)8]

OC C CO
OC Co Co CO
OC C CO

Total M – C bonds = 10,


Total M – M bonds = 1
72. (a) Magnetic moment indicates that there are three unpaired electrons present in chromium. These must
be present in lower energy orbitals which are 3dxy, 3dyz and 3dxz.
73. (c) In both states (paramagnetic and diamagnetic) of the given complex, Ni exists as Ni2+ whose electronic
configuration is [Ar] 3d 84s0.
3d 4s 4p
Ni2+ :

sp3
In the above paramagnetic state, geometry of the complex is sp3 giving tetrahedral geometry.
The diamagnetic state is achieved by pairing of electrons in 3d orbital.
3d 4s 4p

dsp2
Thus the geometry of the complex will be dsp2 giving square planar geometry.

74. (d) [Co( NH 3 ) 5 Br]SO 4 gives white precipitate


78. (d) In metal carbonyls the oxidation state of
of BaSO 4 with BaCl 2(aq)whereas metal is zero. In this case oxidation state of
[Co( NH 3 )5 SO 4 ] Br gives yellow Ni is zero.
79. (c) In it the E.A.N. of V is 35(23 + 12 = 35) so it
precipitate (AgBr) with AgNO3(aq).
can be reduced easily.
75. (d) The correct name is chlorobis
In all other cases the E.A.N. is 36.
(ethylenediamine) nitritocobalt (III).
80. (a) The trans-effect may be defined as the
76. (c) Ions I and IV are the same (trans), with mirror
labelization of ligands trans- to the other
plane through en groups.
trans-directing ligand.
77. (b) NO+ is not a p-acceptor ligand, because in
[Note : The order of ligands in the trans-
it nitrogen atom has no vacant orbital to
directing series is as follows :
accomodate electron pair from the central
CN– ~ CO ~ NO ~ H– > CH3– ~ SC(NH2)2 ~
metal atom.
PR3 > SO3H > NO2– ~ I– ~ SCN– > Br– >
Cl– > Py > RNH2 ~ NH3 > OH– > H2O]
EBD_7587
280 Chemistry Objective MCQs

81. (c) Complex No. of electrons No. of unpaired


in outer d electron (s)
orbital
[Fe(CN)6]3– 3d 5 1 (CN – causes
pairing of
electrons)
[Co(NH3)6 ]3+ 3d 6 0
3– 6
[Co(OX)3] 3d 0
2+ 8
[Ni(H2O)6] 3d 2

[Pt(CN)4]2– 5d 8 0 (CN causes
pairing of
electrons)
[Zn(H2O)6]2+ 3d 10 0

Thus L, M, O and P are diamagnetic.

82. (a) OC CO CO (terminal)

OC Co Co CO

OC CO CO
(bridging)

83. (a) Cl
H
+ Cl C
K Pt H
H
Cl C
H
With increase in synergic bonding, C–C bond length increases in Zeise’s salt.
84. (a)
:O:

+ –
C – +
:O C C O:
+ – – +
:O C Fe Fe C O:
+ – – +
:O C C C C O:
O O
s = 22 and p = 15 bonds
Co-ordination Compounds 281
85. (b) Given compound shows meridional Empirical formula = Na 2 Cu F4 : Van’t Hofff
isomerism.
factor (i) = 3 (given)
86. (a) Four water molecules are coordinated to
Hence, formula of the compound :
copper.
H2O OH2 H--O O Na 2 [Cu F4 ] ® 2Na + + [Cu F4 ]2-
Cu O S 88. (a) t2g orbitals have higher energy by (2/5)Dt.
H2O OH2 H--O O
89. (b) Geometrical isomers of following type of
87. (c) Mole ratio of Na, Cu and square planar complexes is possible.
1.08 0.539 2.16 Ma2b2 type, Ma2bc type and Mabcd type.
F= : : = 2 :1: 4
0.539 0.539 0.539

90. (b) Non –superimposable mirror images are optically active, hence rotate plane polarized light.

a a

M M
b d b
d
c c
whatever angle of molecule
91. (d) Cr(CN)6]3– : Hybridization : d 2sp3 High or low spin is not defined because Cr3+ is an example of
d 3 system of configuration.
92. (a) Cl– is a weak field ligand.
93. (b) [NiCl4]2–, O.S. of Ni = +2
Ni(28) = 3d 8 4s2
3d 4s 4p
2+
Ni :

3d 4s 4p
2+
[NiCl4] :

sp3
No. of unpaired electrons = 2
Magnetic moment, m = 2.82 BM.
EBD_7587
282 Chemistry Objective MCQs
94. (c) Nickel ions are frequently detected by the formation of red precipitate of the complex of nickel
dimethylglyoxime, when heated with dimethylglyoxime.

OH O

CH3 C NOH CH3 C N N C CH3


+ Ni2+ ¾® Ni
CH3 C NOH CH3 C N N C CH3
Dimethylglyoxime
O OH
Nickel dimethylglyoxime

3d 4s 4p

95. (d) Fe2+ :


Fe2+ in K4 [FeII (CN)5(O2)]
3d 4s 4p

2
hybridisation : d sp3

Complex is paramagnetic due to presence of unpaired electron at O–2 i.e., superoxide acting as ligand.
96. (d) Complex X is [Fe(en)3]2+ ; as 'en' is a strong field ligand pairing of electrons will take place.
3d 4s 4p

[Fe(en)3]2+ :

d 2sp3 hybridisation

Hence, hybridisation is d 2sp3 and complex is diamagnetic. As it has 3 bidentate symmetrical 'en'
ligands so it will not show geometrical isomerism.
97. (a) S2 O32 - is monodentate ligand where as other ligands are bidentate.

98. (c) HgI2 (s) + 2I - (aq) ® [Hg I 4 ]2- (aq)


99. (d) Fe(II) state – 3d 6 ; due to strong CN– ligand spin paired complex (d 2sp3) will be formed .
Hence n = 0, m = 0 B.M
Fe(III) state 3d5; d2sp3 complex, n = 1,
m = 1.73 B. M.
100. (b) The number of geometrical isomers
[Co(NH3)2(Cl)4]– –2
AuCl2Br2 – 2
[Co(NO2)(NH3)5]2+ – No isomerism
Haloalkanes and
24
Haloarenes
1. Arrange the following compounds in order of (b) C6 H 5 - CH 2 - CH 2 - CH - CH3
increasing dipole moment : |
Cl
(I) Toluene CH3
(II) m-dichlorobenzene (c) C6H5–CH2–C
(III) o-dichlorobenzene CH3
Cl
(IV) p-dichlorobenzene
(a) I < IV < II < III (b) IV < I < II < III CH2 – CH2 – CH3
(c) IV < I < III < II (d) IV < II < I < III CH2 – Cl
2. Pure chloroform is prepared by (d)
(a) distilling chloral hydrate with aqueous
sodium hydroxide. 5. The Wurtz-Fittig reaction involves condensation
(b) heating ethanol with bleaching powder. of
(c) heating acetone with bleaching powder. (a) two molecules of aryl halides.
(d) reducing carbon tetrachloride. (b) one molecule of each of aryl-halide and
3. The reaction conditions leading to the best yields of alkyl-halide.
C2H5Cl are : (c) one molecule of each of aryl-halide and
phenol.
UV light
(a) C2H6 (excess) + Cl2 ¾¾¾¾® (d) two molecules of aralkyl-halides.
6. For the compounds
dark
(b) C2H6 + Cl2 ¾room
¾ ¾ ¾ ¾ ¾¾®
temperatur e CH3Cl, CH3Br, CH3I and CH3F,
the correct order of increasing C–X bond length
UV light is:
(c) C2H6 + Cl2 (excess) ¾¾¾¾®
(a) CH3F < CH3Cl < CH3Br < CH3I
UV light
(d) C2H6 + Cl2 ¾¾¾¾® (b) CH3F < CH3Br < CH3Cl < CH3I
4. A compound A with molecular formula C10H13Cl (c) CH3F < CH3I < CH3Br < CH3Cl
gives a white precipitate on adding silver nitrate (d) CH3Cl < CH3Br < CH3F < CH3I
solution. A on reacting with alcoholic KOH gives 7. The order of reactivity of the given haloalkanes
compound B as the main product. B on
towards nucleophile is :
ozonolysis gives C and D. C gives Cannizaro
reaction but not aldol condensation. D gives (a) RI > RBr > RCl (b) RCl > RBr > RI
aldol condensation but not Cannizaro reaction. (c) RBr > RCl > RI (d) RBr > RI > RCl
A is : 8. AgNO3 does not give precipitate with chloroform
(a) C6H5 – CH2 – CH2 – CH2 – CH2 – Cl because:
(a) CHCl3 does not ionise in water.
EBD_7587
284 Chemistry Objective MCQs
(b) CHCl3 is insoluble in water. 13. When CH3CH2CHCl2 is treated with NaNH2, the
(c) AgNO3 is insoluble in CHCl3. product formed is
(d) CHCl3 is an organic compound.
9. Consider the following anions. (a) CH3 — CH = CH2
(b) CH 3 — C º CH
O O
NH2
CF3 S O– C6H5 S O–
(c) CH3CH2CH
O O NH2
(I) (II)

Cl

O O (d) CH3CH2CH
NH2

CH3 C O
(IV)
14. CH 3 Br + Nu - ¾
¾® CH 3 - Nu + Br -
(III)
The decreasing order of the rate of the above
When attached to sp3-hydridized carbon, their
reaction with nucleophiles (Nu–) A to D is
leaving group ability in nucleophilic substitution
reaction decreases in the order: [Nu– = (A) PhO–, (B) AcO–, (C) HO–, (D) CH3O–]
(a) I > II > III > IV (b) I > II > IV > III
(c) IV > I > II > III (d) IV > III > II > I
AgCN Reduction (a) A > B > C > D (b) B > D > C > A
10. C2 H5 Br ¾¾¾¾
® X ¾¾¾¾¾® Y , Here
Zn—Hg/HCl (c) D > C > A > B (d) D > C > B > A
Y is 15. In a nucleophilic substitution reaction:
(a) Ethyl methyl amine(b) n-propylamine
R – Br + Cl - ¾¾¾® R – Cl + Br - ,
DMF
(c) Isopropylamine (d) Ethylamine
Which one of the following undergoes complete
11. The chief reaction product of reaction between
inversion of configuration?
n-butane and bromine at 573K is :
(a) C6H5CHC6H5Br (b) C6H5CH2Br
(a) CH3CH2CH2CH2Br
(c) C6H5CHCH3Br (d) C6H5CCH3C6H5Br
(b) CH 3CH 2 CHBr 16. Which of the following statements is wrong?
|
CH3 (a) Ethyl chloride on reduction with Zn-Cu
couple and alcohol gives ethane.
(c) CH3 - CH2 CHBr (b) The reaction of methyl magnesium bromide
|
CH2Br with acetone gives butanol-2.
(c) Alkyl halides follow the following reactivity
(d) CH3 CH 2 CBr2 sequence on reaction with alkenes.
|
CH3 R – I > R – Br > R – Cl > R – I
12. Aryl fluoride may be prepared from arene (d) C2H4Cl2 may exist in two isomeric forms.
diazonium chloride using : 17. Rate of SN2 will be negligible in :
(a) HBF4/D
Br Br
(b) HBF4/NaNO2,Cu, D
(c) CuF/HF
(d) Cu/HF (a) (b)
Haloalkanes and Haloarenes 285
20. The total number of acyclic isomers including
Br Br
the stereoisomers with the molecular formula
C4H7Cl
(c) (d)
(a) 11 (b) 12 (c) 9 (d) 10
18. A major component of Borsch reagent is obtained 21. The major product formed when 1, 1, 1-trichloro-
by reacting hydrazine hydrate with which of the propane is treated with aqueous potassium
hydroxide is:
following ?
(a) Propyne (b) 1-Propanol
Cl
(c) 2-Propanol (d) Propionic acid
Cl
22. Which one of the following reagents is not
NO2 O2N NO2
(a) (b) suitable for the elimination reaction ?
Br
¾¾®
Cl Cl (a) NaI/acetone
NO2 O 2N NO2 (b) NaOEt/ EtOH
(c) (d)
(c) NaOH / H2O
NO2 (d) NaOH / H2O – EtOH
NO2
23. Identify Z in the following serie’s
19. Chlorobenzne reacts with trichloro acetaldehyde
Alc. KOH Br KCN
in the presence of H2SO4. C 2 H 5 I ¾¾ ¾ ¾
¾® X ¾¾
¾2 ® Y ¾¾¾® Z

O (a) CH3CH2CN (b) NCCH2–CH2CN


H2SO4 (c) BrCH2– CH2CN (d) BrCH = CHCN
Cl + H – C – CCl3 ¾¾¾®
2 24. Among the following, the molecule with the
lowest dipole moment is
(a) CHCl3 (b) CH3Cl
The major product formed is:
(c) CH2Cl2 (d) CCl4
Cl 25. Which chloride is least reactive with the
(a) Cl hydrolysis point of view?
C Cl
(a) CH3Cl (b) CH3CH2Cl
Cl (c) (CH3)3CCl (d) CH2 = CH – Cl
Cl 26. The reaction is described as
CH3(CH2)5
(b) Cl C Cl OH– (CH2)5CH3
C – Br ¾¾® HO – C
CH2Cl H H
CH3 CH3

(a) SE 2 (b) SN 1 (c) SN 2 (d) SN 0


(c) Cl CH Cl
CCl3 27. Replacement of Cl of chlorobenzene to give
phenol requires drastic conditions but chlorine
of 2, 4-dinitrochlorobenzene is readily replaced.
(d) Cl CH Cl This is because

Cl
EBD_7587
286 Chemistry Objective MCQs

(a) NO2 makes the ring electron rich at ortho 32. Which of the following is not expected to be
and para positions. intermediate of the following reaction?
(b) NO2 withdraws e– from meta-position.
OH
(c) NO2 donates e– at meta-position. I
(d) NO2 withdraws e– from ortho/para-positions. H2O

28. CH 3 - CH 2 - C H - CH 3 obtained by
|
Cl +
chlorination of n-butane, will be +

(a) meso-form (b) racemic mixture (a) (b)


(c) d-form (d) l-form
29. Which compound in each of the following pairs
+
is most reactive to the conditions indicated ? OH2
CH 2 Br CH 3 +
Br (c) (d)
(A) and (B) (KOH in CH3OH)

33. Among the given halides, which one will give


CH 3 CH 3
same product in both SN1 and SN2 reactions?
Cl Cl
(C) and (D) (NaNH2 in NH3 )
CH 3 CH3

(a) A and C (b) B and C (I) CH3 CH CH2 CH CH3


(c) A and D (d) B and D Br
30. 2-Bromopentane is heated with potassium
ethoxide in ethanol. The major product obtained CH3 Cl
is (II) (III)
(a) 2-Ethoxypentane (b) Pentene-1
Cl
(c) cis-Pentene-2 (d) trans-Pentene-2
31. In the following reaction, compound (B) is (IV) CH3 CH Br

Et
Dry D
Br + Mg ¾¾¾ ®A ¾¾®B (a) (III) only (b) (I) and (II)
ether
(1 eq.)
(c) (III) and (IV) (d) (I), (III) and (IV)

Br NaCN Ni/H Acetic


34. CH3CH 2Cl ¾¾¾¾
® X ¾¾¾¾
2 ® Y ¾¾¾¾
®Z
NaCN Acetic
Br CH CH Cl ¾¾¾¾
® X ¾¾¾¾
® Y ¾¾¾¾
®Z
(a) Br (b) anhydride

Z in the above reaction sequence is


Br (a) CH3CH2CH2NHCOCH3
MgBr
(b) CH3CH2CH2NH2
(c) CH3CH2CH2CONHCH3
(c) (d)
(d) CH3CH2CH2CONHCOCH3
Haloalkanes and Haloarenes 287
35. Which of the following is most reactive towards (a) i > ii > iii (b) ii > iii > i
SN2 reaction? (c) i > iii > ii (d) iii > ii > i
Cl Cl 37. Which compound undergoes nucleophilic
substitution with NaCN at the fastest rate?
(a) (b)

CH3
Cl Cl Br
(a) Br (b)

(c) (d) Br
(c) Br (d)

Cl NO2 38. Identify correct reactivity order for SN1 reaction

36. Identify correct reactivity order for SN1 reaction Cl Cl Cl

Cl Cl Cl
(i) (ii) (iii)
(i) (ii) (iii) O N B
H H
(a) i > ii > iii (b) ii > iii > i
(c) i > iii > ii (d) ii > i > iii
39. Which one of the sequences below is the best synthesis of (E)-3-hexene?

O
– + –OC (CH
(a) ( i) CH Li HBr 3)
¾¾¾¾¾¾¾
2
® ¾¾¾
® ¾¾¾¾¾¾
3 ®
H (ii) H 2O HOC (CH 3 )3

– Br (i ) NaH Na / NH
(b) C CH ¾¾¾® ¾¾¾¾¾¾ ® ¾¾¾¾3¾ ®
(ii )
Br

O
– +
(c) (i )
¾¾¾¾¾¾¾
CH 2 Li HBr
® ¾¾¾
–OCH CH
® ¾¾¾¾¾¾
2 3 ®
Me (ii ) H 2O HOCH 2CH3

– Br (i ) NaH Na / NH
(d) C CH ¾¾¾¾¾ ® ¾¾¾¾¾
(ii ) CH Br
® ¾¾¾¾3¾ ®
3

40. Which of the following reacts at the fast rate F F


f f
with CH3OK in CH3OH ? O2 N NO2
F (c) (d)
F f
f
NO2
(a) (b) NO2 NO 2
41. In the following sequence of reactions
NO 2 Mg HCHO
H 2O
CH3 CH 2 I ¾¾¾® B ¾¾¾¾
® C ¾¾¾ ®D
ether
A

the compound D is
EBD_7587
288 Chemistry Objective MCQs
(a) propanal (b) butanal The compound I is :
(c) n-butyl alcohol (d) n-propyl alcohol
42. The correct product obtained in the reaction (a) CH 2 - CH - CH3

H 3C Cl Cl
Br OH –
H H ¾¾¾® (b) CH 2 - CH2 - CH3
Cl Cl
(c) Cl – CH – CH2 – CH3
Cl
H 3C H
(a)
H OH Cl
(d) CH3 – C – CH3
H 3C OH
(b) Cl
H H
45. Compound that on hydrogenation produces
OH optically inactive compound is
H 3C OH
(c)
H H
H Br
(a)
H 3C H 3C CH 3
(d)
H
H Br
43. 2-phenyl-2-hexanol can be prepared by Grignard (b) CH3
H 2C
synthesis. The pair of compounds giving the
desired product is H Br
(c) H2C
O CH3
C CH3
(a) Br and Ph CH3
Br H
O (d) H C
2
C 2H 5
(b) Br and Ph C
46. Which of the following order is not correct ?
(a) MeBr > Me 2CHBr >
Me3CBr > Et 3CBr(SN 2)
(c) Ph and PhBr (b) PhCH 2Br > Ph CHBrMe > PhCBrMe 2 >
(d) None of these PhCBrMePh(SN 1)
44. In the following reaction sequence : (c) MeI > MeBr > MeCl > MeF (SN 2)
KOH(aq) (i)CH 3MgBr (d) All are correct
I ¾¾¾¾® II ¾¾¾¾¾¾ ® III
(C3H6 Cl 2 ) (ii) H 2O/H + 47. The major product of the following reaction is :
CH3
given
Anhy.ZnCl2 + Conc.HCl
|
C H ONa
¾¾¾¾¾¾¾¾¾® turbidity C6 H5 CH 2 - C - CH 2 - CH3 ¾¾¾¾¾
2 5 ®
| C2 H5OH
immediately
Br
Haloalkanes and Haloarenes 289

CH3 HBr HBr


CH 2 = CH - CH = CH 2¾¾¾® A ¾¾
¾® B
| 80°C
(a) C6 H5CH2 - C - CH 2 - CH3
| Mg
¾¾
¾® C
OC2H5

(b) C6 H5CH = C - CH 2 - CH3


| (a) A is CH 3 - CH = CH - CH 2 Br
CH3
(b) B is CH 3CHBr - CH 2 - CH 2 - Br
(c) C6 H5 CH 2 - C = CHCH 3
| (c) C is cyclopropyl methane
CH3 (d) B on SN2 with aqueous NaOH gives
(d) C6 H5CH 2 - C = CH 2 CH 3CHOH.CH 2CH 2 Br and not
|
CH 2CH3 CH 3CHBr. CH 2 CH 2 OH
48. The compound most reactive towards S N1 51. The final product obtained in the reaction
reaction
(a) Me 3COCH 2 Cl (b) MeOCH 2 Cl Br

(c) C 6 H 5CH 2 CH 2 Cl (d) Cl Mg / ether Heavy water


¾¾ ¾ ¾
¾® A ¾¾ ¾¾ ¾®
49. Which chloro derivative of benzene among the
following would undergo hydrolysis most readily CH 2Cl
with an aqueous sodium hydroxide to furnish
the corresponding hydroxy derivative?
D OD

(a) (b)
NO2
CH 2Cl CH 2Cl
(a) O2 N Cl Br Br

NO (c) (d)
2

CH 2D CH 2OD
(b) O2N Cl
52. The following reaction proceeds through the
intermediate formation of
(c) Me 2N Cl
¾® RBr + CO 2 + AgBr
RCOOAg + Br2 ¾
(d) C6H5Cl (a) RCOO · (b) R·
50. In the following sequence of reactions, which is
incorrect (c) Br · (d) All of these
EBD_7587
290 Chemistry Objective MCQs
53. The compound 57. Although hexafluoroethane (C2F6, b.p. –79ºC)
3Cl / D Br / Fe Zn / HCl and ethane (C2H6, b.p. – 89ºC) differ very much
C 7 H 8 ¾¾ ¾
2 ¾® A ¾¾2¾¾® B ¾¾ ¾¾® C
in their molecular weights, their boiling points
The compound C is differ only by 10ºC. This is due to
(a) low polarizability of F
(a) o-Bromotoluene
(b) nearly similar size of F and H
(b) m-Bromotoluene
(c) p-Bromotoluene (c) both (a) and (b)
(d) 3-Bromo-2, 4, 6-trichlorotoluene (d) Neither of the two
54. The product of 1-bromo-3-chloro cyclobutane 58. Which of the following is a key intermediate in
with Na in presence of dioxane the reaction shown below ?

(a) Br Br Cl NH 2

NaNH , NH
(b) Br Cl ¾¾¾¾¾¾
2 3®
–33 °C

(c) Cl Cl
Cl NH 2
(d) None of these :–
(a) (b)
55. Which of the following has the weakest carbon -
chlorine bond ?

Cl Cl Cl H
Cl NH2 +
(a) (b) + _
(c) (d)
:
CH 3
NH2
CH 3
59. Which of the following structure is more stable?
CH 2 Cl Cl
Z Cl Z Cl
CH 3 –
(c) (d)
(a) (b)

HCl NO2 NO2
¾¾® A
56. Cl3C.CH = CH2¾ Br
¾¾® 2
B Z Cl
Which of the following is correct ? Z Cl

(a) A on reaction with aq. KOH gives
(c) (d)
HOCH 2CH 2 COOK
NO2 – NO2
(b) B can be resolved into d– and l –forms
(c) Both (a) and (b)
(d) Neither (a) nor (b)
Haloalkanes and Haloarenes 291
60. Which of the following is (are) true concerning 63. Silver benzoate reacts with bromine to form
the intermediate in the addition-elimination
mechanism of the following reaction ? COOBr

Ff
(a) (b)
CH ONa
3 ¾® [intermediate] ¾
¾¾ ¾ ¾®
COOAg
NO 2 O CH 3
(c) (d) C 6 H 5Br
Br
OH - OH- H+
64. PhCOCHBr2 ¾¾¾® A ¾¾¾® B ¾¾¾
®C
NO 2 The compound C is –
A = The intermediate is aromatic, B = The (a) PhCH(OH)CHO
intermediate is a resonance stabilised anion, (b) PhCH(OH)COOH
C = Electron withdrawing groups on the benzene
(c) PhCOCBr2
ring stabilises the intermediate |
H
(a) only A (b) only B (d) Ph - C - CH 2 - OH
||
(c) A and C (d) B and C O
61. Identify Z in
CH 3CH 2 CH 2 Br ¾¾¾¾¾
®X
aq. NaOH 14 Cl
400ºC
2 3 Al O 2 2 Cl /H O 65. + NaOH(aq.) ¾¾® Product is
¾¾¾¾ ® Y ¾¾¾¾ ¾
®Z
(a) Mixture of CH3CHClCH2Cl and
CH3CHOHCH2Cl
(b) CH3CHOHCH2Cl
(c) CH3CHClCH2OH 14 14 OH
(d) CH3CHClCH2Cl (a) (b)
OH
62. CH 2 CH 2 + HBr (c) Both (a) and (b) (d) No reaction
66. X in the following reaction is –
peroxide
¾¾ ¾¾® A, then A is CH3 - C - H CCl4
Br2 + || ¾¾¾ ®X
(a) CH 3 – CH 2 Br H - C - CH3

Br (a) (+) 2, 3-Dibromobutane


(b) (–) 2, 3-Dibromobutane
(b) CH 3 CH 3 (c) (±)- 2, 3-Dibromobutane
Br Br (d) meso-2, 3-Dibromobutane
67. The missing reagents R1 and R2 in the following
Br Br
series of reactions are
(c) CH 3 CH 3 R R -
1
CH 3CH 2 Br ¾¾® 2
[ ] ¾¾¾ ® CH 3C HP + Ph 3

(d) BrCH 2 CH 2 Br
EBD_7587
292 Chemistry Objective MCQs
(a) PhLi and Ph3P respectively (a) 2 > 1 > 3 (b) 1 > 2 > 3
(b) Ph3P and PhLi respectively (c) 2 > 3 > 1 (d) 1 > 3 > 2
(c) Ph3P and C2H5ONa respectively 70. Which of the following alkyl halide undergo
rearrangement in SN1 reaction?
(d) Either (b) or (c)
68. The reaction of toluene with chlorine in presence CH3
of ferric chloride gives predominantly : (a) CH3 C CH CH3
(a) benzoyl chloride
(b) m-chlorotoluene CH3 I
(c) benzyl chloride Cl
(d) o- and p-chlorotoluene
(b)
69. Arrange the following in order of decreasing rate
of solvolysis with aqueous ethanol (fastest ® CH3
slowest)
I
CH3 CH3 (c) (d) All of these
H2C C Br Br
(1) (2)

Br

CH3 CHCH2CH(CH3)2
(3)

Answer KEYs
1 (b) 8 (a) 15 (c) 22 (a) 29 (a) 36 (d) 43 (a) 50 (d) 57 (c) 64 (b)
2 (a) 9 (b) 16 (b) 23 (b) 30 (d) 37 (a) 44 (d) 51 (c) 58 (b) 65 (c)
3 (a) 10 (a) 17 (c) 24 (d) 31 (c) 38 (d) 45 (b) 52 (d) 59 (b) 66 (d)
4 (c) 11 (b) 18 (c) 25 (d) 32 (a) 39 (b) 46 (b) 53 (b) 60 (d) 67 (d)
5 (b) 12 (a) 19 (c) 26 (c) 33 (c) 40 (c) 47 (b) 54 (c) 61 (b) 68 (d)
6 (a) 13 (b) 20 (b) 27 (d) 34 (a) 41 (d) 48 (b) 55 (c) 62 (d) 69 (c)
7 (a) 14 (c) 21 (d) 28 (b) 35 (d) 42 (a) 49 (a) 56 (c) 63 (d) 70 (d)
Haloalkanes and Haloarenes 293

1. (b) In p-dichlorobenzene, the two equal dipoles are in opposite direction, hence the molecule has zero
dipole moment. In o- and m- dichlorobenzenes, the two dipoles are at 60° and 120° apart respectively,
and thus according to parallelogram law of forces, the dipole moment of o-dichlorobenzene is much
higher than that of m-isomer. Lastly, toluene with a +I group possesses little dipole moment. Thus the
overall order is

Cl CH3 Cl Cl
Cl
< < <
Cl
Cl
IV I II III
2. (a) Cl3CCH(OH) 2 + NaOH ¾¾
® CHCl3 + HCOONa + H 2O

3. (a) C2H6 (excess) + Cl 2 ¾¾¾¾ UV light


® C2 H 5Cl + HCl
4. (c) Compound A reacts with alc.KOH to give compound B which on further ozonolysis gives C (does
not contains a- H atom) and D (contains a-H atom).This reaction sequence can be achieved by
compounds in option (a) and (c). Since compound A gives white ppt. with AgNO3 preferable option
will be (c) as tert alkyl reacts with AgNO3 more quickly.

CH3 CH3
C6H5 – CH2 – C alc. KOH
¾¾¾¾® C6H5 – CH = C

CH3 CH3
Cl (i) O 3
(A) (ii) Zn/H 2O
(it gives white ppt with AgNO3)
O O
C6H5 – C – H + CH3 – C – CH3
(C) (D)
(it gives cannizaro (It gives aldol condensation only)
reaction only)

5. (b) Reaction between alkyl halide, aryl halide and sodium in presence of ether is known as Wurtz fitting
reaction
C6H5Cl + 2Na + ClCH3 ¾® C6 H5CH3 + 2NaCl
Toluene
6. (a) The correct order of increasing bond length is CH3F < CH3Cl < CH3Br < CH3I
EBD_7587
294 Chemistry Objective MCQs

R – I > R – Br > R – Cl > R – F


7. (a) For a given alkyl group, the order of reactivity is ¬¾¾¾¾¾¾¾¾¾¾¾
increasing bond energy decreasing halogen
reactivity.
This order depends on the carbon-halogen bond energy; the carbon-fluorine bond energy is maximum
and thus fluorides are least reactive while carbon-iodine bond energy is minimum hence iodides are
most reactive.
8. (a) Since CHCl3 is covalent compound it does not produce Cl– ion in H2O, hence no white ppt is formed
during reaction with AgNO3.
O O

9. (b) CF3 S O > Ph S O > CH3 COO > PhO

O O
L.G. Ability

AgCN
10. (a) C2H5Br ¾¾® C2H5NC + AgBr
(X)

Reduction
Zn – Hg / HCl

C2H5NHCH3
(Y) ethyl methyl amine

11. (b) The reaction proceeds via free radical mechanism.


As 2º free radical is more stable than 1º, so CH3CH2CH(Br)CH3 would be formed.
+ –
NºNCl F

12. (a) HBF


¾¾¾®
4
+ N2 + BF3 + NaCl

(Balz-Schiemann's reaction)

NaNH
13. (b) CH3 — CH 2 — CHCl2 ¾¾¾¾

D
NaNH 2
CH3 — CH = CHCl ¾¾¾¾ ® CH 3 — C º CH
D
Final Product
14. (c) The acid character follows the order :
CH3COOH > C6H5OH > H2O > CH3OH
The basic character will follow the order
CH3COO– < C6H5O– < OH– < CH3O–
The stronger the acid, the weaker the conjugate base formed.
15. (c) C6H5CHCH3Br being an optically active secondary alkyl bromide undergoes SN2 nucleophilic
substitution reaction. Hence it undergoes complete inversion of configuration.
H H
– DMF C6H5
C6H5 C — Br + Cl ¾¾¾® Cl — C

H3C CH3
Haloalkanes and Haloarenes 295

CH3 CH3CH 2 CH = CH 2
16. (b) C = O + CH3CH2MgBr ¾¾®
CH3 I
(Its four C 's are different)
CH3 OMgBr
CH3 C CH3CH = CHCH3
CH2CH3
II
H 2O (It has 2 types of carbon)
CH3 OH
C CH 3
CH3 CH2CH3 |
2- methyl butanol-2 CH3 - C = CH 2
(3° alcohol) III
(It has 2 types of carbon)
17. (c) At bridge head position SN1 and SN2 do Grand total of acyclic isomers = 6 + 4 + 2 = 12
not takes place.
heat
18. (c) The major component of Borsch reagent is 21. (d) Cl3C - CH 2 CH 3 + KOH ¾¾¾
®
2,4- dinitrophenyl hydrazine which can be
obtained by reaction of 2,4-dinitrochloro (OH)3 C - CH 2CH3 + 3KCl
benzene and hydrazine

O2N Cl + H.NH NH2


O
NO2 ||
CH3CH 2 C - OH
22. (a) Alkyl chloride or bromide undergo
¾® O2N NH NH2 substitution and get converted to an alkyl
iodide on treatment with a solution of
NO2 sodium iodide in acetone. e.g.
acetone
2,4 -dinitrophenyl hydrazine CH 3CH 2 CH 2 Br ∗ NaI ¾¾ ¾¾ ↑
19. (c) Chloral on reaction with chlorobenzene in CH3CH2CH2I + NaBr
the presence of a catalytic amount of This reaction is also known as Finkelstein
sulphuric acid forms DDT (dichlorodiphenyl Reaction.
Trichloro ethane). alc. KOH Br
23. (b) C 2 H 5 I ¾¾ ¾ ¾® CH 2 = CH 2 ¾¾
¾2®

KCN
H H Cl BrCH 2 - CH 2 Br ¾¾¾® CNCH 2 .CH 2 CN
24. (d) CCl4 has lowest (zero) dipole moment. This
Cl3C – C = O +
is due to its symmetrical tetrahedral
Trichloro H Cl structure. Due to which dipole moment of
acetaldehyde
one bond is cancelled by opposite dipole
moment of the other three bonds.
Cl 25. (d) The non-reactivity of chlorine atom in vinyl
¾®Cl3C – CH chloride is due to resonance stabilisation.
, ∗
Cl CH 2 < CH , Cl ¾¾
↑ CH 2 , CH < C l
DDT
20. (b) C4H7Cl is a monochloro derivative of C4H8 26. (c) Inversion in configuration occurs in SN2
which itself exists in three acyclic isomeric reactions.
forms.
EBD_7587
296 Chemistry Objective MCQs
27. (d) –NO2 group withdraws electrons from on either side of the face forming racemic
o- and p-positions and hence activates the mixture.
–Cl present on that position towards 29. (a) In the first case the reaction gives side chain
nucleophilic substitution. substitution product which is easier in A.
28. (b) Chlorination of alkanes is a free-radical In the second case the reaction will proceed
reaction. Since the intermediate free radical by benzyne mechanism for which ortho
is planar (sp2 hybridised) it can be attacked position w. r. to Cl must have H-atoms.
30. (d) Potassium ethoxide is a strong base, hence causes elimination reaction. The alkene formed is governed
by Saytzeff rule "more substituted an alkene, higher will be its stability".
Br
| -
OC H 5
¾® CH 2 = CHCH 2CH 2CH 3 + CH 3CH = CHCH 2 CH 3
CH 3 CHCH 2CH 2CH 3 ¾¾¾2 ¾
Pentene -1 Pentene - 2 ( more stable )
Further pentene-2 shows geometrical isomerism, cis- and trans- in which trans-isomer having bulkier
groups away from each other is more stable than the cis-isomer.
CH3 CH2CH3 CH3 H
C=C C=C
H H H CH2CH3
cis-Pentene-2 trans-Pentene-2 (more stable)

– +
d d D
31. (c) MgBr ¾¾® ; heat favours elimination.

Br
H
Me Me Me Me
H O H OH
I + +
+
32. (a) H O+ +
H Shift H O —H +
¾¾¾
2
¾® ¾¾¾® ¾¾¾®
2
¾¾®
solvolysis
-
–I

33. (c) SN2 and SN1 same, if C+ not rearrange.


NaCN Ni/H
34. (a) CH3CH 2Cl ¾¾¾¾
® CH3CH 2 CN ¾¾¾¾

(X)

CH 3CH 2 CH 2 NH 2 ¾¾ ¾ ¾ ¾ ¾® CH 3CH 2 CH 2 NHCOCH3 ∗ CH 3COOH


( CH 3CO ) 2 O
(Y ) (Z)

35. (d) 4-nitrobenzyl chloride is likely to react by the 37. (a) Rate of
SN2 mechanism as the strongly electron- 1
withdrawing nitro group would destabilize the SN2 µ .
Steric crowding near to reaction centre
carbocation intermediate of the S N1 38. (d)
mechanism, a benzylic chloride that disfavours
+ + +
the SN1 mechanism.
36. (d) Stablity
+ + + O N –I B
–I max
< < H H
Aromatic Aromatic Anti-aromatic
(i) (ii) (iii)
Haloalkanes and Haloarenes 297

Br (i) NaH
39. (b) C CH ¾¾¾¾® H5C2 — C CH ¾¾¾¾¾¾®
(ii) Br
Na/NH3
H5C2 — C C — C2H5 ¾¾¾¾®
(E) – 3 – hexene
40. (c) The electron withdrawing nitro groups weaken the C – F bond by inductive effect and resonance.
CH 2 CH3 CH 2 CH3
Mg HCHO | |
41. (d) CH 3 CH 2 I ¾¾¾® CH3CH 2 MgI ¾¾¾¾ H2 O
® H - C - OMgI ¾¾¾ ® H - C - OH
Ether
(A) (B) | |
H H
(C) (D)
n - Propyl alcohol
42. (a) In 2° halides of this type the product formed has inverted configuration (SN2 mechanism).
O O MgBr OH
H O 2 1
43. (a) C + Mg Br ¾
¾® C ¾¾¾
2 ®
Ph C CH 3

|
|
CH 3 CH 3 4 6
Ph Ph
3 5
2 – phenyl -2-hexanol

Cl OH O
KOH –H2O (i) CH MgBr
CH3 — C — CH3 ¾¾® CH3 — C — CH3 ¾¾¾¾¾®
3
44. (d) H3C — C — CH3 ¾¾®
(aq.) (ii) H2O / H
(II)
Cl OH
(C3H6Cl2) Unstable OH
(I)
Anhy. ZnCl + HCl
CH3 — C — CH3 ¾¾¾¾¾ ¾® Gives turbidity
2
Lucas Test
immediately
CH3
3° alcohol
(III)

45. (b) (a) H2/Ni


*
H Br H Br
H2/Ni
(b) ¾¾®
(Optically inactive)

(c) H2C H2/Ni H3C


CH3 * CH3
CH3 CH3

H 2/Ni
(d) C 2H 5 H 3C * C 2H 5
H 2C
EBD_7587
298 Chemistry Objective MCQs
46. (b) The more is the stability of intermediate (carbonium ion), the more is the chance of SN 1 mechanism.
+ + + +
The intermediates obtained will be Ph C H 2 (i), Ph C H - Me (ii), Ph C - Me 2 (iii), Ph C MePh (iv).
The stabilty is of the order iv > iii > ii > i.

47. (b) C2H5O–


More
acidic
H CH3 H

C6H5 — CH — C — CH — CH3
(–I effect) (+I effect)
Br
C2H5ONa
C2H5OH

CH3

C6H5 — CH = C — CH 2 — CH 3

Slow – +
48. (b) MeOCH 2 Cl ¾¾¾® MeOCH2 + Cl
(i)
b
+
Me – O = CH2
(ii)

Though (ii) contains +ve charge on oxygen. Since octet around each atom does complete the structure
II is more stable than I.
49. (a) Cl in 2, 4, 6-trinitrochlorobenzene is activated by three NO2 groups at o, and p-positions and hence
undergoes hydrolysis most readily.
50. (d) A is 1, 4-addition product due to conjugation B is obtained by further addition of HBr.
(CH 3CHBr.CH 2 .CH 2 Br ) . B gives C by cyclisation CH 3 – CH – CH 2 . If B reacts by SN2 mechanism,
CH 2
Br on 1° carbon must be replaced by OH - to give CH 3CHBr - CH 2 - CH 2 OH.

Br Br Br

Mg / ether D O.
51. (c) ¾¾¾¾¾
® ¾¾2¾® + DOMgBr

CH 2 Br CH 2 MgBr CH 2D
52. (d) Mechanism of Hunsdiecker’s reaction is

+ Br · ·
R– COO - Ag ¾¾¾¾ ·
2
® RCOOBr ¾¾® RCO O + Br ¾
¾® R + CO 2 ;
– AgBr
Haloalkanes and Haloarenes 299

·
R · + RCOO Br ¾
¾® R – Br + RCOO
CH3 CCl3 CCl3 CH3
3Cl Br /Fe
¾¾® ¾Zn/HCl
2
53. (b) heat
¾¾2¾® ¾¾®
Br Br
54. (c) Bond strength follows the order
C – F > C – Cl > C – Br > C – I

Br

2 + 2Na ¾dioxane
¾ ¾¾® Cl Cl + 2NaBr

Cl
55. (c) Nuclear substituted halogen has double bond character due to resonance.
56. (c) The compound A is
aq.KOH -2 H O
Cl 3C.CH 2 .CH 2Cl ¾¾ ¾¾® (OH )3 C.CH 2 CH 2 OH ¾¾¾
2¾®
HO CH 2 .CH 2 COOK.
*
The compounds B is Cl 3C - C H - CH 2 Br has chiral centre and can be resolved into d- and l- form.
|
Br
57. (c) Small difference in boiling points of C2F6 and C2H6 is due to the fact that (i) the F atom is slightly
larger than H, and (ii) F has low polarizability
58. (b) The benzyne is an intermediate of this type of reaction.
59. (b) Structure (b) is more stable because here negative charge is dispersed to the maximum extent as it is
present on the carbon atom bearing an elecron-withdrawing (–NO2) group

fF
CH 3 O fF CH 3 O fF
+ CH3O - N a + ¾¾¾® –
:

60. (d) ¬¾® –


:

NO 2 NO 2 NO 2

CH 3 O
O CH3
fF

:

¬¾® ¾¾¾® + F-

NO 2 NO 2
aq. NaOH Al2O3
61. (b) CH3 CH 2 CH 2 Br ¾¾¾¾¾ ® CH 3 CH 2 CH 2 OH ¾¾¾¾ ®
heat
Cl / H O
CH 3 CH = CH 2 ¾¾2¾ ¾
2¾® CH .CHOH .CH Cl
3 2
EBD_7587
300 Chemistry Objective MCQs
62. (d) Addition product is formed by anti-Markownikoff's rule.
Hunsdiecke r reaction
63. (d) C 6 H 5 COOAg + Br2 ¾¾ ¾¾ ¾¾ ¾¾® C 6 H 5 Br + CO 2 + AgBr

O–
OH - OH - OH
64. (b) PhCOCHBr2 ¾¾¾® PhCOCHO ¾¾¾ ® Ph – C – C H+
¾¾® PhCH(OH)COOH
(A) O (C)
H
(B)

14 Cl 14 14 OH 14
NaOH(aq.) –
¾¾¾¾® OH
65. (c) high temp.
¾® +
(–HCl) OH
Benzyne
66. (d)
C H Li or
67. (d) ® CH3CH 2 P + Ph 3Br - ¾¾¾¾¾
CH3 CH 2 Br + Ph 3P ¾¾ 6 5 ®
C2 H5ONa

- +
CH3CH = PPh 3 ¬¾® CH3 C H P Ph 3
68. (d) The given reaction is an example of electrophilic subsitution. Further, CH3 group in toluene is
o, p-directing
69. (c) Rate of solvolysis µ stability of C+.
70. (d) Unstable C+ has general tendency to rearrange to more stable C +.
Alcohols, Phenols
25
and Ethers
1. Which one of the following statements is not (c) CH3 CH CH2 CH3
correct?
(a) Alcohols are weaker acids than water. OH
(b) Acid strength of alcohols decreases in the and CH3 CH2 CH CH2 OH
following RCH2OH > R2CHOH > R3COH.
CH3
(c) Carbon-oxygen bond length in methanol,
CH3OH is shorter than that of C – O bond (d) CH3 CH CH2 CH3
length in phenol. O
OH
(d) The bond angle in methanol is
C H and (CH3)2 C CH2 CH3
108.9°.
2. What is the structure of the major product when OH
phenol is treated with bromine water ?
OH OH
OH
O
Br Br Br OH OH
Br 4. PCC
¾¾® (A) ¾¾¾®
+ (B)
H
(a) (b)
Br
H Br (1) MeMgBr
(2) H3O+ NaBH , EtOH
¾¾¾¾® (C) ¾¾¾¾® 4
(D)
OH OH Product (D) in above reaction is:
Br
(c) (d)

OH OH OH
Br OH

3. In the given pair of alcohol, in which pair second (a) (b)


alcohol is more reactive than first towards
hydrogen bromide? OH OH OH

CH3 OH H OH
(c) (d)
OH
(a) and
5. Which of the following reagents would carry out
the following transformation? (D = 2H)
OH O OH

(b) and CH CH3 CCH3 ¾®


?
CCH3
OH
D
EBD_7587
302 Chemistry Objective MCQs

(a) NaBD4 in CH3OH (b) LiAlH4, then D2O 13. Which are not cleaved by HIO4?
(c) NaBD4 in CH3OD (d) LiAlD4, then D2O I. glycerol
6. In the Victor-Meyer’s test, the colour given by II. glycol
1°, 2° and 3° alcohols are respectively: III. 1, 3 propenediol
(a) Red, colourless, blue IV. methoxy-2-propanol
(b) Red, blue, colourless (a) I, II, III, IV (b) I, II
(c) Colourless, red, blue (c) II, III (d) III, IV
(d) Red, blue, violet 14. The reaction of phenol with benzoyl chloride to
7. Rate of dehydration of alcohols follows the order: give phenyl benzoate is known as :
(a) 2° > 1° > CH3OH > 3° (a) Claisen reaction
(b) 3° > 2° > 1° > CH3OH (b) Schotten-Baumann reaction
(c) 2° > 3° > 1° > CH3OH (c) Reimer-Tiemann reaction
(d) CH3OH > 1° > 2° > 3° (d) Gatterman-Koch reaction
8. In Williamson synthesis of mixed ether having a OH
primary and a tertiary alkyl group if tertiary halide
TsCl LiAlH
is used, then : 15. ¾¾¾¾ ® (A) ¾¾¾¾
4
® (B)
Pyridine
(a) Rate of reaction will be slow due to slow
cleavage of carbon-halogen bond.
Product (B) of the above reaction is:
(b) Alkene will be the main product.
(c) Simple ether will form instead of mixed ether.
(d) Expected mixed ether will be formed.
9. An ether (A), C5H12O, when heated with excess
of hot concentrated HI produced two alkyl (a) (b)
halides which when treated with NaOH yielded
compounds (B) and (C). Oxidation of (B) and (C)
gave a propanone and an ethanoic acid Cl
respectively. The IUPAC name of the ether (A)
is: (c) (d)

(a) 2-ethoxypropane (b) ethoxypropane


16. Which one of the following compounds will not
(c) methoxybutane (d) 2-methoxybutane
be soluble in sodium bicarbonate?
10. Allyl phenyl ether can be prepared by heating:
(a) 2, 4, 6 - Trinitrophenol
(a) C6H5Br + CH2 = CH – CH2 – ONa
(b) CH2 = CH – CH2 – Br + C6H5ONa (b) Benzoic acid
(c) C6H5 – CH = CH – Br + CH3 – ONa (c) o - Nitrophenol
(d) CH2 = CH – Br + C6H5 – CH2 – ONa (d) Benzene sulphonic acid
11. Amongst the following alcohols which would 17. Which one of the following substituents at para-
react fastest with conc. HCl and ZnCl2 ? position is most effective in stabilizing the
(a) pentanol (b) 2-methyl butanol
(c) 2-pentanol (d) 2-methyl butan-2-ol O

12. Reagent used to convert allyl alcohol to acrolein phenoxide ion?


is
(a) MnO2 (b) H2O2 (a) – CH3 (b) – OCH3
(c) OsO4 (d) KMnO4 (c) – COCH3 (d) – CH2OH
Alcohols, Phenols and Ethers 303
20. Williamson synthesis of ether is an example of:
OH (a) Nucleophilic addition
ÅN
(b) Electrophilic addition
H CrO Cl –, (PCC), CH Cl (c) Electrophilic substitution
18. ¾¾¾¾¾¾
3
¾¾¾®
2 2

(d) Nucleophilic substitution


21. Phenol on heating with CHCl3 and NaOH gives
CH3 C OH salicylaldehyde. The reaction is called :
(a) Reimer - Tiemann reaction
CH3
(b) Claisen reaction
Product of the reaction is: (c) Cannizzaro’s reaction
OH CHO (d) Hell - Volhard - Zelinsky reaction
22. In which of the following reactions hydrogen gas

(a) (b) will not be evolved?


OH
Na
(a) CH3— CH2— OH ¾¾®
CO2H O K
(b) CH3— CH2— OH ¾¾®
Al
(c) CH3 CH OH ¾¾®
(c) (d)
CH3
OH OH
CH MgBr
(d) CH3 CH OH ¾¾¾¾¾
3
®
19. Consider the following alcohols,
CH3
CH2OH
23. Which is the best reagent to convert isopropyl
(I) alcohol to isopropyl bromide?
CH3 CH3
CH 2OH
?
CH3 CH OH ¾® CH 3 CH Br
(II)
O 2N (a) HBr (b) SOBr2
(c) Br2 (c) CH3MgBr
CH2OH
24. How many isomers of C5H11OH will be primary
(III)
alcohols?
CH3O
CH2OH
(a) 5 (b) 4 (c) 2 (d) 3
(IV) 25. On reaction with sodium, 1 mol of a compound X
Br gives 1 mol of H2. Which one of the following
The order of decreasing reactivities of these compounds might be X?
alcohols towards nucleophilic substitution with (a) CH3CH2CH = CH2
HBr is: (b) CH3COOCH2CH3
(a) III > I > IV > II (b) III > I > II > IV (c) CH3CH2CH2OH
(d) CH2(OH)CH2CH2CH2OH
(c) I > III > IV > II (d) I > III > II > IV
EBD_7587
304 Chemistry Objective MCQs
26. Which of the following reactions would convert
OH O
2-butanol into deuterated compound
CH 3 - CH 2 - CH - CH 3 ? (a) CH 3 CH CH 2 C H
|
D O O
OH
(b) CH3 C CH 2 C H
H SO
4 ® ¾¾¾¾¾¾ (i) BD / THF
(a) ¾¾2 ¾¾ 3 ®
heat (ii) H 2O 2 ,NaOH
O O
OH
H SO
4 ® ¾¾2¾ D / Pt (c) CH3 C CH 2 C OH
(b) ¾¾2 ¾¾ ¾®
heat
OH OH O
(c) PBr3 (i)Mg, diethyl ether.
¾¾¾ ® ¾¾¾¾¾¾¾¾
(ii) D O
® (d) CH 3 CH CH 2 C OH
2

OH 30. A compound of the formula C4H10O reacts with


(d) PBr3 NaOD, D 2O sodium and undergoes oxidation to give a
¾¾¾® ¾¾ ¾¾ ¾®
carbonyl compound which does not reduce
27. To prepare 3-ethylpentan-3-ol, the reagents Tollen’s reagent, the original compound is
needed are: (a) Diethyl ether (b) n-Butyl alcohol
(a) CH3CH2MgBr + CH3COCH2CH3 (c) Isobutyl alcohol (d) sec-Butyl alcohol
(b) CH3MgBr + CH3CH2CH2COCH2CH3 31. What amount of bromine will be required to
convert 2g of phenol into 2, 4, 6-tribromophenol?
(c) CH3CH2MgBr + CH3CH2COCH2CH3 (a) 4.0 (b) 6.0 (c) 10.22 (d) 20.44
(d) CH3CH2CH2MgBr + CH3COCH2CH3 32. Identify X in the following sequence:
28. What is X in the following reaction ?
K Cr O warm
C3H8O ¾¾¾¾
2 2 7
¾® C3H 6O ¾¾¾¾¾
® CHI3
H 2SO4 I 2 + NaOH
OH OCH3 (X)
O
| | (a) CH3CH2CH2OH (b) CH3CH(OH)CH3
X
H2C C - CH 3 ¾¾® H 2C - C - CH3
| | (c) CH3OCH2CH3 (d) CH3CH2CHO
CH 3 CH3 33. tert-Butyl ethyl ether can’t be prepared by which
reaction?
(a) CH 3OH, H 2SO 4 +
-
(b) CH3OH,CH3O Na H+
(a) tert - Butanol + ethanol ¾¾
¾®
(c) H 2 O / H 2SO 4 followed by CH 3OH
(b) tert-Butyl bromide + sodium ethoxide ®
(d) CH 3 MgBr / ether followed by H 3 O +
(c) Sodium tert-butoxide + ethyl bromide ®
29. What is the major product of the following H+
(d) Isobutene + ethanol ¾¾
¾®
reaction?
34. Succinic acid
OH D NH Br
¾¾
® (A) ¾¾¾
D
3
® (B) ¾¾¾2
KOH
® (C) ;
CH3 CH CH 2 CH2 OH
Product (C) will be:
CrO CH2 CO2H CH2 CO2H
¾¾¾¾¾ 3
® Product
Pyridine cold (a) (b)
CH2 CH2 NH2 CH2 NH2
Alcohols, Phenols and Ethers 305
– CH2 CO2H (a) I > II > III > IV (b) I > III > II > V
CH2 CO2 K+
(c) (d) (c) IV > III > II > I (d) IV > III > I > II
CH2 NH2 CH2 CH2 Br 39. The end product of the following sequence of
reactions
35. CH3 CH3
C CH2 CH3¬¾¾ C CH CH3 K Cr O , H SO
CH3 CH3 O OH ¾¾2 ¾2¾7¾2¾¾

OH H 2O
(3° alcohol) (y)
CH 3OH ( excess ) (i) 2CH MgBr
CH3 ¾® ¾¾ ¾¾3 ¾¾®
¾¾ ¾ ¾ ¾ ¾
CH CH CH3 H + ( Cat ) (ii) H 3O +
CH3
OH
(2° alcohol)

Find missing reagents. OH OCH3


(a) x = LiAlH4, y = NaBH4 (a) (b)
(b) x = LiAlH4/AlCl3, y = LialH4 OH
(c) x = LiAlH4, y = LiAlH4/AlCl3
(c) (d)
(d) x = H2/Ni, y = H2/Pt 40. An optically active alcohol of formula C9H12O2
36. Diethyl ether reacts, inspite of its usual inert produced the following compound when refluxed
nature, with :
with KMnO4.
(a) Dilute suphuric acid
(b) Dilute sodium hydroxide O O
(c) Boron trifluoride HO C C OH
(d) Metallic sodium
37. Give the best conditions for this transformation: The original compound showed these properties
also:
OCH3 Na
C9H12O2 ¾¾¾® H2 liberated
¾¾® (A) Br 2
H3C H3C no rapid reaction
O OCH3
CrO 3/H +
¾¾¾® C9H8O3
(a) CH3OH, H+(cat), heat cool
(b) H2O, H+ (cat.), heat What is structure of (A)?
(c) Mg, ether, CH3OH OH
(d) SOCl2, CH3OH OH OH
CH3
38. The order of reactivity of the following alcohols
towards conc. HCl is
CH3 (a) (b)
CH3

F OH F
OH CH3
OH
(I) (II)
OH
CH3 CH3
Ph OH
(c) (d) both (a) and (b)
OH

(III) (IV) CH3


EBD_7587
306 Chemistry Objective MCQs
41. The product of the following reaction, 45. Which of the following synthesis gives 3-methyl-
CH 3 1- hexanol?
|
( i) Hg ( OAc) , H 2O
CH 3 - C- CH = CH 2 ¾¾ ¾¾ ¾2¾¾ ¾® is (a) 2 - bromohexane
| (ii ) NaBH4 Mg (i) H C =O
CH3 ¾¾ ¾¾® ¾¾ ¾2 ¾¾®
C 2H 5OC 2H 5 (ii ) H 3O +

CH3 Mg
| (b) 2 - bromopentane ¾¾ ¾¾®
C2 H 5OC 2H 5
(a) CH 3 - C - CH - CH 3
| | O
CH3 OH (i)
¾¾ ¾¾¾
¾+
®
(ii) H 3O
CH3
| (c) 3 - bromopentane ¾¾ ¾¾®
Mg
(b) CH 3 - C - CH 2 - CH 2 OH C2 H 5OC 2H 5
|
CH3 (i ) CH CH = O
¾¾ ¾ ¾
3 ¾¾®
(ii ) H 3O +
OH
| (d) 1 - bromobutane ¾¾ ¾¾®
Mg
(c) CH 3 - C - C H - CH 3 C2 H 5OC 2H 5
| |
CH3 CH3 ( i) CH COCH
¾¾ ¾ ¾
3 ¾¾
¾3®
(ii ) H 3O +
CH3
| 46. What is the product of the following sequence
(d) HOCH 2 - C - CH 2CH 3 of reactions?
|
CH3
(i) BH / THF
(CH 3 ) 2 C = CH. CH 2 CH 3 ¾¾¾¾¾¾
3 ®
42. An aromatic ether is not cleaved by HI even at -
(ii) H 2 O 2 , OH
525 K. The compound is
(a) C6H5OCH3 (b) C6H5OC6H5 PCC (i ) CH MgBr
¾¾¾® ¾¾ ¾ ¾
3 ¾¾®
(c) C6H5OC3H7 (d) Tetrahydrofuran CH 2Cl 2 (ii ) H 3O +

43. Which of the following reactions will not result (a) 2, 4 - dimethyl - 3 - pentanol
in the formation of anisole? (b) 2, 3 - dimethyl - 3 - pentanol
(a) Phenol + dimethyl sulphate in presence of a (c) 2, 3 - dimethyl - 2 - pentanol
base (d) 2, 2 - dimethyl - 3 - pentanol
(b) Sodium phenoxide is treated with methyl 47. Which of the following are intermediates in the
iodide reaction of excess of CH3MgBr with C6H5COOC2H5
(c) Reaction of diazomethane with phenol to make 2-phenyl - 2-propanol?
(d) Reaction of methylmagnesium iodide with OMgBr
phenol |
44. Which of the following is not true in case of A. C 6 H 5 - C - OCH 2 CH 3
reaction with heated copper at 300ºC? |
CH 3
(a) Phenol ¾ ¾® Benzyl alcohol
(b) Primary alcohol ¾ ¾® Aldehyde O
(c) Secondary alcohol ¾ ¾® Ketone ||
(d) Tertiary alcohol ¾ ¾® Olefin B. C 6 H 5 - C - CH 3
Alcohols, Phenols and Ethers 307

OMgBr 53. Identify the major product,


| OH
C. C 6 H 5 - C - CH 3 +
H
| ¾¾® Product
CH 3
O
(a) A and B (b) A, B and C O OH
(c) A and C (d) B and C
48. Which of the following compound can react with
(a) (b)
hydroxylamine ?
O O
OH OH O
OH
(a) (b)
OH OH (c) (d) O
O
O
OH OH PBr3 Mg, ether
54. (A) ¾¾® (C) ¾¾¾® Grignard reagent
OH
(c) (d) Na2Cr2O7
(B)
H2 SO4
HO OH
OH (3, 4-dimethyl
¾¾®
(D)
H3O+
49. Acetic anhydride reacts with diethyl ether in the 3-hexanol)
presence of anhydrous AlCl3 to give When Grignard reagent reacts with (B) product
(a) CH3COOCH3 (b) CH3CH2COOCH3 (D) will obtained. Reactant (A) of the above
(c) CH3COOCH2CH3 (d) CH3CH2OH
50. In the following reaction. reaction is:
Red P + HI
C 2 H 5 OC 2 H 5 + 4H ¾¾ ¾ ¾¾® 2X + H 2 O ;
X is (a) (b)
OH
(a) ethane (b) ethylene OH
OH
(c) butane (d) propane
51. n-propyl alcohol and isopropyl alcohol can be (c) (d)
chemically distinguished by which reagent OH
(a) PCl5 55. Diols (I-IV) which react with CrO3 in aqueous
(b) reduction H2SO4 and yield products that readily under go
(c) oxidation with potassium dichromate dercarboxylation on heating, are:
(d) ozonolysis
52. An organic compound A reacts with methyl
magnesium iodide to form an addition product OH
which on hydrolysis forms the compound B.
Compound B gives blue colour salt in Victor (I) HO (II) HO OH
OH
Meyer’s test. The compounds A and B are
respectively (III) HO (IV) OH
(a) acetaldehyde, tertiary butyl alcohol OH
(b) acetaldehyde, ethyl alcohol (a) I and II (b) II and III
(c) acetaldehyde, isopropyl alcohol (c) II and IV (d) I and IV
(d) acetone, isopropyl alcohol
EBD_7587
308 Chemistry Objective MCQs

56. CH3CH - CH = CH 2 59. H2C CH CH2 CH2 CH CH3


|
CH3 OH
SOCl O /Zn NaBH
(i) B H
2 6 2 4® Y . H SO ¾¾¾¾ 2
® (A) ¾¾¾®
3
(B) ¾¾¾¾
4
® (C)
¾¾¾¾® X ¾¾¾¾ Pyridine (H O) 2 C5 H 9 ClO
(ii) H 2 O 2 /OH - 140° C
Compound (C) is
What is Y ?

OH
(a) CH3 - CH - CH 2 - CH 2 - O - CH 2 - CH 2 - CH - CH3
| | (a) CH CH CH2 CH CH3
3
CH3 CH3
CH3 - CH - CH 2 - CH 2 - O - CH 2 - CH 2 - CH - CH3 Cl
| | (b) HOCH2— CH2— CH2— CH2— CH2—
CH3 CH3
(b) CH 3 - CH - CH = CH 2 CH2— Cl
| Cl
CH 3
(c) CH3 - CH - CH - O - CH - CH - CH3 (c) HO CH2 CH2 CH2 CH CH3
| | | | (d) HO CH2 CH2 CH CH2 CH3
CH3 CH3 CH3 CH3
Cl
CH3 CH3
+
| | 60. (X) ¾¾¾
H3O
¾® Y+ Z (Y and Z both give
(d) CH3 - C - O - C - CH3
(C5H10O)
| |
C 2 H5 C 2 H5 the Iodoform test). The compound X is –
(a) CH3 – CH = CH – O – CH2 – CH3
57. Identify Z in the sequence:
H
HBr / H 2 O 2
CH 3CH 2 CH = CH 2 ¾¾¾ ¾ ¾® |
C2 H5ONa
(b) CH 3 - C - O - CH 2 - CH 3
Y ¾¾¾¾¾® Z |
(a) (CH3)2COHCH2CH3 CH 3
(b) CH3CH2CH(CH3) – O – CH2CH3 CH3 - C - O - CH 2 - CH3
(c)
(c) CH3(CH2)3– O – CH2CH3 ||
(d) CH3(CH2)4– O – CH3 CH 2
(i) Hg(OAc) /H O
(d) Both (a) and (c)
58. 2 2 ® X + Na
CH3 - CH = CH2 ¾¾¾¾¾¾¾¾ 61. What is the major product of the following
(ii) NaBH 4
reaction ?
0° C
¾¾® Y + CH 3Cl ¾¾ ® Z + HI ¾¾¾
® A+B CH3
What are A and B ?
NBS, benzoyl peroxide CH CH ONa
(a) CH3 – CH2 – CH2 – OH & CH3I ¾¾¾¾¾¾¾¾¾
® ¾¾¾
3 ¾2¾¾®
CCl 4 CH 3CH 2OH

(b) CH3 - CH - OH & CH3I


| Br CH 2 Br
CH3
(c) CH3 - CH - I & CH3OH (a)
|
CH3
(d) CH3 – CH2CH2I & CH3OH OCH 2 CH 3
Alcohols, Phenols and Ethers 309

CH 2 OCH 2 CH 3 64. The structure of the product formed in the


reaction given below is:
(b)
+
H
¾¾®
D
Br
CH 2 OCH 2 CH 3
OH OH
(c)
(a) (b)
O
OCH 2 CH 3
CH 3
(c) (d)
Br
(d) O O
65. The most probable product in the reacion given
OCH 2 CH 3 below is
62. Dehydration of alcohols by conc. H2SO4 takes
place according to following steps : H SO
¾¾¾®
2 4
heat
OH

CH3 CH3 (a) (b)


.. H + +
CH3 — C — CH2.OH ¾¾®
. step-1 CH3 — C — CH2OH 2

H H (c) (d)

CH3 66. Which of the following alcohols is dehydrated


–H2O + most readily with conc. H2SO4 ?
¾¾® CH3 — C — CH2
step-2

H
CH3 CH3
– + CH(OH)CH3 CH(OH)CH3
1, 2-H shift –H
¾¾¾¾® CH3— C — CH 3 ¾¾®
step-4
CH2= C — CH3
step-3 +
The slowest and fastest steps in the above (a) (b)
reaction are
(a) step 1 is slowest, while 3 is fastest. OCH3
(b) step 2 is slowest, while 3 is fastest.
(c) step 2 is slowest, while 4 is fastest.
(d) all steps proceed at equal rate. CH(OH)CH3 CH(OH)CH3
63. In order to get 2-hydroxybenzaldehyde from
phenol, which of the following reagents is (c) (d)
required ?
(a) (CH3CO)2O, H2SO4 (b) CHCl3/NaOH
NO2 Cl
(c) CO2, NaOH (d) CCl4/NaOH
EBD_7587
310 Chemistry Objective MCQs
67. Identify the nature of product in the following 70. Correct increasing order of acidity is as follows
reaction
OH

HO
– (a) H2O, C2H2, H2CO3, phenol
+ K2S2O8 ¾¾® Product
H2O (b) C2H2, H2O, H2CO3, phenol
(c) phenol, C2H2, H2CO3, H2O
OH OH (d) C2H2, H2O, phenol, H2CO3
SO3H 71. Which one/ones of the following reactions will
(a) (b) yield 2-propanol?

SO3H I. H+
CH 2 = CH - CH 3 + H 2 O ¾¾
¾®

OH (i) CH MgI
OH II. CH 3 - CHO ¾¾¾ ¾
3 ¾®
(ii ) H 2O
OH
(c) (d)
(i) C H MgI
III. CH 2 O ¾¾ ¾2 ¾
5¾¾®
(ii ) H 2O
OH
68. Which one of the following compounds has the IV. Neutral KMnO
CH 2 = CH - CH 3 ¾¾ ¾ ¾ ¾ ¾
¾4®
most acidic nature?
(a) I and II (b) II and III
CH2OH (c) III and I (d) II and IV
(a)
72. What is Z in the following sequence of reactions?
OH Z ¾¾¾
PCl
5 ® X ¾¾ ¾ ¾Alc. KOH
¾® Y ¾¾ ¾¾ ¾
2¾¾
4®Z (i ) Conc . H SO
(b) (ii ) H 2O; boil

(a) CH3CH2CH2OH (b) CH3CH(OH)CH3


OH
(c) (c) (CH3CH2)2CHOH (d) CH3CH = CH2
73. Which of the following has lowest boiling point?
OH (a) p-Nitrophenol (b) m-Nitrophenol
(c) o-Nitrophenol (d) Phenol
CH
(d) 74. Arrange the following in increasing order of their
acidity?
69. The major product P in the following reaction is (a) o–cresol, (b) salicylic acid, (c) phenol
H+
(a) c < a < b (b) b < c < a
(CH 3 ) 3 COH + C 2 H 5 OH ¾¾
¾® P
(c) a < b < a (d) a < c < b
(a) (CH 3 )3 COC(CH 3 )3 75. Which of the following diols would cleave into
(b) (CH 3 ) 3 COC 2 H 5 two fragments with HIO 4

(c) C 2 H 5OC2 H 5 (a) 1, 3-hexanediol (b) 2, 4-hexanediol


(c) 1, 6-hexanediol (d) 3, 4-hexanediol
(d) (CH 3 ) 2 C = CH 2
Alcohols, Phenols and Ethers 311
76. In the reaction:
CHO
CH3 (a)
| Heated
CHO
CH 3 - CH - CH 2 - O - CH 2 - CH3 + HI ¾¾¾¾ ®
Which of the following compounds will be COOH
(b)
formed? COOH
(a) CH3 - CH - CH3 + CH3CH 2 OH (c) mixture of (a) and (b)
| OH
CH3 (d) HO
79. Ethanol can be prepared more easily by which
(b) CH3 - CH - CH 2 OH + CH 3CH 3 reaction ?
|
CH3
(i) CH 3 CH 2 Br + H 2 O ¾¾
® CH 3 CH 2 OH
CH3
| (ii) CH 3CH 2 Br + Ag 2 O (in boiling water) ¾¾
®
(c) CH3 - CH - CH 2 OH + CH3 - CH 2 - I
CH3 CH3CH 2OH
|
(d) CH3 - CH - CH 2 - I + CH3 CH 2 OH
77. The following change can be carried out with
O (a) by (i) reaction
? (b) by (ii) reaction
OH ¾¾® (c) Both reactions proceed at same rate
(d) by none
OH 80. 3 moles of ethanol react with one mole of
(a) NaBH 4 (b) LiAlH4 phosphorus tribromide to form 3 moles of
bromoethane and one mole of X. Which of the
(c) H 2 / Pt (d) PCC / CH 2 Cl 2
following is X?
O NaBH 4 (a) H3PO4 (b) H3PO2
78. ¾¾®
¾3
[ ] ¾¾¾¾ ® Product is
(c) HPO3 (d) H3PO3

Answer KEYs
1 (c) 9 (a) 17 (c) 25 (d) 33 (b) 41 (a) 49 (c) 57 (c) 65 (c) 73 (c)
2 (a) 10 (a) 18 (b) 26 (c) 34 (c) 42 (b) 50 (a) 58 (d) 66 (b) 74 (d)
3 (d) 11 (d) 19 (a) 27 (c) 35 (c) 43 (d) 51 (c) 59 (c) 67 (d) 75 (d)
4 (b) 12 (a) 20 (d) 28 (a) 36 (c) 44 (a) 52 (c) 60 (c) 68 (b) 76 (c)
5 (a) 13 (d) 21 (a) 29 (b) 37 (a) 45 (b) 53 (a) 61 (b) 69 (b) 77 (b)
6 (b) 14 (b) 22 (d) 30 (d) 38 (c) 46 (b) 54 (a) 62 (c) 70 (d) 78 (d)
7 (b) 15 (c) 23 (b) 31 (c) 39 (a) 47 (b) 55 (c) 63 (b) 71 (a) 79 (b)
8 (b) 16 (c) 24 (b) 32 (b) 40 (d) 48 (c) 56 (a) 64 (c) 72 (b) 80 (d)
EBD_7587
312 Chemistry Objective MCQs

1. (c) The C – O bond length in alcohols is 142 pm and in Phenol it is 136 pm. The C – O bond length in
phenol is shorter than methanol it is due to the conjugation of unshared pair of electrons on oxygen
with the ring, which imparts double bond character to the C – O bond.

142 pm 136 pm
H C O O
H Phenol H
Methanol
H
2. (a) Phenol has activating (electron releasing)–OH group and bromine water supplies Br + ion easily,
hence under such conditions reaction does not stop at monobromo or dibromo stage but a fully
brominated (2,4,6,-tribromophenol) compound is the final product.

OH OH
Br Br
+ 3 Br2 ¾¾®

Br
2, 4, 6-Tribromophenol

CH3 CH3
+
H
3. (d) CH3 C CH2CH3 ¾¾® CH3 C CH2CH3
+

OH 3° carbocation

H+
CH3 CH CH2CH3 ¾¾® CH3 CH CH2CH3
+

OH 2° carbocation
Stability order of carbocation 3° > 2°.
O O
O
CHO CH
O
4. (b) (A) (B)

OH CH3 OH CH3
CHO CH2 OH
(C) (D)
Alcohols, Phenols and Ethers 313

O OH
NaBD
CCH3 ¾¾¾¾ ®
4
5. (a) CH3OH
CCH3

(Nucleophilic addition takes place) D

6. (b) Victor Meyer’s test: The various steps involved are


HI HI AgNO2 HNO2 KOH
(i) 2OH
RCH RCH¾¾
2 OH®¾RCH 2 I ¾¾¾¾
¾® RCH 2 I ¾¾¾ ®
¾RCH
® RCH
2 NO 2 2¾¾¾¾
2 NO ¾¾¾¾® ® R - C - NO2 ¾¾¾® Blood red colour
||
NOH
Nitrolic acid

R R
HI AgNO2
(ii) RCHOHR ¾¾® CHI ¾¾¾¾
® CHNO2
R R
+
R H – R NO
H O NO KOH
C ¾¾¾® C ¾¾® blue colour
H2O
R NO2 R NO2

HI AgNO 2 HNO22 KOH HNO KOH


(iii) R 3 COH ¾¾® R 3 CI ¾¾ ¾¾ ® R 3 CNO 2 ¾¾¾¾
¾¾ ¾¾ ® No ¾¾ ¾¾¾¾
reaction
N.R. ® Colourless
® Colourless
7. (b) The order of dehydration among three type of alcohols is 3° > 2° > 1° > CH3OH. This behaviour is
related to the relative stabilities of carbocations (3° > 2° > 1°).
8. (b) The tertiary alkyl halide undergoes elimination reaction to give alkenes.

CH3 CH3

CH3 —C—X + NaOC2H5 ¾® CH3 —C== CH 2


2-methyl propene
CH3
Conc. HI
9. (a) CH3CH2OCH(CH3)2 ¾¾¾¾® CH3CH2I + (CH3)2CHI
(A) (B) (C)

NaOH NaOH

CH3CH2OH (CH3)2CHOH

[O] [O]

CH3COOH (CH3)2CO
2 1
hence the IUPAC name of compound is CH3 — CH 2 — O — CH — CH 3

CH3
3
2- ethoxy propane
EBD_7587
314 Chemistry Objective MCQs

10. (a) C6H5Br + NaO – CH2 – CH = CH2 ¾¾


® C6H5 – O – CH2 – CH = CH2 + NaBr
Allyl phenyl ether
11. (d) ZnCl2 + conc. HCl is Lucas reagent. Lucas reagent reacts fastest with tertiary alcohol.
3° alcohol + Lucas reagent = Immediate turbidity
12. (a) MnO2 being a mild oxidising agent stops the oxidation of – CH2OH group at aldehyde stage.
13. (d) HlO4 will not oxidise diol from 1, 3 atom and cannot used for cleavage of ether.

O O

OH Cl C O C
14. (b) + ¾¾¾®

Phenol benzoyl Phenyl benzoate


chloride

OH OTs

TsCl LiAlH
15. (c) ¾¾® ¾¾¾®
4

(A) (B)

16. (c) Due to intramolecular hydrogen bonding withdrawing group than – COCH3. Hence
this will not be soluble in sodium – COCH 3 group more stabilize the
bicarbonate. phenoxide ion at p-position.

OH OH CHO
O
PCC
18. (b) ¾¾®
N
OH OH
O
(3° alcohol cannot be oxidized)
17. (c) Electron withdrawing group stabilises the
benzene ring due to delocalisation of 19. (a) More stable carbocation, more is the rate
charge. toward HBr (acid).
– CH3 and – CH2OH are electron donating 20. (d) This method is suitable for the preparation
group and hence decrease the stability of of a wide variety of unsymmetrical ethers.
benzene ring – OCH3 is weaker electron The nucleophilic substitution of halides
with alkoxide leads to desired product.
– –
OH O O OH
CHCl2 CHO CHO CHO
CHCl3
21. (a) ¾¾¾¾¾® ¾® ¾® +
aq. NaOH, 57°C
OH
Phenol Salicyaldehyde p-hydroxy
(chief product) benzaldehyde
Alcohols, Phenols and Ethers 315

+
+ CH MgBr
22. (d) CH3 CH O H ¾¾¾®
3
CH4 (Major product)

CH3
Acid-base reaction takes place.
CH3 CH3
SOBr2
23. (b) CH3 CH OH ¾¾® CH3 CH Br + SO 2 + HBr
Side products are gases.
In case of HBr elimination also favours to produce alkene.
24. (b) (i) CH3CH2CH2CH2CH2OH

(ii) CH 3 CH 2 CHCH 2OH


|
CH3
CH3
|
(iii) CH3CHCH 2CH 2 OH
(iv) (CH3)3CCH2OH
25. (d) Since 1 mole of compound X reacts with Na to evolve 1 mole of H2 gas, therefore the compound
should have 2 active hydrogen atoms per mole which is possible only in option d.
CH 2 O
OHCH O ++ 2Na ®
CH2 CH 2 CH 2 OH ¾¾® NaOCH
NaOCH 2 CH
2 CH
2 CH
2 CH
2 CH
2 CH
2 ONa + H+22H 2
2 ONa

Br D
| |
PBr3 Mg , ether
26. (c) C 2 H 5 .CHOH.CH 3 ¾¾¾ ® C 2 H 5 – CH - CH 3 ¾¾¾¾ ¾® CH 3 - CH - CH 3
D 2O
27. (c) First write down the structure of the required compound, observe the nature of functional group,

C2 H 5
|
CH3CH2 — C— CH 2CH3
|
OH
and review the memory for preparing the required functional group. Here the required compound is a
3º alcohol having three ethyl groups, so only combination given in option c will introduce three ethyl
groups.

CH2CH3
|
CH 3 CH 2 MgBr + CH 3CH 2 COCH 2 CH3 ¾¾
® CH3CH 2 C C H 2 CH3
|
OH

O OH OCH 3
CH3OH | |
28. (a) CH 2 C - CH 3 ¾¾¾¾ ® H 2 C - C - CH 3
H 2SO4
| |
CH 3 CH 3
EBD_7587
316 Chemistry Objective MCQs

29. (b) CrO3 / pyridine is mild oxidising agent.


1° Alcohol ® Aldehyde
2° Alcohol ® Ketone ;

OH O O
CrO
CH3 CH CH2 CH3 OH ¾¾®
3
CH3 C CH2 C H

30. (d) Since the compound (C4H10O) react with sodium, it must be alcohol (option b, c, or d). As it is
oxidised to carbonyl compound which does not reduce Tollen’s reagent, the carbonyl compound
should be a ketone and thus C4H10O should be a secondary alcohol, i.e. sec-butyl alcohol; other two
given alcohols are 1º.

31. (c) OH OH
Br Br
+ 3Br2 ¾¾®

Br
94 g 3×160 = 480 g

Thus 94 g of phenol require Br2 = 480 g


480
2 g of phenol require
require Br22 == ´ 2 = 10.22g
10.22 g
94
32. (b) Since the compound X (C3H6O) gives iodoform test, it must have –COCH3 groups hence it should be
CH3COCH3. Therefore X should be CH3CH(OH)CH3.
33. (b) (CH 3 )3CBr + NaOC2 H5 can’t be applied for synthesising the ether because sod. ethoxide, being a
strong base, will preferentially cause elimination reaction.
-
OC H
(CH 3 )3CBr ¾¾¾¾®
2 5
(CH 3 ) 2 C = CH 2 + HBr
In isobutene + ethanol, isobutene will form tert-butyl cation which reacts with ethanol, a nucleophile
to form ether.
+ + (i) CH CH OH
H
® (CH 3 )2C CH 3 ¾¾¾¾¾¾®
3 2
(CH3 ) 2C = CH 2 ¾¾¾ (CH 3 )3COCH 2 CH 3
+
(ii) - H

O O
–+
COOH D H2—NH Br2 COOK
34. (c) ¾¾®
—H2O O ¾¾¾®
D NH ¾¾®
KOH
COOH NH2
O O
(A) (B)
succinic Succinamide
anhydride
Alcohols, Phenols and Ethers 317

35. (c) CH3 CH3 CH3


– +
H H
C CH CH3 ¾¾® C CH2 CH3 ¾¾® C CH2CH3

O O
.. CH3 OH

..
..
CH3 CH3

Less hindered carbon

CH3 CH3
AlCl3
C CH CH3 ¾¾® C CH CH3
O O+
..
..

CH3 CH3
AlCl3
CH3 CH3 OH

+ H
¾¾® C CH CH3 ¾¾®CH3 CH CH CH3

x = LiAlH4 CH3 O AlCl3
y = LiAlH 4/AlCl3
36. (c)

OCH3 OCH3
37. (a) CH OH
¾¾¾® ¾¾¾®
3
OCH3
O +
H OH +
H (Acetal)

38. (c) The order of reactivity depends upon the stability of the carbocations formed.

CH3 CH3 CH3 CH3 +

+ F + + Ph
F
From (I) From (II) From (III) From (IV). 1º, Most stable
2º carbocation 2º 2º benzyl carbocation

Remember that presence of electron-withdrawing group intensifies i.e., destabilises the carbocation
thus (i) and (ii) are less stable than (iii). Further (i) is less stable than (ii) because –I effect is more
pronounced in (i) due to less distance between F and positive charge. Thus the stability order of the
four carbocations and reactivity of their parent alcohols will be
IV > III > II > I
K Cr O ,H SO CH OH
OH ¾¾¾¾¾¾¾ ®
2 2 7 2 4
39. (a) COOH ¾¾ ¾
3 ¾®
COOCH3
H 2O H+

OH
CH3MgBr
¾¾¾¾¾
® C CH 3
Excess / H3O+
CH 3
EBD_7587
318 Chemistry Objective MCQs

OH CO2H
CH3 OH O OH
C O
CrO CrO
40. (d) ¾¾®
3
, ¾¾®
3

OH CO2H CH3 CH3

41. (a) Recall that oxymercuration-reduction of alkenes lead to hydration in Markovnikov’s way. Thus

OH
|
(i) Hg (OAc )2 , H 2O
(CH 3 ) 3 CCH = CH 2 ¾¾ ¾ ¾ ¾ ¾ ¾
¾®(CH 3 ) 3 C C H CH 3
(ii ) NaBH 4

42. (b) Due to greater electronegativity of sp2-hybridized carbon atoms of the benzene ring, diaryl ethers are
not attacked by nucleophiles like I–.
43. (d) Phenol has active (acidic) hydrogen so it reacts with CH3MgI to give CH4, and not anisole
® CH 4 + C6 H5OMgI
C 6 H 5 OH + CH 3 MgI ¾¾
44. (a) Phenol does not react with heated Cu at 300ºC.

Mg
45. (b) CH 3 - CH - CH 2 - CH 2 - CH 3 ¾¾¾¾¾®
( C2 H5 ) 2 O
|
Br

O
CH 3 - CH - CH 2 - CH 2 - CH 3 ¾¾
¾® CH 3 - CH - CH 2 - CH 2 - CH 3
| |
MgBr CH 2 CH 2 OMgBr

H O+
¾¾3 ¾¾
® CH 3 - CH - CH 2 - CH 2 - CH 3
|
CH 2 .CH 2 OH
BH /THF
3 [O]
46. (b) (CH 3 ) 2 C = CH.CH 2 CH 3 ¾¾¾¾® (CH 3 ) 2 CH - CH - CH 2 CH 3 ¾¾¾ ®
H2 O2 ,OH - |
OH

O OH
|| |
CH3MgBr
(CH 3 ) 2 CH - C - CH 2 - CH 3 ¾¾¾¾ ¾® (CH 3 ) 2 CH - C - CH 2 - CH 3
H3O- |
CH 3
2, 3 - Dimethyl -3 - pentanol
Alcohols, Phenols and Ethers 319

O OMgBr O
|| | ||
47. (b) C 6 H 5 - C – OC 2 H 5 + CH 3MgBr ¾ ¾® C6 H 5 – C + C2 H5 OMgBr
¾® C 6 H 5 - C - OC 2 H 5 ¾
| |
CH 3 CH 3

(A) (B)

OMgBr OH
| + |
H3O
CH3MgBr
¾¾ ¾ ¾¾® C 6 H 5 - C - CH 3 ¾¾¾® C6 H5 - C - CH3
| |
CH 3 CH3
(C)

O O O
O O
48. (c)
O O O
O
Unstable Stable Unstable

OH O NOH

NH OH
¾¾¾®
2

HO OH O O HON NOH

anhy. AlCl
49. (c) (CH 3CH 2 ) 2 O+ (CH 3 CO) 2 O ¾¾ ¾ ¾¾
3 ® CH COOCH CH
3 2 3
Ether Acid anhydride Ester

50. (a) Ethers are reduced by red P and HI to alkanes through alkyl iodides
red P + HI red P + HI
C 2 H 5 OC 2 H 5 ¾¾ ¾ ¾
¾® 2C 2 H 5 I ¾¾ ¾ ¾
¾® 2C 2 H 6
51. (c) In such reactions, first write down the sequence of reactions and then proceed backward/forward
from a known compound, reagent, or known fact (test).
(i) CH MgI Victor Meyer test
A ¾¾ ¾ ¾
3 ¾® B ¾¾ ¾ ¾ ¾ ¾¾® Blue colour
(ii ) H 2O

Appearance of blue colour in Victor Meyer test indicates that B is 2º alcohol and thus A must be an
aldehyde other than HCHO. Thus among the given options, only option (c) is correct.
52. (c)

O
:OH
OH Å
+
H
53. (a) ¾¾® ¾¾®
O
(stable)
O
(Reso stabilized)
EBD_7587
320 Chemistry Objective MCQs

54. (a) PBr Mg


¾¾®
3
¾¾®
OH Br MgBr
(A) Grignard
reagent
¾¾¾¾

Na Cr O H O+
¾¾¾¾®
2 2 7 + ¾¾® ¾¾®
3
H2SO4
O MgBr O OMgBr OH
(B) (D) (3,4-dimethyl
3-hexanol)

55. (c) (II) HO CrO3


HO ¾¾¾® CO2H CO 2H (Gem) D
aq. H SO
¾¾¾®CH 3 CH2 CO2H
2 4 (– CO2)

O O O
OH
CrO3 D
(IV) ¾¾¾®
aq. H2SO4
CH 3 C C OH ¾¾® CH 3 C CH 3 + CO 2
OH (b-kero acid)

2 6 ® (i) B H
56. (a) CH3 - CH - CH = CH2 ¾¾¾¾¾¾ CH3 - CH - CH 2 - CH 2
(ii) H 2O 2 /OH - | |
|
CH3 CH3 OH

H 2SO4/140°C

CH3 - CH - CH 2 - CH 2 - O - CH 2 - CH 2 - CH - CH3
| |
CH3 (Y) CH3

57. (c) HBr / H 2 O 2 C H ONa


CH 3CH 2 CH = CH 2 ¾¾ ¾ ¾ ¾® CH 3CH 2CH 2CH 2 Br ¾¾2¾
5¾¾® CH 3 ( CH 2 ) 3 OC 2 H 5
In presence of H2O2, HBr adds in anti-Markovnikov’s way (peroxide effect).
58. (d) Going backward from A and B

HI
CH3CH 2 CH 2 I + HOCH3 ¬¾¾ CH3 CH 2 CH 2 OCH3
(A and B) (Z )
CH 3Cl
CH3CH 2 CH 2 ONa
Na
CH 3CH = CH 2 ¾¾
® CH3CH 2 CH 2 OH
(X)
Alcohols, Phenols and Ethers 321

SOCl
59. (c) H 2C = CH - CH 2 - CH 2 - CH - CH 3 ¾¾¾¾ 2
Pyridine
®

OH
Cl

H 2C = CH - CH 2 - CH 2 - CH - CH 3
O3/Zn(H 2O)

O Cl

H - C - CH 2 - CH 2 - CH - CH 3
NaBH 4

Cl

HO - CH 2 - CH 2 - CH 2 - CH - CH3

60. (c) CH3 – C – O – CH2 – CH3 +


H3O

CH2
(X)
CH3 – C – OH + HO – CH2 – CH3
(z)
CH2

CH3 – C – CH3

O
(y)

CH3 CH 2Br CH 2 OCH 2 CH 3

61. (b) NBS, benzoyl peroxide


¾¾¾¾¾¾¾¾¾¾¾® CH3 CH2 ONa
CCl 4 , heat, allylic bromination ¾¾¾¾¾¾¾ ®
CH3CH2OH

Br Br Br

62. (c) Step 2 involves the formation of carbonium ion by the loss of weakly basic H2O molecule. It is slowest
step. Step 4 involves the conversion of an unstable (or intermediate) into a quite stable product,
hence it is fastest step.
EBD_7587
322 Chemistry Objective MCQs

OH OH

63. (b) CHO


D
+ CHCl3 + 3NaOH ¾¾®

:
O H

Å
64. (c) H+ Å Ring
¾¾®
D ¾¾¾® ¾¾®
Expansion
OH OH OH

2
3 1 2
65. (c) H+ 3 1
¾¾¾® +
Ring expansion
¾¾¾¾¾® + 6
(–H2O) leads to relief in
OH 4 5 steric strain
4
1º carbocation
5
3º carbocation
(5-membered ring) (6-membered ring, more stable)

2
3 1
–H +
+ ¾¾®
4 6
or
5
3º carbocation

66. (b) The reaction involves the formation of carbocation as intermediate. Hence more the stability of the
carbocation, more will be the rate of reaction. Let us draw the structure of the corresponding carbocation
and observe the relative stability of the four benzyl carbocations.
The relative stability of the four carbocations is

+ + + +
CHCH3 CHCH3 CHCH3 CHCH3

> > >


OCH3 Cl NO2

67. (d) Oxidation of phenol by K2S2O8 is an example of Elbs persulphate oxidation.


68. (b) Phenol is most acidic because its conjugate base is stabilised due to resonance, while the rest three
compounds are alcohols, hence, their corrosponding conjugate bases do not exhibit resonance.
Alcohols, Phenols and Ethers 323
69. (b) tert - Butyl alcohol is preferentially protonated because its corresponding carbocation is highly
stable than that of C2H5OH

..
+ +
H+ - H2 O CH3CH 2 O
..H
(CH3 )3 COH ¾¾¾
®(CH3 )3 CO H 2 ¾¾¾¾
® (CH3 )3 C ¾¾¾¾¾¾
®
Stable

H -H+
(CH 3 ) 3 C O + CH 2 CH 3 ¾¾¾®(CH 3 ) 3 COCH 2 CH 3
The tert-butyl cation is preferentially attacked by CH3CH2OH rather than bulky (CH3)3COH
70. (d) Such questions can be solved by considering the relative basic character of their conjugated bases
which for H2O, C2H2, H2CO3 and C6H5OH are
- -
OH, HC º C : , HCO 3- , C 6 H 5 O -
More the possibility for the dispersal of the negative charge, weaker will be the base. Thus the
relative basic character of the four bases is

HCO 3- < C 6 H 5O - < - OH < HC º C -


Eqivalent Non-equivalent Oxygen can accommodate
resonating structures negative charge easily

Due to resonance
Thus the acidic character of the four corresponding acids will be
H 2 CO3 > C 6 H 5OH > H 2 O > HC º CH
OH
|
H+
71. (a) I. CH 3CH = CH 2 + H 2 O ¾¾
¾® CH 3 C H CH 3

OH
|
(i) CH 3MgI
II. CH 3 CHO ¾¾ ¾ ¾¾® CH 3 C H CH 3
(ii ) H 2O

III. CH 2 O ¾(¾
i ) C2 H 5 MgI
¾¾ ¾ ¾® C 2 H 5 CH 2 OH
(ii ) H 2O

KMnO
IV. CH 2 CH = CH 2 ¾¾ ¾¾
4 ® CH C H C H
3 2
neutral | |
OH OH
72. (b) Reagents used in the various steps indicate that the compound Z has an alcoholic group. This set of
reactions is possible only when Z is CH3CHOHCH3.

PCl alc.KOH
CH3CH(OH)CH3 ¾¾¾
5
® CH3 CH Cl CH3 ¾¾¾¾® CH3CH = CH 2
(Z) (X) (Y)
(i) Conc. H2SO4 , (ii) H2 O, boil
Markovnikov's hydration

In options a and c, Y cannot be converted back into d by the given series of reactions.
EBD_7587
324 Chemistry Objective MCQs

73. (c) o-Nitrophenol has intramolecular H-bonding.

OH OH OH
CH3 COOH
74. (d)
o-Cresol (a) Salicylic acid (b) Phenol (c)

Electron releasing groups (–CH3, –OCH3, –NCH3 etc) intensify the negative charge of phenoxide ion,
i.e., destablises it hence decreases ionization of parent phenol, therefore decreases acidity. While
electron withdrawing groups (–NO2, –COOH, –CHO etc.) increases acidity.
HIO
75. (d) CH 3 - CH 2 - CH - CH - CH 2 - CH 3 ¾¾¾

2 CH 3 - CH 2 - CHO
| |
OH OH

76. (c) In the cleavage of mixed ethers having two different alkyl groups, the alcohol and alkyl iodide formed
depends on the nature of alkyl groups. When primary or secondary alkyl groups are present, it is the
lower alkyl group that forms alkyl iodide therefore
CH3
D
|
CH 3 - CH - CH 2 - O - CH 2 - CH 3 + HI ¾¾® CH3 - CH - CH 2 OH + CH3CH 2 I
|
CH3

77. (b) LiAlH4 will give the desired compound.

O OH
NaBH4
78. (d) O ¾¾¾¾¾®
(reduction)
O OH
Ozonide

79. (b) Heavy metal ions, particularly Ag+, catalyse SN1 reaction because of the presence of empty orbital.
+ –
slow +
CH3CH2Br + Ag+ [CH3CH2 ... X .. Ag] ¾¾®
(–AgX)
OH
CH3CH 2 ¾¾¾®CH3CH 2OH
from H2O

80. (d) 3C 2 H 5 OH + PBr3 ¾


¾® 3C 2 H 5 Br + H 3 PO 3
[X ]
@unacademyplusdiscounts_link

https://telegram.me/Unacademyplusdiscounts_link

https://telegram.me/Unacademyplusdiscounts_link

https://telegram.me/Unacademyplusdiscounts_link

Join Us Now For all study Materials for free


Aldehydes, Ketones
26 and Carboxylic Acids
OH

PCC dil. NaOH (B) Ozonolysis


1. ¾¾® (A) ¾ ¾D¾® C+D
B
Product (B) is:
From the ozonolysis products, the two isomers
A and B can be distinguished with the help of
O O
(a) Fehling solution (b) Tollen’s reagent
(c) Haloform test (d) only spectroscopy
(a) (b) 4. The correct set of reagents to carry out the
following conversion
O ?
O CH= CH 2 ¾¾
¾®

(c) (d) CH 2CHO

(a) B2 H6 /diglyme, H 2 O 2 / NaOH,


O
EOH PCC/CH 2Cl2
2. (P) ¾¾¾
D
®
(b) Br2 , NaNH2 , HgSO4 /H2SO4
EOH
(Q) ¾¾¾
D
® Ph — CH2— OH + Ph — CO2 (c) H2O /H2SO4 , CrO3 /H2SO4
O
(d) None of the above
(R) ¾¾®
3
P + Q, Structure of (R) is: 5. Which of the following gives ethanal CH 3CHO
on acid hydrolysis?
(a) Ph CH CH CH 3
O CH 3
CH3 (a)
(b) Ph CH C
CH3 H
O
CH3
(b) CH 3
(c) Ph C CH CH3
O H
CH3
(d) Ph C CH2 CH 3
(c)
O CH 3

Ozonolysis
O CH 3
3. X+Y (d)
O OH
A
EBD_7587
326 Chemistry Objective MCQs
6. The following reaction O
OH Anhyd. OH
+ HCN + HCl ¾¾®
ZnCl (b) + CH3 NH CH3
2
CHO
is known as
(a) Perkin reaction O
(b) Gattermann aldehyde synthesis (c) (CH3)3CCH + (CH3)2NH
(c) Kolbe’s reaction
(d) None of these form an enamine.
(d) Gattermann-Koch reaction
11. In the reaction sequence
7. What is X in the following conversion ?
O OH – D
2CH3CHO ¾¾¾® A ¾¾® B ; the product
CH 2 OH ||
X
¾¾® C -H B is :
(a) CH3– CH2 – CH2– CH2– OH
(a) K2Cr2O7, H2SO4/H2O (b) CH3–CH = CH – CHO
(b) HIO4 (c) CH3–CH2 –CH2–CH3
(c) PCC/CH2Cl2 O
ôô
(d) OsO4, (CH3)3C.COOH, (CH3)3COH, OH– (d) CH3 – C – CH3
8. What is the product of the following reaction ?
12. Which one of the following reactions will not
O result in the formation of carbon-carbon bond?
(C H ) P + – CH –
:

¾¾¾ ¾¾¾¾
6 5 3
¾
2
® (a) Reimer-Tieman reaction
DMSO
(b) Friedel Craft’s acylation
(a) 2 - methyl - 1- pentene
(b) 4 - methyl - 1- pentene (c) Wurtz reaction
(c) 2 - methyl - 2- propyloxirane (d) Cannizzaro reaction
(d) 1 - pentene 13. Which is major product formed when acetone is
O heated with iodine and potassium hydroxide?
(a) Iodoacetone (b) Acetic acid
9. HNO Ca(OH) (c) Iodoform (d) Acetophenone
¾¾®
3
(A) ¾¾¾®
2
(B)
O
14. NH OH H+
Product (B) in this reaction is: ¾¾¾¾
2
® (A) ¾¾¾
® (B)

LAH
¾¾¾
® (C) ; product (C) of the reaction is:
O
O
(a) (b) OH

O O
(a) (b)
N N
(c) (d)
H H
10. Which of the following pairs of reactants is most
effective in forming an enamine? (c) (d)
O N N
O OH
(a) CH3CH2CH + [(CH3)2CH]2 NH
H H
Aldehydes, Ketones and Carboxylic Acids 327
15. End product of the following sequence of
O
reactions are:
H +
18. ¾® X , most likely the compound X is
O
1. I + NaOH, D
¾¾¾¾¾¾
2
+ ® R R
2.H
C CH3 3. D
OH OH
O
O (a) (b)
(a) yellow ppt. of CHI3, R
R R R
COOH
O OH OH
(b) yellow ppt. of CHI3, R R R
(c) (d)
CHO
O R
(c) yellow ppt. of CHI3, 19. The reagents employed to carry the following
transformation
COOH
(d) yellow ppt. of CHI3, COOH O O
O OH
H
Br ?
C H CHO ¾¾
®
O O O O
a
16. ¾® H 3C CH 3 H 3C CH 3

CHO (a) LiAlH4 , H 2SO 4 / heat


b
¾® (b) PCC / CH 2Cl 2 followed by HIO 4
(c) NaBH 4 / CH 3OH followed by HIO 4
Identify appropriate reagents for the above
reaction: (d) O 3 followed by (CH 3 ) 2 S
20. What is the product of the following reaction ?
(a) a = Br2/CCl4, b = aq. KOH
+
(b) a = Br2/H , b = aq. KOH CHO
(c) a = Br2/H+, b = alc. KOH |
NaCN, ethanol
(d) a = Br2/HO–, b = aq. KOH ¾¾¾¾¾¾®
17. The starting compounds needed to make the dil. HCl
following compound ? |
O Br
||
KOH , CH OH
¾¾¾ ¾ ¾
3 ¾® — CH = C - C - CH2 CH3 OH OH
| H CN H Cl
CH3
(a) benzaldehyde and 3-pentanone (a) (b)
(b) acetophenone and 2-butanone
(c) acetophenone and butanal Br Br
(d) benzaldehyde and 2-pentanone
EBD_7587
328 Chemistry Objective MCQs
23. Identify Z in the sequence
CN OH D P O H O / H+
CH3 COONH 4 ¾¾
® X ¾¾¾
2 5
® Y¾¾¾¾®
2 Z
H H H Cl D
(a) CH3CH2CONH2 (b) CH3CN
(c) (d) (c) CH3COOH (d) (CH3CO)2O
O
Br Br (i) SOCl
24. Ph CH CH C O H ¾¾¾¾ 2
®
(ii) NH
21. Arrange the following in order of decreasing 2

acidity
(A); Product (A) of the reaction is:

(A) (B) O O O
(a) Ph CH CH C CH2 NH
CH 2
O
(b) Ph CH CH C NH
(C)
O
(a) C > A > B (b) C > B > A
(c) Ph CH CH C H
(c) A > C > B (d) A > B > C
22. What is the product of the following reaction ? (d) Ph CH CH NH

NH
(1) EtONa , EtOH 25. (X)C4H7OCl ¾¾¾
3
® C4H9OH
H ¾ ¾ ¾ ¾ ¾ ¾¾®
( 2 ) CH 3 I Br
¾¾¾2
KOH
® CH3CH2CH2NH2; compound (X) is:

O O O O
CH3
(a) (b) C Cl
(a) H Cl CH3
CH3
(c) OH (d) Cl CHO
OH Cl
H 3C
O O
H
(b) PhMgBr
3 2 CH MgBr H + /H O
26. ¾¾¾¾® ¾¾¾¾¾
® ¾¾¾¾®
(1 eq.) (1 eq.) (1 eq.)

O
Product.
H3C
H The product formed in the reaction is –
(c)

OH OH
OH
CH 3
(d) H (a) Me Ph

O
Aldehydes, Ketones and Carboxylic Acids 329

O O CN
|| ||
(b) Me - C - C - Ph C – CH3
OH (b)
OH
|
(c) Me - C - COOH OH
|
Ph CH2 – C – CH3
(c)
O O CN
|| ||
(d) Ph - C - C - O - Me OH
O
C – COOH
OH
(d)
27. CH3
OH ¾¾¾¾¾¾® X + H2O
H2SO4
OH 29. Calculate number of molecules of Grignard
Product (X) of the reaction is– reagent consumed by 1 molecule of following
compound.
O
O H
OH
(a) (b) O O
O O O
OH O
HO NO2
O O (a) 5 (b) 2 (c) 3 (d) 1
30. Which of the following will not give butyl acetate
(c) O (d)
when treated with 1- butanol
OH HO
28. In a set of reactions, acetic acid yielded a product
D. O
||
SOCl Benzene
(a) CH 3 - C - Cl (pyridene )
2 ® (A) ¾¾¾¾
CH3COOH ¾¾¾¾ ® (B)
AlCl3 O
||
HCN HOH
(b) CH 3 - C - OH(H 2SO 4 )
¾¾¾® (C) ¾¾¾® (D)
O O
The structure of (D) would be – || ||
(c) CH 3 - C - O - C - CH 3 (pyridene )
O
||
COOH (d) CH 3 - C - H(H 2SO 4 )
31. Tischenko reaction is a modification of:
CH2 – C – CH3 (a) Aldol condensation
(a) (b) Claisen condensation
OH (c) Cannizzaro reaction
(d) Pinacol-pinacolon reaction
EBD_7587
330 Chemistry Objective MCQs
32. Formaldehyde can be distinguished from 37. What is the product of the following reaction.
acetaldehyde by the use of :
(a) Schiff's reagent (b) Tollen's reagent H KOH , CH OH
¾¾ ¾ ¾ ¾
3 ¾®
?
(c) I2/Alkali (d) Fehling’s solution H heat
33. Clemmensen reduction of a ketone is carried out
in the presence of :
(a) LiAlH4
(b) Zn-Hg with HCl CHO
(c) Glycol with KOH
(d) H2 with Pt as catalyst (a) (b)
34. Which of the following is the product of aldol
condensation? CHO
O OH O
(a) OH (b)
(c) (d)
38. Compound ‘A’ (molecular formula C3H8O) is
O treated with acidified potassium dichromate to
O
HO form a product ‘B’ (molecular formula C3H6O).
(c) (d) ‘B’ forms a shining silver mirror on warming with
ammonical silver nitrate. ‘B’ when treated with
OH an aqueous solution of H2NCONHNH2.HCl and
35. Cannizaro’s reaction is not given by : sodium acetate gives a product ‘C’. Identify the
CHO structure of ‘C’
(a) CHO (b)
(a) CH 3CH 2 CH < NNHCONH2
CH 3
(c) CH3CHO (d) HCHO
(b) CH 3 , C < NNHCONH2
|
CHO CH3

36.
(i) KOH/D
[X] . What is [X]? (c) CH3 , C < NCONHNH2
(ii) H2O/H + |
CH3
CHO
(d) CH 3CH 2CH < NCONHNH2
CH2 – OH
39. The most reactive compound towards formation
(a) of cyanohydrin on treatment with KCN followed
by acidification is
COOH (a) benzaldehyde
(b) p-nitrobenzaldehyde
CHO (c) phenylacetaldehyde
(d) p-hydroxybenzaldehyde
(b) 40. An organic compound ‘A’ has the molecular
formula C3H6O. It undergoes iodoform test.
COOH When staturated with HCl it gives ‘B’ of molecular
formula C9H14O. ‘A’ and ‘B’ respectively are
CH2 – OH

(c)
(a) Propanal and mesitylene
CHO (b) Propanone and mesityl oxide
CH2 – OH (c) Propanone and 2,6-dimethyl-2, 5-heptadien-
4-one
(d) (d) Propanone and mesitylene oxide

COO K+
Aldehydes, Ketones and Carboxylic Acids 331
41. What is D in the following sequence of reactions? O O
NaBH 4 HBr
O¾¾ ¾¾® A¾¾¾® B¾(¾
i) Mg, Et 2O
¾ ¾¾ ¾® ( i ) EtONa
CH 3OH ( ii ) H 2C =O + ? ¾¾ ¾¾ ¾® COOEt

|
(ii) H O+
( iii ) H3O + 3

PCC (a) Diethyl carbonate (EtO)2 C = O


C ¾¾¾® D
CH 2 Cl 2
(b) Ethyl acetate CH 3COOEt
(a) CHO (b) COOH (c) Ethyl formate HCOOEt
(d) Diethyl oxalate Et. OOC. COOEt
OH CH 3 44. The final product of the following sequence of
(c) (d) reactions is
CHO OH
CH 3
CHO OHC
NBS, CCl4 , heat Mg / ether
42.
(i) NaOH/100ºC
? ¾¾¾¾¾¾¾ ® ¾¾¾ ¾ ¾®
CO 2
(ii) H+/H2O benzoyl peroxide

CHO OHC C(CH 3 )3


H 3O +
The major product in the above reaction is ¾¾¾
¾® ?

COOH HOOC CH 3

(a) (a)
COOH HOOC
H 3C C CH 3
OH HOOC
CH 2 COOH
(b)
CH 2 COOH
COOH HO

O (b)

O
C(CH 3 ) 3
(c)

CH 3
O
(c) HOOC
O

CH2OH HOH2C C(CH 3 ) 3

(d)
COOH
CH2OH HOH2C
(d)
43. Identify the missing reagent in the reaction given
below
C(CH 3 ) 3
EBD_7587
332 Chemistry Objective MCQs
45. Identify Z in the following reaction sequence. O
-
O
MnO 4 SOCl2 CH 3COONa
CH 3 CHO ¾¾¾¾ ® X ¾¾¾¾ ® Y ¾¾¾¾¾ ¾®Z (a) (b) O
Dil.H 2SO 4 D O
(a) CH3COCH2COONa O
O
(b) (CH3CO)2O
(c) CH3CO – O – COCH2Cl O
(c) O (d)
(d) CH3CO – OCOCHCl2
46. In the following sequence of reactions, what is 49. The correct product of the following sequence
D? of reactions

CH3 O
EtONa
[O]
¾¾¾¾


¾¾®
¾ A 2 COOH
OEt
H3O

NaOH, H O , Χ D
¾¾ ¾¾∗2¾¾↑ ¾¾® ?
SOCl 2 Na N3 heat H3O
A ¾¾ ¾
¾® B ¾¾ ¾® C ¾¾
¾® D
(a) Primary amine (a) 4 - heptanone
(b) An amide (b) 4 - methyl - 3 - hexanone
(c) 2 - ethyl pentanoic acid
(c) Phenyl isocyanate
(d) 2 - propyl butanoic acid
(d) A higher hydrocarbon 50. The correct product of the following reactions
47. Which one of the following compounds
undergoes decarboxylation upon heating ?
O
|| O O
2 EtONa
¾¾¾ ¾↑
(a) || COOH
OH 2 CH3I
EtO COOH
OEt
O
(i) LiAlH
¾¾¾¾¾
4↑
(ii) H2O
O
||
(a) 2, 2-dimethyl propane diol
(b) || OH (b) 2-methyl-1-propanol
(c) 2, 2-dimethyl propanedioic acid
O
O (d) 2-methyl propanoic acid
O
|| || 51. Arrange the following carbonyl compounds in
(c) decreasing order of their reactivity in nucleophilic
COOH
OH
O addition reaction.
O
|| || CHO CHO COCH3 CHO
(d)
COOH
OH
48. Which lactone is formed by heating the following
hydroxy acid ?
HO
| | O NO2 CH3
|| (i) (ii) (iii) (iv)
COOH
OH (a) (ii) > (iii) > (i) > (iv) (b) (ii) > (i) > (iv) > (iii)
(c) (iii) > (ii) > (i) > (iv) (d) (iii) > (i) > (iv) > (ii)
Aldehydes, Ketones and Carboxylic Acids 333

O 56. Oxidation of acetaldehyde with selenium dioxide


produces
(a) ethanoic acid (b) methanoic acid
D Zn(Hg)/HCl
52.
O ¾¾ ® (A) ¾¾¾¾¾ ® (B) (c) glyoxal (d) oxalic acid
H O 57. Which of the following will respond to Cannizzaro’s
In the above reaction, product (B) is: reaction ?
(a) 2,2-Dimethylpropanal
(b) Acetaldehyde
(a) (b) (c) Propionaldehyde
(d) Cinnamaldehyde
58. If 3-hexanone is reacted with NaBH4 followed
(c) (d) by hydrolysis with D2O, the product will be

53. Correct order of reactivity of following compounds


towards Grignard reagent?
(a) CH3CH2CH(OH)CH2CH2CH3
O O
(b) CH3CH2CD(OH)CH2CH2CH3
CH3 C H H C H (c) CH3CH2CH(OD)CH2CH2CH3
(I) (II)
(d) CH3CH2CD(OD)CH2CH2CH3
O 59. Propanal on reaction with dilute NaOH forms
(a) CH3CH2CH2CH2CH2CHO
CH3 C (b) CH3CH2CH(OH)CH2CH2CHO
(III) (c) CH3CH2CH2CH(OH)CH2CHO
(a) I > II > III (b) II > I > III (d) CH3CH2CH(OH)CH(CH3)CHO
(c) II > III > I (d) I > III > II 60. Which is not true about acetophenone?
54. What is the final product (B) of this sequence? (a) It reacts with 2,4-dinitrophenylhydrazine to
form 2, 4-dinitrophenylhydrazone
CH3
(b) It reacts with Tollen’s reagent to form silver
mirror
Br
¾¾¾ 2
® A ¾¾¾¾¾¾
®B
1. KCN (c) It reacts with I2/NaOH to form iodoform
hight +
2. H3O , heat (d) On oxidation with alkaline KMnO4 followed
by hydrolysis it gives benzoic acid
CH3 CO2H 61. The order of decreasing ease of reaction with
ammonia is
CO2H
(a) (b)
(a) anhydrides, esters, ethers
(b) anhydrides, ethers, esters
CH3 CH2CO2H (c) ethers, anhydrides, esters
(d) esters, ethers, anhydrides
62. Br NH (alc.)
CH3CH 2 COOH ¾¾¾®
2 X ¾¾¾¾¾
3 ®Y
(c) (d) Red P
CO2H Y in the above reactions is
(a) lactic acid (b) ethylamine
55. 1-Phenylethanol can be prepared by reaction of
(c) propylamine (d) alanine
benzaldehyde with
63. In the mechanism of Hoffmann reaction, which
(a) methyl bromide intermediate rearranges to alkyl isocyanate ?
(b) ethyl iodide and magnesium
(a) Bromamide (b) Nitrene
(c) methyl bromide and aluminium bromide
(d) methyl iodide and magnesium (c) Nitroso (d) Amide
EBD_7587
334 Chemistry Objective MCQs
64. A colourless liquid, at room temperature, reacts 72. Among the given compounds, th e most
with soda-lime to form sodium salt of a carboxylic susceptible to nucleophilic attack at the carbonyl
acid and ammonia gas. The liquid is group is
(a) propanoic acid (b) formamide (a) MeCOCl (b) MeCHO
(c) propanamide (d) methyl enthanoate (c) MeCOOMe (d) MeCOOCOMe
65. Which of the following on oxidation followed by 73. Ethyl acetate is obtained when methylmagnesium
hydrolysis gives pyruvic acid ? iodide reacts with
(a) Acetaldehyde cyanohydrin (a) ethyl formate (b) ethyl chloroformate
(c) acetyl chloride (d) carbon dioxide
(b) Acetone cyanohydrin
(c) Formaldehyde cyanohydrin 74. An ester is boiled with KOH. The product is
cooled and acidified with concentrated HCl. A
(d) None of these
white crystalline acid separates. The ester is
66. Identify the correct order of boiling points of the
following compounds : (a) methyl acetate (b) ethyl acetate
CH3CH2CH2CH2OH CH3CH2CH2CHO (c) ethyl formate (d) ethyl benzoate
1 2 75. Hydrolsis of an ester gives a carboxylic acid which
CH3CH2CH2COOH on Kolbe’s electrolysis yields ethane. The ester is
3 (a) ethyl methanoate (b) methyl ethanoate
(a) 1 > 2 > 3 (b) 3 > 1 > 2 (c) methyl methanoate(d) methyl propanoate
(c) 1 > 3 > 2 (d) 3 > 2 > 1 76. What is the missing reagent in the synthesis
67. Carboxylic acids are more acidic than phenol and
shown below
alcohol because of
(a) intermolecular hydrogen bonding O O
(b) formation of dimers (i) EtONa
¾¾¾¾¾®
(c) highly acidic hydrogen (ii) Reagant
EtO COOH
OEt
(d) resonance stabilization of their conjugate
base COOH
68. Which of the following compounds will react with |
NaOH, H2O (i) H2O , H +
NaHCO3 solution to give sodium salt and carbon ¾¾¾¾¾® ¾¾¾¾¾®
D (ii) D
dioxide ?
(a) Acetic acid (b) n-Hexanol (a) bromocyclopentane
(c) Phenol (d) Both (b) and (c) (b) 1, 5 - dibromo pentane
(c) 1, 4 - dibromo butane
69. Which of the following acids has the smallest
(d) 1, 1 - dibromo cyclopentane
dissociation constant ?
77. The correct product of the following sequence
(a) CH3CH(F)COOH (b) FCH2CH2COOH
of reactions
(c) BrCH2CH2COOH (d) CH3CH(Br)COOH
70. Which of the following orders of relative strengths
(i) LiAlH
PBr3
of acids is correct ? (CH3 )2CHCOOH ¾¾¾¾¾
(ii) H O

¾¾¾ ®
2
(a) FCH2COOH > ClCH2COOH > BrCH2COOH
KCN H O+ , H+
(b) ClCH2COOH > BrCH2COOH > FCH2COOH ¾¾¾¾ ® ¾¾¾¾¾
2 ®
DMSO D
(c) BrCH2COOH > ClCH2COOH > FCH2COOH (a) (CH 3 ) 2 CH CH Br.COOH
(d) ClCH2CO2H > FCH2COOH > BrCH2COOH
(b) (CH 3 ) 2 CH CH 2 COOH
71. Acetic acid on heating in presence of P2O5 gives
(a) acetic anhydride (b) acetylene (c) (CH 3 ) 2 CH CH 2 CH 2 NH 2
(c) peracid (d) no reaction (d) (CH 3 ) 2 C = CHCOOH
Aldehydes, Ketones and Carboxylic Acids 335
78. Carboxylic acid group does not give the usual CH3CH3
addition and elimination reactions of aldehydes
and ketones because —C–C—
(c)
(a) O–H bond is more polar than C = O group OH OH
(b) carboxylate ion gets ionised — CH – CH —
(c) carboxylate ion gets stabilised by resonance (d)
OH OH
(d) it exists as – COOH and there is no carbonyl
83. In Cannizzaro reaction given below
group
:OH - ..
79. Which one of the following orders of acid strength 2PhCHO ¾¾¾® PhCH 2OH + PhCO 2–
is correct? the slowest step is :
(a) the transfer of proton to the carbonyl group
(a) RCOOH > HOH > ROH > HC º CH (b) the abstraction of proton from the carboxylic
(b) RCOOH > HOH > HC º CH > ROH group
(c) RCOOH > HC º CH > HOH > ROH (c) the deprotonation of Ph CH2OH
(d) RCOOH > ROH > HOH > HC º CH (d) the attack of : OH at the carboxyl group
80. Which of the following is incorrect ?
(a) FeCl3 is used in detection of phenols. X
84. CH3 COOH
(b) Fehling solution is used in detection of Y
glucose. In the above sequence of reaction X and Y are
(c) Tollen’s reagent is used in detection of respectively
unsaturation. (a) H2/Pt ; Br2
(d) NaHSO3 is used in detection of carbonyl (b) KMnO4; H2/Pt
compounds. (c) KMnO4 (aq); HI/P
NaCN H O+ (d) NH2 – NH2/KOH, HI/P
81. (CH3)2CO ¾¾¾® A ¾¾¾®
3
B 85. Consider the following transformations :
(HCl) D
CaCO heat I
In the above sequence of reactions A and B are CH3COOH ¾¾¾¾
3 ¾® B ¾¾®
® A ¾¾ 2
C
NaOH
(a) (CH3)2C(OH)CN, (CH3)2C(OH)COOH The molecular formula of C is
(b) (CH3)2C(OH)CN, (CH3)2C(OH)2 OH
(c) (CH3)2C(OH)CN, (CH3)2CHCOOH |
(a) CH 3 - C - CH 3 (b) ICH2 — COCH3
(d) (CH3)2C(OH)CN, (CH3)2C = O |
I
82. Acetophenone when reacted with a base,
(c) CHI3 (d) CH3I
C2H5ONa, yields a stable compound which has
86. Silver benzoate will react with bromine in CCl4 to
the structure. form :
COOBr
COOAg
— C = CH – C —
(a) (a) (b)
CH3 O
Br Br
— CH – CH2C —
(b) COOH
CH3 O (c) (d)
EBD_7587
336 Chemistry Objective MCQs
87. The compound that undergoes decarboxylation 92. Which of following compound is hemiacetal?
most readily under mild condition is
COOH COOH HO OMe
CH2COOH O O OH
(a) (b) (a) (b)

COOH CH2COOH
COOH O H OH
(c) (d) (c) (d) All of these
O
88. Which of the following reaction can produce
R – CO – Ar ?
AlCl3 O NH – NH
(a) ArCOCl + H - Ar ¾¾¾ ® 93. C16H16 ¾¾®
3
(B) ¾¾¾
2 2
– ¾®
;
C8H8O HO /D
(A)
(b) RCOCl + ArMgX ¾¾
®
reactant (A) in this reaction is:
(c) ArCOCl + RMgX ¾¾
®
AlCl
(d) RCOCl + H - Ar ¾¾ ¾ 3®
Ph CH3
89. The end product (C) in the following sequence of
(a) C C
reactions is Ph CH3

1% HgSO CH MgX [O ]
HC º CH ¾¾ ¾ ¾¾
4 ® A ¾¾ ¾
3 ¾
¾® B ¾¾®
¾ ( C) Ph CH3
20% H 2SO 4 H 2O
(b) C C
(a) acetic acid (b) isopropyl alcohol CH3 Ph
(c) acetone (d) ethanol
90. Predict the product ‘B’ in the sequence of reactions
Ph Ph
30% H 2SO4 NaOH C C
CH º CH ¾¾¾¾¾® A ¾¾¾¾
®B (c)
% HgSO4 CH3 CH3
(a) CH3COONa
(b) CH3COOH (d) both (b) and (c)
(c) CH3CHO
O OH
(d) CH3 - CH - CH 2 - CHO
| HCN
OH 94. R C R R C R
(catalyst)
91. A substance C 4H10O yields on oxidation a CN
compound, C4H8O which gives an oxime and a
positive iodoform test. The original substance Which of following can be used as a catalyst in the
on treatment with conc. H2SO4 gives C4H8. The
structure of the compound is above reaction?
(a) CH3CH2CH2CH2OH O
(b) CH3CHOHCH2CH3
(a) Cl– (b) CH3 C O–
(c) (CH3)3COH
(d) CH3CH2–O–CH2CH3 (c) EtO– (d) HSO 4–
Aldehydes, Ketones and Carboxylic Acids 337
95. Which of the following reagent(s) used for the (a) Cannizaro’s reaction
conversion? (b) Rosenmund reduction
(c) Wolf-Kishner reduction
(d) Clemmenson reduction
O O 98. Tollen’s reagent and Fehling solutions are used
¾¾® to distinguish between
OH
CO2Et (a) acids and alcohols
(b) alkanes and alcohols
(a) glycol/LiAlH4/H3O+
(c) ketones and aldehydes
(b) glycol/NaH/H3O+ (d) n-alkanes and branched alkanes
(c) LiA1H4 99. In the presence of a small amount of phosphorous,
(d) NaBH4 aliphatic carboxylic acids react with chlorine or
96. Among the following the order of reactivity bromine to yield a compound in which a-hydrogen
towards nucleophilic addition is has been replaced by halogen. This reaction is
(a) CH3CHO > CH3COCH3 > HCHO known as :
(a) Wolff - Kishner reaction
(b) HCHO > CH3CHO > CH3COCH3
(b) Rosenmund reaction
(c) CH3CHO > HCHO > CH3COCH3
(c) Etard reaction
(d) CH3COCH3 > CH3CHO > HCHO
(d) Hell - Volhard - Zelinsky reaction
97. The reaction, 100. Phthalic acid reacts with resorcinol in the
Zn ( Hg )/Conc. HCl [H ] presence of concentrated H2SO4 to give:
CH 3CHO ¾¾¾¾¾¾¾¾¾¾
® CH 3 CH3
(a) Phenolphthalein (b) Alizarin
is (c) Coumarin (d) Fluorescein

Answer KEYs
1 (b) 11 (b) 21 (a) 31 (c) 41 (a) 51 (b) 61 (a) 71 (a) 81 (a) 91 (b)
2 (b) 12 (d) 22 (a) 32 (c) 42 (b) 52 (b) 62 (d) 72 (a) 82 (a) 92 (d)
3 (c) 13 (c) 23 (c) 33 (b) 43 (a) 53 (b) 63 (b) 73 (b) 83 (a) 93 (d)
4 (a) 14 (b) 24 (b) 34 (b) 44 (b) 54 (d) 64 (b) 74 (d) 84 (c) 94 (c)
5 (a) 15 (c) 25 (a) 35 (c) 45 (b) 55 (d) 65 (a) 75 (b) 85 (c) 95 (a)
6 (b) 16 (c) 26 (c) 36 (a) 46 (c) 56 (c) 66 (b) 76 (c) 86 (c) 96 (b)
7 (c) 17 (a) 27 (b) 37 (a) 47 (d) 57 (a) 67 (d) 77 (b) 87 (b) 97 (d)
8 (a) 18 (b) 28 (d) 38 (a) 48 (c) 58 (c) 68 (a) 78 (c) 88 (a) 98 (c)
9 (c) 19 (c) 29 (a) 39 (b) 49 (a) 59 (d) 69 (c) 79 (a) 89 (c) 99 (d)
10 (c) 20 (a) 30 (d) 40 (d) 50 (a) 60 (b) 70 (a) 80 (c) 90 (d) 100 (d)
EBD_7587
338 Chemistry Objective MCQs

OH O O
| || ||

PCC HO
¾¾¾®

||
1. (b) ¾¾¾
® D
(aldol)
(A) (B)

O O
KOH
¾¾¾®
D
2. (b) (aldol)
(P)

Ph — CHO ¾¾¾¾¾® PhCH 2OH + PhCOO -


KOH
D
(Q ) ( Cannizzaro) CH3 CH3
Ph — CH C O3
¾¾¾® PhCHO + O
CH3 CH3
(R) (Q) (P)

CHO CH2CHO

O O3
¾¾®
3
+ CH3CHO ¾¾® + CH2O
3. (c)
(Responds haloform test)
A B

B2 H6 /diglyme. PCC
4. (a) CH = CH 2 ¾¾¾¾¾¾ ® CH 2 - CH 2 OH ¾¾¾¾
CH Cl
®
H 2O 2 , NaOH 2 2

CH 2CHO

O CH 3 HO CH3
5. (a) H + /HOH – H 2O
¾¾¾¾¾
® ¾¾¾ ¾® CH 3CHO
O H HO H

6. (b) C6 H6 + HCN + HCl ¾¾¾¾¾®


2
[C6 H5CH = NH ] ¾¾® C6 H5CHO
anhy. ZnCl

7. (c) Pyridinium chloro chromate selectively oxidises a primary alcohol to an aldehyde


PCC
e.g. R CH 2 OH ¾¾¾¾
CH Cl
® RCHO
2 2

H3C H3C
8. (a) O + CH 2 = P (C 6 H 5 )3 ¾¾® CH 2 + O = P (C6 H5 )3
n–H7C 3 n–H7C3
Aldehydes, Ketones and Carboxylic Acids 339

O OH O
|| ||
HNO COOH Ca(OH)
9. (c) ¾¾¾¾¾¾¾
3 ® ¾¾
® ¾¾¾¾2¾ ® + CaCO3
(Oxidising agent) COOH

10. (c) 2° amine must be less sterically hindered. In option (b) and (c) 2° amine is being used but due to more
reactivity of aldehyde compared to cyclohexanone. Option (c) is correct.

O O O
|| |
11. (b) CH3 РC РH + OH CH2 РC РH ¨ = H2C = C РH

O O OH O O
D
CH3 – C – H + CH2 – C – H ¾® CH3 – C – CH2 – C – H ¾® CH3 – CH = CH – C – H
(Aldol condensation) H

12. (d) Canizzaro’s reaction is a disproportionation reaction of aldehyde in which one molecule of aldehyde
is reduced to alcohol whereas other is oxidised to salt of carboxylic acid
C6H5CHO + KOH ¾¾ ® C6H5CH2OH + C6H5COOK
13. (c) CH3COCH3 + 3I2 + 4KOH ¾¾
® CHI3 + CH3COOK + 3HI + 3H2O

O O H
|| N–OH || |
H+ N
NH2OH ¾¾¾¾¾¾® NH ¾¾¾
LAH
®
14. (b) ¾¾¾¾® (Beckmann
rearrangement)
amide amine

O O O
(i) I2 +NaOH
||

¾¾¾¾¾¾ ® (iii) D
||

||

15. (c) (ii) H +


¾¾¾® + CO2
C–CH3 CO2H
||
O

CHO CH–OH CHO CHO


H+ ¾¾¾® Br E2
¾¾¾¾ ®
16. (c) ¾¾¾
® B r2 alc.KOH

O O
|| || KOH , CH 3OH
17. (a) C6 H5 - C + CH 2 - C - CH 2 - CH 3 ¾¾ ¾ ¾ ¾ ¾®
| | D
H CH 3

OH O O
| || ||
-H2O
C 6 H 5 - CH - CH - C - CH 2 CH 3 ¾¾¾¾ ® C6 H5 - CH = C - C - CH 2CH3
| |
CH 3 CH3
EBD_7587
340 Chemistry Objective MCQs

18. (b) Reaction of 4,4-dialkylcyclohexadienone with an acid to form phenol with the migration of one of the
alkyl groups to the adjacent carbon is known as dienone-phenol rearrangement.

O OH CHO
OH OH

19. (c) NaBH HIO


¾¾¾¾

¾¾¾

O
O O CH3OH O O O

20. (a) Only –CHO group reacts with CN– ion and the reaction is

OH
NaCN, ethanol
C O ¾¾¾¾¾¾® C
dil. HCl
H H CN

21. (a) (C) contains the most reactive methylene group followed by (A) then (B).

O O O O O O

22. (a) H C H ONa, EtOH H ¾CH


¾¾ 3I ® H
¾¾2 ¾
5 ¾¾ ¾¾®
Na – NaI CH 3

heat PO H O+
23. (c) CH3COONH 4 ¾¾¾¾ ® CH3CONH 2 ¾¾¾
2 5 ® CH CN ¾¾¾¾
3
3 ® CH3COOH
( - H 2O)
X
( -H2O) Y
( - NH3 )
Z

O O
|| SOCl 2 ||
PH – CH = CH – C – OH ¾¾¾® Ph–CH=CH–C–Cl
:

24. (b) NH2

O
||
Ph–CH=CH–C–NH

O O
·· Br2 + KOH
25. (a) X = CH3 CH 2 CH 2 C Cl + NH 3 ¾¾®CH3 CH 2 CH 2 C NH 2

CH3 CH2 CH2 NH2

26. (c)
O O OMgBr OH
O O
PhMgBr +
¾¾¾® Ph – C – C – OMgBr ¾¾¾®
MeMgBr H /H2O
Ph – C – C – OMgBr ¾¾¾® Ph – C – COOH

O Me O Me
Aldehydes, Ketones and Carboxylic Acids 341

27. (b)
OH
H

OH
X

O
SOCl2 ÷÷ Benzene
28. (d) ® CH3 - C - Cl ¾¾¾¾®
CH 3 COOH ¾¾¾¾
AlCl 3
A

O OH OH
||
C – CH3 C – CH3 C – CH3
HOH
HCN
¾¾¾® ¾¾¾®
COOH
CN
D
B C

31. (c) Tishchenko reaction is a modification of


H Canizzaro’s reaction. This reaction involves
disproportionation of an aldehyde lacking
[1] C a hydrogen atom in the alpha position in
|||
C the presence of an alkoxide. The reaction
product is an ester. Catalysts are aluminium
O alkoxide or sodium alkoxide.
29. (a) O O In Cannizzaro’s reaction the base is sodium
O hydroxide and the oxidation product is a
[3] carboxylic acid and the reduction product
[1] HO NO 2
is an alcohol.
30. (d) Acid chlorides, anhydrides, acids give ester O C6H5CH2ONa
with alcohol 2 ¾¾¾¾¾®
50 – 60°, 4 hr
ROH + R’COCl ¾
¾® ROCOR’ + HCl H

ROH + (R 'CO)2O ¾
¾® O
ROCOR’ + RCOOH

ROH + R’COOH ¾
¾® ROCOR’ + H 2 O O
90%
EBD_7587
342 Chemistry Objective MCQs

32. (c) Only acetaldehyde and methyl ketones give iodoform test.

Zn-Hg/HCl
33. (b) C O ¾¾¾¾® CH2

Ketone

CH3 CH3 O

34. (b) CH3 —C== O+ CH3COCH3 ¾® CH3 —C— CH 2 —C— CH3

OH
4 - Hydroxy - 4 - methyl - 2 - pentanone

35. (c) Only those aldehydes which do not have a-H atom undergo Cannizaro’s reaction. Hence CH3CHO
will not undergo Cannizaro’s reaction as it has 3 aH atoms.
36. (a) It is an example of intramolecular cannizzaro reaction.
37. (a) It is an example of intramolecular aldol condensation

CHO CHO
H H ¾¾¾¾¾¾
KOH, CH3OH -H O
® OH ¾¾ ¾
2¾®
heat

+
38. (a) C 3 H 8 O ¾K Cr O / H H NCONHNH 2
¾2 ¾2¾7 ¾¾® C 3 H 6 O ¾¾2 ¾ ¾ ¾ ¾¾ ®C
A B (–CHO)
Since B reduces Tollen’s reagent, it indicates that it has an –CHO group, so it must be CH3CH2CHO.
Hence
H NNHCONH
CH 3CH 2CH 2OH ® CH 3CH 2CHO ¾¾2 ¾ ¾ ¾ ¾
¾2®
CH 3 CH 2 CH = NNHCONH 2
[ A] [ B] [C]

39. (b) –NO2 group is electron withdrawing which enchances the addition of CN– ion (a nucleophilic
additon reaction).

O NOH
|| ||
CaCO3 heat NH 2OH
40. (d) CH 3COOH ¾¾ ¾¾® (CH 3COO) 2 Ca ¾¾¾® CH3C CH3 ¾¾¾¾® CH3 C CH3
[A ] [B] [C] Acetoxime

NaBH HBr
41. (a) O ¾¾¾¾
4® OH ¾¾
¾® Br
CH3OH

(i) Mg. Et O PCC


¾¾¾¾¾®
2
CH 2OH ¾¾¾¾ ® CHO
(ii) H2C = O CH Cl 2 2
Aldehydes, Ketones and Carboxylic Acids 343
42. (b) This is intramolecular Cannizzaro’s reaction.

O
||||
43. (a) Et O
¾¾¾® - C - OEt

CH 3 CH2Br CH2COOH

NBS, CCl , heat (i) Mg/ether


44. (b) ¾¾¾¾¾¾¾
4 ® ¾¾¾¾¾®
+
benzoyl peroxide (ii) CO 2, H3O

C(CH 3 ) 3 C(CH3)3 C(CH3)3

Allylic subsitution

- + CH COONa
¾® CH 3 COCl ¾¾ ¾
MnO / H SOCl 2 3 ¾ ¾® CH COOCOCH
45. (b) CH 3CHO ¾¾ ¾4 ¾¾® CH 3COOH ¾¾ ¾ heat
3 3
X Y Z

[O] SOCl2 NaN


46. (c) C 6 H 5CH 3 ¾¾®
¾ C 6 H 5COOH ¾¾ ¾¾® C 6 H 5 COCl ¾¾ ¾
3 ® C H CON
6 5 3
[ A] [B] [C]

é O ù
ê || .. ú
heat
¾® êC6 H 5 - C - N :ú ¾
¾¾ ¾® C6 H 5 NCO
ê ú Phenyl isocyanate , [ D]
ë û
Unstable Benzoyl nitrene
47. (d) b - Keto acids are decarboxylated by following mechanism
H7C3 H7C3 H7C3
C=O D
O ¾¾® ¬¾®
O OH O
H
O
OH O
48. (c)
| | || -H O
2 1 O
5 3 1 ¾¾¾
2¾®
3 5
4 2 OH
4

O O O
EtONa NaOH
49. (a) 2 OEt ¾¾¾¾
HO
+ ® OEt ¾¾¾¾
+
®
H
3

O O
O
D 6 2
OH ¾¾¾¾¾¾® 7 4 1
decarboxylation 5 3
4 – Heptanone
EBD_7587
344 Chemistry Objective MCQs

O O 3 1
O O HOH2C CH2OH
LiAlH 4
50. (a) 2 EtONa ¾¾¾¾® 2
¾¾¾¾® EtO COOH
OEt H 2O
EtO COOH
OEt 2 CH3I CH3 CH3 CH3 CH3
51. (b) Aldehyde will be more reactive than ketone.
D
COCH3 Clemmensen
52. (b) b-keto acid ¾¾ ® ¾¾¾¾¾®
Reduction

53. (b) –CH3 group give + M and + I but + M work only at ortho and para position.
Reactivity µ positive charge on carbonyl carbon so that's why reactivity II > I > III.
KCN H O+
® Ph – CH2 – C º N ¾¾¾® Ph – CH2 – CO2H (B)
3
54. (d) (A) Ph – CH2 – Br ¾¾¾
C6 H 5
|
55. (d) 1-Phenylethanol ( CH 3 C HOH ) is a 2º alcohol and can be prepared by the reaction of benzaldehyde
with Grignard reagent CH3MgI.
56. (c) Selenium dioxide oxidises aldehydes and ketones at a-position to carbonyl group.
SeO
CH 3 CHO ¾¾¾
2 ® OHC.CHO

CH3
|
57. (a) (a) H3C - Ca - CHO (b) H 3C a - CHO
|
CH3
2,2 -dimethylpropanal acetaldehyde (a-H)
(no a-H)

a a
(c) CH3CH2 CHO (d) C6H5CH = CHCHO
propiona ldehyde cinnamaldehyde
(a-H) (a-H)
58. (c) Remember that during reduction with NaBH4, a hydride ion is transferred by NaBH4 to carbonyl
carbon and a proton (or deutron) is transferred from the solvent (in the second step) to carbonyl
oxygen.

O ,
O OD
|| | |
NaBH 4
C2 H5 , C, C3 H 7 ¾¾ ¾¾
↑ C2 H5 , C H , C3 H 7 ¾¾¾¾
D 2O

® C 2 H5 - CH - C3 H 7
(–OD )
3,Hexanone
CH3 O b
CH3 OH CH3
| || – | | |
59. dil. OH
(d) CH CH CHO + CH CH ¾¾¾®
3 2 2 CH – CH CHCHO 2
a
Propanal (2 molecules)
Note that it is the a-carbon (and not b–) that is adding on the carbonyl oxygen of the other propanal
molecule.
60. (b) Acetophenone (C6H5COCH3) being a ketone, does not reduce Tollen’s reagent.
61. (a) Weaker the base, better the leaving group and hence more is the reactivity of the compound. The
relative basic character of the four groups is
NH 2- > OR - > OCOCH 3- > Cl -
Aldehydes, Ketones and Carboxylic Acids 345
Thus the relative leaving ability of the four groups is
NH -2 < OR - < OCOCH3- < Cl -
Hence the relative reactivity of the four acyl derivatives toward nucleophile is

O O O O
|| || || ||
R - C - NH 2 < R - C - OR < R - C - OCOCH 3 < R - C - Cl
Amide Ester Acid anhydride Acid chloride

red P, Br 3® NH CH 3CHNH 2 COOH


62. (d) 2 ® CH CHBr COOH ¾¾¾
CH 3CH 2COOH ¾¾¾¾¾ 3
α -Aminopropionic acid (alanine)

O é O ù
|| ê || . . ú Rearrangement
¾®êR - C - N :ú ¾¾¾¾¾¾® RAlkyl -N=C=O
Br2 / KOH
63. (b) R - C - NH 2 ¾¾ ¾¾ isocyanate
ê Acylnitrene ú
ë û
64. (b) Only amides (but not acids and esters) undergo hydrolysis in presence of soda-lime to form sodium
salt of a carboxylic acid and ammonia gas. Further, since the given compound is a liquid, it must be
formamide, because propanamide is a solid.

OH O O
| || ||
[O] H + , H 2O
65. (a) CH3 C HCN ¾¾¾
® CH3 - C – CN ¾¾¾¾¾ ® CH 3 C– COOH
Acetaldehyde cyanohydrin

66. (b) Due to –I effect of the –COOH group, H-bonds in acids are much stronger than in alcohols; while
aldehydes do not exhibit H-bonding.
67. (d) In carboxylates (conjugate base of carboxylic acids), resonance is more significant because the two
resonating structures are similar, while in phenoxide, the resonating structures are not equivalent,
alkoxide ions do not show resonance.
68. (a) Only acetic acid is more acidic than carbonic acid (conjugate acid of NaHCO3) hence it dissolves in
NaHCO3,

® CH 3COO - Na + + H 2 CO 3 (H 2 O + CO 2 )
CH3COOH + NaHCO3 ¾¾
stronger acid weaker acid

while phenol and n-hexanol are less acidic than carbonic acid and hence do not dissolve in NaHCO3.
69. (c) Bromine is less electronegative than F, further in BrCH2CH2COOH, Br is more away from the –COOH
group than in CH3CHBrCOOH.

O O O
|| || ||
70. (a) F - CH 2 – C - OH > Cl - CH 2 – C - OH > Br – CH 2 – C- OH
(Electron-withdrawing character
decreasing from L to R)
71. (a) P2O5 is a dehydrating agent and removes a molecule of water from two molecules of acetic acid
PO
CH 3 COOH + HOOCCH 3 ¾¾¾
2 5 ® CH COOCOCH
3 3
EBD_7587
346 Chemistry Objective MCQs
72. (a) More the magnitude of positive charge on the carbonyl carbon, higher will be its reactivity toward
nucleophilic attack.
O O O O
|| || || ||
CH3 - C - Cl CH3 - C - H CH3 - C - OCOCH3 CH3 - C - OCH3
Chlorine being more electron-withdrawing than O develops more positive charge on carbonyl carbon.
The order of reactivity of the various carboxyl derivatives toward nucleophile is
RCOCl > RCHO > (RCO) 2 O > RCOOR
Most reactive Least reactive

O OMgBr O
|| | ||
CH 3MgBr ¾
¾® C - OC 2 H 5
73. (b) Cl - C- OC2 H5 Cl - C - OC2 H5
| |
Ethylchloroformate
CH3 CH3
Ethyl acetate

74. (d) Ethyl benzoate on hydrolysis gives benzoic acid which is a solid, other esters give CH3COOH and
HCOOH, both of which are liquids.
75. (b) Ethane is obtained by the electrolysis of sodium or potassium salt of ethanoic acid (CH3COOH),
hence the ester must be ethanoate CH3COOR.

O O O O

76. (c) EtONa Br Br


EtO OEt ¾¾¾
COOH ¾® EtO COOH
OEt
Na Na

O O COOH
(i) NaOH / D
EtO COOH
OEt + +
(ii) H2O / H
¾¾¾¾¾¾ ®
(iii) D

KCN +
LiAlH 4 PBr3 H
77. (b) – COOH ¾¾ ¾¾ ® – CH 2OH ¾¾¾ ® -CH 2 Br ¾¾¾® -CH 2CN ¾¾
¾® -CH 2COOH
H 2O DMSO H 2O

78. (c) Due to resonance in carboxylate ion, the double bond character of C = O bond in carboxylic acids is
greatly reduced as compared to that in aldehydes and ketones.

O O O
C ¬¾® C C
O O O

79. (a) R COOH > ROH > HC º CH. Water is more acidic then alcohol.
80. (c) Unsaturation is not detected by Tollen’s reagent, a reagent for differentiating between aldehydes
and ketones.
OH OH
81. (a) NaCN / HCl H O+
(CH 3 ) 2 CO ¾¾ ¾ ¾¾® (CH 3 ) 2 C ¾¾¾
3 ¾
®(CH 3) 2 C
CN COOH
[A] [B]
Aldehydes, Ketones and Carboxylic Acids 347

O O
C2H5O– –
82. (a) – C – CH3 ¾¾¾® – C – CH2
– C2H5OH


O O O O O OH
– H 2O
– C – CH2 + C – ¾® – C – CH2 – C – ¾¾®
– – C – CH2 – C –
–OH
CH3 CH3 CH3

D –H2O
O
– C – CH = C –

CH3
O – O O O–
O || |
fast Ph – C – H
83. ˆˆˆ†
(a) Ph – C– H + OH–‡ˆˆˆ Ph – C – H ¾¾¾¾® Ph – C + Ph – C –H
slow | |
OH OH H
O OH
+ || |
H exchange
¾¾¾¾¾¾ ® Ph – C + Ph – C –H
fast | |
O– H
84. (c) KMnO4 converts – CH3 group of toluene into – COOH while HI reduces – COOH group into – CH3
group.
Heat H 3C I2 + NaOH
85. (c) CH3COOH + CaCO3 ® (CH3COO)2Ca ¾¾¾® H C > CO ¾¾¾¾¾ ® CHI3
3 (C)
(A) ( B)
86. (c) This is Hunsdiecker’s reaction.

COOAg Br
| |
CCl
+ Br2 ¾¾¾

+ AgBr + CO2

Silver benzoate Bromobenzene

87. (b) b-Ketoacids undergo decarboxylation easily.


88. (a) ArCOR can be prepared by the combination of ArH + RCOCl and not by ArCOCl + RMgX because
here the ArCOR formed will further react with RMgX to form 3º alcohol, ArC(OH)R2 as the final
products.
1%HgSO CH MgX
89. (c) HC º CH ¾¾¾ ¾
¾4 ® CH CHO ¾¾ ¾
3 ¾® CH 3CHOHCH 3 ¾[¾®
3 ¾ O]
¾ CH 3COCH 3
20%H 2SO4 [A ] H 2O [ B] Acetone [C]

30%H SO4 NaOH


90. (d) CH º CH ¾¾¾2¾¾ ® CH 3CHO ¾¾ ¾
¾® CH 3CHOHCH 2 CHO
HgSO4 [ A] [ B]

Conc.H SO Oxidation
91. (b) C 4 H8 ¬¾¾¾¾¾
2 4¾ C H O ¾¾¾¾¾
4 10 ® C4 H8O (R - COCH3 )
( - H 2O)
Thus C4H8O should be CH3CH2COCH3, hence C4H10O should be CH3CH2CHOHCH3
EBD_7587
348 Chemistry Objective MCQs
92. (d) Hemiacetal ® presence of alcohol and ether on same carbon.
O
Ph Ph || NH 2 — NH2 , HO- , D
O3
93. (d) (A) C=C ¾¾® (B) Ph – C – CH3 ¾¾¾¾¾¾¾¾ ® Ph–CH2–CH3
CH3 CH3
(cis & trans)

94. (c) Base stronger than cyanide is used as a catalyst.


95. (a) O O
+
3 H O
OC2H5 ¾¾® OH
O O
CH2OH
½
O
CH2OH
(i) LiAlH
4
¬¾¾¾+¾ OH
OH (ii) H3O
O
96. (b) Presence of alkyl group in carbonyl compounds decreases their reactivity towards nucleophilic
addition. Further greater the number of such groups lesser will be the reactivity towards
nucleophilic addition \ correct order is
HCHO > CH3CHO > CH3COCH3
97. (d) The reaction given is a Clemmenson reduction.
98. (c) All aldehydes and ketones show reaction with Tollen's reagent and Fehling solutions, but
ketones do not show this reaction.
Note: Benzaldehyde do not give reaction with Fehling solution.
99. (d) Hell-Volhard-Zelinsky Reaction :
It is a-substitution when a carboxylic acid having a-hydrogens is treated with chlorine or bromine
in presence of small amount of red phosphorous.
X
|
P
R - CH 2 COOH + X 2 ¾¾
® R C H - COOH + HX(X = Cl, Br)

O
C
O
HO
COOH C
Conc. H 2SO4
100. (d) +2 OH¾¾¾¾¾¾ ®
COOH
+ H2O

HO O OH
Fluorescein
27 Amines
1. The conversion of benzene diazonium chloride
to bromobenzene can be accomplished by NHCOCl
(a) Reimer-Tiemann reaction (c)
(b) Friedel-Crafts reaction
(c) Gattermann reaction
NCO
(d) Azo-Coupling reaction (d)
2. C7H9N has how many isomeric forms that
contain a benzene ring? 7. In a reaction of aniline a coloured product C was
obtained.
(a) 4 (b) 5 (c) 6 (d) 7 CH3
3. The amine that does not react with acetyl N
chloride is NH2
NaNO CH3
(a) CH3NH2 (b) (CH3)2NH ¾¾¾2® B ¾¾¾¾¾¾¾® C
HCl Cold
(c) (CH3)3N (d) None of these
4. In the diazotisation of arylamines with sodium A
nitrite and hydrochloric acid, an excess of The structure of C would be :
hydrochloric acid is used primarily to
(a) – N = N – CH2– N –
(a) supress the concentration of free aniline
available for coupling
(b) supress hydrolysis of phenol CH3
(c) ensure a stoichiometric amount of nitrous CH3 CH3
acid
(d) neutralise the base liberated (b) –N=N–
5. Nitrosoamines (R2N – N = O) are insoluble in
water. On heating with conc. H2SO4, they give
secondary amines. The reaction is called CH3
(a) Liebermann nitroso reaction – NH – NH – –N
(b) Etard reaction (c) CH3
(c) Fries reaction
(d) Perkin reaction CH3
6. Aniline reacts with phosgene and KOH to form –N=N– –N
(d)
OH CH3
8. Reaction of cyclohexanone with dimethylamine
(a) in the presence of catalytic amount of an acid
forms a compound if water during the reaction is
continuously removed. The compound formed
O is generally known as
C Cl
(b) (a) an amine (b) an imine
(c) an enamine (d) a Schiff’s base
EBD_7587
350 Chemistry Objective MCQs

Me H /H2 O + NH3
D CH3CH2CN ¾¾¾¾® A ¾¾¾ ® B ¾¾¾¾
®C
9. ¾® D
Å
N Me
NaOBr
¾¾¾¾® A ¾¾¾® B ¾¾¾¾ ®C
OH
(a) CH3CH2CONH2 (b) CH3CH2COBr
n-Bu Et
(c) CH3CH2NH2 (d) CH3CH2CH2NH2
The alkene formed as a major product in the above X
elimination reaction is 14. CH 3 - CH 2 C º N ¾¾® CH 3CH 2 CHO.
Me Me The compound X is
(a) SnCl2 / HCl / H2O, boil
(a) (b) (b) H2 / Pd – BaSO4
(c) LiAIH4 / ether
(c) Me (d) CH2 = CH2 (d) NaBH4 / ether / H3O+
15. Tautomerism will be exhibited by
NH2 (a) (CH3)3CNO (b) (CH3)2NH
(c) R3CNO2 (d) RCH2NO2
Br (i) NaNO /HCl 16. Hydrolysis of CH 3CH 2 NO 2 with 85% H 2SO 4
10. ¾¾®2
H O
(A) ¾¾¾¾¾¾
2 ¾® (B),
2 (ii) H3PO2
gives
(a) CH3CH2OH (b) C2H6
Product (B) in this reaction is:
(c) CH 3CH = NOH (d) CH 3COOH
Br 17. Towards electrophilic substitution, the most
Br Br
Br reactive will be
(a) (b)
(a) Nitrobenzene
Br (b) Aniline
Br
(c) Aniline hydrochloride
(d) N-Acetylaniline
Br 18. 3, 5-dibromotoluene can be best synthesised by
Br
Br Br CH 3
(c) (d)
(a) Br / H O
HNO2
¾¾¾¾¾
2 2 ®
¾¾¾¾ ®
11. p-Chloroaniline and anilinium hydrogen chloride H3PO2
can be distinguished by:
NH 2
(a) Sandmeyer reaction
CH 3
(b) Carbylamine reaction
(c) Hinsberg’s reaction
(b) Fuming HNO NH
(d) AgNO3 ¾¾¾¾¾¾ 3®
¾¾¾3®
H2SO4 HONO
12. Consider the following sequence of reactions : ¾¾¾¾ ®
CuBr
Reduction CH 3
Compound[A] ¾¾¾¾¾
® [B]
(c) Br /Fe
¾¾¾¾
2HNO
® CH 3CH 2 OH ¾¾¾¾
2 ®
D
The compound [A] is
(a) CH3CH2CN (b) CH3NO2 CH 3
(c) CH3NC (d) CH3CN
(d) CH Cl/AlCl
13. What will be the final product in the following ¾¾¾¾¾¾
3 3®
D
reaction sequence?
Br
Amines 351
19. Which of the following compounds is an 22. Which of the following reagents will be useful to
enamine? distinguish between
(a) N=C= N H 3C NH 2 and

(b) NH2 CH 2 NH 2 ?

(c) N (a) Dilute HCl


(b) C6H5SO2 and OH–/H2O
(d) N
(c) HONO then b-naphthol
20. Which of the following reactions will not give (d) AgNO3 in H2O
N, N- dimethyl benzamide ?
23. Which of the following is the strongest base ?
COOC2H5 + (CH3)2NH ¾®
(a) NH 2 NH 2

(b) CONH 2 + CH 3 MgI ¾¾


® (a) (b)

(c) COCl + (CH 3 ) 2 NH ¾¾


® NO 2
NH
NH
(c) (c)
(d) COOCO +

24. The cyclobutyl methylamine with nitrous acid


(CH 3 ) 2 NH ¾ ¾® gives
21. The product – (C) obtained in the following sequence
of reactions is CH2 OH
(a) (b)
NH2
Br
(1) HONO Sn / HCl
Br ¾¾¾¾¾ ® A ¾¾ ¾¾® (c) (d) All of these
(2) CuCl

25. Which of the following amines will react with


NO2 (1) HONO
B ¾¾¾¾¾® C cyclohexanone to give enamine ?
(2) H3PO 2

Cl (a) (CH 3 ) 2NH (b)


(a) Br Br (b) N
|
H
Br Br
NO 2
(c) (d) All of these
Cl
N
(c) Br Br (d) None of these |
H
EBD_7587
352 Chemistry Objective MCQs
26. In the reaction shown below, the major product(s)
formed is/are
OH
(b)
NH2 N=N

acetic anhydride
¾¾¾¾¾¾¾® Product(s)
CH2Cl2
NH2
Z O OH

H N=N
N CH3 (c)

O
(a) + CH3COOH
NH2
O
(d) N=N
NH2

H + CH3COOH
(b) N CH3 28. In the following reactions, the product S is
O O H3C
¾¾¾¾® ¾¾®
H
N CH3
O
(c) H + H2O
N CH3
O (a)
O

+ –
NH3CH3COO
(b)

(d) H
N CH3
O O
27. In the following reactions, the major product W (c)
is
OH
NH2 , NaOH
NaNO2 , HCl
¾¾¾¾¾®
0°C
V ¾¾¾¾¾¾¾® W (d)

29. In the reaction,


Boiling H 2O
CH 3CN + 2 H ¾ ¾® X ¾¾ ¾ ¾ ¾ ¾® Y , the
(a) term Y is
N=N OH
(a) acetone (b) ethanamine
(c) acetaldehyde (d) dimethylamine
Amines 353

HgCl2 35. The most basic compound among the following is


30. In the reaction C 6 H 5 NH 2 + CS2 ¾¾¾¾ ® (a) Acetanilide (b) Benzylamine
D
the product obtained is (c) p-Nitro aniline (d) Aniline
(a) phenyl isocyanide 36. Which of the following compounds gives dye
(b) phenyl cyanide test?
(c) p-amino benzene sulphonic acid (a) Aniline (b) Methylamine
(d) phenyl isothiocyanate (c) Diphenylamine (d) Ethylamine
31. Arrange the following amines in the order of 37. Acetanilide on nitration followed by alkaline
increasing basicity. hydrolysis mainly gives
(a) o-Nitroacetanilide
NH2 NH2 NH2 (b) p-Nitroaniline
(c) m-Nitroaniline
(a) CH3NH2 < < < (d) 2, 4, 6-Trinitroaniline
CH3
38.
OCH3 NO2
(CH CO) O Br2/CH3COOH
NH2 NH2 NH2 ¾¾¾¾®
3 2
A ¾¾¾¾¾®B
+
H /H O
¾¾¾®
2
X.
(b) < < < CH3NH2 NH2

What is X?
OCH3 NO2
CH3
NH2 NH2 NH2
Br
(a)
(c) < < < CH3NH2

NH2
NO2 OCH3
CH3
NH2 NH2 NH2
(b)
(d) < < < CH3NH2 Br
NH2
OCH3 NO2
32. The final product formed when Methyl amine is CH3
treated with NaNO2 and HCl is: COCH3
(a) Diazomethane (b) Methylalcohol (c)
(c) Methylcyanide (d) Nitromethane
33. The order of basicity of amines in gaseous state
is : NH2
(a) 1° > 2° > 3° > NH3 (b) 3° > 2° > NH3 > 1° CH3
(c) 3° > 2° > 1° > NH3 (d) NH3 > 1° > 2° > 3°
34. Carbylamine forms from aliphatic or aromatic
primary amine via which of the following (d)
intermediates ? COCH3
(a) Carbanion (b) Carbene NH2
(c) Carbocation (d) Carbon radical
EBD_7587
354 Chemistry Objective MCQs
39. Which reaction sequence would be best to (a) CH3 – CH2 – CO – NH2
prepare 3-chloroanilne from benzene ? (b) CH3 – CN
(a) Chlorination, nitration, reduction (c) (CH3CO)2O
(b) Nitration, chlorination, reduction (d) CH3 – COOH
(c) Nitration, reduction, chlorination
46. The order of basicity of the compounds
(d) Nitration, reduction, acetylation, chlorination,
hydrolysis O
40. A compound ‘A’ when treated with HNO3 (in
presence of H2SO4) gives compound ‘B’, which
N N N N
is then reduced with Sn and HCl to aniline? The
compound ‘A’ is H H is
H
(a) toluene (b) benzene
(I) (II) (III) (IV)
(c) ethane (d) acetamide
41. An organic compound (A) on reduction gives
compound (B). (B) on treatment with CHCl3 and
alcoholic KOH gives (C). (C) on catalytic (a) IV > I > III > II (b) I > III > II > IV
reduction gives N-methylaniline. The compound (c) III > I > IV > II (d) II > I > III > IV
A is 47. Complete reduction of benzene-diazonium
(a) Methylamine (b) Nitromethane chloride with Zn/HCl gives:
(c) Aniline (d) Nitrobenzene
(a) Aniline (b) Phenylhydrazine
42. Predict about the relative boiling point of the
following two amines. (c) Azobenzene (d) Hydrazobenzene
48. Conversion of benzene diazonium chloride to
chlorobenzene is an example of which of the
NCH3 ; H3C NH following reactions?
(a) Claisen (b) Friedel-craft
I II
(c) Sandmeyer (d) Wurtz
(a) Boiling point of I > II
(b) Boiling point of II > I 49. In a set of reactions p-nitrotoluene yielded a
(c) Both should have equal boiling points product E.
(d) It can’t be predicted
43. Which statement is true regarding the following CH3
structure?
Br Sn/HCl
N ¾¾¾®
2 B ¾¾¾¾
®C
FeBr3
CH3 C3H7
NaNO
CuBr
C2H5 ¾¾¾¾ 2 ® D ¾¾¾ ®E
NO2 HCl HBr
(a) It is a chiral molecule The product E would be:
(b) It exists in two resolvable optically active
forms CH3 CH3
(c) Both (a) and (b) Br Br Br
(d) Neither (a) nor (b)
(a) (b)
44. Which one of the following is not an oxidation
product of a primary amine?
(a) A hydroxylamine Br Br
(b) A nitroso compound
(c) A nitro compound CH3 CH2 Br
(d) All of these Br
45. Identify Z in the following sequence of reactions– (c) (d)
D P2O5
CH 3 COONH 4 ¾¾® X ¾¾¾® Y Br
H 2O/H
¾¾¾¾
®Z Br
Amines 355
50. Which of the following will produce isopropyl 56. What is the product of the following series of
amine – reactions?
NH 2
NH OH
2 4® LiAlH Cl
(I) (CH 3 ) 2 CO ¾¾¾¾® X ¾¾¾¾
NaNO 2 , HCl CuCN
¾¾¾¾¾¾ ® ¾¾¾¾
®
NH3 LiAlH 4 H2 O, 0 °C
(II) CH 3 - CH 2 - CHO ¾¾¾® X ¾¾¾¾
® (1) CH MgBr
heat ¾¾¾¾¾¾
3

(2) H2O, H
NH3
(III) (CH3 ) 2 CH - OH + PCl5 ¾¾
® X ¾¾¾ ® O = C - CH 3 COOCH 3
heat Cl Cl
(IV) CH3 - CH 2 - CH 2 - NH 2 ¾¾¾ ® (a) (b)
(a) I, II (b) II, III
(c) I, III (d) IV only
51. Acetaldoxime reacts with P2O5 to give HO - CH - CH 3 CH 2 CN
(a) Methyl cyanide (b) Methyl cyanate Cl Cl
(c) Ethyl cyanide (d) None of these (c) (d)
52. Identify the prdouct C in the series
Na / C H OH
CH 3CN ¾¾¾2¾5¾¾® A ¾¾¾
¾2 ®B HNO 57. The starting reagents needed to make the azo
compound shown below
Cu / 573K
¾¾¾¾
¾® C
CH3 – CH 2 N=N OH
(a) CH 3COOH (b) CH 3CH 2 NHOH
(c) CH 3CONH 2 (d) CH 3CHO NH2
53. An organic compound ‘A’ having molecular
formula C2H3N on reduction gave another
(a) + ethylamine
compound ‘B’. Upon treatment with nitrous acid,
‘B’ gave ethyl alcohol. On warming with
chloroform and alcoholic KOH, B formed an OH
offensive smelling compound ‘C’. The compound
C 2 H5 OH
‘C’ is
(b) +
(a) CH3CH 2 NH2 (b) CH 3 CH 2 N ®
=C
NH 2
(c) CH3 C º N (d) CH 2 CH 2 OH
54. Ethyl isocyanide on hydrolysis in acidic medium NH2 NH 2
generates
(a) propanoic acid and ammonium salt (c) +
(b) ethanoic acid and ammonium salt
(c) methylamine salt and ethanoic acid
OH C 2 H5
(d) ethylamine salt and methanoic acid
55. Primary nitro compounds react with nitrous acid NH 2 OH
to form nitrolic acids which dissolve in NaOH
giving
(d) +
(a) yellow solution
(b) blue solution
(c) colourless solution C 2 H5
(d) red solution
EBD_7587
356 Chemistry Objective MCQs
58. Which of the following does not give N–ethyl 62. Aniline is reacted with bromine water and the
cyclopentylamine as major product ? resulting product is treated with an aqueous
solution of sodium nitrite in presence of dilute
HCl. The compound so formed is converted into
tetrafluoroborate which is subsequently heated
(a) NH 2 + CH 3CHO ¾H / Pt
¾2 ¾¾® dry. The final product is
H / Pt
(a) p-Bromofluorobenzene
(b) O + CH 3CH 2 NH 2 ¾¾2 ¾
¾® (b) p-Bromoaniline
O (c) 2, 4, 6-Tribromofluorobenzene
|| (d) 1, 3, 5-Tribromobenzene
(c) CH3CH2NH2
C - Cl ¾¾¾¾¾¾ ® 63. Arrange the following three compounds in order
Pyridine
of decreasing acidity
(i) LiAlH , Et O
¾¾¾¾¾¾¾
4 2 ®
+ + +
(ii) H2O
NH3 NH3 NH3
CH COCl
(d) NH 2 ¾¾¾¾¾
3 ®
Pyridine (1) LiAlH4
¾¾¾¾¾ ® (A) (B) (C)
(2) H 2O
59. Match the compounds in List I with their nature
from List II, as seen in aqueous medium NO2 CH3
List I List II (a) B > A > C (b) B > C > A
I. Acetamide A. Acidic (c) C > B > A (d) C > A > B
II. Benzonitrile B. Basic 64. In the reaction
III. Triethylamine C. Neutral
CH Br
3
IV. Phenol ¾¾¾¾¾¾¾¾¾¾¾¾¾¾¾¾¾¾¾¾¾®
CH 2 CN NaNH ,NH , - 80°C
2 3
(a) I–C; II–C; III–B; IV–A
(b) I–B; II–C; III–C; IV–A the products obtained are
(c) I–C; II–B; III–B; IV–C CH3
(d) I–A; II–A; III–C; IV–B |
60. The correct order of basicities of the following (a) CH - C º N
compounds is
(b) CH 2 .NH 2
NH
CH3 C CH3 CH2 NH2 (c) CH 2 .OH
NH2 (d) None of these
1 2 65. In the acidic reduction of nitrobenzene, which
O of the following is the intermediate?
(a) C 6 H 5 - N = O
(CH3 )2 NH CH3 C NH2
(b) C 6 H 5 NH - NHC 6 H 5
4 (c) C 6 H 5 - N = N - C 6 H 5
3
O
(a) 2 > 1 > 3 > 4 (b) 1 > 3 > 2 > 4 ­
(c) 3 > 1 > 2 > 4 (d) 1 > 2 > 3 > 4 (d) C 6 H 5 - N = N - C6 H 5
61. In the following reaction, X is 66. ‘Z’ in the following sequence of reactions is
Bromination NaNO2 /HCl
X ¾¾¾¾¾® Y ¾¾¾¾¾®
HNO /H SO Zn/HCl
Boiling C6 H6 ¾¾¾¾¾¾
3 2 4 ® W ¾¾¾¾®
Z ¾¾¾¾¾ ® D
C H OH Tribromobenzene
2 5
NaNO 2 H 2O/H 3PO 2
(a) benzoic acid (b) salicylic acid X ¾¾¾¾ ® Y ¾¾¾¾¾¾ ®Z
HCl
(c) phenol (d) aniline
Amines 357

COOH (a) All the four are aromatic


(b) I, III and IV are aromatic

(a) (b) (c) I, II and III are aromatic


(d) I and III are aromatic
70. Ethylene can be prepared in good yield by
OH Cl
heat
(a) CH 3 CH 2 N + (CH 3 )3 I - ¾¾¾
®
(c) (d) CH 2 = CH 2 + (CH3 )3 N + HI
heat
(b) CH 3CH 2 N + (CH3 ) 3 OH - ¾¾¾
®
67. An organic compound (A) on reduction gives
compound (B). (B) on treatment with CHCl3 and CH 2 = CH 2 + (CH3 )3 N + H 2O
alcoholic KOH gives (C). (C) on catalytic (c) Both (a) and (b)
reduction gives N-methylaniline. The compound
(d) heat
A is CH 3CH 2 NH 2 ¾¾¾
®
(a) Methylamine (b) Nitromethane CH 2 = CH 2 + NH3
(c) Aniline (d) Nitrobenzene 71. Which of the following leads to carbon-carbon
double bond?
heat
68. Cl– H3N+ N+ H3Cl– ¾¾®
(a) 1º Amine + RCHO ®
Z; Z is
(b) 2º Amine + R2CO ®
(a) H2N NH2 (c) 2º Amine + RCHO ®
(d) Both (b) and (c)
(b) 72. In Hofmann bromamide degradation, one of the
N
H important steps is the migration of
(a) an alkyl group without its electron pair to
(c) electron deficient N atom.
N (b) an alkyl group with its electron pair to
H
electron deficient O atom.
(c) an alkyl group with its electron pair to
(d)
N electron rich N atom.
H (d) an alkyl group with its electron pair to
69. Pyrrole and pyridine both are basic and form salts electron deficient N atom.
with acids?
O
ôô
+
H
+ (i) OD - /Br
H
¾® ¾® 73. (CH 3 )3 C - C - NH 2 ¾¾¾¾¾¾

Product P
.. + .. + (ii)D 2O
N N N N
H H2 H
I II III IV is
Which of the following statement is true (a) (CH3)3 CNH2 (b) (CH3)3 CNHD
regarding the aromatic character of the four (c) (CH3)3CND2 (d) no reaction
species?
EBD_7587
358 Chemistry Objective MCQs
74. Identify product D in the following reaction 78. The correct stability order of the following
sequence: resonance structures is

CH3
K 2 Cr2 O7 : H 2SO 4 + – + –
CH3 C CH2CH2OH ¾¾¾¾¾¾¾
H 2 O, heat
®A H2C = N = N H 2C – N = N
(I) (II)
CH3
– + – +
(i) LiAlH 4 H2C – N = N H2C – N = N
SOCl (CH3 )2 NH diethyl ether
¾¾¾®
2
B ¾¾¾¾¾ ® C ¾¾¾¾¾¾
(ii) H 2O
®D (III) (IV)
(2 mole)
(a) (I) > (II) > (IV) > (III)
CH3
(b) (I) > (III) > (II) > (IV)
(a) CH3CCH2C N (c) (II) > (I) > (III) > (IV)
CH3 (d) (III) > (I) > (IV) > (II)
CH3 79. Introduction of a methyl group in ammonia
markedly increases the basic strength of ammonia
(b) CH3CCH2CH2 N(CH3)2 in aq. solution, introduction of the second methyl
CH3 group increases only marginally the basic
CH3 N(CH3)2 strength of methyl amine in water. This is due to
(a) different type of hybridisation in the two
(c) CH3CCH2CHN(CH3)2 amines.
CH3 (b) protonated dimethyl amine is more solvated
CH3 than methyl amine.
(c) protonated dimethyl amine is more solvated
(d) CH3CCH2CHN(CH3)2
than the protonated methyl amine.
CH3 OH (d) protonated dimethyl amine is less stable
75. The correct order of decreasing basic character than the protonated methyl amine.
of the three aliphatic primary amines is 80. Which of the following amines can be resolved
NH2 NH2 into two enantiomers?
I II

NH2 .. ..
III
N CH2CH3 N CH3
(a) I > II > III (b) III > II > I
(c) I > II » III (d) I = II º III H CH3 H CH3
76. Which of the following can undergo electrophilic I II
substitution when treated with nitrous acid at ..
0º C? H3C NH2
..
(a) C6H5NH2 (b) C6H5NHCH3 N C CH2CH3
(c) C6H5N(CH3)2 (d) None
CH3 H CH3
77. When aniline reacts with oil of bitter almonds
III IV
(C6H5CHO) condensation takes place and benzal
derivative is formed. This is known as (a) I, IV (b) I, II
(a) Million's base (b) Schiff's reagent
(c) Schiff's base (d) Benedict's reagent (c) I, III, IV (d) III, IV
Amines 359

Answer KEYs
1 (c) 9 (d) 17 (b) 25 (d) 33 (c) 41 (d) 49 (b) 57 (b) 65 (a) 73 (c)
2 (b) 10 (b) 18 (a) 26 (a) 34 (b) 42 (b) 50 (c) 58 (c) 66 (b) 74 (b)
3 (c) 11 (d) 19 (c) 27 (a) 35 (b) 43 (a) 51 (a) 59 (a) 67 (d) 75 (a)
4 (a) 12 (d) 20 (b) 28 (a) 36 (a) 44 (d) 52 (d) 60 (b) 68 (c) 76 (c)
5 (a) 13 (c) 21 (c) 29 (c) 37 (b) 45 (d) 53 (b) 61 (d) 69 (b) 77 (c)
6 (d) 14 (a) 22 (c) 30 (d) 38 (b) 46 (b) 54 (d) 62 (c) 70 (b) 78 (b)
7 (d) 15 (d) 23 (b) 31 (c) 39 (b) 47 (a) 55 (d) 63 (a) 71 (d) 79 (d)
8 (c) 16 (d) 24 (d) 32 (b) 40 (b) 48 (c) 56 (a) 64 (a) 72 (d) 80 (d)

1. (c) Diazonium salts react with copper powder and hydrogen bromide to form bromobenzene. This
reaction is known as Gattermann reaction.
+ –
N NCl Br

Cu
¾¾®
HBr

2. (b) C7H9N having one benzene ring may be represented as C6H5CH4N, may be in the form of 1º and 2º
amines in the following five isomeric forms.

NH2 NH2 NH2 CH2NH2 NHCH3

CH3

CH3
CH3
(I) (II) (III) (IV) (V)

3. (c) The compounds containing active H-atoms (H atoms attached to N, O or S) react with CH3COCl to
form acetyl derivatives.
4. (a) Excess of HCl is used to convert free aniline to aniline hydrochloride otherwise free aniline would
undergo coupling reaction with benzenediazonium chloride.
5. (a) R 2 NNO + H 2O ® R 2 NH + HNO 2
EBD_7587
360 Chemistry Objective MCQs

D
6. (d) C6 H5 NH 2 + COCl2 + KOH ® C6H5 NH.COCl + HCl ¾¾® C6 H 5 NCO + HCl
7. (d) The reaction can be completed as follows:

NH2
NaNO /HCl CH3
¾¾¾¾¾®
2
N2Cl + N
(diazotisation) CH3
‘A’ ‘B’ N, N- Dimethylaniline
(Aniline) Benzene

¾¾¾®
diazonium chloride cold

CH3
N=N N
CH3
‘C’
p-Dimethylaminazobenzene

OH
8. (c) O + HN(CH3)2 ¾¾® N (CH3)2

–H O
2
¾¾® N (CH3)2

Enamine

9. (d) Hofmann's rule : When theoretically more than one type of alkenes are possible, the alkene containing
least alkylated double bond is formed. Hence

Me Me
D
¾® + CH2 = CH2 + H2O
+ ..
OH N Me N – Me
n-Bu | |
Et n-Bu

NH2
Br Br
Br Br
10. (b) (A) (B)

Br
Br
11. (d) p-Chloroaniline and anilinium hydrogen chloride can be distinguished by AgNO3. Anilinium hydrogen
chloride will give white ppt of AgCl on reaction with AgNO3, but p-chloronoaniline will not react with
it because Cl is directly attached to benzene nucleus.
Reduction HONO
12. (d) CH3CN ¾¾¾®CH3 – CH2 – NH2 ¾¾¾® CH3CH2OH
(A) (B)
Amines 361

H /H 2O + 3® NH
13. (c) CH3CH2CN ¾¾¾¾ ® CH3CH 2COOH ¾¾¾
D
(A)

Hoffmann bromamide reaction


CH3CH 2CONH 2 ¾¾¾¾¾¾¾¾¾¾¾
® CH3CH 2 NH2
NaOBr
(B) (C)

14. (a) It is Stephen’s reaction.


SnCl 2 /HCl H 2O
CH3CH2C º N ¾¾¾¾¾ ® CH 3CH 2 CH = NH.HCl ¾¾¾ ® CH 3CH 2 CHO + NH 4 Cl
15. (d) Nitro compounds having a-hydrogen show tautomerism

+ O + OH
RCH2– N RCH = N
O– O–
16. (d) 1º Nitroalkanes on hydrolysis with boiling 85% H2SO4 give acids.
heat + -
CH 3 CH 2 NO 2 + H 2 O + H 2SO 4 ¾¾¾® CH 3 COOH + [ NH 3 OH] HSO 4

O
.. + – ..
17. (b) NO2 NH2 NH3Cl H–N–C–CH3

Nitrobenzene Aniline Aniline hydrochloride N- Acetylaniline


+
Nitrobenzene and aniline hydrochloride have electron- withdrawing (–NO2 and – N H3 ) groups,
hence these will undergo electrophilic substitution with difficulty. Aniline and N– acetylaniline
(acetanilide) have electron– releasing groups, however –NHCOCH3 is less electron- releasing than
– NH2 due to delocalisation of lone pair of electron on N toward carbonyl group. Hence aniline
(having – NH2) will undergo electrophilic substitution most easily.

CH 3 CH 3 CH 3
HONO
18. (a) Br / H O
¾¾2¾ ¾
2¾® ¾¾ ¾
¾®
Br H 3PO 2
Br Br Br
NH 2 NH 2

19. (c) Enamine (ene for carbon-carbon double bond and amine for amine group). >C = C – N<
20. (b) (a) C 6 H 5COOC 2 H 5 + (CH 3 )2 NH ® C 6 H 5CON (CH 3 ) 2 + C 2 H 5OH
(b) C 6 H 5CONH 2 + CH3MgI ® C 6 H 5CO NH MgI + CH 4
(c) C 6 H 5COCl + (CH3 )2NH ® C 6 H 5CON (CH 3 ) 2 + HCl
(d) C 6 H 5CO.O.COC 6 H 5 + (CH 3 )2 NH ® C 6 H 5CON (CH 3 ) 2 + C 6 H 5COOH
EBD_7587
362 Chemistry Objective MCQs

NH 2 Cl Cl
Br Br Br Br
Br Br
HONO Sn/HCl
21. (c) ¾¾¾¾¾®
CuCl
¾¾¾¾
®
Sandmeyer
NH 2 Cl
NO 2 NO 2 Br Br
(i) HONO
¾¾¾¾¾¾¾¾¾®
(ii) H3 PO2
elimination of NH2
22. (c) The first compound will give azo dye

H 3C N=N

OH
23. (b) Aliphatic amines are stronger base than aromatic amines. Further the order 2° > 1° > 3° > NH 3
OH H OH
| | HÅ
Å
CH 2 NH 2 :N = O ¾¾® CH - N - N - O ¾¾ ¾®
:

24. (d) 2
|

:
H
H OH
| | Å
CH 2 - ÅN - N:- ¾-¾ 2 H 2O
¾ ¾® CH 2 - N º N : ¾ ¾® +
| | + : N º N:
H OH
-H +

CH2

Å
-H +
¾¾¾®

OH -

OH

25. (d) Carbonyl compounds containing at least one a -hydrogen atom react with a secondary amine to give
enamine

R R
> CH - C = O + HN ¾¾ ¾¾ ® > C = C-N
> >
R - H 2O R
Primary amines also form enamines

| | |
> CH - C = O + H 2 N - R ¾
¾® > C = C - N
> H > CH - C = N - R
R
enamine imine
In second case enamine – amine tautomerism is possible and equilibrium lies completely on the imine
side.
Amines 363
gg
26. (a) — NH 2 group is acetylated by acetic anhydride in methylene chloride (solvent).
COCH3 O O
CH2Cl2 +
CH2 NH 2 + (CH3CO)2O ¾¾® CH2 NH + CH3COOH C NH2 ¬¾® C NH2
Note that —CONH2 group does not undergo acetylation because here lone pair of electrons is
delocalised.


NCl
OH
NH2 , NaOH
OH
NaNO , HCl
27. (a) ¾¾¾¾®
2
¾¾¾¾¾¾¾¾¾®
0 °C

(V) (W)

(i) O NH
28. (a) ¾¾¾¾
2
® ¾¾®
3
(ii) Zn, H2O

OH

NH –2H O N
¾¾®
2

OH

29. (c) H O , boil


CH 3CN + 2H ® CH 3CH = NH ¾¾2 ¾ ¾
¾® CH 3CHO
X Y

Note that cyanides are reduced only by 2H atoms, so reduction stops at aldimine stage which on
hydrolysis with water gives aldehyde; it is an example of Stephen’s reaction.

30. (d) C 6 H 5 NH 2 + CS 2 ® C 6 H 5 NH .CS.SH ¾HgCl


¾¾ ¾2 ® C H N = C = S + HgS + 2HCl
6 5

The reaction is called mustard oil reaction.


31. (c) Aliphatic amines are more basic than aromatic amines thus methylamine is most basic. Electron
donating groups increase the basicity whereas electron withdrawing groups decrease the basicity
of the aromatic amines. Thus p-methoxyaniline is more basic then aniline which is further more basic
then p-nitroaniline.

NH2 NH2 NH2

CH3 — NH2

NO2 OCH3
EBD_7587
364 Chemistry Objective MCQs
32. (b) CH 3 NH 2 + HNO 2 ¾¾ ® CH 3OH + N 2 + H 2O
33. (c) The correct order of relative basicity of amines in the gas phase is 3° > 2° > 1° > NH3
The alkyl group releases electron and thus, tends to disperse the positive charge of the alkyl ammonium
ion and therefore stabilises it. Since, NH4+ (from NH3) has no such alkyl group, it is not stabilised to
such an extent as alkylammonium ion.
34. (b) RNH2 + CHCl3 + 3KOH ¾® RNC + 3KCl + 3H2O
Mechanism
OH - / D &&
CHCl3 ¾¾¾¾¾¾® CCl2
a- elimination
Dichlorocarbene
H H H
H Cl
+ +
R N + CCl2 ¾¾® R N CCl2 ¾¾® R N C ¬¾¾ R N C

¾
Cl Cl
H H H H

¾
®
H +
R Nº C H
¬¾¾
R N C

¾
Cl

¾®
OH
+
R Nº C + HOH

35. (b) CH2–NH2 compound is most basic due to presence of localized lone pair of electron

on nitrogen atom while other compounds have delocalized lone pair of electron.
36. (a) Only 1º aromatic amines undergo coupling reactions to form a dye

NHCOCH3 NHCOCH3 NH2


37. (b) Nitration H2O/OH
¾¾¾® ¾¾¾®

NO2 NO2
p-Nitroaniline

CH3 CH3 CH3 CH3

(CH CO) O
3 2 Br /CH COOH
2 3 H +
¾¾¾¾® ¾¾¾¾¾¾® ¾®
38. (b)
Br Br
NH2 NHCOCH3 NHCOCH3 NH2
(–NHCOCH3 is more electron-releasing than –CH3 group)

NO2 NO2 NH2

HNO3, H2SO 4 Cl /Fe reduction


39. (b) ¾¾¾¾¾® ¾¾®
2
¾¾¾¾®
333 K
Cl Cl
Amines 365
(a) When chlorination is done earlier than nitration, chlorobenzene formed at first step would introduce –
NO2 group in ortho-position, not in m-position.
(b) Again if –NO2 group is reduced earlier than the chlorination step, –NH2 group formed on reduction
will again introduce –Cl in o-position.

40. (b) HNO / H SO Sn / HCl


A ¾¾ ¾3 ¾ ¾
2 ¾
4 ® B ¾¾ ¾¾® C H NH
6 5 2

This indicates that B is C6H5NO2 and hence A is C6H6

NO2 NH2 NC NHCH3

reduction CHCl H /Catalyst


41. (d) ¾¾¾¾® ¾¾¾¾® 3
¾¾¾¾®
2
alc. KOH

(A) (B) (C) N-Methylaniline

42. (b) In II, intermolecular H-bonding between similar molecules is possible, while this is not so in I.
43. (a) In amines, N is sp3 hybridised and thus has pyramidal shape. In the given structure, since the three
alkyl groups are different, and the fourth corner of the pyramid is occupied by lone pair of electrons,
the molecule is chiral. However, the two enantiomers of the amine are not resolvable because of their
rapid interconversion through a transition state having planar structure (sp2 hybridised nitrogen)
44. (d) All of the three are oxidation products of a 1º amine.

H OH O
[O] .. [O]
[O] +
R – .N. – H ¾® R – .N. – H ¾¾® R – N = O ¾® R – N
O–
1º Amine Hydroxylamine Nitroso Nitro

D 2 5 ® CH CN ¾¾¾¾
3 D /P O H OÅ
45. (d) CH3COONH 4 ¾¾® CH3 CONH 2 ¾¾¾¾ 3 ® CH3 COOH
(X) (Y) (Z)

46. (b) The order of basicity is I > III > II > IV.
The lone pair of electrons on N is more readily available for protonation in I and III then in II.
III contains an oxygen atom which has – I effect due to which it is less basic than I. In compound
IV lone pair of e–s on N–atom is contributed towards the aromatic sextet formation and hence
is not at all available for protonation. Hence option (b) is correct.
47. (a) On reduction with Zn and HCl, C6H5N2Cl forms aniline as the main product.

CuCl/HCl
48. (c) C6 H 5 N 2 Cl ¾¾¾¾¾
® C6H 5Cl + N2
Benzene or Cu + HCl Chloro benzene
diazoniumchloride

The above reaction is known as Sandmayer’s reaction.


EBD_7587
366 Chemistry Objective MCQs

CH3 CH3 CH3


Br Br
Br Sn/HCl
49. (b) ¾¾® 2
¾¾®
FeBr 3

NO2 NO2 NH2

¾¾¾¾¾
CH3 CH3
Br Br
HBr ¾¾¾¾® NaNO2
¾¾®
HCl
CuBr

Br N + º NCl

NH 2OH LiAlH
50. (c) (CH3 )2 CO ¾¾¾¾ ® (CH3 ) 2 CH = N - OH ¾¾¾¾
4 ® (CH ) CHNH
3 2 2

PCl
5 ® (CH ) CHCl ¾¾¾
3 ® (CH ) CHNH NH
(CH3 ) 2 CHOH ¾¾¾ 3 2 3 2 2

51. (a) PO
CH 3CH = NOH ¾¾
2¾5 ® CH CN + H O
3 2

Na / C H OH HNO Cu
52. (d) ¾® CH 3CH 2 NH 2 ¾¾ ¾
CH 3CN ¾¾ ¾2¾5¾ ¾2 ® CH CH OH ¾¾® CH CHO
3 2 3
(A) ( B) 573 K ( C)

reduction HNO
53. (b) A ¾¾¾® B ¾¾¾®
2
C2H5OH
(C2H3N)
Carbylamine reaction
Offensive smell (C)

Given reactions indicate that B has 1º NH2 group, and thus A, C2H3N, should be CH 3C º N . Hence
C should be CH3CH2NC

reduction 3 ® CH CH N = C CHCl ®
CH3 C º N ¾¾¾¾® CH3 CH 2 NH 2 ¾¾¾¾ 3 2 KOH
A B C

+
H
= C + H 2 O ¾¾¾
54. (d) CH3 CH 2 N ® ® CH 3CH 2 NH 2 + HCOOH

55. (d) Red solution

NH 2 N 2 Cl
CN
Cl Cl
NaNO /HCl Cl
56. (a) ¾¾¾¾¾
2 ¾
® ¾¾¾
CuCN
¾®
H2O, 0°C

CH 3 - C = N - MgBr CH 3 - C = O

Cl Cl + NH 3 + HOMgBr
(i) CH MgBr (ii) H O/H ∗
¾¾¾¾¾¾
3 ® ¾¾¾2 ¾¾↑
Amines 367

+ -
57. (b) C2H5 ¾
NaNO - HCl
NH 2 ¾¾ ¾ ¾
2 ¾¾®
C2 H5 N º N - Cl
0 - 5°C

+ -
C2 H5 N º N - Cl + OH ®

C2 H5 N= N OH.

Pyridine
58. (c) COCl + CH 3CH 2 NH 2 ¾¾¾¾ ®

(i)LiAlH , Et O
CONHCH 2CH 3 ¾¾¾¾¾¾¾¾ ®
4 2
(ii)H O CH 2 NHCH2CH3
2

N-ethyl cyclopentyl methyl amine

59. (a) Acetamide is basic due to the presence of lone pair of electrons of N; it is also acidic because its
conjugate base shows resonance.

O O O-
|| || |
base
® CH 3- C - NH - ¬¾® CH 3 - C = NH
CH 3- C - NH 2 ¾¾¾

(more stable due to –ve charge on O)


Benzonitrile (C6H5CºN) acts as an electrophile (Lewis acid) due to electron deficiency of C of CN as
well as nucleophile (Lewis base) in nature due to presence of lone pair electrons on N; hence it is
neutral Triethylamine and phenol are basic and acidic in nature respectively.
60. (b) Guanidine is most basic because its conjugate acid is stabilized by two equivalent resonance
structures,.

NH NH2 + NH NH2
2
||
H + | + || | +
CH 3- C - NH 2 ¾¾¾® CH3- C - NH 2 ¬¾® CH3- C - NH 2 ¬¾® CH3- C = N H 2

Further 2º amines are more basic than 1º, while amides are least basic due to delocalisation of the
lone pair of electrons on N

. .-
:O : : O:
|| . . | +
CH3- C- NH 2 ¬¾® CH 3- C = NH 2

Thus the correct order of basicities is

NH CH 3 O
ôô ô ôô
CH 3 - C - NH 2 > CH 3 - N H > CH 3CH 2 NH 2 > CH 3 - C - NH 2
EBD_7587
368 Chemistry Objective MCQs
61. (d) Proceed backward ; tribromobenzene is produced by boiling compound Z with C2H5OH ; Z in turn is
obtained by diazotisation of Y, so Y and Z should have –NH2 and –N2Cl groups respectively, in
addition to three Br atoms. Hence X should be C6H5NH2

NH2 NH2 N2Cl


Br Br Br Br Br Br
bromination NaNO2 C2H5OH
¾¾¾¾¾® ¾¾¾¾® ¾¾¾¾®
HCl boil

Br Br Br
X Y Z

NH2 NH2 N2Cl F


Br Br Br Br Br Br
Br2 water NaNO2 (i) BHF 4
62. (c) ¾¾¾¾® ¾¾¾¾® ¾¾¾¾®
HCl (ii) heat

Br Br Br
63. (a) The electrons withdrawing groups decrease the basic character and increase the acid character by
inductive effect and resonance.
64. (a) Nitriles having a -hydrogen atom form alkyl derivatives with RBr in presence of NaNH 2 / NH 3 .
65. (a) Nitrobenzene is reduced in the following manner through various intermediates to form aniline as
the final product.

2[ H ] 2[ H ] 2[ H ]
C 6 H 5 NO 2 ¾¾¾® C 6 H 5 NO ¾¾¾® C 6 H 5 NHOH ¾¾¾® C 6 H 5 NH 2
nitrobenzene nitrosobenzene phenylhydroxyla mine aniline

NO2 NH2 N2Cl

HNO Zn/HCl NaNO H PO


¾¾®
3
¾¾¾¾¾¾® ¾¾®
2
¾¾¾®
3 2
66. (b) H2SO4 (acidic reduction) HCl (reduction)

W X Y Z
NO2 NH2 NC NHCH3

reduction CHCl H /Catalyst


¾¾¾¾® ¾¾¾¾®
3
¾¾¾¾®
2

67. (d) alc. KOH

(A) (B) (C ) N-Methylaniline

heat
68. (c) + + ¾¾¾® +NH4Cl
N N N
– –
Cl H3 H3Cl H
Amines 369

H +
69. (b) ¾¾¾®
.. or +
N N N
H H H H
I (Sextet present, II (not aromatic)
hence aromatic) (Aromatic sextet is not
(e pair on N forms a part present because e pair is
of aromatic sextet) involved in protonation)

H +
or ¾¾¾®
+
N N N
H
III (sextet present, hence aromatic) IV (sextet present,
(e pair on N, not a part of sextet) hence aromatic)

70. (b) This is an example of Hofmann elimination which generally takes place by E2 mechanism and the
latter requires a strong base (recall that OH– is a strong base than I–). The NH2– , being a strong base,
can’t be eliminiated easily.
OH –H O
C = O ¾¾® C ¾¾®
2
71. (d) RNH2 + C = NR
NHR
1º Amine Aldehyde/ketone Carbinolamine Imine
(Schiff's base)

CH3 CH3 NR2 CH2


R2NH + C = O ¾¾¾® C ¾¾¾® C = NR2
R R OH R
2º Amine Aldehyde / ketone Carbinolamine Enamine
(R = H) (R=Alkyl)

72. (d) For this one should remember that Hofmann’s degradation involves migration to electron deficient
nitrogen atom, hence the alkyl group will migrate with its bonding pair of electrons.

O O O O
.. – OBr .. OH
- .–. ..
R C NH2 ¾¾® R C NHBr ¾¾® R C .N. Br ¾® R C N
..
Amide
(note that N is electron-deficient)

.. H O
¾¾® O C NR ¾¾®
2
RNH2+ CO32–
Alkyl isocyanate Amine
73. (c) Remember that in Hofmann rearrangement, the two original H atoms of the –CONH2 group are
removed by base (OH–) and new H’s are derived from H2O.
O
ôô
(i) OD – /Br
(CH 3 )3 C - C - NH 2 ¾¾¾¾¾2¾
® (CH 3 )3 C - ND 2
(ii) D 2 O
EBD_7587
370 Chemistry Objective MCQs

CH3 O CH3 O
ô ôô ô ôô
74. (b) (A) CH3 — C — CH 2 — C — O — H (B) CH3 — C — CH 2 — C— Cl
ô ô
CH3 CH3

CH3 O CH3
ô ôô | CH3
(C) CH3 — C — CH 2 — C — N — CH3 (D) CH3 – C – CH2 – CH2 – N
ô ô | CH3
CH3 CH3 CH3
75. (a) Note the point of difference in the given compounds which here lies at b-carbon. In I, II, III, the
b-carbon atoms are sp3, sp2 and sp hybridised respectively which in turn cause the difference in their
s-character. We know that more is the s character of an atom, greater will be its electron-withdrawing
nature. Thus sp (50% s character) hybridised carbon is most electron-withdrawing, while sp3 (25%
s-character) is least electron-withdrawing. Further, we know that presence of an electron-withdrawing
group decreases basicity of an amine. Thus
NH2 NH2 NH2
b b b
I. (b – sp3 hybridised) II. (b – sp2) III. (b – C sp2)
+
76. (c) Nitrosonium ion, N O from HONO is a weak electrophile, hence it can attack only on highly activated
benzene nucleus, provided proper position, p- or o- is free.
H3C CH3
..
N N(CH3)2

+
NO
¾¾®

NO
77. (c) Benzaldehyde reacts with primary aromatic amine to form schiff's base
C6 H 5CH = O + C 6 H 5 NH 2 ¾
¾® C 6 H5 CH = NC 6 H 5
Benzaldehyde Aniline Benzylidene aniline

+ - - +
78. (b) H 2C = N + = N - > H 2 C- - N + º N > H 2 C - N = N > H 2 C - N = N
I III II IV
Octet complete, Octet complete, Octet incomplete, Octet incomplete,
6 covalent bond, 6 covalent bond, 5 covalent bond, 5 covalent bond,
–ve charge on N –ve charge on C –ve charge on N –ve charge on C

79. (d) The basic character of an amine in water is determined by (i) electron availability on the N atom and
(ii) the extent of stabilization of the cation (protonated amine) due to solvation by hydrogen bonding
H OH2 H OH2
H+ + H+ +
CH3 N H ¾¾¾® CH3 N H OH2 ; CH3 N H ¾¾¾® CH3 N H OH2

H H 2 OH CH3 CH3
+
Methyl amine Protonated methyl amine, CH 3NH 3 Dimethyl amine Protonated dimethyl amine
+
(Highly stabilized) (Lesser stabilized than CH 3NH 3)
80. (d) Most of the simple amines those having smaller alkyl (groups) can’t be resolved , although they may
contain three different alkyl groups.
28 Biomolecules
1. In fructose, the possible optical isomers are 8. The hormone which controls the processes of
(a) 12 (b) 8 (c) 16 (d) 4 burning of fats, proteins and carbohydrates to
2. Glucose molecule reacts with X number of liberate energy in the body is
molecules of phenylhydrazine to yield osazone. (a) cortisone (b) adrenaline
The value of X is (c) thyroxine (d) insulin
(a) three (b) two (c) one (d) four 9. Two vitamins absorbed from intestine along with
3. Which L-sugar on oxidation gives an optically fats are
active dibasic acid (2 COOH groups)? (a) A, D (b) A, B (c) A, C (d) D, B
10. Which one of the following is an amine hormone ?
(a) Thyroxine (b) Oxypurin
(c) Insulin (d) Progesterone
CHO 11. Complete hydrolysis of starch gives :
CHO
H OH (a) glucose only
H OH
(a) HO H (b) HO (b) galactose and fructose in equimolar
H amounts
HO H
H OH (c) glucose and galactose in equimolar
CH2OH amounts
CH2OH
CHO CHO (d) glucose and fructose in equimolar amounts
12. Accumulation of which of the following
H OH HO H molecules in the muscles occurs as a result of
(c) H OH (d) H OH vigorous exercise ?
H OH HO H (a) Glycogen (b) Glucose
CH2OH CH2OH (c) Pyruvic acid (d) L-lactic acid
13. Which of the following will not show
4. In cells the net production of ATP molecules mutarotation?
generated from one glucose molecule is (a) Maltose (b) Lactose
(a) 46 (b) 32 (c) 36 (d) 40 (c) Glucose (d) Sucrose
5. Which is correct statement? 14. An organic compound with the formula C6H12O6
forms a yellow crystalline solid with
phenylhydrazine and gives a mixture of sorbitol
(a) Starch is a polymer of a-glucose and mannitol when reduced with sodium. Which
(b) In cyclic structure of fructose, there are four among the following could be the compound?
carbons and one oxygen atom
(c) Amylose is a component of cellulose
(d) Proteins are composed of only one type of (a) fructose (b) glucose
amino acids (c) mannose (d) sucrose
6. Vitamin D is also known as 15. When a-D-glucose and b-D-glucose ar e
(a) sunshine vitamin dissolved in water in two separate beakers I and
(b) ascorbic acid II respectively and allowed to stand, then –
(c) growth vitamin
(d) reproductive vitamin
7. Night blindness is caused by deficiency of
(a) vitamin B12 (b) vitamin A (a) specific rotation in beaker I will decrease
(c) vitamin C (d) vitamin E while in II will increase upto a constant value
EBD_7587
372 Chemistry Objective MCQs
(b) the specific rotation of equilibrium mixture 22. Which of the following statement(s) is (are) true?
in two beakers will be different
(c) the equilibrium mixture in both beakers will
be leavorotatory (i) All amino acids contain one chiral center.
(d) the equilibrium mixture in both beakers will (ii) Some amino acids contain one, while some
contain only cyclic form of glucose contain more chiral center or even no chiral
16. The pKa values for the three ionizable groups X, center.
Y and Z of glutamic acid are 4.3, 9.7 and 2.2 (iii) All amino acids found in proteins have L
respectively configuration.
(iv) All amino acids found in proteins have 1°
HO2C CH2 CH CO2H amino group.
(a) (ii), (iii) & (iv) (b) (ii) & (iii)
X Z (c) (i), (iii) & (iv) (d) (i) & (iv).
+ NH3
Y 23. In osazone formation, glucose reacts with three
The isoelectric point for the amino acid is: molecules of phenylhydrazine. Which statement
(a) 7.00 (b) 3.25 (c) 4.95 (d) 5.95 is true regarding this?
17. A vitamin that contains both N and P is
(a) Vitamin C (b) Vitamin K (a) All the three molecules react in similar
(c) Vitamin B12 (d) Vitamin D fashion
18. Match List I (name of vitamin) with List II (b) Two molecules react in similar manner, while
(deficiency result/disease) and select the correct the third reacts in different way
answer using the codes given below the lists : (c) All the three molecules react in different
List I List II ways
I. Ascorbic acid A. Beri-beri (d) None of the above is true
24. Natural glucose is termed D-glucose because :
II. Retinol B. Cracked lips
(a) – OH on the second carbon is on the right
III. Riboflavin C. Scurvy side in Fischer projection
IV. Thiamine D. Night blindness (b) – OH on the sixth carbon is on the right side
(a) I - B, II - A, III - C, IV - D in Fischer projection.
(c) – OH on the fifth carbon is on the right side
(b) I - A, II - B, III - C, IV - D
in Fischer projection.
(c) I - D, II - C, III - B, IV - A (d) It is dextrorotatory.
(d) I - C, II - D, III - B, IV - A 25. Which of the following statement is not correct?
19. Nucleotides in DNA are linked by – (a) Amylopectin is a branched polymer of a -
(a) hydrogen bond glucose.
(b) 3', 5'-phosphodiester bond (b) Cellulose is a linear polymer of b-glucose
(c) glycosidic bond (c) Glycogen is the food reserve of plants
(d) peptide bond (d) All proteins are polymers of a - amino acids.
20. If one str and of DNA has the sequence 26. Amylopectin is a polymer of
ATGCTTGA, the sequence in the complimentary (a) a – D – glucose (b) amino acid
strand would be (c) b – D – glucose (b) amylase.
(a) TACGAACT (b) TCCGAACT 27. Which of the following is a non-reducing sugar?
(a) Lactose (b) Fructose
(c) TACGTACT (d) TACGTAGT
(c) Sucrose (d) Maltose
21. Which of the following pairs can be distinguished 28. Glycosidic linkage is actually an :
by Fehling’s solution ? (a) Carbonyl bond (b) Ether bond
(c) Ester bond (d) Amide bond
(a) Glucose and fructose 29. Which of the following does not reduce
(b) Glucose and sucrose Benedict’s solution?
(c) Methanal and ethanal (a) Glucose (b) Fructose
(d) Hydroxypropanone and benzaldehyde. (c) Sucrose (d) Aldehyde
Biomolecules 373
30. Hydrolysis of sucrose is called CHO
(a) hydration (b) saponification CHO
(c) esterification (d) inversion HO H
HO H
31. Assume that a particular amino acid has an (a) H OH (b) HO H
isoelectric point of 6.0. In a solution at pH 1.0, H OH H OH
which of the following species will predominate? CH3
CH2OH
R R
CHO
+
(a) H3NCHCO2H (b) H2NCHCO2H HO H
R (c) HO H (d) None of these
R
+
H OH
(c) H3NCHCO2 (d) H2NCHCO2– CH2OH
32. A mixture of two amino acids having pI 9.60 and 38. The enantiomer of a-D-(+)-glucose is
5.40 can be separated (a) b-D-(+)-glucose (b) a-D-(–)-glucose
(c) a-L-(–)-glucose (d) b-L-(–)-glucose.
(a) by adjusting the pH of the solution at 9.60
(b) by adjusting the pH of the solution at 4.20 39. CN
(c) by adjusting the pH of the solution at 7.0 CHO
HO H
(d) by adjusting the pH of the solution at 7.5. HO H NaCN/HCN HO H
33. The correct statement in respect of protein ¾¾¾¾®
H OH H OH
haemoglobin is that it
(a) acts as an oxygen carrier in the blood CH2OH CH2OH
(b) forms antibodies and offers resistance to I
diseases
(c) enzymes are specific biological catalysts CN
that can normally function at very high H OH
temperature (T ~ 1000K) + HO H
(d) enzymes are specific biological catalysts H OH
that possess well-defined active sites
34. Which of the following carbohydrates is not CH2OH
related to (+)–glucose? II
(a) Amylopectin (b) Amylose Compounds I and II may be grouped as
(c) Inulin (d) Glycogen (a) diastereomers (b) epimers
35. What will happen when D-(+)-glucose is treated (c) C–2 epimers (d) all of the three.
with methanolic —HCl followed by Tollens’ 40. Which of the following evolves carbon dioxide,
reagent ? on oxidation with periodate ?

(a) A black ppt. will be formed (a) CHO (b) CH2OH


(b) A red ppt. will be formed ½ ½
CHOH CO
(c) A green colour will appear ½
½
(d) No characteristic colour or ppt. will be CHOH CHOH
formed. ½ ½
36. Glucose-D has a great tendency to be converted CH2OH CH2OH
into cyclic isomer. Which two carbon atoms get
joined through ‘O’ to form this hemiacetal ? (c) Both (d) None
(a) C1 and C4 (b) C1 and C5 41. Imino acid among these compounds is
(a) serine (b) proline
(c) C1 and C6 (d) C2 and C6
(c) tyrosine (d) lysine
37. Among the three compounds shown below, two 42. Which amino acid is achiral?
yield the same product on reaction with warm (a) alanine (b) valine
HNO3. The exception is: (c) proline (d) None of these
EBD_7587
374 Chemistry Objective MCQs

43. Which is not a true statement? 52. The dipeptide, Gly. Ala has structure –
O
+ || –
(a) a-carbon of a-amino acid is asymmetric (a) H 3 NCH 2 CNHCHCO 2
(b) All proteins are found in L-form |
CH 3
(c) Human body can synthesize all proteins
they need O
(d) At pH = 7 both amino acids and carboxylic + || –
groups exist in the ionised form (b) H 3 NC H CNHC H 2CO 2
|
44. Which of the following is a steroid hormone? CH 3
(a) Cholesterol (b) Adrenaline
O
(c) Thyroxine (d) Progesterone ||
– +
45. Vitamin B6 is known as (c) O2CCH–NH–C–CH2NH3
(a) pyridoxine (b) thiamine |
(c) tocopherol (d) riboflavin CH3
46. A sequence of how many nucleotides in O
messenger RNA makes a codon for an amino – || +
acid? (d) O 2 CCH 2 NH - CCHNH3
|
(a) Three (b) Four CH3
(c) One (d) Two
53. All of the following statements apply to proteins
47. The process by which synthesis of protein takes except
place based on the genetic information present (a) Proteins generally have no definite melting
in m-RNA is called point
(a) Translation (b) Proteins contain the grouping —CONH—
(b) Transcription (c) Proteins have high molecular weight
(c) Replication (d) Proteins can only contain the elements C,
(d) Messenger hypothesis H, O and N.
48. Which of the following substances is not 54. The reason for double helical structure of DNA
present in nucleic acids? is the operation of:
(a) Cytosine (b) Adenine (a) Electrostatic attractions
(b) van der Waals forces
(c) Thymine (d) Guanidine
(c) Dipole - Dipole interactions
49. Which of the following is not present in a (d) Hydrogen bonding
nucleotide? 55. Among the following organic acids, the acid
(a) Guanine (b) Cytosine present in rancid butter is:
(c) Adenine (d) Tyrosine (a) Pyruvic acid (b) Lactic acid
50. Iso-electric point of alanine is (pH = 6). At which (c) Butyric acid (d) Acetic acid
pH, maximum concentration of zwitter ion of 56. Which of the following enzyme converts starch
alanine will be present? into maltose?
(a) pH > 6 (b) pH < 6 (a) Diastase (b) Maltase
(c) pH = 6 (d) pH = 7 (c) Zymase (d) Invertase
57. Which statement is incorrect about peptide
HgSO 4
51. H C C H ¾¾¾® (A) bond?
H2SO4
(1) NH + HCN
¾¾¾¾¾¾®
3
+
(B) ; (a) C–N bond length in proteins is longer than
(2) H3O
usual C–N bond length
Product (B) of given reaction is: (b) Spectroscopic analysis shows planar
structure of - C - NH - bond
(a) Glycine (b) Alanine ||
(c) valine (d) Leucine O
Biomolecules 375
(c) C–N bond length in proteins is smaller than 59. Which of the following structures represents
usual C–N bond length thymine ?
(d) None of these O NH2
5 (CH3CO)2 O
58. D-(+)-Glucose ¾¾¾¾¾¾ ® D-(+)-Glucose HN N
pentaacetate (a) (b)
Which statement is true about glucose O N O N
pentaacetate ? H H
(a) It will react with phenylhydrazine but not NH2 OH
with Tollens’ reagent.
(b) It will react with Tollens’ reagent but not N N CH3
(c) (d)
with phenylhydrazine.
(c) It will react with both of the above HO N HO N
mentioned reagents. 60. Among the following vitamins the one whose
(d) It will react neither with phenylhydrazine deficiency causes rickets (bone deficiency) is
nor with Tollens’ reagent. (a) Vitamin A (b) Vitamin B
(c) Vitamin D (d) Vitamin C

Answer KEYs
1 (b ) 7 (b ) 13 (d ) 19 (b ) 25 (c) 31 (a ) 37 (b ) 43 (b ) 49 (d ) 55 (c )
2 (a) 8 (d ) 14 (a ) 20 (a) 26 (a) 32 (a ) 38 (c ) 44 (d ) 50 (c) 56 (a )
3 (a) 9 (a ) 15 (a ) 21 (b ) 27 (c) 33 (a ) 39 (d ) 45 (a) 51 (b ) 57 (c )
4 (c) 10 (a ) 16 (b ) 22 (b ) 28 (b ) 34 (c ) 40 (b ) 46 (a) 52 (a) 58 (d )
5 (a) 11 (a ) 17 (c ) 23 (b ) 29 (c) 35 (d ) 41 (b ) 47 (a) 53 (d ) 59 (d )
6 (a) 12 (d ) 18 (d ) 24 (c) 30 (d ) 36 (b ) 42 (d ) 48 (d ) 54 (d ) 60 (c )

1. (b) Fructose has 3 chiral centres and hence 5. (a) Starch is also known as amylum which
number of optical isomers are 23 = 8 occurs in all green plants. A molecule of
starch (C6H10O5)n is built of a large number
2. (a) CHO CH=N.NHPh
of a-glucose rings joined through oxygen
3PhNHNH2
CHOH ¾¾¾¾® CH=N.NHPh + PhNH2+ NH3 atom.
6. (a) Vitamin D is also known as sunshine
(CHOH)3 (CHOH)3 vitamin.
CH2OH CH2OH 7. (b) Vitamin A.
8. (d) Insulin.
3. (a) (a) and (d) are L– sugar but (a) gives an 9. (a) Vitamin A, D are absorbed from the intestine
optically active dibasic acid. by fats.
4. (c) C 6 H 12 O 6 + 6 O 2 + 2 ATP ¾
¾® 10. (a) Thyroxine is an amine hormone.
11. (a) Starch is a mixture of amylose &
6CO 2 + 6H 2 O + 38ATP amylopectin polysaccharides and monomer
\ Net ATP molecules evolved = 36 is glucose. Thus on complete hydrolysis it
gives only glucose.
EBD_7587
376 Chemistry Objective MCQs
12. (d) Glucose ¾¾¾¾¾¾® 21. (b) Glucose is a hemiacetal, so in presence of a
(does not need
stored in oxygen; base (alkaline medium is provided by
theform of ¯
need only
Glycogen enzymes)
Fehling’s solution) it can develop —CHO
O group in the form of open chain structure
Pyruvic acid ¾¾2¾
® CO2 + H 2O
¯
which responds Fehling’s solution. Sucrose
Lactic acid is a glycoside (acetal), i.e. its hemiacetal OH
During vigorous exercise sufficient oxygen groups (one due to glucose and another
is not available to meet the energy demand due to fructose) are not free, so it can’t attain
so, energy is derived through conversion —CHO group. Hence it will not respond
of pyruvic acid to lactic acid. Fehling’s solution.
13. (d) Sucrose does not contain a free aldehydic 22. (b) Although D-alanine is a constituent of a
or ketonic group hence it does not show bacterial cell walls, it is not found in proteins.
mutarotation. 23. (b) First molecule of phenylhydrazine
14. (a) Since the compound forms a yellow unedrgoes nucleophilic addition on
crystalline solid, i.e. osazone with carbonyl (–CHO in glucose and >CO in
phenylhydrazine, it may be an aldohexose fructose) group. Second molecule of the
or a ketohexose. Further, since on reduction, reagent oxidizes –CHOH– at position 2(in
compound forms a mixture of sorbitol and aldoses) or –CH2OH at position 1 (in
mannitol, it must be a ketohexose, i.e. ketoses) to form –CHO or C=O
fructose. Recall that glucose on reduction respectively. The third molecule again
gives only one alcohol glucitol (Sorbitol) undergoes nucleophilic addition on the
15. (a) a-D-glucose or b-D-glucose when dissolved newly developed carbonyl group to form
in water and allowed to stand, following
osazone.
equilibrium is established.
a-D-glucose ƒ Open chain form 24. (c) Fischer gave the prefix “D” to compounds
ƒ b-D-glucose whose bottom chiral has its OH to the right.
(+111°) (+19°) So natural glucose is called D-glucose or
Specific rotation of a-form falls, while that dextrose.
of b-form increases until a constant value Structure of D-Glucose :
of +52.5º is reached at equilibrium. This 1CHO
phenomenon is known as mutarotation. 2
H OH
4.3 + 2.2 3
16. (b) In acidic medium = 3.25 HO H
2 4
H OH
17. (c) Vitamin B12 contains both N and P 5
18. (d) Ascorbic acid - Scurvy H OH
6 CH OH
Retinol - Night blindness 2
Riboflavin - Cracked lips D-Glucose
Thiamine - Beri-Beri 25. (c) Glycogen is called animal starch and is found
Thus option (d) is correct. in all animal cells. It constitutes the reserve
19. (b) Phosphate is linked to 3rd carbon of 1 and food material.
5th carbon of the other sugar molecules. 26. (a) Amylopectin is a polymer of a-D-glucose.
20. (a) On the basis of structure of guanine and 27. (c) Sucrose is a non-reducing sugar as it does
complementary bases present in them, we not reduce Fehling reagent and Tollen's
can say that if the sequence of bases in one reagent.
strand of DNA is I, then the sequence in the
28. (b) Glycosidic linkage is actually an ether bond
second strand should be II
as the linkage forming the rings in an
A:T : G : C :T :T : G :A I
oligosaccharide or polysaccharide is not
T : A : C : G : A : A : C : T II
Biomolecules 377
just one bond, but the two bonds sharing 32. (a) Every amino acid exists exclusively as
an oxygen atom e.g. sucrose dipolar ion when the pH of the solution is
HO equal to its isoelectric point (pI), hence at
this pH it does not migrate to either electrode,
HO while at other pH, an amino acid migrates
OH either to cathode or to anode depending
O
O upon its pI. Thus at pH 9.60, amino acid
OH with pI 5.40 will exist as an anion and migrate
HO O
to anode; while that with pI 9.60 will not
OH migrate to any electrode.
HO OH 33. (a) Haemoglobin acts as an oxygen carrier in
the blood since it reacts with oxygen to form
HOCH2
unstable oxyhaemoglobin which easily
O breaks to give back haemoglobin and
oxygen.
OH
34. (c) Inulin is a polysaccharide of fructose, while
OH all others are polysaccharides of glucose ;
OH although all the four has same molecular
O formula, (C6H10O5)n.
glycosidic link
35. (d) Reaction of D-(+)-glucose with methanolic
—HCl leads to formation of methyl
HOCH2 O glucoside (C1—OH group is methylated)
which, being acetal, is not hydrolysable by
HO base, so it will not respond Tollen’s reagent.
CH2OH 36. (b) Glucose exists as glucopyranose (a six
OH membered ring structure) which is formed
through C1 and C5.
29. (c) Sucrose, being a non-reducing sugar, does 37. (b) a and c give the same product.
not reduce Benedict’s solution. Remember 38. (c) In the D family the more dextrorotatory
that fructose has an a-hydroxy ketonic anomer is named a-D-. In the L family the
group, which is also reducing group more laevorotatory anomer is named a L.
(different from ordinary ketonic group)
39. (d) When structures I and II are C–2 epimers, it
30. (d) Since sucrose is dextrorotatory while
implies that these are epimers and
hydrolysis product of sucrose, having
diastereomers too.
equimolar mixture of glucose and fructose,
40. (b) A ketonic group having —OH group on
is laevorotatory. Hence the hydrolysed
both sides is removed as CO 2 during
product of sucrose is known as invert sugar
oxidation with periodate.
and the hydrolysis of sucrose is known as
41. (b) Proline contains imino (secondary amino),
inversion.
31. (a) In acidic medium Zwitter ion convert into NH group
+ – + 42. (d) All are chiral
NH3 CH COO + H
43. (b) All proteins are not found in L-form but
R they may be present in form of D or L.
44. (d) Progesterone (Gestogens) is a steroid
+
hormone, which controls the development
¾¾
® NH3 — CH — COOH
and maintainance of pregnancy. Thryoxine
ô
R and adrenaline are amine hormones.
EBD_7587
378 Chemistry Objective MCQs
45. (a) Vitamin B6 is called pyridoxine. It is found 55. (c) Butyric acid also known as butanoic acid
in fruits, green-vegetables, milk, etc. is found in milk and butter and is a product
46. (a) The sequence of bases in mRNA are read of anaerobic fermentation. It has an
in a serial order in groups of three at a time. unpleasant smell and acid taste.
Each triplet of nucleotides (having a specific 56. (a) Maltose is obtained by partial hydrolysis
sequence of bases) is known as codon.
of starch by the enzyme diastase present in
Each codon specifies one amino acid.
Malt.
Further since, there are four bases,
therefore, 43 = 64 triplets or codons are 2(C6H10O5)n + nH2O ¾¾¾¾®
Diastase
possible.
nC6H12O6 (Maltose)
47. (a) Synthesis of polypeptide is known as
translation. For this process three type of 57. (c) Due to resonance,

RNA are essential. O O
+
NH – C – NH – ¬¾® – C = NH – ,
||
48. (d) Guanidine, H 2 N - C - NH 2 , is neither a C – N bond acquires some double bond
character, hence shorter in length
purine nor a pyrimidine base
49. (d) Tyrosine is an a-amino acid, and neither a 58. (d) During acetylation of (+)-glucose, it is the
purine nor a pyrimidine base C1 —OH of the hemiacetal that is acetylated
50. (c) At isoelectric point, conc. of Zwitter ion will and not the C5—OH that forms the ring
be maximum. (cyclic structure). Since equilibrium with the
O open-chain aldehyde is prevented, the
HgSO4 || penta-acetate does not respond the
51. (b) HC CH ¾¾¾¾
H 2SO4
® CH3 —CH aldehydic reactions.

NH 2 H, OH H, OAc
NH3 + HCN|
¾¾¾¾¾
+
® CH3 — CH — COOH
H
H OH H OAc
52. (a) By convention, the amino acid with the free Ac2O
amino group (N-terminal) is written at the HO H O ¾¾¾® AcO H O
H OH H OAc
left end and the one with the unreacted
H H
carboxyl group (C-terminal) at the right end.
Thus, the structure of Gly. Ala is CH2OH CH2OH
D-(+)-Glucose a or b-Glucose penta acetate
O
+ || –
H 3 NCH 2 CNHCHCO 2 59. (d) The correct structure of thymine is
|
CH 3 OH
53. (d) Statement (d) is not correct. Some proteins
N CH3
also contain S, with C, H, O and N.
54. (d) The two polynucleotide chains of DNA
molecules are twisted around a common axis HO N
but run in opposite directions to form a right Thymine (T)
handed helix. The two chains are joined
together by specific hydrogen bonds. 60. (c) Deficiency of vitamin D causes rickets.
29 Polymers
1. Which of the following is not an example of (c) ( CH2 – CH = CH – CH2 – CH – CH2 )n
addition polymer?
(a) Polystyrene (b) Nylon CN
Cl
(c) PVC (d) Polypropylene
2. Which of the following is not a biopolymer? (d) ( CH2 – C = CH – CH2 ) n
(a) Proteins (b) Rubber 8. Low density polythene is prepared by
(c) Cellulose (d) RNA (a) Free radical polymerisation
3. Thermosets are: (b) cationic polymerisation
(a) cross-linked polymers (c) anionic polymerisation
(b) don’t melt or soften on heating (d) Ziegler-Natta polymerisation
(c) cross-linking is usually developed at the 9. The polymer used for optical lenses is :
time of moulding where they harden (a) polypropylene
reversibly (b) polyvinyl chloride
(d) all of the above (c) polythene
4. Which of the following statements is not correct (d) polymethyl methacrylate
for fibres? 10. Polymer used in bullet proof glass is
(a) lexan (b) PMMA
(a) Fibres possess high tensile strength and (c) nomex (d) kevlar
high modulus. 11. Glyptals are chiefly employed in
(b) Fibres impart crystalline nature. (a) toy making
(c) Characteristic features of fibres are due to (b) surface coating
strong intermolecular forces like hydrogen (c) photofilm making
bonding. (d) electrical insulators
(d) All are correct. 12. Which of the following is currently used as a
5. Perlon is tyre cord?
(a) Rubber (b) Nylon-6 (a) Terylene (b) Polyethylene
(c) Terylene (d) Orlon (c) Polypropylene (d) Nylon - 6
6. Which one of the following is a chain growth 13. The plastic household crockery is prepared by
polymer? using
(a) Nucleic acid (b) Polystyrene
(c) Protein (d) Starch (a) melamine and tetrafluoroethane
7. Structure of some important polymers are given. (b) malonic acid and hexamethyleneamine
Which one represents Buna-S? (c) melamine and vinyl acetate
CH3 (d) melamine and formaldehyde
14. Mark out the most unlike form of polymerization
(a) ( CH2 – C = CH – CH2 )n of CH2 = CH – CH = CH2

(b) ( CH2 – CH = CH – CH2 – CH – CH2 )n H CH2


(a) C=C
C6H5 CH2 H n
EBD_7587
380 Chemistry Objective MCQs
22. Which of the following organic compounds
H H polymerizes to form the polyester Dacron?
(b) C=C
CH2 CH2 (a) Propylene and para HO—(C6H4)— OH
n (b) Benzoic acid and ethanol
CH = CH2 CH = CH 2 (c) Terephthalic acid and ethylene glycol
– CH2 – CH – CH2 – CH – (d) Benzoic acid and para HO–(C6H4)—OH
(c)
n 23. Biodegradable polymer which can be produced
from glycine and aminocaproic acid is :
CH2 CH2
(a) PHBV (b) Buna - N
(d) – C –– C – (c) Nylon 6, 6 (d) Nylon 2- nylon 6
n 24. Caprolactum is used for the manufacture of :
15. The condensation of hexamethylenediamine with
sebacoyl chloride at 525 K gives
(a) nylon-6,20 (b) nylon-6,01 (a) Nylon - 6 (b) Teflon
(c) nylon-6,10 (d) None of these (c) Terylene (d) Nylon - 6,6
16. Example of condensation polymer is 25. If a polythene sample contains two monodisperse
(a) Formaldehyde ® meta-formaldehyde fractions in the ratio 2 : 3 with degree of
(b) Acetaldehyde ® para-aldehyde polymerization 100 and 200, respectively, then
(c) Acetone ® mesityl oxide its weight average molecular weight will be :
(d) Ethene ® polyethene (a) 4900 (b) 4600 (c) 4300 (d) 5200
17. The mass average molecular mass & number 26. Synthetic polymer bakelite can be prepared from
average molecular mass of a polymer are 40,000 following compounds
and 30,000 respectively. The polydispersity index (a) Styrene and vinyl chloride
of polymer will be (b) Acrylonitrile and vinyl chloride
(c) Adipic acid and ethylene glycol
(d) Phenol and formaldehyde
(a) < 1 (b) > 1 (c) 1 (d) 0 27. Among cellulose, polyvinyl chloride, nylon and
18. Given the polymers (i) Nylon-6, 6; (ii) Buna-S; natural rubber, the polymer in which the
(iii) Polythene. Arrange these in increasing order intermolecular force of attraction is weakest is
of their inter-molecular forces (lower to higher) (a) Nylon (b) Polyvinyl chloride
(c) Cellulose (d) Natural Rubber
28. Which one of the following is an example of
(a) (i) > (ii) > (iii) (b) (ii) > (iii) > (i) thermosetting polymers?
(c) (ii) < (iii) < (i) (d) (iii) < (i) < (ii)
(a) Neoprene (b) Buna-N
19. What is the percentage of sulphur used in
vulcanization of rubber? (c) Nylon 6, 6 (d) Bakelite
(a) 05% to 30% (b) 03% to 25% 29. Which of the following polymer is a polyamide?
(c) 10% to 20% (d) 05% to 25% (a) Terylene (b) Nylon
20. Which of the following is used in vulcanization (b) Rubber (d) Vulcanised rubber
of rubber ? 30. Ebonite is a
(a) S8 (b) CF4 (c) Cl2F2 (d) C2F2 (a) natural rubber
21. Which one of the following statement is not true? (b) synthetic rubber
(a) In vulcanization the formation of sulphur (c) highly vulcanized rubber
bridges between different chains make (d) polypropene
rubber harder and stronger. 31. Melamine plastic crockery is a codensation
(b) Natural rubber has the trans -configuration polymer of
at every double bond
(a) HCHO and melamine
(c) Buna-S is a copolymer of butadiene and
(b) HCHO and ethylene
styrene
(d) Natural rubber is a 1, 4 - polymer of isoprene (c) melamine and ethylene
(d) None of these
Polymers 381
32. Nylon 6,6 is a polyamide obtained by the reaction (a) M n = (M w )1/2 (b) Mn = M w
of
(a) COOH(CH2)4 COOH + NH2C6H4NH2– (p) (c) M w > M n (d) M w < M n
(b) COOH(CH2)4 COOH + NH2 (CH2)6 NH2 37. Polymer formation from monomers starts by
(c) COOH (CH2)6 COOH + NH2 (CH2)4 NH2 (a) condensation or addition reaction between
(d) COOHC6H4 COOH– (p) + NH2 (CH2)6 NH2 monomers
33. Which of the following is not correctly represented? (b) coordinate reaction between monomers
(c) conversion of monomer to monomer ions
(a) Terylene
(d) hydrolysis of monomers.
O O Benzoyl peroxide or
38. n(CF2 = CF2 ) ¾¾¾¾¾¾¾¾
®X
— OCH2–CH2–C– –C — (NH 4 )2 S2 O8
n Here, X is :
(b) Neoprene (a) RMMA (b) PVC
(c) PAN (d) None of these
— CH2 — C = CH — CH 2 — 39. Which pair of polymers have similar properties ?

Cl
n
(c) Nylon-6, 6 (a) Nylon, PVC
O O (b) PAN, PTFE
(c) PCTFE, PTFE
— NH–(CH2)6– NH– C–(CH2)4– C–O —n (d) Bakelite, alkyl resin
40. Among the following, the wrong statement is
CH3
(a) PMMA is plexiglass
(d) PMMA — CH2–C (b) SBR is natural rubber
(c) PTFE is teflon
COOCH3 n
(d) LDPE is low density polythene
34. The monomer of the polymer; 41. When condensation product of hexamethylene-
diamine and adipic acid is heated to 525 K in an
atmosphere of nitrogen for about 4-5 hours, the
product obtained is
CH3 CH3 (a) solid polymer of nylon 6, 6
|
ÚÚÚÚÚCH 2 - C - CH 2- C + (b) liquid polymer of nylon 6, 6
| is (c) gaseous polymer of nylon 6, 6
CH3 CH3
(d) liquid polymer of nylon 6
CH3 42. In which of the following polymers, empirical
(a) H 2C = C formula resembles with monomer?
(a) Bakelite (b) Teflon
CH3 (c) Nylon-6, 6 (d) Dacron
(b) CH3CH = CHCH3 43. Which is a polymer of three different monomers?
(c) CH3CH = CH2 (a) ABS (b) SBR
(d) (CH3)2C = C(CH3)2 (c) NBR (d) Nylon-2, 6
35. Which of the following polymers do not involve 44. The polymer which has conducting power is
cross linkages? (a) polyethylene (b) polybutadiene
(a) Melmac (b) Bakelite (c) polystyrene (d) polyacetylene
(c) Polythene (d) Vulcanised rubber 45. Which of the following compound is used for
preparation of melamine formaldehyde polymer?
36. Number average molecular mass, Mn and weight NH2
average molecular mass (M w ) of synthetic
polymers are related as (a)
EBD_7587
382 Chemistry Objective MCQs
N 48. Which of the following statements is false?
N N
(a) Artificial silk is derived from cellulose.
(b) Nylon-6, 6 is an example of elastomer.
(b) (c) The repeat unit in natural rubber is isoprene.
H2N NH2
NH2 (d) Both starch and cellulose are polymers of
H2N NH2 glucose.
N 49. Which one of the following sets forms the
biodegradable polymer?
N N
(c)

NH2 (a) CH2 = CH – CN and CH2 = CH – CH = CH2


(b) H2N – CH2 – COOH and
NH2
H2N–(CH2)5 – COOH
(c) HO – CH2 – CH2 – OH and
(d)
N HOOC COOH
H2N
46. Match the polymers in column - A with their main
uses in column - B and choose the correct (d) CH = CH2 and
answer:
Column - A Column - B CH2 = CH – CH = CH2
(A) Polystrene (i) Paints and lacquers 50. Which one of the following is an example of a
(B) Glyptal (ii) Rain coats thermosetting polymer?
(C) Polyvinyl (iii) Manufacture of toys (a) ( CH 2 - C = CH - CH 2 ) n
|
chloride Cl
(D) Bakelite (iv) Computer discs
(a) (A) - (ii), (B) - (i), (C) - (iii), (D) - (iv) (b) ( CH 2 - CH ) n
|
(b) (A) - (ii), (B) - (iv), (C) - (iii), (D) - (i) Cl
(c) (A) - (iii), (B) - (iv), (C) - (ii), (D) - (i)
H H O O
(d) (A) - (iii), (B) - (i), (C) - (ii), (D) - (iv) | | || ||
47. The compound which cannot be used as a (c) ( N - (CH 2 )6 - N - C - (CH 2 ) 4 - C ) n
plasticizer, is
(a) di-n-butylphthalate OH OH
(b) tricresyl phosphate CH2 CH2
(c) di-n-octyphthalate (d)
(d) diethyl phthalate n

Answer KEYs
1 (b) 6 (b) 11 (b) 16 (c) 21 (b) 26 (d) 31 (a) 36 (c) 41 (b) 46 (d)
2 (b) 7 (b) 12 (d) 17 (b) 22 (c) 27 (d) 32 (b) 37 (a) 42 (b) 47 (d)
3 (d) 8 (a) 13 (d) 18 (c) 23 (d) 28 (d) 33 (a) 38 (d) 43 (a) 48 (b)
4 (d) 9 (d) 14 (d) 19 (a) 24 (a) 29 (b) 34 (a) 39 (c) 44 (d) 49 (b)
5 (b) 10 (b) 15 (c) 20 (a) 25 (a) 30 (c) 35 (c) 40 (b) 45 (c) 50 (d)
Polymers 383

1. (b) Nylon is a condensation polymer


2. (b) Since proteins, cellulose and RNA control various activities of plants and animals, they are called
biopolymers. They are found in living organisms.
3. (d) These are characteristics of thermosets.
4. (d) All the given statements about fibres are correct.
5. (b) Perlon is nylon-6. It is prepared from a single monomer (caprolactam) having a potential amino group
at one end and a potential carbonyl group at other end.
6. (b) Polystyrene is a chain growth polymer.

CH = CH2

Na, Heat
7. (b) nCH 2 = CH - CH = CH 2 + n ¾¾¾¾¾®
Polymerisation
1, 3-Butadiene
Styrene

( CH
— 2 – CH = CH – CH2 – CH – CH2 —
)n
Butadiene -
Styrene copolymer
(SBR or BUNA - S)

8. (a) Ethene on free radical polymerisation gives low density polythene.


9. (d) Polymethyl methacrylate is hard, fairly rigid. It is used for optical lenses.
10. (b) PMMA and polycarbonate are used in bullet proof glass
11. (b) Glyptal is an alkyd resin obtained from polyhydric alcohols and polybasic organic acids or their
anhydrides. They are used for surface coatings.
12. (d) Nylon tyre cord is made from high tenacity continuous filament yarn by twisting and plying.
13. (d) The unbreakable plastic household crockery is made from copolymer of formaldehyde (HCHO) and
melamine.

H2N N NH2

N N Polymerisation
+ HCHO ¾¾¾¾¾¾® Melmac
Formaldehyde
NH2
Melamine

14. (d) In polymerisation of 1,3-butadiene either 1,4-polymerisation or 1,2-polymerisation occurs. In case of


1,4-polymerisation, the double bond shifts at C2 and C4 carbon, while the chain propagates from C1
and C4 end. In this either trans or cis polymeric chain is formed.
Option (a) and (b) represent 'trans' and 'cis' 1,4 polymerisation respectively.
Option (c) resembles 1,2 polymerisation, where as option (d) most unlikely to happen.
EBD_7587
384 Chemistry Objective MCQs

O O O O
|| || || ||
D
15. (c) H 2 N - (CH 2 -)6 NH 2 + Cl - C - (CH 2 )8 - C - Cl ¾¾® -(HN - (CH 2 ) 6 - (NH - C - (CH 2 )8 - C -
)n
Hexamethylene diamine Sebacoyl chloride Nylon 6, 10
16. (c) Polymers formed by condensation process with elimination of small molecule like H2O, CO2 etc. are
known as condensation polymers.
O
CH3 CH P
Eg. dil. 3
C = O + H 2 CH - C - CH3 ¾¾¾ ® C = CH - C - CH 3 + H 2 O
HCl
CH3 Acetone P CH3 Mesityl oxide
O

17. (b) Average number molecular weight Mn = 30,000

Average mass molecular weight Mw = 40,000

w M 40, 000
Polydispersity index (PDI) = M = 30, 000 = 1.33
n

18. (c) Nylon – 6, 6 is a fibre with hydrogen bonds thus have strongest intermolecular forces. Buna-S is a
elastomer whereas polythene is thermoplastic. the intermolecular forces of attraction in thermoplastics
is intermediate between elastomer and fibres. Therefore forces in elastomers are weakest.
19. (a) Vulcanization is a process in which natural rubber is treated with sulphur and certain organic
compounds which accelerate the reaction between the rubber and sulphur. Thus about 5% S is used
for making tyre rubber, 20-25% S for making ebonite and 30% S for making battery case rubber.
20. (a) SF6 is used in the vulcanisation of rubber. Sulphur is heated with polymer to introduce cross-linking
and thus, form tough polymer.
CH3 H CH2 CH2 CH3 H
21. (b) C=C C=C C=C
– CH2 CH2 CH3 H CH2
CH2–
Natural rubber
Natural rubber
(All cis configuration)
All statements except (b) are correct
O O
DD || ||
22. (c) n HOCH2CH2OH + n HOOC COOH ¾¾® -( OCH2CH2–O– C C- )n

Ethylene glycol Terephthalic acid Terylene


It is resistant to mineral and organic acids. It is used for blending with wool to provide better crease,
in safety helmets and aircraft battery boxes.
23. (d) H 2 N ¾ CH 2 ¾ COOH + H 2 N ¾ (CH 2 )5 ¾ COOH ® -(HN ¾ CH 2 ¾ CO ¾ NH ¾ (CH 2 )5 ¾ CO)-
Glycine Aminocapric acid Nylon-2-nylon-6

O O
H
NH N
24. (a) ¾¾¾¾¾® Nylon–6
n
(Caprolactum)

25. (a)
Polymers 385

26. (d) Phenol and formaldehyde undergo condensation polymerisation under two different conditions
to give a cross linked polymer called bakelite.
27. (d) Nylon and cellulose, both have intermolecular hydrogen bonding, polyvinyl chloride has dipole-
dipole interaction, while natural rubber has van der Waal forces which are weakest.
28. (d) Polymers which change irreversibly into hard and rigid material on heating are known as thermosetting
polymers e.g bakelite.
29. (b) Nylon is a polyamide fibre. It is prepared by the condensation polymerisation of adipic acid
(HOOC(CH2)4COOH) and hexamethylene diamine (H2N(CH2)6NH2).
30. (c) Ebonite is a hard highly vulcanized rubber, containing 20-25%, sulphur.
31. (a) Melamine plastic crockery is a copolymer of HCHO and Melamine.

32. (b) n HOOC(CH 2 ) 4 COOH + n H 2 N(CH 2 ) 6 NH 2


Adipic acid Hexamethylene diamine

O O
525K
¾¾¾¾¾¾® [—
C – (CH 2)4 – C –NH–(CH 2)6 – NH —
]n
Polymerisation
Nylon 6, 6

33. (a) Terylene is prepared by condensing terephthalic acid and ethylene glycol

O O
|| ||
n HOOC COOH + n HOCH2CH2OH ¾® C C – OCH2 – CH2 – O
Terephthalic acid Ethylene glycol Terylene
n

34. (a) Addition of monomers follows isoprene rule

CH3 CH3 CH3


C = CH2 C = CH2 H 3C +

CH3 CH3 ¾¾® C – CH2– C – CH2


H 3C

H T H T CH3

35. (c) Polythene is a linear polymer

Mw
36. (c) PDI =
Mn

For synthetic polymer, PDI > 1


\ M w > Mn
37. (a) Polymerisation starts either by condensation or by addition reactions between monomers
Benzoyl peroxide or
38. (d) ® ( CF2 - CF2 )
n (CF2 = CF2 ) ¾¾¾¾¾¾¾¾
(NH 4 )2 S2 O8
PTFE
(X)
Here X is polytetrafluoroethylene. So, none of these i.e., option (d) is correct choice.
EBD_7587
386 Chemistry Objective MCQs

39. (c) PCTFE and PTFE both have same carbon backbone.
-[ CIFC - CF2 –]n -[ F C - CF –]
2 2 n
PCTFE (polymonochloro trifluoroethylene) PTFE (poly tetrafluoro ethylene)

40. (b) SBR is styrene-butadiene rubber, is a synthetic rubber.


41. (b) The condensation polymerisation of hexamethylene diamine and adipic acid is done in solution form
by interface technique. In this, liquid nylon polymer is obtained.
42. (b) In addition homopolymers such as teflon, empirical formula resembles with monomer.
43. (a) ABS is acrylonitrile-butadiene-styrene rubber which is obtained by copolymerisation of acrylonitrile,
1, 3-butadiene and styrene.

CH 2 = CH + CH 2 = CH + CH 2 = CHC6 H5 ¾¾
®
| | Styrene
CN CH = CH 2
Acrylonitrile 1,3- Butadiene

é CH 2 - CH- CH 2 CH = CHCH 2 - CH 2 - CH —
— ù
ê | | ú
ëê CN C6H5ûú n

ABS rubber
44. (d) Polyacetylene, due to presence of double bonds, is a conducting polymer.
45. (c)
46. (d) Column-A Column-B
Polymer Use
Polystyrene Manufacture of toys
Glyptal Paints and lacquers.
P.V.C. Rain Coats
Bakelite Computer discs
47. (d) Generally high boiling esters or haloalkanes act as plasticizer.
48. (b) Nylons (polyamides) are fibres.
49. (b) Biodegradable polymer is nylon-2, 6 which is copolymer of glycine (H2N – CH2– COOH) and amino
caproic acid (H2N–(CH2)5 – COOH).
O O
n H2N – CH2 – COOH + nH2N – (CH2)5– COOH ¾® —
( HN – CH2 – C – NH – (CH2) 5 – C —
)n
Glycine Amino caproic acid Nylon – 2, 6

50. (d) Glyptal is used in the manufacture of paints and lacquers.


Chemistry in
30
Everyday Life
1. Terfenadine is commonly used as a/an 9. An antibiotic with a broad spectrum
(a) tranquilizer (b) antihistamine (a) kills the antibodies
(c) antimicrobial (d) antibiotic (b) acts on a specific antigen
2. Which of the following term means pain killer? (c) acts on different antigens
(a) Antibiotic (b) Analgesic (d) acts on both the antigens and antibodies
10. An ester used as medicine is
(c) Antipyretic (d) Penicillin
(a) ethyl acetate (b) methyl acetate
3. Which of the following is a bactericidal antibiotic? (c) methyl salicylate (d) ethyl benzoate
(a) Ofloxacin (b) Tetracycline 11. Select the incorrect statement.
(c) Chloramphenicol (d) Erythromycin (a) Equanil is used to control depression and
hypertension.
4. Which one of the following is used as Antihistamine?
(b) Mifepristone is a synthetic steroid used as
(a) Omeprazole (b) Chloranphenicol “morning after pill”.
(c) Diphenhydramine (d) Norethindrone (c) 0.2 per cent solution of phenol is an antiseptic
5. Which of the following statements about aspirin while its 1.0 per cent solution is a disinfectant.
(d) A drug which kills the organism in the body
is not true?
is called bacteriostatic.
(a) It is effective in relieving pain. 12. Which of the following acts as an antioxidant in
(b) It is a neurologically active drug. edible oils ?
(c) It has antiblood clotting action. (a) Vitamin B (b) Vitamin C
(c) Vitamin D (d) Vitamin E
(d) It belongs to narcotic analgesics. 13. H1 – Receptor antagonists is a term associated
6. Sulphonamides act as with :
(a) Antiseptic (b) Analgesic (a) Antiseptics (b) Antihistamins
(c) Antimicrobials (d) Antipyretic (c) Antacids (d) Analgesics
14. The structure given below is known as
7. Various phenol derivatives, tincture of iodine
(2 – 3%) I2 in (water / alcohol) and some dyes
O
like methylene blue are P
CH 2 - C - NH
(a) antiseptics (b) disinfectants H H S CH3
(c) analgesics (d) antipyretics
CH3
8. Which of the following is a hypnotic drug? N COOH
(a) luminal (b) salol O H
(c) catechol (d) chemisol
(a) Penicillin F (b) Penicillin G
(c) Penicillin K (d) Ampicillin
EBD_7587
388 Chemistry Objective MCQs
15. The correct structure of the drug paracetamol is (a) A - I, B - II, C - III, D - IV
(b) A - II, B - III, C - IV, D - I
OCH3 OH
(c) A - III, B - I, C - II, D - IV
(d) A - IV, B - I, C - II, D - III
(a) (b) 19. An antipyretic is
(a) quinine (b) paracetamol
(c) luminal (d) piperazine
CONH2 NHCOCH3 20. Salol can be used as
(a) antiseptic (b) antipyretic
(c) analgesic (d) None of these
OCH3 OH 21. Which one of the following is employed as a
tranquilizer?
(a) Naproxen (b) Tetracycline
(c) (d) (c) Chlorpheninamine (d) Equanil
22. The insecticide containing 99% g – isomer of
benzene hexachloride is known as
CONH2 COCH3 (a) lindane (b) TNT
16. Which one of the following can possibly be used (c) malathion (d) methoxychlor
as analgesic without causing addiction and mood 23. The following compound is used as
modification?
O
(a) Diazepam
(b) Morphine O – C – CH3
(c) N-acetyl-para-aminophenol
(d) Tetrahydrocannabinol COOH
17. Antiseptics and disinfectants either kill or
prevent growth of microorganisms. Identify (a) an anti-inflammatory compound
which of the following statements is not true: (b) analgesic
(c) hypnotic
(d) antiseptic
(a) Chlorine and iodine are used as strong 24. Amoxycillin is semi-synthetic modification of
disinfectants. (a) penicillin (b) streptomycin
(b) Dilute solutions of boric acid and hydrogen (c) tetracyclin (d) chloroampheniol
Peroxide are strong antiseptics. 25. Which of these is a hypnotic ?
(c) Disinfectants harm the living tissues. (a) metaldehyde (b) acetaldehyde
(d) A 0.2% solution of phenol is an antiseptic
(c) paraldehyde (d) None of these
while 1% solution acts as a disinfectant.
26. Which of the following is used as an
18. Match the chemicals in Column I with their uses
antibiotic?
in Column II.
(a) ciprofloxacin (b) paracetamol
(c) ibuprofen (d) tocopherol
Column I Column II
27. The drug used for prevention of heart attacks
(A) Sodium perborate (I) Disinfectant
is
(B) Chlorine (II) Antiseptic
(c) Bithional (III) Milk bleaching agent
(a) aspirin (b) valium
(D) Potassium stearate (IV) Soap (c) chloramphenicol (d) cephalosporin
Chemistry in Everyday Life 389
28. Omeoprazole and lansoprazole are used as – 38. End of detergents have
(a) ester group (b) sodium sulphate
(c) aldehyde (d) amine group
39. Which of the following represents a synthetic
(a) antifertility (b) antiallergic detergent?
(c) antibiotic (d) antacid
29. Sulpha drugs are used for (a) C15H31COOK
(a) precipitating bacteria (b) CH3[CH2]16COONa
(b) removing bacteria
(c) decreasing the size of bacteria C12H25 SO3Na
(c)
(d) stopping the growth of bacteria
30. The drug which is effective in curing malaria is
(d) None of these
(a) quinine (b) aspirin
40. Which of the following represents soap ?
(c) analgin (d) equanil
(a) C17H35COOK (b) C17H35COOH
31. Heroin is a derivative of
(c) C15H31COOH (d) (C17H35COO)2Ca
(a) cocaine (b) morphine
41. An antibiotic contains nitro group attached to
(c) caffeine (d) nicotine
aromatic nucleus. It is
32. The formulation of dettol contains
(a) penicillin (b) streptomycin
(a) chloroxylenol (b) terpineol
(c) tetracycline (d) chloramphenicol
(c) alcohol (d) All of these
33. Which of the following is an antidiabetic drug? 42. Aspirin can be prepared by the reaction of
(a) Insulin (b) Penicillin (a) Salicyldehyde with acetic anhydride in
(c) Chloroquine (d) Aspirin presence of H2SO4
34. When salicylic acid is treated with acetic (b) Salicylic acid with methanol in presence of
anhydride we get H2SO4
(a) aspirin (b) paracetamol (c) Salicylic acid with acetic anhydride in
(c) salol (d) none of these presence of H2SO4
35. Which artificial sweetener contains chlorine ? (d) Cinnamic acid with acetic anhydride in
(a) Sucralose (b) Alitame presence of H2SO4
(c) Aspartame (d) Saccharin
43. Which is correct about vanillin?
36. Structuraly biodegradable detergents, should
(a) A flavouring agent having vanila flavour
contain (b) p-hydroxy-m-methoxy benzaldehyde
(c) A food additive
(a) normal alkyl chain (d) All of these
(b) branched alkyl chain 44. Which is correct about saccharin?
(c) phenyl side chain
(d) cyclohexyl side chain O
37. Substance used for the preservation of coloured C
(a) It is NH
fruit juices is SO2
(a) benzene
(b) benzoic acid (b) It is 600 times sweeter than sugar
(c) phenol (c) It is used as sweetening agent
(d) sodium meta bisulphite (d) All of these
EBD_7587
390 Chemistry Objective MCQs

45. Benzalkonium chloride is a (c) antiseptic and disinfectant


(d) antibiotic-broad spectrum
(a) cationic surfactant and antiseptic 48. Artificial sweetner which is stable under cold
(b) anionic surfactant and soluble in most of conditions only is :
organic solvents (a) Saccharine (b) Sucralose
(c) cationic surfactant and insoluble in most of (c) Aspartame (d) Alitame
organic solvents 49. Which one of the following compounds is an
(d) cationic surfactant and antimalarial anti-fertility drug?
46. Which one of the following is employed as
antihistamine ?
(a) Chloramphenicol (a) Aspirin (b) Chloromycetin
(b) Diphenylhydramine (c) Saheli (d) Penicillin
(c) Norothindrone 50. Chemically heroin is
(d) Omeprazole (a) morphine monoacetate
47. Chloroamphenicol is an : (b) morphine dibenzoate
(a) antifertility drug (c) morphine diacetate
(b) antihistaminic (d) morphine monobenzoate

Answer KEYs
1 (b) 6 (c) 11 (d) 16 (c) 21 (d) 26 (a) 31 (b) 36 (b) 41 (d) 46 (b)
2 (b) 7 (a) 12 (d) 17 (b) 22 (a) 27 (a) 32 (d) 37 (b) 42 (c) 47 (d)
3 (a) 8 (a) 13 (b) 18 (c) 23 (b) 28 (d) 33 (a) 38 (b) 43 (d) 48 (c)
4 (c) 9 (c) 14 (b) 19 (b) 24 (a) 29 (d) 34 (a) 39 (c) 44 (d) 49 (c)
5 (d) 10 (c) 15 (b) 20 (a) 25 (c) 30 (a) 35 (a) 40 (a) 45 (a) 50 (c)
Chemistry in Everyday Life 391

1. (b) Terfenadine is commonly used as 18. (c) The correct matching is as follows :
antihistamine.
Column I Column II
2. (b) Analgesic means pain killer.
(A) Sodium perborate Milk bleaching agent
3. (a) Bactericidal are the drugs that kills bacteria.
(B) Chlorine Disinfectant
Ofloxacin works by stopping the growth of
bacteria. This antibiotic treats only bacterial (C) Bithional Antiseptic
infections. (D) Potassium stearate Soap
4. (c) Diphenhydramine is used as antihistamine. 19. (b) Paracetamol is an antipyretic
5. (d) Aspirin is an non-narcotics analgesic. 20. (a) Salol is phenyl salicylate used as antiseptic.
6. (c) Sulphonamides act as antimicrobials. 21. (d) Equanil is an important medicine used in
7. (a) Antiseptic drugs cause destruction of depression and hypertension.
micro-organism that produce septic 22. (a) Lindane or gammexane is g isomer of BHC.
diseases e.g. Dettol, savlon, boric acid, 23. (b) It is acetyl salicylic acid i.e., aspirin which is
phenol, iodoform, KMnO4 and some dyes used as analgesic and antipyretic.
such as methylene blue, genatian violet. 24. (a) Amoxycillin is semisynthetic modification
8. (a) These drugs induce sleep and are habit of penicillin
forming, common example of hypnotic 25. (c) Paraldehyde is a hypnotic.
drugs are luminal and saconal. 26. (a) Ciprofloxacin is used as an antibiotic, while
9. (c) Broad spectrum antibiotics act on different paracetamol, ibuprofen and tocopherol are
antigens. respectively antipyretic, pain killer and
10. (c) Oil of winter green or methyl salicylate is vitamin E.
used as medicine. 27. (a) Due to anti-blood clotting action of
11. (d) Bacteriostatic drugs inhibit the growth of aspirin, it is used to prevent heart attack.
organisms while bactericidal drugs kill the 28. (d) Antacids decrease acidity in stomach.
microorganisms. 29. (d) Sulpha drugs (antibacterials and antibiotics)
12. (d) Vitamin E is an antioxidant present in edible are group of drugs which are derivatives of
sulphanilamide.
oils.
30. (a) Substances used for the treatment of malaria
13. (b) The term “antihistamine” refers only to H1
are antimalarials, e.g. quinine, chloroquine.
antagonists, which is also known as
31. (b) Heroin is acyl derivative of morphine.
H1-receptor antagonists and H1-antihistamine.
32. (d) Dettol (antiseptic) is a mixture of 4.8%
14. (b) It is the known structure of penicillin G or
chloroxylenol + 9.9% terpineol and absolute
benzyl penicillin
alcohol.
15. (b) 33. (a) Insulin is antidiabetic drug.
16. (c) We know that N-acetyl-para-aminophenol 34. (a)
(or paracetamol) is an antipyretic which can
OH OCOCH3
also be used as an analgesic to relieve COOH COOH
pains. (CH3CO)2O
¾¾¾¾¾®
NaOH
17. (b) Dilute solutions of boric acid and hydrogen
Salicylic acid Acetyl salicylic acid
peroxide are weak antiseptics. (Aspirin)
EBD_7587
392 Chemistry Objective MCQs
42. (c) Asprin is prepared by reaction of salicylic
HO acid with acetic anhydride in presence of
OH H2SO4.
Cl HO
O Cl O
35. (a) is
C – OH O O
HO O || ||
O OH C C
OH Cl
+ CH3 O CH3
sucralose.
COOH
36. (b) Structurally, biodegradable detergents
should contain branched alkyl chain. OCOCH3
H SO
2 4®
37. (b) Benzoic acid is used as preservative as ¾¾¾¾
sodium benzoate. Acetyl salicylic acid
38. (b) A detergent molecule consists of a large (Aspirin)
hydrocarbon group that is non-ionic and a 43. (d) All are characteristics of vanillin.
sulphonate (SO3–Na+) or a sulphate (SO4– 44. (d) All are characteristics of saccharin.
Na +) group that is ionic. Examples of 45. (a) Benzalkonium chloride, also known as
detergents are sodium-n-dodecyl benzene alkyldimethylbenzylammonium chloride is
sulphonate, sodium dodecyl sulphate etc. nitrogenous cationic surface active agent
39. (c) The most widely used domestic detergent belonging to the quaternary ammonium
is the sodium dodecyl benzene sulphonate group. It is used as antiseptic.
(SDS). +
N Cl –
CH3 – (CH2)11 SO3Na CnH2n +1
n = 8, 10, 12, 14, 16, 18
40. (a) Soaps are the sodium or potassium salts of 46. (b) Diphenylhydramine also known as benadryl
higher fatty acids e.g., C 17 H37 COOK is an antihistamine.
(potassium stearate). These are obtained by 47. (d) Chloroamphenicol is a broad spectrum
alkaline hydrolysis of oils and fats. The antibiotic.
48. (c) Aspartame is stable under cold conditions.
reaction is called saponification.
49. (c) The antifertility drugs are those chemical
41. (d) Chloramphenicol is
compounds which prevent pregnancy in
O woman. These, drugs controls the menstrual
||
O2N CH - CH - NH - C – CHCl 2
cycle and ovulation and so used as birth
| | control drug or contraceptives.
OH CH 2 OH 50. (c) Morphine diacetate.
Analytical
31
Chemistry
1. An aqueous solution of a salt X turns blood red (a) 85.36 % (b) 14.64 %
on treatment with CNS– and blue on treatment (c) 58.63 % (d) 26.14 %
with K4[Fe(CN)6]. X also gives a positive chromyl 8. In Kjeldahl’s method, CuSO4 acts as
chloride test. The salt X is :
(a) oxidising agent (b) reducing agent
(a) CuCl2 (b) FeCl3
(c) hydrolysing agent (d) catalytic agent
(c) Cu(NO3)2 (d) Fe(NO3)3
2. KMnO4 reacts with oxalic acid as : 9. In Lassaigne’s test, the organic compound is
fused with a piece of sodium metal in order to
(a) increase the ionisation of the compound.
(b) decrease the melting point of the compound.
MnO 4- + C 2 O 24- + H + ¾
¾® Mn 2 + + CO 2 + H 2 O (c) increase the reactivity of the compound.
Hence, 50 ml of 0.04 M KMnO4 is acidic medium (d) convert the covalent compound into a
is chemically equivalent to mixture of ionic compounds.
(a) 100 ml of 0.1 M H2C2O4 10. A white sodium salt dissolves readily in water to
(b) 50 ml of 0.2 M H2C2O4 give a solution which is neutral to litmus. When
(c) 50 ml of 0.1 M H2C2O4 silver nitrate solution is added to the solution, a
(d) 25 ml of 0.1 M H2C2O4 white precipitate is obtained which does not
3. For detection of sulphur in an organic compound dissolve in dil. HNO3. The anion could be
sodium nitroprusside is added to the sodium
extract. A violet colour is obtained due to (a) CO 32 - (b) Cl - (c) SO 24 - (d) S2 -
formation of 11. An aqueous solution of a substance gives a white
(a) Fe(CN) 2 precipitate on treatment with dil. HCl which
(b) K 3Fe(CN) 5 NS dissolves on heating. When H2S is passed
through the hot acidic solution, a black precipitate
(c) Na 4 [Fe(CN)5 NOS]
is obtained. The substance is a
(d) Na 4 Fe(CN ) 6
4. 0.45 g of acid (molecular weight 90) is neutralised (a) Hg 22 + salt (b) Cu2+ salt
by 20 ml of 0.5N caustic potash. The basicity of
acid is (c) Ag+ salt (d) Pb2+ salt
(a) 1 (b) 2 (c) 3 (d) 4 12. A is a lighter phenol and B is an aromatic
5. The cation that will not be precipitated by H2S carboxylic acid. Separation of mixture of A and B
in the presence of dil. HCl is : can be carried out easily by using a solution of
(a) Pb2+ (b) Cu2+ (c) Co2+ (d) As 3+ (a) sodium hydroxide
6. A pink coloured salt turns blue on heating. The (b) sodium sulphate
presence of which cation is most likely ?
(c) calcium chloride
(a) Co2+ (b) Cu2+ (c) Zn2+ (d) Fe2+ (d) sodium bicarbonate
7. 0.5 g mixture of K 2Cr2 O 7 and KMnO4 was 13. A metal chloride dissolves appreciably in cold
treated with excess of KI in acidic medium. I2 water. When placed on a platinum wire in Bunsen
liberated required 100 cm3 of 0.15 N Na 2S2O 3 flame no distinctive colour is noticed, the cation
solution for titration. The percentage amount of would be
K 2Cr2 O 7 in the mixture is (a) Mg2+ (b) Ba2+ (c) Pb2+ (d) Ca2+
EBD_7587
394 Chemistry Objective MCQs

OH OCH3 OH (a) As2S3 and CdS (b) CdS, NiS and ZnS
(c) NiS and ZnS (d) Sulphide of all ions
14. 19. 3.92 g of ferrous ammonium sulphate react
N
completely with 50 ml KMnO4 solution. The
10
percentage purity of the sample is
(p-ethyl phenol) (p-methyl anisole) (p-ethyl benzyl alcohol)
(a) 50 (b) 78.4 (c) 80 (d) 39.2
Above compounds can be differentiated by using 20. 1.25 g of a sample of Na2CO3 and Na2SO4 is
the reagent: dissolved in 250 ml solution. 25 ml of this solution
(a) NaOH, Tollen's reagent, FeCl3 neutralises 20 ml of 0.1N H2SO4. The % of Na2CO3
(b) CrO3, Tollen's reagent, FeCl3 in this sample is
(c) Tollen's reagent, CrO3, FeCl3 (a) 84.8% (b) 8.48% (c) 15.2% (d) 42.4%
(d) Na, Tollen's reagent, FeCl3 21. An aqueous solution of FeSO4, Al2(SO4)3 and
15. Copper wire test for halogens is known as chrome alum is heated with excess of Na2O2 and
(a) Duma’s Test (b) Beilstein’s Test filtered. The materials obtained are:-
(c) Liebig’s Test (d) Lassigne’s Test (a) A colourless filtrate and a green residue.
16. In the separation of Cu and Cd2+ in 2nd group
2+ (b) A yellow filtrate and a green residue.
qualitative analysis of cations, tetrammine (c) A yellow filtrate and a brown residue.
copper (II) sulphate and tetrammine cadmium (II) (d) A green filtrate and a brown residue.
sulphate react with KCN to form th e 22. The formula of gas is [CO]x. If its vapour density
corresponding cyano complexes. Which one of is 140, the value of x will be:
the following pairs of the complexes and their (a) 2.5 (b) 3.0 (c) 5.0 (d) 6.0
relative stability enable the separation of Cu2+ 23. If 0.2 gram of an organic compound containing
and Cd2+? carbon, hydrogen and oxygen on combustion,
(a) K3[Cu(CN)4] more stable and K2[Cd(CN)4] yielded 0.147 gram carbon dioxide and 0.12 gram
less stable water. What will be the content of oxygen in the
substance ?
(b) K2[Cu(CN)4] less stable and K2[Cd(CN)4]
(a) 73.29% (b) 78.45%
more stable
(c) 83.23% (d) 89.50%
(c) K2[Cu(CN)4] more stable and K2[Cd(CN)4] 24. Which of the following statements is incorrect ?
less stable
(a) Fe2+ ion also gives blood red colour with
(d) K3[Cu(CN)4] less stable and K2[Cd(CN)4] SCN– ion.
more stable (b) Fe3+ ion also gives blood red colour with
17. Sodium carbonate cannot be used in place of SCN– ion.
(NH4)2CO3 for the identification of Ca2+, Ba2+ and (c) On passing H2S into Na2ZnO2 solution a
Sr2+ ions (in group V) during mixture analysis white ppt of ZnS is formed.
because : (d) Cupric ion reacts with excess of ammonia
(a) Mg2+ ions will also be precipitated. solution to give deep blue colour of
(b) Concentration of CO32– ions is very low. [Cu(NH3)4]2+ ion.
(c) Sodium ions will react with acid radicals. 25. An aqueous solution of colourless metal sulphate
(d) Na+ ions will interfere with the detection of M gives a white precipitate with NH4OH. This
Ca2+, Ba2+, Sr2+ ions. was soluble in excess of NH4OH. On passing
18. A solution containing As3+, Cd2+, Ni2+ and Zn2+ is H2S through this solution a white ppt. is formed.
made alkaline with dilute NH4OH and treated with The metal M in the salt is
H2S. The precipitate obtained will consist of

(a) Ca (b) Ba (c) Al (d) Zn


Analytical Chemistry 395
26. Volume of 3% solution of sodium carbonate (a) NiSO4 (b) BaS2O3
necessary to neutralise a litre of 0.1 N sulphuric (c) PbS2O3 (d) CuSO4
acid 34. 0.24 g of a volatile liquid on vaporization gives
(a) 176.66 ml (b) 156.6 ml
45 ml of vapours at NTP. What will be the vapour
(c) 116.0 ml (d) 196.1 ml
density of the substance ?
27. The Lassaigne’s extract is boiled with dil. HNO3
(Density of H2 = 0.089 g L–1)
before testing for halogens because
(a) 95.39 (b) 39.95 (c) 99.53 (d) 59.93
35. The salt used for performing ‘bead’ test in
(a) silver halides are soluble in HNO3 qualitative inorganic analysis is
(b) Na2S and NaCN are decomposed by HNO3
(c) Ag2S is soluble in HNO3 (a) K 2SO 4 . Al 2 (SO 4 )3 .24H 2 O
(d) AgCN is soluble in HNO3
28. For preparing 250 mL of N/20 solution of Mohr’s (b) FeSO 4 . ( NH 4 ) 2 SO 4 4H 2 O
salt, the amount of Mohr’s salt needed is (c) Na ( NH 4 )HPO 4 .4H 2O
(a) 9.8 g (b) 4.9 g (c) 19.6 g (d) 3.2 g
29. Potassium chromate solution is added to an (d) CaSO 4 .2H 2O
aqueous solution of a metal chloride. The 36. A gas “X” is passed through water to form a
precipitate thus obtained is insoluble in acetic saturated solution. The aqueous solutions on
acid. When precipitate is subjected to flame test treatment with the AgNO 3 gives a white
the colour of the flame is preciptate. The saturated aqueous solution also
(a) lilac (b) apple green dissolves magnesium ribbon with evolution of a
(c) crimson red (d) golden yellow colourless gas “Y”. Identify ‘X’ and ‘Y’.
30. Three separate samples of a solution of a single (a) X = CO2, Y = Cl2 (b) X = Cl2, Y = CO2
salt gave these results. One formed a white (c) X = Cl2, Y = H2 (d) X = H2 , Y = Cl2
precipitate with excess ammonia solution, one
formed a white precipitate with dil. NaCl solution 37. In Kjeldahl’s method for the estimation of N 2 ,
and one formed a black precipitate with H2S. The potassium sulphate and copper sulphate are
salt could be used. On the basis of their functions which of
(a) AgNO3 (b) Pb(NO3)2 the following statement(s) is/are correct?
(c) Hg(NO3)2 (d) MnSO4 I. Potassium sulphate raises the bpt. and
ensures complete reaction.
OEt
31. Compound CH 3 CH and II. Copper sulphate acts as catalyst.
OEt III. Potassium sulphate acts as catalyst and
(P) (Acetal)
copper sulphate raises the bpt.
CH3 CH2 O CH2 CH2 can be (a) Only III is correct (b) I and II are correct
(Q)
(c) Only II is correct (d) None is correct
differentiated by:
38. Aniline is usually purified by
(a) H3OÅ, Na (b) H3OÅ, Tollens' test
Å (a) chromatographic technique
(c) H3O , Fehling test (d) All of these
32. Th e best meth od for th e separ at ion of (b) steam distillation
naphthalene and benzoic acid from their (c) by addition of oxalic acid
mixture is: (d) fractional crystallization
(a) Distillation (b) Sublimation 39. Prussian blue is formed when
(c) Chromatography (d) Crystallisation (a) ferrous sulphate reacts with FeCl3
33. A salt on treatment with dil. HCl gives a pungent (b) ferric sulphate reacts with K 4 Fe(CN) 6
smelling gas and a yellow precipitate. The salt
(c) ferrous ammonium sulphate reacts with
gives green flame when tested. The solution
gives a yellow precipitate with potassium FeCl3
chromate. The salt is (d) ammonium sulphate reacts with NO2
EBD_7587
396 Chemistry Objective MCQs
40. 0.59 g of an organic substance when treated with [Y] + K2Cr2O7 + H2SO4 ¾¾® green solution.
caustic soda evolved ammonia which required [X] and [Y] is:
20 cc of N/2 sulphuric acid for neutralisation. The (a) SO32–, SO2 (b) Cl– ,HCl
percentage of nitrogen is 2–
(c) S , H2S (d) CO32–, CO2
(a) 26.32% (b) 40% 46. 0.75 g platinic chloride of a mono- acidic base on
(c) 53.6% (d) 63.6% ignition gives 0.245 g platinum. The molecular
41. An organic compound has C and H percentage weight of the base is
in the ratio 6 : 1 and C and O percentage in the (a) 75.0 (b) 93.5 (c) 100 (d) 80.0
ratio 3:4 the compound is 47. The sodium extract prepared from sulphanilic acid,
contains SCN - . It gives blood red colouration
(a) HCHO (b) CH3OH with
(c) CH3CH2OH (d) (COOH)2 (a) a mixture of Na2S and CS2
42. The presence of magnesium is confirmed in the (b) FeCl3
qualitative analysis by the formation of a white (c) FeSO4
crystalline precipitate of (d) Na2SO3
(a) Mg(HCO 3 ) 2 (b) MgNH 4 PO 4 48. An orange precipitate of II group is dissolved in
conc. HCl; the solution when treated with excess
(c) MgNH 4(HCO3 )3 (d) MgCO3
of water turns milky due to formation of
43. A salt which gives CO2 with hot conc.H2SO4
(a) Sn (OH)Cl (b) Sb(OH )Cl 2
and also decolourizes acidified KMnO4 on
warming is: (c) SbOCl (d) Sb(OH ) 2 Cl
(a) HCO 3- (b) CO 32- 49. A substance on treatment with dil.H2 SO 4
(c) Oxalate (d) Acetate liberates a colourless gas which produces
44. 2.79 g of an organic compound when heated in (I) turbidity with baryta water and
Carius tube with conc. HNO3 and H3PO4 formed (II) turns acidified dichromate solution green.
is converted into MgNH4.PO4 ppt. The ppt. on The reaction indicates the presence of
heating gave 1.332 g of Mg 2 P 2 O 7. The
percentage of P in the compound is (a) CO 32- (b) S2 - (c) SO 32 - (d) NO 3-
50. Which of the following gives a precipitate with
Pb(NO3)2 but not with Ba(NO3)2?
(a) 23.33% (b) 13.33% (a) Sodium chloride
(c) 33.33% (d) 26.66% (b) Sodium acetate
45. [X] + H2SO4 ¾¾® [Y] a colourless gas with (c) Sodium nitrate
irritating smell, (d) Sodium hyrogen phosphate

Answer KEYs
1 (b) 6 (a) 11 (d) 16 (a) 21 (b) 26 (a) 31 (d) 36 (c) 41 (a) 46 (b)
2 (c) 7 (b) 12 (d) 17 (a) 22 (c) 27 (b) 32 (b) 37 (b) 42 (b) 47 (b)
3 (c) 8 (d) 13 (a) 18 (d) 23 (a) 28 (b) 33 (b) 38 (b) 43 (c) 48 (c)
4 (b) 9 (d) 14 (b) 19 (a) 24 (a) 29 (b) 34 (d) 39 (b) 44 (b) 49 (c)
5 (c) 10 (b) 15 (b) 20 (a) 25 (d) 30 (b) 35 (c) 40 (a) 45 (a) 50 (a)
Analytical Chemistry 397

1. (b) Fe3+, radical gives blood red with CNS–


æ x 0.5 - x ö 100 ´ 0.15
\ ç + ÷ =
Fe3+ +3 CNS – ¾¾
® Fe(CNS)3 è 49 31.6 ø 1000
Ferric sulphocyanide
(Blood red colouration) where 49 is Eq. wt. of K 2Cr2 O 7 and 31.6 is
Fe3+ gives blue colour on treatment with
K4[Fe(CN)6] Eq. wt. of KMnO4 .
4Fe3+ + 3K4 [Fe(CN)6] ¾® On solving, we get x = 0.073 g
12K+ + Fe4[Fe(CN)6]3 Percentage of K 2Cr2 O 7
Ferricferrocyanide
(Prussian blue) 0.0732 ´ 100
= = 14.64%
Cl– radical gives chromyl chloride test. 0.5
2. (c) Equiv. mass of KMnO4 8. (d) Kjeldahl’s method depends upon the fact
molar mass molar mass that most of the organic compounds
MnO 4- = = . containing nitrogen are quantitatively
7-2 5
decomposed to give (NH4 )2SO 4 when
Equiv. mass of oxalic acid
heated strongly with conc. H2SO4. In this
molar mass molar mass method CuSO4 acts as catalytic agent.
C 2 O 24 - = =
2(4 - 3) 2 9. (d) To convert covalent compounds into ionic
compounds such as NaCN, Na2S, NaX, etc.
Meq. of KMnO4= 50 × 5 × 0.04 = 10 = meq of
10. (b) NaCl is a salt of strong acid and strong
H2C2O4 = 50 × 2 × 0.1 = 10. Hence (c).
base hence on dissolution will give neutral
3. (c) Na 2S + Na 2 [Fe(CN)5 NO] ¾¾
® solution.
Sodium nitroprusside NaCl + AgNO3 ¾® AgCl ¯ + HNO3
11. (d) PbCl2 is insoluble in cold water, soluble in
Na 4 [Fe(CN)5 NOS]
Sodium thionitroprusside
hot water and PbS is black ppt in acidic
medium.
4. (b) Eq. of acid = Eq of base, 12. (d) Carboxylic acids dissolve in sodium
0.45 20 ´ 0.5 bicarbonate, while phenol does not.
\ = , = E.wt = 45
E.wt 1000 13. (a) MgCl2 is soluble in water and does not give
flame test. Other give flame test.
M.wt 90
Basicity = = =2 OH
E.wt 45 CHO
5. (c) Co+2 ion is precipitated by H2S in presence
of NH4OH which is a group reagent of group 14. (b) CrO
¾¾¾

IV in cationic analysis.
6. (a) Zn2+ salts are white. Usually Fe2+ salts are
rarely pink. Cu2+ salts are usually blue in CO2H
hydrated form. Co2+ is pink in aqueous
solution. Tollen 's
¾¾¾¾®
7. (b) Let the amount of the K 2Cr2 O 7 in the
mixture be x g, then amount of KMnO4
will be (0.5 – x) g
EBD_7587
398 Chemistry Objective MCQs
15. (b) Beilstein's test: Organic compounds
12 0.147
containing halogens when heated over Cu 23. (a) % of C = ´ ´ 100 = 20.045 %
wire loop give blue or green colour flame 44 0.2
due to formation of volatile copper halides. 2 0.12
16. (a) K 3[Cu(CN) 4] is more stable whereas % of H = ´ ´100 = 6.666 %
18 0.2
K2[Cd(CN)4] is less stable.
17. (a) If Na2CO3 is used in place of (NH4)2CO3. It % of O = 100 - ( 20.045 + 6.666) = 73.29 %
will precipitate group V radicals as well as 24. (a) Only Fe3+ ions give blood red colouration
magnesium radicals. The reason for this is with SCN– ions.
the high ionization of Na2CO3 in water into Fe3+ + SCN - ¾¾
® [Fe(SCN)]2+
Na+ and CO2– 3 . Now the higher concentration (dark red)
of CO23 – is available which exceeds the
solubility product of group V radicals as 25. (d) Zn 2 + + 2NH 4 OH ¾¾
® Zn(OH) 2 + 2NH 4+
well as that of magnesium radicals. White ppt.
Zn(OH) 2 + 2NH 4 OH ¾¾
®
18. (d) As3+ and Cd2+ are the radicals of group II,
whereas Ni2+ & Zn2+ are the radicals of group Zn(OH)2 + 2NH4OH ¾¾
®(NH4 )2 ZnO2 + 2H 2O
IV. The solubility product of group IV Soluble
radicals is higher as compared to group II.
(NH 4 )2ZnO 2 + H 2S ¾¾
® ZnS + 2NH 4 OH
NH4OH increases the ionisation of H2S by
White ppt.
removing H+ of H2S as unionisable water.
H2S 2H+ + S2–; 26. (a) Normality of 3% Na2CO3.

H + + OH - ¾ ¾® H 2 O 3 ´ 100
N= = 0.566N
Thus excess of sulphide ions are present 53 ´ 100
which leads to the precipitation of all the For H2SO4 sol. N1 = 0.1, V1 = 100 mL
four ions. For Na2CO3 sol. N2 = 0.566. N2 ?
Note : HCl decreases ionisation of H2S Now apply N1V1 = N2V2
whereas NH4OH increases the ionisation of N1V1 0.1 ´ 100 mL
H2S. V2 = N = = 176.66 mL
2 0.566
50 ´ 1
19. (a) Eq of KMnO4 used = = 0.005 27. (b) Na2S and NaCN, formed during fusion with
1000 ´ 10 metallic sodium, must be removed before
\ Eq of FAS reacted = 0.005 adding AgNO3, otherwise black ppt. due to
\ weight of FAS needed Na2S or white precipitate due to AgCN will
= 0.005 × 392 = 1.96 g be formed and thus white precipitate of AgCl
20. (a) Let the amount of Na2CO3 present in the will not be identified easily.
mixture be x g . Na2SO4 will not react with
H2SO4. Then Na 2S ∗ 2AgNO3 ¾¾
↑ 2NaNO3 ∗ Ag 2S →
Black
x 20 ´ 0.1 ´10
= \ x = 1.06g
53 1000 NaCN ∗ AgNO3 ¾¾
↑ NaNO3 ∗ AgCN →
\ Percentage of Na2CO3 White

1.06 ´ 100 NaCl + AgNO3 ¾


¾® NaNO 3 + AgCl ¯
= = 84.8%
1.25 white
21. (b) Green residue is due to Fe(OH)2 and yellow
boil
Na 2S + 2HNO 3 ¾¾
¾® 2 NaNO 3 + H 2S ­
filtrate is due to chromate ions CrO 24-
boil
22. (c) 28 × x = 140 Þ x = 5 NaCN + HNO 3 ¾¾
¾® NaNO 3 + HCN ­
Analytical Chemistry 399
28. (b) The ionic equation for oxidation of Mohr’s 35. (c) Microcosmic salt. Na(NH 4 )HPO 4 .4H 2 O
salt is
is used for bead test
Fe2+ ¾¾
® Fe3+ + e - Na.( NH 4 )HPO 4 ® NaPO3 + NH3 + H 2 O

392 NaPO 3 + CoO ® NaCo.PO 4 (blue)


Now Eq. of Mohr’s salt = = 392
1 36. (c) Cl2 + H2O ® HCl +HClO
Strength = Normality × Eq. mass HCl + AgNO3 ® AgCl ¯ + HNO3
White ppt
1
= ´ 392 = 19.6g / lit Mg + 2HCl ® MgCl2 + H2 ­
20
Hence X is Cl2 and Y is H2
Thus for preparing 250 ml of N/20 Mohr’s 37. (b) K2SO4 raises the bpt. and ensure complete
salt solution, Mohr ’s salt needed reaction and CuSO4 acts as catalyst.
19.6 38. (b) Aniline is miscible in steam and has b.p.
= ´ 250 = 4.9 g lower than water, hence it is collected as
1000
distillate along with steam.
29. (b) BaCl2 + K2CrO4 ® BaCrO4 + 2KCl. BaCrO4 39. (b) 2Fe 2 (SO 4 )3 + 3K 4 [(Fe(CN) 6 ] ®
is insoluble in acetic acid and Ba gives apple
green colour in flame test. Fe 4 [ Fe (CN ) 6 ]3 + 6K 2SO 4
30. (b) Pb(NO3)2 + 2NH4OH ® prussian blue
Pb(OH)2 ¯ + 2NH4NO3 40. (a) Percentage of nitrogen by Kjeldahl’s method
(white ppt)
1.4 ´ N ´ V
Pb(NO3)2 + 2HCl ® PbCl2 ¯ + 2HNO3 =
Wt. of organic compound
(white ppt)
Pb(NO3)2 + H2S ® PbS ¯ + 2HNO3 1.4 ´ 20 1
(black) = ´ = 26.32%
0.532 2
OEt 41. (a) % ratio of C : H is 6 : 1 andC : O is 3 : 4 or 6 : 8
H OÅ
31. (d) CH3 – CH ¾¾¾¾
3 ® \ % ratio of C : H : O is 6 : 1 : 0
OEt ( Total = 15)
(P) (Acetal)
% of C = 6 ´ 100 = 40
CH3CHO; differentiated by Na, Fehling 15
Tollen's test 40 / 12 = 3 . 33 = 1
CH 3 – CH 2 – O – CH 2 – CH 3
(Q) 1
% of H = ´ 100 = 6.6
15
H OÅ
¾¾¾¾ 3 ® EtOH 6.6 / 1 = 6.6 = 2
32. (b) Among the given compounds naphthelene 8
is volatile but benzoic acid is non-volatile % of O = ´100 = 53.3
15
(it forms a dimer). So, the best method for
their separation is sublimation, 53.3 / 16 = 3.3 = 1
33. (b) Gas evolved is SO2
simple ratio: CH2O
Wt. of 45 ml. of vapours at NTP \ The compound is HCHO
34. (d) V.D. =
Wt. of 45 ml. of H 2 at NTP 42. (b) MgCl 2 + Na 2 HPO 4 + NH 4 OH ®

0.24 g Mg ( NH 4 ) PO 4 + 2 NaCl + H 2 O
= = 59.93 white ppt .
-1
45 ml. ´ 0.000089 g ml
EBD_7587
400 Chemistry Objective MCQs

43. (c) Na 2C2O 4 + H 2SO4 ® 46. (b) Apply the formula M.wt

Na 2SO4 + H 2O + CO + CO2 Acidity æ wt of Pt salt ´ 195 ö


= ç - 410 ÷÷
2 çè wt of Pt ø
CO burns with blue flame 2CO + O 2 ® 2CO 2
Na 2C2O 4 + H 2SO 4 (dil) ®
\ M.wt = 1 æç 0.75 ´195 - 410 ö÷ = 93.5
Na 2SO4 + H 2 C2O 4 2 è 0.245 ø

2KMnO 4 + 3H2SO4 + 5H 2 C2 O4 ® K 2SO4 +47.


2MnSO + 8H-2 O++Fe
(b) 4 SCN 10CO
3+ 2
¾ ¾® Fe(SCN )3
® K 2SO 4 + 2MnSO 4 + 8H 2 O + 10CO 2 blood red ppt.

44. (b) Percentage of P 48. (c) Sb 2S3 + 6HCl ® 2SbCl 3 + 3H 2S


62 wt . of Mg 2 P2 O 7 SbCl 3 + 6H 2 O ® SbOCl + 2HCl
= × × 100 ppt ( White )
222 wt. of compound
49. (c) Na 2SO 3 + 2HCl (dil) ®
62 1.332
= × × 100 = 13.33% 2 NaCl + H 2 O + SO 2
222 2.79
45. (a) SO2 and H2S both being reducing agents, K 2 Cr2 O7 + H 2SO4 + 3SO 2 ®
can turn acidified dichromate solution green.
K 2SO4 + Cr2 (SO4 )3 + H 2 O
SO2 can be obtained by the action of acid
Green
upon sulphite while H2S is evolved by the
50. (a) Pb(NO3)2 +2NaCl ® PbCl2 ¯ + 2NaNO3 ;
action of acid upon sulphide. However, SO2
BaCl2 is soluble in water
has a burning sulphur smell which is
irritating. H2S has rotten egg like smell.
@unacademyplusdiscounts_link

https://telegram.me/Unacademyplusdiscounts_link

https://telegram.me/Unacademyplusdiscounts_link

https://telegram.me/Unacademyplusdiscounts_link

Join Us Now For all study Materials for free

You might also like